Вы находитесь на странице: 1из 495

S

C
I
S
PHY @ HSC

Stephen Bosi
John OByrne
Peter Fletcher
Joe Khachan
Jeff Stanger
Sydney, Melbourne, Brisbane, Perth, Adelaide
and associated companies around the world

Sandra Woodward

Contents
Series features
How to use this book
Stage 6 Physics syllabus grid

vi
viii
x

Module 1 Space
Module 1 Introduction
Chapter 1 Cannonballs, apples, planets and gravity
1.1 Projectile motion
1.2 Gravity
1.3 Gravitational potential energy
Practical experiences
Chapter summary
Review questions

2
4
4
10
16
20
22
22

Chapter 2 Explaining and exploring the solar system 26


2.1 Launching spacecraft
26
2.2 Orbits and gravity
35
2.3 Beyond Keplers orbits
41
2.4 Momentum bandits: the slingshot effect
44
2.5 Im back! Re-entry
46
Practical experiences
52
Chapter summary
53
Review questions
54
Chapter 3 Seeing in a weird light: relativity
3.1 Frames of reference and classical relativity
3.2 Light in the Victorian era
3.3 Special relativity, light and time
3.4 Length, mass and energy
Practical experiences
Chapter summary
Review questions

58
58
61
64
69
75
76
76

Module 1 Review

80

Module 2 Motors and Generators


Module 2 Introduction

82

Chapter 4 Electrodynamics: moving charges and


magnetic fields
84
4.1 Review of essential concepts
84
4.2 Forces on charged particles in magnetic fields 89
4.3 The motor effect
90
4.4 Forces between parallel wires
93
Practical experiences
97
Chapter summary
98
Review questions
98

Chapter 5 Induction: the influence of changing


magnetism
5.1 Michael Faraday discovers electromagnetic
induction
5.2 Lenzs law
5.3 Eddy currents
Practical experiences
Chapter summary
Review questions

100
100
104
106
109
110
110

Chapter 6 Motors: magnetic fields make the world


go around
6.1 Direct current electric motors
6.2 Back emf and DC electric motors
6.3 Alternating current electric motors
Practical experiences
Chapter summary
Review questions

114
114
120
121
126
127
127

Chapter 7 Generators and electricity supply: power


for the people
7.1 AC and DC generators
7.2 Transformers
7.3 Electricity generation and transmission
Practical experiences
Chapter summary
Review questions

130
130
136
141
148
149
149

Module 2 Review

152

Module 3 From Ideas to Implementation


Module 3 Introduction

154

Chapter 8 From cathode rays to television


8.1 Cathode ray tubes
8.2 Charges in electric fields
8.3 Charges moving in a magnetic field
8.4 Thomsons experiment
8.5 Applications of cathode rays
Practical experiences
Chapter summary
Review questions

156
156
160
164
165
167
170
171
171

Chapter 9 Electromagnetic radiation: particles


or waves?
9.1 Hertzs experiments on radio waves
9.2 Black body radiation and Plancks hypothesis
9.3 The photoelectric effect
9.4 Applications of the photoelectric effect
Practical experiences
Chapter summary
Review questions

174
174
178
182
184
185
186
187
iii

Contents

Chapter 10 Semiconductors and the electronic


revolution
10.1 Conduction and energy bands
10.2 Semiconductors
10.3 Semiconductor devices
10.4 The control of electrical current
Practical experiences
Chapter summary
Review questions

188
189
190
193
197
201
202
202

Chapter 11 Superconductivity
11.1 The crystal structure of matter
11.2 Wave interference
11.3 X-ray diffraction
11.4 Crystal structure
11.5 Electrical conductivity and the crystal
structure of metals
11.6 The discovery of superconductors
11.7 The Meissner effect
11.8 Type-I and type-II superconductors
11.9 Why is a levitated magnet stable?
11.10 BCS theory and Cooper pairs
11.11 Applications of superconductors
Practical experiences
Chapter summary
Review questions

204
204
205
207
208

Module 3 Review

224

209
211
212
212
213
215
217
220
221
221

Module 4 Quanta to Quarks


Module 4 Introduction

226

Chapter 12 From Rutherford to Bohr


228
12.1 Atomic timeline
228
12.2 Rutherfords model of the atom
229
12.3 Plancks quantised energy
231
12.4 Spectral analysis
232
12.5 Bohrs model of the atom
235
12.6 Bohrs explanation of the Balmer series
236
12.7 Limitations of the RutherfordBohr model 239
Practical experiences
241
Chapter summary
242
Review questions
243
Chapter 13 Birth of quantum mechanics
13.1 The birth
13.2 Louis de Broglies proposal
13.3 Diffraction
13.4 Confirming de Broglies hypothesis
13.5 Electron orbits revisited
13.6 Further developments of atomic theory
19241930
Practical experiences
Chapter summary
Review questions
iv

247
247
248
250
251
252
253
256
256
257

Chapter 14 20th century alchemists


14.1 Discovery of the neutron
14.2 The need for the strong force
14.3 Atoms and isotopes
14.4 Transmutation
14.5 The neutrino
14.6 Was Einstein right?
14.7 Binding energy
14.8 Nuclear fission
14.9 Chain reactions
14.10 Neutron scattering
Practical experiences
Chapter summary
Review questions

260
260
261
262
263
265
266
268
269
270
272
273
274
275

Chapter 15 The particle zoo


15.1 The Manhattan Project
15.2 Nuclear fission reactors
15.3 Radioisotopes
15.4 Particle accelerators
15.5 The Standard Model
Practical experiences
Chapter summary
Review questions

279
279
280
282
286
292
295
296
297

Module 4 Review

300

Module 5 Medical Physics


Module 5 Introduction

302

Chapter 16 Imaging with ultrasound


16.1 What is ultrasound?
16.2 Principles of ultrasound imaging
16.3 Piezoelectric transducers
16.4 Acoustic impedance
16.5 Types of scans
16.6 Ultrasound at work
Practical experiences
Chapter summary
Review questions

304
304
305
308
310
312
315
317
318
318

Chapter 17 Imaging with X-rays


17.1 Overview and history: types of X-ray images
17.2 The X-ray tube
17.3 Types of X-rays
17.4 Production of X-ray images
17.5 X-ray detector technology
17.6 Production of CAT X-ray images
17.7 Benefits of CAT scans over conventional
radiographs and ultrasound
Practical experiences
Chapter summary
Review questions

320
320
321
322
324
326
326
329
330
331
331

Contents

Chapter 18 Imaging with light


18.1 Endoscopy
18.2 Medical uses of endoscopes
Practical experiences
Chapter summary
Review questions

333
333
336
338
339
339

Chapter 19 Imaging with gamma rays


19.1 Isotopes and radioactive decay
19.2 Half-life
19.3 Radiopharmaceuticals: targeting tissues
and organs
19.4 The gamma camera
19.5 Positron emission tomography
Practical experiences
Chapter summary
Review questions

340
340
343

Chapter 20 Imaging with radio waves


20.1 Spin and magnetism
20.2 Hydrogen in a magnetic field
20.3 Tuning in to hydrogen
20.4 It depends on how and where you look
20.5 The MRI scanner
20.6 Applications of MRI
Practical experiences
Chapter summary
Review questions

354
354
355
357
359
360
362
363
364
364

Module 5 Review

366

344
346
347
350
351
351

Module 6 Astrophysics
Module 6 Introduction

368

Chapter 21 Eyes on the sky


21.1 The first telescopes
21.2 Looking up
21.3 The telescopic view
21.4 Sharpening the image
21.5 Interferometry
21.6 Future telescopes
Practical experiences
Chapter summary
Review questions

370
370
373
374
377
380
382
383
384
384

Chapter 22 Measuring the stars


22.1 How far?
22.2 Light is the key
22.3 The stellar alphabet
22.4 Measuring magnitudes
22.5 Colour matters
Practical experiences
Chapter summary
Review questions

388
388
389
394
397
400
403
405
405

Chapter 23 Stellar companions and variables


23.1 Binary stars
23.2 Doubly different
23.3 Variable stars
23.4 Cepheid variables
Practical experiences
Chapter summary
Review questions

407
407
411
413
415
418
418
419

Chapter 24 Birth, life and death


24.1 The ISM
24.2 Star birth
24.3 Stars in the prime of life
24.4 Where to for the Sun?
24.5 The fate of massive stars
24.6 How do we know?
Practical experiences
Chapter summary
Review questions

422
422
423
425
428
430
433
435
436
436

Module 6 Review

438

Module 7 Skills
Module 7 Introduction

440

Chapter 25 Skills stage 2


25.1 Metric prefixes
25.2 Numerical calculations
25.3 Sourcing experimental errors
25.4 Presenting research for an exam
25.5 Australian scientist
25.6 Linearising a formula

442
442
443
445
446
447
447

Chapter 26 Revisiting the BOS key terms


26.1 Steps to answering questions

448
449

Numerical answers
Glossary
Index
Acknowledgements
Formulae and data sheets
Periodic table

452
454
465
471
473
474

S
C
I
S
Y
PH
@ HSC
AGE FOR NSW STUDENTS
CK
PA
S
IC
YS
PH
E
ET
PL
M
CO
THE
in2 Physics is the most up-to-date physics package written for the NSW Stage 6 Physics syllabus. The
materials comprehensively address the syllabus outcomes and thoroughly prepare students for the HSC exam.
Physics is presented as an exciting, relevant and fascinating discipline. The student materials provide
clear and easy access to the content and theory, regular review questions, a full range of exam-style
questions and features to develop an interest in the subject.

in2 Physics @ HSC student book


The student book closely follows the NSW Stage 6 Physics
syllabus and its modular structure.
It clearly addresses both the contexts and the prescribed focus
areas (PFAs).
Modules consist of chapters that are broken up into
manageable sections.
Checkpoint questions review key content at
8
regular intervals throughout each chapter.
Physics Philes present short, interesting
snippets of relevant information about
physics or physics applications.
Physics Features highlight important real-life
examples of physics.
Physics For FunTry This! provide hands-on
activities that are easy to do.
Physics Focus brings together physics concepts
in the context of one or more PFAs and provides
students with a graded set of questions to
develop their skills in this vital area.

From ideas to
on
implementati

From
cathode rays
to television

glass

anode (positive)

electrons
'boil' off
the heated
cathode

collimator

electron
beam

heater

cathode (negative)

Figure 8.5.3

electrons attracted
to the positive anode

The components of an electron gun used in


oscilloscopes and CRT televisions

both cathode ray

Television

electron gun

magnetic
coils

fluorescent screen

Figure 8.5.5

A television picture tube


showing the electron gun,
deflection coils and
fluorescent screen

only one beam.

Each student book includes an interactive student CD containing:


an electronic version of the student book.
all of the student materials on the companion website with live
links to the website.
vi

mask
blue
beam

electron
guns

red
beam

R
G
B

green
beam

focusing
coils

Time

sawtooth voltage for timebase

Figure 8.5.4

Time

mask

fluorescent
screen

fluorescent
screen

electron
beams

holes in
mask

Figure 8.5.6

guns
A colour CRT television set has three electron
that will only strike their respective coloured
phosphor dots with the aid of a shadow mask.

sinusoidal vertical voltage

A sawtooth voltage waveform on the horizontal


beam
deflection plates of a CRO sweeps the electron
waveform
across the screen to display the sinusoidal
on the vertical deflection plates.

used the principles of the cathode ray


Cathode ray tube (CRT) television sets
are now being superseded by plasma
tube for most of the 20th century. These
which use different operating principles
and liquid crystal display television sets,
sharper image. However, the CRT
and allow a larger display area with a
place in this form of
television holds quite a significant historical
communication.
television set is shown in Figure 8.5.5.
A schematic diagram of a colour CRT
of the CRO. The main difference is the
Its basic elements are similar to those
field coils placed outside the tube
method of deflecting the electrons. Magnetic
fields inside it. The magnitude and
produce horizontal and vertical magnetic
degree and direction of electron beam
direction of the current determine the
rule for the force on charged particles
deflection. Recall your right-hand palm
field will deflect the electrons
in a magnetic field. The vertical magnetic
deflect them vertically.
horizontally; the horizontal field will
scanning the beam from left to right
The picture on the screen is formed by
the television switches the beam on and
and top to bottom. The electronics in
in order to reproduce the transmitted
off at the appropriate spots on the screen
images, colour television sets need to
picture. However, to reproduce colour
green phosphors on the screen. Three
and
blue
red,
of
intensity
the
control
one aimed at one particular colour. The
separate electron guns are used, each
in groups of red, blue and green dots
coloured dots on the screen are clustered
cannot be distinguished by eye
that are very close to each other and generally
For this reason a method of guiding the
without the aid of a magnifying glass.
coloured dots was devised. A metal
different electron beams to their respective
8.5.6) and consisting of an array of
sheet, known as a shadow mask (Figure
hole guides the three beams to
Each
screen.
holes, is placed behind the phosphor
the beams move horizontally and vertically.
their respective coloured phosphor as
need the shadow mask since they had
Black and white television sets did not

168

phosphor dots
on screen

vacuum

beam to move up or down in


The vertical deflection plates cause the
For example, a sinusoidal voltage will
synchronisation with an input voltage.
as a trace) on the screen.
display a sinusoidal waveform (known

electron
beam

deflecting
coils

try this!
Do not aDjust your
horizontal!
If you have access to an old
black and white TV set or an old
style monochrome computer
monitor, try holding a bar
magnet near the front of the
screen and watch how the
image distorts. This occurs
because the magnetic field
deflects the electrons that strike
the screen. DO NOT do this with

a colour TV set. This can


magnetise the shadow mask and
cause permanent distortion of
the image and its colour. You
can move a bar magnet near the
back of a colour TV set to
deflect the electrons from the
electron gun and therefore
distort or shift the image
without causing permanent
damage to the TV set.

used
Can an osCillosCope be
as a television set?

ray oscilloscope (CRO) and CRT


he similarity between the cathode
be used as a television set. In
television suggests that a CRO can
that have made use of the CRO as
fact, there have been some devices
in principle, it can be used as a
you would a computer monitor. So,
why did they need to deflect the
television. One is then forced to ask
fields rather than with electric
beam in a television set with magnetic

fields as in the CRO?


be made in the same design as
In principle all television sets could
with
and cheaper to deflect the beam
a CRO; however, it is much easier
the tube rather than embed electrodes
a magnetic field on the outside of
is a little trickier. So now
in the glass and inside the vacuumthis
deflect the beam of the CRO using
another question arises: why not
CROs?.
in cheaper
magnetic fields, wouldnt it result
instruments. The horizontal
Cathode ray oscilloscopes are precision
to very high frequencies in order
sweep rate must be able to be increased
to
quickly. Electric fields can be made
to detect signals that change very
extra power requirements.
change very quickly without significant
system requires higher and higher
However, a magnetically deflected
in
and vertical deflection frequencies
voltages with increasing horizontal
same
the
in the coils, and therefore,
order to maintain the same current
a significantly greater power
angle of beam deflection thus having
sets, however, only operate at
requirement. Cathode ray tube television
horizontal and vertical frequencies.
fixed and relatively low scanning
the mass market to deflect with a
Thus it is simpler and cheaper for
magnetic field.

CheCkpoint 8.5
1
2
3
4

Outline the purpose of a CRO.


List the main parts of a CRO.
in the CRO.
Describe the role of each of these parts
the cathode ray tube CRO and CRT TV.
State the similarities and differences between
169

in2 Physics @ HSC Activity Manual


Chapter 8
from cathode rays
to television

MODULE

A write-in workbook
that provides a
structured approach
to the mandatory
practical experiences,
both first-hand and
secondary-source
investigations.
Dot point and skills
focused.

from ideas to
tion
implementa

Chapter 8

from Cathode rays


to television
aCtivity 8.1
first-hand investigation

Changing pressure of
discharge tubes

the occurrence of different striation


first-hand information to observe
Perorm an investigation and gather
discharge tubes.
patterns for different pressures in

Physics skills

in this activity include:


The skills outcomes to be practised
12.1 perform first-hand investigations
12.2 gather first-hand information
14.1 analyse information
syllabus grid on pages viviii.
skills outcomes can be found in the
The complete statement of these

Aim

To observe the striation patterns for different

different patterns that


Because of this development,
could be seen depended on the pressure.
the air molecules. To do this,
but it can be made to conduct by ionising
Normally air is considered to be an insulator,
an electric field). At high pressures these
that are always in air are accelerated (with
to ionise the air
energy
the very small fraction of free electrons
sufficient
gain
not
losing their energy and, as a result, do
electrons collide frequently with the air,
molecules, thereby acquiring enough
travel further before colliding with air
they are
atoms. As pressure is reduced, these electrons
in turn, can ionise other atoms. When
will produce more free electrons that,
energy to ionise the air molecules. This
show (known as a discharge). The lower
to be absorbed by atoms, we see a light
discharge.
a
able to travel far enough to gain the energy
producing
and
can travel before colliding with gas molecules
the pressure, the further the electrons
excited (increasing in energy) and
electrons around the gas atom becoming
will also
The light that is emitted is a result of the
energy they can have in an atom). Light
lowest
(the
state
ground
the
to
return
re-emitting the photon of light as they
ground state, emitting photons. As every
with ions and the electrons return to the
be produced when free electrons recombine
the element with which the electron collides.
the colour of light seen will vary with
element has a distinct set of energy levels,
be passed through air. The
it was found that electric current could

Equipment

The patterns are


can carry out the experiment first hand.
If you have the apparatus at school, you
is very dark.
discharge tubes at different pressures
induction coil
DC power supply
connecting wires
Alternatively, you can use the simulations

them.

Risk assessment

Method
1
2
3
4

8.1.1.
Set up the equipment as shown in Figure
in the tube.
Observe the patterns and note the pressure
series.
Replace the tube with the next in the
and
Repeat the process of observing the patterns
your set.
in
tubes
the
noting the pressure for each of

DC power supply

tube
Figure 8.1.1 Induction coil and discharge

pressures in discharge tubes.

Hypothesis

in Part B and make observations from

hard to see unless the room

HAZARD

The voltages necessary to operate the


coils and may produce unwanted X-rays.
used,
High voltages are produced by induction
the tube. Generally, the higher the voltage
the tube and the pressure of the gas in
tubes depend upon the dimensions of
of unwanted X-rays.
the greater the danger of the production
a minimum of 1 m away from the equipment.
Use the lowest possible voltage and stand

Theory

which the pressure could be reduced


Plcker collaborated to create a tube in
Ever since Heinrich Geissler and Julius
ray tube have advanced tremendously.
atom and developing uses for the cathode
substantially, our understanding of the

69

68

in2 Physics @ HSC Teacher Resource


Editable teaching materials, including teaching
programs, so that teachers can tailor lessons to
suit their classroom.
Answers to student book and activity manual
questions, with fully worked solutions and
extended answers and support notes.
Risk assessments for all first-hand
investigations.

in2 Physics @ HSC companion website


Visit the companion website
in the student lounge
and teacher lounge
of Pearson Places
Review questions
auto-correcting multiple-choice
questions for each chapter.
Web destinationsa list
of reviewed websites that support
further investigation.

For more information on the in2 Physics series,


visit www.pearsonplaces.com.au
vii

How to use this book


in2 Physics @ HSC is structured to enhance student
learning and their enjoyment of learning. It contains many
outstanding and unique features that will assist students
succeed in Stage 6 Physics. These include:
Module opening pages introduce a range of contexts for
study, as well as an inquiry activity that provides
immediate activities for exploration and discussion.

2
Context

Key ideas are clearly highlighted with a


and
indicate where domain dot points
Syllabus flags
appear in the student book. The flags are placed as
closely as possible to where the relevant content is
covered. Flags may be repeated if the dot point has
multiple parts, is complex or where students are
required to solve problems.
3

Motors and
Generators

l v = l0 1
tv =

mv =

Figure 4.0.2

of interference of electromagnetic radiation, and examine how this was applied to


crystals using X-rays. Then we will see how the BCS theory of superconductivity
made use of the crystal structure of matter.

try thiS!

CheCkpoInT 11.1

Crystals in the kitChen

Explain what is meant by the crystal structure of matter.

Look at salt grains through a


magnifying lens. Each grain is
a single crystal that is made from
the basic arrangement of sodium
and chlorine atoms shown in
Figure 11.1.1. Although the
grains mostly look irregular due
to breaking and chipping during
the manufacturing process,
occasionally you will see an
untouched cubic or rectangular
prism that reflects the underlying
crystal lattice structure.

11.2 Wave interference


The wave nature of light can be used to measure the size of very small spaces.
Recall that two identical waves combine to produce a wave of greater amplitude
when their crests overlap, as shown in Figure 11.2.1a (see in2 Physics @
Preliminary sections 6.4 and 7.4). The overlapping waves will cancel to produce
t=0s
a resulting wave of zero amplitude when the crest of one wave coincides with the
trough of the other (Figure 11.2.1b). This addition and subtraction is called
constructive and destructive interference respectively and is a property of all
wave phenomena.
t=1s
As an example, two identical circular water waves in a ripple tank overlap (see
Figure 11.2.2). The regions of constructive and destructive interference radiate
outwards along the lines as shown. Increasing the spacing between the sources
t = 3 s (Figure 11.2.2b).
causes the radiating lines to come closer together
a

Figure 11.2.1 Two identical waves (red, green) travelling in opposite directions can add (blue)
t=1s

(a) constructively or (b) destructively.t = 5 s

The interference of identical waves from two sources can also be represented
by outwardly radiating transverse waves (see Figure 11.2.3). The distance that a
twave
= 3 s travels is known as its path length. t = 6 s Constructive interference occurs
when the difference in the path length of the two waves is equal to 0, , 2, 3,
4 or any other integer multiple of the wavelength . Destructive interference
occurs
when the two waves are half a wavelength out of step. This corresponds to
t=4s
t=7s
a path length difference of /2, 3/2, 5/2 etc.

t=5s

lines of destructive
interference

lines of constructive
interference

t=4s

t=0s

11.1 The crystal structure of matter

waves
in phase

destructive
interference
constructive
interference

204

t=7s

1
2

evil tWinS

he most extreme massenergy


conversion involves antimatter.
For every kind of matter particle
there is an equivalent antimatter
particle, an evil twin, bearing
properties (such as charge) of
opposite sign. Particles and their
antiparticles have the same rest
mass. When a particle meets its
antiparticle, they mutually
annihilateall their opposing
properties cancel, leaving only
their massenergy, which is
usually released in the form of
two gammaray photons. Matter
antimatter annihilation has been
suggested (speculatively) as a
possible propellant for powering
future interstellar spacecraft.

1 v2
1
m0c 1 + 2 = m0c 2 + m0v 2
2
2 c

E = mc 2

Figure 3.4.6

One of the four ultra-precise superconducting spherical


gyroscopes on NASAs Gravity Probe B, which orbited
Earth in 2004/05 to measure two predictions of general
relativity: the bending of spacetime by the Earths
mass and the slight twisting of spacetime by the
Earths rotation (frame-dragging)

In general relativity, Einstein showed that gravity


occurs because objects with mass or energy cause this
4D spacetime to become distorted. The paths of
objects through this distorted 4D spacetime appear to
our 3D eyes to follow the sort of astronomical
trajectories you learned about in Chapter 2 Explaining
and exploring the solar system. However, unlike
Newtons gravitation, general relativity is able to handle
situations of high gravitational fields, such as
Mercurys precessing orbit around the Sun and black
holes. General relativity also predicts another wave that
doesnt require a medium: the ripples in spacetime
called gravity waves.

where m is any kind of mass. In relativity, mass and energy are regarded as the
same thing, apart from the change of units. Sometimes the term mass-energy is
used for both. m0 c 2 is called the rest energy, so even a stationary object contains
energy due to its rest mass. Relativistic kinetic energy therefore:
m0c 2
mv c 2 m0c 2 =
m0c 2
v2
1 2
c
Whenever energy increases, so does mass. Any release of energy is
accompanied by a decrease in mass. A book sitting on the top shelf has a slightly
higher mass than one on the bottom shelf because of the difference in
gravitational potential energy. An objects mass increases slightly when it is hot
because the kinetic energy of the vibrating atoms is higher.
Because c 2 is such a large number, a very tiny mass is equivalent to a large
amount of energy. In the early days of nuclear physics, E = mc 2 revealed the
enormous energy locked up inside an atoms nucleus by the strong nuclear force
that holds the protons and neutrons together. It was this that alerted nuclear
physicists just before World War II to the possibility of a nuclear bomb. The
energy released by the nuclear bomb dropped on Hiroshima at the end of that
war (smallish by modern standards) resulted from a reduction in relativistic mass
of about 0.7 g (slightly less than the mass of a standard wire paperclip).

Discuss the implications of


mass increase, time dilation
and length contraction for
space travel.

Worked example
qUESTIon
When free protons and neutrons become bound together to form a nucleus, the reduction in
nuclear potential energy (binding energy) is released, normally in the form of gamma rays.
Relativity says this loss in energy is reflected in a decrease in mass of the resulting atom.

Each chapter concludes with:


a chapter summary
review questions, including literacy-based questions
(Physically Speaking), chapter review questions
(Reviewing) and physics problems (Solving
Problems). Syllabus verbs are clearly highlighted as
and where appropriate
Physics Focusa unique feature that places key
chapter concepts in the context of one or more
prescribed focus areas.

Figure 11.2.2 Interference of water waves for


two sources that are (a) close
together and (b) further apart

19

Imaging with
gamma rays

PRACTICAL EXPERIENCES

ChAPTER 19

This is a starting point to get you thinking about the mandatory practical
experiences outlined in the syllabus. For detailed instructions and advice, use
in2 Physics @ HSC Activity Manual.

Figure 11.2.3 Constructive and destructive interference between

Activity 19.1: Bone scAns

identical transverse waves from two sources

Perform an investigation to
compare a bone scan with
an X-ray image.

205

Chapters are divided into short, accessible sections


the text itself is presented in short, easy-to-understand
chunks of information. Each section concludes with
a Checkpointa set of review questions to check
understanding of key content and concepts.

A bone scan is performed to obtain a functional image of the bones and so can be
used to detect abnormal metabolism in the bones, which may be an indication of
cancer or other abnormality. Because cancer mostly involves a higher than normal
rate of cell division (thus producing a tumour), chemicals
involved in metabolic processes in bone tend to accumulate in
higher concentrations in cancerous tissue. This produces areas
of concentration of gamma emission, indicating a tumour.
Compare the data obtained from the image of a bone scan
with that provided by an X-ray image.
Discussion questions
1 Identify the best part of the body for each of these
diagnostic tools to image.
2 Compare and contrast the two images in terms of
the information they provide.

Figure 19.6.1

Chapter summary

The number of protons in a nucleus is given by the


atomic number, while the total number of nucleons is
given by the mass number.
Atoms of the same element with different numbers of
neutrons are called isotopes of that element.
Many elements have naturally occurring unstable
radioisotopes.
In alpha decay an unstable nucleus decays by emitting
an alpha particle (-particle).
In beta decay, a neutron changes into a proton and
a high-energy electron that is emitted as a beta particle
(-particle).
In positron decay, a positronthe antiparticle of the
electronis emitted.

Activity 19.2: HeAltHy or diseAsed?


Typical images of healthy bone and cancerous bone are shown. The tumours show
up as hot-spots. Use the template in the activity manual to research and compare
images of healthy and diseased parts of the body.
Discussion questions
1 Examine Figure 19.4.2. There is a hot-spot that is not cancerous near the
left elbow. Explain.
2 In the normal scan (Figure 19.6.2a), the lower pelvis has a region of high
intensity. Why is this? (Hint: It may be soft tissue, not bone. Looking at
Figure 19.6.2b might help you with this question.)
3 State the differences that can be observed by comparing an image of
a healthy part of the body with that of a diseased part of the body.

PHysicAlly sPeAking
Below is a list of topics that have been discussed throughout
this chapter. Create a visual summary of the concepts in
this chapter by constructing a mind map linking the terms.
Add diagrams where useful.

Radioactive
decay

350

Radiation

Radioisotope

Neutron

Proton

Beta decay

Gamma
decay

Antimatter

Bone scan

Positron
decay

Half-life

Bones scans of (a) a healthy person and


(b) a patient with a tumour in the skeleton

mEdICAL
PhySICS

When a positron and an electron collide, their total


mass is converted into energy in the form of two
gamma-ray photons.
In gamma decay a gamma ray (g) is emitted from a
radioactive isotope.
The time it takes for half the mass of a radioactive
parent isotope to decay into its daughter nuclei is the
half-life of the isotope.
Artificial radioisotopes are produced in two main ways:
in a nuclear reactor or in a cyclotron.
A gamma camera detects gamma rays emitted by
a radiopharmaceutical in the patients body.
PET imaging uses positron-emitting
radiopharmaceuticals to obtain images using gamma
rays emitted from electronpositron annihilation.

Review questions

Comparison of an X-ray and bone scan of a hand

Gather and process secondary


information to compare a
scanned image of at least one
healthy body part or organ with
a scanned image of its
diseased counterpart.

Figure 19.6.2

viii

73

constructive
interference

t=6s

1
2

72

Figure 11.1.1 Crystal structure of sodium chloride. The red spheres represent positive
sodium ions, and the green spheres represent negative chlorine ions.

1
Rearrange:
mvc 2 m0c 2 = (mv m0)c 2 m0v 2
2
In other words, at low speeds, the gain in relativistic mass (mv m0)
multiplied by c 2 equals the kinetic energya tantalising hint that at low speed
mass and energy are equivalent. It can also be shown to be true at all speeds,
using more sophisticated mathematics. In general, mass and energy are
equivalent in relativity and c 2 is the conversion factor between the energy unit
(joules) and the mass unit (kg). In other words:

c2

here are two more invariants in special relativity.


Maxwells equations (and hence relativity)
requires that electrical charge is invariant in all
frames. Another quantity invariant in all inertial frames
is called the spacetime interval.
You may have heard of spacetime but not know
what it is. One of Einsteins mathematics lecturers
Hermann Minkowski (18641909) showed that the
equations of relativity and Maxwells equations become
simplified if you assume that the three dimensions of
space (x, y, z) and time t taken together form a
fourdimensional coordinate system called spacetime.
Each location in spacetime is not a position, but rather
an eventa position and a time.
Using a 4D version of Pythagoras theorem,
Minkowski then defined a kind of 4D distance
between events called the spacetime interval s given by:
s 2 = (c time period)2 path length2
= c 2t 2 ((x)2 + (y)2 + (z)2)
Observers in different frames dont agree on the
3D path length between events, or the time period
between events, but all observers in inertial frames
agree on the spacetime interval s between events.

from ideaS to
implementation

A crystal is a three-dimensional regular arrangement of atoms. Figure 11.1.1


shows a sodium chloride crystal (ordinary salt also called rock salt when it comes
as a large crystal). The crystal is made from simple cubes repeated many times,
with sodium and chlorine atoms at the corners of the cubes. Crystals of other
materials may have different regular arrangements of their atoms. There are
14 types of crystal arrangements that solids can have.
The regular arrangement of atoms in crystals was a hypothesis before
Max Von Laue and his colleagues confirmed it by X-ray diffraction experiments.
William and Lawrence Bragg took this method one step further by measuring
the spacing between the atoms in the crystal. Let us first look at the phenomenon

v2
m0c 2 1 2
c

Surprising discovery
crystal, constructive interference,
destructive interference, path length,
diffraction grating, Bragg law,
phonons, critical temperature,
type-I superconductors,
type-II superconductors,
critical field strength, vortices,
flux pinning, BCS theory, Cooper pair,
coherence length, energy gap, spin

v2
= m0c 2 1 2
c

m0c 2

v2
1 2
c
Using a well-known approximation formula that you might learn at university,
(1 x )n 1 nx for small x:

1. The history of physics

InQUIRY ACtIVItY

Chapter openings list the key words of each chapter and


introduce the chapter topic in a concise and engaging way.

Just as an improved understanding of the conducting properties of


semiconductors led to the wide variety of electronic devices, research
into the conductivity of metals produced quite a surprising discovery
called superconductivity. This is the total disappearance of electrical
resistance below a certain temperature, which has great potential
applications ranging from energy transmission and storage to public
transport. An understanding of this phenomenon required a detailed
understanding of the crystal structure of conductors and the motion
of electrons through them.

mv c 2 =

m0

TwISTIng SPACETImE ...


And YoUR mInd

A simple homopolar motor

83

Superconductivity

c2

c2

82

11

v2

v2

PHYSICS FEATURE

Many of the devices you use every day have electric motors. They spin your DVDs,
wash your clothes and even help cook your food. Could you live without them,
and how much do you know about how they work?
The essential ingredients for a motor are a power source, a magnetic field
and things to connect these together in the right way. Its not as hard as you
think. All you need is a battery, a wood screw, a piece of wire and a cylindrical or
spherical magnet. Put these things together as shown in Figure 4.0.2 and see
if you can get your motor to spin. Be patient and keep trying. Then try the
following activities.
1 Test the effects of changing the voltage you use. You could add another
battery in series or try a battery with a higher voltage.
2 Try changing the strength of the magnet by using a different magnet or
adding another. What does this affect?
3 Try changing the length of the screw, how sharp its point is or the material
it is made from. Does it have to be made of iron?

A generator produces electricity


in each of these wind turbines.

The kinetic energy formula K = 1 mv 2 doesnt apply at relativistic speeds,


2
even if you substitute relativistic mass mv into the formula. Classically, if you
apply a net force to accelerate an object, the work done equals the increase in
kinetic energy. An increase in speed means an increase in kinetic energy. But
in relativity it also means an increase in relativistic mass, so relativistic mass
and energy seem to be associated. Superficially, if you multiply relativistic
mass by c 2 you get mv c 2, which has the same dimensions and units as energy.
But lets look more closely at it.

t0
1

The first recorded observations of the relationship between electricity and


magnetism date back more than 400 years. Many unimagined discoveries
followed, but progress never waits. Before we understood their nature, inventions
utilising electricity and magnetism had changed our world forever.
Today our lives revolve around these forms of energy. The lights you use to
read this book rely on them and the CD inside it would be nothing but a shiny
coaster for your cup. We use magnetism to generate the electricity that drives
industry, discovery and invention. Electricity and magnetism are a foundation for
modern technology, deeply seated in the global economy, and our use impacts
heavily on the environment.
The greatest challenge that faces future generations is the supply of energy.
As fossil fuels dry up, electricity and magnetism will become even more
important. New and improved technologies will be needed. Whether its a hybrid
car, a wind turbine or a nuclear fusion power plant, they all rely on applications
of electricity and magnetism.

Space
How does this formula behave at low speeds (when v 2/c 2 is small)?

Mass, energy and the worlds most famous equation

Solve problems and analyse


information using:
E = mc2

BUIld YoUR own eleCtRIC motoR

Figure 4.0.1

Seeing in a
weird light:
relativity

Isotope

reviewing
1

Recall how the bone scan produced by a radioisotope


compares with that from a conventional X-ray.

Analyse the relationship between the half-life of


a radiopharmaceutical and its potential use in the
human body.

Explain how it is possible to emit an electron from the


nucleus when the electron is not a nucleon.

Assess the statement that Positrons are radioactive


particles produced when a proton decays.

Discuss the impact that the production and use of


radioisotopes has on society.

Describe how isotopes such as Tc-99m and F-18 can


be used to target specific organs to be imaged.

Use the data in Table 19.6.1 to answer the questions:


a Which radioactive isotope would most likely be
used in a bone scan? Justify your choice.
b Propose two reasons why cesium-137 would not
be a suitable isotope to use in medical imaging.

Nucleon

Alpha decay

PET

Table 19.6.1
Scintillator

Properties of some radioisotopes

Radioactive souRce

Radiation emitted

Half-life

C-11
Tc-99m
TI-201
I-131
Cs-137
U-238

+, g
g
g
, g

20.30 minutes
6.02 hours
3.05 days
8.04 days
30.17 years
4.47 109 years

351

How to use this book

Other features

Module reviews provide a full range of exam-style


questions, including multiple-choice, short-response
and extended-response questions.

from ideas to
implementation
4

The review contains questions in a similar style and proportion


to the HSC Physics examination. Marks are allocated to
each question up to a total of 25 marks. It should take you
approximately 45 minutes to complete this review.
5

Experimental data from black body radiation during


Plancks time showed that predicted radiation levels
were not achieved in reality. Planck best described
this anomaly by saying that:
A classical physics was wrong.
B radiation that is emitted and absorbed is
quantised.
C he had no explanation for it.
D quantum mechanics needed to be developed.

extended response

Figure 11.13.4 shows a cathode ray tube that has


been evacuated. Which answer correctly names each
of the labelled features?

III

Explain, with reference to atomic models, why


cathode rays can travel through metals. (2 marks)

Outline how the cathode ray tube in a TV works


in order to produce the viewing picture. (2 marks)

Give reasons why CRT TVs use magnetic coils and


CROs use electric plates in order to deflect the
beams, given that both methods work. (2 marks).

In your studies you were required to gather


information to describe how the photoelectric effect
is used in photocells.
a Explain how you determined which material was
relevant and reliable.
b Outline how the photoelectric effect is used in
photocells. (3 marks)

II

multiple choice
(1 mark each)
1 Predict the direction of the electron in Figure 11.13.1
as it enters the magnetic field.
A Straight up
B Left
C Right
D Down

A
B

Figure 11.13.1

An electron in a magnetic field


C

The diagrams in Figure 11.13.2 represent


semiconductors, conductors and insulators. The
diagrams show the conduction and valence bands,
and the energy gaps. Which answer correctly labels
each of the diagrams?

A
B
C
D

Figure 11.13.4 An evacuated cathode ray tube

II

III

Conductor
Insulator
Insulator
Semiconductor

Insulator
Conductor
Semiconductor
Conductor

Semiconductor
Semiconductor
Conductor
Insulator

A
B
C
D

II

III

Critical
temperature
Superconductor
material
Critical
temperature
Normal material

Superconductor
material
Critical
temperature
Normal material

Normal material

Superconductor
material

II

Figure 11.13.2

The graph in Figure 11.13.3 shows how the


resistance of a material varies with temperature.
Identify each of the parts labelled on the graph.

Superconductor
material
Critical
temperature

II

III

Cathode
Striations

Anode
Cathode

Anode

Faradays
dark space
Striations

Faradays
dark space

10

Justify the introduction of semiconductors to replace


thermionic devices. (4 marks)

11

Magnetic levitation trains are used in Germany and


Japan. The trains in Germany use conventional
electromagnets, whereas the one in Japan uses
superconductors. Compare and contrast the two
systems. (3 marks)

12

Determine the frequency of red light, which has


a wavelength = 660 nm. (Speed of light
c = 3.00 108 m s1)
Calculate the energy of a photon that is emitted
with this wavelength. (Plancks constant
h = 6.63 1034 J s) (4 marks)

Figure 11.13.3

Physics for FunTry This! activities are short, handson activities to be done quickly, designed to provoke
discussion.
Physics Features are a key feature as they highlight
contextual material, case studies or prescribed focus
areas of the syllabus.

III

II

Normal material

I
Striations
Faradays
dark space
Crookes
dark space
Cathode

A complete glossary of all the key words is included at


the end of the student book.

Energy bands

Resistance ()

Physics Philes present short, interesting items to


support or extend the text.

III

Temperature (K)

Resistance varies with temperature

224

225

Practical experiences
The accompanying activity manual covers all of the
mandatory practical experiences outlined in the syllabus.
in2 Physics @ HSC Activity Manual is a write-in
workbook that outlines a clear, foolproof approach to
success in all the required practical experiences.
Within the student book, there are clear cross-references
to the activity manual: Practical Experiences icons refer to
the activity number and page in the activity manual. In
each chapter, a summary of possible investigations is
provided as a starting point to get
students thinking. These include
PRACTICAL
the aim, a list of equipment and
EXPERIENCES
Activity 10.2
discussion questions.
Activity Man

The final two chapters provide essential reference


material: Skills stage 2 and Revisiting the BOS
key terms.
In all questions and activities, except module review
questions, the BOS key terms are highlighted.

in2 Physics @ HSC Student CD


This is included with the student book and contains:
an electronic version of the student book
interactive modules demonstrating key concepts

MODULE

ual, Page 94

Chapter 6
motors: magnetic fields
make the world go around

motors and
generators

aCtIVItY 6.2
First-hand investigation

the companion website on CD

Risk assessment

Motors and torque

Solve problems and analyse information about simple motors using:


= nBIA cos

Method

Physics skills

Cut a length of cotton-covered wire so that the wire is long enough


to wrap around the exterior of a matchbox three times (as shown in
Figure 6.2.2).

Leave a straight piece (approx. 10 cm long) hanging out and then wind
the remainder of the wire around the box 2 times. Leave another
straight piece the same length as at the start, on the opposite side.

Wrap the straight pieces around the loops so that they tie both ends.

Fan out the loops so that you get equally spaced loops and that it
looks like a bird cage (see Figure 6.2.3).

Push out the middle of the paper clip as shown and Blu-Tack to
the bench.

Slip the straight pieces of wire through the paper clip supports.
Unwrap the cotton from these parts.

Connect an AC power supply to the paper clips.

Place two magnets so that a north pole and a south pole face on
opposing sides of the cage.

Turn on. You may need to give the cage a tap to get it spinning.

The skills outcomes to be practised in this activity include:


12.4 process information
14.1 analyse information
The complete statement of these skills outcomes can be found in the syllabus grid on pages viiviii.

Aim

Hypothesis

Theory
The motor effect means that a current-carrying wire experiences
a force when placed in a magnetic field. This is the basis for
the workings of a motor.
For a motor to work as needed, the motion resulting from
the motor effect needs to be circular and the force needs to be
adjusted so the direction of rotation does not change.

Question
Figure 6.2.1 shows the simplified workings of a motor that you
will be making. Label all the parts of the motor.

48

insulated wire from which insulation can be removed easily


magnets
magnetic field sensor and data logger (if available)
paperclips

matchbox
wire
b

loop wire
through

Figure 6.2.2 Equipment set-up 1

cage fanned out

paper clip

alligator clip
wires

power
source

Figure 6.2.3 Equipment set-up 2

Record your observations of the motor.

The complete in2 Physics @ HSC package

How did adding more magnets affect how the motor ran?

Remember the other components of the complete package:

When the current is increased, what changes occurred?

Results
N

C:

D:

Figure 6.2.1 Simplified motor

Blu-Tack
connecting wires with alligator clips
power supply

a link to the live companion website (Internet access


required) to provide access to the latest information and
web links related to the student book.

A:
B:

Equipment



in2 Physics @ HSC companion website at Pearson Places


49

in2 Physics @ HSC Teacher Resource.

ix

Stage 6 Physics syllabus grid


Prescribed focus areas
1. The history of physics

H1. evaluates how major advances in scientific understanding and


technology have changed the direction or nature of scientific thinking

Feature: pp. 12, 29, 72

2. The nature and practice of physics

H2. analyses the ways in which models, theories and laws in physics
have been tested and validated

Focus: p. 79

3. Applications and uses of physics

H3. assesses the impact of particular advances in physics on the


development of technologies

Feature: pp. 12, 29, 307,


334, 346

Focus: pp. 25, 246, 299

Focus: pp. 57, 79, 129,


173, 223, 246, 259, 278
4. Implications for society and the
Environment

H4. assesses the impacts of applications of physics on society and the


environment

Feature: pp. 29, 307, 344

5. Current issues, research and


developments in physics

H5. identifies possible future directions of physics research

Feature: pp. 391, 410

Focus: pp. 113, 173, 353


Focus: pp. 79, 113, 173,
223, 353, 386

Module 1 Space
1. The Earth has a gravitational field that exerts a force on objects both on it and around it
Students learn to:

Page Students:

define weight as the force on an object


due to a gravitational field

13

perform an investigation and gather information to determine a value for


Act. 1.2
acceleration due to gravity using pendulum motion or computer-assisted
technology and identify reason(s) for possible variations from the value 9.8ms2

Page

explain that a change in gravitational


potential energy is related to work done

16

gather secondary information to predict the value of acceleration due to gravity


on other planets

Act. 1.3

define gravitational potential energy as


the work done to move an object from
a very large distance away to a point
in a gravitational field:
mm
EP = G 1 2
r

16

analyse information using the expression:

Act. 1.3

F = mg
to determine the weight force for a body on Earth and for the same body
on other planets

2. Many factors have to be taken into account to achieve a successful rocket launch, maintain
a stable orbit and return to Earth
Students learn to:

Page Students:

Page

describe the trajectory of an object


undergoing projectile motion within the
Earths gravitational field in terms of
horizontal and vertical components

7, 9,
23, 24

solve problems and analyse information to calculate the actual velocity of


a projectile from its horizontal and vertical components using:
vx2 = ux2

v = u + at
vy2 = uy2 + 2ayy
x = uxt
y = uyt +12 ayt2

describe Galileos analysis of projectile


motion

perform a first-hand investigation, gather information and analyse data to


calculate initial and final velocity, maximum height reached, range and time of
flight of a projectile for a range of situations by using simulations, data loggers
and computer analysis

explain the concept of escape velocity


in terms of the:
gravitational constant
mass and radius of the planet

18

identify data sources, gather, analyse and present information on the contribution 29
of one of the following to the development of space exploration: Tsiolkovsky,
Act. 2.1
Oberth, Goddard, Esnault-Pelterie, ONeill or von Braun

Act. 1.1

Stage 6
Physics syllabus grid
outline Newtons concept of escape
velocity

18

identify why the term g forces is used


to explain the forces acting on an
astronaut during launch

31

discuss the effect of the Earths orbital


motion and its rotational motion on the
launch of a rocket

34

analyse the changing acceleration of


a rocket during launch in terms of the:
Law of Conservation of Momentum
forces experienced by astronauts

30, 33

analyse the forces involved in uniform


circular motion for a range of objects,
including satellites orbiting the Earth

25, 32, solve problems and analyse information to calculate the centripetal force acting
34, 37, on a satellite undergoing uniform circular motion about the Earth using:
54, 55
mv 2
F = r

compare qualitatively low Earth and


geo-stationary orbits

43

define the term orbital velocity and the 36, 40, solve problems and analyse information using:
quantitative and qualitative relationship 56
r3
GM
=
between orbital velocity, the
2
4
2
T
gravitational constant, mass of the
central body, mass of the satellite and
the radius of the orbit using Keplers
Law of Periods
account for the orbital decay of
satellites in low Earth orbit

46

discuss issues associated with safe


re-entry into the Earths atmosphere
and landing on the Earths surface

47

identify that there is an optimum angle


for safe re-entry for a manned
spacecraft into the Earths atmosphere
and the consequences of failing to
achieve this angle

47

37, 54,
55
Act. 2.2

39, 43,
56

3. The solar system is held together by gravity


Students learn to:

Page Students:

Page

describe a gravitational field in the


region surrounding a massive object in
terms of its effects on other masses
in it

13

present information and use available evidence to discuss the factors affecting
the strength of the gravitational force

Act. 1.3

define Newtons Law of Universal


Gravitation:
mm
F = G 1 22
d

11

solve problems and analyse information using:


mm
F = G 1 22
d

23, 24,
25, 37,
54, 55

discuss the importance of Newtons


Law of Universal Gravitation in
understanding and calculating the
motion of satellites

35, 38

identify that a slingshot effect can be


provided by planets for space probes

44

xi

Stage 6
Physics syllabus grid
4. Current and emerging understanding about time and space has been dependent upon earlier models
of the transmission of light
Students learn to:

Page Students:

Page

outline the features of the aether model 61


for the transmission of light
describe and evaluate the MichelsonMorley attempt to measure the relative
velocity of the Earth through the aether

62

gather and process information to interpret the results of the Michelson-Morley


experiment

62
Act. 3.2

discuss the role of the MichelsonMorley experiments in making


determinations about competing
theories

62

outline the nature of inertial frames of


reference

58

perform an investigation to help distinguish between non-inertial and inertial


frames of reference

60
Act. 3.1

discuss the principle of relativity

58

analyse and interpret some of Einsteins thought experiments involving mirrors


and trains and discuss the relationship between thought and reality

66

describe the significance of Einsteins


assumption of the constancy of the
speed of light

65

analyse information to discuss the relationship between theory and the evidence
supporting it, using Einsteins predictions based on relativity that were made
many years before evidence was available to support it

78

identify that if c is constant then space


and time become relative

65

discuss the concept that length


standards are defined in terms of time
in contrast to the original metre
standard

79

explain qualitatively and quantitatively


the consequence of special relativity in
relation to:
the relativity of simultaneity
the equivalence between mass and
energy
length contraction
time dilation
mass dilation

64, 69 solve problems and analyse information using:


E = mc2
lv = l0 1

v2

66, 69,
72, 77,
78

c2

t0
tv = 1

v2
c2

m0
mv = 1
discuss the implications of mass
increase, time dilation and length
contraction for space travel

v2
c2

70, 73

Module 2 Motors and Generators


1. Motors use the effect of forces on current-carrying conductors in magnetic fields
Students learn to:

Page Students:

Page

discuss the effect on the magnitude of


the force on a current-carrying
conductor of variations in:
the strength of the magnetic field in
which it is located
the magnitude of the current in the
conductor
the length of the conductor in the
external magnetic field
the angle between the direction
of the external magnetic field and
the direction of the length of the
conductor

92

Act. 4.1

xii

perform a first-hand investigation to demonstrate the motor effect

Stage 6
Physics syllabus grid
describe qualitatively and quantitatively 94
the force between long parallel currentcarrying conductors:
II
F
=k 1 2
l
d

solve problems using:


II
F
=k 1 2
l
d

94

define torque as the turning moment of


a force using:
t = Fd

115

solve problems and analyse information about the force on current-carrying


conductors in magnetic fields using:
F = BIlsin

92
Act. 4.1

identify that the motor effect is due to


the force acting on a current-carrying
conductor in a magnetic field

90,
116

solve problems and analyse information about simple motors using:


t = nBIAcos

117
Act. 6.2

describe the forces experienced by


a current-carrying loop in a magnetic
field and describe the net result of
the forces

117

identify data sources, gather and process information to qualitatively describe the 91, 119
application of the motor effect in:
Act. 6.1
the galvanometer
the loudspeaker

describe the main features of a DC


electric motor and the role of each
feature

115

identify that the required magnetic


fields in DC motors can be produced
either by current-carrying coils or
permanent magnets

115

2. The relative motion between a conductor and magnetic field is used to generate an electrical voltage
Students learn to:

Page Students:

outline Michael Faradays discovery of


the generation of an electric current by
a moving magnet

100

perform an investigation to model the generation of an electric current by moving 101


a magnet in a coil or a coil near a magnet
Act. 5.1

Page

define magnetic field strength B as


magnetic flux density

101

plan, choose equipment or resources for, and perform a first-hand investigation


to predict and verify the effect on a generated electric current when:
the distance between the coil and magnet is varied
the strength of the magnet is varied
the relative motion between the coil and the magnet is varied

Act. 5.1

describe the concept of magnetic flux


in terms of magnetic flux density and
surface area

101

gather, analyse and present information to explain how induction is used in


cooktops in electric ranges

108
Act. 5.2

describe generated potential difference


as the rate of change of magnetic flux
through a circuit

103

gather secondary information to identify how eddy currents have been utilised in
electromagnetic braking

Act. 5.2
113

account for Lenzs Law in terms of


conservation of energy and relate it to
the production of back emf in motors

105,
120

explain that, in electric motors, back


emf opposes the supply emf

120

explain the production of eddy currents


in terms of Lenzs Law

106

3. Generators are used to provide large scale power production


Students learn to:

Page Students:

Page

describe the main components of a


generator

131

plan, choose equipment or resources for, and perform a first-hand investigation


to demonstrate the production of an alternating current

Act. 5.1

compare the structure and function of


a generator to an electric motor

135

gather secondary information to discuss advantages/disadvantages of AC and DC


generators and relate these to their use

135
Act. 7.1

describe the differences between AC


and DC generators

135

analyse secondary information on the competition between Westinghouse and


Edison to supply electricity to cities

141
Act. 7.2

gather and analyse information to identify how transmission lines are:


insulated from supporting structures
protected from lightning strikes

146
Act. 7.3

discuss the energy losses that occur as 144


energy is fed through transmission lines
from the generator to the consumer
assess the effects of the development
of AC generators on society and the
environment

147

xiii

Stage 6
Physics syllabus grid
4. Transformers allow generated voltage to be either increased or decreased before it is used
Students learn to:

Page Students:

Page

describe the purpose of transformers in


electrical circuits

136

perform an investigation to model the structure of a transformer to demonstrate


how secondary voltage is produced

Act. 7.3

compare step-up and step-down


transformers

137

solve problems and analyse information about transformers using:


Vp
np
=
Vs
ns

137
Act. 7.3

identify the relationship between the


ratio of the number of turns in the
primary and secondary coils and the
ratio of primary to secondary voltage

137

gather, analyse and use available evidence to discuss how difficulties of heating
caused by eddy currents in transformers may be overcome

139
Act. 7.3

explain why voltage transformations are


related to conservation of energy

139

gather and analyse secondary information to discuss the need for transformers in
the transfer of electrical energy from a power station to its point of use

145
Act. 7.3

explain the role of transformers in


electricity substations

142

discuss why some electrical appliances


in the home that are connected to the
mains domestic power supply use a
transformer

136,
144

discuss the impact of the development


of transformers on society

147

5. Motors are used in industries and the home usually to convert electrical energy into more useful forms
of energy
Students learn to:

Page

Students:

Page

describe the main features of an AC


electric motor

124

perform an investigation to demonstrate the principle of an AC induction motor

Act. 6.3

gather, process and analyse information to identify some of the energy transfers
124,
and transformations involving the conversion of electrical energy into more useful 153
forms in the home and industry
Act. 7.3

Module 3 From Ideas to Implementation


1. Increased understandings of cathode rays led to the development of television
Students learn to:

Students:

Page

explain why the apparent inconsistent


157
behaviour of cathode rays caused
debate as to whether they were charged
particles or electromagnetic waves

Page

perform an investigation and gather first-hand information to observe the


occurrence of different striation patterns for different pressures in discharge
tubes

Act. 8.1

explain that cathode ray tubes allowed


the manipulation of a stream of
charged particles

perform an investigation to demonstrate and identify properties of cathode rays


using discharge tubes:
containing a Maltese cross
containing electric plates
with a fluorescent display screen
containing a glass wheel

Act. 8.2

analyse the information gathered to determine the sign of the charge on


cathode rays

Act. 8.2

solve problem and analyse information using:


F = qvBsin
F = qE
and
V
E=
d

162,
164

157

identify that moving charged particles


in a magnetic field experience a force

164

identify that charged plates produce an


electric field

161

xiv

Stage 6
Physics syllabus grid
describe quantitatively the force acting 164
on a charge moving through a magnetic
field:
F = qvBsin
discuss qualitatively the electric field
strength due to a point charge, positive
and negative charges and oppositely
charged parallel plates

160

describe quantitatively the electric field 161


due to oppositely charged parallel plates
outline Thomsons experiment to
measure the charge/mass ratio of an
electron

165

outline the role of:


electrodes in the electron gun
the deflection plates or coils
the fluorescent screen
in the cathode ray tube of
conventional TV displays and
oscilloscopes

167

2. The reconceptualisation of the model of light led to an understanding of the photoelectric effect and
black body radiation
Students learn to:

Page

Students:

Page

describe Hertzs observation of the


effect of a radio wave on a receiver and
the photoelectric effect he produced
but failed to investigate

182

perform an investigation to demonstrate the production and reception of


radio waves

Act. 9.1

outline qualitatively Hertzs experiments 175


in measuring the speed of radio waves
and how they relate to light waves

identify data sources, gather, process and analyse information and use available
evidence to assess Einsteins contribution to quantum theory and its relation to
black body radiation

Act. 9.2

identify Plancks hypothesis that


radiation emitted and absorbed by the
walls of a black body cavity is
quantised

179

identify data sources, gather, process and present information to summarise the
use of the photoelectric effect in photocells

184
Act. 9.3

identify Einsteins contribution to


quantum theory and its relation to
black body radiation

179

solve problems and analyse information using:


E = hf
and
c = f

181
Act. 9.3

explain the particle model of light in


terms of photons with particular energy
and frequency

179

process information to discuss Einstein and Plancks differing views about


whether science research is removed from social and political forces

Act. 9.4

identify the relationships between


179
photon energy, frequency, speed of light
and wavelength:
E = hf
and
c = f

xv

Stage 6
Physics syllabus grid
3. Limitations of past technologies and increased research into the structure of the atom resulted
in the invention of transistors
Students learn to:

Page

Students:

Page

identify that some electrons in solids


are shared between atoms and move
freely

189

perform an investigation to model the behaviour of semiconductors, including


the creation of a hole or positive charge on the atom that has lost the electron
and the movement of electrons and holes in opposite directions when an
electric field is applied across the semiconductor

Act.
10.1

describe the difference between


conductors, insulators and
semiconductors in terms of band
structures and relative electrical
resistance

189

gather, process and present secondary information to discuss how shortcomings


in available communication technology lead to an increased knowledge of the
properties of materials with particular reference to the invention of the transistor

Act.
10.2

identify absences of electrons in a


191
nearly full band as holes, and recognise
that both electrons and holes help to
carry current

identify data sources, gather, process, analyse information and use available
evidence to assess the impact of the invention of transistors on society with
particular reference to their use in microchips and microprocessors

Act.
10.2

compare qualitatively the relative


number of free electrons that can drift
from atom to atom in conductors,
semiconductors and insulators

190

identify data sources, gather, process and present information to summarise the
effect of light on semiconductors in solar cells

Act.
10.3

identify that the use of germanium in


early transistors is related to lack of
ability to produce other materials of
suitable purity

199

describe how doping a semiconductor


can change its electrical properties

193

identify differences in p and n-type


semiconductors in terms of the relative
number of negative charge carriers and
positive holes

193

describe differences between solid


state and thermionic devices and
discuss why solid state devices
replaced thermionic devices

199

4. Investigations into the electrical properties of particular metals at different temperatures led to the
identification of superconductivity and the exploration of possible applications
Students learn to:

Page

Students:

Page

outline the methods used by the Braggs 208


to determine crystal structure

process information to identify some of the metals, metal alloys and compounds 211
that have been identified as exhibiting the property of superconductivity and their
critical temperatures

identify that metals possess a crystal


lattice structure

209

perform an investigation to demonstrate magnetic levitation

Act.
11.1

describe conduction in metals as a free


movement of electrons unimpeded by
the lattice

209

analyse information to explain why a magnet is able to hover above a


superconducting material that has reached the temperature at which it is
superconducting

Act.
11.1

209
identify that resistance in metals is
increased by the presence of impurities
and scattering of electrons by lattice
vibrations

gather and process information to describe how superconductors and the effects
of magnetic fields have been applied to develop a maglev train

Act.
11.1

describe the occurrence in


215
superconductors below their critical
temperature of a population of electron
pairs unaffected by electrical resistance

process information to discuss possible applications of superconductivity and the 219


effects of those applications on computers, generators and motors and
Act.
transmission of electricity through power grids
11.1

discuss the BCS theory

215

discuss the advantages of using


superconductors and identify
limitations to their use

217

xvi

Stage 6
Physics syllabus grid

Module 4 From Quanta to Quarks


1. Problems with the Rutherford model of the atom led to the search for a model that would better explain
the observed phenomena
Students learn to:

Page

Students:

Page

discuss the structure of the Rutherford


model of the atom, the existence of the
nucleus and electron orbits

230,
244

perform a first-hand investigation to observe the visible components of the


hydrogen spectrum

Act.
12.1

analyse the significance of the


hydrogen spectrum in the development
of Bohrs model of the atom

236

process and present diagrammatic information to illustrate Bohrs explanation of


the Balmer series

236
Act.
12.1

define Bohrs postulates

236

solve problems and analyse information using:


1
1
1
= R 2 2

ni
nf

233,
245
Act.
12.1

discuss Plancks contribution to the


concept of quantised energy

231

analyse secondary information to identify the difficulties with the RutherfordBohr model, including its inability to completely explain:
the spectra of larger atoms
the relative intensity of spectral lines
the existence of hyperfine spectral lines
the Zeeman effect

Act.
12.2

describe how Bohrs postulates led to


the development of a mathematical
model to account for the existence of
the hydrogen spectrum:
1
1
1
= R 2 2

n
n
f
i

237,
244

discuss the limitations of the Bohr


model of the hydrogen atom

239

2. The limitations of classical physics gave birth to quantum physics


Students learn to:

Page

Students:

Page

describe the impact of de Broglies


proposal that any kind of particle has
both wave and particle properties

250,
259

solve problems and analyse information using:


h
=
mv

249,
258

define diffraction and identify that


interference occurs between waves that
have been diffracted

250,
257

gather, process, analyse and present information and use available evidence to
assess the contributions made by Heisenberg and Pauli to the development of
atomic theory

255
Act.
13.1

describe the confirmation of de Broglies 251,


proposal by Davisson and Germer
257
explain the stability of the electron
orbits in the Bohr atom using
de Broglies hypothesis

253,
257

xvii

Stage 6
Physics syllabus grid
3. The work of Chadwick and Fermi in producing artificial transmutations led to practical applications
of nuclear physics
Students learn to:

Students:

Page

define the components of the nucleus


261,
(protons and neutrons) as nucleons and 278
contrast their properties

Page

perform a first-hand investigation or gather secondary information to observe


radiation emitted from a nucleus using Wilson Cloud Chamber or similar
detection device

Act.
14.1

discuss the importance of conservation


laws to Chadwicks discovery of the
neutron

261,
275

solve problems and analyse information to calculate the mass defect and energy
released in natural transmutation and fission reactions

267,
277

define the term transmutation

263

describe nuclear transmutations due to


natural radioactivity

263

describe Fermis initial experimental


observation of nuclear fission

269

discuss Paulis suggestion of the


existence of neutrino and relate it to
the need to account for the energy
distribution of electrons emitted in
-decay

266,
276

evaluate the relative contributions of


electrostatic and gravitational forces
between nucleons

261

account for the need for the strong


nuclear force and describe its
properties

262

explain the concept of a mass defect


using Einsteins equivalence between
mass and energy

267

describe Fermis demonstration of


a controlled nuclear chain reaction
in 1942

270,
275

compare requirements for controlled


and uncontrolled nuclear chain
reactions

271,
275

4. An understanding of the nucleus has led to large science projects and many applications
Students learn to:

Page

Students:

Page

explain the basic principles of a fission


reactor

280,
298

gather, process and analyse information to assess the significance of the


Manhattan Project to society

280
Act.
15.1

describe some medical and industrial


applications of radioisotopes

283,
298

identify data sources, and gather, process, and analyse information to describe
the use of:
a named isotope in medicine
a named isotope in agriculture
a named isotope in engineering

284,
Act.
15.2

describe how neutron scattering is used 272,


as a probe by referring to the properties 298
of neutrons
identify ways by which physicists
286,
continue to develop their understanding 299
of matter, using accelerators as a probe
to investigate the structure of matter
discuss the key features and
components of the standard model of
matter, including quarks and leptons

xviii

292,
298

Stage 6
Physics syllabus grid

Module 5 Medical Physics


1. The properties of ultrasound waves can be used as diagnostic tools
Students learn to:

Page

Students:

Page

identify the differences between


ultrasound and sound in normal
hearing range

305

solve problems and analyse information to calculate the acoustic impedance of


a range of materials, including bone, muscle, soft tissue, fat, blood and air and
explain the types of tissues that ultrasound can be used to examine

312

describe the piezoelectric effect and


308
the effect of using an alternating
potential difference with a piezoelectric
crystal

gather secondary information to observe at least two ultrasound images of


body organs

Act.
16.1

define acoustic impedance:


Z =
and identify that different materials
have different acoustic impedances

310,
311

identify data sources and gather information to observe the flow of blood through
the heart from a Doppler ultrasound video image

Act.
16.2

describe how the principles of acoustic


impedance and reflection and
refraction are applied to ultrasound

311

identify data sources, gather, process and analyse information to describe how
ultrasound is used to measure bone density

315
Act.
16.3

define the ratio of reflected to initial


intensity as:

310

solve problems and analyse information using:


Z =
and

310,
311

I r Z2 Z 1
=
Io Z + Z 2
2
1

I r Z2 Z 1
=
Io Z + Z 2
2
1

identify that the greater the difference


in acoustic impedance between two
materials, the greater is the reflected
proportion of the incident pulse

310

describe situations in which A scans, B


scans and sector scans would be used
and the reasons for the use of each

312

describe the Doppler effect in sound


waves and how it is used in ultrasonics
to obtain flow characteristics of blood
moving through the heart

315

outline some cardiac problems that can 316


be detected through the use of the
Doppler effect

2. The physical properties of electromagnetic radiation can be used as diagnostic tools


Students learn to:

Page

Students:

Page

describe how X-rays are currently


produced

321

gather information to observe at least one image of a fracture on an X-ray film


and X-ray images of other body parts

Act.
17.1

compare the differences between soft


and hard X-rays

322

gather secondary information to observe a CAT scan image and compare the
information provided by CAT scans to that provided by an X-ray image for the
same body part

Act.
17.1

explain how a computed axial


tomography (CAT) scan is produced

326

perform a first-hand investigation to demonstrate the transfer of light by


optical fibres

Act.
18.1

describe circumstances where a CAT


scan would be a superior diagnostic
tool compared to either X-rays or
ultrasound

329

gather secondary information to observe internal organs from images produced


by an endoscope

Act.
18.1

explain how an endoscope works in


relation to total internal reflection

334

discuss differences between the role of


coherent and incoherent bundles of
fibres in an endoscope

336

explain how an endoscope is used in:


observing internal organs
obtaining tissue samples of internal
organs for further testing

337

xix

Stage 6
Physics syllabus grid
3. Radioactivity can be used as a diagnostic tool
Students learn to:

Page

Students:

Page

outline properties of radioactive


isotopes and their half-lives that are
used to obtain scans of organs

340,
343,
344

perform an investigation to compare an image of bone scan with an X-ray image

Act.
19.1

describe how radioactive isotopes may


be metabolised by the body to bind or
accumulate in the target organ

344

gather and process secondary information to compare a scanned image of at least Act.
one healthy body part or organ with a scanned image of its diseased counterpart 19.2

identify that during decay of specific


radioactive nuclei positrons are
given off

342

discuss the interaction of electrons and 342


positrons resulting in the production of
gamma rays
describe how the positron emission
349
tomography (PET) technique is used for
diagnosis

4. The magnetic field produced by nuclear particles can be used as a diagnostic tool
Students learn to:

Students:

Page

identify that the nuclei of certain atoms 355


and molecules behave as small
magnets

Page

perform an investigation to observe images from magnetic resonance image


(MRI) scans, including a comparison of healthy and damaged tissue

Act.
20.1

identify that protons and neutrons in


354
the nucleus have properties of spin and
describe how net spin is obtained

identify data sources, gather, process and present information using available
evidence to explain why MRI scans can be used to:
detect cancerous tissues
identify areas of high blood flow
distinguish between grey and white matter in the brain

Act.
20.1

explain that the behaviour of nuclei


with a net spin, particularly hydrogen,
is related to the magnetic field they
produce

355

gather and process secondary information to identify the function of the


Act.
electromagnet, radio frequency oscillator, radio receiver and computer in the MRI 20.1
equipment

describe the changes that occur in the


orientation of the magnetic axis of
nuclei before and after the application
of a strong magnetic field

355

identify data sources, gather and process information to compare the advantages
and disadvantages of X-rays, CAT scans, PET scans and MRI scans

Act.
20.2

define precessing and relate the


frequency of the precessing to the
composition of the nuclei and the
strength of the applied external
magnetic field

356

gather, analyse information and use available evidence to assess the impact of
medical applications of physics on society

Act.
20.3

discuss the effect of subjecting


precessing nuclei to pulses of radio
waves

357

explain that the amplitude of the signal 359


given out when precessing nuclei relax
is related to the number of nuclei
present
explain that large differences would
360
occur in the relaxation time between
tissue containing hydrogen bound water
molecules and tissues containing other
molecules

xx

Stage 6
Physics syllabus grid

Module 6 Astrophysics
1. Our understanding of celestial objects depends upon observations made from Earth or from space
near the Earth
Students learn to:

Page

discuss Galileos use of the telescope to 371


identify features of the Moon
Act.
21.1
discuss why some wavebands can be
more easily detected from space

373

define the terms resolution and


sensitivity of telescopes

375

discuss the problems associated with


ground-based astronomy in terms of
resolution and absorption of radiation
and atmospheric distortion

373,
378

Students:

Page

identify data sources, plan, choose equipment or resources for, and perform an
investigation to demonstrate why it is desirable for telescopes to have a large
diameter objective lens or mirror in terms of both sensitivity and resolution

377
Act.
21.2

outline methods by which the resolution 378,


and/or sensitivity of ground-based
380
systems can be improved, including:
adaptive optics
interferometry
active optics

2. Careful measurement of a celestial objects position in the sky (astrometry) may be used to determine
its distance
Students learn to:

Page

Students:

Page

define the terms parallax, parsec,


light-year

388

solve problems and analyse information to calculate the distance to a star given
its trigonometric parallax using:
1
d =
p

Act.
22.1

explain how trigonometric parallax can


be used to determine the distance to
stars

388

gather and process information to determine the relative limits to trigonometric


parallax distance determinations using recent ground-based and space-based
telescopes

Act.
22.2

discuss the limitations of trigonometric


parallax measurements

389

3. Spectroscopy is a vital tool for astronomers and provides a wealth of information


Students learn to:

Page

Students:

Page

account for the production of emission


and absorption spectra and compare
these with a continuous black body
spectrum

390

perform a first-hand investigation to examine a variety of spectra produced by


discharge tubes, reflected sunlight, or incandescent filaments

Act.
22.3

describe the technology needed to


measure astronomical spectra

390

analyse information to predict the surface temperature of a star from its intensity/ Act.
wavelength graph
22.4

identify the general types of spectra


produced by stars, emission nebulae,
galaxies and quasars

393

describe the key features of stellar


spectra and describe how these are
used to classify stars

395

describe how spectra can provide


information on surface temperature,
rotational and translational velocity,
density and chemical composition of
stars

393

xxi

Stage 6
Physics syllabus grid
4. Photometric measurements can be used for determining distance and comparing objects
Students learn to:

Page

Students:

Page

define absolute and apparent


magnitude

398

solve problems and analyse information using:

400

M = m 5 log
and
IA
= 100 (m
IB

d
10

mA)/5

to calculate the absolute or apparent magnitude of stars using data and


a reference star
explain how the concept of magnitude
can be used to determine the distance
to a celestial object

399

perform an investigation to demonstrate the use of filters for photometric


measurements

Act.
22.5

outline spectroscopic parallax

401

identify data sources, gather, process and present information to assess the
impact of improvements in measurement technologies on our understanding of
celestial objects

Act.
22.6

explain how two-colour values (i.e.


colour index, BV) are obtained and
why they are useful

401

describe the advantages of


photoelectric technologies over
photographic methods for photometry

397

5. The study of binary and variable stars reveals vital information about stars
Students learn to:

Page

Students:

Page

describe binary stars in terms of the


means of their detection: visual,
eclipsing, spectroscopic and
astrometric

411

perform an investigation to model the light curves of eclipsing binaries using


computer simulation

Act.
23.1

explain the importance of binary stars


in determining stellar masses

408

solve problems and analyse information by applying:


4 2r 3
m 1+ m 2 =
GT 2

420

classify variable stars as either intrinsic 413


or extrinsic and periodic or non-periodic
explain the importance of the period
luminosity relationship for determining
the distance of cepheids

xxii

416

Stage 6
Physics syllabus grid
6. Stars evolve and eventually die
Students learn to:

Page

Students:

Page

describe the processes involved in


stellar formation

423

present information by plotting HertzsprungRussell diagrams for:


nearby or brightest stars
stars in a young open cluster
stars in a globular cluster

Act.
24.1

outline the key stages in a stars life


in terms of the physical processes
involved

428

analyse information from an HR diagram and use available evidence to determine 437
the characteristics of a star and its evolutionary stage

describe the types of nuclear reactions


involved in Main-Sequence and postMain Sequence stars

425,
430

present information by plotting on a HR diagram the pathways of stars of 1, 5


and 10 solar masses during their life cycle

discuss the synthesis of elements in


stars by fusion

425,
430

explain how the age of a globular


cluster can be determined from its
zero-age main sequence plot for a
HR diagram

433

explain the concept of star death in


relation to:
planetary nebula
supernovae
white dwarfs
neutron stars/pulsars
black holes

429,
431

437

xxiii

1
Context

Figure 1.0.1 The knowledge of how things


move through space,
influenced by gravity, has
transformed the way we work,
play and think.

Space

Modern physics was born twice. The first time (arguably) was in the 17th century
when Newton used his three laws of motion and his law of universal gravitation to
connect Galileos equations of motion with Keplers laws of planetary motion. Then
early in the 20th century, when many thought physics had almost finished the job of
explaining the universe, it was unexpectedly born again. Einstein, in trying to
understand the nature of light, proposed the special and general theories of
relativity (and simultaneously helped launch quantum mechanics).
Space was the common threadKepler, Galileo, Newton and Einstein were all
trying to understand the motion of objects (or light) through space.
Newtons laws of mechanics and his theory of gravitation led to space
exploration and artificial satellites for communication, navigation and monitoring of
the Earths land, oceans and atmosphere. Einsteins theory of relativity showed that
mass and energy are connected, and that length, mass and even space and time
are rubbery. Relativity has come to underlie most new areas of physics developed
since then, including cosmology, astrophysics, radioactivity, particle physics,
quantum electrodynamics, anything involving very precise measurements of time
and the brain-bending string theory.
So, whenever you use the global positioning system (GPS), consult Google
maps, check the weather report or make an international call on your mobile phone,
remember that the technology involved can be traced directly back to physics that
started 400 years ago.

Figure 1.0.2 The revolution in our

Inquiry activity

understanding of the universe


started with the humble question
of how projectiles move.

Go ballistic!
The path through the air of an object subject only to gravity and air resistance,
is called a ballistic trajectory. If the object is compact and its speed is low, then
air resistance is negligible and its trajectory is a parabola.
Investigate parabolic trajectories using a tennis ball, an A4 piece of paper,
a whiteboard or a blackboard and a digital camera.
1 On a board about 2m wide, draw an accurate grid of horizontal and vertical lines
10cm apart.
2 With a firmly mounted camera, take a movie of a tennis ball thrown slowly in
front of the board. Try different angles and speeds to get eight or more frames
with the ball on screen, and get as much of a clear parabolic shape (including
the point of maximum height) as you can.
3 Using video-editing software, view the best movie, frame by frame, on a
computer. If your software allows it, create a single composite image with all
the balls positions shown on one image, to show the parabolic trajectory.
4 If you cant do that, then for each frame, on the board, and using the grid,
estimate the x- and y-coordinates of the balls centre to the nearest 5cm
or better. Some video software allows you to read the x- and y-coordinates
(in pixels) by clicking on the image.
5 Plot a graph of x versus y to produce a graph of the parabolic trajectory. The graph
might be a bit irregular because of random error in reading the blackboard scale.
6 Video the trajectory of a loosely crumpled-up piece of A4 paper. Now air
resistance is NOT negligible. Does the trajectory still look like an ideal parabola?

Cannonballs,
apples, planets
and gravity
What goes up must come down

projectile, trajectory, parabola,


ballistics, vertical and horizontal
components, Galileantransformation,
range, launch angle, time of flight,
inverse square law, law of universal
gravitation, universal gravitation
constant G, gravitational field g, test
mass, central body, density,
gravimeter, low Earth orbit,
gravitational potential energy, escape
velocity, gravitationallybound

One of the powers of physics is that it enables us to find connections


between seemingly unconnected things and then use those
connections to predict new and unexpected phenomena. What
started as separate questions about the shape of the path of
cannonballs through the air and the speed of the Moons orbit around
the Earth eventually led to the law of gravitation. This explained how
the solar system works, but also led to the development of artificial
satellites and spacecraft for the exploration of the
solar system.

1.1 Projectile motion


Up and down, round and round
Before Galileo Galilei (15641642), it was a common belief that an object such
as a cannonball projected through open space (a projectile) would follow a path
(trajectory) through the air in a nearly straight line until it ran out of impetus
and then drop nearly straight down in agreement with the ideas of Aristotle.
However, through experiments (Figure1.1.1) in which he rolled balls off the
edge of a table at different speeds and then marked the position of collisions with
the ground, Galileo demonstrated that the trajectory of a falling ball is actually
part of a parabola (see Figure 1.1.2). Remember that a parabola is the shape of
the graph of a quadratic equation. The immediate result of Galileos discovery
was that the art of firing cannonballs at your enemies became a science (ballistics).
However, there were also more far-reaching, constructive consequences.

Figure 1.1.1 Galileos laboratory notes on his experiments


showing that projectiles follow parabolic paths
4

space

Vertical displacement

Opponents of Copernicus heliocentric universe claimed that if


the Earth was rotating and orbiting the Sun, then a person jumping
vertically into the air would have the ground move under their feet,
so that they would land very far away from where they started.
Galileo argued that a person jumping from a moving Earth is like
a projectile dropped by a rider on a horse (representing the Earth)
moving with a constant velocity (Figure 1.1.3). From the riders point
of view, the projectile would appear to drop vertically, straight to the
ground, accelerating downwards the whole time. A bystander who is
stationary relative to the ground would see the rider, horse and
projectile whoosh past and, like any other projectile, the dropped
object would appear to follow a parabolic trajectory.
Figure 1.1.2
Galileo argued that the parabolic motion of the projectile was
made up of two separable parts: its accelerating vertical motion as
seen by the rider, and its constant horizontal velocity (which is the
same as that of the horse). Recall from your Preliminary physics
course that these two contributions to velocity are called vertical and
horizontal components (see in2 Physics @ Preliminary section 2.2, p 26).
Galileo then argued that the Earth doesnt zoom away under your feet
because at the moment you jump upwards you already have the same horizontal
component of velocity as the Earths surface. Relative to the Earths surface,
your horizontal velocity is zero and so you land on the same spot.
In connecting the two problems of projectile motion and a moving Earth,
Galileo developed two important new concepts. The first is the idea that
the parabolic trajectory of a projectile can be divided into vertical and
horizontal components. The second is the idea of measuring motion relative
to another moving observer (or frame of reference). The formula
vB (relativetoA)=vBvA (see in2 Physics @ Preliminary, p8) is used to transform
velocities relative to different frames of reference. This formula is sometimes
called the Galilean transformation.

Components of a trajectory
The ideal parabolic trajectory is an approximation that works under two
conditions:
1 Air resistance is negligible (gravity is the only external force).
2 The height and range (horizontal displacement) of the motion are both
small enough that we can ignore the curvature of the Earth.
a

Horizontal displacement

This graph of a parabolic trajectory shows the


vertical and horizontal components of
displacement separately. The projectile
positions are plotted at equal time intervals.

Describe Galileos analysis


of projectile motion.

Describe the trajectory of an


object undergoing projectile
motion within the Earths
gravitational field in terms
of horizontal and vertical
components.

Figure 1.1.3 Trajectory of the riders projectile as seen by (a) the rider and (b) an observer on the ground
5

Cannonballs,
apples, planets
and gravity
The first condition is true for compact and low-speed projectiles. The second
is true in almost all human-scale situations, typically at or near the Earths surface.
Lets analyse an example of ideal projectile motion. Recall that the acceleration
due to gravity is g=9.8ms2 (see in2 Physics @ Preliminary section 1.3). Here we
are going to write it as a vector g. Clearly its direction is downwards.
Consider the trajectory of a ball. We start by separating the horizontal and
While the ball is in the air, the only
vertical components of its motion.
external force on it is gravity acting downwards, so there is a constant vertical
acceleration ay=g, illustrated by the changing vertical spacing of projectile
positions plotted at equal time intervals in Figure 1.1.2.
The net horizontal force is zero, so, consistent with Newtons first law,
horizontal velocity is constant (ax=0), which is clear from the equal horizontal
spacing of the projectile positions plotted at equal time intervals in Figure 1.1.2.
We can recycle the kinematics (SUVAT) equations from the Preliminary
course. (See in2 Physics @ Preliminary section 1.3.)
(4)
s = vt
(1)
s = ut +1 at2
2
u+v

s=
t
(2)
(5)
v2 = u2 + 2as



2
v = u + at
(3)

PRACTICAL
EXPERIENCES
Activity 1.1

Activity Manual, Page


1

Table 1.1.1

Equations of projectile motion

Horizontal components

Vertical components

ux = u cos i

uy = usini

vx = ux

vy = uy + gt

x = uxt

1 2
gt
y = uyt +

vx2 = ux2

vy2 = uy2 + 2gy

75

90

60
45

Here we need to apply them separately to the vertical (y) and horizontal (x)
components of motion. Instead of displacement s, well use x=xfxi for
horizontal displacement and y=yfyi for vertical displacement. Well put
subscripts on the initial and final vertical velocities (uy and vy for example). We
only need to use SUVAT equations 3, 4 and 5. i is the launch angle (between
Remember to adjust the sign
the initial velocity u and the horizontal axis).
of g to be consistent with your sign convention. In problems involving gravity, up
is normally taken as positive, making the vector g negative (i.e. g = 9.8 m s2).
In the syllabus, vx2 = ux2 is included for completeness; but is unnecessary,
as it can be derived from vx=ux.
Some properties of ideal parabolic trajectories are:
At the maximum height of the parabola, vertical velocity vy=0.
The trajectory is horizontally symmetrical about the maximum height position.
The projectile takes the same time to rise to the maximum height as it takes to
fall back down to its original height.
For horizontal ground, initial speed = final speed.
Maximum possible height occurs for a 90 launch angle. The maximum
possible range (for horizontal ground) occurs for a 45 launch angle
(Figure 1.1.4).
Independent of their initial velocity, all objects projected horizontally from the
same height have the same time of flight as one dropped from rest
from the same height, because they all have a zero initial vertical velocity
(Figure 1.1.5).

30
15

Figure 1.1.4 For a fixed initial speed, maximum range


occurs for a 45 angle of launch and maximum
height occurs for a 90 angle of launch.
6

space

100mm

Ballistics is a drag

ir resistance or drag introduces deceleration in both the


vertical and horizontal directions, distorting the ballistic
trajectory from an ideal parabola. As a projectile becomes less
compact, air resistance increases relative to weight. The range
decreases, the trajectory becomes less symmetrical, and the
final angle becomes steeper. The launch angle for maximum
range decreases. In extreme cases (for example, a loosely
crushed piece of paper), the trajectory seems to approach
Aristotles prediction: it moves briefly in a nearly straight line
and then drops nearly vertically.
no air resistance

Figure 1.1.5 Multiflash photo of two falling


objects. All horizontally projected
objects have the same time of
flight as an object dropped from
rest from the same height.

Target practice
You now have all the equations you need to do
some damage, so lets launch some projectiles.
Safety warning! The following worked example
may seem dangerously long because it illustrates
several alternative methods of solving projectile
problems rolled into one.

increasing air resistance

Figure 1.1.6 The effect of increasing air resistance

Worked example
Question
You throw a ball into the air (Figure 1.1.7). You release the ball 1.50m above the ground,
with a speed of 15.0ms1, 30.0 above horizontal. The ball eventually hits the ground.
Answer the following questions, assuming air resistance is negligible.
a For how long is the ball in the air before it hits the ground (time of flight)?
b What is the balls maximum height?
c What is the balls horizontal range?
d With what velocity does the ball hit the ground?

Solve problems and analyse


information to calculate the
actual velocity of a projectile
from its horizontal and vertical
components using:
v x2 = u x2
v = u + at
vy2 = uy2 + 2ayy
x = uxt
1
y = uyt + ayt2
2

1.50 m

Figure 1.1.7 Throwing a ball into the air


7

Cannonballs,
apples, planets
and gravity

Solution
Always draw a diagram! Divide the motion into vertical (y) and horizontal (x) components.
Choose the origin to be the point of release, so xi=yi=0.
This is not always the most convenient choice of origin.

Use the sign convention +& + .


Components of initial velocity u (Figure 1.1.8):
ux=+ucosi = +15.0cos30.0 = +13.0ms1
uy=+usini = +15.0sin30.0 = +7.50ms1
The only external force is gravity so vertical acceleration is g=9.80ms2. There is no
horizontal force, therefore ax= 0ms2 (constant horizontal velocity).
fin

al

yu

ial

ini

i = 30.0

it
loc

ve

uy

ve

vy

loc

ity

ux

vx

Figure 1.1.8 Components of initial and final velocities


a The ball hits the ground when vertical displacement y=1.50m.

Find final vertical velocity: vy2 = uy2 + 2gy = 7.502 + 29.801.50 = 85.65

vy =




85.65 = 9.255ms1 (must be downwards), so vy = 9.255ms1

vy = uy + gt = 9.255 = +7.50 + (9.80)t


9. 255 7. 50
Rearrange, solve:
t=
= 1.71s
9 .80
The ball hits the ground 1.71s after being thrown.
Find t:

1
Alternative method using the quadratic formula y = uyt + gt2 = 1.50m
2
1
2
Substitute, rearrange: 1.50 + 7.50t + 9.80t = 0
2
7.5 7.502 + 4 4.90 1.50
= 0.179s or +1.71s
2 4.90

Quadratic, solve for t: t =

Two solutions: Reject the physically irrelevant negative solution, so t=1.71s.

b At maximum height, vertical velocity vy = 0, so use vy2 = uy2 + 2gy.


0 = uy2 + 2gymax = 7.502 + 2(9.80)ymax
7.502
Rearrange, solve:
ymax =
= +2.87m above the point of release,
2 9.80
so height above ground = 2.87m + 1.50m = 4.37m above the ground.

Alternative method

1
Use vy = uy + gt to find the time t when vy = 0, then use y = uyt + gt2 to find
2
vertical displacement.

c From part a, we know the time of flight t=1.71s.



8

Horizontal displacement in this time is:


x = uxt = +13.0ms11.71s = +22.2m = 22.2m (to the right)

space
d x-component of final velocity: vx = +13.0ms1

y-component of final velocity: vy = 9.255ms1 (down) (from part a)

To find magnitude, use Pythagoras theorem (see Figure 1.1.8):


v = v x2 +v y2 = 132 + 9.2552 = 15.96 16.0ms1
v y 9.25
Direction: tanf =
, so f = 35.4 down from horizontal
=
v x 13.0
Alternative magnitude calculation

Negligible air resistance, mechanical energy = kinetic energy + gravitational


potential energy and is conserved (see in2 Physics @ Preliminary section 4.2). Near
the Earths surface, gravitational potential energy U=mgh. Using the ground as h = 0:
Ki + Ui = Kf + Uf
1 2
1
Cancel m:
mv + mghi = mvf2 + mghf
2 i
2
1
1
Substitute: 15.02 + 9.801.50 = vf2 + 0
2
2

Rearrange, solve: v f = 15.02 + 2 9.80 1.50 = 15.94 15.9ms1

This is the same as for the previous method within the three-figure precision of the
calculation, but doesnt tell us the direction.

In the previous example, time of flight was determined by the vertical


However, if the
componentthe flight ended when the ball hit the ground.
projectile hits a vertical barrier such as a wall, then the time of flight is determined
by the horizontal component.

Worked example
Question
Suppose you kick a ball at 22.0ms1, 20.0 above the horizontal, towards a wall 21.0m
away (Figure 1.1.9). Ignore air resistance and the balls radius.
a What is the balls time of flight (before hitting the wall)?
b At what height does the ball hit the wall?
c Is that the greatest height reached by the ball?

Solution

Solve problems and analyse


information to calculate the
actual velocity of a projectile
from its horizontal and vertical
components using:
v x2 = u x2
v = u + at
vy2 = uy2 + 2ayy
x = uxt
1
y = uyt + ayt 2
2

Figure 1.1.9 The ball hits the wall.

Choose the origin to be the initial position, so xi=yi=0. Use the sign convention +
and +.
ux= 22.0cos20.0 (right) = +20.7ms1
uy= 22.0sin20.0 (up) = +7.52ms1
9

Cannonballs,
apples, planets
and gravity
a The ball hits the wall when the horizontal displacement x=+21.0m.

Substitute: x = uxt = +21.0m = +20.7ms1t


+21.0 m

= 1.014s 1.01s
+20.7 m s1
1
b The ball hits the wall at a height (vertical displacement) of y = uyt + gt2.
2

Rearrange, solve: t =

Substitute, solve: y = +7.521.014 + 0.59.801.0142 = +2.587

The ball hits the wall 2.59m above ground.

c Check if the ball reaches maximum height of the parabola before hitting the wall.

Time of flight = 1.01s. vy = 0 at maximum height of parabola.

Find the time taken to reach maximum height.

Substitute: vy = 0 = uy + gt = +7.52 + 9.80t

Rearrange, solve: t =

7.52
= 0.767s which is less than time of flight
9.80
The ball would reach the maximum height of the parabola before hitting the wall,
therefore the final height is NOT the maximum height for the trajectory.

Checkpoint 1.1
1
2
3
4
5
6

Determine the horizontal acceleration of a projectile in flight. Determine its vertical acceleration. (Assume
negligible air resistance.)
What angle of launch gives maximum horizontal range? What angle of launch gives the maximum possible height?
(Assume negligible air resistance.)
What is another name for air resistance?
If you throw a ball horizontally from the roof, and drop another at the same time, which one will hit the ground first?
Describe the two conditions that must apply so that a trajectory is a parabola.
List the 8 equations used in calculations of projectile motion. Explain why at least one of them is unnecessary.

1.2 Gravity
In Ptolemys universe, the Sun, Moon and planets each had a separate clockworklike mechanism to keep it in motion. Copernicus and Kepler greatly improved
the picture, but Isaac Newton finally showed there was a single mechanism for
them allthe force of gravity.
The calculations of parabolic trajectories in section 1.1 work well close to the
Earths surface where g is constant. However, if were going to venture out into
space, we cant use these simple equations. We need to look at the force of gravity
on a larger scale.

Newtons law of universal gravitation


Newton assumed several properties of gravity (see in2 Physics @ Preliminary
section 13.5):
All massive objects (that is, objects with mass) attract each other. The larger
the masses, the larger the force.
10

space
Like light intensity, the magnitude of the force decreases with distance
according to the inverse square law (see in2 Physics @ Preliminary sections
6.1 and 15.1). However, astronomer Ismael Boulliau had suggested this
before him.
The law of gravitation is universalit applies throughout the universe and is
responsible for the orbits of all the planets and moons.
All this is expressed mathematically as the law of universal gravitation:
FG = G

m1m2
d2

where FG is the magnitude of the force of gravitational attraction between


Define Newtons Law of
Universal Gravitation:
two masses m1 and m2 and d is the distance between their centres of mass (see
in2 Physics @ Preliminary section 3.6). The universal gravitational constant G
mm
F =G 1 2

(big G) is 6.671011 Nm2kg2 in SI units. It should not be confused with
d2
2
little g, 9.8ms .
More properties of gravitation:
The direction of the force acts along the line joining the
centres of the two masses and is always attractive.
The formula is strictly correct for point masses and spheres,
but works well for non-spheres.
The formula must be modified if one mass penetrates the
surface of the othergravity would not approach infinity if
you were to burrow towards the centre of the Earth.
You can see the feeble force of gravity acting
The resultant force on a mass due to the presence of other
between objects in your garage. John Walkers
masses is the vector sum of the individual forces on the first
Fourmilab website describes step by step how
mass due to each of the other individual masses.
you can perform a crude version of the

Try this!

Slightly attractive

Worked example
Question
Calculate the gravitational force between the Earth and the Moon.
Data: Earths mass mE = 5.971024kg

Moons mass mM = 7.351022kg

Average EarthMoon distance dEM = 3.84108m

Universal gravitational constant G = 6.671011 N m2 kg2

Solution

FG = G

Cavendish experiment in your own garage (see


Physics Focus How to weigh the Earth at the
end of this chapter), using commonly found
household items and a video camera.
If youre feeling too lazy to do it yourself,
you can just download sped-up videos of the
experiment in progress.

mEmM
dEM 2

6.67 1011 5.97 1024 7.35 1022


(3.84 108 )2

= 1.981020 N

Figure 1.2.1 Cavendish apparatus at home

11

Cannonballs,
apples, planets
and gravity
Now lets try an example with more than two masses.

Worked example
Moon

Question
Earth

spacecraft

Figure 1.2.2 A spacecraft in the


EarthMoon system
FSE

A 1000kg spacecraft is in the vicinity of the EarthMoon system. The spacecraft is at the
origin, the Moon is on the positive yaxis and the Earth is on the positive xaxis (Figure
1.2.2). Given that the Earthspacecraft and Moonspacecraft distances are 3.82108m
and 3.91107m respectively, calculate the resultant gravitational force on the
spacecraft.
Data: Earths mass mE = 5.971024kg

Moons mass mM = 7.351022kg

Universal gravitational constant G = 6.671011 N m2 kg2

Solution
Force due to Moon: FSM = G

FSM

Fres

spacecraft

Figure 1.2.3 Gravitational force vector


diagram. Note; This does not
resemble the position vector
diagram in Figure 1.2.2.

d SM 2

6.67 1011 1000 7.35 1022

Force due to Earth: FSE = G

mSmM

mSmE
d SE 2

(3.91 107 )2

6.67 1011 1000 5.97 1024


(3.82 108 )2

Dont underestimate the power of boredom


Boredom part 1
ored? Dont just write graffititry revolutionising physics! In 1665, an
outbreak of bubonic plague around London closed Cambridge University,
so Isaac Newton (aged 23) escaped for 2 years to his mothers farm. He was
not a very good farmer, so he fended off his city-boy boredom by inventing
calculus and using prisms to show that white light is actually a mixture of
colours (the spectrum). To top this off, when he saw an apple fall off his
mothers tree, he wondered if the force accelerating the apple downwards was
also responsible for keeping the Moon orbiting the Earth.
So he began formulating his theory of gravitation. His mathematics professor
was so impressed that a couple of years after Newton returned to Cambridge,
he resigned and handed his professorship to Newton.
After this initial investigation, it took Newton another 20 years to fully
develop and finally publish his law of universal gravitation.
12

= 2.729N (+x direction)

Magnitude of resultant: Fres = 3.2072 + 2.7292 = 4.21 N


3.207
, so = +49.6 from the xaxis
Direction: tan =
2.729

PHYSICS FEATURE

= 3.207N (+y direction)

1. The history of
physics

3. Applications and
uses of physics

Figure 1.2.4 Graffiti carved on a stone at


the Kings School in Grantham,
England, by Isaac Newton,
then about 10 years old

space

Weight and gravitational fields


As far as we know, the universal gravitational constant G is a fundamental
constant, unchanging with position or time. But the acceleration due to gravity g
is different on other astronomical bodies, at different heights and even at
different positions on the Earths surface.
Recall that weight w=mg is defined as the force on an object due to gravity
(see in2 Physics @ Preliminary section 3.2); in other words, FG = w= mg. Little g,
the acceleration due to gravity, can also be thought of as the strength of the
gravitational field. However, the word weight is usually reserved for the case
in which the gravitational field is due to a body of astronomical size, such as
a planet.
Any massive object can be described as being surrounded by a gravitational
field, a region within which other objects experience an attractive force. Just as
for electrical and magnetic fields (see in2 Physics @ Preliminary sections 10.6,
12.3 and 12.4), we can draw diagrams of gravitational field lines (Figure 1.2.6).
The arrows on the field lines around a mass, point in the direction of the force
acting on another (normally much smaller) test mass. Gravitational field
is a vector (g). The density of the field lines at any particular point in space
represents g, the magnitude of the field at that point, and the direction of the
field lines represents the direction of this vector. Field lines run in radial
directions from point masses or spherical masses.
Using a small test mass m, lets derive g, the magnitude of the gravitational
field due to a planet of mass M. The weight w of the test mass is defined as the
force on m due to the planets gravity; that is:
mM
w=mg=FG=G
d2

PRACTICAL
EXPERIENCES
Activity 1.2

Activity Manual, Page


5

Describe a gravitational field


in the region surrounding a
massive object in terms of its
effects on other masses in it.
Define weight as the force
on an object due to a
gravitational field.

Boredom part 2

t is said that, at age 17, Galileo was attending church and, bored,
was watching a lantern swing from the ceiling. Using his pulse as a
stopwatch, he observed that the oscillation period of a pendulum barely
changed as its amplitude gradually decreased. Back at home he started
experiments confirming that the oscillation period depends on pendulum
length L, but not at all on mass and only slightly on amplitude. He
proposed (correctly) that pendulums could be used to create the first
accurate mechanical clocks.
We now know that, consistent with Galileos observations, for a simple
mass-on-string pendulum the formula for oscillation period T is:
T = 2

L
g

The formula is an approximation, but if the maximum swing angle is


less than 15 from vertical, the formula is correct within 0.5%. With this
formula and a pendulum, you can measure the value of littleg, which
varies slightly between locations around the world.

Figure 1.2.5 Young Galileo watches a swinging


lantern in Pisa cathedral.
13

Cannonballs,
apples, planets
and gravity
Divide both sides by test mass m:

g=

FG
M
=G 2
m
d

Newtons equation for gravitational force is symmetricalyou can choose


either mass as the test mass and calculate the field around the other and still get
the same magnitude of force when you multiply them together because of
Newtons third law (see in2 Physics @ Preliminary section 3.5)the two masses
are an actionreaction pair. However, if one of the masses is much larger (such as
a planet), it is more convenient to calculate the field around it and use the
smaller mass as the test mass.
In astronomical situations where one of the bodies (such as a planet or star) is
very much larger, the larger body is sometimes called the central body. Because
of its large mass, the central body experiences negligible gravitational
accelerations compared with a small test mass.
Strictly speaking, the acceleration g is the acceleration of the test mass
towards the common centre of mass of the whole system of two masses.
However, if the central body is much larger than the test mass, we can ignore its
acceleration, so g effectively becomes the acceleration of the test mass towards the
central body.
Gravitational field is a vector, so when calculating the resultant field due to
several bodies, the approach is identical to calculating the resultant gravitational
force due to several bodiescalculate the field due to each individual mass and
then find the vector sum of the fields.

Worked example
Question
Figure 1.2.6 Gravitational field lines around the
Earth (a) on an astronomical scale
and (b) near the surface

Calculate gE the magnitude of the gravitational field at the Earths surface.


Data: Earths mass mE = 5.971024kg

Earths radius rE = 6.37106m

Universal gravitational constant G = 6.671011 N m2 kg2

Solution

gE =

GM E
d2

The test mass is at the Earths surface, d = rE


Substitute:

gE=

6.67 1011 5.97 1024


(6.37 106)2

= 9.81 ms2

This should be a very familiar result.

PRACTICAL
EXPERIENCES
Activity 1.3

Activity Manual, Page


11

14

Variations in gravitational field


Newtons gravitation equation says that the magnitude of a planets gravitational
field depends on the mass of the planet and decreases with distance from the
planets centre. For example, on Earth, the value of g is 0.28% lower at the top
of Mt Everest than at sea level. Also, because the Earth has a slightly larger radius
near the equator than at the poles (the equatorial bulge), g is slightly lower at
the equator. Except at the poles, there is an additional (fictitious) decrease in g

space

nte

ractiv

measurements that gets more severe as one approaches the equator. Because of the
Earths rotation, the (downward) centripetal acceleration (see
in2 Physics @ Preliminary section2.3) of the ground appears to be subtracted from
the true value of g. In fact this centripetal effect is responsible for the formation
of the equatorial bulge, which was predicted by Newton before it was measured.
The Sun and Moon also exert a weak gravitational force on objects at the
Earths surface, so the magnitude and direction of g vary slightly, depending on
the positions of the Sun and Moon. Variation in g caused by the positions of the
Sun and Moon relative to the oceans is responsible for the pattern of tides.
Strictly speaking, Newtons gravitation equation written in the form
above assumes that the planet is a perfectly uniform sphere. Close to the surface
of a planet, local deviations from uniform density can result in small local changes
in the magnitude and direction of g. The magnitude will be slightly larger than
average when measured on the ground above rock (such as iron ore) of high
density (mass per unit volume) and lower above rock containing low-density
minerals (such as salt or oil), an effect exploited by geologists in mineral
exploration. The Earths crust is less dense than the mantle, so variations in
thickness of the crust also affect g. Variation in g is measured using a gravimeter,
the simplest kind being an accurately known mass suspended from a sensitive
spring balance.
Variations in g on larger distance scales around the Earth can be measured
using satellites orbiting in low Earth orbit. Deviations in the orbital speed of
satellites indicate that, in addition to the equatorial bulge, Earth is also slightly
pear-shapedpointier at the North Pole than the South Pole.

M o d u le

Hookes law

saac Newton had enemies, and Robert


Hooke (16351703) was probably his
greatest. They argued bitterly over
(among other things) who first suggested
the inverse square law for gravity. Hooke
was an accomplished experimental
physicist, astronomer, microscopist,
biologist, linguist, architect and
inventor. He is best remembered for the
discovery of (biological) cells and
the invention of the spring balance (see
in2 Physics @ Preliminary section 3.2),
which exploits Hookes law F=kx.
The force F exerted by a spring is
proportional to x, the change in spring
length. The spring constant k is a
measure of the springs stiffness. A
calibrated spring balance can measure
weight, and, if used with an accurately
calibrated mass, it can be used
as a gravimeter to measure g.

haviour of springs

notes on the be
Figure 1.2.7 Hookes

15

Cannonballs,
apples, planets
and gravity

Checkpoint 1.2
1
2
3
4
5
6

Write down Newtons law of universal gravitation.


Define weight.
What part of Newtons formula for gravitational force is responsible for the inverse square law behaviour?
What are two names for the quantity g?
List three factors responsible for (real) variations in g around the Earth.
Outline the differences between G and g.

1.3 Gravitational potential energy


Weve already mentioned gravitational potential energy (GPE) U=mgh
(see in2 Physics @ Preliminary section 4.1) in part d of the worked example
accompanying Figure1.1.7. This formula for GPE is an approximation that only
works close to the Earths surface, where g is very nearly constant. Its good
enough for projectile motion but, as you now know, g decreases with distance, so
we need a more accurate formula to understand energy on an astronomical scale.

Work and GPE


Explain that a change in
gravitational potential energy
is related to work done.

Define gravitational potential


energy as the work done to
move an object from a very
large distance away to a point
in a gravitational field:
mm
EP = G 1 2
r

For clarity well use the symbol EP instead of U to denote gravitational potential
energy calculated using the more accurate formula, even though the two symbols
are really interchangeable. Potential energy is energy stored by doing work
against any force (such as gravity) that depends only on position; therefore,
gravitational potential energy EP is energy stored by doing work against the force
of gravity. It can be shown (using calculus to derive the work done against
gravity by changing the separation of two masses) that:
E P = G

m1m2
r

where m1 and m2 are two masses separated by a displacement (or separation) r


and G is the universal gravitational constant. Note that EP is always negative and
EP
approaches zero as displacement r approaches infinity (Figure 1.3.1).
for a separation r is the work that would need to be done by a force opposed to
gravity in moving the masses together, starting at infinite separation where
EP=0 and bringing them to a separation of r (with no net change in speed).
Equivalently, EP is the work done by gravity while the masses are moved
apart, starting at a separation of r to a position of infinite separation (with no
The gravitational potential energy does not depend on
net change in speed).
the path taken by the masses to get to their final positions; it depends only on
the final separation r.
The formula isnt affected by the choice of which mass to move, although
normally we treat a large mass such as the Sun or a planet as an immoveable
central body and the smaller mass as a moveable test mass. The formula seems to
imply that EP approaches negative infinity as the test mass approaches the centre
of a planet. However, this formula no longer applies in this form once one mass
penetrates the surface of the other.
16

space

+GmtmE
r E2

FG

rE

2rE

3rE

4rE

5rE
separation

GmtmE
rE

EP

Figure 1.3.1 Plots of gravitational force (FG) and gravitational potential energy (EP) versus separation

between a test mass mt and the Earth mE, starting at one Earth radius rE. The vertical FG
and EP axes are not drawn to the same scale.

Worked example
Question
A piece of space junk of mass mJ drops from rest from a position of 30000km from the
Earths centre. Calculate the final speed vf it attains when it reaches a height of 1000km
above the Earths surface. Assume that above 1000km, air resistance is negligible.
Data: Earths mass mE = 5.971024kg

Earths radius rE = 6.37106m

Universal gravitational constant G = 6.671011 N m2 kg2

Solution
Air resistance is negligible, so total mechanical energy (kinetic + potential energy) is
conserved. Assume that because of the enormous mass of the Earth, its change in velocity
is negligible. Use the Earth as the frame of reference. Dont forget to convert to SI units.

Cancel mJ:
Substitute:

K i + EPi = Kf + EP
m
m
mm
1
1
mJvi2 G J E = mJvf2 G J E
ri
rf
2
2
0 6.67 1011

5.97 1024

1 2
5.97 1024
11
=

6.67

10

v
f
30.0 106 2
(6.37 + 1.00) 106

106 106

Rearrange, solve: v f = 2 6.67 1011 5.97 1024

7.37 30.0
= 9030 m s1 = 9.03 km s1
Note that this result doesnt depend on mJ.

17

Cannonballs,
apples, planets
and gravity

Escape velocity: what goes up ?


Outline Newtons concept
of escape velocity.

Isaac Newton showed that what goes up doesnt necessarily come down. Normally,
if one fires a projectile straight up, the object will decelerate until its velocity
changes sign and it falls back down. However, if a projectiles initial velocity is
high enough, the 1/d2 term in the gravity equation will cause the acceleration g to
decrease with height too rapidly to bring the projectile to a stop so it will never
turn backit can escape the planets gravitational field. The minimum velocity
that allows this is called the escape velocity. Strictly speaking, its really a speed,
because the initial direction of the projectile isnt critical.
Newton treated the projectile as a cannonball (with no thrust) so that, other
than the initial impulse from the cannon, the only force acting on it is gravity.
He conceived escape velocity using his force equation, and the escape velocity
formula can be derived from it. However, a more modern derivation using energy
is easier and similar to the previous worked example.
Let m be the mass of a projectile, M the mass of a planet, ve the initial speed
and r the initial position (the planets radius if you are on the surface). Assume air
resistance is negligible, so total mechanical energy (KE + GPE) is conserved (see
in2 Physics @ Preliminary section 4.2).
Ki + EPi = Kf + EPf
The escape velocity represents the minimum limiting case where the projectile
just reaches infinite displacement with zero speed; in other words, Kf=EPf=0.
1
GmM
mve 2
=0+0
2
r

Rearrange, cancel m:

ve =


Explain the concept of escape
velocity in terms of the:
gravitational constant
mass and radius of the planet.

2GM
r

If the initial speed is greater than this, the projectile will maintain a non-zero
speed even as it approaches infinite displacement. Note that the escape velocity
depends only on the planets mass and the projectiles starting position r but not
on the projectiles mass.
You may be puzzled that in the above derivation, the total mechanical energy
(sum of KE and GPE) was exactly zero. This means that the escaping projectile
has just enough (positive) KE to overcome its negative potential energy. When
the mechanical energy is less than zero, there is not enough KE to overcome the
GPE and the two masses are said to be gravitationally bound. When the total
mechanical energy ME>0, the KE can overcome the GPE and the two bodies
are no longer bound together. This concept of binding also applies to the other
three fundamental forces (including electromagnetism, which binds electrons to
the nucleus of an atom).
The escape velocity from the Earths surface is:
2 6.67 10 11 5.97 1024
6.37 10

18

= 11 200 m s 1 = 11.2 km s 1

space
This idealised escape velocity needs to be modified when applied to real
spacecraft. First, the derivation ignores air resistance in the atmosphere
(hundreds of kilometres thick), which would increase the escape velocity.
Second, in a real rocket, engines produce an extra forcethrustthat can
accelerate a craft to a higher altitude where the escape velocity is lower. It also
ignores other sources of gravitational fields such as the Sun, Moon and planets.
The escape velocity for a projectile under the gravitational influence of more
than one body is given by:
ve total = ve12 + ve22 +
where ve total is the escape velocity for the total system and ve1, ve2 are the
escape velocities from the individual bodies within the system, calculated for the
projectile using the same starting position in space.

Ultimate frisbee

as the first artificial object to leave the solar system a giant steel
frisbee? In the 1950s, the US started testing nuclear bombs
underground, to minimise atmospheric nuclear fallout. In 1957, during
Operation Plumbbob in the Pascal-B test, a nuclear bomb was detonated at
the bottom of a 150m shaft sealed with concrete and a 900kg, 10cm thick
steel cap. The steel cap fired upwards at enormous speed and was never
seen again. Before the test, it was estimated that an extreme upper limit for
the speed of the steel cap would be 67 kms1. This is well above the
escape velocity for the whole solar system (43.6 kms1 from Earth), starting
an urban myth that it beat the Voyager probes (launched in 1977) out of the
solar system. A later, more realistic, estimate suggested that, at most, the
cap had a speed of 1.4 kms1, reaching an altitude of less than 95 km.

Checkpoint 1.3
1
2
3
4
5

Define under what circumstances it is suitable to use the simplified formula U = mgh for gravitational potential
energy (GPE).
Write down the more accurate formula for GPE.
What limit does GPE approach as the separation of the two masses approaches infinity?
On what factors does Newtons idealised escape velocity depend?
What other factors affect escape velocity in realistic situations?

19

Cannonballs,
apples, planets
and gravity

PRACTICAL EXPERIENCES
CHAPTER 1

This is a starting point to get you thinking about the mandatory practical
experiences outlined in the syllabus. For detailed instructions and advice, use
in2 Physics @ HSC Activity Manual.

Activity 1.1: Projectiles


Perform a first-hand
investigation, gather
information and analyse data
to calculate initial and final
velocity, maximum height
reached, range and time of
flight of a projectile for a
range of situations by using
simulations, data loggers
and computer analysis.

A ball is rolled down a ramp, whose dimensions will be known to you. Predict where
the ball will land.
Equipment: aluminium track, ball bearing, metre ruler, measuring tape, shoe.
ball bearing

track

ruler

Figure 1.4.1 Equipment set-up


for this activity

Discussion questions
1 List assumptions you have made in order to make an estimate of the range.
2 Assess how reliable is your method.
3 Explain how changing the original angle of the ramp will affect the range
of the ball.

Activity 1.2: Determining the value of acceleration


due to gravity
Perform an investigation and
gather information to determine
a value for acceleration due
to gravity using pendulum
motion or computer-assisted
technology and identify
reason(s) for possible variations
from the value 9.8ms2.

Use the motion of a pendulum to gather


data to determine the acceleration due
to gravity.
Equipment: pendulum (string and mass),
retort stand and clamp, stopwatch,
metre ruler, data logger.

retort stand

string

mass

Figure 1.4.2 Pendulum apparatus set-up

20

space
Discussion questions
1 Explain what you did in order to make the experiment reliable.
2 Galileo originally thought that the period of the pendulum did not depend
at all on the amplitude of the swing. Is this true? Explain how you can take
this into account in your experiment.
3 How does your value compare with the accepted value?
4 Outline another method that would allow you to achieve the same aim.

Activity 1.3: GravityOut of this world


Use the spreadsheet template to gather appropriate information to help you predict
the acceleration due to gravity at the surface of other planets.

Figure 1.4.3 Spreadsheet template


Process the information you have gathered using the spreadsheet template. Complete
the template to calculate the values of acceleration due to gravity on other planets.
Discussion questions
1 Determine which planet has the largest value for acceleration due to gravity
at its surface.
(Note that the gas giants Jupiter, Saturn, Uranus and Neptune dont have
a well-defined boundary between the atmosphere and a solid planet surface.
The visible surface is fluid, i.e. gas and/or liquid.)
2 Identify the factors that affect the acceleration due to gravity.

Gather secondary information to


predict the value of acceleration
due to gravity on other planets.
Present information and use
available evidence to discuss
the factors affecting the strength
of the gravitational force.

Analyse information using the


expression:
F = mg
to determine the weight force
for a body on Earth and for the
same body on other planets.

21

Cannonballs,
apples, planets
and gravity

Chapter summary

If air resistance (drag) is negligible and g is very nearly


constant (for example near a planets surface), then the
trajectory of a projectile is a parabola.
The formula for transforming velocity within one frame
of reference into one relative to another frame of
reference is called the Galilean transformation:
vB(relativetoA)=vBvA
Parabolic projectile motion can divided into vertical and
horizontal components. The vertical component has
a downward acceleration of g and the horizontal
component has a constant velocity.
In parabolic projectile motion, the equations of
motion are:
Horizontal components:
ux = ucosi, vx = ux, x = uxt, vx2 = ux2
Vertical components:
uy = usini, vy = uy + gt, y = uyt + 1 gt2,
2
vy2 = uy2 + 2gy
For horizontal ground, the maximum possible range
occurs for a 45 launch angle. The maximum possible
height occurs for a 90 launch angle.
All objects projected horizontally from a particular
height have the same time of flight as one dropped from
rest from the same height.
If a trajectory ends when the projectile hits the ground,
time of flight is determined by the vertical component.
If the projectile hits a vertical barrier, then time of flight
is determined by the horizontal component.
Newtons law of universal gravitation:
mm
FG = G 1 2 2
d

Gravitational acceleration (g) towards a central body


such as a planet is also called its gravitational field. It
depends on the central body mass M and the distance d
from its centre:
M
g =G 2
d
The force of gravity on an object in that field is called
its weight: w=mg.
Gravitational field g measured near the Earths surface
varies slightly with distance from the Earths centre and
density of the surrounding material. The centripetal
acceleration of the Earths surface also decreases
measured values of g (only an apparent effect).
Gravitational potential energy (GPE) is the work done
by a force opposing gravity in moving masses together
starting at infinite separation and bringing them to
a separation of r (with no net change in speed).
The simple formula for GPE (U = mgh) is an
approximation that only works at or near the surface
of a planet. The more accurate expression is:
mm
E P = G 1 2
r
EP approaches zero as separation of the two masses
approaches infinity.
The minimum initial velocity that a projectile needs to
have in order to escape a planets gravitational field is
called escape velocity:

ve =

2GM
r

Review questions
Physically speaking
Complete each definition by using a keyword taken from the list at the beginning of the chapter.
To approach infinite distance from a massive central body, a projectile must start with _________________.
The path of a projectile is known as a _________________.
The formula for converting velocities between frames of reference is the _________________.
A projectiles maximum horizontal displacement is its _________________.
Universal gravitation and the intensity of light both follow the _________________.
Close to the Earths surface and subject only to gravity, a projectiles path is a _________________.
The acceleration of a _________________ near the central body equals the gravitational field.
Close to the Earths surface, all objects projected horizontally from the same height have the same_________________.
A _________________ is apparatus used to assist in mineral exploration.
If drag is negligible, then a projectiles range is determined only by initial velocity and _________________.

22

space

Reviewing
1 Given that the Earth rotates, account for why when you jump straight up,
you land on the same spot.

2 The high jump and the long jump both involve a run-up and then a jump.
Using ideas from projectile motion, briefly compare and contrast the ideal
characteristics of the run-up and jump for the two sports.

3 A projectile takes 1.25s to reach its maximum height. What is its time of

Solve problems and analyse


information to calculate the
actual velocity of a projectile
from its horizontal and vertical
components using:
vx2 = ux2
v = u + at
vy2 = uy2 + 2ayy
1
x = uxt; y = uyt + ayt2
2

flight, assuming the ground is horizontal and drag is negligible?

4 Explain why (assuming negligible air resistance) all objects projected


horizontally from the same height have the same time of flight as an object
dropped from that height, regardless of their initial speed.

5 Predict what would happen to the magnitude of the gravitational force


between two masses:
a if one of the masses were doubled
b if both masses were doubled
c if the distance between the masses were doubled.

6 Describe how (and explain why) g would differ slightly from average at a point
on the Earths surface above an oil deposit.

7 Youve seen diagrams of electrical field lines around positive charges in which

Solve problems and analyse


information using:
mm
F =G 1 2
d2

the arrows point outwards (see in2 Physics @ Preliminary section 10.6).
Briefly discuss the possibility of a planet with gravitational field lines that
point outwards. Propose how you would expect a test mass to behave there.

8 Without doing a calculation, deduce the speed at which a meteorite would hit
the Earths surface if it started from rest at a very large distance from the
Earth. Justify your answer. Ignore air resistance and gravity of other
astronomical bodies. (Hint: The value is one already calculated elsewhere in
this chapter.)

9 Read the definition of gravitational potential energy EP in section 1.3 page 16.
Explain why it is necessary to specify in the definition that the work is done
with no net change in speed. (Hint: What other form of energy is involved?)

Solving problems
10 Repeat the calculation in the worked example accompanying Figure 1.1.7,
assuming that the ball lands on the flat roof of a 2.5m high garage, instead
of the ground.

11 Consider the worked example accompanying Figure 1.1.9. Keeping


everything unchanged except initial speed:
a What would the initial speed of the ball need to be if the ball hit the wall
when it was just at its maximum possible height? What would be its time
of flight?
b What would the initial speed of the ball need to be if the ball hit the
ground just in front of the wall? What be would its time of flight?

Solve problems and analyse


information to calculate the
actual velocity of a projectile
from its horizontal and vertical
components using:
vx2 = ux2
v = u + at
vy2 = uy2 + 2ayy
1
x = uxt; y = uyt + ayt2
2

12 By considering the vertical component of velocity and ignoring air resistance,

derive an expression (containing initial speed u and launch angle ) for the
time taken for a projectile near the Earths surface to reach its maximum
height. Then show that the time of flight for a projectile fired over horizontal
ground is given by:
2u
t=
sin
g

23

Cannonballs,
apples, planets
and gravity

13 A marble rolls horizontally off the edge of a 1.00m high table with a speed
of 3.00ms1. Calculate the speed with which it hits the ground, by:
a using the equations of projectile motion
b assuming the conservation of mechanical energy (using the simple
version of the equation for GPE).

Solve problems and analyse


information using:
mm
F =G 1 2
d2

14 Repeat the calculation in the worked example accompanying Figure 1.2.2,


with the positions of the Moon and Earth swapped.

15 Using your own mass, calculate the maximum force of gravity exerted by

the planet Mars (m=6.421023kg) on you, given that the closest


approach of Mars to Earth is approximately 5.61010m. How close
would you need to stand to the centre of mass of a 10tonne truck for the
magnitude of the gravitational force it exerts on you to be the same?
(1tonne = 1000kg)

16 Show that g is 0.28% lower on top of Mt Everest (8848 m) than at sea


level. Data: Mean Earth radius rE=6.367106 m.

17 Calculate the change in GPE in moving a 10kg object from an initial


position 1000km above the surface of the Earth to a final position at a
distance from the Earth equivalent to the mean orbital radius of the Moon
(r=3.84108m). Assume the Moon is on the opposite side of its orbit
at the time and you can ignore its gravitational effect.

18 Using the data and answer from Question 17, calculate the speed at which
you would need to project the 10kg object radially outwards from the
initial position so that it would just reach the final position, stop and fall
back to Earth. (You can ignore air resistance above an altitude of
~1000km.)

19

a Calculate the velocity required for a projectile to escape the Suns


gravitational field (mSun=1.991030 kg) if launched from the orbital
radius of the Earth (1.501011m), if the Earth and other planets
werent there.
b Using part a and Earths escape velocity (11.2 km s1), show that the
total escape velocity from the solar system for a projectile launched
from Earth is 43.6 kms1. Assume the projectile doesnt pass near
other planets.

Extension
Solve problems and analyse
information to calculate the
actual velocity of a projectile
from its horizontal and vertical
components using:
v x2 = u x2
v = u + at
vy2 = uy2 + 2ayy
1
x = uxt; y = uyt + ayt2
2

20 By considering the horizontal component of displacement for a projectile


and your answer for Question 12, derive an expression (containing initial
speed u and launch angle ) for the horizontal range. Either by using
calculus or by considering the properties of trigonometric functions, show
that the maximum range is attained for a launch angle of 45.

21 A wildlife reserve ranger needs to hit a monkey in a tree with a tranquiliser


dart in order to capture and examine it. The barrel of the dart gun is
pointing exactly at the monkey. The angle between the barrel of the dart
gun and the horizontal is not 90. At the instant the ranger fires, the
monkey is startled and drops from rest to the ground below.

Re

24

iew

Q uesti o

Show that the dart will hit the monkey. (Hint: Show that by the time the
dart reaches the horizontal position of the monkey, both the dart and
the monkey have the same vertical position. Assume that air resistance
is negligible.)

space

PHYSICS FOCUS
How to weigh the Earth

torsion wire

1. The history of physics


Solve problems and analyse information using:
mm
F =G 1 2
d2
Analyse the forces involved in uniform circular
motion for a range of objects, including satellites
orbiting the Earth.

Newton first tested his law of universal gravitation


by showing that gravity was responsible for both
the acceleration of a falling apple (9.8ms2) and the
centripetal acceleration (see in2 Physics @
Preliminary section 2.3) of the orbiting Moon.
However, he didnt know the Earths mass ME or the
value of G, but by using ratios of acceleration and
distance squared, he showed that GME = ad2 is the
same for an apple and the Moon, confirming that
the same law of gravity applied to both.
In 1798, the Earths mass was finally measured.
Henry Cavendish (17311810) (who discovered
hydrogen) measured the average density of the Earth
E to be 5.448 times denser than water (1000kgm3).
The experiment was designed by John Michell
(17241793) (who first predicted the existence of
black holes). Since the Earths radius was accurately
known, this was equivalent to both weighing the Earth
and measuring the value of G.
Cavendish used an extremely sensitive torsional
balance (Figure 1.4.4) to measure the tiny
gravitational attraction between two small lead
spheres m (attached to a thin 1.86m rod) and two
nearby large lead spheres M. From the angle of twist
in the calibrated torsion wire, he determined the
gravitational force between the spheres. From this,
and using Newtons equation for gravitational force,
he calculated the Earths average density.
Vibrations, temperature variations and slight air
movements would disturb the apparatus, so it was built
into a small sealed building, with Cavendish outside,
operating the apparatus via cords and pulleys, and
making observations through telescopes in the walls.

F m

Figure 1.4.4 (a) Schematic and (b) cutaway view of the apparatus
used by Cavendish to weigh the Earth

The shift in position of the smaller masses was


about 4mm.
1 Because of Earths gravitational field, the Moon
must accelerate towards Earth. Why doesnt the
Moon crash into Earth?
2 Using Cavendishs value for Earths density ,

the definition =

mass
and the mean radius
volume

rE=6.37106m, calculate the Earths mass and


compare it with the modern value.
3 Using Newtons universal gravitation equation,
Cavendishs value for mE, the modern values for rE
and g=9.8ms2, calculate G and compare it with
the modern value.
4 Using the modern value for G, calculate the total
gravitational force between the spheres measured
by Cavendish. (Hint: Calculate the force between
a small and a large sphere in a single pair and
double it. Ignore the thin rod etc.)
Distance between sphere centres r=0.225m,
large sphere mass M=158kg,
small sphere mass m=0.73 kg
5 Typical laser printer paper weighs 0.080kgm2.
Calculate the size (in mm) of a square piece of
printer paper that would have a weight on Earth
equivalent to the force in Question 4.

25

Explaining
and exploring
the solar system
Getting up there

propellant, impulse, exhaust velocity,


reaction device, thrust, payload, g-force,
effectively weightless, lift-off, Keplers laws,
satellite, ellipse, orbital velocity, eccentric,
semimajor axis, periapsis, apoapsis, perihelion,
aphelion, perigee, apogee, hyperbola, closed or
stable orbit, geosynchronous, geostationary,
medium Earth orbit, semi-synchronous,
gravity assist, slingshot effect, re-entry,
orbital decay, drag, lift, supersonic, hypersonic,
shock wave, heat shield, ablation

How many times have you been told to stop dreaming and
be practical? For scientists and engineers, both dreams
and practical know-how were potent tools to turn the
understanding of the physics of gravity and motion into the
technology of space travel. Most of the important pioneers
of rocketry were inspired to pursue dreams of space travel
by reading Jules Vernes (18281905) story From the
Earth to the Moon, or the stories of HGWells
(18661946). But they also had a solid
grounding in physics and engineering.

2.1 Launching spacecraft


In his book Philosophiae Naturalis Principia Mathematica (Principia for short),
Newton used his law of gravity and laws of motion to explain Keplers laws
of planetary motion, but also predicted the launching of artificial satellites
and projectiles capable of escaping Earths gravity. Once you understand the
physics behind something, it becomes possible to create new technology.
In the case of space flight, it took 300 years to release the potential buried
within Newtons equations, via the 2000-year-old Chinese technology
of fireworks.

A bite-size history of rocketry

Figure 2.1.1 The Apollo 11 mission: the launch


of a Saturn 5 boosterthe largest
rocket in historyon its way to
deliver the first humans to the Moon
26

For most of the history of rocketry, starting with the invention by the
Chinese of gunpowder (the first rocket fuel or propellant) sometime
between 300bc e and 850 c e , the technology was driven mainly by military
applications. The Chinese invented the first rockets or fire arrows (fireworks
tied to arrows). Some of the early milestones of this history are summarised
in Table2.1.1 in the Physics Feature Fire Arrows on page 29.
Only in the 20th century were civilian and scientific applications
of rocketry (space exploration, Earth monitoring and communications)
finally considered to be potentially as important as the military ones.

space
Here well concentrate on the important
rocket researchers of the 20th century, the period
in which the most rapid scientific advances took
place. Below is a list their most important
contributions.

PRACTICAL
EXPERIENCES
Activity 2.1

Activity Manual, Page


14

Konstantin Tsiolkovsky (18571935)


Tsiolkovsky (also Tsiolkovskii), a Russian mathematics teacher, derived the basic
rocketry equations including the Tsiolkovsky rocket equation (see Physics Phile
This is rocket science, p 30), used Newtons definition of escape velocity to
calculate it for Earth, and proposed multi-stage rockets and steerable thrusters.
He advocated the use of liquid propellants (including liquid hydrogen) because
they could be controlled using valves and would give a larger impulse than solids
(see in2 Physics @ Preliminary section 4.5). He also wrote science fiction,
predicting space stations, and space colonies using biological recycling of food
and oxygen and airlocks for moving between a spacecraft and vacuum.
Robert Goddard (18821945)
Goddard, a US physicist, invented and tested many practical aspects of rockets,
launching the first liquid-propellant rockets (liquid oxygengasoline) in 1926.
He confirmed experimentally that rockets work in vacuum and showed that an
hourglass-shaped deLaval steam nozzle greatly increased rocket efficiency. He
launched the first scientific payload (camera, thermometer and barometer) that
parachuted back to Earth, and steered rockets using vanes to direct exhaust gas
and a gimballed (pivoted) nozzle under the automatic control of a gyroscope.
He even experimented with very futuristic ion thrusters. Goddard attracted
public ridicule by predicting travel to the Moon (see in2 Physics @ Preliminary
Physics Phile p 43). He was mostly ignored by the US government, but he
strongly influenced Oberth, von Braun and Korolyov (see below).
Robert Esnault-Pelterie or REP (18811957)
REP, a French aircraft designer, wrote on interplanetary travel, calculated the
energies and flight times for trips to the Moon, Venus and Mars and proposed
atomic energy to power interplanetary craft. With Andr Hirsch, he established
the REPHirsch Prize for aeronautics, the first winner being Oberth (below). In
1931, Esnault-Pelterie conducted early experiments with liquid propellants
(petrolliquid oxygen, benzenenitrogen peroxide and tetranitromethane) and
developed a gimballed nozzle.
Herman Oberth (18941989)
The German physicist Oberths PhD thesis describing space travel was initially
rejected as utopian (though it was later accepted), so he published it as an
influential book By Rocket into Planetary Space. In it he developed equations for
space flight, proposed a design for a two-stage rocket using hydrogenoxygen
propellant and described craft for human space exploration. A follow-up book
won him the REPHirsch Prize, which he used to purchase rocket engines for
research assisted by his student Wernher von Braun. He worked (with vonBraun)
on both the Nazi V2 rocket program and later the American rocket program. In
1953 he published Man in Space, proposing space stations, space-based
telescopes and space suits.

Figure 2.1.2 Konstantin Tsiolkovsky

Figure 2.1.3 Robert Goddard

Figure 2.1.4 Robert Esnault-Pelterie

Figure 2.1.5 Herman Oberth


27

Explaining
and exploring
the solar system

Figure 2.1.6 Wernher von Braun

Figure 2.1.7 Sergey Korolyov

Figure 2.1.8 Gerard ONeill

28

Wernher von Braun (19121977)


As a student, von Braun (German physicist and aeronautical engineer) tested
Oberths rocket engines. He was an early amateur researcher in the Spaceflight
Society, which was taken over by the Nazis. Under the Nazis von Braun led the
team that developed the alcoholoxygen-fuelled A4 (or V2) rocket used on
Allied cities including London, killing and wounding thousands. After the war,
he joined the US armys nuclear missile program. He dreamed of a civilian
space program. In magazines and television, he publicly promoted exploration
to the Moon and Mars with permanent colonies and orbiting space stations
serviced by re-usable shuttle-type craft.
In 1957 the USSR launched Sputnik, the first artificial satellite, shocking
the US and leading to the space race of the 60s between the USSR and the
US. In response, a civilian space agency, the National Aeronautics and Space
Administration (NASA), was formed, and in 1960 von Braun became director
of its Marshall Space Flight Center. He became a major figure in the race to
the Moon (the Apollo missions) announced in 1961 by President Kennedy.
He led the project to construct the largest rocket ever builtthe Saturn5
(Figure 2.1.1).
As is well known, the US won the race to the Moon in 1969, although they
spent much of the 60s catching up to many USSR space firsts. The race also
led to rapid development of civilian satellites for communications, Earth
surfaceatmospheric monitoring and scientific space exploration.
Sergey Korolyov (also Sergei Korolev) (19071966)
Korolyov, a Ukrainian-born Russian aircraft designer, was known only as the
Chief Designer of the USSR space programhis name was kept secret until
his death. He helped set up the Jet Propulsion Research Group, which
launched liquid-fuelled rockets in 1933, and led to the USSR government
forming the Jet Propulsion Research Institute, with Korolyov as Deputy Chief.
During Stalins Great Purge of 1938, Korolyov was imprisoned for 6 years,
then released to become a rocket designer in the nuclear missile program,
where he quickly improved on the design of captured Nazi V2 missiles.
Like his US rival von Braun, he dreamed of space travel and tried to
convince his government to allow civilian projects. In 1957, he was allowed to
launch the first artificial satellite Sputnik into orbit, starting the space race.
He oversaw a string of space firsts (and failures): first animal (dog) in orbit,
first unmanned Moon landing, first image of the unseen side of the Moon,
first man and first woman in orbit, first extra-vehicular activity (space walk),
first fly-pasts of Venus and Mars and more. Launch failures of four N1 boosters
(rival to von Brauns Saturn 5) and Korolyovs death in 1966 helped to lose
the race to the Moon for the USSR.
Gerard ONeill (19271992)
ONeill, a US physicist, invented the particle storage ring used in particle
accelerators, and an early wireless computer network. He led development of
a satellite positioning systema precursor to the US global positioning system
(GPS). Through conferences, papers and books, he was an energetic advocate
of space travel. He proposed colonies in cylindrical spacecraft positioned at

space
Lagrange points. (These are five stable locations around pairs of orbiting
bodies such as Earth and Moon at which a test mass can remain indefinitely,
requiring little or no thrust.) He suggested that colonists would live on the
inner surface of these cylinders 3km in radius and 20km long. The cylinders
would spin, using centripetal force, to simulate gravity, and the inside would be
covered with Earth-like geography.

Identify data sources, gather,


analyse and present information
on the contribution of one of the
following to the development of
space exploration: Tsiolkovsky,
Oberth, Goddard, EsnaultPelterie, ONeill or von Braun.

PHYSICS FEATURE
Fire arrows

he following table is a
very incomplete summary
of some of the highlights of
the 24-century long history
of rocketry.

Figure 2.1.9 The Chinese character


for rocket translates
literally as fire-arrow.

1. The history of physics

3. Applications and uses


of physics

4. Implications for society


and the environment

Table 2.1.1 Some milestones in the pre-20th century history of rocketry


300 BCE to 850

At some time between these dates, the Chinese invent gunpowder and fireworks.

11501200

The Chinese develop the first rockets, fire arrows (fireworks tied to arrows), and
projectile weapons including grenades and cannons are used against invading
Mongols.

12001300

Invading Mongols bring Chinese rocket technology to Europe and the Arabian
Peninsula.

15291556

Conrad Haas (Austria) proposes the first designs for multi-staged rockets.

1687

Isaac Newton publishes Philosophiae Naturalis Principia Mathematica containing


his three laws of motion and the law of universal gravitation. He defines escape
velocity and predicts artificial satellites.

~1730

German Colonel von Geissler manufactures rockets (up to 54 kg) for warfare.

1792, 1799

Sultan Tipu (India) uses iron-cased 1km range rockets against British troops.

18031806

Impressed by Tipu, Sir William Congreve (Britain) develops more accurate 3km
range rockets up to 136 kg, which were used successfully against Napoleons ships
and against the Americans in the war of 1812.

19th century

Engineers, scientists, inventors and crackpots experiment with non-military


applications of rockets.

1821
18611865

Rocket-propelled harpoons are used to hunt whales.


Rockets are used in the American Civil War.

1865

Science fiction writer Jules Verne (France) publishes From the Earth to the Moon.

1903

Konstantin Tsiolkovsky (Russia) publishes reports in which he applies rigorous


physics to rocketry and discusses the possibility of space travel.

29

Explaining
and exploring
the solar system

Forces and rockets

This IS rocket
science!

sing calculus and Newtons


second law, Tsiolkovsky derived
his famous delta v rocket
equation:
m
v = v e ln i
mf
where v is the magnitude of
velocity change during a rocket
burn, veis the exhaust velocity and
mi and mf are the initial and final
masses of the rocket (plus
remaining propellant). However, a
recently discovered pamphlet by
William Moore showed he had
derived a similar equation in 1813.

Analyse the changing


acceleration of a rocket
during launch in terms of the:
Law of Conservation
of Momentum
forces experienced
by astronauts.

To understand the forces exerted on rockets and astronauts during take-off,


we first need to define some terms.

Thrust
Tsiolkovsky called a rocket a reaction device. This is because the burning
propellant forms hot, high-pressure exhaust gas that is forced through the nozzle
at high exhaust speed ve. By Newtons third law (see in2 Physics @ Preliminary
section 3.5), the large force exerted on the exhaust gas results in a reaction (called
thrust) back on the rocket, pushing it forward. You can use momentum to
calculate the thrust. (See in2 Physics @ Preliminary Worked example, p70.)
Lets analyse the [rocket + propellant + exhaust] as our system. The forces
ejecting exhaust out of the nozzle are internal forces, so they cant change the
net momentum of the system; therefore, momentum gained by the ejected
exhaust (meve) must be cancelled by the [rocket + propellant] gaining equal
and opposite momentum. An increase in momentum (impulse) of the
[rocket + propellant] implies acceleration and, hence, a force (called thrust FT).
Suppose the speed of the exhaust gas ve is constant over a time period t.
The impulse J=FTt exerted on the [rocket + propellant] and the impulse
(meve) exerted on the exhaust gas are equal in magnitude:
J=FTt = (meve) = veme
Rearrange:

FT = v e

me
t

me is the mass of exhaust gas lost per unit time.


Increasing exhaust
t
speed ve is important in rocket design because it increases thrust FT. Even if
thrust FT=mRa is constant, because the mass mR of the [rocket + propellant] is
rapidly reducing, the acceleration a rapidly increases during launch.

FTwhere
= ve

Worked example
Question
The thrust equation doesnt only apply to rockets. A fireman holding a hose was not prepared
when the water was turned on and was knocked over by the unexpected thrust. Water exited
the spout with a speed 39.0ms1 and with a flow rate 470Lmin1.
Calculate the force that knocked him over. (Water density is 1000 kg m3 = 1.00kgL1.)

Solution
Mass flow rate of water = 470Lmin11.00kgL1/60s = 7.833kgs1
me
Equation for thrust: FT = ve
= 39.0ms17.833kgs1 = 305N
t

Rocket engines and stages


There are two basic kinds of rocket engine, those using solid and those using
liquid propellant (Figure 2.1.10). Solid propellant engines are simpler and can
achieve maximum thrust faster, but cannot be controlled once they start. Liquid
propellant engines are more complicated and slower to start, but can be
controlled and produce greater thrust.
A multi-stage rocket can deliver a heavier payload (space cargo) because, when
the propellant in each stage is finished, that stage can be jettisoned (falling into
the ocean) reducing the rockets mass and so allowing for greater payload mass.
Rockets from the space-race era were typically liquid-propelled stages stacked
30

space
vertically. More recent rockets such as the US Space Shuttle and the European
Ariane 5 use a combination of liquid- and solid-propelled stages stacked side-byside. Russian rockets, such as the Proton, use only liquid propellant. Of course,
multi-stage rockets are also more complicated and so have more ways to fail.
In the US, smaller satellites and military missiles are launched using simpler,
solid-fuel rocket engines.

fuel
solid fuel
oxidiser
mixture

g-force
oxidiser
Maybe youve heard that astronauts are squashed by g-force when a rocket
accelerates on take-off. Often the term g-force is used to quantify the effects on
pumps
your body of accelerations experienced in a roller-coaster, car or aeroplane. Its not
combustion
an accurate name, because its value is more closely related to acceleration than
chamber
force, but it is used extensively in aeronautics and astronautics, so well give you
combustion
a commonly used definition. The g refers to acceleration expressed in units of
chamber
g=9.8ms2. The force refers to the fact that a net external force is responsible for
nozzle
nozzle
that acceleration and it is this force and resulting reaction forces within a body that are
responsible for the effects of g-force. Sometimes the term g-load is used instead.
exhaust
exhaust
Lets start with the vertical component of motion and g-force. Consider three
Figure 2.1.10 Typical (a) solid-propellant
situations:
and (b) liquid-propellant
1 While you are sitting or standing still (not accelerating), there is no net force
rocket engines
on you. Your body is compressed by a pair of balanced forcesweight mg
downwards, and the upward normal force N from the seat or floor. (See in2
Physics @ Preliminary p45.) This compression causes the internal effects of
weight. Your body is experiencing the compressive effect of 1 unit of Earth
gravity (i.e. g-force = 1).
2 If you are accelerating upwards, such as riding the bottom of a curve on
a roller-coaster or in a rocket during take-off, the net force is upwards
the normal force from the seat is larger than your weight. Your body compresses
more than usual, as though you are heavier. If your net upward acceleration is
9.8ms2 (1g), then your body is compressed as though you are in a
gravitational field of 1+1=2 units of Earth gravity (i.e. g-force=2).
3 In free-fall (or in orbit), the normal force from the chair disappears and you
Identify why the term g forces
are no longer compressed. You feel effectively weightless (see in2 Physics @
is used to explain the forces
Preliminary pp3738), even though at typical Space Shuttle altitudes you
acting on an astronaut during
actually have ~90% of your weight on Earth. In this case you are experiencing
launch.
g-force=0, the same effect as 0 units of Earth gravity.
The term g-force usually means apparent weight divided by true weight
on Earth. Apparent weight is what appears on a set of bathroom scales. Bathroom
scales actually measure the magnitude of the normal force, not true weight, so,
to calculate g-force, first calculate normal force. Consider vertical motion only.
Weight mg is down and normal force N is up. Let av be vertical acceleration and
let up be positive:

Fnet = mav = N + (mg)
Apparent weight N:

N = mav + (+mg)

The apparent weight increase is caused by the increase in normal force N due
to the term mav, which is simply the net force accelerating you.
Vertical g-force = N mav + ( + mg)
=
mg
mg
a
Vertical g-force = gv + 1
31

Explaining
and exploring
the solar system

The First
Astronauts?

ccording to Chinese legend,


in 1500 a senior bureaucrat
(a Mandarin) called Wan Hu
tried to launch himself into
space by tying 47 gunpowder
rockets to a chair. He failed to
become the first astronaut by
dying in the explosion at launch.
In about 1806 in France,
Claude Ruggieri launched
a rocket containing a sheep
~300m into the air, parachuting
it back to Earth alive. The police
prevented him from turning a
small boy into the first astronaut
by the same method.

The agv term in the g-force formula represents the effects of your acceleration
due to the net force and the +1 represents the background effects of your real
weight.
Notice that in free-fall your acceleration is g, so the g-force is 1+1=0.
If you are holding a rope or are strapped into a harness being pulled upwards
instead of sitting or standing, then the above discussion and formulae still apply
except that now your body is being stretched and the normal force is replaced by
the tension force.
The horizontal component of g-force, such as when you accelerate or
decelerate in a car, is easier to calculate. (See in2 Physics @ Preliminary Physics
Phile gWhiz p11.) As there is no horizontal component of weight mg, the
g-force equation simplifies:
Horizontal g-force = agh
To calculate the resultant g-force, combine the vertical and horizontal
components of g-force using vector addition.

Worked example
Question
You round the bottom of an upturning curve on a roller-coaster at a speed of 36.0km h1.
The curve is circular with radius 5.00m. Calculate the g-force you would experience.

Solution
Analyse the forces involved in
uniform circular motion for
a range of objects, including
satellites orbiting the Earth.

At that moment, you experience (upward) centripetal acceleration (see in2 Physics @
Preliminary section 2.3). Calculate the vertical g-force.
36
v = 36 km h1 =
m s1 = 10 m s1
3.6
ac =
Vertical g-force =

v 2 102
=
= 20.0 m s2
R 5.00
av
+20.0
+1 =
+ 1 = 3.04
g
9.80

One effect of g-force is that the apparently increased weight of the blood
drains it from the head, affecting vision and consciousness. On average, 45 g
causes dimming of vision, 56 g visual blackout and above 6 g you experience
loss of consciousness (g-LOC). Much larger g-forces can be tolerated for periods
of less than about 4seconds.
To increase g-force tolerance during launch, astronauts face the direction of
acceleration. This orientation is called eyeballs in, because the eyeballs are
effectively pushed into their sockets. Also, the seats are oriented with the head
and body lying horizontally (Figure 2.1.11). In this way, g-force doesnt easily
force blood into or out of the head. Fighter pilots and astronauts also wear
gsuits containing inflatable bladders in the trousers which squeeze blood out of
the legs and back into the head.
Using a powerful cannon to launch a satellite (see Newtons thought
experiment in section 2.2) would not work because of the enormous g-force
from the initial explosion.
32

space

Figure 2.1.11 Gemini 3 astronauts Gus Grissom


and John Young strapped into their
horizontally oriented seats are being
prepared for launch (1965).

Warning! The terms g-force and g-load are not SI quantities. They are
informal terms and are sometimes used carelessly. Sometimes g-force and g-load
are used to mean the same thing. Sometimes g-load is used to mean only the net
acceleration in units of g, not including the effect of gravity. When reading
g-force or g-load data, be careful to check which definition is being used.

Force during take-off


Here well account for the forces, accelerations and g-force experienced during
a typical launch. This example is for a Space Shuttle, but the principles apply to
other craft. In the Shuttle, the solid rockets boosters and the main liquidpropelled engines fire-up at the same time, while in a more traditional multistage rocket each stage fires sequentially.
Figure 2.1.12 is a graph of g-force experienced by everything within the
Shuttle during launch. Just before lift-off (or take-off ), the vector sum of thrust
plus the force exerted by the gantry (the crane-like structure holding the rocket)
plus the rockets weight is zero, so there is no acceleration. Because acceleration
is zero, the net force on the astronaut is also zero, so the astronauts weight is
balanced by the normal force exerted by the seat: g-force=1 (point A).
After lift-off, thrust is larger in magnitude than rocket weight plus air
resistance, so the rocket (and astronaut) accelerate upwards. Now the seat exerts
a normal force greater than the astronauts weight. The g-force is greater than 1,
increasing steadily, along with acceleration as the mass of remaining propellant
decreases. Note that throughout the launch process, the craft is pitching over
from vertical to horizontal motion, so the gravitational contribution to g-force is
becoming progressively smaller in the direction of motion.
The ~0.2 drop in g-force between points A and B is due to throttle down;
air resistance-induced pressure on the Shuttle surface reaches a dangerous
maximum (maxQ), so thrust is deliberately reduced until the atmosphere thins
out. Thrust is increased again and g-force increases to between 2 and 3 (pointB).
As the mass of remaining propellant decreases, acceleration (and g-force)
increases until fuel in the boosters (or lower stage in a traditional rocket) runs
out. Acceleration decreases dramatically, but the boosters (or empty stages) are
discarded (point C), and the remaining engines provide the thrust. Acceleration
increases again as propellant mass decreases. To avoid the astronauts and payload

Analyse the changing


acceleration of a rocket
during launch in terms of the:
Law of Conservation
of Momentum
forces experienced
by astronauts.

33

Explaining
and exploring
the solar system
external tank
separation

C solid rocket

E in orbit

D accelerating up

booster
separation

to orbital speed

3.5

3.0

B reduced

air resistance

g-force

2.5

lift-off

2.0
1.5
1.0

0.5
0

E
100

200

300

400

500

600

Time (s)

Figure 2.1.12 g-force during a typical Shuttle launch

Analyse the forces involved in


uniform circular motion for
a range of objects, including
satellites orbiting the Earth.

Discuss the effect of the


Earths orbital motion and its
rotational motion on the launch
of a rocket.

Earths rotation

being subjected to a dangerously high g-force, the thrust must be reduced to


limit g-force to 3 g (pointD).
Once the rocket is in orbit, the rockets stop firing (point E). The only force
acting now is weight (providing the centripetal force of orbit), so the rocket and
the astronaut are both in free-fall and effectively weightless; the astronaut
experiences a zero g-force.

Running start
It takes a lot of fuel to get a spacecraft to a high enough altitude and high
enough speed to achieve orbit. You can get higher if (like a pole vaulter) you get
a run-up before lift-off. Given that Earth rotates rapidly, a rocket already has a
large easterly tangential velocity at launch. So, if you launch towards the east,
you can use less propellant, carry a larger payload or go into a higher orbit. The
closer you are to the equator, the faster your initial speed u (Figure 2.1.13).
At the equator u=465ms1.

Worked example
Question
Compare Earths rotational tangential speed vT at the rocket launch facilities at Woomera,
Australia (used during the 60s and 70s), and Kourou, French Guiana.

v1
v2
tangential velocity

Figure 2.1.13 Rockets are usually launched


towards the east, to take
advantage of Earths rotation.
The effect is greatest at the
equator.

Data:

Earths radius rE = 6.37106m

Earths rotational period T=86 164s

Woomera: Latitude 31.1S, longitude 136.8E.

Kourou:

Solution
Tangential speed: v T =

Latitude 5.2 N, longitude 52.8W


2R

T

(See in2 Physics @ Preliminary p 29.)

Radius of rotation R depends on lat, the latitude angle: R = rEcoslat


34

space

vT =

2rE
2 6.37 106
cos lat =
cos lat = 465 cos lat
T
86164

Not enough hours


in a day

Woomera: 465cos31.1 = 398ms1


Kourou:

465 cos 5.2 = 463ms

To explore the solar system, you need to reach Earths escape


velocity. As the Earth also orbits the Sun, you can get an extra boost
from Earths orbital speed (about 3.0104ms1) if you launch at
a time of the year when the Earths orbital motion points in the desired
direction (Figure 2.1.14).

direction of Sun

orbital
motion

he absolute period of Earths rotation


as determined from the orientation of
distant stars is only 23h, 56min and 4s,
or 86164s (a sidereal day). The normal
24 hour mean solar day is longer because
it includes the extra time needed for the
Earths rotation to catch up with the extra
component of the Suns apparent motion
in the sky due to the Earths orbit.

Earth

Figure 2.1.14 Interplanetary missions require higher launch


rotational motion

velocities. These launches need to take


advantage of the Earths orbital velocity.

Checkpoint 2.1
1
2
3
4
5
6



7
8

Which two people led the space race between the USSR and the US?
What event triggered the US government to fund a large civilian rocket program?
What are Lagrange points?
Name the two basic kinds of rocket engine.
Explain why a multi-stage rocket allows a heavier payload.
What is the vertical g-force on someone:
a standing stationary?
b in free-fall?
c in orbit?
Explain why the acceleration of a firing rocket increases with time.
Explain one reason why most rockets are launched towards an easterly direction.

2.2 Orbits and gravity


How and why do spacecraft stay up there? Newton, using his three laws of motion
and his law of universal gravitation, showed not only that gravity provided the
centripetal force required to keep the Moon in orbit around the Earth, but also
that he could use his laws to explain all of Keplers laws of planetary motion.
Newton developed a thought experiment to understand orbit. He imagined
standing on a mountain (Figure2.2.1) and firing a projectile horizontally from
a powerful cannon. Gravity would accelerate the projectile towards the ground,

Discuss the importance of


Newtons Law of Universal
Gravitation in understanding
and calculating the motion
of satellites.

35

Explaining
and exploring
the solar system
curving it downwards until impact. If you increased the projectiles initial
velocity, it would travel further around the Earth and its trajectory would be less
steeply curved. Eventually you would reach an initial velocity at which the
curvature of the trajectory exactly matched the curvature of the Earth itself, so
that the projectile would never catch up to the groundthe projectile was now
in a circular orbit. The projectile would become an artificial satellite (an object
in orbit around a much larger one) in the same way that the Moon is a natural
satellite of Earth. Newton also showed that if you increased the velocity further,
the orbit would become an ellipse. At high enough velocity (escape velocity), the
projectile would never return.

Figure 2.2.1 This illustration from Newtons


Principia illustrates the
principles behind orbits, and
launching an artificial
satellite.

PRACTICAL
EXPERIENCES
Activity 2.2

Activity Manual, Page


17

Vi
B

Vf
R

R
Vf

Vi

Figure 2.2.2 This diagram shows the


derivation of the centripetal
acceleration formula.
Define the term orbital velocity
and the quantitative and
qualitative relationship between
orbital velocity, the gravitational
constant, mass of the central
body, mass of the satellite and
the radius of the orbit using
Keplers Law of Periods.

Deriving centripetal force


You have seen the formula for centripetal acceleration ac, but where does it come
from? Look at Figure2.2.2. Suppose an object moves uniformly in a circle. The
magnitudes of the initial and final velocities (vi and vf ) are the same, so call them
both v (v=vi=vf ). The distance travelled d=vt in going from A to B is R
(using radians), giving R=vt, which can be rearranged to:
vt
=
R
To find the instantaneous acceleration rather than the average over a long time,
use a t (and therefore ) that approaches zero.
Consider the v vector diagram on the right in Figure2.2.2. It is an isosceles
triangle with two equal sides of length v. For approaching zero, the length of
v approaches the length of the arc v between vf and vi so:
vv
If you combine this with the previous equation, eliminating , then in
the limit:
v 2t
v =
R
v
v2
Divide both sides by t:
= ac =
t
R
Then of course, to get centripetal force, multiply centripetal acceleration
by mass (see in2 Physics @ Preliminary p 46). Dont forget that the magnitude
of tangential velocity is v = 2pr T. For objects in orbit, the tangential velocity is
also called the orbital velocity.

Heaven and Earth


Newton showed that the force acting in the heavens to keep the Moon in orbit
was the same one acting on small projectiles on Earth. He assumed that in both
cases the force is given by his law of universal gravitation. At that time, neither
the Earths mass ME nor the value of G were known, but if you rearrange the
formula:
M m
F = ma = G E
d2
their product GME = ad2 should evidently be the same constant for the orbiting
Moon and a falling apple on Earth. Newton showed that this was true.

36

space
Worked example
Question
Show that GME = ad2 is the same for a falling apple and the orbiting Moon. Assume that
the Moons orbit is circular.
Data:

Earths radius rE=6.37106m

Average EarthMoon distance dEM = 3.84108m

Moons orbital period T=27.3days


Solution
Apple on Earth: GME = ad2 = 9.80ms2(6.37106m)2 = 3.981014m3s2
Moon in orbit: Centripetal acceleration ac =

v
r

2r
4 2r
a = 2
T
T
Orbit radius r = dEM; T=27.3243600 = 2.359106s

Solve problems and analyse


information using:
mm
F =G 1 2
d2
Solve problems and analyse
information to calculate the
centripetal force acting on a
satellite undergoing uniform
circular motion about the Earth
using:
mv 2
F =
r
Analyse the forces involved in
uniform circular motion for
a range of objects, including
satellites orbiting the Earth.

Orbital velocity v =

GME = ad2 =

4 2dEM 3
T

4 2 (3.84 108 m)3


6

(2.359 10 s)

= 4.02 1014 m3 s 2

The two values agree within ~1%.

Extension: (Hard) Can you think of a reason for the ~1% discrepancy? (Hint: See Physics
Phile Finding new planets page40.)

Keplers laws and satellites


Johannes Kepler (15711630), using Tycho Brahes data, showed that the known
planets and Earth orbit the Sun in ellipses that obey his three laws of planetary
motion, but he had no idea why. An ellipse is a circle, stretched along one
dimension. The more stretched the ellipse, the more eccentric it is. Most of the
orbits of planets of our solar system are very nearly circular, not very eccentric.
Comets have highly eccentric orbits.
Figure 2.2.3 defines some properties of elliptical orbits. The semimajor axis
is half the length of the ellipses longest axis. It can also be thought of as a kind of
average radius for the orbit. For a circular orbit, the semimajor axis is the radius.
The point of closest approach of the orbit to the central body is called the
periapsis. The furthest point is the apoapsis. If the central body is the Sun, then
these points can also be called perihelion and aphelion (helios is Sun in Greek).
If the central body is the Earth, then they are the perigee and apogee (geos is
Earth in Greek).
Let a be the semimajor axis and let dA and dP be the distances from the
central body to the aphelion and perihelion respectively, then look at Figure
2.2.3a and confirm that a is the average of these: a=(dA+dP)/2.
Keplers laws of planetary motion in the
You had a sneak preview of
Preliminary text (see in2 Physics @ Preliminary p 250). Here they are again:
1 The orbits of the planets are ellipses, with the Sun at one focus (Figure 2.2.3a).

37

Explaining
and exploring
the solar system
2 A line joining the planet to the Sun sweeps out equal areas in equal times. This
means the planet travels faster when it is closer to the Sun (Figure 2.2.3b).
3 For all planets orbiting the Sun, the square of the orbital period T is
proportional to the cube of the semimajor axis a. This is the law of periods.
T2
a3

constant
= constant

These laws hold for any orbiting system of two bodies if the mass of the
central body is very much larger than the mass of the other. They also apply to
circular orbits, since a circle is a special case of an ellipse.
If c is the half the distance between the foci of an ellipse, then eccentricity e is
defined as e = ac . A circle is an ellipse with zero eccentricity (e=0); both foci are
For a circle, the semimajor axis becomes the
together at the circle centre.
radius, a=r. Orbits of planets in our solar system are very nearly circular, the
two most eccentric being Mercury with e=0.2056 and Mars with e=0.0934.
Pluto has e=0.2482 but, sadly for Pluto-fans, it was demoted to a dwarf planet
in 2006.
a

Sun

semimajor axis

focus

Discuss the importance of


Newtons Law of Universal
Gravitation in understanding
and calculating the motion
of satellites.

perihelion
(closest to
the Sun)

focus

aphelion
(farthest from Sun)

focus

Area

Area

Figure 2.2.3 (a) A highly eccentric elliptical orbit. (b) Keplers law of areas: a line
joining the planet to the Sun sweeps out equal areas in equal times.

Newton derived Keplers laws from his laws of motion and gravitation. He
showed that Keplers first law (law of elliptical orbits) follows from the inverse
square law. By including his three laws of motion, he also proved Keplers second
law (law of equal areas). These derivations are beyond the syllabus; however,
showing Keplers third law, the law of periods, for a circular orbit is very easy.
Careful! Dont confuse the semimajor axis a with centripetal acceleration ac.
Suppose a satellite of mass m orbits a central body of mass M and m<<M
so that the acceleration of M is negligible. In a circular orbit, the satellites
acceleration is centripetal:
2 r
4 2r
v2
ac =
and v =
ac =
r
T
T2
where v is orbital velocity, r is orbital radius and T is orbital period.
The magnitude of gravitational force exerted on m is:
F = mac =

38

GmM
2

ac =

GM
r2

4 2r
T2

T2
r3

42
(which is constant)
GM

space
We derived this for a circular orbit, but it is also true for elliptical orbits if
we replace radius r with semimajor axis a.
T2
a3

42
GM

Because here the value of Keplers constant is explicit, we will call this the
explicit form of Keplers law of periods.

Solve problems and analyse


information using:

Worked example

r3
T

GM
4 2

Question
From Earth, you observe (almost edge on) the orbit of Jupiters moon Ganymede and
determine its orbital period to be T=7.15 Earth days. You measure the width of the orbit
to be w=2.14109m. Assuming the orbit is circular, determine Jupiters mass.

Solution
r=

Assuming orbit is circular:


Explicit form of Keplers third Law:
Rearrange and evaluate:

M=

w
= 1.07 109 m
2

r 3 GM
=
T 2 4 2
4 2r 3
GT 2

4 2 (1.07 109 )3
6.67 1011 (7.15 24 3600)2

= 1.90 1027 kg

An early triumph of the law of universal gravitation occurred when Newtons


friend Edmund Halley used it to show that the trajectory (Figure2.2.4) of a
comet he had observed (now called Halleys Comet) fitted with the trajectories of
two previously observed comets. Halley concluded it was the same comet, and
correctly predicted that it would return every 76 years.
The success of Newtons law of universal gravitation was not simply that it
could be used to explain Keplers laws, which were already known, but that
it could be used to predict other phenomena not yet observed (such as space
travel). It could also be used to accurately predict small deviations of planets from
Keplers ideal orbits around the Sun. For example, when planets pass near each
other, local effects of gravity perturb them from perfect Keplerian orbits. The law
of universal gravitation can be used to predict these deviations very accurately.
Halley included the effects of perturbations due to planets in his comet
calculations. Similar deviations in the orbit of Uranus were attributed to the
gravity of a then unknown planet. Neptune, that new planet, was found in 1846
within 1 of the position predicted using the law of universal gravitation. Several
astronomers contributed to both the calculations and the observations, resulting
in arguments about who deserved credit for discovering Neptune.
The first obvious failure of Newtons gravitation law was in explaining the
observation that the position of the perihelion of Mercurys orbit was not fixed,
but was precessing around the Sun. (See in2 Physics @ Preliminary p255.)
Perturbations due to gravity of other planets and other mechanical effects such as
the Suns equatorial bulge were able to explain 99.23% of the precession, but the
remaining 0.77% required an improved theory of gravitationEinsteins theory
of general relativity.

Uranus

Earth
Halleys
Comet

Neptune

Mars
Jupiter
Saturn

Figure 2.2.4 The eccentric orbit of Halleys


Comet in relation to the nearly
circular orbits of Earth and
other planets. The angle
between the comets orbit and
the plane of the orbits of the
planets is not apparent here.

39

Explaining
and exploring
the solar system

Finding new planets

he mass of a large planet is not negligible


compared with the mass of its star, so both
the planet and the star orbit the centre of mass
of the system with the same period; the planet
orbits on a large ellipse and the (more massive)
star on a small one. Planets outside our solar
system (extra-solar) are normally too far and
faint to be seen directly in telescopes, but
astronomers can deduce the presence of a very
massive planet by detecting the small wobble of
the star it orbits, using the Doppler effect. (See
in2 Physics @ Preliminary p259.) As the star
moves towards and then away from us
periodically, its spectrum shifts towards bluer
and then redder wavelengths in succession.

Define the term orbital


velocity and the quantitative
and qualitative relationship
between orbital velocity, the
gravitational constant, mass
of the central body, mass of
the satellite and the radius of
the orbit using Keplers Law
of Periods.

large planet
centre of mass

star

Figure 2.2.5 A large, undetectable distant planet can


cause its parent star to wobble detectably.

Worked example
Question
Derive an expression for the magnitude of orbital velocity for a satellite in a circular orbit, in
terms of mass of the central body M and orbital radius r. Use this expression to calculate
the Moons orbital speed, assuming a circular orbit.
Data:

Average EarthMoon distance d = 3.84108m


Earths mass Me = 5.971024kg

Solution
Gravitational acceleration:

ag =

GM
r2

Resulting centripetal acceleration: ac = ag =

v 2 GM
= 2
r
r

Rearrange:
v=


Moons orbital speed:

v=

GM
for circular orbits
r

6.67 1011 5.97 1024


3.84 108

= 1020 m s1

Note that orbital speed is independent of the mass of the satellite. One of the
consequences of this equation is that for circular orbits, the smaller the radius the
faster the orbital speed.

40

space

Checkpoint 2.2
1
2
3
4
5
6
7
8
9
10
11

Who first described the principles behind launching artificial satellites?


If an object is in a circular orbit and you increase the orbital speed slightly, describe what happens to the shape
of the orbit.
State the formula for centripetal acceleration.
State Keplers laws. What condition is required in order to apply them to other systems of two bodies?
Define the terms periapsis, perihelion and perigee.
Define the terms apoapsis, aphelion and apogee.
Compare the speed of a satellite at periapsis and apoapsis. Is there an exception?
A circle is also an ellipse. True or False. Explain.
Define the semimajor axis of a circle of radius r.
Name the planet in our solar system with the most eccentric orbit.
Outline the main reason the planets sometimes deviate from Keplers laws. Name the planet that was discovered
because of this and describe (briefly) how.

2.3 Beyond Keplers orbits

central
body

hyperbolic

Isaac Newton showed that all orbits consistent with the Law of
Universal Gravitation and his laws of motion fall into one of four possible
circular
types: the first two suggested by Keplers first lawcircles and ellipses
parabolic
as well as parabolas and hyperbolas. (Remember that a hyperbola is the
curve you get when you graph y = xk ). The central body (if very massive)
would be at the focus of all these curves.
Consider Figure2.3.1 (and Figure 2.2.1). Suppose we increase the
elliptical
speed of the satellite executing the circular orbit. The orbit will become
Figure2.3.1 Gravity allows four possible kinds of orbits.
elliptical. If we further increase the speed so that we reach escape velocity,
the satellite wont return. (See Escape velocity: What goes up ? p 18).
The orbit is a parabola. If we now increase the speed beyond escape velocity,
the orbit becomes a hyperbola.
All these orbits are symmetrical in shape and speedthe speed of
the satellite at any position is identical to its speed at a mirror image position
(although obviously the direction is different).
Circles and ellipses are called closed or stable orbits because the satellite
follows a closed curve, repeating its motion periodically and indefinitely if
undisturbed. However, an object approaching a much larger mass doesnt always
fall into a stable orbit around it. Assuming that it doesnt hit the surface of the
larger mass, an object moving fast enough will execute an open orbita parabola
or a hyperbolaflying off after the encounter, never to return.
The kind of orbit depends on the sign of the mechanical energy ME:
ME = K + E P =

1 2 GmM
mv
2
r

where m is the mass of the satellite and M is the central mass. Here, r is the
instantaneous distance from the central mass. For a circular orbit, r is constant
(= the orbital radius), but r varies with time for an elliptical orbit. In the absence
41

Explaining
and exploring
the solar system

Try this!
Plasticine orbital mechanics
Isaac Newton showed that all possible orbits
are examples of conic sections, curves
resulting from taking planar slices of a cone at
different angles. Make an accurate cone out of
plasticine or foam and use a sharp knife to cut
sections to reveal orbital shapes:
Cut horizontally = circle.
Cut parallel to one edge of the cone = parabola.
Cut less steeply than the parabola = ellipse.
Cut more steeply than the parabola = hyperbola.

circle

ellipse

hyperbola

of resistive forces such as air resistance, the mechanical energy is


constant for any orbit. For stable orbits with semimajor axis a:
GmM
ME =
2a
This implies that the lower (more negative) the energy, the
smaller the orbit.
There are three possibilities:
Orbit is closed (circle or ellipse), ME<0. The orbiting
object is gravitationally bound, v<ve (v=orbital velocity,
ve=escape velocity).
Orbit is a parabola, ME=0. The object is borderline
gravitationally unbound, v=ve.
Orbit is a hyperbola, ME>0. The object is gravitationally
unbound, v>ve.
Note that the parabolic orbit described above doesnt have
the same properties as the parabolic trajectory of a projectile
near the Earths surface.
The projectile motion equations in section 1.1 dont
apply to this parabola. The x- and y-components of the motion
dont separate as easily as for down-to-Earth projectile motion.
A projectile passing by a large massive body (without being
captured) follows a hyperbolic orbit (Figure2.3.3), reaching
maximum orbital speed at closest approach. The two arms of a
hyperbolic orbit (like all hyperbolas) tend towards asymptotes;
that is, the further from the central mass an object is, the closer
its path tends towards a straight line and constant speed. Very
rarely, if the projectiles speed coincidentally equals the escape
velocity, it would follow a parabolic orbit.

Worked example

parabola

Figure 2.3.2 All orbits are conic sections.

asymptotes to
hyperbola

Question
A comet of unknown mass mC passes the Sun (MS = 1.991030kg).
At perihelion, the centre-to-centre distance is r=5.081010m and
the speed of the comet is v=7.50104ms1.
What type of orbit is it? Do you expect the comet to return?

Solution
Gm M
GM
1
1
ME = mCv 2 C S = mC v 2 S
2
r
r
2
hyperbola

Figure 2.3.3 A projectile not captured by a


large massive body follows
a hyperbolic orbit (or very
rarely, a parabolic one).

1
6.67 1011 1.99 1030
= mC (7.50 104)2

5.08 1010
2

= mC ( +2.0 108 )
ME > 0, so the orbit is hyperbolic and the comet is not expected to return
(unless later interaction with another body reduces its energy sufficiently).
(Careful! When subtracting two numbers of very similar size, check
that the answer is not smaller than the smallest significant figure in
the calculation.)

42

space
Strictly speaking, when calculating gravitational potential energy, we really
should include the effect of other astronomical bodies such as nearby stars.
However, as long as the other bodies are either too small or too far away, we can
get away with calculating the two-body ME using only the masses of the central
The rules relating ME to the shape of the orbit only
and orbiting body.
apply to the two-body ME.

Orbits close to home


Compare qualitatively low Earth
The vast majority of artificial satellites (and all Space Shuttle and International
and geostationary orbits.
Space Station missions) are launched into what are called low Earth orbits or LEOs
(1602000km above the Earths surface, Figure 2.3.4). Because the atmosphere
Altitude (km) Radius (km)
extends into this range, such orbits also experience some air resistance and so are
GEO
temporary. The LEO band is heavily used and now highly contaminated by
35 794 42 164
orbital debris (space junk), which can be dangerous in a collision. The positions
of more than 8500 objects larger than 10cm are monitored to avoid this.
GPS
However, LEOs have several advantages over higher orbits. Because the
20 230 26 600
altitude (height above sea level) is small, it requires less fuel to reach. Some
commercial communications satellites use LEOs because, being close to Earths
1602000 65308370
surface, transmitters need less power. Military and civilian surveillance and surface
monitoring satellites (for example those used by Google Maps) often use LEOs
LEO
because, being close, photographic resolution of images is high. Also, the orbital
period is short (between 1.5 and 2hours), so there is rapid coverage of the
Earths surface.
Surrounding the Earth are the Van Allen radiation belts, two belts of energetic
Figure 2.3.4 Ranges of typical satellite orbital
electrons and ions trapped from the solar wind by the Earths magnetic field (see
radii. GEO, geostationary orbit;
in2 Physics @ Preliminary p307). Vehicles in LEO below ~1000km altitude are
GPS, global positioning system
protected from this radiation by distance and the atmosphere, so, except for the
(semi-synchronous); and LEO, low
Apollo Moon missions, human spaceflights do not venture above LEO.
Earth orbit. Orbits lying between
LEO and GEO are medium Earth
The explicit form of Keplers third law allows us to choose a useful orbital
orbits (MEO). Below LEO is called
period. For example, if the orbit is circular with a radius of 42164km, then
sub-orbital.
the orbital period of the satellite equals the rotational period of the Earth
(23h 56min 4s; see Physics Phile Not enough hours in a day, p 35). Such
Solve problems and analyse
a satellite is called geosynchronous and would appear over the same patch of
information using:
Earth at the same time each day.

r 3 GM
=
A special case of a geosynchronous orbit is the circular easterly
T 2 4 2
orbit aligned directly over the equator. The satellite follows the
Earths rotation exactly and so appears stationary in the sky.
This is a geostationary or Clarke orbit (Figure 2.3.4).
Fixed Earth-bound dish antennas can be pointed permanently
Are you serious?
at a geostationary satellite. Most communications and
n 1945 in a Wireless World magazine article,
broadcast satellites use this orbit.
science-fiction writer Arthur C Clarke (who wrote
Another special case of a geosynchronous orbit is the
2001: A Space Odyssey) first described how three
Tundra orbit, a highly elliptical orbit that is highly inclined
geostationary satellites could be used to allow
to the equator and designed to dwell for extended periods
continuous, worldwide radio communication, a feat
each day over high latitudes.
not achieved until the mid-60s.
Medium Earth orbits or MEOs have radii between LEO
Aspects of the idea had already been suggested
and geosynchronous orbits. A semi-synchronous satellite
by earlier researchers, although Clarke produced
has an orbital period exactly half of Earths rotation period.
the most complete plan. He didnt patent his multiGlobal positioning system (GPS) satellites are in circular
billion dollar idea because a lawyer convinced him
it was too far-fetched to be taken seriously.
semi-synchronous orbits (Figure 2.3.4) at various angles to

43

Explaining
and exploring
the solar system
the equator, so that at any time from anywhere at least six satellites are
contactable. Arctic regions are poorly covered by geostationary satellites, so
Russia uses communications satellites in Molniya orbitshighly elliptical,
semi-synchronous orbits inclined to the equator, designed to dwell over Arctic
regions for many hours each day. Russia and the USA both use such orbits for
spy satellites.

Checkpoint 2.3
1
2
3
4
5
6

Name the four shapes of orbits allowed by Newtons law of universal gravitation.
Describe the orbit normally followed by a projectile passing (without capture) a massive body. Explain why it is
rarely a parabola.
List the three different conditions on the sign of two-body orbital mechanical energy, and name the orbits that result.
Outline why low Earth orbits are temporary.
Describe the Van Allen radiation belts.
Define the terms geostationary, geosynchronous and semi-synchronous.

2.4 Momentum bandits:


the slingshot effect

Identify that a slingshot effect


can be provided by planets for
space probes.

44

Now you know how to get into orbit, but what are you going to do while youre
up there? To tour the solar system on very little propellant, you will need to learn
to hitchhike and steal!
When exploring the outer parts of the solar system with a space probe, you
can save fuel and increase payload space by hitching a ride with some of the
planets. Planets carry an enormous reserve of orbital kinetic energy (and
momentum). By steering a space probe into a temporary hyperbolic orbit around
a planet, you can steal some of its momentum using gravity assist (or the
slingshot effect).
Consider Figure2.4.1. A space probe that approaches a planet (for example
Jupiter) with an inbound speed of Vin, is steered into a hyperbolic orbit by the
Jupiters gravity and leaves in a different direction with an outbound speed Vout.
Judged from Jupiters frame of reference (Figure2.4.1a), Jupiter is the central
mass and, as orbits are symmetrical, the two speeds are equal. Using lower case
text for velocities measured in Jupiters reference frame, vout = vin.
To calculate velocities as seen by an observer stationary relative to the Sun,
we simply add Jupiters velocity (relative to the Sun) to each velocity in Jupiters
reference frame. If we do this, it turns out that Vout>Vin; the trip around Jupiter
has increased the speed of the probe (Figure2.4.1b). Momentum is conserved,
so the probes gain in momentum is exactly balanced by the planets loss of
momentum. However, because of the planets enormous mass, the decrease in
its speed is immeasurably small.

space
We can also treat gravity assist as a perfectly elastic collision (see in2 Physics
@ Preliminary p66). The planet pulls the probe rather than pushing and the
collision is gradual, but the conservation of momentum still applies. It is elastic
because gravity conserves mechanical energythere is no friction. The planet
is like an enormously heavy cricket bat and the probe is a small, highly elastic
superball. In the bats frame of reference, the bat is a stationary immovable
object (see in2 Physics @ Preliminary p66), so in a highly elastic collision the
balls speed before and after the collision is practically unchanged. In the frame
of reference of the batsman, however, the ball has been given an increase in speed
by the moving bat.
In 1973, Mariner 10 first used gravity assist (from Venus) to achieve
a flyby of Mercury to gather images and other measurements. The Galileo probe
was launched from a Space Shuttle in 1989, reached Jupiter in 1995 and
studied Jupiters moons until 2003. To reduce the explosion hazard to the
Shuttles astronauts during launch, Galileos fuel requirement was decreased
by using gravity assist, once from Venus and twice from Earth, to slingshot
Galileo to Jupiter. The reduction in Earths orbital velocity was of the order
of only 1018ms1.

spacecrafts
velocity outbound
vout = vin

spacecrafts
trajectory

Jupiter

spacecrafts
velocity
inbound

resultant Vout

Vout > Vin

Worked example

Jupiter

Question
For gravity assist, the maximum possible speed increase occurs for the (unrealistic) extreme
limit at which the spacecraft executes a nearly 180 turn, parallel to the planets orbital
motion (Figure2.4.2).

Jupiters velocity
relative to the Sun
resultant Vin

Figure 2.4.1 Gravitational slingshot in

Show that for this special case, the change in speed of the craft in the Suns frame of
reference is twice the orbital speed of the planet Vp. You can assume that in the Suns
frame, the probes speed is always larger thanVp.

(a) Jupiters frame of


reference and (b) the Suns
frame of reference

Solution
Be careful! Change in speed VoutVin is the change in the magnitude of the velocity, but
it is not the same as the magnitude of change in velocity |VoutVin|.
All labelled velocities in Figure2.4.2 lie in one dimension. Upper case variables denote
quantities in the Suns reference frame, and lower case variables the planets frame.
As usual, bold = vectors, italic = magnitudes and sign = direction.
Vin, Vout and Vp are respectively the spacecrafts incoming and outgoing speeds and
the planets orbital speed in the Suns frame of reference. Because of orbital symmetry, in
the planets frame the spacecrafts incoming and outgoing speeds vin and vout are equal.
Let them both equal v.

vout

vin
b

Vout

Using the Galilean transformation formula vB(rel.toA)=vBvA to convert velocities


in Figure2.4.2a from the planets frame and using the sign convention + :

vin = (v) = Vin Vp = (Vin) (+Vp)

(1)

vout = (+v) = Vout Vp = (+Vout) (+Vp) v = Vout Vp (2)


Equate (1) and (2):
Rearrange:

v = Vin + Vp

v = Vin + Vp = Vout Vp

Vout Vin = V = 2Vp

Vp

Vin

Figure 2.4.2 Hypothetical 180 hyperbolic


orbit in (a) the planets frame
of reference (b) the Suns frame

45

Explaining
and exploring
the solar system

Hitchhikers guide to the solar system

nstead of selling burgers, two university students


took summer holiday jobs at the NASA Jet Propulsion
Laboratory and made the next half century of solar
system exploration possible. In 1961, Mike Minovitch
proved that gravity assist was possible, and in 1965
Gary Flandro showed (just in the nick of time) that in
1976, 1977 and 1978 the planets would, by luck, be
suitably aligned for a space probe to hitchhike across
the solar system using gravity assist from Jupiter and
Saturn to explore the outer planets Jupiter, Saturn,
Uranus and Neptune. This grand tour would not be
possible again until about 2157. Two of the resulting
missions, Voyager 1 and Voyager 2, provided enormous
advances in planetary science, including the nowfamiliar spectacular images of the outer planets and
their moons. Voyager 1 is now at the boundary between
the solar system and interstellar space.

Voyager 1
Saturn
12 Nov 80

Jupiter
9 July 79

Saturn
26 Aug 81

Jupiter
5 Mar 79

Earth 5 Sept 77
20 Aug 77

Pluto
Aug 89

Uranus
27 Jan 86

Voyager 2
Neptune
01 Sept 89

Figure2.4.3 Voyager 1 and Voyager 2 take the grand tour of the solar
system using gravity assist.

Checkpoint 2.4
1
2
3
4

In a gravity-assist manoeuvre, a probe increases its momentum. Explain how momentum is conserved and why
this might not be obvious to an observer.
During a slingshot manoeuvre, what is the shape of the probes orbit in the planets reference frame?
What can we say about the probes speed (far from the planet), before and after the slingshot manoeuvre, when
viewed from the planets frame?
If VP is the orbital speed of the planet, what is the maximum possible speed increase for a probe executing
gravity assist?

2.5 Im back! Re-entry


It is time to come home. You made it through the launch safely so youre feeling
lucky. However, you still have another dangerous hurdle to jumpcoming back
to Earth or re-entry.

Orbital decay
Account for the orbital decay
of satellites in low Earth orbit.

46

Craft in low Earth orbits do experience some drag or air resistance, because the
atmosphere gradually thins out with altitude in a very roughly exponential trend
(Figure2.5.1). At altitudes above ~1000km, drag is considered negligible. Most
Earth-sensing satellites orbit below this altitude, and so have a limited lifetime.
Drag converts orbital kinetic energy into thermal energy, causing the orbital
radius to decrease (orbital decay). The lower the orbit, the greater the air density
(and drag) and so the faster the orbital decay. Sustained orbits are not possible at

space
100

Density (kg m3)

altitudes below ~160km (sub-orbital). Satellites in low


orbits can counteract decay by firing rockets from time
to time, but only until the propellant runs out.
Eventually, a craft in a decaying orbit spirals down
towards the Earths surface, to burn up catastrophically
like a meteor due to the thermal energy produced by
the enormous drag (see in2 Physics @ Preliminary
p57). Drag can be enhanced by solar activity such as
temporary increases in the Suns output of ultraviolet
radiation, which inflates the upper atmosphere,
making the orbital decay rate unpredictable. Normally
a satellite will burn up completely in the atmosphere,
although parts of larger craft (such as the Russian Mir
Space Station or the US Skylab) occasionally make it to
Earths surface.

105

1010

1015

100

200

300

400 500 600


Altitude (km)

700

800

900 1000

Figure2.5.1 Graph of typical air density in the Earths atmosphere


versus altitude

Rest In Pieces

n the late 70s, NASAs first space station, Skylab, was


crippled with low propellant and damaged gyroscopes.
Enhanced solar activity increased its orbital decay and the first
launch of the newly developed Space Shuttle was delayed,
preventing repair missions. So, in July 1979, it fell on Australia.
Pieces too large to burn up on re-entry landed in a line between
Esperance and Rawlinna, Western Australia. Many pieces are in
the Esperance Museum, but the largest (Figure2.5.2) is now in
the United States Space & Rocket Center. The Shire of Esperance
sent the US government a $400 fine for littering. The fine was
finally paid in April 2009 in a Californian radio publicity stunt,
using listeners donations.

of Skylab
Figure2.5.2 The largest surviving fragment

Safe re-entry corridor


If a spacecraft is carrying a human crew, or if the craft needs to be retrieved, then
plunging into a fiery re-entry like a meteorite is not an option. A safer return
from low Earth orbit normally starts by retro-firing rockets to slow the craft
down so it begins to fall into a lower energy, lower altitude orbit. There the
higher air density starts to slow the craft further. At the bottom of LEO, orbital
speed is nearly 7.8kms1. Orbiting spacecraft could not carry enough propellant
to ease down to the surface. The crew has no choice but to head bravely towards
the Earth at the correct angle, using only high technology and clever physics to
protect them.
Safe re-entry is a balance between two forces: drag and lift. Drag is the
deceleration force; lift is the force that keeps an aeroplane in the air. Air moving
relative to the craft creates pressure differences. If pressure underneath is greater
than that above, lift results. The shape and orientation of the craft and the
re-entry angle all affect the ratio of these two forces.

Discuss issues associated with


safe re-entry into the Earths
atmosphere and landing on the
Earths surface.
Identify that there is an
optimum angle for safe re-entry
for a manned spacecraft into
the Earths atmosphere and the
consequences of failing to
achieve this angle.

47

Explaining
and exploring
the solar system

s
in

cie
ffi

nt d

rag

re-entry corridor
sive drag
c
ex es

Figure2.5.3 The safe re-entry corridor

The three main issues behind safe re-entry are:


1 minimising the effects of deceleration (g-force)
2 managing the effects of heating
3 landing the craft safely in the right place.
The first two issues lead to the existence of a narrow range of safe re-entry
angles and speeds (Figure2.5.3). Drag is both good and bad. Drag provides the
spacecraft with brakes, but it also produces the copious amounts of thermal
energy that could destroy the craft. If the approach into the atmosphere is at too
shallow an angle, drag will be too small, the air flow will provide too much lift
and the craft will skip over the atmosphere instead of entering. If the angle is
too steep, drag will be too large, producing excessive heat and deceleration g-force,
which would destroy the craft and crew.

Deceleration
As at launch, astronauts seats during re-entry are oriented perpendicular to, and
facing (eyeballs in) the direction of acceleration, but this time acceleration is
opposite to velocity, so they look backwards.
Traditional re-entry vehicles (such as were used in the 1960s and 1970s) were
teardrop-shaped capsules with the blunt end pointing forward (Figure2.5.4).
They allowed very little or no control once re-entry had begun and provided very
little lift. Such a re-entry is called ballistic re-entry and requires larger re-entry
angles. This kind of capsule subjected the astronauts to a maximum re-entry
g-force of anywhere between 6 and 12. The Apollo re-entry angle was between
5.2 and 7.2
The Space Shuttle introduced in 1981 has wings that provide lift and flightcontrol structures (such as elevons, a rudder/speed brake and a body flap) that
allow considerable control over the descent, adjusting the vehicles aerodynamics
to the changing density of the air, and making re-entry more gentle, with a
maximum g-force of about 23. This degree of control also widens the safe re-entry
corridor, allowing a gentle, low-g 12 re-entry. This is called glide re-entry.
To further decrease descent speed without excessive g-force, the Shuttle performs
a series of Sshaped turns by rolling and banking, gently enhancing drag.
The Russian Soyuz capsule, in use continuously (in modified form) since the
1960s, is a more spherical variation of the traditional capsule shape but with
attitude control thrusters, which provide some glide control during re-entry.
It usually yields a g-force of 45, but sometimes up to about 8 for a completely
ballistic re-entry.
Ballistic re-entries are high acceleration but quickbetween 10 and 15
minutes. A full glide re-entry is low acceleration but slowShuttle re-entry,
for example, takes about 45 minutes. Soyuz is intermediate and takes about
30 minutes.
Heating
On re-entry, vehicles travel at well above the speed of sound. The speed of sound
is sometimes called Mach1 (after Ernst Mach (18381916) a physicist and
philosopher who studied gas dynamics). Twice the speed of sound is called
Mach2 and so on. Supersonic means travelling faster than Mach1. Hypersonic
usually means faster than Mach5 (oversimplifying somewhat).
Pressure builds up in front of projectiles. Sudden pressure changes normally
propagate away as sound (at the speed of sound). In supersonic flight, however,
48

space
this pressure wave is too slow to move out of the projectiles way, so the pressure
builds up to very high levels, forming a shock wavethe air equivalent of the
bow wave in front of a speed boat.
The enormous mechanical energy of orbit must go somewhere. Drag
converts it to thermal energy. Contrary to common sense, in hypersonic flight,
a blunt projectile with more drag actually gets less hot than a more streamlined
one. In the 1950s, Harvey Julian Allen proved this theoretically and explained
why the sharp nose cones of intercontinental ballistic missiles were vaporising on
re-entry. Hypersonic wind tunnel tests (see Figure2.5.5) confirmed his theory.
In hypersonic flight, much of the heat generation takes place in the shock
wave (the dark line wrapping around the front of both projectiles in Figure2.5.5).
The shock wave does not touch the blunt projectile (Figure2.5.5a) and so
doesnt transfer the heat efficiently to it, but it does touch the tip of the sharp
projectile (Figure2.5.5b), which gets much hotter. For this reason, re-entry
vehicles (including the Space Shuttle) are blunt at the front, and hence the
traditional teardrop shape of capsules.
The blunt front of the vehicle is also coated with a suitable heat shield with
very high melting and vaporisation temperatures. It is also highly insulating to
slow the rate of thermal conduction. Thermal insulating materials in most
applications are almost always very porous because tiny pockets of gas are very
poor thermal conductors. Some insulator materials are also designed to be highly
light-absorbing (black) in the visible and near infra-red parts of the spectrum
because such surfaces, when hot, also radiate thermal energy away more
efficiently (radiative cooling).
Tiles on the Shuttle surface are made of 90% porous silica fibre, which is an
excellent high melting point insulator, but it is brittle. The tiles on the hottest
parts (the underside and leading edges) are also coated with a tough black glass
to enhance radiation of thermal energy, but also to provide mechanical strength.
Broken tiles were believed to be responsible for the destruction during re-entry
of the Shuttle Discovery in 2003.
In more traditional space capsule ballistic re-entry, drag is higher, so heat is
generated more rapidly, and insulation and radiation alone are not enough. In
these cases, the insulating heat shield is also designed to vaporise and erode
(ablation). The hot, vaporised and ablated material carries thermal energy away
rather than conduct it to the capsule, similar to the way in which evaporating
sweat carries away excess heat from your skin. The pressure from this ablation
also helps to push away the hot gas convecting from the shock wave. The shield
must be thick enough to last the journey and provide sufficient insulation.
Two modern examples of ablating materials are phenolic impregnated carbon
ablator (PICA) and silicone impregnated reusable ceramic ablator (SIRCA). In
the Chinese space program, one of the ablation materials used is blocks of oak
wood. Its cheap and easy to work. As it chars, it forms charcoal, which is porous
and almost pure carbon, making it an extraordinarily good thermal insulator
with a very high melting point. Another advantage is that porous carbon is very
black and radiates thermal energy efficiently. However, it is mechanically weaker
than more high-tech ablation materials.
During re-entry, superheated air surrounding the vehicle is ionised. The air
becomes a plasmaa conductive soup of free positive and negative charges that,
like the Earths ionosphere (see in2 Physics @ Preliminary pp1534), reflects

Figure 2.5.4 Re-entry vehicles: (a)Gemini


19641966 (b)Apollo 19661975
(c)Soyuz 1960present
(d)Space Shuttle 1981present
49

Explaining
and exploring
the solar system
a

Figure2.5.5 Hypersonic wind tunnel tests. (a) The crescent-shaped shock wave is detached from
the blunt projectile, but (b) touches the tip of the sharp projectile.

radio waves, so the astronauts cannot communicate with the Earth for several
minutes during re-entry. This problem has been solved for the Shuttle by
communicating via a satellite above it, since only the bottom of the Shuttle has
significant ionisation.

Landing
Drag depends on the projectiles cross-sectional area and speed. Drag cannot stop
a projectile completely because, during deceleration, drag decreases until it
exactly cancels the weight of the projectile and deceleration stopsthe projectile
has reached terminal speed (see in2 Physics @ Preliminary p45). The terminal
speed of a capsule is too high for it to land safely. To slow the capsule further for
the landing, drag is enhanced (and terminal speed decreased) by using parachutes
to increase the effective area of the capsule.
The final touchdown could be on land (typical of Russian missions) or a
splashdown in the water (typical of US missions pre-Shuttle). Russian Soyuz
also has soft-landing engines that fire just before it touches the ground.
The Space Shuttle lands on a runway, much like an aeroplane (Figure2.5.6)
but it uses parachutes to help it brake. During landing, the Shuttle (which has
been described as being like flying a brick with wings) is controlled entirely by
computer.
Another issue is accurate targeting of the landing site. The steeper the
re-entry angle, the smaller the horizontal component of motion (range) and so
the more accurate the prediction of the final landing site. However, the Shuttle
makes up for its shallow re-entry, because its aeroplane-like flight-control
structures allow adjustment of the landing path. The shape of the landing path is
also designed to be more forgiving. The Shuttle approaches the runway roughly
opposite to the landing direction. Fourminutes from touchdown, it does a
heading-alignment loop, to adjust precisely to the direction of the runway
(Figure 2.5.6).

50

space

Altitude 25 000 m

Mojave
Runw
ay 23
E
d
Airfor wards
ce Ba
se

Figure 2.5.6 Scale drawing of the relatively gentle descent of the Space Shuttle. The Shuttle is
drawn at 1minute intervals to touchdown. The squares on the ground are 10 nautical
miles (18.5km) wide.

Checkpoint 2.5
1
2
3
4
5
6
7
8
9
10
11

Define orbital decay and explain what causes it.


Because of drag, satellites at altitudes below ~1000km can do nothing to combat orbital decay. True or False?
Explain.
What other astronomical body can affect the rate of orbital decay? Explain.
Discuss how drag is good and bad for re-entry.
Outline what can happen if a spacecraft attempts re-entry with too shallow or too steep an angle.
Explain why astronauts face backwards during re-entry, unlike at launch.
Outline why occupants of the Space Shuttle experience lower g-force during re-entry than in the more traditional
re-entry vehicles.
Define the terms supersonic and hypersonic.
What is a shock wave?
Outline why a pointy hypersonic projectile is more likely to melt than a blunt one.
Explain why a capsule with a parachute slows down more than without one.

51

Explaining
and exploring
the solar system

PRACTICAL EXPERIENCES
CHAPTER 2

This is a starting point to get you thinking about the mandatory practical
experiences outlined in the syllabus. For detailed instructions and advice, use
in2 Physics @ HSC Activity Manual.

Activity 2.1: Development of space exploration


Identify data sources, gather,
analyse and present
information on the contribution
of one of the following to the
development of space
exploration: Tsiolkovsky,
Oberth, Goddard, EsnaultPelterie, ONeill or von Braun.

Use the template provided in the activity manual to extract information about
your chosen scientist. Process this information to make a short oral presentation
to the class.
Discussion questions
1 For the scientist that you have researched, list their main contributions
to space exploration.
2 Explain how later scientists have benefited from this research.
Extension
3 Werner von Brauns great Russian rival, the Chief Designer for the USSR
space program Sergey Korolyov, is not as familiar as some of the names
mentioned in the syllabus, despite leading the launch of the first artificial
satellite, Sputnik, in 1957. This is probably because his name was kept
secret by the communist government of the USSR until after his death in
1966. You may also want to research his contribution to space exploration.

Activity 2.2: Uniform circular motion


Solve problems and analyse
information to calculate the
centripetal force acting on
a satellite undergoing uniform
circular motion about the
Earth using:

F =m

v2
r

Perform an experiment that will allow you to determine the relationship between
the radius of a satellites orbit around the Earth and its gravitational force.
Equipment: string, rubber stopper, mass carrier and masses, electronic scales,
glass or plastic tube, paperclip, sticky tape, metre ruler, stopwatch.
Discussion questions
1 From your experimental data,
determine the mathematical
relationship between the
orbital radius of a satellite and
its tangential velocity for a
given centripetal force.
2 Describe the method you
would use to determine the
centripetal force on a small
model satellite.

tension
glass or
plastic tube
paperclip

string

mass carrier

Figure 2.6.1 Force on mass moving in a


horizontal circle

52

mg

Chapter summary







Rocket thrust is the reaction to the force exerted on the


exhaust exiting the nozzle.
For a fixed mass of exhaust per unit time, thrust
increases as exhaust speed increases.
As the mass of remaining propellant decreases, a rockets
acceleration (and g-force) increases.
There are two kinds of rocket engines: liquid and solid
propelled.
By jettisoning used stages, a rockets mass is decreased,
allowing more payload to be carried.
g-force is the apparent weight experienced during
acceleration, divided by true weight on Earth.
a
a
Vertical g-force = v + 1. Horizontal g-force = h .
g
g
To increase g-force tolerance, astronauts are seated with
bodies horizontal, and looking in the direction of the
acceleration (eyeballs in) for both lift-off and re-entry.
A spacecraft launched eastwards has the extra initial
velocity of Earths rotation. This is greatest at the
equator (465ms1).
Launches in the direction of Earths orbital velocity
obtain an initial velocity boost of 3.0104ms1.
Gravity provides the centripetal force for satellite orbits:

mv 2
(for circular orbits)
R
Keplers laws (apply to any two bodies if the central
body has a very much larger mass):
1 Orbits of planets are ellipses, with the Sun at one
focus.
2 Planets sweep out equal areas in equal times.
T2
3 Law of periods:
= a constant
a3
a 3 GM
Explicit form of Keplers third law:
=
T 2 42
Eccentricity is a measure of the elongation of an ellipse.
A circle is an ellipse of zero eccentricity.
Periapsis is the position of closest approach to the
central mass and fastest orbital speed (perihelion for the
Sun, perigee for Earth).
Apoapsis is the position of furthest distance from the
central mass and slowest orbital speed (aphelion for
the Sun, apogee for Earth).
If the mass of a satellite is not negligible when compared
to that of the central body, then both masses orbit with
the same period around the systems centre of mass.

Fc =

space

GM
r
Two-body orbital mechanical energy
1
GmM :
ME = mv 2
2
r
Magnitude of orbital speed: v =

ME<0 (bound), orbit is closed or stable (circle


or ellipse); velocity < escape velocity
ME=0 (borderline unbound) orbit is parabolic;
velocity = escape velocity
ME>0 (unbound), orbit is hyperbolic;
velocity > escape velocity
GmM
.
For stable orbits, ME =
2a
Orbits are symmetrical in shape and speed.
Low Earth orbit (LEO): altitude between ~160 and
~2000km.
At altitudes below ~1000km, drag causes orbital decay.
Upper atmosphere can be inflated by increased solar UV
radiation, increasing drag.
Orbits below ~1000km are protected from the Van
Allen radiation belts by the atmosphere and distance.
Geosynchronous orbit: T = 1 sidereal day
(23h 56m 4s).
Circular geosynchronous orbits over the equator are
called geostationary. These orbits are used extensively
for communication satellites (r=42164km).
A space probe entering a temporary hyperbolic orbit
behind an orbiting planet can gain momentum via
gravity assist or the slingshot effect.
For gravity assist, the maximum possible change in
speed of the probe in the Suns frame of reference is
twice the planets orbital speed Vp.
Drag converts orbital KE into thermal energy.
Safe re-entry angle: if angle is too low, the craft will skip
off atmosphere; if angle is too high, g-force and heating
rate are too large.
Much heating takes place in the hypersonic shock wave.
Blunt-fronted re-entry vehicles are used because the
shock wave is detached from the craft.
Heating of a spacecraft on re-entry is reduced by an
insulating and radiating heat shield.
Traditional capsules also use ablation of the heat shield
to dissipate heat.
Parachutes decrease terminal velocity by increasing the
effective cross-sectional area.

53

Review questions

Explaining
and exploring
the solar system

Physically speaking
For each type of orbit, fill in the missing information. One has been done already.

Name of orbit

Sign of two-body ME

Open or closed

v >, =, < vescape

Bound or unbound

Negative

Closed

<

Bound

Geostationary
Slingshot (in planets frame)
Elliptical
Hohmann (see Physics Focus)
Parabolic
Hyperbolic
Halleys Comet
Circular
Molniya

Reviewing
Solve problems and analyse
information to calculate the
centripetal force acting on a
satellite undergoing uniform
circular motion about the
Earth using:
2

mv
r
Solve problems and analyse
information using:
mm
F =G 1 2
d2
Analyse the forces involved
in uniform circular motion
for a range of objects,
including satellites orbiting
the Earth.
F =

1 What was the first solid rocket propellant and who invented it?
2 Assuming that propellant is burned at a constant mass per unit time, use the
equation for thrust to explain why forcing exhaust gas through a narrow nozzle
increases thrust.

3 List the advantages of both solid and liquid propellants.


4 Discuss why the vertical g-force formula has a + 1 term, but the horizontal
formula doesnt.

5 Describe a situation during launch in which astronauts would experience a g-force


greater than zero but less than 1.

6 At the bottom of a bungee jump with the cord attached to the ankles, one can
easily experience a g-force of 3, the maximum normally allowed for Shuttle
launches. Describe three important differences in the way the g-force is
experienced in these two situations.

7 Explain why launch facilities are usually built as close to the equator as
is practical.

8 Describe the circumstances under which a star would not sit at the focus of
a planets elliptical orbit.

9 Discuss why we only briefly see Halleys Comet with an unaided eye every 76 years,
even though it is in orbit continuously around the Sun.

10 Outline what happens to the period of a satellite if its semimajor axis is reduced
by a factor of 4.

11 A space probe approaches a planet in a hyperbolic orbit. Discuss the condition


that must be fulfilled to move it into a stable orbit of the planet and describe how
it might be achieved.

12 By re-examining the gravity assist worked example on page 45, show that the
magnitude of change in the probes velocity in the Suns reference frame is twice
the probes initial speed in the planets reference frame (that is, |VoutVin|=2v).

13 A satellite is in a highly elliptical orbit around Earth such that, at perigee, it is


briefly at an altitude of less than 1000km. Over many orbits, the altitude at
apogee decreases (the orbit becoming more circular). Explain why this occurs.

14 List two reasons why (human-crewed) space stations are always in low Earth orbit.
54

space

Solving Problems
15 Calculate the two-body gravitational potential energy for a system consisting

Analyse the forces involved in


uniform circular motion for a
range of objects, including
satellites orbiting the Earth.

of a 1.00kg test mass sitting on the surface of the Earth. How far would
the test mass need to be from the Sun so that the two-body GPE of the
test massSun system is the same value? Estimate roughly where that
position would be in relation to the orbital radii of the planets.

16 Typically, at launch, the Shuttles main engines, with an effective exhaust


velocity of 4460ms1, produce a thrust of 5.45106N. The two solidfuel rocket boosters, with an effective exhaust velocity of 2640ms1,
produce 1.250107N each.
a Calculate the combined rate (in kgs1) at which propellant is used
at launch.
b Assuming a mass at launch of 2.03106kg, calculate the Shuttles
acceleration at launch and 1minute later, assuming the above
specifications remain constant. (Hint: Dont forget gravity.)

17 On a roller-coaster, you round the top of a circular hump in the track of

5.00m radius. You have a g-force meter with you and at the moment
youre at the top it reads a vertical g-force of 0.00.
a What is your weight at that moment?
b What is the magnitude of the normal force exerted on you by the seat
at that moment?
c What is your centripetal acceleration?
d Calculate your speed at the top.
e Assuming friction and air resistance are negligible, calculate your
horizontal g-force at that moment.

18 Prunella spins a weight (mass m) on a string (length L) in a horizontal

Solve problems and analyse


information to calculate the
centripetal force acting on a
satellite undergoing uniform
circular motion about the Earth
using:

circle (Figure2.6.2) to illustrate the relationship between orbital speed


and centripetal force for an orbiting satellite. Renfrew says: Because of
the weight, the string isnt horizontal so the orbital radius is R=Lsin,
and the centripetal force is Fc=T sin.

Prunella then says: Yeah, but as long as the orbital speed v is high

enough, will be very close to 90 so you can use the approximation that
string tension T is the centripetal force and the string length L represents
orbital radius R.
v2
Show that as long as orbital speed v fulfils the condition
>7g,
R
then L is no more than 1% larger than the true orbital
radius R and T is no more than 1% larger than the true
centripetal force Fc.

Solve problems and analyse


information using:
mm
F =G 1 2
d2

mv 2
r
Analyse the forces involved in
uniform circular motion for a
range of objects, including
satellites orbiting the Earth.
F =


T sin

mg

Figure 2.6.2 Spinning weight model


of a satellite

55

Explaining
and exploring
the solar system

19 In Gerard ONeills proposed colonies in space, people would live on the


inner surfaces of rotating cylinders 3km in radius and 20km long.
Gravity would be simulated via the reaction force to the centripetal force
exerted by the cylinder on the occupants inside. Calculate the rotation rate
(in revolutions per hour) that would yield artificial gravity, mimicking the
magnitude of gravity at the Earths surface.
Solve problems and analyse
information using:
r 3 GM
=
T 2 4 2

Define the term orbital velocity


and the quantitative and
qualitative relationship
between orbital velocity, the
gravitational constant, mass
of the central body, mass of
the satellite and the radius of
the orbit using Keplers Law
of Periods.

20 Using the mass of Jupiter calculated in the worked example on page 39,
21

predict the period of Jupiters moon Callisto, given that its semimajor axis
is 1.89106km.
2r
and the explicit form
Using the equation for tangential velocity v =
T
of Keplers law of periods, re-derive the expression from section 2.2, for
the magnitude of orbital velocity of a satellite in circular orbit:
v=

GM

r

22 Using the explicit form of Keplers third law, calculate the radii of:
a a geostationary orbit
b a circular semi-synchronous orbit.

23 Draw a table in which you compare the kinetic energy, the two-body

potential energy and the two-body mechanical energy of a geosynchronous


and a circular semi-synchronous satellite of mass 2000kg. Use the data
from Question22 to confirm that the two-body mechanical energy for both
orbits is:
GmM
ME =
2a

24 A comet passes the Sun (MS = 1.991030kg). At perihelion, the

centre-to-centre distance is 8.551010m and the speed of the comet


is 6.69106ms1. What kind of orbit is it? Do you expect the comet
to return?

25 Gravity assist can also be used as a brake. Show that if the diagram in the
planets frame in Figure2.4.1a is unchanged, but the planets orbital
velocity in Figure2.4.1b is reversed, then the probes speed in the Suns
frame would decrease.

26 A Soyuz capsule with a crew of three (7460kg) is in a circular orbit at

Re

56

iew

Q uesti o

an altitude of 336km, having completed a mission to the International


Space Station.
a Calculate its kinetic energy.
b Ignoring the effect of its soft-landing engines, calculate how much
thermal energy is generated by drag during its re-entry.
Earths mass ME = 5.971024kg
Earths radius at the landing site rE=6366km

space

PHYSICS FOCUS

final orbit

L Driving lesson in orbit


initial
orbit

3. Applications and uses of physics


satellite

Once in a stable orbit, a spacecraft is coasting. One


only needs to do work (burn fuel) to swap between
orbits. The following are two important examples of
manoeuvres between orbits.
Changing lanes: Circular orbits around Earth (or other
body) are like lanes on a highway, the smaller radius
orbits being the fast lanes. In 1925, Walter Hohmann
devised a fuel-efficient method for changing lanes. To
move from a small orbit to a larger one, an accelerating
engine burn of just the right impulse will convert the
circular orbit to an elliptical Hohmann transfer orbit
(Figure 2.6.3), which joins the initial and final orbits.
Once in the elliptical orbit, the engine is shut off again.
Once the craft reaches the apoapsis of its elliptical
orbit, a second accelerating burn of the right impulse
will convert the elliptical orbit into a (now larger radius)
circular orbit. This method can also be used in reverse
to drop down to a smaller orbit, in which case the two
engine burns must be decelerations.
Hohmann transfer orbits around the Sun can also
be used for travel between planets. The launch must be
timed so the planet is at the periapsis of the Hohmann
orbit just as the probe arrives.
Overtaking: Sometimes spacecraft need to
rendezvous (meet) in orbit, such as when the Space
Shuttle needs to dock with the International Space
Station. If youre way behind a craft in the same orbit
and you need to catch up, you should paradoxically
slam on the brakesbriefly retrofire (fire your engines
in reverse) to reduce speed (Figure 2.6.4). This
temporarily reduces KE (and total mechanical energy)
so you drop down into a lower energy, smaller radius
overtaking orbit, more than regaining your speed.
Smaller radius orbits are faster, so you eventually catch
up with the other craft. When youre about to overtake
it in the fast lane, fire your engines forwards to
increase mechanical energy, so you pop back up into
the original orbit rendezvous.

elliptical
transfer orbit

Figure 2.6.3
A Hohmann elliptical
transfer orbit

Figure 2.6.4 To overtake, first slow down.


1 When you fire a rocket to change your speed,
does this violate momentum conservation? Explain.
2 Why doesnt one just fly from the initial orbit to the
final orbit in a straight line instead of following a
Hohmann orbit? Does this violate Newtons first law?
Explain.
3 Show that to transfer from an initial circular orbit
of radius Ri to a larger final one of Rf, you must use
a Hohmann transfer orbit with a semimajor axis a,
where Ri + Rf = 2a. (Hint: Draw some diagrams.)
4 Why cant the Hohmann transfer orbit be a circular
orbit? (Hint: Draw another diagram.)
5 Why is the Shuttle in Figure2.6.4 pointing the
wrong way?
6 When overtaking, you temporarily reduce your KE.
Why does your KE increase again in the overtaking
orbit? You may use the fact that for a stable orbit,
GmM
the two-body ME=
(section 2.3).
ME =
2a

Extension
7 A Space Shuttle releases a communications satellite
into an initial orbit of radius r=6.66106m.
Using a Hohmann transfer orbit, the satellite
moves up to a geosynchronous orbit of
rgeo=4.216107m. Using the result of
Question 3, calculate how long the transfer takes.
(Hint: The transfer path is half an orbit.)

57

3
special relativity, general relativity,
inertial frame of reference, invariant,
principle of relativity, fictitious forces,
Maxwells equations, electromagnetic
wave, mechanical medium,
luminiferous aether, interferometry,
beamsplitter, aether drag, Michelson
Morley experiment, null result,
postulate, simultaneity, Lorentz factor,
time dilation, proper time, twin paradox,
proper length, length contraction, rest
mass, proper mass, relativistic mass,
spacetime, mass-energy,

Seeing in a weird
light: relativity
Just some minor problems
You may have heard it said that some physicists think that a theory of
everything is just around the corner. This attitude is not new. Many
physicists thought this about what is now called classical Newtonian
physics, towards the end of the late 19th century. There were just a few
minor problems with understanding the way light travels through space
that needed to be fixed, and then the job of physics would be finished.
Well, those minor problems with light led to the twin pillars of
modern physics: quantum mechanics and relativity. Einstein was a key
player in both, especially relativity, which comes in two parts. Special
relativity replaced Newtons mechanics and the later general relativity
replaced Newtons universal gravitation. A century later, relativity still
defies common sense, bending space, time and the mind, but it has
not yet failed any experimental test. In this chapter we will only deal
with the theory of special relativity.
Common sense is nothing more than a deposit
of prejudices laid down by the mind before
you reach eighteen.
A Einstein

3.1 Frames of reference and


classical relativity
Outline the nature of inertial
frames of reference.

Discuss the principle of


relativity.

58

Before we talk about relativistic weirdness, recall inertial frames of reference


(see in2 Physics @ Preliminary section 3.3). Inertial frames of reference are
required for observers using Newtons laws or the Galilean transformation
formula for relative velocity (see section 1.1). An inertial frame is any nonaccelerating (including non-rotating) reference frame. Inertial frames can move
at a constant velocity relative to each other.
There are no absolute velocities and there is
All velocities are relative.
no special absolutely stationary inertial frame. The classical Newtonian laws of
mechanics and gravity are unchanged (or invariant) when transforming from
one inertial frame to another (even though the values of some measurements
such as velocity might change). This is called the principle of relativity.
To transform a situation from one inertial frame to another, you simply apply
the Galilean transformation formula to all the velocities. Youve already seen an
example of this in the gravity assist example (Figure2.4.2).

space

nte

ractiv

Because there is no special inertial frame, no experiment purely within


your own frame can detect the velocity of your frame, so absolute velocity is
meaningless. You can only compare your frames velocity relative to others. An
example of this is waiting to depart in a train, looking out the window (Figure
3.1.1) to see that a train next to you is moving slowly away, only to find a few
seconds later that, in fact, relative to the station it is your train that is moving.
Your acceleration (including vibrations) was negligibleyou felt no effect of your
uniform velocity.
However, you can feel the acceleration of a non-inertial reference frame,
and measure it using an accelerometer. The simplest accelerometer is a pendulum.
If a pendulum hangs vertically in a car, your horizontal acceleration is zero. If you
are accelerating horizontally, the pendulum will hang obliquely (Figure 3.1.2).
If you are observing from within a non-inertial (accelerating) frame, Newtons
laws appear to be violated. Objects can appear to change velocity without a true
net external force; in other words, you experience fictitious forces or pseudoforces (see in2 Physics @ Preliminary p 39). For example, in a car taking a corner,
you experience the sensation of being thrown outwards by a fictitious centrifugal
force. If viewed from the inertial frame of the footpath, you evidently are pulled
inwards by a true centripetal force. (Weve cheated a bit. The Earth is turning,
so the footpath is not strictly an inertial frame. However, Earths radius is so
large that in most human-scale situations, fictitious forces due to Earths rotation
are negligible.)
Another view of tests for non-inertial reference frames is that they involve
detecting fictitious forces. Its a two-step process. First, analyse an object within
that frame of reference and decide what true external Newtonian forces must act
on the object. Then, look for apparently extra or missing forcesevidence of
a non-inertial frame. For example, judged from the inertial frame of the ground,
the downward weight mg and the upward normal force N of the seat are the
only true forces on an astronaut during launch. Within the accelerating rocket, the
sensation of enhanced weight (downwards) associated with g-force has the same
magnitude as N but is apparently in the wrong direction and is therefore fictitious.
A pendulum accelerometer hangs obliquely within an accelerating car as
though there is a fictitious horizontal component of weight. In free-fall (or orbit),
the apparent absence of weight is also fictitious. Your frame accelerates downwards,
so true weight becomes undetectable to you, as though your true downward
weight is cancelled by a fictitious upward gravity. The effects of neither force
show up separately on an accelerometer.

M o d u le

Figure 3.1.1 Who is really moving?

Try this!
Fictitious fun
While sitting on a playground
merry-go-round with a friend,
try playing catch with a slow
moving tennis ball. The fictitious
centrifugal and Coriolis forces
will cause the ball to appear to
follow warped trajectories,
making it difficult to catch.

Worked example

mg

question
mg tan 5

A Christmas decoration is hanging obliquely inside your car, 5 from vertical and pointing
towards the cars left side. Describe quantitatively the cars motion (no skidding!).

Solution
Only two true external forces act on the decoration: tension and weight (Figure3.1.2).
Because there is an angle between them, they arent equal and opposite, so the decoration
experiences a net real force and acceleration sideways (in this case centripetal). The net
force and acceleration point towards the right side of the car, so the accelerometer
(and the car) is steering towards the right.

left

right

mg

Figure 3.1.2 Festive season pendulum


accelerometer
59

Seeing in a
weird light:
relativity
The centrifugal force perceived by the occupants of the car to be pulling the decoration
toward the left side of the car is fictitious.
From Figure3.1.2, the magnitude of the centripetal acceleration is:
F
ac = c = g tan5=9.800.0875=0.86ms2
m

Note: This test is subjectiveit requires personal judgement (hence possible


bias). No measurement alone can identify a force as fictitious. For example, no
pure measurement can tell the difference between true weightlessness and the
fictitious weightlessness of free-fall. You can only tell the difference by looking
Activity Manual, Page
21
down and seeing the Earth below; judgement says there should be gravity, but
you cant feel it. The inability of measurement alone to distinguish the effects of
true gravity from the effects of g-force is what Einstein used as the starting point
Perform an investigation to
for his re-writing of the law of gravitation in his theory of general relativity, but
help distinguish between
youll have to wait until university physics to learn about that!
non-inertial and inertial
This approach to distinguishing between inertial and non-inertial frames
frames of reference.
relies on a classical concept of force. In Einsteins relativity, the concept of force
is more complicated and is used much less.
The term fictitious force doesnt
mean the observed effects are
imaginary, as the victims of a cyclone
Fictitious cyclone?
or astronauts who are subject to
Yeah right!
g-force can attest. It simply means
ffects associated with
that the apparent force doesnt fit
so-called fictitious
Newtons definition of a true force.
forces of Earths rotation
It is always possible to
are not always negligible.
re-analyse fictitious forces using an
The Coriolis force is a
inertial frame and to account for
fictitious tangential
all observed effects using only true
force appearing in
Newtonian forces.

PRACTICAL
EXPERIENCES
Activity 3.1

rotating frames of
reference and is
associated with the
formation of cyclones.

Figure 3.1.3 Satellite photo


of a cyclone

Checkpoint 3.1
1
2
3
4
5
6
7

60

Define an inertial reference frame.


Recall the Galilean transformation formula for relative velocities.
Outline why we usually treat the Earth as an inertial frame, given that it is rotating.
Discuss whether or not centripetal force is fictitious.
In free-fall, you dont experience any extra apparent forces. Are you in an inertial frame? Explain.
What apparatus would distinguish true weight from apparent weight due to g-force ?
The values of some measurements such as velocity might change, but the laws of mechanics are the same in all
frames of reference. True or False? Explain.

space

3.2 Light in the Victorian era


The 19th century was a period of enormous advance in the study of electricity
and magnetism. Faraday, Ampere, Oersted, Ohm and others, through theory and
experiment, produced a large collection of equations and phenomena. There
were hints of connections between electricity and magnetisman electrical
current can produce a magnetic field (see in2 Physics @ Preliminary section 12.3)
and a changing magnetic field can induce a changing electric field or current
(section 4.1).
The Scottish theoretical physicist, James Clerk Maxwell (18311879)
collected the existing equations to reduce them down to the minimum number.
He reduced them down to eight equations (which expanded to 20 when he
included all the x-, y- and z-components). A self-taught electrical engineer called
Oliver Heaviside (18501925), using the newly developed mathematics of
vectors, reduced Maxwells equations to four. We now call those four equations
Maxwells equations.
It puzzled Maxwell that his equations were almost symmetrical in their
treatment of electrical and magnetic fieldsalmost but not quite. So he added
a term to his equations, assuming that a changing electrical field can induce a
magnetic field (not previously observed). When he did this, he showed that an
oscillating magnetic field would induce an oscillating electric field and vice versa,
resulting in a self-sustaining electromagnetic wave. From his equations he
calculated the speed of that wave to be equal to the speed of light in a vacuum
(which is now called c and equals 2.998792458108ms1).
It was either an astonishing coincidence or strong circumstantial evidence
that light is an electromagnetic wave (see in2 Physics @ Preliminary p 84).
Heinrich Hertz (18571894) experimentally confirmed the predicted speed and
properties of these electromagnetic waves.

Figure3.2.1 James Clerk Maxwell

What is lights medium?


Until then, every existing kind of wave needed a mechanical medium; for
example, sound propagates through air, earthquakes through rock, musical
vibrations along a violin string, ripples along water and so on (see in2 Physics
@ Preliminary section5.3). To sustain a wave, a medium needs two properties:
resilience (or stiffness) and inertia (any density- or mass-related property).
The higher the stiffness and the lower the inertia, the higher the wave speed.
It was assumed that light also needs a medium, which was called
luminiferous aether or just aether (US spelling: ether). Luminiferous means
light-bearing, and aether was the air breathed by the gods of Greek mythology.
So Maxwell developed a model for aether, assigning it bizarre mechanical
properties consistent with the behaviour and enormous speed of light. It needed
to be far less dense than air but much stiffer than any known material. Despite
its stiffness, aether was assumed to penetrate all materials effortlessly. Conversely,
it needed to be able to be penetrated without resistance by all objects that move
freely through space, including Earth hurtling around the Sun.
If you shout with the wind blowing behind you, then, relative to you, the
velocity of sound would be higher than if the air were still. This is because the
velocity of sound (and other mechanical waves) is the sum of its velocity relative
to the medium and the velocity of the medium itself. In other words, mechanical

Outline the features of


the aether model for the
transmission of light.

61

Seeing in a
weird light:
relativity
waves seem to obey the Galilean transformation. It was assumed that light should
also obey it, so the speed of light should be affected by the motion of the aether.
However, Maxwells equations appeared to allow only one particular value for
The Galilean transformation and Newtons
the speed of light in a vacuum.
laws imply it is impossible for the speed of light to appear to be the same to all
observers with different relative speeds. Perhaps the speed specified by Maxwells
equations is the speed relative to the aether only. However, this meant that the
aether represented a preferred reference frame for Maxwells equations, which was
inconsistent with the classical principle of relativity.

M and M
Describe and evaluate the
MichelsonMorley attempt to
measure the relative velocity of
the Earth through the aether.
Discuss the role of the
MichelsonMorley experiments
in making determinations about
competing theories.

Figure 3.2.2 Interference pattern in a


Michelson interferometer
illuminated by a mercury
vapour lamp. Patterns of
different shapes (such as
vertical bands) are possible
and depend on exactly how the
interferometer is aligned.

PRACTICAL
EXPERIENCES
Activity 3.2

Activity Manual, Page


25

Gather and process information


to interpret the results of the
MichelsonMorley experiment.

62

Given that the Earth was supposed to be hurtling around the Sun, through the
aether at 3104ms1, the resulting aether wind (or aether drift) relative to
Earth should affect measurements of light speed differently according to the time
of day and time of year as the Earth rotated and orbited the Sun, changing its
orientation relative to the aether.
So in the 1880s, the experimentalist Albert Michelson (18521931), joined
later by Edward Morley (18381923), attempted to measure changes in the speed
of light throughout the day due to this shifting aether wind. They used
a very sensitive method called interferometry (see section21.5), which
Michelson had used some years earlier to accurately measure the speed of light.
Recall constructive and destructive interference (see in2 Physics @ Preliminary
p102 and p126). If two light beams are projected onto a screen, then a bright
fringe occurs at places where the two beams are in phase (constructive
interference). Where they are out of phase, destructive interference results in a
dark fringe. The pattern of bright and dark fringes is called an interference
pattern (Figure 3.2.2).
Interference turns a pair of monochromatic (single wavelength) light
beams into an extremely sensitive ruler for which the interference fringes are like
magnified ruler markings one light wavelength apart. For visible light, this
spacing is less than 8107m and corresponds to time intervals of less than
31015s. If the two light beams travel via different paths, then a very small
change in the length of one path will change the relative phase, resulting in
a detectable change in the position of fringes in the interference pattern.
A change in wave speed along one of those paths should have a similar effect
on phase.
Michelson and Morley set up an interferometer in which the light was divided
into two perpendicular beams or arms by passing it through a half-silvered
mirror or beamsplitter (Figure3.2.3). The apparatus was built on a heavy stone
optical bench floating in mercury, to allow rotation and damp out vibrations.
They assumed that if one interferometer arm was pointing parallel to the aether
wind, the speed of light should be slightly different in the two arms. The time of
flight of the light in the arm parallel to the aether wind should be slightly longer
than that of light along the perpendicular arm. As the Earth (or the apparatus)
rotates, this speed difference, as measured by the positions of the interference
fringes (Figure 3.2.2), should change with the angle.
Figure 3.2.4 summarises the classically predicted effect of aether wind on the
resultant light speed in the two arms of the interferometer. Lets calculate the
expected time difference. Suppose the total distance from beamsplitter MS to M1
(or M2) is L, then the round-trip for each arm is 2L.

space
a

M1
source

Ms

aether
wind

l1
l2
M2

M2

Ms

eyepiece

source

Figure3.2.3 The MichelsonMorley interferometer


drawn as (a) a simplified schematic
and (b) an actual ray diagram.
Multiple reflections were used to
make the effective length of the arms
very long hence more sensitive to
changes in light speed. MS is the
half-silvered beamsplitter mirror.

relative to the aether, then the resultant light speed is c 2 v 2 (Figure3.2.4a)


and the time taken for light to do a round-trip is:
2L
c2 v2

2L

1
1

v2
c2

In the arm parallel to the aether wind (speed v), for half the trip against the
wind, the speed of light would be cv, and for the other half with the wind it
would be c+v, so the time taken would be longer:
L
L
2L
t2 =
+
=

c v c +v
c

1
1

v2

c2
(Check that you agree that since v is smaller than c, time in the parallel arm t2
is longer than time in perpendicular arm t1.)
Other factors such as thermal expansion or contraction of the apparatus
could cause apparent drift in the interference pattern, but the shift due to
rotation of the apparatus (or the Earth beneath it) would be a sine wave with
a period equal to the rotation period of the apparatus, so any drift not due to
rotation could be detected and subtracted. Michelson and Morley graphed the
position of interference fringes versus rotation angle at different times of the
day, but concluded that the small observed shifts could be explained as drift in
the experimental apparatus. Over several years, scientists repeated the
measurements, with some reports of possible changes in interference over the
day; but eventually the consensus was that any observed effect was well below
what was expected by the aether theory and could be explained by drift in
the apparatus.
George Fitzgerald and Hendrik Lorentz attempted to squeeze the Galilean
transformation into Maxwells equations, concluding that charged particles
(such as charges in atoms) moving through the aether with speed of v must

C 2 V 2
V

C
V

aether wind

t1 =

aether wind

In the arm perpendicular to the aether wind (speed v), if c is light speed

M1

C V

C+ V

Figure3.2.4 Classical effect of aether wind on light


velocity. (a) The resultant velocity
perpendicular to the wind and (b)
resultant velocity parallel to the wind.
Blue = light velocity relative to aether,
green = aether velocity and red =
resultant light velocity

shrink in the direction of motion by a factor of 1 v 2/c 2. The interferometer


arm parallel to the aether wind would shrink just enough to compensate for
the change in light speed and hence cancel the expected change in the
interference pattern.
63

Seeing in a
weird light:
relativity
Another reason suggested for failure to see the shift was that perhaps aether
only penetrates transparent objects, so aether was trapped by large opaque
mountains and valleys or buildings and dragged along by the moving Earth,
similar to the way in which air is trapped in the fur of a running dog. A flea
conducting scientific experiments on the dogs skin would be unaware that outside
the fur, air is whooshing backwards relative to the dog. This idea was called aether
drag. If this were true, then at the tops of mountains, closer to outer space, the
aether wind might be detectable. Some experimenters repeated the experiment on
mountains without success (apart from a controversial partial result).
Failure to detect undeniable effects of aether wind caused some physicists to
question if it even existed. Maxwells equations only mention electric and
magnetic fields. The aether is not required by the equations. Einstein assumed it
didnt exist, but said that relativity was not an attempt to explain Michelson and
Morleys negative result, but rather, he was motivated by the properties of
Maxwells equations. However, in physics, when experiment and theory say the
Today almost all physicists
same thing, youre probably on the right track.
agree that there is no aether.
The MichelsonMorley experiment is often called the most famous failed
experiment. It was not exactly a failure. In 1907, Michelson was awarded the
The result of an experiment that fails
Nobel Prize for Physics for his work.
to find evidence of an expected effect despite careful design and execution is more
correctly called a null result. This null result was one of the most important in
the history of physics, because it helped bring about a whole new way of seeing
the universe.

Checkpoint 3.2
1
2
3
4
5
6
7
8

Describe Maxwells circumstantial evidence that light is an electromagnetic wave.


Discuss why it was assumed that light required a medium or aether to propagate in.
Maxwells equations predicted that the speed of light should depend on the speed of the medium, but this was
contradicted by the MichelsonMorley experiment. True or false? Explain.
In the classical analysis of the MichelsonMorley interferometer, which arm required the longer time of flight?
Is it correct to say that the MichelsonMorley experiment didnt show any change in the interference pattern? Explain.
Outline how Fitzgerald and Lorentz explained the apparent absence of evidence for aether wind.
Describe aether drag.
Discuss which played a greater role in motivating Einsteins work, the work of Michelson and Morley or that of Maxwell.

Explain qualitatively and


quantitatively the consequence
of special relativity in relation
to:
the relativity of simultaneity
the equivalence between
mass and energy
length contraction
time dilation
mass dilation.

64

3.3 Special relativity, light and time


Although relativity is Einsteins theory, many of the underlying ideas or
mathematical formulae were inspired or anticipated by others including Poincar,
Lorentz and Minkowski. Einstein, being a theoretician, did not conduct
laboratory experiments. However, he is famous for making good use of the
Gedankenexperiment or thought experiment to boil abstract ideas down into
simple concrete ones. Theory can sometimes be derived by imagining an
experiment being done, even if it is impractical. We will mention some of his
thought experiments in this section.

space

Speed of light
Newton regarded space and time as absolute. In practical terms, this means that
the length of 1 metre, the duration of 1 second and the geometric properties of
shapes would be the same to all observers everywhere. Not all physicists agreed,
but the success of Newtons laws silenced any philosophical discussion.
However, Maxwells theory (and the MichelsonMorley experiment) pointed
to the speed of light in a vacuum being constant to all observers. So Einstein said
one of three things must be wrong: the principle of relativity (the invariance of
laws of mechanics in all inertial reference frames), Maxwells equations or the
Galilean transformation (the basis of all of classical mechanics).
The principle of relativity seemed very fundamental to Einstein, so he didnt
reject that. In fact, he extended Galileo and Newtons principle of relativity to
include all laws of physics, not just mechanics. He called it his first postulate.
Following a suggestion by Jules Henri Poincar (18541912), Einstein
decided that as the speed of light in a vacuum was invariant in all inertial frames,
then that must also be a law of nature, which he called the postulate of the
constancy of the speed of light.
Maxwells equations accurately described electromagnetic phenomena, so
Einstein didnt want to reject them. So it must be the Galilean transformation
(and hence all of classical mechanics) that was wrong. But it is difficult to see how
something so simple could possibly be wrong.
Suppose you are on a moving train, shining a torch towards the front of the
carriage. To your eyes, the light travels the length of the carriage L. To you, its
speed is the length of the carriage divided by the time t it took to get there c=L/t.
To an observer at the train station, the light travelled the length of the carriage
plus the distance D the carriage travelled in that time: c=(L + D)/t. The
arithmetic is so laughably simple. How could both observers possibly get the same
value for c? It could only be possible if you and the observer at the train station
In other words, if the
disagree on the lengths L or D or the time period t.
speed of light is constant then length (space) and/or time are not absolutethey
must depend on the state of motion of the observer.
So why had no-one noticed until 100 years ago? Classical mechanics had
successfully described phenomena for three centuries, but it had never been tested
Classical mechanics and the
for things moving at close to the speed of light.
Galilean transformation are accurate approximations at speeds well below the
speed of light. Only when the properties of light itself were examined, did the
problems become obvious.
the speed of light is the ultimate
Einstein showed (in several ways), that
speed limitno observer can reach the speed of light. As a teenager, he asked
What would the world look like if I rode on a light beam? He answered as an
adult with a thought experiment. A light beam is a wave of oscillating electric and
magnetic fields moving at the speed c. If you were in the same reference frame as
the light beam, you would observe stationary electric and magnetic fields that
vary as sine waves in space, but are constant in time. This is not an allowable
solution to Maxwells equations, so it is not possible for an observer to travel at
the speed of lightit is the ultimate speed limit.

Simultaneity
Einstein demonstrated that simultaneity is relative. Events apparently simultaneous
to one observer are not necessarily so to all observers. Lets use Einsteins own

Describe the significance of


Einsteins assumption of the
constancy of the speed of light.
Identify that if c is constant
then space and time become
relative.

Whats so
Special about
Relativity?

instein called his 1905


replacement theory for
Newtons mechanics special
relativity. It is a special case in
the mathematical sense of being
restricted to particular
conditionsto inertial reference
frames. Einsteins general
relativity came 11 years later
and was generalised to include
non-inertial reference frames.
It replaced Newtons gravity.
65

Seeing in a
weird light:
relativity

O2

O1

platform

Figure3.3.1 Lightning strikes at A and B


appear simultaneously to
observer O1 but not to O2.

Solve problems and analyse


information using: E = mc2
l v = l0 1
tv =

c2

t0
1

mv =

v2

v2
c2

m0
1

v2
c2

Analyse and interpret some of


Einsteins thought experiments
involving mirrors and trains and
discuss the relationship
between thought and reality.

thought experiment. Observer O1 is standing on a train station platform


equidistant from points A and B, which appear to O1 to be struck simultaneously
by two bolts of lightning (Figure 3.3.1). Because the light travelled the same
distance from both points and the light reached O1 at the same time, O1 judges
that the lightning bolts struck simultaneously.
Suppose observer O2 is sitting in a high speed train passing the platform
without stopping. O1 calculates that at the moment the lightning struck, O2 on
the train was also equidistant from A and B and so naively assumes that O2 would
also see the events as simultaneous. However, by the time the light reached O1s
eyes, O2 was closer to B and had already seen the light from B but not A, and
Only if two events occur simultaneously at the
concluded B was struck first.
same place, will all observers agree that they were simultaneous.
Similar arguments can show that the order of events is relative. Since an effect
must come after its cause, relativity places restrictions on possible chains of cause
and effect. Because the speed of light is the universal speed limit, two events
separated by distance cannot have any influence over each other over a time scale
shorter than the time required for light to travel between them. Because no signal
or influence can travel between a cause and its effect faster than light, apparent
changes in the simultaneity or order of events based on the passage of light is more
than just an optical illusionit represents a fundamental limitation of reality.

Time dilation
The relativity of simultaneity suggests that time itself is a bit rubbery. Relativity
predicts that the rate of passage of time differs, depending on the velocity of
the observer.
Consider the following thought experiment (Figure3.3.2). Suppose you
(observer 0) are on a train, moving with speed v relative to the ground, while
measuring the speed of light by shining a light pulse vertically towards a mirror on
the train ceiling, a distance D from the light source. Outside on the ground is
observer v who appears to you to be rushing past with horizontal speed v and
watching your experiment. Both you and observerv regard your own reference
you both agree on three things: (1) the speed
frame as stationary. However,
of light, (2) your relative horizontal speed v and (3) the height of the mirror D.
You both agree on the height of the mirror because you are both in the same
reference frame with respect to vertical components.
Your light source also contains a detector capable of timing the interval t0
between the emission and detection of the light pulse. You do the experiment
and use the speed formula c = 2D/t0 to obtain the correct speed of light.
a

v
mirror

mirror
D
source and
detector

vtv
source

detector

Figure3.3.2 Measuring the speed of light on a train as seen by (a) observer 0 within the train and
(b) observer v from the reference frame on the ground
66

space
Observerv disagrees about what happened. She was carrying an accurate
stopwatch and timed the event independently, getting a longer time interval of tv.
Using Pythagoras theorem, she calculates that the path length of the light
(Figure3.3.2b) was not 2D, but rather:
1
2

2 D 2 + ( vt v )2 = 4 D 2 + (vt v )2

She agrees on the speed of light c so:


4 D 2 + (vt v )2

c=

tv

Square and rearrange: c2tv2 = 4D2 + (vtv)2 but observer 0 says c = 2D/t0
Eliminate D:
Rearrange:

c2tv2 = c2t02 + (vtv)2


tv2(c2 v2) = c2t02
tv =

t0
1

v2
c2

The factor 1 / 1 v 2/ c 2 is called the Lorentz factor (abbreviated as ).


It is always larger than 1, which means that if t0 is the time between ticks on
observer0s clock then observer v will see observer0s clock whiz by at speed v,
ticking more slowly with a time tv between ticks. This is time dilation. The word
dilation means spreading out, just like the time between clock ticks.
An observer moving relative to you and observing a clock ticking (or any
series of events) stationary in your frame, will judge events to happen more slowly
than you observe them. Note that the dilation is only observed in clocks in other
You cant observe time dilation in clocks in your own
frames of reference.
frame, no matter how fast others think youre moving.
the effect is symmetrical; that is,
As there is no preferred inertial frame,
observers can be swapped. You both agree on your relative speed v but both insist
the other observer is moving and their clock is running slow. You are both right
because time is relative.
A time interval observed on a clock that is stationary relative to the observer is
To generalise this idea, t0
called the proper time for that reference frame.
can represent the time between any two events (such as the ticks of a clock) that
occur in the same place in the frame of the observer. If the events are separated in
space, then extra time is required to allow for light to travel between the positions
of the two events.
Global positioning system (GPS) receivers estimate your position by
measuring how long it takes the GPS signal to travel from the satellites to your
receiver. The orbital speed of the satellites is large enough that the calculation
needs to take time dilation into account. (It also takes into account a larger effect
from general relativity: time runs more slowly in a stronger gravitational field.)
The Lorentz factor approaches infinity as speed approaches the speed of light
(Figure3.3.3). This means that time in a frame of reference approaching the speed
of light (relative to the observer) will come to a complete stop. In other words,
nothing can be seen to happen in such a frame, which is another reason why the
speed of light is the ultimate speed limit.

7
6
5
tv 4
t0 3
2
1
0

0.2

0.4
0.6
Speed VC

0.8

1.0

Figure3.3.3 Plot of the ratio of a time


interval in a moving reference
frame to proper time (tv/t0)
versus speed in units of c. Note
that at speeds below ~0.1c,
the ratio1, so time behaves
nearly classically.
67

Seeing in a
weird light:
relativity

Worked example

Muon and on
and on

ast-moving muons are produced


in the upper atmosphere by
cosmic ray bombardment. Time
dilation extends their normally short
lifetimes long enough to allow many
of them to make it to Earth, where
they are a significant component of
Earths background radiation.)

Figure3.3.4 Both twins think the others


clock is moving slower, but
who is older at the end of
the journey?

question
A muon is like a heavy electron, and at low speed it decays with a mean lifetime of
2.2106s. Suppose a beam of muons is accelerated to 80% of the speed of light.
What would their mean lifetime be in the laboratory reference frame?

Solution
Lifetime in the muons frame: t0=2.2106s
Speed of muons frame: v=0.80c
t0
Lifetime in the laboratory frame is tv: t v =
1

2.2 106
2

1 0.80

= 3.7 106 s

c
The twin paradox
When two people pass by quickly, observing each other, they both think the
others clock is running slower. The principle of relativity says you are both right.
The twin paradox is a thought experiment in special relativity. Bill goes for an
intergalactic cruise travelling at close to the speed of light (in Earths frame),
while Phil stays on Earth (Figure3.3.4). During the flight, they both correctly
conclude that the other twins frame is moving, and so he is ageing more slowly.
But what happens when Bill comes back home? Observations in the same
frame should agree. It turns out that Bill is younger than Phil. Does this violate
the principle that all inertial frames of reference are equivalent? No. Bill turned
around (accelerated) to come home. The situation is no longer symmetrical.
Special relativity isnt enough to explain what Bill saw from his accelerating
frame (he needs general relativity). However we have no difficulty talking about
what (non-accelerating) Phil saw.
By turning around and coming back, Bill left his original inertial frame and
re-entered Phils frame, so he should agree with Phil. Phil remained in his inertial
frame all along, so his conclusions (that Bill was moving and so is younger) have
been consistent with special relativity throughout and, in his frame, correct.
If instead Phil had hopped into another craft and caught up with Bills inertial
frame, then Bills original conclusion would have been correct and Phil would
have been younger.
This prediction has been confirmed using highly precise, twin atomic clocks
and an aeroplane.

Checkpoint 3.3
1
2
3
4
5
6
7
8

68

State Einsteins first postulate and its alternative name.


State Einsteins second postulate and its alternative name.
Outline why Newtons classical mechanics is so successful despite a fundamental error (the Galilean transformation).
Explain why the speed of light places restrictions on possible chains of cause and effect.
Write the formula for time dilation.
A clock moving towards you appears to slow down. If the clock were moving in the opposite direction, would it speed up?
What is the name given to a time interval measured on a clock that is stationary in your frame of reference?
In the twin paradox, during a period of constant relative motion, both Bill (astronaut) and Phil (earthling) observe the
other twins watch ticking more slowly. Whos observation is actually correct?

space

3.4 Length, mass and energy


The formula for time dilation has already upset our common sense. However,
once the clocks start talking to the rulers and the masses, things can only get
more bizarre.

Solve problems and analyse


information using: E = mc2
l v = l0 1

Length contraction
There is a grain of truth in Lorentz and Fitzgeralds suggestion (section3.2)

tv =

that the arm of a Michelson interferometer contracts by a factor of 1 v 2/ c 2


in the direction of motion. Their formula was correct, but their interpretation
that it resulted from motion through the (non-existent) aether was wrong. Also,
the contraction doesnt happen in the frame of reference of the experimenter.
Moreover, their hypothesis was ad hoc; it was designed only to patch a hole
in the old theory without resulting in any additional testable predictions. So
Einstein re-interpreted their mathematics in light of his theory of relativity.
If an object is moving with speed v relative to the observer, the length of the
object in the direction of that motion will be observed to be contracted
according to the formula:
l v = l0 1

v2
c2

m0
1

v2
c2

Explain qualitatively and


quantitatively the consequence
of special relativity in relation
to:
the relativity of simultaneity
the equivalence between
mass and energy
length contraction
time dilation
mass dilation.

v2
c2

where l0 is the length judged by an observer who is stationary relative to the


object (proper length) and lv is the length judged by an observer in a frame
The length contraction only
moving with speed v relative to the object.
takes place in the dimension parallel to the motion. Just like time dilation:
1 the effect is symmetrical, which means the observers can be swappedboth
insist it is the other persons ruler that is too short
2 you cannot observe a Lorentz contraction within your own frame.

c2

t0
1

mv =

v2

1.2
1
0.8
lv 0.6
l0
0.4

Imagine that observer1 and observer2 are trying to measure the length of
0.2
a rod, but all they have is a stopwatch. They already know accurately (and agree
on) their relative speed v. Observer1 is holding the rod and observer2 is holding
0
0
0.2
0.4 0.6 0.8
1.0
the stopwatch. They whoosh past each other almost touching, both looking at
Speed VC
the watch.
Figure3.4.1 Plot of the ratio of length in a
Observer2 is stationary relative to the watch (Figure3.4.2a), so he knows the
moving reference frame to
reading on his watch is his proper time. As the rod passes by, the watch reads
proper length (lv/l0) versus
zero at the start of the rod and t2 at the end, so the rod took a time t2 to pass by.
speed in units of c. Note that as
Therefore he calculates that the length of the rod in his frame is lv=vt2.
speed approaches c, lv shrinks
to zeroanother reason why the
Observer1 is stationary relative to the rod (Figure3.4.2b), so she knows that
speed of light is unattainable.
its length for her is the proper length l0. She agrees that the watch says t2, but the
moving watch seemed to be ticking too slowly, so the number on the watch must
be too small. Using the time dilation formula,
she calculates that the time t1 in her frame
a
b
was longer:
t2
v
t1 =
v
2
v
1 2
c
Figure3.4.2 Measuring the length of a rod using a stopwatch as seen by (a) observer 2,
60

55

60

60

10

50

15

45

20

40

35

30

25

55

10

50

15

45

20

40

35

30

25

holding the watch, and (b) observer 1, holding the rod


69

Seeing in a
weird light:
relativity

Observer1 then calculates that the length of the rod is l0=vt1 or


vt 2
.
l0 =
v2
1 2
c
But observer2 says that lv=vt2, so by substitution and rearrangement

b
v

C'
A'

B'
c

question
v

A'

B'

c2

Worked example

D'

C'

v2 .

l v = l0 1

The distance travelled by light in one year, 9.461015m, is called a light-year (ly).
The nearest star to our Sun is Proxima Centauri, 4.2light-years away.
Suppose you are travelling to Proxima Centauri at three-quarters of the speed of light.
a Calculate how long it takes to get there from Earth (measured using your
on-board clock).
b Discuss whether this answer is a contradiction.

Figure3.4.3 A fast-moving vehicle appears


contracted horizontally, but also
rotated away from the observer.
The car is depicted when (a)
stationary, (b) moving at high
speed and (c) viewed from
above. Corner C is normally out
of sight, but at high speed, the
vehicle moves out of the way
fast enough to allow light
reflected from C to reach your
eyes at O, allowing you to see
the cars back and side at the
same time. This is called
TerrellPenrose rotation.

Solution
a Both you and Earth-bound observers agree on your relative speed 0.75c. In the
spaceships frame, the distance to Proxima Centauri is contracted:
l v = l 0 1

t=

v2
c

= 4.2 1 0.752 = 2.78 ly

l v 2.78 ly
=
= 3.7 years
c 0.75c

3.7 years is less than the 4.2 years that light takes to get there in Earths frame.
b
This is not a contradiction because in the spaceships frame, light would only
take 2.78 years because lv = 2.78 ly.
a

Earth

Proxima
Centauri

b
v

v
Earth

Proxima
Centauri

Figure3.4.4 Trip to Proxima Centauri as seen by


(a) earthlings and (b) the astronauts

Discuss the implications of


mass increase, time dilation
and length contraction for
space travel.

70

Note that in the last example, the astronauts thought they experienced a
short trip because the distance travelled was contracted, whereas the earthlings
thought the astronauts felt their trip was short because their time had slowed.

space

Relativistic mass
If you measure the mass m0 of an object at rest in your frame (rest mass or
proper mass) and use the classical definition of momentum p=m0v, then in
collisions, momentum is not necessarily conserved for all reference frames.
However, momentum is conserved if one instead uses p=mvv where mv
is the relativistic mass:
m0
mv =
v2
1 2
c
The relativistic mass of an object increases as its speed relative to the observer
increases. As speed approaches c, the mass approaches infinity, so the force
required to accelerate an object to the speed of light becomes infinite. This is yet
another reason why the speed of light cannot be reached.
When accelerating particles in accelerators, this increase in mass needs to be
taken into account, otherwise the machines wont work.
7
6

rains A and B are about to


collide head-on, each with
a speed 0.5c relative to the
station. So, relative to train B,
train A is moving at the speed
of light, right? Wrong! The
replacement for Galileos relative
velocity rule in 1dimension is:

v (A rel. to B) =

vA v B
v v
1 A 2B
c

The speed of trainA relative to


train B is:
0.5c (0.5c )
= 0.8c
0.5c (0.5c )
1
c2

5
mv 4
m0 3
2

Figure3.4.5 Plot of the ratio of relativistic mass

1
0

Relativistic
train crash

0.2

0.4
0.6
Speed CV

0.8

mv in a moving reference frame to


rest mass m0 versus speed in units
of c. As speed approaches c, the
relativistic mass approaches infinity.

Worked example
question
A medical linear accelerator (linac) accelerates a beam of electrons to high kinetic energies.
These electrons then bombard a tungsten target, producing an intense X-ray beam that can
be used to irradiate cancerous tumours. A typical speed for electrons in the beam is 0.997252
times the speed of light.
Calculate the Lorentz factor and hence the relativistic mass of these electrons, given the
rest mass is 9.111031kg.

Solution

Lorentz factor =

1
1

mv =

m0
1

v2

1
1 0.997252

= 13.5

c2
= 9.11103113.5 = 1.231029kg

c2

Note: When calculating Lorentz factors close to the speed of light, use a greater number of
significant figures than usual, because you are subtracting two numbers of very similar size.
71

Seeing in a
weird light:
relativity
Solve problems and analyse
information using:
E = mc2
l v = l0 1
tv =

c2

t0
1

mv =

v2

v2
c2

Mass, energy and the worlds most famous equation

The kinetic energy formula K= 1 mv2 doesnt apply at relativistic speeds,


2
even if you substitute relativistic mass mv into the formula. Classically, if you
apply a net force to accelerate an object, the work done equals the increase in
kinetic energy. An increase in speed means an increase in kinetic energy. But
in relativity it also means an increase in relativistic mass, so relativistic mass
and energy seem to be associated. Superficially, if you multiply relativistic
mass by c2 you get mvc2, which has the same dimensions and units as energy.
But lets look more closely at it.

m0
1

v2
c2

PHYSICS FEATURE
Twisting spacetime ...
and your mind
1. The history of physics

here are two more invariants in special relativity.


Maxwells equations (and hence relativity)
requires that electrical charge is invariant in all
frames. Another quantity invariant in all inertial frames
is called the spacetime interval.
You may have heard of spacetime but not know
what it is. One of Einsteins mathematics lecturers
Hermann Minkowski (18641909) showed that the
equations of relativity and Maxwells equations become
simplified if you assume that the three dimensions of
space (x,y,z) and time t taken together form a
fourdimensional coordinate system called spacetime.
Each location in spacetime is not a position, but rather
an eventa position and a time.
Using a 4D version of Pythagoras theorem,
Minkowski then defined a kind of 4D distance
between events called the spacetime interval s given by:
s2 = (ctime period)2 path length2

= c2t2 ((x)2 + (y)2 + (z)2)
Observers in different frames dont agree on the
3D path length between events, or the time period
between events, but all observers in inertial frames
agree on the spacetime interval s between events.

72

Figure 3.4.6 One of the four ultra-precise superconducting spherical


gyroscopes on NASAs Gravity ProbeB, which orbited
Earth in 2004/05 to measure two predictions of general
relativity: the bending of spacetime by the Earths
mass and the slight twisting of spacetime by the
Earths rotation (frame-dragging)

In general relativity, Einstein showed that gravity


occurs because objects with mass or energy cause this
4D spacetime to become distorted. The paths of
objects through this distorted 4D spacetime appear to
our 3D eyes to follow the sort of astronomical
trajectories you learned about in Chapter2 Explaining
and exploring the solar system. However, unlike
Newtons gravitation, general relativity is able to handle
situations of high gravitational fields, such as
Mercurys precessing orbit around the Sun and black
holes. General relativity also predicts another wave that
doesnt require a medium: the ripples in spacetime
called gravity waves.

space
How does this formula behave at low speeds (when v2/c2 is small)?
mv c 2 =

m0c 2
1

v2

v2
= m0c 2 1 2
c

1
2

c2
Using a well-known approximation formula that you might learn at university,
(1 x )n1 nx for small x:
v2
m0c 2 1 2
c

1
2

1 v2
1
m0c 2 1 + 2 = m0c2 + m0v2
2
2 c

1
m v2
2 0
In other words, at low speeds, the gain in relativistic mass (mv m0)
multiplied by c2 equals the kinetic energya tantalising hint that at low speed
mass and energy are equivalent. It can also be shown to be true at all speeds,
using more sophisticated mathematics. In general, mass and energy are
equivalent in relativity and c2 is the conversion factor between the energy unit
(joules) and the mass unit (kg). In other words:

Rearrange:

mvc2 m0c2 = (mv m0)c2

E = mc2
where m is any kind of mass. In relativity, mass and energy are regarded as the
same thing, apart from the change of units. Sometimes the term mass-energy is
used for both. m0c2 is called the rest energy, so even a stationary object contains
energy due to its rest mass. Relativistic kinetic energy therefore:
m0c 2
mv c 2 m0c 2 =
m0c 2
2
v
1 2
c
Whenever energy increases, so does mass. Any release of energy is
accompanied by a decrease in mass. A book sitting on the top shelf has a slightly
higher mass than one on the bottom shelf because of the difference in
gravitational potential energy. An objects mass increases slightly when it is hot
because the kinetic energy of the vibrating atoms is higher.
Because c2 is such a large number, a very tiny mass is equivalent to a large
amount of energy. In the early days of nuclear physics, E=mc2 revealed the
enormous energy locked up inside an atoms nucleus by the strong nuclear force
that holds the protons and neutrons together. It was this that alerted nuclear
physicists just before World War II to the possibility of a nuclear bomb. The
energy released by the nuclear bomb dropped on Hiroshima at the end of that
war (smallish by modern standards) resulted from a reduction in relativistic mass
of about 0.7g (slightly less than the mass of a standard wire paperclip).

Evil twins

he most extreme massenergy


conversion involves antimatter.
For every kind of matter particle
there is an equivalent antimatter
particle, an evil twin, bearing
properties (such as charge) of
opposite sign. Particles and their
antiparticles have the same rest
mass. When a particle meets its
antiparticle, they mutually
annihilateall their opposing
properties cancel, leaving only
their mass-energy, which is
usually released in the form of
two gamma-ray photons. Matter
antimatter annihilation has been
suggested (speculatively) as a
possible propellant for powering
future interstellar spacecraft.

Discuss the implications of


mass increase, time dilation
and length contraction for
space travel.

Worked example
question
When free protons and neutrons become bound together to form a nucleus, the reduction in
nuclear potential energy (binding energy) is released, normally in the form of gamma rays.
Relativity says this loss in energy is reflected in a decrease in mass of the resulting atom.
73

Seeing in a
weird light:
relativity

Exploding
a myth

t is commonly believed (wrongly)


that Einstein was involved in the
US nuclear bomb project. Perhaps
this is because, during World War
II, the nuclear physicists Leo
Szilard, Eugene Wigner and
Edward Teller, knowing such a
bomb was possible and worried the
Nazis might build one, wrote a
letter to President Roosevelt
suggesting the US beat them to it.
They asked their friend Einstein to
sign it because, being the most
well-known scientist at the time,
he would be taken seriously. Apart
from that, Einstein did two days
work on the theory behind uranium
enrichment.

Calculate how much energy is released when free protons, neutrons and electrons
combine to form 4.00g of helium-4 atoms (2 protons + 2neutrons + 2 electrons). At room
temperature and pressure, each 4g of helium gas is about 25L, roughly the volume of an
inflatable beach ball.
Data:

Mass of proton mp=1.6726221027kg

Mass of neutron mn=1.6749271027kg

Mass of electron me=9.111031kg

Mass of helium atom mHe=6.6464761027kg

c2 = 8.98761016m2 s2

Solution
Total mass of the parts:

mT = 2(mp+mn+me) = 2(1.672622 + 1.674927 + 0.000911)1027kg

= 6.696921027kg

Reduction in mass:

m = mT mHe = (6.69692 6.646476)1027kg

= 5.04441029kg

Binding energy per He atom:


E = mc2 = 5.04441029kg8.98761016m2 s2

= 4.53371012J

Binding energy for 4.00 g (0.004 kg):


4.53371012J
0.004 kg = 2.731012J
mHe
This much energy would be released by the explosion of more than 600 tonnes of TNT.

Some physicists dislike the definition of relativistic mass mv of a moving object


and prefer to talk only about the energy of an object (and its rest mass m0). There
are problems with the definition, including the fact that relativistic mass doesnt
behave like a scalar, because it can be different along different directions.

Checkpoint 3.4
1
2
3
4
5
6
7
8

74

Discuss why, if Lorentz and Fitzgerald came up with the correct formula for length contraction, Einstein gets the
credit for explaining relativistic length contraction.
Write the formula for length contraction. Would a ruler moving lengthwise relative to you appear shorter or longer?
Define the term proper length.
To what limit does observed length of a moving object tend as speed approaches c?
Write the formula for relativistic mass. Would a mass moving relative to you appear larger or smaller?
Use relativistic mass to justify the statement that the speed of light is the universal speed limit.
Define all the terms in the equation E = mc2 and explain what the equation means.
Explain why an atom weighs less than the sum of its parts.

PRACTICAL EXPERIENCES

space

CHAPTER 3

This is a starting point to get you thinking about the mandatory practical
experiences outlined in the syllabus. For detailed instructions and advice, use
in2 Physics @ HSC Activity Manual.

60

20

10

20 30 40
0 10
50

90 8

0 70 60 50

40

30

Discussion questions
1 The principle method for detecting a non-inertial frame is measurement
of acceleration. Describe an example of a non-inertial frame in which
a typical accelerometer would not appear to measure an acceleration or
detect extra fictitious forces.
2 Is there a test that can be performed within a frame of reference to tell
if the effect measured by the accelerometer is the result of acceleration
of the frame or due to an actual additional force?

80

Perform an investigation that allows you to distinguish between inertial and


non-inertial frames of reference.
Equipment: protractor, string, mass (50 g), tape, cardboard, chair on wheels
or skateboard.

Perform an investigation to
help distinguish between
non-inertial and inertial frames
of reference.

70

Activity 3.1: Fact or fiction: Inertial and


non-inertial frames of reference

Figure 3.5.1 An accelerometer

Activity 3.2: Interpreting the MichelsonMorley


experiment results
Use simulations to gather data from the MichelsonMorley experiment. You will
gather data as though there is and is not an aether, and then interpret the results.
There are many MichelsonMorley experiment simulations available. Two
web-based examples are given on the companion website.

Gather and process information


to interpret the results of the
MichelsonMorley experiment.

Discussion questions
1 Describe what Michelson and Morley were expecting to observe if aether
were present.
2 Using the data you have gathered, explain how your observations support
or refute the existence of the aether.
3 Recall the interpretation put forward by Michelson and Morley.
4 Discuss the importance of this experiment.
Extension
1 Research the history of how long the belief in aether persisted in some
physicists after the publication of special relativity in 1905.
2 Read the following paper, which contains a thorough review of the history
of the MichelsonMorley experiment, including historical letters to and
from several researchers:
Shankland, RS, 1964, MichelsonMorley Experiment, American Journal
of Physics, vol.32, p 16.

75

Chapter summary

Seeing in a
weird light:
relativity

Inertial reference frames are those that do not accelerate.


Principle of relativity: The laws of mechanics are the
same in all inertial reference frames. Einstein extended it
to all laws of physics (first postulate of relativity).
When judged within a non-inertial frame, fictitious
forces are perceived.
Maxwells equations for electromagnetism predicted
only a single possible speed for light, which was assumed
to be relative to a hypothetical medium called aether.
Michelson and Morley failed to detect changes in speed
due to aether wind, using an interferometer. Fitzgerald
and Lorentz made the ad hoc suggestion that things
contract when moving relative to the aether, hiding the
effect of the changing relative speed of light.
Einstein and others argued that aether was not required
by Maxwells equations and was inconsistent with the
principle of relativity.
Second postulate of relativity: The speed of light is
constant to all observers.
The speed of light is the fastest possible speed.
The finite speed of light means different observers
disagree on the simultaneity and order of events. Only
events at the same time and place are agreed by all
observers to be simultaneous.
1
Lorentz factor: =
v2
1 2
c
Proper time t0 is a time interval measured on a clock
stationary in the observers frame.

Proper length l0 is the length of an object stationary in


the observers frame.
Proper or rest mass m0 is the mass of an object stationary
in the observers frame.
t0
Time dilation: t v =
v2
1 2
c
Clocks (and all time-dependent phenomena) evolve in
time more slowly if they are moving relative to the
observers frame.
v2
Length contraction: l v = l 0 1 2
c
Length lv of an object moving relative to the observers
frame contracts in the direction of motion.
m0
Relativistic mass: mv =
v2
1 2
c
Mass of an object mv moving relative to the observers
frame increases.
Two observers in separate inertial frames will agree on
their relative speed v.
However, both observers will judge the other observer to
be moving and, hence, subject to time dilation, length
contraction and relativistic mass increase. They disagree,
but both are correct because these three quantities are
relative. Only when two observers are in the same frame
will they agree on these.
Mass and energy are equivalent: E = mc2. A small mass
is equivalent to a large energy.

Review questions
Physically speaking
Use the words below to complete the
following paragraph:

Inertial _______________ have _______________ status in _______________ mechanics.


_______________s laws apply in these frames. If one performs measurements

Galileo, Newton, Einsteins, Maxwell,

in _______________, then _______________ forces might be perceived. Classical


mechanics and _______________ relativity both agree that physical laws are

constancy, fictitious, change,

_______________ in _______________ frames. However, they disagree on the

non-inertial frames, length, observer,


classical, value, invariant, mass, time,
frames, speed, inertial, special

_______________ of the speed of light. According to _______________s equations,

the _______________ of the speed of light does not _______________ between frames,
so light doesnt obey the transformation formula of _______________. Because of
this, measurements of _______________, _______________ and _______________ within
a reference frame moving relative to the _______________, will depend on the
_______________ of that frame.

76

space

Reviewing
1 You have a priceless Elvis Presley doll hanging from
your rear-vision mirror at a constant angle from
vertical. Elviss feet lean towards the front of the car.
Are you driving:
A forwards at uniform speed?
B backwards at uniform speed?
C forwards but accelerating?
D forwards but decelerating?

Solve problems and analyse information using:


E = mc2
l v = l0 1
tv =

exerted on you by the seatbelt fictitious? Centrifugal


force normally refers to the fictitious force you feel
pushing you outwards when you steer a car. Some
people have suggested re-defining centrifugal force as
the outward reaction force you exert on the seat belt
in response to the centripetal force it exerts on you.
Re-defined in this way, is centrifugal force still
fictitious? Justify your answers.

3 At the end of the 19th century, no-one was able to


travel at close to the speed of light, and clocks, rulers
and mass balances werent sensitive enough to
measure relativistic changes. So why did the
problems with classical physics start to become
obvious then?

4 Explain why interferometry is an extremely sensitive


method for measuring short differences in time
or length.

5 Explain why Michelson and Morley performed their


experiment at different times of the day and year.

6 If we were an entire civilisation of blind people relying


on sound instead of light to decide the simultaneity
of events, would our equations for relativistic length,
time and mass contain c=340ms1 (the speed of
sound in air) instead? Whats so special about the
speed of light? Discuss.

7 In Figure 3.3.2b, the dimensions of the light path


have been drawn correctly. However, for simplicity,
two aspects of the trains appearance to observer v
have been left out. Describe two changes that would
need to be made to Figure 3.3.2b to represent these
effects more correctly.

8 Suppose our relativistic twins Bill and Phil both got


into spacecraft, went off in opposite directions and
took journeys at relativistic speeds that were mirror
images (judged from Earth). Predict and explain:
a how their apparent ages will compare when they
come back home
b how their apparent ages will be judged by stay-athome earthlings.

mv =

c2

t0
1

2 In a car that is cornering, is the centripetal force

v2

v2
c2

m0
1

v2
c2

9 Prunella and Renfrew, two observers in inertial frames


moving relative to each other, will always agree on
their relative speed v. A third observer, Thor, standing
between them, sees them both coming towards him
from opposite directions, at equal speeds. Is it correct
to say that relative to Thor, Prunella and Renfrew are
both moving at a speed of v|2?

10 A stretch-limo drove into a small garage at near light


speed. The garage attendant slammed the garage
door behind the car. For a brief time the attendant
saw that the relativistically shortened limo was
completely contained between the closed garage door
and the rear garage wall. A short time later, the stillmoving car smashed through the back wall. As far as
the driver was concerned, the garage was shortened
and the limo was too long for the garage so the limo
was never contained between a closed door and the
intact back wall. Reconcile the two differing accounts
of what happened. (Hint: See section 3.3.)

11 Show that mc2 has the units and dimensions of energy.


12 In a perfectly inelastic collision, two colliding objects
stick together. In a symmetrical inelastic collision
between two identical objects, the final speed is zero
in the frame of their centre of mass. Given that massenergy is conserved in an inertial frame, is the mass of
the system the same as before the collision? Explain.
(Hint: What happens to kinetic energy in an inelastic
collision?)

77

Seeing in a
weird light:
relativity

Solving Problems
13 Depending on your answer to Question 1, calculate
the magnitude of your speed or acceleration if the
Elvis Presley doll hangs at a constant angle of 10
from vertical.

Solve problems and analyse information using:


E = mc2
l v = l0 1

14 The caption for Figure 3.2.3b states that increasing


the length of the arms would increase sensitivity to
changes in the speed of light. Justify this, using the
equations given in that section.


L = L 1 v 2 c2
then the difference t2t1 between the times of flight
for the two arms would be zero. Use the equations
given for t2 and t1 in section3.2.

16 In the worked example of your trip to Proxima


Centauri (Figure 3.4.4), one member of the crew had
a mass 80kg at launch. Assuming his normal diet
and physiology were maintained, what would you
expect his mass to be during the trip:
a as measured on the spaceship?
b as judged from Earth?

17 Your rival in the space race plans a trip to Alpha


Centauri, which is slightly further away (4.37ly). She
wants to do the trip in 3.5years (one-way) as judged
by her own on-board clock.
a What speed (as a fraction of c) does she need to
maintain?
b How long does the trip take as judged from Earth?

19 For subatomic particles, a more conveniently sized


(non-SI) unit of energy is the electron volt (eV). The
conversion is E(eV)=E(J)/e where e=1.601019C,
the charge on an electron. A mega-electron volt (MeV)
is 106eV.

For the worked example on page 71, show that the


kinetic energy of the electron in the medical linac
beam is 6.4MeV (me=9.111031kg). What is the
total energy of that electron?

20 Estimate the total energy (in joules) released by the

Re

78

iew

Q uesti o

Hiroshima bomb (m0 = 0.7g).

c2

v2
c2

21 In their rest frame, muons have a mean lifetime of

2.2106s. However, measurements (at various


altitudes) of muons produced by cosmic rays indicate
that, on average, they travel 6.00103m from where
they are produced in the upper atmosphere before
decaying. Calculate their average speed (as a fraction
of c).

22 Show that if the speed of light were infinite, the


following equations would revert to their classical
form.
a
b

18 Calculate the total energy in the two gamma ray


photons produced when an electron meets a positron
(an anti-electron) (me=9.111031kg).

v2

m0

mv =

that (in agreement with Fitzgerald and Lorentzs


suggestion) if the length L of the interferometer arm
parallel to the aether wind shrinks to

c2

t0

tv =

15 Supposing the aether hypothesis were correct, show

v2

2 2

l v = l0 1 v c
tv =

t0

2 2

1v c

mv =

m0

2 2

1 v c

v (A rel. to B) =

vA vB

1 vA vB / c 2

23 Research the history of relativity and list up to five


historically important experimental confirmations of
its predictions. Make a timeline of the events. Note
that some experiments may pre-date relativity. For
example, in 1901 W Kaufmann measured the
increase in an electrons mass as its speed increased.
If possible, identify whether such examples came to
Einsteins attention before he formulated his theory.
Analyse information to discuss the relationship between
theory and the evidence supporting it, using Einsteins
predictions based on relativity that were made many
years before evidence was available to support it.

space

PHYSICS FOCUS
Cant measure the speed
of light

he French metric system, which evolved into the


Systme International dUnits or SI units, was
originally based on artefact standards. The standard
metre bar and kilogram were real objects (or artefacts)
in Paris. Artefacts can degrade or be damaged, and
making copies for standards labs is expensive, slow
and unreliable. Artefact standards are now being
replaced by fundamental physical property standards.
One second is now defined as a certain number of
periods of oscillation of a very stable light frequency in
the spectrum of cesium-133 (in atomic clocks).
Measurement standards often involve sensitive
interferometry. The metre was changed in 1960 from
the original bar to a certain number of wavelengths
(measured interferometrically) of a colour from the
krypton-86 spectrum.
Being invariant, the speed of light is very useful for
standards. Interferometric measurements of the speed
of light became so precise that the weakest link was
the experimental difficulty in reproducing the
So in 1983 the
krypton-86 standard metre.
speed of light was fixed by definition at exactly
299792458ms1 and the standard metre was
redefined as the distance travelled by light in
1|299792458 of a second. Now, any lab with an
interferometer and an atomic clock can produce its
own standard metre.
Since 1983, by definition, the speed of light can
no longer be measured. Traditional procedures for
measuring the speed of light should now be called
measuring the length of a metre.
The last artefact standard, the platinumiridium
kilogram in Paris, appears to be changing mass slightly.
The Avogadro project at Australias CSIRO is trying to
develop a replacement for it with a procedure for
making and testing (almost) perfect spheres of silicon
that could be made in standards labs around the world
without the need to copy the original sphere directly.
The spheres are measured using interferometry, with the
best result so far being an overall distortion from
sphericity of 30nm and an average smoothness of 0.3nm.

2. The nature and practice of physics

3. Applications and uses of physics

5. Current issues, research and


developments in physics
Discuss the concept that length standards are
defined in terms of time in contrast to the original
metre standard.

1 Explain why standards based on fundamental


physics properties are preferable to artefacts.
2 Justify (in light of relativity) the statement that the
speed of light is an especially good property on
which to base a measurement standard.
3 The 1960 metre standard was based on light from
krypton-86. Explain why it needed to specify the light
source and why the new metre standard doesnt.
4 Given that the value of the speed of light is now
arbitrarily fixed, discuss why they didnt just make
the speed of light a nice round number such as
3.00000000108ms1.
5 A single atomic layer of silicon is approximately
5.41010m thick. For the best silicon sphere in
the Avogradro project, to approximately how many
atomic layers does the reported distortion from
sphericity and average smoothness correspond?

Figure 3.5.2 One of CSIROs accurate silicon spheres

79

The review contains questions in a similar style and proportion to


the HSC Physics examination. Marks are allocated to each question
up to a total of 30marks. It should take you approximately
54 minutes to complete this review.

Multiple choice
(1 mark each)
1 Ignoring air resistance, all projectiles fired
horizontally from the same height above horizontal
ground will have the same:
A horizontal velocity.
B time of flight.
C range.
D final speed.

Which of the following orbits has a two-body


mechanical energy greater than zero?
A Geostationary
B Elliptical
C Parabolic
D Non-returning comet

You have just rounded the top of a curve on a rollercoaster. The g-force meter you are carrying reads
exactly zero. Which one of the following is true?
A Your weight is the centripetal force.
B Your weight is zero.
C Your weight is equal and opposite to the normal
force exerted on you by the seat.
D Your weight is equal and opposite to the tension
in your body.

80

The MichelsonMorley experiment demonstrated that:


A the aether wind was undetectable.
B waves do not require a medium.
C one arm of the interferometer contracted in
response to the aether wind.
D aether is trapped by mountains and valleys and
dragged along with the Earth.

Observer A on the ground, watches a train


(containing observer B) rush past at speed v. Both
make measurements of things in each others frame
of reference. From the following list of statements,
choose the statement they disagree on.
A The other observers frame of reference is moving
with speed v.
B The apparent length of my own metre ruler is
longer than the apparent length of other
observers metre ruler.
C Observer Bs watch appears to run slower than
observer As watch.
D The height of the train carriage ceiling is 2.2 m
above the carriage floor.

Short response
6 The escape velocity from the Earths surface, based

on Newtons original concept, is 11.2 kms1. Briefly


explain two ways in which this number is not quite
applicable to real Earth-surface launches. (2 marks)

7 Calculate the potential energy of a 2500 kg satellite


in a geostationary orbit around the Earth. Assume
a sidereal day is 23h 56min 4s. (3 marks)

8 In their rest frame, charged pions have a mean

lifetime of 2.60108s. A particular beam of


charged pions travel an average distance of 30m
before decaying. Calculate their speed (as a fraction
of the speed of light). (4 marks)

9 Explain why if you are in a circular orbit and you


briefly retro-fire your engines to slow down, you move
to a faster orbit. (3 marks)

space
10 A 9000 kg helicopter is parked at the equator and
then later near the North Pole. Assuming the two
locations are chosen so that the gravitational potential
energy is the same at the two spots, estimate the
difference in relativistic mass at the two locations.
At which location will the mass be larger? (Hint:
Velocity is low, so use the classical expression for
kinetic energy.) (3 marks)

Extended response
11 Critically discuss the following proposition: The
MichelsonMorley experiment was an embarrassment
for physics because, despite a large effort, it failed to
find what it was looking for and so it should be
relegated to the dustbin of physics history. (5 marks)

12 The following formula relates the length of a



pendulum (L) to the period of its swing (T).


L
T = 2
g
During your studies in physics you carried out an
experiment to determine acceleration due to gravity.
Describe and explain a method you would use to
perform this measurement. (5 marks)

81

2
Context

Figure 4.0.1 A generator produces electricity


in each of these wind turbines.

82

Motors and
Generators
The first recorded observations of the relationship between electricity and
magnetism date back more than 400 years. Many unimagined discoveries
followed, but progress never waits. Before we understood their nature, inventions
utilising electricity and magnetism had changed our world forever.
Today our lives revolve around these forms of energy. The lights you use to
read this book rely on them and the CD inside it would be nothing but a shiny
coaster for your cup. We use magnetism to generate the electricity that drives
industry, discovery and invention. Electricity and magnetism are a foundation for
modern technology, deeply seated in the global economy, and our use impacts
heavily on the environment.
The greatest challenge that faces future generations is the supply of energy.
As fossil fuels dry up, electricity and magnetism will become even more
important. New and improved technologies will be needed. Whether its a hybrid
car, a wind turbine or a nuclear fusion power plant, they all rely on applications
of electricity and magnetism.

Figure 4.0.2 A simple homopolar motor

INQUIRY ACTIVITY
Build your own electric motor
Many of the devices you use every day have electric motors. They spin your DVDs,
wash your clothes and even help cook your food. Could you live without them,
and how much do you know about how they work?
The essential ingredients for a motor are a power source, a magnetic field
and things to connect these together in the right way. Its not as hard as you
think. All you need is a battery, a wood screw, a piece of wire and a cylindrical or
spherical magnet. Put these things together as shown in Figure 4.0.2 and see
if you can get your motor to spin. Be patient and keep trying. Then try the
following activities.
1 Test the effects of changing the voltage you use. You could add another
battery in series or try a battery with a higher voltage.
2 Try changing the strength of the magnet by using a different magnet or
adding another. What does this affect?
3 Try changing the length of the screw, how sharp its point is or the material
it is made from. Does it have to be made of iron?

83

Electrodynamics:
moving charges
and magnetic
fields
Strange but true

electric current, conventional


current, magnetic field, force,
direct current (DC), alternating current
(AC), voltage, potential difference,
right-hand grip rule,
right-hand palm rule, motor effect,
current-carrying conductor

Some of the most surprising discoveries in science have come from


the relationship between electricity and magnetism. Who would have
thought that a current-carrying wire can act as a magnet? Could
anyone have guessed that moving a magnet could create electricity,
or that moving charges get a push from magnetic fields?
All these amazing facts were discovered by people who were
curious, creative and dedicated. They were interested
enough to find out how things work, and now its your
turn. Dust off your imagination and get ready to
picture the concepts behind many applications of
electricity and magnetism.

4.1 Review of essential concepts


In this module you will develop and apply an understanding of the relationship
between electricity and magnetism. First we need to review some essential
concepts you encountered in the Preliminary course (see in2 Physics @
Preliminary Chapters 10 and 12). You will need to recall these concepts
throughout this module and apply them to new situations.

Electric current
In the presence of an electric field, charged particles and free ions will move
towards a region with an opposite electric charge. In this module we will be
considering the movement of electrons within metal conductors such as copper.
If you connect a wire to the terminals of a battery, as in Figure 4.1.1, an electric
field is set up along the length of the wire between the two ends. The free
electrons in the metal wire are then attracted towards the positive terminal. The
movement of these electrons is called an electric current. Protons are bound
tightly inside the nuclei of atoms and the nuclei are essentially fixed in position.
This means that when a wire is connected to a battery the only charges free to
move inside the wire are the free electrons.
84

motors and
generators
An electric current is defined as the rate of flow of net charge through
a region. In the wire in Figure 4.1.1, a number of electrons will flow through the
area A each second, and each electron carries a charge of 1.6 1019 coulombs.
If the current is 1ampere (or 1amp), it carries 1 coulomb of charge per second
through area A, which is more than 6billion billion electrons each second.
We now know that the nature of an electric current is actually a flow of electrons.
However, in electric circuits we often consider currents as if they were a flow of
positive particles. This type of current is called conventional current and it is in the
opposite direction to the flow of the negatively charged electrons. This confusing
situation has arisen because current was first thought to be a flow of positive particles.
Conventional methods for determining the direction of other physical quantities,
such as magnetic fields and forces, have been developed using the conventional
direction of current. So we will stick with these historical conventions.
It is important throughout this module to consider the direction of an
electric current as the direction in which a positive charge would flow through a
conductor. In Figure 4.1.1, conventional current flows from the positive terminal
towards the negative terminal, as indicated by the arrow along the wire.
An electrical current in which the charges only flow in one direction is
called direct current (DC). This current is commonly used in small portable
electronic devices and is supplied by a battery. One way of illustrating this type
of current graphically is shown in Figure 4.1.2.
The red line on the graph is a direct current measured by a digital ammeter.
The sign of the current (+ or ) represents the direction in which the current is
travelling. You can see that in this example the current has a constant value and
direction over time.
In contrast to direct current, an alternating current (AC) is continually
changing direction. The sign of the terminal at each end of an AC circuit
alternates between positive and negative over time. Each time this occurs, the
electric field within the wire changes direction. This reverses the direction of the
force on the charges within the wire and the current changes direction accordingly.
This type of current is good for transporting electrical energy over large distances
and is commonly used in larger appliances. In Figure 4.1.2, an AC current
measured by a digital ammeter is shown as a blue line. As the current changes
direction the blue line moves above or below the horizontal
axis. The corresponding change in sign of the current
indicates a change in the currents direction.

Potential difference, emf and voltage


The work done by any electrical device can be traced back to
the creation of a difference in electrical potential energy. In
Figure 4.1.1, a chemical reaction separates the charges inside
the battery. This causes a difference in electrical potential
energy between the positive and negative terminals of the
The battery energy given to each coulomb
battery.
of charge within the battery is measured in volts and is
commonly called an emf (). In an ideal battery it is equal to
the voltage measured at the terminals when the battery is not
connected to a circuit. For this type of battery this is usually
about 1.5 V.

area
A

Figure 4.1.1 A battery creates an electric


field within the wire and
a current flows.

DC

+
AC
Current (A)

Direct current and alternating current

copper wire

10 20

30

40

50

Time
(ms)

Figure 4.1.2 A graph of AC and DC over time

Flick of a switch

he charges in a current-carrying wire travel along


the wire much more slowly than the speed at
which you normally walk. Why then does a light
come on instantly when you flick the switch? All the
free electrons in the wire start moving at the same
time under the influence of the electric field in the
wire. When you flick the switch, this field travels
along the wire at close to the speed of light. Almost
instantly all the free electrons are moving and losing
energy in the light bulb.
85

Electrodynamics:
moving charges
and magnetic fields

Birds on a wire

ave you ever wondered why


birds can happily sit on
power lines and not get
electrocuted? No, those wires are
not insulated. For a current to
flow through a bird on a wire,
there would have to be a potential
difference between its feet. If the
bird could stand on the wire and
touch any other object such as
the ground or another wire then it
would get the shock of its life.

Figure 4.1.3 There is no potential


difference between
the birds feet.

When a wire is connected to the batterys terminals (see Figure 4.1.1), an


electric field is set up within the wire. The electric field drives electrons from the
negative terminal, through the wire to the positive terminal. As the electrons
move through a circuit element, such as a light bulb, they collide with the ions in
the metal lattice. During these collisions they lose kinetic energy to the metal
lattice. The metal lattice then loses this energy as heat; in the light bulb it is also
lost as visible light.
The energy lost in the circuit element corresponds to a loss of electrical
The difference in potential
potential energy by the charges in the current.
energy per unit of charge between the two points either side of a circuit element
is known as the potential difference, potential drop or voltage (V). We can
measure this potential difference by connecting a voltmeter to the circuit in
parallel with the circuit element.

Resistance and Ohms law


Recall that the structure of a metallic conductor is essentially a lattice of metal
atoms (or ions) surrounded by a sea of free electrons. If a potential difference V
is established within the metal, these free electrons will flow as a current I.
The amount of current that flows due to this potential difference is
V
determined by the electrical resistance of the material, which is defined by R =
I
and measured in ohms (). A material that has a relatively high resistance
will only conduct a relatively small current for a given potential difference.
The resistance in a conductor is a result of the collisions of the moving
charges with the ions of the metal lattice. Basically, the more collisions the free
electrons have with the lattice the higher the resistance. Recall that the length,
cross-sectional area, temperature and type of material within the conductor
influence resistance.
In many circuit components, the ratio of voltage divided by current is a
This relationship is known as Ohms law and describes the
constant.
relationship in which V and I are proportional for a circuit, or circuit
component, with a fixed resistance R.

Resistance and power


The wire filament of the light bulb in Figure 4.1.1 has a very high resistance and,
therefore, a large amount of the available energy within the circuit is lost within
the bulb. This energy heats the filament in the blub to such a high temperature
that it emits visible light.
Whether energy in a circuit is lost as heat or in turning an electric motor, the
rate at which energy is converted into another form is called electric power P:
Power =

energy transferred
time taken for transfer

Recall that:
P = IV
V2
Substituting Ohms law V = IR: P = I2R or P =
R
where P is power in watts (W).
Watts are equivalent to joules per second (Js1), so we can determine the
energy lost:
Energy = Pt
where energy is in joules (J) and time is in seconds (s).
86

motors and
generators

Magnetic fields produced by electric currents


A magnetic field exists in a region of space if a magnet at some point in that
space would experience a magnetic force. From in2 Physics @ Preliminary
section 12.3, you know that a current-carrying conductor produces a magnetic
field. You can observe this circular magnetic field around a current-carrying wire
if you place a magnetic compass needle at various locations around the wire
(see Figure 4.1.4).
We represent magnetic fields by solid lines, called magnetic field lines, and
label these lines with the symbol B. These lines represent the places where the
magnetic field has the same strength. We also place arrow heads along these
By convention,
magnetic field lines to indicate the direction of the field.
the arrow heads indicate the direction in which the north pole of a magnet
would point within the magnetic field (see the compass needles in Figure 4.1.4).
You can work out the direction of the magnetic field in Figure 4.1.5
by using the right-hand grip rule. Grip the wire with your right hand and
point the thumb in the direction of the conventional current along the wire.
Remember, this is the direction in which positive particles would flow, from the
positive to the negative terminal. Your curled fingers will now point in the
direction of the magnetic field around the wire.
There are several conventions we need to recall when we draw twodimensional diagrams showing currents and magnetic fields. For example, when
we view the situation shown in Figure 4.1.5 from above, we represent this as
shown in Figure 4.1.6. The symbol in the centre indicates that the current is
coming towards you (out of the page). When viewing the situation from below,
we represent this as shown in Figure 4.1.7. The symbol in the centre indicates
the current is flowing away from you (into the page). Notice that the magnetic
field lines shown in Figures 4.1.6 and 4.1.7 are drawn more closely near the wire,
where the field becomes stronger.
The and symbols indicate that the current is flowing out of and
into the page respectively. You can remember this convention if you imagine that
is the head of an archers arrow coming out of the page at you. The crossed
feathers in the back of the arrow are represented by , indicating that the arrow
is pointing away from you.
We also represent magnetic fields in two-dimensional diagrams as shown in
To show that the magnetic field points into or out of the
Figure 4.1.8.
page, we use or respectively. Use the right-hand grip rule to determine the

Figure 4.1.6 Magnetic field lines around


a conventional current going
out of the page

Figure 4.1.4 A straight wire carrying a


current deflects the compasses
around it in a circular pattern.
electric current I

magnetic field B

Figure 4.1.5 The curled fingers point in the


direction of the magnetic field
when the thumb points in the
direction of the conventional
current along the wire.

Figure 4.1.7 Magnetic field lines around


a conventional current going
into the page
87

Electrodynamics:
moving charges
and magnetic fields
conventional current I

Figure 4.1.8 Magnetic field lines into ()


and out of () the page for a
wire carrying a conventional
current upwards in the plane
of the page

(a)

direction of the magnetic field around the wire in Figure 4.1.8. You should see
that the magnetic field lines would go into the page on the right-hand side of the
wire (represented by ) and come out of the page on the left (represented by ).
An extension of the situations you have reviewed above is the magnetic field
around a current-carrying wire loop. Again we use the right-hand grip rule to
determine the direction of the magnetic field around the current-carrying wire
(Figure 4.1.9). Notice that the magnetic field in the centre of the loop always
points in the same direction, no matter where your hand is around the loop.
In the following chapters there are many situations that involve loops of wire
carrying currents; therefore, it is important you are familiar with the magnetic
field that is produced around them.
Most applications of magnetic fields in current-carrying wire loops actually
involve more than one loop. Each loop is called a turn, and many turns together
are known as a solenoid (Figure 4.1.10). A solenoid is simply a long coil of wire,
and the magnetic field produced is similar to that of a bar magnet, with a north
The direction of the magnetic field through
and south pole at each end.
the centre of the solenoid is determined by using a special version of the righthand grip rule (Figure 4.1.10). In this situation, you must curl your fingers in
the direction of the conventional current around the solenoid and your thumb
will point in the direction of the magnetic field. Your thumb will point to the
end of the solenoid that forms a north pole. A coil such as this is used to make
an electromagnet or simply to produce a magnetic field.

(b)

Figure 4.1.9 (a) The right-hand grip rule


can be used for a current loop.
(b) Magnetic field lines around
a single wire loop

Figure 4.1.10 The right-hand grip rule is


used to find the direction of
the magnetic field inside the
solenoid.

The North Pole?

id you know that the


Earths north geographic
pole is actually its south
magnetic pole? The north pole
of a magnet is attracted towards
the north geographic pole and
therefore it must be a south
magnetic pole. Did you also
know that the Earths magnetic
field changes direction? At
irregular intervals of about
250000 years the polarity of
the Earths magnetic field flips
and points in the opposite
direction. Scientists are not
sure of the effects of this flip
or for how long the field
disappears during each flip.

Geographic
North Pole

Magnetic
South Pole

Magnetic
North Pole

Geographic
South Pole

Figure 4.1.11 The Earth acts as though it has a south magnetic pole near the geographic
north pole! The magnetic north pole is the place to which the north end of
a compass appears to point.

88

motors and
generators

Checkpoint 4.1

1 Define the nature and direction of conventional direct current.


2 Construct two-dimensional diagrams illustrating the magnetic field around a current-carrying wire from two
different perspectives (end-on and side-on).
3 Sketch and compare two diagrams illustrating the magnetic field around a bar magnet and a current-carrying
solenoid.

4.2 Forces on charged particles


in magnetic fields
We have already seen that free charged particles move when placed in an electric
field, because they experience a force (Figure 4.1.1). This force is caused by the
attraction and repulsion that charged particles experience within the field.
Knowing why this occurs, it might seem strange to hear that a charged particle
also experiences a force due to a magnetic field. There is, however, one important
A charged particle only experiences a force in a magnetic field
difference.
when the particle is moving relative to the magnetic field, or if the strength of
the magnetic field is changing. It is important to know that stationary charges or
charges moving parallel to the magnetic field do not experience a force.
In Figure 4.2.1a, a positively charged particle, lets say a proton of charge q,
is travelling upwards with a velocity v within a horizontal magnetic field B. The
proton experiences a force F in a direction perpendicular to both the magnetic
field and the direction in which it is moving. The force is given by F = qvB. The
way we can tell the direction in which the force is acting is to introduce another
right-hand rule. This rule is commonly called the right-hand palm
a
rule (or the right-hand push rule) and is illustrated in Figure 4.2.1b.
To find the direction of the force that a positive particle will
experience when moving through a perpendicular magnetic field:
1 Place your open right hand with the fingers pointing in the
direction of the magnetic field (north to south).
2 Place your thumb at right angles to your fingers and in the
direction in which the particle is travelling.
b
3 The force on the positive particle will be directed out of your
palm and at right angles to your hand.
We can conclude that the proton in Figure 4.2.1a will experience
a force out of the page at right angles to both the magnetic field and
the direction in which the proton is travelling. Note that if we know
the direction of any two of the three quantities represented in the
right-hand palm rule, we can use this rule to determine the direction
of the third quantity.

v
B
S

direction in which the


particle is travelling
with velocity v

B
direction of the
magnetic field
direction of the force F
on the positive particle

Figure 4.2.1 The right-hand palm rule is used


to find the direction of a force
acting on a positively charged
particle in a magnetic field.

89

Electrodynamics:
moving charges
and magnetic fields

Try this!
Bending beams of particles
Ask your teacher if they can show you a beam of electrons
in a Crookes magnetic deflection tube (see Figure 4.2.2).
With teacher supervision, bring the north pole of a magnet
close to the front of the tube and observe the effect on the
beam of electrons. By the right-hand palm rule, negative
particles experience a force out of the back of your hand.
Try to predict the direction in which electrons will be
deflected when you place the south pole of the magnet
close to the front of the tube. Now try it and then explain
it to a friend. Do they agree with your explanation?

Figure 4.2.2 A Crookes magnetic deflection tube


produces a beam of electrons.

Checkpoint 4.2
1 Explain why a stationary charged particle experiences a force when you move a magnet past it.
2 Identify the direction in which the proton in Figure 4.2.1a would be moving for it to experience a force into the page.

4.3 The motor effect

Identify that the motor effect


is due to the force acting on
a current-carrying conductor
in a magnetic field.

PRACTICAL
EXPERIENCES
Activity 4.1

Activity Manual, Page


27

90

In the previous section we saw that charges moving in a magnetic field


If a current-carrying conductor is placed in an external
experience a force.
magnetic field, the wire also experiences a force and this is called the motor
effect. This effect occurs because the charges within the wire are travelling
through the magnetic field and experience a force, just as they would if they were
Remember, we are considering
free charged particles (see Figure 4.2.1a).
current to be a flow of positive particles. It is actually the negative electrons
that experience the force from the magnetic field. It works just fine to use
conventional current and consider the force is acting on positive particles to keep
things simple.
We can now use the right-hand palm rule we saw in section 4.2 to work out
the direction of the force on a current-carrying wire in an external magnetic field
To find the direction of the force that acts on a current(see Figure 4.3.1).
carrying wire that is perpendicular to an external magnetic field:
1 Place your open right hand with the fingers pointing in the direction of the
magnetic field (north to south).
2 Place your thumb at right angles to your fingers and in the direction in which
the conventional current is flowing (from the positive to negative terminals
in the circuit).
3 The force experienced by the wire will be directed out of your palm at right
angles to your hand.

motors and
generators
When we allow current to flow through a wire
within the magnetic field in Figure 4.3.1, we see
that the wire moves out of the page, at right angles
to both the magnetic field and the direction of the
conventional current. Each positive particle of the
conventional current within the wire in Figure 4.3.1
experiences a force due to its motion within the
external magnetic field. Since these positive particles
are within the wire, the force acts on the wire.

I
S

direction in which the I


current is flowing

B
direction of the
magnetic field
F

Figure 4.3.1 The right-hand palm

PHYSICS FEATURE

Identify data sources, gather and process


information to qualitatively describe the
application of the motor effect in:
the galvanometer
the loudspeaker.

Loudspeakers

n excellent example of an application of the motor


effect is a loudspeaker. This device is a key part
of telephones, televisions and any other appliance in
which an electrical signal needs to be converted into
sound for us to hear. Figure 4.3.2a shows the most
fundamental parts of a typical loudspeaker labelled
A to D. Figure 4.3.2b illustrates its operation via the
motor effect.
A loudspeaker contains a current-carrying coil (C),
which is commonly called the voice coil. This coil is
wound around a hollow cardboard tube (B) and the
tube is fixed to the cone of the speaker (A). The voice
coil is suspended inside a cylindrical permanent
magnet (D) that provides a uniform magnetic field at
right angles to the coil. An alternating current is passed
through the voice coil, causing the cone to rise and fall
due to the motor effect. Each time the speaker cone
pushes outwards on the air, it creates a wave of
pressure that travels away from the speaker. These
waves of air pressure are sound waves. By varying the
frequency of the alternating current in the voice coil,
the frequency (or pitch) of the sound can be varied.
This means that speakers can generate a variety of
sounds and reproduce sounds recorded elsewhere.
At the moment in time shown in Figure 4.3.2b,
the current is travelling out of the page on the left
side of the voice coil. Using the right-hand palm rule

Figure 4.3.2 A loudspeaker converts electrical energy into sound.


(a) A cut-away section showing the parts of the
loudspeaker and (b) a simplified cross section showing
the direction of the magnetic field

direction of the force


on the positive particle

rule for a current

you can see that a force will be exerted upwards on


the voice coil on this side. This force is exerted in the
same direction around the circumference of the voice
coil and causes the speaker cone to move upwards.
So we can see that now we know about the motor
effect and the right-hand palm rule we can explain
how some everyday things work.
a

B
C

direction of force
on the voice coil

A cardboard cone

B cardboard tube fixed


to cardboard cone and
wrapped in voice coil
C wire coil (called the
voice coil)
D permanent cylindrical
magnet

91

Electrodynamics:
moving charges
and magnetic fields

Quantifying the motor effect


If we place a current-carrying wire within a magnetic field (see Figure 4.3.3) the
force on the conductor is given by:
Solve problems and analyse
information about the force on
current-carrying conductors in
magnetic fields using:
F = BIlsin

F = BIlsin
where F is magnitude of the force on the wire in newtons (N), l represents the
length of the wire inside the magnetic field in metres (m), I is the size of the
current in amps (A), B is the strength of the magnetic field in tesla (T) and is
the acute angle between the magnetic field and the wire.
Note that you can rearrange this equation and make any of the variables the
subject to find their values.

Worked example
Question
If the wire in Figure 4.3.3 has a length of 5 cm within a 0.2 T magnetic field, the wire is at
30 to the field and it contains a current of 0.5 A, what is the force exerted on the wire?

I
B
l

SOLUTION

First we convert the length of the wire into the appropriate units: 5cm = 5/100 = 0.05 m

sin =

Figure 4.3.3

a
l

a = l sin

Then substitute:

F = BIlsin

= 0.2 0.5 0.05 sin30

= 2.5 103 N out of the page

Qualitative analysis of factors affecting the motor effect


Discuss the effect on the
magnitude of the force on a
current-carrying conductor of
variations in:
the strength of the magnetic
field in which it is located
the magnitude of the current
in the conductor
the length of the conductor
in the external magnetic field
the angle between the
direction of the external
magnetic field and the
direction of the length of
the conductor.

92

If you have an equation that describes a relationship, the easiest way to see
how the variables affect the subject of the equation is to place some numbers in
the equation and see what happens.
Remembering that the motor effect is the force F that a current-carrying
conductor experiences in a magnetic field, lets look at how the other variables in
the equation affect the magnitude of F. As you read through this section keep
referring back to the equation and check that you come to the same conclusions.

Magnetic field strength, B


If the value of B was very small, then the right side of the equation would be
multiplied by a very small number. Conversely, if the value of B was very large,
the right side of the equation would be multiplied by a very large number. Since
the magnitude of the force F is equal to the right-hand side of the equation,
F is clearly directly related to B.
To take our analysis one step further, consider multiplying the value of B by 2.
Looking at the equation, we see that if we do this, the effect on the value of F is
We can now say the force F is directly
the same as multiplying F by 2.
proportional to magnetic field strength B; that is, as B increases by some factor
(say 2 times) F also increases by that same factor.
Having understood the analysis above, it should be easy now to see the
following relationships.

motors and
generators
Current, I
As for magnetic field strength, by inspecting the formula we can see
that force F will be directly proportional to current I.
Length, l
Similarly, F is directly proportional to the length l of the currentcarrying conductor within the magnetic field. Be particularly careful to
remember that l is the total length of the wire within the magnetic field.
It is noteworthy that l is regarded as a vector, but current I is not.
Angle,
When the wire is parallel to the magnetic field, the angle is zero. Inspecting
Figure 4.3.4 you can see that if is zero degrees then sin is also zero. When
you substitute zero for sin in the motor effect equation, you see that the
This shows us the interesting situation that the force
force must be zero.
on a current-carrying conductor in a magnetic field is zero when the
conductor is parallel to the magnetic field lines.
When the current-carrying conductor is perpendicular to the magnetic
field, is 90. From Figure 4.3.4 you can see that sin90 is 1. Since 1 is the
maximum value for sin, when we substitute 1 into the equation the force F
will be the maximum value it can be for each set of the other variables.
This shows that the force F is a maximum value when a current-carrying
conductor is perpendicular to the magnetic field B.
Inspecting Figure 4.3.4 you can see that as increases from 0 to 90 the
value of sin increases towards a value of 1. The rate of this increase is not
So we can only say that
constant (i.e. the graph is not a straight line).
force F depends on , as it is not directly proportional to .

Nanotube
Loudspeakers:
No Magnets

group of Chinese researchers has


developed a loudspeaker that
consists only of a thin film of carbon
nanotubes driven by an AC input
signal. The sound generation is
attributed to a thermoacoustic effect.
Changes in the current flowing
through the film are reflected in the
films temperature. Those temperature
changes excite pressure waves in the
surrounding air and these are sound
waves. The film is flexible and can be
stretched and still operate unimpeded.
Perhaps loudspeakers wont have
magnets in the near future!

sin
1

90

180

270

360

Checkpoint 4.3
1 Describe the relative directions of the force, the current and the
magnetic field when a current-carrying wire experiences the
maximum possible force due to the motor effect.
2 Compare the relationships of B, I, l and to F in the equation
F = BIlsin.
3 Explain the motor effect.

Figure 4.3.4 Graph of


versus sin

4.4 Forces between parallel wires


In many applications of electric circuits there are wires bundled tightly together
and running parallel to each other. If we want to explore the interaction of
these wires, we need to bring together two of the facts we have learned so far.
The first is that current-carrying conductors produce a magnetic field.
The second is that a current-carrying conductor experiences a force when inside
a magnetic field. We will also need to apply two of the right-hand rules we have
learned to determine the direction of the magnetic fields and the forces.
93

Electrodynamics:
moving charges
and magnetic fields

Qualitative analysis

Try This!
the motor effect
Take a piece of insulated wire
about 510 metres long.
Stretch it out between two
retort stands so that there are
two pieces of wire running
parallel within a few
centimetres of each other.
Connect the ends to a 12 V
battery and insert a tapping
key switch at one end of the
circuit. When connected
briefly, the currents will run
antiparallel to each other.
Caution: Connect these wires
for a very short time only, as
they carry a large current.
Predict what will happen when
you press the switch. Now
observe! How did you go?

Let us first consider the situation in which we have two parallel current-carrying
wires with currents that are travelling in the same direction.
In Figure 4.4.1a we use the right-hand grip rule to determine the direction of
the magnetic fields around the wires. Now, to understand what is happening to
each wire, we will consider what is happening to one wire at a time.
In Figure 4.4.1b we are looking at what is happening to wire 2. The magnetic
field generated by the current in wire 1 travels into the page around wire 2.
Using the right-hand palm rule, we can see that wire 2 experiences a force
towards wire 1.
In Figure 4.4.1c we now see what is happening to wire 1. The magnetic field
of wire 2 comes out of the page around wire 1. Therefore the right-hand palm
rule shows that wire 1 experiences a force towards wire 2.
The conclusion we can come to is that when two parallel currentcarrying conductors have currents travelling in the same direction, the two
conductors are forced towards each other.

II
F
=k 12
l
d

94

I2

F1

wire 1

wire 2

magnetic fields
around parallel wires

Describe qualitatively and


quantitatively the force
between long parallel currentcarrying conductors:

I1

wire 1

wire 2

magnetic field
due to I1

I1

I2

F2

wire 1

wire 2

magnetic field
due to I2

Figure 4.4.1 Determining the forces on two parallel wires


with currents flowing in the same direction

Now lets consider the two parallel current-carrying wires with currents that
are travelling in opposite directions.
In Figure 4.4.2a the right-hand grip rule shows us the direction of the
magnetic fields around the wires. To understand what is happening to each wire,
we will again consider each wire in turn.
Lets look at what is happening to wire 2 first (Figure 4.4.2b). The right-hand
grip rule shows the magnetic field of wire 1. This field travels into the page
around wire 2. The right-hand palm rule shows that wire 2 experiences a force
away from wire 1.
Figure 4.4.2c shows what is happening to wire 1. The magnetic field of wire 2
goes into the page around wire 1. The right-hand palm rule then shows that
wire 1 experiences a force away from wire 2.
The conclusion we can now come to is that when two parallel currentcarrying conductors have currents travelling in the opposite direction, the two
conductors are forced away from each other.
It may be easy for you to remember the two conclusions above about the
direction of forces on parallel wires, although remembering the result is generally
less important than knowing how you got there. If you forget the conclusions

motors and
generators
a

wire 1

I1

wire 2

wire 1

magnetic fields
around antiparallel wires

I2

wire 2

magnetic field
due to I1

I1

I2

wire 1

wire 2

magnetic field
due to I2

above and you know how to work them out yourself you can never get them
wrong. You will apply similar methods in other problems later in this module.
So if you are comfortable with these methods now, it will be easier later. If at any
time you have trouble using your right-hand rules, come back to the relevant part
of this chapter and revise. You will meet the skills you have used here several more
times yet and each occasion is a chance to test your knowledge.

Figure 4.4.2 Determining the forces on two


current-carrying wires with
currents in opposite directions

Quantifying the relationship


Using our right-hand rules, we have determined that parallel wires exert forces on
each other. To quantify these forces, lets start with an equation we have seen
already. Recall the equation for the force on a current-carrying conductor:
F = BIlsin
For parallel wires, each wire is at right angles to the magnetic field of the other
wire. The sin term in the above equation is therefore equal to 1 (see Figure 4.3.4)
so the equation becomes:
F = BIl
Inspecting this formula we can see that the current I and length l can be
measured relatively easily. To work out F, the size of the force, we now need to
calculate B, the strength of the magnetic field around the wire.
The strength of the magnetic field around a current-carrying conductor
can be determined using the equation:
I
B=k
d
where the proportionality constant k is 2 107 NA2, I is the current in amps,
and d is the distance away from the wire in metres.
Lets consider the situation in Figure 4.4.1b. The magnetic field is being
produced by wire 1, so the current I1 will be used in calculating the magnetic
field strength. The force we are calculating is acting on wire 2, so the current we
should use in this part of the calculation is I2. Combining the two previous
equations and inserting the correct currents in each we get:
F =k

Rearranging this gives:

I1
I l
d 2

II
F
=k 1 2
l
d

where F is the force on each wire in newtons and l is the length the wires are
parallel in metres, so F/l is the force on each metre of wire. k is the proportionality
constant 2 107 NA2, I1 and I2 are the currents in the two wires in amps, and
d is the distance between the two wires in metres.
95

Electrodynamics:
moving charges
and magnetic fields
1.5 A

1.0 A

Worked example
QUESTION
For the situation shown in Figure 4.4.3 calculate the magnitude of the force acting on
each wire.

0.5 m

SOLUTION
2 cm

Figure 4.4.3 Two parallel current-carrying


wires

Feeling
the pinch

he piece of copper
pipe shown in
Figure 4.4.4 was
crushed by lightning.
Just like two parallel
wires carrying currents
in the same direction,
the sides of this pipe
were pulled together
when a current of more
than 100000 amps
was present.

Figure 4.4.4
Dramatic evidence of
parallel conductors
experiencing a force

From Figure 4.4.3, l = 0.5m, l1 = 1.5A, l2 = 1.0A and d = 2/100 = 0.02m.


II
F
Use:
=k 1 2
l
d
I1 I 2
l
Rearrange to make F the subject: F = k
d
Substitute:

F=

2 107 1.5 1.0


0.5
0.02

= 7.5 106 N

Note that as this question asked for only the magnitude of the force, you do not have to
include a description of the direction. If asked, you should add that the force on each wire is
directed towards the other wire.

More qualitative analysis


Our last stop in our look at parallel wires is to analyse the relationships expressed
in the equation:
II
F
=k 1 2
l
d
If we follow the process we used in section 4.3, we can see the following
relationships:
F is directly proportional to the length l. To see this easily we rearrange the
formula to make F the subject. As l increases F increases.
F is also proportional to both I1 and I2.
F is inversely proportional to the distance d between the two wires. This
means that as the distance increases F decreases, or as the distance decreases
F increases.

Checkpoint 4.4
1 Identify the two key facts that explain the interactions of two parallel current-carrying conductors.
2 Describe the interactions of two parallel current-carrying conductors.

96

PRACTICAL EXPERIENCES

motors and
generators

CHAPTER 4
This is a starting point to get you thinking about the mandatory practical
experiences outlined in the syllabus. For detailed instructions and advice, use
in2 Physics @ HSC Activity Manual.

Activity 4.1: The motor effect


Observe the effect of a current-carrying wire that is placed in an external magnetic
field and relate it to the mathematical formula:

F = BIlsin
Equipment: 2 strong horseshoe magnets or ceramic magnets on an iron yoke,
long wire, power supply, retort stand, clamp.

hanging from
a retort stand

Perform a first-hand
investigation to demonstrate
the motor effect.
Solve problems and analyse
information about the force on
current-carrying conductors in
magnetic fields using:
F = BIlsin

flexible wire

ceramic magnets
on an iron yoke

low voltage DC
power supply

Figure 4.5.1 Experiment set-up

Discussion questions
1 Describe the motor effect.
2 Discuss what happened when the current direction was changed.

97

Chapter summary

Electrodynamics:
moving charges
and magnetic fields

Current is the rate of flow of charge through a region,


and in circuits the direction of a current is that of a
positive charge, called conventional current.
A current in which the charges only flow in one
direction is called direct current (DC). A current in
which the charges move back and forth is an alternating
current (AC).
Resistance is a measure of how easily a current flows and
it is defined as the ratio of voltage over current.
Electrical power is the rate at which energy is transferred
within a circuit component.
Electric currents produce a magnetic field around a
wire. The direction of this field can be determined by
the right-hand grip rule.
A magnetic field is depicted by lines with an arrow
indicating the direction in which the north pole of a
magnet points within the field.
The symbols and are used to show the direction of
a magnetic field into and out of the page respectively.

The symbols and are used to show the direction


of a current into and out of the page respectively.
Charged particles moving in a magnetic field experience
a force. When these charges are moving in a wire, the
wire experiences a force called the motor effect.
The right-hand palm rule relates the perpendicular
directions of force, magnetic field and motion in the
motor effect.
A loudspeaker is a great example of an application of the
motor effect.
The motor effect is quantified by the equation
F = BIlsin where the force is proportional to the
magnetic field strength (B), current (I), length of the
wire within the magnetic field (l) and the angle between
the wire and the magnetic field ().
Parallel current-carrying wires experience the motor
effect due to each others magnetic fields and this
II
F
phenomenon is quantified by the equation = k 1 2 .
l
d

Review questions
Physically speaking
1

Across
4 Equation to determine the force between
two current-carrying wires (7, 3)

2
4

3
5

7 emf stands for this (13, 5)


9 Application of the motor effect that
converts electrical energy to sound
10 Quantity related to the energy given
to electrons in a circuit

6
8

11 Force experienced by current-carrying


wire in an external magnetic field (5, 6)

Down
1 Electrons moving in one direction (6, 7)
2 Unit of power

10

11

98

3 The branch of electricity that deals with


moving charges and magnetic fields
5 Rate of use of electrical energy
6 Opposition to the flow of electrons
8 Device that converts electrical energy
to kinetic energy using the motor effect

motors and
generators

Reviewing
1
2

Describe the difference between DC and AC.

Explain why a bird can sit on an electrical power cable


and not get electrocuted.

Describe what is meant by metals having a sea of


electrons.

a Recall the factors that affect the resistance of


a wire.
b State how they affect it.

Given that the definition of power is P = W/t, show


that the equation for electrical power is P = VI.

Describe what happens to a charged particle in


a magnetic field.

Compare the paths of two charged particles entering


magnetic fields. The first is in a constant magnetic
field and the second is in a magnetic field that is
gradually increased.

16 Determine the direction the particle will move when it


enters the magnetic field that is shown.
a

Outline the journey of an electron through the circuit


in Figure 4.1.1, noting the energy transformation.

9 State the motor effect in words.


10 Sketch a graph to show how changing the angle
of the wire in a magnetic field changes the force
experienced.

11 Explain how current can create sound in


a loudspeaker.

12 Draw a labelled energy transformation diagram


of a loudspeaker.

+
b

17 Determine the force on a current-carrying wire


(I =2A) of length 0.5m that is placed in a magnetic
field of 3T.

18 A physics student did an experiment to measure


the force on a wire placed in an external magnetic
field. The field is altered and the results are
recorded below.

Force (N)

Magnetic field (T)

0.05
0.08
0.11
0.17

0.1
0.2
0.3
0.4

a Graph the results.


b Determine the value of the gradient of the graph
and what the gradient represents.
c Given that the length of the wire is l = 0.2m and
the current is I = 2A, comment on the accuracy
of the students results.

solving Problems
13 Calculate the resistance in a circuit that has a battery
supplying 10V and current flow of 2.3A.

14 Determine what happens to current in a circuit when


the thickness of the wire is doubled and the voltage is
increased to four times the original.

15 Using the right-hand palm rule, determine the


direction of the unknown quantity B, I or F.
a

iew

Q uesti o

Re

99

5
electromagnetic induction,
Faradays law, magnetic flux,
magnetic field strength B, magnetic flux
density, emf, perpendicular area, Lenzs
law, law of conservation of energy, eddy
currents, induction cooktops, resistive
heating, eddy current braking

Induction:
the influence
of changing
magnetism
Electromagnetic induction
The discovery of electromagnetic induction was a giant step on the path
to modern technology. Our understanding of this phenomenon required
a great deal of new physics and involved the work of many individuals.
One man, Michael Faraday, led the way with his experimental genius
and intuitive diagrammatic reasoning.
Faraday lacked the mathematical skills to numerically describe his
discoveries, but James Clerk Maxwell took Faradays understanding and
eventually quantified all electromagnetism. Faradays law and Lenzs
law provide us with the tools to explain and predict
eddy currents and to understand their applications.
Later they will help us in our quest to uncover the
secrets of motors, generators and transformers.

5.1 Michael Faraday discovers


electromagnetic induction

Outline Michael Faradays


discovery of the generation
of an electric current by a
moving magnet.

100

Michael Faraday (17911867) was born into a working-class family in London


in 1791. He received little formal schooling and started work at the age of 12.
At 14 he became a bookbinders apprentice and set about educating himself with
the books he was able to access. Over this time he developed a keen interest in
science and began attending scientific lectures. In 1813 he became a research
assistant for the prominent scientist Sir Humphry Davy (17881829). In the
following years Faraday became renowned as one of the greatest experimental
scientists.
One of his numerous experimental discoveries was the phenomenon of
Electromagnetic induction is the
electromagnetic induction in 1831.
generation of an electric current by a changing magnetic field. Faraday showed
that when he moved a magnet near a wire coil, a current flowed within the coil.
He first moved a magnet into one end of a wire coil (Figure 5.1.1a). As he did
this he measured a current in the coil on a galvanometer (a type of ammeter).

motors and
generators
He noticed that this current was only induced while the magnet was moving.
He moved the magnet out of the coil, and this time he measured a current in
Following many
the opposite direction within the coil (Figure 5.1.1b).
other experiments, he put forward his general principle of electromagnetic
induction that a changing magnetic field can cause a current to be generated in
a wire. This change can be caused by either the relative motion of the field and
the coil or by a change in the strength of the magnetic field.
In light of Faradays conclusion, lets offer a simple way to understand his
observations. In Figure 5.1.1 we can consider the change in the magnetic field
through the coil to be represented by the number of magnetic field lines passing
within the coil. In Figure 5.1.1a, notice that the magnetic field around the bar
magnet is not uniform. You see that as you get closer to the poles of the magnet
the magnetic field lines get closer together, indicating that the field is stronger.
Therefore, as the magnet gets closer to the coil more field lines pass within the
coil, indicating that the magnetic field within the coil gets stronger. So, as the
magnet gets closer to the coil the strength of the field within the coil is changing
and this induces the current. This gives us a general explanation for
electromagnetic induction. Now lets take things a bit further.

Faradays law: explaining electromagnetic induction


Electromagnetic induction can be summarised by Faradays law.
The induced emf in a coil is proportional to the product of the number
of turns and the rate at which the magnetic field changes within the turns.
Faradays law is quantified by an equation, and it is very useful to analyse the
equation to understand the relationships involved. This equation is:
= n(B/t)
Let us spend a little time now looking at what each of the variables in this
equation means and then we can understand the relationships.

Magnetic flux B
Magnetic flux is a measure of the amount of magnetic field passing
through a given area. There are two variables that determine the value of
magnetic flux: the strength of the magnetic field B and the area the field is
passing through A. Magnetic flux B is measured in weber (Wb) and can be
expressed by the equation:
B = BA
Magnetic field strength B, a quantity we are already familiar with,
is also called magnetic flux density. This quantity is measured in tesla (T), or
equivalently in webers per square metre (Wb m2). This is a measure of the field
strength per square metre.
A is an area that is perpendicular to the magnetic field lines. If a magnetic
field passes through a circular wire loop of area A (Figure 5.1.2a) and the loop is
at an angle to the field (see Figure 5.1.2b), then the field passes through an
effective area A that is perpendicular to the field and is smaller than area A (see
Figure 5.1.2c). Note that flux could also be calculated using the perpendicular
component of the magnetic field strength, B, and the total area A.

a
N

A
b
N

Figure 5.1.1 The changing magnetic field of


a moving magnet can induce a
current in a coil of wire.

Our suns
magnetic
influence

arge outbursts from the Sun cause


changes in the strength of the
magnetic field at the Earths surface.
This changing magnetic field induces
currents in long metal pipelines and the
wires of power grids, especially at high
latitudes. Currents of 1000 amps have
been measured in pipelines in Alaska,
causing accelerated corrosion. Large
currents in power grids have overloaded
circuits and left millions of people in
the dark for hours.

PRACTICAL
EXPERIENCES
Activity 5.1

Activity Manual, Page


33

Perform an investigation to model


the generation of an electric
current by moving a magnet in
a coil or a coil near a magnet.
Define magnetic field strength B
as magnetic flux density.
Describe the concept of magnetic
flux in terms of magnetic flux
density and surface area.

101

Induction:
the influence of
changing magnetism
This line is the perpendicular
height of area A.
B
A

Figure 5.1.2 (a) A circular coil of area A. (b) A side view of the coil at an angle to the magnetic
field. (c) The effective area of the coil perpendicular to the magnetic field viewed
from point P (b)

The product of the magnetic flux density B and the area A gives a
measure of the total amount of magnetic field passing through that area. This
is the magnetic flux B.
The delta () symbols in Faradays law mean a change in some quantity. So
the two terms with delta symbols attached are differences between an initial
value and a final value. The term B stands for the change in the magnetic
flux and is given by:
B = B final B initial
The t term represents a period of time. This is the period of time over
which the change in flux B is measured.
We can see that the B/t term in Faradays law is actually the rate
of change of magnetic flux, just as acceleration is the rate of change of velocity
(aav=v/t). The term B/t tells us how fast the flux is changing.

emf
The symbol stands for emf, measured in volts. It is the difference in electrical
potential between the two ends of a coil (X and Y in Figure 5.1.3).
This emf creates an electric field within the wire of the coil and a current is
established, provided the circuit is complete (X and Y are not connected in
Figure 5.1.3). A current will flow as long as there is a change in the magnetic
field within the coil.
Qualitative analysis of Faradays law
Lets now look at the relationship between emf and the other terms in the
equation. Recall that the equation for Faradays law is:
= n(B/t)

B
N

X Y

Figure 5.1.3 A moveable magnet passes


through a stationary coil with
terminals X and Y.
102

The emf is proportional to the rate of change of the magnetic flux (B/t).
If there is a large change in flux in a small amount of time, then B/t is
large; therefore, the emf produced will be large. This emf is responsible for the
induced current in a closed loop of wire. If the emf is large, then so is the
induced current. Another way of saying this is that if we wanted to induce a
large current, we would change the magnetic field within the coil as much as
possible in the shortest time possible. To do this in the example we have seen,
we could move the magnet more quickly, use a stronger magnet or make the
perpendicular area as large as possible for the coil.
The emf is also proportional to n, the number of turns in the coil. Again, if
we wanted to create a large current we would want to have as many turns in the
coil as possible.

motors and
generators
By inspecting all the variables in Faradays law we can conclude that
an induced potential difference, and therefore an induced current in a coil,
is proportional to the rate of change of magnetic flux (B/t) and the
number of turns (n) within the coil. Further, we can conclude that induced
currents are produced by changing the magnetic field strength B, the relative
motion between B and the area A, or changing the perpendicular area A of
the coil.
Our next challenge is to find the direction of an induced current, and for
that we need Lenzs law. We will cover this in section 5.2.

Try this!
skipping currents

ake a length of wire about


3040 metres long and
connect the ends to a sensitive
ammeter. Lay the wire out in
a large open space and swing it
like a skipping rope. Can you
induce a current by changing the
size of the loop and using the
Earths magnetic field? Can you
explain why this should work?

Induction without relative motion


Now that we have a basic understanding of electromagnetic induction, we will
look at another example.
In one of his early experiments Faraday wound two coils of wire around an
iron ring (Figure 5.1.4). He noticed that a current was induced in coil B for a
short time after the current in coil A was switched on or off. To explain this
induced current, recall that a current travelling through a wire produces a
magnetic field around the wire. We can then follow similar reasoning to that
in the different situation described earlier. The list of events below explains
why Faraday observed induced currents for a short time after he connected or
disconnected coil A. The steps below are indicated in Figure 5.1.5, which
shows the currents in both coils and the magnetic field produced in coil A.
1 Coil A is connected by closing the switch and a current begins to flow.
2 The current in coil A produces a magnetic field around the iron ring
(see Figure 4.1.9). This field gets stronger as the current increases to
its maximum value. The changing magnetic flux from coil A causes
a changing magnetic flux in coil B, which induces an emf and
therefore a current in coil B. The rate of change of magnetic flux
power
in coil B is positive, rapid at first but then slows down (Figure
supply
5.1.5), so the induced current is positive, high at first but
decreases rapidly.
3 The current and magnetic field in coil A both reach their
maximum value. The rate of change of magnetic flux in coil B is
now zero, so the induced emf and current are also zero.
4 Coil A is disconnected by opening the switch. The current in coil A
decreases rapidly, but does not stop immediately because there is
normally a brief spark in the switch that allows current to continue
flowing briefly.
5 The current in coil A, and therefore the magnetic field it produces,
rapidly decreases. The changing magnetic flux from coil A causes a
changing magnetic flux in coil B that, in turn, induces an emf and
therefore a current. The rate of change of magnetic flux in coil B is
negative, initially high but rapidly decreases. Therefore, the induced
current is negative, high at first but decreases rapidly to zero.
The example above illustrates our previous conclusions that it is the
change in the magnetic field (more precisely, the changing magnetic flux)
passing through a wire coil that induces a current, and the induced current is
proportional to the rate of change of magnetic flux. Now we can see that the
direction of the induced current is determined by whether the magnetic flux
is increasing or decreasing.

Describe generated potential


difference as the rate of
change of magnetic flux
through a circuit.
layers of copper coil
interwound with
cotton and calico

soft
iron
ring

switch

coil B
coil A

Figure 5.1.4 Basic set-up of Faradays experiment

IA

BA

IB
2
1

5
4

Figure 5.1.5 The behaviour of currents and


magnetic fields in Faradays
iron ring experiment
103

Induction:
the influence of
changing magnetism

Checkpoint 5.1
1 Outline Faradays discovery of induction by a moving magnet and summarise his conclusion.
2 Define magnetic flux in terms of magnetic flux density.
3 Using the term magnetic flux, explain why removing a magnet quickly from a coil induces a relatively large current.

5.2 Lenzs law


Heinrich Lenz (18041865) independently made many of the same discoveries
as Faraday. He also devised a way to predict the direction of an induced current
This method
in a closed conducting loop due to a changing magnetic field.
is called Lenzs law and it states that an induced current in a closed conducting
loop will appear in such a direction that it opposes the change that produced it.
This means that the induced magnetic field from a wire loop will oppose the
change in magnetic flux that causes the induced current. Let us look at the
example shown in Figure 5.2.1 to understand this better.
a

direction of
movement

direction of
movement

S
N
N

Figure 5.2.1 (a) Bar magnet moves towards a wire loop.


(b) The magnetic field due to the induced current

As the north pole of the magnet gets closer to the wire loop in Figure 5.2.1a,
the magnetic flux passing through the coil increases. This change in flux induces
a current in the wire loop. Now lets find the direction of the current to agree
with Lenzs law.
The north pole of the magnet is coming towards the coil, so the magnetic
flux pointing downwards through the coil is increasing. Lenzs law says that the
magnetic field produced by the induced current should oppose this change in
flux, so the induced flux should point upwards through the coil. Using the
right-hand grip rule for a wire coil or solenoid, we see that the current would
need to flow in the direction shown in Figure 5.2.1b, to produce an upwardspointing magnetic field within the loop. This field is pointing in the opposite
direction to the changing magnetic flux, so it reduces the changing magnetic flux
from the approaching magnet. Simply, you can think of the interaction of these
two magnetic fields as if the north poles of two bar magnets are facing each
other. The two north poles repel each other, and in this way the induced field
acts to minimise the increasing magnetic flux within the coil.
104

motors and
generators
Now, lets look at what would happen when you move a magnet away from
the loop with its north pole facing the loop (Figure 5.2.2).
In this case the magnetic flux in the coil is decreasing and pointing
downwards. The induced magnetic field should therefore be also pointing
downwards to add to the reducing field and try to minimise the change. Using
the right-hand grip rule, we see that we need a current as shown to produce a
downwards-pointing induced magnetic field. Again, you can think of the
interaction of these two magnetic fields as the interaction of two magnets in
which a north pole is facing a south pole. The two ends of the magnets are
attracted to one another and in this way the induced field acts to add to the
decreasing flux within the coil. These examples leave us with Lenzs law as
a tool to determine the direction of an induced current in a wire coil due to
a changing magnetic flux.

The law behind Lenzs law: the law of conservation


of energy
While we were learning about Lenzs law you may have wondered why the
current needs to produce a magnetic field to oppose the change in flux? Well the
This law states that
answer lies in the law of conservation of energy.
energy cannot be created or destroyed, only converted from one form to another.
In the case of Lenzs law, it is the fact that energy cannot be created that is
important.
If the induced current in Figure 5.2.1 was in a direction that added to the
changing flux through the coil there would be an attractive force on the magnet.
This would mean that the magnets motion would cause it to be pulled through
the coil. The amount of energy you could get from the induced current (heat
from electrical resistance) and the induced magnetic field would be much more
than that put in initially to move the magnet and change the flux through the
coil. This would mean you would be getting something (energy) for free
(without doing any work), which is not possible.
Energy must be ultimately converted from the work done to move the
magnet into heat energy from the electrical resistance within the wire coil.
There must be a balance between the energy that goes into a system and
the energy that comes out. So, whether you push the magnet towards the loop
or pull it away from the loop, you will always experience a force that resists the
motion. This force is an attraction or a repulsion between the magnetic fields
of the magnet and the wire loop.
So far in this chapter we have explained the cause of magnetic induction
using Faradays law, used Lenzs law to find the direction of an induced current,
and the right-hand grip rule to find the direction of this currents magnetic field.
Keep these ideas in mind, as we will apply them in the next two chapters.

direction of
movement

S
N

Figure 5.2.2 Induced current due to a


decreasing magnetic field in
accordance with Lenzs law

Account for Lenzs law in terms


of conservation of energy and
relate it to the production of
back emf in motors.

Checkpoint 5.2
1
2
3

Define Lenzs law.


Describe the induced current and magnetic field in Figure 5.2.2 if the south pole of the permanent magnet is
pointing towards the wire coil.
Justify the current and magnetic field shown in Figure 5.2.1b in terms of the law of conservation of energy.

105

Induction:
the influence of
changing magnetism

5.3 Eddy currents


Explain the production of eddy
currents in terms of Lenzs Law.

We have seen that charges moving in a magnetic field experience a force in


accordance with the right-hand palm rule (sections 4.2 and 4.3). This effect
occurs for free charges and charges within conductors. When a current-carrying
wire experiences a force within an external magnetic field, we call this the motor
effect. When charges within a wire experience a force within a changing
magnetic field, inducing a current, we call this electromagnetic induction.
Many conductors that experience a changing magnetic field and produce an
induced current are much larger than a wire. We call these induced currents
eddy currents.
Eddy currents can be produced by the relative motion of a conductor
and a magnetic field. These eddy currents are small loops of current within the
conductor. They are the same as induced currents in wires subjected to changing
magnetic flux, except that the currents are not confined to a loop of wire. These
currents are set up in accordance with Lenzs law and produce magnetic fields
that act to minimise the change in magnetic flux within the path of the current.
Figure 5.3.1 shows a square piece of copper sheet swinging like a pendulum
through a magnetic field. When this piece of metal moves through the magnetic
field, we notice that there is a braking effect, slowing its swing. After a few
swings it comes to rest. It stops much more quickly than it does when we remove
the magnetic field. To explain this situation we can use the right-hand rules or
approach the problem in terms of Lenzs law. When we use Lenzs law, we use the
right-hand grip rule for solenoids or coils to predict the direction of magnetic
fields and eddy currents.
As the copper square swings into the magnetic field on the left of Figure
5.3.1 lets consider what happens to a positive charge (as shown) on the leading
edge of the square. If this positive charge was within a piece of wire it would
experience a force F1 upwards as shown. If this was a square loop of wire, this
force would generate a conventional current moving anticlockwise around the
loop. As this charge is not confined to a wire, the charge moves upwards initially
and then loops around to form a complete circuit (an eddy current). Using the
right-hand grip rule, we can see that the eddy current (I1) shown would produce

direction of swing

I1

F2 causing
braking effect

F1

F3

I2

+
F4 causing
braking effect

Figure 5.3.1 A square metal sheet is swung through a uniform magnetic field between two bar
magnets. A braking effect is observed due to induced currents and their magnetic
fields, in accordance with Lenzs law.

106

motors and
generators
a magnetic field out of the page (indicated by the N for the north pole of the
currents magnetic field). The flow of positive charges in the direction of F1 is a
current. This current experiences a force due to the uniform magnetic field. This
force F2 opposes the motion of the copper, acting as a braking effect.
As the copper square leaves the magnetic field (on the right in Figure 5.3.1)
Lenzs law tells us that the eddy current should produce a force to slow the squares
departure from the field. The positive charge shown experiences a force F3
upwards, as shown. This causes a flow of positive charges in the direction of F3.
This current experiences a force due to the uniform magnetic field. This force F4
opposes the motion of the metal sheet, again acting as a braking effect.
If you have access to the equipment that demonstrates the situation in Figure
5.3.1, then try observing it for yourself. You may also be able to observe the
effect of cutting slots through the piece of metal swinging in the field. The slots
limit the size of the eddy currents that can be produced and therefore the size of
the induced magnetic fields, and the braking effect is considerably less. We will
meet this idea of reducing the size of eddy currents again in chapters 6 and 7.

Try this!
Racing magnets
Find two identical magnets. Get a piece of copper or aluminium sheet
and a sheet of a non-metal, such as glass, with a surface similar in
smoothness to the surface of the metal. Place the two sheets at the
same angle (say 60) to the table surface and place the magnets at the
same height on each sheet (see Figure 5.3.2). Now predict which
magnet is going to win the race and why. Now race! Did everyone agree?
Explain your observations to a friend.
aluminium or
copper

glass

60

Figure 5.3.2 Which magnet will win?

107

Induction:
the influence of
changing magnetism

PHYSICS FEATURE

PRACTICAL
EXPERIENCES
Activity 5.2

Gather, analyse and present information to explain


how induction is used in cooktops in electric ranges.

Activity Manual, Page


39

Induction cooking

nduction cooktops are a great example of a growing


application of eddy currents. The main appeal of
induction cooking is its efficiency and fast heating.
Heat is not transferred to the pan from a hot plate or
flame in induction cooking. The heat is generated
within the pan itself and then flows into the food
being cooked. This means that minimal heat is lost to
the air before reaching the food, making this much
more efficient than other cooking methods.
The operation of an induction cooktop is illustrated
in Figure 5.3.3. A rapidly changing strong magnetic
field is generated in a large wire coil, using an
alternating current. Both the intensity and direction of
this field change continuously over very short periods
of time. The resulting rapidly changing magnetic flux
within the base of the frying pan induces strong eddy
currents, causing resistive heating.
Resistive heating by eddy currents occurs when the
charges flowing within the metal collide with the ions
in the metal lattice. Kinetic energy is transferred to
the metal ions as vibrations, and this increases
the temperature of the metal. The amount of heat Q
produced by resistive heating is proportional to the
resistance of the material R and the square of the
current I, as shown by Joules law:

Q = Pt = I2Rt
where Q is in joules, power P is in watts or joules
per second (Js1), I is in amps A, R is in ohms ()
and t is in seconds (s).

eddy currents
produced in
base of frypan
ceramic surface
coil supplied with
high frequency AC

AC

rapidly changing
magnetic field
B

Figure 5.3.3 The frypan on an induction cooktop up heats due


to eddy currents.

From Joules law we can see that a large current


and relatively high resistance would result in a large
amount of resistive heating. This explains why
specialised cookware is required to gain maximum
efficiency from induction cooking.
Another method of heat production within
induction cookware is a process called magnetic
hysteresis losses. When a magnetic field is applied to,
and then removed from, a magnetic material such as
iron, a permanent magnetic field remains within the
material. If a magnetic field is then introduced in the
opposite direction to this remnant field, some energy
is expended reducing the remnant field to zero before
the field can build in the other direction. Energy is
dissipated in this process as heat in the material and
therefore also raises the temperature.

Checkpoint 5.3
1
2

108

Explain the formation of eddy currents in a small, flat, square metal sheet that falls between the poles of a magnet.
Describe how Lenzs law can be used to predict the formation of eddy currents.

PRACTICAL EXPERIENCES

motors and
generators

CHAPTER 5

This is a starting point to get you thinking about the mandatory practical
experiences outlined in the syllabus. For detailed instructions and advice, use
in2 Physics @ HSC Activity Manual.

Activity 5.1: Generating electric current


Using the equipment listed, write up an investigation that will allow you to
generate alternating current. Once you have produced alternating current,
investigate how changing the distance between a coil and a magnet, the strength of
the magnet and the relative motion between the coil and the magnet will affect the
electric current produced.
Equipment: coil of wire (transformer coil), galvanometer, magnet (either
electromagnet or a series of permanent magnets).
Discussion questions
1 Describe the relationship between the distance between the coil and the
magnet and the electric current produced.
2 Determine how the strength of the magnet affects the current produced.
3 What effect does making the magnet move instead of the coil and vice
versa have on the current produced?

Activity 5.2: Making use of Eddy currents


Research induction cooktops and check to see if advertised claims about their
efficiency are true. Look at the use of eddy currents in braking. What forms of
transport use it and where could it be applied?
Discussion questions
1 Explain why AC and not DC must be used for an induction cooktop to
work.
2 Discuss the efficiency claims of induction cooktops in comparison to
traditional cooktops.
3 List the advantages and disadvantages of eddy current braking. (Hint: See
Physics Focus on page 112.)

Perform an investigation to
model the generation of an
electric current by moving a
magnet in a coil or a coil near
a magnet.
Plan, choose equipment or
resources for, and perform a
first-hand investigation to
predict and verify the effect on
a generated electric current
when:
the distance between the coil
and magnet is varied
the strength of the magnet
is varied
the relative motion between
the coil and the magnet is
varied.
Plan, choose equipment or
resources for, and perform a
first-hand investigation to
demonstrate the production
of an alternating current.
Gather, analyse and present
information to explain how
induction is used in cooktops
in electric ranges.
Gather secondary information
to identify how eddy currents
have been utilised in
electromagnetic braking.

109

Chapter summary

Induction:
the influence of
changing magnetism

Michael Faraday discovered that a current can be


generated by a changing magnetic field when moving
a magnet within a wire coil.
Magnetic field strength B in tesla (T) is equivalent to
magnetic flux density in webers per square metre
(Wbm2).
Magnetic flux B is a measure of the magnetic field
passing through a certain area. This is equal to the
magnetic flux density B multiplied by the perpendicular
area A through which the field is passing.
An emf produced in a coil is proportional to the rate
of change of magnetic flux and the number of turns in
the coil.

Induced currents are produced by changing the


magnetic flux due to relative motion, changing the flux
density or changing the perpendicular area.
Lenzs law and the right-hand grip rule can be used to
predict the direction of an induced current.
Lenzs law states that an induced current in a closed
conducting loop will appear in such a direction that it
opposes the change that produced it.
Lenzs law can be explained in terms of the law of
conservation of energy.
The production of eddy currents can be explained in
terms of Lenzs law.
Applications of eddy currents include induction
cooktops and eddy current braking.

Review questions
Physically speaking
The theme of this word search is induction.
There is a twist; there is no list provided, so
you have to work out the words that have been
included. Find the 10 hidden words that have to
do with induction, list them and write their
definitions.

110

W N

motors and
generators

Reviewing
1 Outline Faradays experiment that led to the discovery

11 A conductive wire is placed on rails of an electrical


circuit and forced to move to the right, as shown in
the diagram below.

of electromagnetic induction.

2 Recall the factors that affect the size of the induced

wire

emf that is created by induction.

bulb

3 Magnetic flux is a measure of the magnetic field


passing through a certain area. Use this statement to
explain how magnetic flux can be altered.

rails

4 Outline how emf can be used to produce current.


5 Describe the place of relative motion in inducing emf.
6 Explain how Lenzs law supports the law of

a Determine which is the positive and which is the


negative end of the wire.
b Determine the direction of the current in the
circuit.

conservation of energy.

7 Compare induced current in a wire and eddy currents.


8 Explain why a solid square piece of copper swinging
through a magnetic field will slow more quickly than
one with slits in it.

12 Explain how eddy currents can be a problem.


13 Give examples of how eddy currents can be of use.
14 Predict the direction of the eddy currents in the
following examples.
a

9 Compare and contrast the use of eddy currents in


induction cooktops and electromagnetic braking.
(You may need to refer to Physics Focus on
page 112.)

square metal sheet

Solving problems

rotating metal disc


15 Justify the claim that induction cooktops need

10 Using Lenzs law, predict the direction of current

special cookware.

in the following situations. Sketch these diagrams,


showing the induced currents.
a
c
S

expanding wire loop


Y

iew

Q uesti o

Re

111

Induction:
the influence of
changing magnetism

PHYSICS FOCUS
Eddy Currents Stop Me
in My Tracks

rakes are used in vehicles when it is necessary to


reduce speed quickly. When a vehicle brakes,
friction is usually used to convert kinetic energy into
heat. Typically this involves a brake pad made of a
heat-resistant material being forced against a metal
disc attached to a wheel. This leads to wear on brake
components and requires the ongoing cost of
replacement. Some high-speed trains and modern
roller-coasters now use eddy current braking. This
method requires no physical contact between brake
components and therefore no wear and tear.
Some roller-coasters use stationary magnets to
induce eddy currents in the moving roller-coaster.
High speed trains in Europe have electromagnets
fixed to the train, to induce eddy currents in the rails
beneath, as shown in Figure 5.4.1.

N
S

S
N

N
S

S
N

N
S

S
N

coils

N
S

S
N

top of rail

Figure 5.4.1 An eddy current brake in action


A series of current-carrying coils are suspended
a few millimetres above the top of the rail. Their
magnetic fields are produced with a polarity that is
in the opposite direction to the neighbouring coil
(Figure 5.4.1) by alternating the direction of the direct
current within the coil.

112

top of
stationary
rail

coil wound
around soft
iron core

location
of coil

+
I
rail

direction
of trains
motion

Figure 5.4.2 (a) Simplified diagram of eddy current braking system.


(b) Induced eddy currents in rail seen from above

The operation of these brakes is outlined in


Figure 5.4.2. A direct current is passed through the
coil, producing a magnetic field and the soft iron
core intensifies the field. These fields extend into the
iron rail and, as the train moves, the top of the rail
experiences a changing magnetic flux. In accordance
with Lenzs law, this changing flux is opposed
by the production of eddy currents and their
associated magnetic fields (see Figure 5.4.2b).
The braking effect is derived from the force that
each current experiences due to the magnetic field
of the other. For example, the eddy current in the rail
experiences a force due to the magnetic field of
the coil and an equal and opposite force is
experienced by the coil due to the magnetic field
of the eddy current.
The kinetic energy of the train is converted to
heat through resistive heating and magnetic
hysteresis losses. A graph of typical rail heating
is shown in Figure 5.4.3. The adoption of this
technology is not yet widespread, as there are
environmental and structural problems associated
with excessive rail heating. Use is limited to rail lines
with rails that can withstand the temperature
changes without affecting their performance.

Change in rail temperature (C)

motors and
generators

Extension

30

6 Research other methods of applying eddy currents


to braking and compare these methods to the one
outlined above.
7 The kinetic energy lost when braking can be
recycled by regenerative braking systems.
Research these systems and outline their operation.

25
20
15
10
5
0
0

20

40

60

80

100

120

140

160

Braking force (kN)

H4. Assesses the impacts of applications


of physics on society and the environment

Figure 5.4.3 Graph of rail heating


1 Identify and describe the function of eddy currents
in train eddy current brake systems.
2 Calculate the amount of kinetic energy Ek that
must be dissipated as heat to stop a train with a
mass m of 5 104 kg travelling at a velocity v of
300kmh1, using Ek = mv2. (Remember to
convert velocity to the appropriate units.)
3 If typical braking forces of 100 kN are applied,
identify the associated change in track
temperature.
4 Assess the environmental impact of an excessive
rise in rail temperature if eddy current braking was
used on a high speed train in Australia.
5 Identify further research that could be conducted
to allow the widespread adoption of train eddy
current brakes.

H5. Identifies possible future directions


of physics research

Gather secondary information to identify


how eddy currents have been utilised in
electromagnetic braking.

113

Motors: magnetic
fields make the
world go around
The magic of motors

DC motors, coils, armature, rotor,


stator, electromagnets, torque,
commutatorsplitring, commutator
brushes, currentcarrying loop,
galvanometer, back emf, supply emf,
single-phase, three-phase,
AC induction motor, squirrelcage
rotor, shaded-pole induction motor

Much of the activity and infrastructure that makes our lives of


convenience possible is hidden from our view. Electric motors
contribute significantly to our modern existence but remain a
mystery. To most they are magical devices that convert electricity into
motion with the flick of a switch. What better way to appreciate our
comfortable lives than to understand one of the things that drives it.
This chapter will equip you to contemplate how one invention,
the electric motor, contributes to your quality of life in
the age of technology.

6.1 Direct current electric motors


Direct current (DC) motors transform electrical potential energy into rotational
kinetic energy. They can be powered by relatively lightweight batteries and are
therefore easily portable. The most common type of DC motor found in
battery-operated toys is shown in Figure 6.1.1.
These motors typically draw a current from
a number of batteries. This current flows
armature or rotor
commutator brushes
through a wire coil within an external
magnetic field. The coil experiences a force
we know as the motor effect, which causes the
motor to rotate.
The essential components that allow
a DC motor to operate are summarised in
Table 6.1.1.

stator
(curved magnets)

commutator contacts

coils

Figure 6.1.1 Parts of a simple DC motor


114

motors and
generators
Table 6.1.1 Parts of a DC motor
Part

Description and role

Coils

Many loops of wire that carry a direct current. These wires experience
a force (due to the motor effect) that causes the motor to turn when
current is flowing.
This is the part of a motor that contains the main current-carrying
coils or windings. For DC motors this is the rotor, but for AC motors it
is usually the stator.
In a DC motor, the rotor consists of coils of wire wound around
a laminated iron frame. The frame is attached to an axle or shaft that
allows it to rotate. The iron frame is laminated to reduce heating and
losses due to eddy currents. The iron itself acts to intensify the
magnetic field running through the coils.
Stationary permanent magnets (electromagnets in large DC motors)
that provide an external magnetic field around the coils. Permanent
magnets are curved to maximise the amount of time the sides of the
coil are travelling perpendicular to the magnetic field, to maintain
maximum torque.
A device with metal semicircular contacts that reverse the direction of
the current flowing in each coil at every half rotation. This reversal of
the current makes continuing rotation possible in a DC motor.
Conducting contacts (generally graphite or metal) that connect the
commutator to the DC power source.

Armature

Rotor

Stator

Commutator split-ring

Commutator brushes

Describe the main features


of a DC electric motor and
the role of each feature.
Identify that the required
magnetic fields in DC motors
can be produced either by
current-carrying coils or
permanent magnets.

Brushless DC motors

he designs of many modern DC motors are more complex


than the simple example we study in this chapter. The
brushless DC motor has numerous significant advantages, and
is used commonly in computer cooling fans (Figure 6.1.2). The
cylindrical permanent magnets in the rotor have alternating
poles around the circumference. Small electronics switch the
direction of the current in the stator coils. The magnetic field
from the stator coils rotates and this causes the rotor to rotate.

N
S
N

Figure 6.1.2 The brushless DC motor of a computer cooling fan

Now that we have seen the basic structure of a DC motor, we need to turn our
attention to understanding and explaining its operation. In section 4.3 we
encountered the motor effect, but now we will apply it to rotating a currentcarrying coil. Figure 6.1.1 shows that motors contain many coils of wire. For
simplicity we will begin by looking at a single coil and how it would act as a motor.

Lets torque about rotating coils


A torque is the turning effect (or turning moment) of a force. The force
that causes an electric motor to turn is the motor effect, so we must see how to
calculate a torque before we can begin to perform calculations for motors.
The idea of torque is illustrated in Figure 6.1.3a, which shows a force F due to
a person sitting on a see-saw. In this case the force is acting perpendicular to the
The magnitude () of the torque involved is calculated using
see-saw.
the equation:
= Fd

Define torque as the turning


moment of a force using:
= Fd

115

Motors: magnetic
fields make the
world go around

pivot

where torque in newton metres (Nm) is given by the distance d from the pivot
If the force is not acting
in metres multiplied by the force F in newtons.
at right angles to the see-saw (Figure 6.1.3b), it is the perpendicular component
F of the force that is used. As F = Fcos, the formula for torque becomes:
= Fd = Fcosd

b
d

F
cos =
F

Notice that the angle between F and F is the same as the angle the
see-saw makes with the horizontal. We will use this fact again soon. Note that
you could also use = Fd.

Worked example
QUESTION

Figure 6.1.3 A force acting on the end of


a see-saw exerts a torque.
(a) Force at right-angles to
the see-saw and (b) force
angled to the see-saw.

Identify that the motor effect is


due to the force acting on a
current-carrying conductor in
a magnetic field.

axis of rotation

FYZ

FWX

Calculate the torque exerted on the see-saw in Figure 6.1.3b if a force of 980N was acting
at a distance of 4.0 m from the pivot at an angle of 30.

SOLUTION

= Fd = Fcosd

= 980 cos30 4.0

= 3390 Nm

Quantifying torque on a coil


When a current-carrying loop (or coil) is placed in a magnetic field, it is the
force we know as the motor effect that applies a torque on the coil. Figure 6.1.4a
If we know the
shows a current-carrying loop within a magnetic field.
direction of the current in the coil, we can use the right-hand palm rule to give
us the direction of the forces on each side of the coil. Recall from our previous
study of the motor effect that charges moving parallel to the magnetic field do
not experience a force. This explains why only the forces FWX and FYZ act on the
coil between points W and X, and Y and Z respectively, when it is in the position
shown in Figure 6.1.4a. During the coils rotation, forces are experienced by sides
XY and WZ, but they essentially cancel each other out and the coil is not free
to move in these directions. Now lets apply our new understanding of torque to
this coil.
The forces FWX and FYZ apply a torque to the coil, and at the moment
shown in Figure 6.1.4a the magnitude of each torque () is given by:

= F d

b
FWX

WX

rotation

axis of rotation

YZ

FYZ

Figure 6.1.4 (a) The current-carrying coil is


able to rotate about the axis of
rotation (dashed line). (b) The
same coil a short time later has
rotated slightly, as seen from
point A along the axis of rotation.
116

We will use the wire between points W and X as our example and consider
the situation in Figure 6.1.4b in which the coil has rotated. We must now
account for the fact that force FWX is not acting perpendicular to the coil, so we
use the formula seen previously:
= Fd = FWXcosd
As the force FWX is the motor effect, we recall the equation:
FWX = BIlsin
Note that in this equation is the angle we met in section 4.3. It is the
angle of a current-carrying wire to the magnetic field through which it passes.
This is not the same angle as in the cos term of the previous equation. Since the

motors and
generators
two sides of the coil that experience a force (wx and yz in Figure 6.1.4) are
always perpendicular to the field, = 90. This makes sin = 1 and therefore:
FWX = BIl
Combining this equation with = FWXcosd (using the other !) gives:
= BIlcosd
where l is the length W to X (WX) and d is half the length of X to Y (XY/2).
Now we consider the whole coil. The two forces FWX and FYZ are equal and
both make the coil rotate in the same direction. The total torque on the coil is:
total = 2BIlcosd
The product 2ld is the area of the coil A. This gives:
= BIAcos
Motors have more than one coil, so we make one final adjustment to this
expression, giving:
= nBIAcos
where n is the number of turns in the coil.
This formula quantifies the
torque (turning effect) on a rotating coil in newton metres (Nm). It is important
to note that the angle in this equation is the angle of the coil relative to the
magnetic field, as shown in Figure 6.1.4b.

Solve problems and analyse


information about simple
motors using:
= nBIAcos

Worked example
QUESTION

axis of
rotation

The coil in Figure 6.1.5 contains 50 turns and is carrying a 6.0 102 A current in a
5.0 103 T magnetic field. Calculate the torque on the coil if the coil has rotated 30
relative to the field.

= 4.5 105 Nm

cm

cm

SOLUTION
Using = nBIAcos and converting the lengths to metres:
= 50 5.0 103 6.0 x 102 (0.06 0.05) cos30

F
B

Figure 6.1.5

Operation of a simple DC motor


Lets follow the rotation of a coil within a simple DC motor to understand how
motors work. We will analyse the situation shown in Figure 6.1.7. In Figure
6.1.7a the coil of a DC motor is connected to a battery via a split-ring
commutator at A. At the moment shown in Figure 6.1.7a, the two curved metal
contacts of the commutator splitring direct the conventional current around
the coil through points W, X, Y and Z in turn. Figure 6.1.7b shows the coil in
Figure 6.1.7a as seen from the point A along the axis of rotation. The coil is
seen end-on, and we can see the direction in which the current flows along sides
These sides experience a force F that imposes a torque
WX and YZ.
on the coil, causing it to turn. The magnitude of the torque on the coil at
positions 15 is shown on the graph in Figure6.1.7c. Lets now follow this
coil through half a rotation and see what happens to the current and torque
on the coil.
Recall that the torque on the coil in Figure 6.1.7 is given by the equation:

PRACTICAL
EXPERIENCES
Activity 6.2

Activity Manual, Page


48

Describe the forces


experienced by a currentcarrying loop in a magnetic
field and describe the net
result of the forces.

= nBIAcos
117

Motors: magnetic
fields make the
world go around
In this example the values of all the variables on the right-hand side of the
The variation in the cos
equation remain constant, except for the angle .
term accounts for the variation in the magnitude of the perpendicular force F
experienced by the sides WX and YZ. This force determines the amount of torque
acting on the coil, and therefore the torque varies accordingly as the coil rotates.
At position 1 in Figure 6.1.7b, the perpendicular force F on the coil is at a
maximum and therefore the torque is at a maximum. This occurs when is
zero and cos is 1.
At position 2, the coil has rotated 45 with respect to the magnetic field. The
magnitude of the perpendicular force (F) acting on the coil is less than at
position 1. This means that the torque acting on the coil has reduced. At this
point, cos is approximately 0.7. Note that real motors have curved magnets
in the stator to ensure that each coil maintains = 0 as long as possible. This
provides maximum torque for the longest time possible during each rotation.
In the example in Figure 6.1.7, the magnets in the stator are not curved, so
only at positions 1 and 5 is = 0.
At position 3, the perpendicular component of the force F has dropped to
zero. Consequently, there is no torque acting on the coil at this instant. Here
is 90 and cos is 0, so torque is zero. At this point the brushes and splitring
within the commutator have broken contact momentarily and no current
flows in the coil. The current needs to be reversed in the coil at this point, so
that the motor continues to turn for the next half rotation.
Between position 3 and 4, the commutator has reconnected the coil and
reversed the direction of the current flowing in the coil (now flowing through
points Z, Y, X and W in turn). The opposite sides of the split-ring
commutator are now connected to the terminals of the battery. If the current
had remained in the original direction once the motor rotated past point 3,
the forces on the coil would have been in the opposite direction to the initial
rotation. This would stop further rotation.
As the motor moves through position 4, the perpendicular component of the
force F is increasing. This means the torque is increasing and will reach a

Try this!
Model a simple motor

o get a better understanding of


the forces on a coil as it rotates,
make a model of a simple motor.
You need some coat-hanger wire,
a few bamboo skewers, Blu-Tack,
two small magnets, a cork and
paper. Construct a coil as shown in
Figure 6.1.6. Here the green
skewer indicates the magnetic
field. The red skewer points on the
magnets represent the direction of
the force due to the motor effect.
Work with a friend and model the
forces on a coil as it rotates.

Figure 6.1.6 A simple model of


an electric motor
b
axis of
rotation

a
X
F

YZ

WX

c
Position

= 0

WX

Torque
0 max.

YZ
F

= 45

WX

Figure 6.1.7 (a) A simplified model of a DC motor.


(b) The coil as seen from point A in part a,
showing forces and current over a half
rotation. (c) Torque on the coil over a
half rotation
118

= 90

YZ
F

F
YZ
F

YZ

WX
F

= 45

WX
F

= 90

Time

motors and
generators
maximum again at position 5. At position 5 the coil has performed half a
rotation and the pattern we have seen from positions 1 to 5 will repeat until
the coil is back to its original orientation at position 1.

PRACTICAL
EXPERIENCES
Activity 6.1

PHYSICS FEATURE

Activity Manual, Page


42

Galvanometers

he galvanometer was developed in the 1800s to measure the


relative strength and direction of electrical currents. Its
descendant is the analogue ammeter (Figure 6.1.8), which is
calibrated in units of amperes and its basic structure is shown in
Figure 6.1.9. In an ammeter, the coil is connected in parallel to a
low-resistance wire within the ammeter. When a current is passed
into the ammeter, only a small amount of the total current flows
through the coil and this is proportional to the total current.
When a current passes through the coil shown in Figure 6.1.9,
it experiences a force due to the motor effect. This force exerts a
torque on the coil, causing it to rotate around the pivot. A spring
provides resistance to the torque, and the needle comes to rest
when the torque on the coil equals the torque from the spring. The
torque on the coil is proportional to the current passing through
the coil. Therefore the amount the coil and needle move indicates
the size of the current.
To ensure that the needle moves (or deflects) by the same
amount with each ampere of current being measured, the magnets
are curved around the coil and iron core. This ensures that the
magnetic field is always perpendicular to the flow of current along
the sides of the coil. A uniform maximum torque will then be
experienced by the coil through its whole range of movement.
This means that the scale you read on the ammeter can be uniform
(i.e. the scale has the same-sized divisions throughout).

Identify data sources, gather and process


information to qualitatively describe the
application of the motor effect in:
the galvanometer
the loudspeaker.

Figure 6.1.8 A moving coil ammeter

pointer

force

permanent magnet

spring

coil

pivot

moving
coil

soft iron
core
magnetic field

Figure 6.1.9 Cross-section of a moving coil


galvanometer

Checkpoint 6.1
1
2
3

Explain how each part of a DC motor contributes to its operation.


Define torque and explain how torque varies during the rotation of a DC motor.
Compare the features of a DC motor and a galvanometer.
119

Motors: magnetic
fields make the
world go around
B

Current (A)

motor current
0
0

Time (ms)

Figure 6.2.1 Graph of net current for a DC


motor averaged over many cycles

Account for Lenzs Law in


terms of conservation of energy
and relate it to the production
of back emf in motors.

6.2 Back emf and DC electric motors


We now know that when a current is applied to the coil of a motor, the coil
experiences a force. This force exerts a torque on the coil and the rotor begins
to rotate. If we continue to apply the same current to the coil, the net force on
the coil continues increasing the motors speed (recall Newtons second law).
This may make you wonder why a motor doesnt just keep speeding up. We
know from experience that they dont because our toys with motors dont
accelerate forever.
The main reason DC motors reach a maximum operating speed is that a
Back emf is a
back emf is generated in the coil as the motor rotates.
potential difference across the terminals of the motor created by the changing
magnetic flux passing through the wire coils within the motor (see section 5.1).
Look back at Figure 6.1.7 for a moment. At position 1 the magnetic flux
B through the coil is zero. At position 2 in Figure 6.1.7b, the magnetic field
is pointing right to left through the coil and the magnetic flux is increasing.
This tells us that there should be an emf induced that would produce a
magnetic field pointing from left to right to reduce the increasing magnetic
flux. This field would be produced by a current flowing through Z, Y, X and
W in turn. We can see from Figure 6.1.7a that this would be a current that
opposes the one generated by the supply emf (through W, X, Y and Z in
turn). The result of this would be that the net current in the coil would be less
than the supply emf could generate. Another way to think about it is to recall
the equation for Faradays law:
= n(B/t)

Try this!
a back emf
Connect a DC motor to a
battery and place an ammeter
in the circuit to measure the
current flowing through the
motor. Predict how the amount
of current flowing will change
if you apply a significant load
on the motor. Try it! How did
you go?

Explain that, in electric


motors, back emf opposes
the supply emf.

120

We can see that this changing flux B over time t would generate a
potential difference or emf .
This potential difference (back emf ) is in the opposite direction to
the applied potential difference (supply emf) that causes the rotor to turn. As
the speed of the motor increases the back emf increases, as B/t increases.
Eventually this potential difference cancels out most of the applied potential
difference and virtually no current flows through the coil. At this point there
is no net torque acting on the rotor and it turns at a constant speed.
Let us analyse this situation using Figure 6.2.1. At time A on the graph,
the motor is connected to a DC power source. The applied potential
difference causes a current (blue line) to flow in the motors coil and this
builds quickly to its maximum value. Once the coil starts turning, a back emf
is generated due to the changing flux within the coil. This back emf is in the
opposite direction to the applied potential difference and therefore reduces
the net potential difference, which in turn reduces the current flowing in the
coil as the speed of rotation increases.
At time B, the motor has reached its maximum rotational speed. Here
most of the supply emf has been cancelled out by the back emf. If the motor
has no load attached to it, only a small current continues to flow. This residual
current is required to overcome any friction within the motor and any voltage
drop due to losses such as resistive heating. There is no net torque on the coil
between times B and C and the motor operates at a constant speed.

motors and
generators
At time C, a large load is applied to the motor, such as the motor turning
a wheel to move a toy car. The motor slows down quickly under this load and
the amount of back emf is reduced. This means that the applied potential
difference is greater and therefore a larger current flows through the coil.
At time D, a larger current continues to flow through the coils. If the
motor is not designed to handle the resistive heating produced by this larger
current, the motor may burn out. Burn out occurs when insulation melts at
high temperature, and may cause other components to melt. The motor will
then cease to operate efficiently.

Back emf
measures
motor speed

Checkpoint 6.2
1
2
3

Define the term back emf.


Describe the relationship between back emf and supply emf during
the operation of a motor.
Analyse the production of back emf in terms of Lenzs law.

n some motors back emf is


used to measure their speed.
The input voltage is turned off
for a short amount of time and
the back emf is measured. Since
the back emf is proportional to
the speed of the motor, a
calibration can be applied and
the speed can be calculated.

6.3 Alternating current electric


motors
Many large appliances in your home contain motors that use single-phase
240V alternating current (AC). Single-phase AC was illustrated in Figure 4.1.2
as a blue curve, showing a current changing direction many times a second. To
connect an appliance to the mains power, you insert a plug (Figure 6.3.1) into
the wall socket. This allows single-phase AC to be supplied by the active pin and
the circuit to be completed by the neutral pin.
Some industrial motors require more power and torque than can be
supplied by single-phase AC. These motors are typically supplied with threephase 415V AC by a plug with four or five pins. Figure 6.3.2 shows the AC
signal available for three-phase equipment. This involves three alternating
currents that can be applied to different coils within a motor at any one time.

active

neutral

earth

Current

+
0

Time

Figure 6.3.1 A typical plug for an electrical


appliance in Australia

Figure 6.3.2 Each of the three AC in


three-phase power is 120
out of phase with the others.

121

Motors: magnetic
fields make the
world go around
AC motors have the same basic components as DC motors but lack the
need for others. They all contain a rotor and stator. Their magnetic fields can be
generated by current-carrying coils. They utilise the motor effect to transform
electrical potential energy into rotational kinetic energy. Lets explore examples
of the AC induction motor that exist both in industry and in the home.

Three-phase AC induction motors


PRACTICAL
EXPERIENCES
Activity 6.3

Activity Manual, Page


51

The simplest induction motor is the three-phase AC induction motor


(Figure6.3.3). These motors are used in industry for their efficiency and
In Figure 6.3.5a,
reliability. Three-phase AC is fed to the coils in the stator.
each pair of magnetic poles in the stator is fed one phase of the AC signal.
The peak current of each phase is reached sequentially around the stator
(Figure 6.3.5b), creating a magnetic field (the stator field) that rotates.

electromagnets

rotor

Figure 6.3.3 In this cutaway image of a three-phase AC induction


motor, you can see the stator, consisting of electromagnets
arranged to form a hollow cylinder. Within the stator sits
the rotor, which is mounted on the motors shaft.

Universal motors

he most common type of motor in appliances


around the house is the universal motor. This
type of motor is capable of using both AC and
DC. Figure 6.3.4 shows that coils act as
electromagnets in the stator of these motors.
These provide stronger fields and are lighter
than permanent magnets. The coils are
connected in series with the rotor coils and use
a single-phase alternating current. These motors
are found in many household appliances in
which a variable speed is required, such as drills
and blenders.

armature coils

carbon brushes
shaft

stator coils
segmented
commutator

spring-loaded
brush holders

Figure 6.3.4 A typical universal electric motor, showing the main components.
Some motors would have additional stator coils. The commutator
feeds current to the armature coils in the position where most
torque will be experienced.

122

motors and
generators
a

conducting bars

Current

+
0

Time

i
rotor

ii

iii
2

end ring

Figure 6.3.5 (a) In a three-phase motor, as the current in each pair of opposite

coils peaks, the field appears to rotate, dragging the rotor around
with it. (b) A squirrel cage rotor. The rotor is made of iron
laminations to cut down undesirable eddy currents. The induced
currents flow lengthwise in copper or aluminium rods which are
joined at the ends (as in a squirrel cage).

stator pole

These motors contain a squirrel cage rotor (Figure 6.3.5b) that does not
Around the circumference of these
require the input of an external current.
rotors are a number of parallel conducting bars. These bars are joined at the ends
by an end ring that allows current to flow from one bar to another. The rotating
stator field induces a current in these bars and a magnetic field is induced in
accordance with Lenzs law. This induced magnetic field interacts with the rotating
field from the stator and the resulting forces cause a torque on the rotor. This
causes the rotor to spin without the need for a commutator as in a DC motor.
As these motors do not need brushes and therefore have fewer moving parts, they
are more efficient and more reliable.
Another way to understand the operation of an AC induction motor is to
consider a positive particle within one of the conducting bars in the stator.
Lets consider the bar marked A in Figure 6.3.6a. As the rotating magnetic field
moves upwards past bar A this is equivalent to the bar moving downwards in a
stationary magnetic field. Figure 6.3.6b shows this equivalent situation in which
bar A moves relative to the magnetic field. Using the right-hand palm rule
(see section 4.2) we see that a positive particle in bar A would experience a force
into the page. This is equivalent to a current being induced into the page in bar A.
Now we must use the right-hand palm rule (see section 4.3) to deduce the direction
of the motor effect on a current-carrying conductor. This indicates that a force is
exerted upwards on bar A and this is in the same direction as the rotating field in
Figure 6.3.6a. This force on bar A is the same as the force experienced by each bar
as the magnetic field rotates. These forces, in the same direction as the rotating
stator field, exert a torque on the rotor and are responsible for its rotation.
a

stator

force on bar A due


to motor effect

B
squirrel cage
conducting bars

rotating
magnetic
field

B
I
motion of bar A
relative to the
magnetic field

Figure 6.3.6 (a) The magnetic field due to


one phase of an AC inductor
motor. (b) The force acting on
squirrel cage rotor bar A in
part (a) due to the rotating
magnetic fields
123

Motors: magnetic
fields make the
world go around

Single phase AC induction motors

Try this!
Induction exerts forces
Suspend a small piece of
a lightweight conductor (e.g.
aluminium foil) from fishing line.
Use your right-hand palm rule to
predict the force acting on the foil
as you move a strong magnet
vertically past the foil. Can you
observe any movement? If not,
use the scientific method to
discover why its not working
and try again.

Describe the main features of


an AC electric motor.
Gather, process and analyse
information to identify some of
the energy transfers and
transformations involving the
conversion of electrical energy
into more useful forms in the
home and industry.

A more complicated, but widely used, AC induction motor is the shaded-pole


AC induction motor shown in Figure 6.3.7. These motors require only a single
phase of AC and can be found in most household electric fans. In these motors
the alternating current is passed through a coil wrapped around the soft iron
casing. The stator field induced by the alternating current passes through the
casing and through the squirrel-cage rotor. Figure 6.3.7a shows the rotor
removed and leaning on the motor. A cross-section of a squirrel cage rotor is also
shown (Figure 6.3.7b) and clearly shows the conducting bars within the rotor.
Four small copper shading rings can be seen within the stator in Figure
6.3.7a. These are inserted into the stator on each side of the rotor on opposite
poles. The currents induced in these shading rings in accordance with Lenzs law
act to delay the magnetic flux passing through the rotor. This produces an
asymmetric magnetic field passing through the rotor shown in Figure 6.3.7c.
This leads to a changing magnetic field in each cycle of the alternating current
that sweeps across each pole of the stator. The sweeping change in magnetic field
strength across the rotor is essentially the same as the rotating magnetic field we
studied in the three-phase AC induction motor. This rotating magnetic field
causes a torque on the rotor in the same way as outlined for the three-phase
induction motor.
b

conducting bar

squirrel cage rotor


shading rings

rotor
shading rings delay
the phase of part of
the field to produce
a rotating field

thick, copper
shading ring

Figure 6.3.7 (a) A shaded-pole AC induction motor taken from a small household fan, with
(b) a cross-section of the rotor. The conducting bars can be seen clearly within
the laminated rotor. (c) The principle of a simple single-phase, shaded pole
induction motor. The distorted (or shaded) field causes the rotor to turn in
one direction in preference to the other.

124

motors and
generators

The right motor for the job


Now that we have seen a few examples of some common electric motors, lets
consider for a moment why they are chosen for their common applications.
The initial price and operating cost are key factors in making a decision.
Operating costs depend on factors such as energy efficiency and replacement of
parts due to wear and tear. The amount of torque required and how often a
motor will be put under load are also critical factors. These will determine how
much current is needed, the strength of the magnetic field required and
numerous other parameters. Of course there are other constraints including
portability, size and weight to consider. All these factors lead to a lot of
homework for an engineer who is trying to design a machine or appliance
driven by a motor.
A summary of the characteristics of the different types of motors discussed
in this chapter is provided in Table 6.3.1. Can you see why they are used in
their common applications?
Table 6.3.1 Characteristics of motors
Type

Advantages

Disadvantages

Common applications

Simple (brushed) DC
motor

Efficiency (%)
4090

Low cost, battery powered,


speed easily controlled

Toys, power tools, treadmill


exercisers, automotive
starters

Brushless DC motor

3090

AC universal motor

4060

Three-phase AC
induction motor

7090

Single-phase (shadedpole) AC induction motor

2035

Long working life, low


maintenance, high efficiency
High starting torque,
compact design, high
running speeds
High starting torque, high
power, high efficiency, good
power to weight ratio
Inexpensive, long working
life, high power, multi-speed

Short working life, high


maintenance (brushes),
sparking and ozone
production
High cost of some designs,
requires a controller
Less efficient than
equivalent DC motor
Requires three-phase power

Inefficient, low starting


torque

CD and DVD players,


computer hard drives
Blenders, vacuum cleaners,
hair dryers, portable power
tools, sewing machines
Industrial machinery, pumps
and compressors
Fans

Checkpoint 6.3
1
2
3

Compare the features of an AC induction motor and a simple DC motor.


Construct a flow chart to account for the operation of a three-phase AC induction motor.
Justify the choice of a three-phase AC induction motor for use in industrial machinery.

125

Motors: magnetic
fields make the
world go around

PRACTICAL EXPERIENCES
CHAPTER 6

This is a starting point to get you thinking about the mandatory practical
experiences outlined in the syllabus. For detailed instructions and advice, use
in2 Physics @ HSC Activity Manual.

Activity 6.1: Applications of the motor effect


Identify data sources, gather
and process information to
qualitatively describe the
application of the motor
effect in:
the galvanometer
the loudspeaker.

Discussion questions
1 Outline how the motor effect is used to make music in a loudspeaker.
2 Explain how a loudspeaker differs from a motor in its use of the
motor effect.
3 Determine the difference between the way in which the motor effect
is used in a loudspeaker and in a galvanometer.

armature

Part A: Make a loudspeaker and determine how the motor effect is used to make
it work.
Equipment: two horseshoe magnets, strong sticky tape, thin insulated wire,
cardboard, power supply, alligator clips and wires.
Part B: Make a working galvanometer and determine the differences in this
application of the motor effect.
Equipment: PVC-covered copper wire (150 cm) with bare ends, wooden base
board, armature block, magnets, split pins, knitting needle, rivets, wire
strippers, drinking straw, rheostat (1015 ohms, rated at 5 A or more).

brush

Activity 6.2: Motors and torque


commutator

Figure 6.4.1 A simple motor

Solve problems and analyse


information about simple
motors using:
= nBIA cos

Make a motor like the one shown and note what factors change its performance.
Calculate the torque of your motor.
Equipment: insulated wire, magnets, magnetic field sensor and data logger
(if available), paperclips, Blu-Tack, connecting wires with alligator clips,
power supply.
Discussion questions
1 Investigate the factors that determine the effectiveness of the motor.
2 Calculate the amount of torque in your motor and list ways in which
torque can be increased.

Activity 6.3: AC induction motors


Perform an investigation to
demonstrate the principle of
an AC induction motor.

Using the equipment supplied, make a model of an AC induction motor and relate
each part to the parts in a real AC motor.
Equipment: aluminium foil, fishing line, retort stand and clamp, ceramic
magnet.
Discussion questions
1 Outline how the metal is made to move.
2 Explain why the AC induction motor is so efficient.

126

Chapter summary

The main features of a DC motor are the rotor (coils,


shaft and frame), stator (permanent or electromagnets),
commutator splitring and commutator brushes.
The roles of these components are summarised in
Table 6.1.1.
Torque is the turning effect of a force F. = Fd where
d is the distance from the pivot to the point where the
force is applied.
A current-carrying loop in an external magnetic field
experiences forces due to the motor effect that generate
a torque.
The torque on a current-carrying coil can be
quantified by the relation = nBIA cos where n is the
number of turns in the coil, B is the strength of the
magnetic field, I is the strength of the current in the
coil, A is the area of the coil and is the angle between
the plane of the coil and the magnetic field lines.
The torque on a coil in an external magnetic field varies
as the coil rotates. The torque is at a maximum when
the plane of the coil is parallel to the magnetic field

motors and
generators

and zero when the coil is perpendicular to the


magnetic field.
The current-carrying coil of a galvanometer experiences
a torque due to the motor effect. This torque is balanced
by a spring and this causes the needle to deflect by an
amount proportional to the current flowing. This allows
the determination of the magnitude and direction of the
current being measured.
When a DC motor rotates, its coils experience an
induced emf (back emf ) set up in accordance with
Lenzs law. This back emf opposes supply emf and
reduces the current flowing through the motors coils.
AC induction motors contain a squirrel-cage rotor
(conducting bars, shaft and frame) and stator
electromagnets.
AC induction motors generate a rotating magnetic field
that induces currents in the squirrel-cage rotor. These
current-carrying conductors in the rotor experience
a force due to the motor effect, which exerts a torque
on the rotor.

Review questions
Physically Speaking

Reviewing

The terms in the following list belong to two distinct groups.


Group these terms into the two groups and add the definition
of each term. Create a diagram to display the relationship
between them.

1 a Identify the type of motor in Figure 6.4.2.

Split-ring commutator
Squirrel cage
Brushes
Stator

b Identify each of the labelled features.


c Construct a table to list the parts you have
identified and the role each plays.

2 Explain why radial (curved) magnets in a motor allow

Fan
Bearings
Armature
Magnets

for greatest efficiency.

3 Describe the differences and similarities in the way


permanent and current-carrying coils produce
magnetic fields within a DC motor.

D
E

4 Define a galvanometer and outline what it is used for.


5 A galvanometer has a spring attached to the centre
of it, distinguishing it from a simple DC motor. Give
reasons for its presence.

6 Recall Lenzs law and explain how Lenzs law accounts


for the conservation of energy.

Figure 6.4.2

7 Describe what is meant by back emf.


8 Determine how back emf is produced in a motor.
9 Explain what a manufacturer does to a motor to
account for back emf.

127

Motors: magnetic
fields make the
world go around

10 a Label the parts of the AC motor in Figure 6.4.3.


b Explain what you could do to this motor to make it
into a DC motor.
B

18 Students undertook to measure the torque produced


by a simple DC motor.

The motor contained 100 turns and the square


armature was 0.03m in length. The motor is attached
to a piece of string holding a mass (Figure 6.4.4).

D
A

motor

motor shaft

r
edge of table
string

mass

Figure 6.4.3

11
12
13
14

Figure 6.4.4

Give examples of where an AC motor would be used.


Compare and contrast DC and AC motors.

The motor was turned on and allowed to wind up the


mass until it stalled and stopped. At this point the
radius of the windings r was recorded. The current
supplied to the motor was gradually increased and
the process repeated.

Explain how an AC induction motor works.


Explain how a single-phase induction motor gets
started.

15 Distinguish between situations in which AC universal


motors and AC induction motors would be best
suited.

The table of results is shown below.


solving Problems
16 Calculate the maximum torque that is generated by a
force of 460N applied to an object at a distance of
3m from its axis of rotation.

17 Calculate the torque in a square coil with sides of


length 3cm. The current in the coil is 2A and it is
placed in a magnetic field of 0.3T.

Re

128

Q uesti o

a
b
c
d

iew

= F d = mg d

Mass = 0.5kg

Torque (Nm)

Current a

Radius (m)

0.27

0.1

0.055

0.54

0.2

0.11

0.81

0.3

0.165

1.08

0.4

0.22

1.35

0.5

0.276

1.62

0.6

0.331

Draw a graph of torque versus current.


Determine the gradient of the line.
What quantity does this value represent?
From this value, determine the magnetic field
in which the armature is spinning.

motors and
generators

PHYSICS FOCUS
Linear motors

igure 6.4.5 is of a maglev traina train that floats


on its rail and moves at very high speeds. You will
study the floating mechanism in detail in Module 3
From Ideas to Implementation, but lets have a look
at how the train actually moves forward.
The maglev train operates solely on electric power.
The propulsion method is via an electric motorbut
one with a differenceit is a linear motor. This is, in
principle, an electric motor that has been unwrapped
and flattened. Magnetic fields on the train and rail
are continually created to attract and repel each other.
It is these interactions that apply the forces to propel
the train forward.
Linear motors produce motion by moving in a
straight line rather than the traditional rotational
motion. There are two main parts: the stator unrolled
(the primary) and laid flat on the rail, and the
secondary, which is the glider on the train that floats
over the rail.

3. Applications and uses of physics

Why use this type of motor? Reasons include:


It has no moving parts so there is no wear and tear.
The train rides on an air cushion, so less energy is
lost due to friction.
Electromagnets are used for braking, so the train
is a lot quieter.
1 Outline how a DC motor works.
2 Outline how an AC induction motor works.
3 Compare the uses for an AC induction motor with
that of an AC universal motor.
4 Draw a diagram of a DC motor and explain what
can be done to it to make it an AC motor.
5 Determine how torque is calculated in a motor.
6 Justify the use of linear motors in such applications
as the maglev train.
7 Evaluate the cost associated with maglev trains and
standard trains.

Research
8 Find out exactly how a linear motor works.
9 Compare the torque produced in a standard motor
with the linear force in a linear motor.

Figure 6.4.5 A linear motor propels this maglev train.

129

Generators and
electricity supply:
power for the
people
Technology that changed our lives

generator, transformer, step-down


transformer, step-up transformer,
flux leakage, magnetic hysteresis,
power stations, substation,
transmission towers, insulators,
lightning protector

Widespread access to affordable electricity has arguably been the


single greatest catalyst for change in modern society. It has had an
impact on every aspect of our lives, from our health to our wealth and
even what we do in our leisure time. Some key developments that
enabled this revolution were the invention of the AC generator
and the transformer. Their significance is far beyond
the reaches of the power lines they service and, as
we shall see, they involve more than meets the eye.

7.1 AC and DC generators

Permanent magnets
or electromagnets
provide an external
magnetic field.

In chapter 6, we saw that the real value of electric motors was that they convert
electrical potential energy into rotational kinetic or mechanical energy. A logical
question that follows is how do we get the electrical potential energy that turns
these motors?. For many of our household applications, the answer is we use
a generator. When you switch on an electrical appliance at home, you may not
realise where the energy comes from. In some distant power plant there is a huge
rotating machine as big as a building that looks like a huge electric motor. The
difference is that this generator is producing electricity instead of using it.
The simplest AC motor design is the synchronous AC
motor (Figure 7.1.1). If this motor was supplied with 50Hz
AC from the power point, it would spin at 50 revolutions
per second in synchronisation with the electrical signal. This
S
design has no practical application, but more complicated
designs are used in clocks and tape drives, due to their
constant speed.

AC supply
Slip-ring commutator continuously
connects the rotating coil to the AC supply.

130

Figure 7.1.1 A synchronous motor uses a slip-ring commutator


to feed AC to the motor.

motors and
generators
Although it lacks applications as a motor, this design would produce an
electric current if you turned the coil with your hands. In this way it is acting as
When we turn the motor ourselves, its coil experiences a
a generator.
changing magnetic field and an emf is induced (recall Faradays law). If the loop
is connected by a circuit, a current flows, and we have turned the motor into
a generator.
The parts of a generator are essentially the same as those of a motor (see
Table 7.1.1). The difference is that we physically turn a generator and it produces
electrical potential energy. So the operation of a generator is the opposite of that
of a motor. In this section we will only consider simple generators; we will see a
more complicated version in section 7.3.

Describe the main components


of a generator.

Table 7.1.1 The parts of a simple generator and their function


Part

Description and role

Armature

The armature is the part of a generator that contains current-carrying coils. These carry an induced
current caused by a changing magnetic field. For simple generators these are the coils in the rotor, but
in other generators the armature is in the stator.
These are the many loops of wire that carry electrical current. In many generators there are two sets of
coils. One set is the electromagnets that provide the magnetic field (in simple generators this field is
provided by permanent magnets). The other set is in the armature and these electromagnets carry the
current produced by the generator.
The rotor generally consists of coils of wire wound around a laminated iron frame. The frame is attached
to an axle or shaft that allows it to rotate. The iron frame is laminated to reduce heating and losses due
to eddy currents. The iron itself acts to intensify the magnetic field passing through the coils.
In simple generators the stator is the stationary permanent magnets or electromagnets that provide an
external magnetic field around the rotor. These magnets are curved to maximise the amount of time the
sides of the rotor coil are travelling perpendicular to the magnetic field. In some generators the stator
contains the armature coils and a magnet turns as part of the rotor to produce a changing magnetic field.
A simple DC generator contains semicircular metal contacts (a split ring) that reverse the direction of the
current flowing out of the rotor coil every half rotation. This reversal of the current ensures that the
current being produced is DC.
A simple AC generator has two circular metal contacts. Each slip ring is connected to one end of the
coils in the rotor. These provide an alternating current that changes direction every half rotation. In more
complicated examples these provide a current to an electromagnet in the rotor and a current is produced
in the stator.
Brushes are conducting contacts (generally of graphite or metal) that connect the commutator to the
external circuit.

Coils

Rotor

Stator

Commutator
split ring
Commutator
slip ring

Commutator
brushes

Figure 7.1.2a shows a typical hand-operated generator found in schools. This


generator has the features of both an AC and a DC generator. With the flick of
a switch you can connect to the components for either type of generator and
produce a current by winding the handle. If you have access to one of these make
sure you try it out and try to identify all its parts.

A simple AC generator

PRACTICAL
EXPERIENCES
Activity 7.1

Activity Manual, Page


54

Figure 7.1.3a shows a simple model of an AC generator. This generator


therefore contains a slip-ring commutator that simply connects each end of the
rotating coil to an external circuit. The rotor of the generator shown is being
turned by hand. Lets follow a full rotation of this generator as shown in Figure
7.1.3b and use the graph (Figure 7.1.3c) to gain an understanding of its
operation. Figure 7.1.3c shows the amount of flux B passing within the coil
WXYZ. It also shows the induced emf caused by the rate of change of flux
B/t in accordance with Faradays law (see section 5.1). This emf would result
in a current flowing within the coil, if the output terminals were connected.
131

Generators and
electricity supply:
power for the people

slip-ring commutator
to produce AC

slip-ring commutator
to produce DC

S
coils or
windings

rotor and
armature

stator
magnets

output terminals

Figure 7.1.2 (a) A typical AC and DC generator found in high schools and (b) a close-up
showing the main parts of this generator (see Table 7.1.1 for details)

At position 1 (Figure 7.1.3b), the plane of the coil WXYZ lies parallel to the
magnetic field. This means that no magnetic field lines are passing within
the coil, so the amount of magnetic flux B within the coil is zero. Recall
Faradays law:

Try This!
Modelling generators
Use the model of a simple
motor suggested in section 6.1
to model the operation of a
generator. Add slip-ring and
split-ring commutators by
sticking pieces of aluminium
foil around the cork. Ensure
you can describe and explain
the current that would be
produced during a full rotation.

ractiv

nte

M o d u le

132

= n(B/t)
which shows that the emf induced in the coil is proportional to the rate of
change in magnetic flux B/t passing within the coilnot the flux value
itself. At position 1, the emf is at a maximum because the rate of change of the
magnetic flux is at its maximum. This can be seen in Figure 7.1.3c in which
the tangents to the curve (illustrating the slope of the curve) for the magnetic
flux B represent the rate of change of magnetic flux B/t. The tangent at
position 1 is at the maximum positive value and, therefore, so is the induced
emf, according to Faradays law.
If the output terminals of the generator are connected, the induced emf
in the coil will cause a current to flow. At position 1, this occurs in the
direction marked (i.e. through Z, Y, X and W in turn). To determine the
direction of this current, lets consider a positive charge in the wire between
points W and X at position A. The right-hand palm rule shows that a charge
moving upwards at position A, as the coil rotates within the magnetic field,
will experience a force towards point W. You could also determine the direction
of the current using Lenzs law. Review Lenzs law and then give it a try.
At position 2 the coil has rotated 45. The amount of flux passing through the
coil has increased from zero and will continue to increase until position 3. The
rate of change of magnetic flux B/t at position 2 has decreased and will
continue to decrease until the coil reaches position 3. Around position 3 the
flux is changing relatively slowly. At position 3 the flux reaches a maximum
and then starts to decrease. Since the rate of change of flux B/t is zero
(as shown by the slope of the tangent) at position 3, the emf must also be
zero according to Faradays law. This means there will be no current flowing
through the coil at this point.

b
a

rotation
X

B
A
W

Position

WX

YZ
YZ
YZ

+max.

WX

and B

max. 0

YZ

WX

motors and
generators

WX

tangents show
rate of change

WX

YZ

YZ

WX

YZ

7
WX

YZ

8
axis of
rotation

output terminals

WX

WX

YZ

Time

Figure 7.1.3 (a) A simplified model of an AC generator and (b) the coil as seen from point P in part (a), showing the direction
of the current flowing over one rotation. (c) A graph of magnetic flux and induced emf over one rotation

Once the coil rotates past point 3, the flux B begins to decrease and the rate
of change of flux B/t is increasing. We can see this as the slope of the
curve for B gets bigger (as shown by a tangent to the curve). When the flux
B is decreasing, B=BfinalBinitial is a negative value and so is
B/t. This means that according to Faradays law the induced emf now has
the opposite sign to the sign it had before position 3. This means that the
current produced by this emf is in the opposite direction to the one that
would have flowed before position 3. This explains why we get an AC current,
because at positions 3 and 7 in Figure 7.1.3b the current will change direction.
Between positions 3 and 5 the flux decreases and is zero at position 5. The rate
of change of flux increases to a maximum at position 5. This means that the
emf increases to a maximum at point 5 and, therefore, so does the induced
current. Notice that in Figure 7.1.3b the current flowing along the side of the
coil WX is now flowing into the page. You can use the right-hand palm rule to
show that the current is now flowing through points W, X, Y and Z in turn.
This further illustrates that the current has reversed direction at point 3 and
we are producing an AC current at the output terminals.
Follow the rotation through to position 9 and you will see that the current
changes direction again at position 7 and the full cycle is completed at
position 9. Each rotation induces an emf in the coil that is in one direction
for half a rotation and in the opposite direction for the other half of a rotation.
This alternating emf can produce an alternating current and our explanation
of the operation of AC generators is complete.

A simple DC generator
Now that we have seen how an AC generator works, lets look at a DC generator.
Figure 7.1.5a shows a simple model of a DC generator. The obvious
difference is the split-ring commutator connecting the coil to the external circuit.
We have seen this situation in a simple DC motor (see Figure 6.1.7) but now
it is acting as a DC generator. Note that the graph for emf is the emf that would
be measured at the output terminals. This is therefore affected by the inclusion
of a split-ring commutator that is fundamental to the functioning of a DC
generator. Lets focus on what is different about the operation of a DC generator.

A Linear
Generator
magnet
motion of magnet

coil

Figure 7.1.4 A torch without batteries

ith energy shaping up to be


the single biggest issue for
humans in the future, there are
many new devices available to
help us save energy. A torch
without batteries is just one
example. These torches contain
a permanent magnet that can
slide back and forth inside a coil
of wire. The current generated in
the coil is stored in a capacitor
and is then ready for use in the
torch.

133

Generators and
electricity supply:
power for the people
As before, we see that the change in the rate of change of magnetic flux
B/t in the coil causes a changing induced emf (Figure 7.1.5c). The
operation of this generator is the same as the AC example until we get to
position 3. At position 3 the brushes of the commutator reverse the contact
between the coil and the external circuit.
The current flowing in the coil WXYZ is in the same direction as for an AC
generator (see Figure 7.1.3b). The difference is that now this current only
flows in one direction in the external circuit. As the current in the coil
changes direction at position 3, the split-ring commutator reverses the
connection. This causes the current to flow in the same direction in the
external circuit throughout the entire rotation.
From the graph of emf in Figure 7.1.5c you can see that the emf induced in
the external circuit rises and falls but is always in the same direction. This
produces a current that also rises and falls and is always in the same direction.
This is DC. We usually see a DC (or emf ) represented on a graph as a
straight line with one value (e.g. the red line in Figure 4.1.2). This might be
the case with a current from a battery, but not for the output direct from a
DC generator.
At position 7 we see that again the split-ring commutator reverses the contact
to the external circuit. A current in this external circuit would continue to
flow in the same direction, even though the current in the coil again changes
direction.

Regenerative
braking

n energy-efficient way to
slow a moving vehicle is to
convert the kinetic energy of the
vehicle into electricity. This
energy can then be stored and
reused to get the vehicle moving
again. Traditionally braking was
achieved by using friction and
the energy was lost as heat.
Modern trains and electric hybrid
automobiles now use these
systems to increase efficiency.
The electric motors on the
wheels of the vehicle are used as
generators when the brakes are
applied, and energy is stored in
batteries or a capacitor.

Position

b
1

2
a
X
S

axis of
rotation

+max.

WX

YZ

YZ

tangents show
rate of change
B

WX

YZ
Z

WX
Y

and B
max.

YZ

WX

WX
YZ

YZ
WX
YZ

7
output terminals

WX
YZ

8
WX

WX

YZ
Time

Figure 7.1.5 (a) A simplified model of an DC generator and (b) of the coil as seen from point P in part (a), showing one complete rotation.
(c) A graph of magnetic flux and induced emf over this rotation

134

motors and
generators

Comparing AC and DC generators


The differences in the structure and operation of simple AC and DC generators
The two styles of commutator are essential to the
should now be clear.
production of either AC or DC and are the main difference. As DC generators
have more complicated commutators, they are generally less reliable. The
continual electrical reconnection and the extra mechanical wear in the split-ring
commutator reduces the service life of a DC generator and increases running
costs. Electrical arcing between the split ring and the brushes also produces
electromagnetic radiation. This can cause interference in other electronics and to
radio communication.
When we consider the operation of simple generators, we see that both styles
of commutator connect the rotating coils to an external circuit. The commutators
in a DC generator perform the additional function of reversing the direction of
the connection to the outside circuit each half revolution. This acts to maintain
the current in the external circuit flowing in one direction, even though the
current within the rotating coil is changing direction. We will see in section 7.3
that more complicated generators produce their current in the stator. We will
learn that this is the case in a generator used in a large-scale AC power plant.
This configuration is best for generating high currents and is designed to produce
three-phase electricity.

PHYSICS FEATURE
Comparing motors
and generators

Describe the differences


between AC and DC generators.
Gather secondary information
to discuss advantages/
disadvantages of AC and DC
generators and relate these to
their use.

Compare the structure and function


of a generator to an electric motor.

dynamotor. These can be used to convert DC


electricity to different voltages. This could occur if the
set contained a DC motor and DC generator. As we will
see in the next chapter, there is a handy device that
can easily convert AC to different voltages, but it is
more difficult with DC. Motor-generators can also be
used to convert AC to DC, DC to AC and AC to AC at
another frequency. Modern electronics have now
allowed all these conversions to be made without
the use of a motor-generator, but there are still some
applications where electronics are not practical.

comparison between motors and generators is now


relatively easy. First, take a look at Tables 6.1.1
and 7.1.1 to identify the similarities and differences.
You may have noticed that we had already seen the
principles behind the operation of a generator when
we studied back emf in motors (section 6.2). The
induced emf that opposes supply emf in a motor
would produce a current if there was no supply emf
and you would have a generator.
Figure 7.1.6 shows the
AC voltage is applied to the
combination of a motor and a
motor coil through slip rings
generator. This illustrates the
conversion of mechanical energy to
AC voltage output is through
slip rings and brushes
electrical energy in the generator
and back again in the motor. It also
N
shows the similarity in construction
mechanical energy
input
is
converted
of both an AC motor and an AC
into electrical energy
S
generator. This combination of a
generator
motor and a generator is commonly
known as a motor-generator set or
Figure 7.1.6 A motor-generator set or dynamotor

mechanical
energy output

motor

back emf generated


by turning of motor
voltage applied to motor

135

Generators and
electricity supply:
power for the people

Checkpoint 7.1
1
2
3

Use labelled diagrams to outline the structure of simple AC and DC motors.


Compare the structure and function of motors and generators.
Describe and contrast the structure and function of AC and DC generators.

7.2 Transformers
Discuss why some electrical
appliances in the home that
are connected to the mains
domestic power supply use
a transformer.
Describe the purpose of
transformers in electrical
circuits.

Our everyday use of electricity requires a large variety of electrical currents and
voltages. With DC, this usually means that we have the right battery for the job.
With AC, its another story altogether. When you plug in your laptop computer,
it typically requires an AC input of about 18.5V, but the average value of the
voltage from the power point is about 240 V. (Note: the Australian standard is
now 230 V.) The conversion of AC to the correct voltage occurs, with a range of
+10% to 6%, in a small box on the laptops power cable, which contains a
device called a transformer.
changing magnetic flux

Try This!
exploring
electromagnetism
To observe the magnetic flux
produced by a changing current,
place a wire with many loops near or
wrapped around a magnetic compass.
Connect a battery to your coil and
observe the compass needle as you
connect and disconnect the circuit.

340

200

22

32

0 280 3
0 26
00

24

14

40

20

160

60

80

100

12

Figure 7.2.2 A current through a coil


induces a magnetic flux.

136

AC supply

voltage Vp

voltage Vs

primary coil
with np turns

load

secondary coil
with ns turns

Figure 7.2.1 In an ideal transformer, the iron core ensures that all the flux generated
in the primary coil also passes through the secondary coil.

Transformers are devices that can easily convert alternating


currents to higher or lower voltages. They consist of two coils of insulated
wire wound around an iron core (see Figure 7.2.1). You may notice that
they have the same basic construction as Faradays iron ring we studied
Faraday observed
earlier (see Figure 5.1.4). As we saw in Chapter 5,
an induced current in a secondary coil only when there was a changing
current in the primary coil, and this is illustrated in Figure 5.1.5. The
changing current in the primary coil created a changing magnetic flux that
passed through the secondary coil. This changing flux induced an emf in
It is
the secondary coil and an induced current was measured.
important to note that when a direct current was applied to the primary
coil it did not produce a continually changing magnetic field, so
transformers are only practical for AC.
To quantify the voltage conversions performed by transformers,
we recall Faradays law from Chapter 5:
= n(B/t)

motors and
generators
Lets consider applying a 50Hz input voltage Vp across the primary coil of
the transformer in Figure 7.2.1, which has np turns (or loops) of wire. If we
assume there are no losses, this will produce the same changing magnetic flux
B/t within both coils. This changing magnetic flux will produce a 50Hz
alternating voltage Vs across the secondary coil with ns turns. Substituting into
Faradays law gives:
B
B
Vp = np
and Vs = ns
t
t

Dividing one expression by the other gives:


Vp
Vs

np
ns

This expression shows us the relationship between the number of turns and
the voltages across each coil of a transformer.
If we analyse the expression above, we can see that if the primary coil has
more turns (loops) than the secondary coil (i.e. np/ns>1), the voltage Vp across
the primary coil will be greater than the voltage Vs across the secondary coil.
This is called a step-down transformer because it produces AC with a
lower voltage. When the secondary coil has more turns than the primary coil (i.e.
np/ns < 1), the voltage Vs measured across the secondary coil is higher than the
This is called a step-up transformer because it produces
input voltage Vp.
an AC output with higher voltage. Figure 7.2.3 is a simplified diagram showing
the change in magnitude of the output voltage Vs compared to the input voltage
Vp for these two types of transformers.

Identify the relationship


between the ratio of the
number of turns in the primary
and secondary coils and the
ratio of primary to secondary
voltage.
Vp

Vs

Worked example
Figure 7.2.3 The simplified graphs show

QUESTION

the AC output of a step-up


transformer (blue line) and
a step-down transformer (red
line) in the secondary coil of a
transformer. The input voltage
in the primary coil is shown for
comparison (black line).

Calculate the number of turns required in the secondary coil of a transformer to produce
18.5V AC from 230V AC if there are 100 turns in the primary coil.

SOLUTION
Using:

Vp
Vs

np
ns

V
ns = np s
Vp

18.5
Substituting Vp = 230 V, Vs = 18.5V and np = 100 gives: ns = 100
230
= 8 turns
and rearranging to make ns the subject gives:

Transformer efficiency and design


The law of conservation of energy states that energy cannot be created or
destroyed, but can be converted from one form to another. This means that the
amount of energy you put into a transformer must equal the amount you get
out. This is true for all transformers, but some of the energy that went into a
transformer is always converted to heat. It is for this reason that transformers
are never 100% efficient.

Compare step-up and stepdown transformers.


Solve problems and analyse
information about transformers
using:

Vp
Vs

np
ns

137

Generators and
electricity supply:
power for the people
Efficiency is often expressed as a percentage and can be determined using the
following equation:
useful power output
Efficiency (%) =
100
total power input
The losses of energy that occur within a transformer cause the useful electrical
power output to be less than the power input. We saw in Chapter 4 that power is
the rate at which energy is converted and can be determined using the following
relationships:
V2
P = IV, P = I2R and P =
R
where P is electrical power in watts (W), I is current in amps (A), V is potential
difference (or voltage) in volts (V) and R is resistance in ohms ().
In formulating the equation:
Vp np
=
Vs ns
we assume that there are no losses; that is, that transformers are ideal.
We will
continue this assumption whenever we do calculations for transformers, but it is
important to consider transformers in a more realistic way, to understand their
losses and design considerations.
Using the equation above, we can determine another relationship for a
transformer that illustrates the changes in current. In an ideal transformer the
power input in the primary coil (Pp) and the power output in the secondary coil
(Ps) would be:
Pp = IpVp and Ps = IsVs

Neon lights

eon lights are one very


visible application of
transformers. Typically about
25000 volts are required to
excite the gas in a neon tube, so
a step-up transformer is used to
convert the voltage from 230V
AC. In Figure 7.2.4, the red light
is produced by neon gas and the
violet-blue light is produced
by argon.

Figure 7.2.4 Neon and argon


discharge tubes

138

Rearranging and substituting into the relationship above gives:


I s Pp np
=
I p Ps ns
Pp = Ps for an ideal transformer, so:
I s np
=
I p ns

This illustrates that both the voltage and the current output of a
transformer change with respect to the input. We have therefore gained another
insight into the workings of transformers. By inspecting both equations:
Vp np
I s np
=
and
=
I p ns
Vs ns

we can see that


for a step-up transformer (ns > np), the transformer increases
for a
the voltage but decreases the current of AC electricity. Alternatively,
step-down transformer (ns < np) the voltage decreases but the current increases.
Lets see how this situation satisfies the law of conservation of energy.
Since energy (in joules) must be conserved and power (in joules per second) is
the rate at which energy is converted, then power must also be conserved if no
energy is being stored in a transformer. As power is equal to the product of the
current and voltage:
Pp = Ps and IpVp = IsVs

motors and
generators
As we already know, the voltage is changed in a transformer and, for the above
relationship to hold, the change in current must be inversely proportional to the
change in voltage. We have already shown that this is the case by deriving the two
expressions:
Vp np
I s np
=
and
=
I p ns
Vs ns
Combining these expressions gives:
I s Vp
=
I p Vs

This expression shows that current and voltage are inversely proportional.
For example, in a step-up transformer Vs is larger than Vp so Vp/Vs must be less
than 1. In this case, Is/Ip is also less than 1. This means that Is is less than Ip.
We now see that transformers obey the law of conservation of energy
when they alter the current and voltage of AC electricity. For an ideal transformer,
the input power is equal to the output power and this is maintained by the inverse
relationships between input and output current and voltage.
These findings explain the design features evident in Figure 7.2.5, which
shows a step-down transformer. The increase in current in the secondary coil is
the reason a larger diameter wire is used for the secondary coil.
As discussed in Chapter 5, larger currents produce greater losses due to
resistive heating. The amount of heat Q produced by resistive heating is
proportional to the resistance of the material R and the square of the currentI
as shown by Joules law:
Q = Pt = I2Rt
where Q is in joules, power P is in watts or joules per second (Js1), I is in
amps, R is in ohms and t is in seconds (s). So we see that larger currents produce
more heat.
The resistance of a metallic conductor is described by the following equation:
l
R=
A
where is resistivity, l is length and A is cross-sectional area. This equation shows
that resistance in a wire is proportional to its length and inversely proportional to
its cross-sectional area. So, by increasing the diameter of the wire in the secondary
coil of the transformer (Figure 7.2.5), the resistance is decreased and this
minimises resistive heating.
resistive heating is one of the mechanisms
We have now identified that
responsible for energy losses in all transformers. The transformer shown in Figure
7.2.5 is a very common style of transformer in household appliances because it
minimises another loss mechanismflux leakage. The transformer in Figure
7.2.5 has a central iron core around which the primary and secondary coils are
The overall shape of the iron core acts to contain and direct the
wound.
magnetic flux from the primary coil through the secondary coil. This ensures that
the maximum amount of flux passes through the secondary coil and therefore
maximises induction in the secondary coil. If a significant amount of flux did not
pass through the secondary coil, then energy would not be transferred and could
be lost by other mechanisms.

Explain why voltage


transformations are related
to conservation of energy.

secondary
coil
primary coil

Figure 7.2.5 A step-down transformer in which


the coils are wound around a
central laminated iron core

Gather, analyse and use


available evidence to discuss
how difficulties of heating
caused by eddy currents in
transformers may be overcome.

139

Generators and
electricity supply:
power for the people

a
primary coil

cross soft iron core


section
I

increasing
magnetic
flux

secondary
coil

b
solid core

increasing
magnetic
flux
eddy currents
cross section

c
laminated
core

eddy current
cross section

Figure 7.2.6 (a) A simple model of a


transformer. (b) Large eddy
currents are set up in the core.
(c) Smaller eddy currents are
set up in a laminated core.

The Sounds
of AC

f you ever get a chance to see


a large industrial transformer
you will probably hear it hum.
This is the sound of the
laminations in the core vibrating.
As the direction of 50Hz AC
output alternates back and forth,
the magnetic field within the
core also changes direction. The
alternating magnetic flux exerts
forces on the core laminations
and causes them to vibrate.

140

The remaining mechanisms for losses in transformers are sometimes referred


The first of these is loss due to magnetic hysteresis as the
to as core losses.
magnetic field continually changes direction. We were introduced to this effect
in our discussion of induction cooktops (see Chapter 5 Physics Feature p 108).
In induction cookware hysteresis served a useful purpose in heating food, but in
a transformer we wish to minimise the effect. This can be done by making the
core of a transformer from a magnetically soft iron that does not stay magnetised
once an external magnetic field is removed. Magnetically soft iron is therefore
used in transformer cores to reduce heat loss through magnetic hysteresis. This
soft iron is also used in the rotors of motors and generators and in the stators of
AC motors to reduce losses due to magnetic hysteresis.
The other type of core loss in transformers is resistive heating due to
eddy currents. To reduce this effect, the soft iron cores of transformers are made
from thin laminated sheets that are electrically insulated from each other. Figure
7.2.6a shows a simple set-up of two coils that would act as a transformer. This
diagram shows a moment in time in which the current in the primary coil is
producing an increasing magnetic flux in the direction indicated. In accordance
with Lenzs law, currents would be induced in both the secondary coil and as
eddy currents in the iron core (Figure 7.2.6b). To minimise the energy lost as
heat due to these eddy currents, the core is made from thin laminations (Figure
7.2.6c) that are electrically insulated from one another. This minimises the
magnitude of the eddy currents and therefore the amount of heat lost through
resistive heating.
We have now seen that transformers come in many shapes and sizes, but they
all have the same basic structure. They contain a magnetically soft, laminated
iron core and two coils of insulated wire. Large transformers can be up to 99%
efficient, but there are always some losses as heat. These losses, although small in
proportion, can affect the performance of a transformer, if allowed to build up.
In small transformers, air removes heat by convection, but in large transformers
oil is used (Figure 7.2.7). The oil is passed through cooling tubes and heat is
dissipated to the air. This type of transformer is used in electrical substations and
at power plants as part of the distribution of electricity to the community. We
will explore this more in the next section.

low voltage
terminals

high voltage
terminals
oil tank

laminated core
low voltage coils

high voltage coils

cooling
tubes

Figure 7.2.7 An industrial transformer used in the transmission of electricity

motors and
generators

Checkpoint 7.2
1
2
3
4
5
6

Identify a device that uses a transformer and outline the role the transformer plays in the operation of this device.
Define step-up and step-down transformers.
Explain why transformers do not work with DC electricity.
Calculate the voltage produced by a transformer for which np/ns is 0.01 and the input voltage is 12V.
Recall the law of conservation of energy and analyse its application to the operation of a transformer.
Identify the losses that occur in transformers and outline the design features that minimise these losses.

7.3 Electricity generation and


transmission
PHYSICS FEATURE

PRACTICAL
EXPERIENCES
Activity 7.2

Activity Manual, Page


58

Edison and Westinghouse

n the late 1800s a rivalry developed in the US over


the type of electricity that should be generated.
Thomas Edison (18471931) had established a
system based on DC electricity. It was primarily used
for household lighting and Edison had patents on
much of the technology involved. One of the major
drawbacks of this system was that it had to be
distributed at the voltage used in households, as
transformers could not be used with DC to easily alter
the voltage and current. Transmission therefore
required large currents, and this limited the distance
DC could be transmitted because of the huge power
losses as a result of resistive heating. This meant that
power stations had to be dotted all over a large city
and distribution was not practical in rural areas.
When Edison was establishing his system there
was no viable alternative to DC. Nicola Tesla
(18561943), an employee of Edison at one stage,
eventually developed AC motors and generators.
The patents for these were promptly purchased by
George Westinghouse (18461914). Alternating
current had the advantage that voltage and current
could be changed using transformers. It could
therefore be generated at low voltages and converted
to high voltages for transmission using low currents.
This allowed transmission over long distances

Figure 7.3.1 (a) Thomas Edison and (b) George Westinghouse


with acceptable energy losses, and made AC more
economical.
Edison embarked on a campaign to discredit the
use of AC and promote it as unsafe. To do this he used
an AC generator to publicly electrocute animals,
including an elephant. The showdown between
Edisons DC and Westinghouses AC came when a
proposal was put forward to generate hydroelectricity
at Niagara Falls. Several designs were considered,
including those from Westinghouse and Edison.
Westinghouse was awarded the contract and, due to
its many advantages, AC electricity was eventually
adopted as the standard worldwide.
Gather secondary information on the competition
between Westinghouse and Edison to supply
electricity to cities.

141

Generators and
electricity supply:
power for the people

AC power generation and delivery

Figure 7.3.2 A coal-fired power station


a

C
B

three phase
output

stator
N

A
S

neutral

Current or EMF

b
+
0V

B
10

20

30

40

Time
(ms)

Figure 7.3.3 (a) A three-phase generator. The DC


output is fed to the rotating magnet via
slip rings (not shown). The ends of each
of the three stator coils are connected
togetherthis becomes the neutral.
(b) The other ends carry the high voltages,
which are one-third of a cycle apart.
Explain the role of transformers
in electricity substations.

142

Today, more than 99% of the worlds electricity is generated as AC.


Most of the electricity generated in New South Wales is produced by
burning coal in a power plant like the one shown in Figure 7.3.2.
Heat from the burning of coal in these facilities is used to produce
steam. This steam is used to turn turbines, which are connected to large
generators that produce electricity. These generators typically produce
three-phase 23kV AC at 50Hz. Figure 7.3.3 shows the main features of
a three-phase generator used in a power station. Once generated,
electricity is transmitted to consumers through a vast network, such as
the one summarised in Figure 7.3.4.
The most common use of this energy is in our homes. It arrives there
in wires above your street or below your feet in underground cables. In
order to get there efficiently, electricity goes through the following steps
on the way to your home.
The 23kV 50Hz output voltage from a power stations generators is
passed through step-up transformers to produce 330kV or even 500kV
AC. High voltages mean low currents and this reduces power losses
during transmission due to the resistance of the lines. Large transmission
lines are used to transport this electrical energy over long distances to a
terminal substation. Fuel sources for power stations are typically a long
way from major population centres, so this long-distance transmission is
essential.
At the end of the transmission line, step-down transformers in
a terminal substation convert the voltage to 132kV or 66kV for
transmission to zone substations (see Figure 7.3.5). Higher voltages are
less important for these shorter transmission distances. As the voltages
decrease, the effort required to insulate each wire from the others
decreases and so does the cost involved.
At zone substations this power is again stepped-down using
transformers. Most commonly it is converted to 11kV AC and
distributed into communities.
Typically, transformers on power poles finally step-down the AC to
400V and 230V for industrial and household use respectively. These
voltages are low enough so that discharges cannot occur through air
within appliances, and the insulation of wires from each other is easy
and economical.
A typical power pole in a suburban street resembles the one shown
in Figure 7.3.6. Three of the four main wires shown carry the three
phases produced in the power plants generator (see Figure 7.3.3). The
other wire is the neutral that completes the circuit. The potential
difference between neutral and a single phase is 230V. The difference
between any two of the three phases is 400V. A single phase and the
neutral are connected to each household and all four wires deliver
three-phase power to industry (see Figure 7.3.4).

motors and
generators

step-up transformers
GENERATION

POWER STATION
23 kV
50 Hz

330 kV or 500 kV
AC transmission

HIGH VOLTAGE
TRANSMISSION LINES
TRANSMISSION

TERMINAL
SUBSTATION
132 kV or 66 kV
AC transmission

step-down transformers
132 kV & 66 kV AC
SUB-TRANSMISSION

ZONE
SUBSTATION

11 kV
AC distribution
step-down transformers
11 kV AC
DISTRIBUTION

400 V and 230 V


AC distribution

pole step-down
transformer

CONSUMER

household
single-phase
230 V AC

3 2

230 V
400 V

factory
three-phase 400 V AC

Figure 7.3.4 A typical electricity supply network in New South Wales


143

Generators and
electricity supply:
power for the people

four wires carrying


three-phase AC and
the neutral wire

insulator

Figure 7.3.5 An electricity substation

Figure 7.3.6 A power pole in Marrickville, Sydney. Note that the four
main wires are not insulated. They are spaced far
enough apart to stop discharges occurring. They are
insulated from the pole by small ceramic insulators.

PHYSICS FEATURE
Transformers in the Home

he use of transformers does not end when


electricity is fed into your home.
The
components of many household appliances require
higher or lower voltages than the 230V available from
the wall socket. A good example is the microwave
oven. The component within a microwave oven that
produces the microwaves typically requires thousands

of volts, while the control circuits and control panel


on the front only use small voltages. This means that
a microwave oven would require both step-up and
step-down transformers to supply the power for its
components.
Discuss why some electrical appliances in the
home that are connected to the mains domestic
power supply use a transformer.

Power losses during transmission and distribution


Discuss the energy losses that
occur as energy is fed through
transmission lines from the
generator to the consumer.

In order to deliver electrical power efficiently across NSW the power loss between
The main
the generator and the customer must be as small as possible.
losses during transmission result from the resistance in the transmission wires and
the induction of eddy currents. First, lets look at losses due to resistance.
Using a relationship we have seen before:
P = I2R
we see that power loss P in transmission lines is proportional to the resistance R
and the current I squared. Therefore, if we double the resistance in transmission
wires we would double the power loss. But if we double the current we increase
So obviously, we want the resistance in
the power loss by a factor of four.
transmission wires as low as possible but, even more importantly, we want to use
a relatively small current. This is achieved using transformers to step-up AC to as

144

motors and
generators
high a voltage as practical. During this process the current is reduced to a
minimum.
Another way to illustrate why we want to reduce transmission current comes
from another relationship we saw earlier:
l
R=
A
where is resistivity, l is length and A is cross-sectional area.
This equation shows that the resistance R of a wire is proportional to its
length l. This means that, as the distance of transmission increases, the resistance
also increases. A typical resistance for one type of high-voltage transmission line
is about 0.4 ohms per kilometre. We can see that if large currents are used,
transmission will quickly become uneconomical over the hundreds of kilometres
typically required. A large proportion of the energy generated would be lost as
heat due to resistive losses. The best solution is again to minimise the current
being transmitted.
The two previous paragraphs highlight the importance of transformers in the
distribution of electricity. Transformers can readily step-up voltages and in doing
so they minimise the magnitude of the current involved. They can also step-down
the voltage when required to a value suitable for the consumer. This ability of
transformers has changed modern society enormously, and we will take a closer
look at this soon.
l
also shows us that the resistance of a wire is inversely
The equation R =
A
proportional to its cross-sectional area A. So making the diameter of the wire
larger will reduce its resistance. Unfortunately, thick copper wires are very heavy
and require larger structures to support them. Larger structures cost more money,
so a compromise is required. Aluminium is used as a conductor in high-voltage
transmission lines. Although it has a higher resistance than copper, it is much
lighter, so the wires have a larger diameter to reduce resistance without
being too heavy.
Losses during AC transmission also occur due to the formation of
eddy currents. The constantly alternating current in transmission lines
produces a constantly changing magnetic flux. This can induce eddy
To
currents in nearby conductors and energy will be lost.
minimise this, transmission lines are held at a distance from metal
transmission towers by insulators. Losses due to eddy currents also
occur in the cores of transformers. Recall that these losses are
minimised by constructing these cores from thin laminations of
magnetically soft iron.

Gather evidence and analyse


secondary information to
discuss the need for
transformers in the transfer of
electrical energy from a power
station to its point of use.

PRACTICAL
EXPERIENCES
Activity 7.3

Activity Manual, Page


62

lightning protector
insulators

Transmission structures
A typical high-voltage AC transmission tower is shown in Figure
7.3.7. The three sets of wires carry the three phases generated in the
power station. As with lower voltage transmission (Figure 7.3.6),
high-voltage transmission lines are not coated with insulation.
These three sets of wires must be kept at large distances from
each other and from the metal tower. This not only minimises losses
due to eddy currents, it also minimises the chance of current
flowing through the air (electrical discharge) between conductors
or to the ground through the tower. In ideal conditions, 500kV

high voltage
transmission
lines

Figure 7.3.7 A high-voltage transmission tower


145

Generators and
electricity supply:
power for the people
Gather and analyse information
to identify how transmission
lines are:
insulated from supporting
structures
protected from lightning
strikes.

transmission lines must be approximately half a metre away from the tower that
supports them. In conditions in which the insulators are wet and covered in
pollution deposits, this distance must be greater. Typically, therefore, these wires
are held more than a metre from their supporting tower.
Figure 7.3.8 shows that insulators used for lines are typically long chains of
ceramic components. These components are disc shaped and have deep
corrugations on their underside. These features increase the distance any current
would have to travel in a discharge. The corrugations also minimise the chance
of pollution deposits settling on the surface and keeps some of the insulator dry
in wet weather.

High-Voltage DC Transmission

odern electronics have made the task of converting high-voltage AC to


high-voltage DC (HVDC) relatively simple, and HVDC transmission is now
economically viable for distances of more than several hundred kilometres.
These systems lose less power due to eddy currents induced in metal support
structures and require only two transmission lines. For short distances, these
savings do not offset the cost of the extra electronics, but if the transmission
distance is long enough, the savings are significant. HVDC is the method of
choice for transmitting electricity by underwater cables due to the excessive
losses involved. The Victorian and Tasmanian electricity grids are connected
across Bass Strait via a 400kV DC link called Basslink. This provides a twoway link along which electricity flows according to the demand at either end.
In this way, either state can make up for a shortfall of electricity in the other.

The impact of AC generators and transformers on society

insulator
arcing horn
cement
ball

conductor

Figure 7.3.8 (a) A technician services


insulators on a power line.
(b) Insulators used are
typically long chains of
ceramic components.

146

cup
porcelain

The development of AC generators and transformers in the 20th


century enabled the widespread availability of electricity. AC
generators efficiently convert the energy from sources such as coal
into electricity. Transformers enabled the efficient long-distance
transmission of this electricity to the households of whole nations.
These developments have arguably been the single greatest
catalysts for change in modern society.
Before the availability of household electricity, a significant part
of the daily ritual was the lighting of fires for cooking and heating.
Refrigeration was generally not available, so many perishable foods
could only be eaten fresh or when in season. None of the electrical
appliances that make our lives so convenient had been developed,
so much more time was devoted to chores and manual tasks.
As you can imagine, these differences would have resulted in a
very different life for the average citizen in a society without
electricity.
The availability of electricity has also had many positive and
negative effects on society.
The ability to transmit electricity efficiently over long distances
has allowed more people in large cities to live further from the

motors and
generators
city centre. It is not uncommon in modern society for people to commute
large distances to work.
People in most rural areas enjoy the same access to electricity as city dwellers.
This has improved their quality of life and allowed many people to remain in
country areas, who might have otherwise decided to move to the city.
The quality of life improved when electricity was affordable in every home.
This changed the way we live through the many appliances available. Many
of us find it hard to conceive a life without a refrigerator, television, DVD
player or computer.
Tasks performed by electrical machinery decreased the amount of unskilled
labour required and increased unemployment in certain parts of society.

Assess the effects of the


development of AC generators
on society and the
environment.
Discuss the impact of the
development of transformers
on society.

The widespread generation of electricity has also had a significant effect on


the environment.
When industry was able to move away from the power plants in city centres
this took pollution away from many peoples homes. This improved the
quality of their environment and improved their health.
The availability of electric power for heating has reduced the need to burn
wood or coal in houses, significantly improving air quality in large cities.
The use of hydro-electricity generation required the
construction of large dams. These destroyed the habitat of
many plants and animals. It also displaced many people from
their homes, and this continues today as our demand for
electricity grows.
Lightning protection
The construction of long-distance power lines also required
igure 7.3.7 shows a typical high-voltage
the destruction of habitat and increased rates of erosion
transmission tower. The two wires at the very
where vegetation has been removed.
top of the tower are called lightning protectors or
Radioactive waste from nuclear power plants requires
shield conductors.
These wires are placed
long-term storage and poses a threat to the environment if
above
the
tower
and
transmission
lines to
not contained.
significantly reduce the chance of lightning
The burning of fossil fuels for the production of electricity
has led to a significant increase in atmospheric carbon
striking the transmission lines. At regular
dioxide levels. Scientists believe this will have significant
intervals these wires are connected directly to
consequences in the near future through global climate
the ground. These connections act to transfer
change.
huge currents from lightning strikes directly to
The burning of fossils fuels has also produced pollutants
earth. This protects the transmission system
such as sulfur dioxide. This contributes to the formation of
from dangerous spikes in current and voltage,
acid rain, which can damage forests, aquatic life and manwhich can damage substation transformers and
made structures.
cause blackouts.

PHYSICS FEATURE

Checkpoint 7.3
1
2
3
4
5

Explain how transformers allow the transmission of AC power over long distances.
Identify the role of transformers in household appliances.
Outline the energy losses in high-voltage transmission lines and the steps taken to minimise them.
Describe how transmission lines are insulated from supporting structures and protected from lightning strikes.
Assess the impact of transformers and AC generators on society and the environment. Justify your answer.

147

Generators and
electricity supply:
power for the people

PRACTICAL EXPERIENCES
CHAPTER 7

This is a starting point to get you thinking about the mandatory practical
experiences outlined in the syllabus. For detailed instructions and advice, use
in2 Physics @ HSC Activity Manual.

Activity 7.1: AC and DC generators


Gather secondary information
to discuss advantages/
disadvantages of AC and DC
generators and relate these to
their use.

Research how AC and DC generators work then place the information that you
have found into a table outlining the advantages and disadvantages of each. Use this
information to create a list of applications that would benefit from using each.
Discussion questions
1 Compare the structure of AC and DC motors.
2 List applications for which AC and DC generators are preferred and
explain your choice.

Activity 7.2: Edison and Westinghouse


Analyse secondary information
on the competition between
Westinghouse and Edison to
supply electricity to cities.

Research the competition between Westinghouse and Edison to dominate supply


of electricity. Use this information to present a 5 minute speech on the topic.
Discussion questions
1 Outline the benefits of both DC and AC electricity supplies.
2 State reasons that Edison used to show that AC was more dangerous.

Activity 7.3: Power to the people


Gather and analyse information to
identify how transmission lines are:
insulated from supporting structures
protected from lightning strikes.
Perform an investigation to model the
structure of a transformer to demonstrate
how secondary voltage is produced.
Gather, analyse and use available
evidence to discuss how difficulties of
heating caused by eddy currents in
transformers may be overcome.
Gather and analyse secondary
information to discuss the need for
transformers in the transfer of electrical
energy from a power station to its point
of use.
Gather, process and analyse information
to identify some of the energy transfers
and transformations involving the
conversion of electrical energy into more
useful forms in the home and industry.

148

In this investigation, you will gather first-hand data to see how a


transformer works. You will then gather information from secondary
sources in order to look into how heating due to eddy currents are
overcome in a transformer.
Equipment: transformer with removable coils, power supply,
ammeter, voltmeter, connecting wires, light bulb.
Discussion questions
1 Outline the difference between a step-up and step-down
transformer.
2 Explain why the first-hand data does not fit the equation for
a transformer.
3 Outline methods used to overcome heating in transformers due
to eddy currents.

Chapter summary

The difference between motors and generators is that


motors use electricity to produce mechanical energy and
generators produce electricity from mechanical energy.
The main components of a generator are the armature,
coils, rotor, stator, split-ring or slip-ring commutator
and commutator brushes.
A current is produced in the rotating coil of a generator
due to the changing magnetic flux within the coil.
An AC generator contains a slip-ring commutator; a
DC generator contains a split-ring commutator.
The operation of a transformer relies on the changing
magnetic flux from the primary coil passing through the
secondary coil. This changing flux then induces an emf
and produces a current in the secondary coil.
The law of conservation of energy states that energy
cannot be created or destroyed, only converted from one
form to another.
Transformers alter the voltage and current of AC
electricity. There is an inverse relationship between the
voltage and current produced by a transformer. If the
voltage is increased the current is decreased and vice
versa. This is in agreement with the law of conservation
of energy.
For an ideal transformer, the relationship between
the number of turns in the coils of a transformer and
the voltages in either coil is:
Vp np
=
Vs ns

motors and
generators

A transformer that increases the voltage of AC electricity


is called a step-up transformer; one that decreases the
voltage is called a step-down transformer.
Loss mechanisms in transformers include flux leakage,
resistive heating by eddy currents and hysteresis loses.
These are minimised by using a suitably shaped and
magnetically soft laminated iron core.
The increase in AC voltage and subsequent decrease in
current in transformers allows the efficient transmission
of AC power over long distances.
Household electrical appliances contain components
that require various voltages and it is transformers
within these appliances that provide these voltages.
The main losses in the transmission of electricity are due
to resistive losses and eddy currents. These losses are
minimised through appropriate isolation of conductors
and minimising resistance.
Transmission lines are isolated and insulated from their
supporting structures by insulating components called
insulators. The higher the voltage, the larger the length
of the insulator required to minimise the occurrence of
electrical discharge.
Transformers and AC generators have enabled the
widespread distribution of AC electricity. This has had
an enormous impact on society and the environment.
These impacts include changes in where many people
live, their quality of life, the nature of their employment,
their health and that of our environment.

Review questions
Physically speaking
Copy and complete the table below by adding a definition for each key term.

Term

Definition

Step-up transformer
Generator armature
Generator
Substation
High-voltage insulator

149

Generators and
electricity supply:
power for the people

Reviewing
A

X
B

Look at Figure 7.4.1. Identify each of the parts that have been labelled.

3
4

Explain how you could tell the difference between an AC and DC motor.

Outline methods of reducing energy lost along the path from generation
to consumer.

List the applications that would be best suited to using an AC generator


and those that would be more appropriate using DC generators.

State the features of AC and DC generators that make them appropriate


to their use.

a List the effects that generators have had on society and the environment.
b Assess these effects.

Give reasons why transmission lines need to be insulated from supporting


structures and describe how it is done.

1
2

Figure 7.4.1 An AC generator

Create a table to compare and contrast the parts and function of an electric
motor and a generator.
a Identify where energy is lost during the distribution of AC from the time
the electricity is generated to when it reaches the household consumer.
b Describe the energy that is lost.

10 Explain how transmission lines are protected from lightning strikes.


11 a Identify parts AE of the transformer in Figure 7.4.2.
b Identify the function of each of these parts.
c If np = 250 and ns = 750, identify if this is a step-up or a step-down
transformer.
A
changing magnetic flux

B
E

voltage Vp

voltage Vs

Figure 7.4.2 Parts of a transformer

12
13

Explain how current is produced in the secondary coil of a transformer.


Discuss how the number of turns in the coils of a transformer determines
the voltage in the secondary coil.

14 Show that Vp
Vs

np is true for a transformer that is 100% efficient.


ns

15 Outline the energy transfers for a transformer that is not 100% efficient.
16 Determine how currents in primary and secondary coils of a transformer
are related.

17 Explain why transformers are needed in order to bring electricity effectively


from a power station to the consumer.

18 Many appliances that are used in the home have a transformer attached to
their power cord. Explain the presence and need for these.

150

motors and
generators
19 Give examples of how transformers have impacted on the way in which
we live.

20 Discuss the production of eddy currents in transformers and the problems


that arise from them.

21 Outline methods of reducing the effects of eddy currents in transformers.

Solving Problems
22 Calculate the voltage in the secondary coil of a transformer that has 20 turns
in the primary coil and 300 turns in the secondary coil if the input is 240V.

23 Determine the efficiency of a transformer that has a turns ratio of 10:500,


an input voltage of 50V and output of 200V.

24 Complete the following table.


Coils in primary

Primary voltage (V)

100

320

240

50000

Coils in secondary

Secondary voltage (V)

Step-up or step-down

200
50
30500

240

25 Complete the following table by calculating the unknown quantities.

6
240

0.5

Current in secondary
coil (A)

Voltage in secondary
coil (V)

Turns ratio

240
2

12

0.1

1000

iew

Q uesti o

Voltage in primary
coil (V)

Re

Current in primary
coil (A)

151

The review contains questions in a similar style and proportion to


the HSC Physics examination. Marks are allocated to each question
up to a total of 30 marks. It should take you approximately
54minutes to complete this review.

Multiple choice

Look at Figure 7.5.2.

(1 mark each)
1 Two current-carrying wires of different lengths are

III

placed side by side as shown in Figure 7.5.1.


0.5 cm

The length of the 2m wire and the separation


between the two wires are doubled. Identify the
magnitude of the new force acting on each wire.
A F
B 2F
C 4F
D F/2

152

Figure 7.5.1

0.8 m

IV

2m

Identify the answer that has the correct part matched


to the correct role.

Part

Role

a
b

Armature

Provides magnetic field

Brushes

Commutator

Magnet

Allow current direction to change


within the motor
Maintains electrical contact without
tangling wires
Part of motor or generator that
contains current-carrying coils or
windings

Determine the output voltage from a household


transformer for which the ratio of windings is 40:500.
A 3000V
B 19V
C 0.05V
D 240V

Figure 7.5.2 A generator

II

Identify the answer that lists its parts correctly.

II

III

IV

Axis of
rotation

Force

Output
terminals

Current

Force

Current

Axis of
rotation

Output
terminals

Axis of
rotation

Output
terminals

Force

Current

Output
terminals

Axis of
rotation

Current

Force

A series of substations can be found in suburban


streets. The role of these substations is mainly to:
A step up the voltage.
B step down the voltage.
C boost the power being transmitted.
D add extra branches of wires to serve more houses.

motors and
generators

Extended questions
6

Describe the main features of an AC induction motor


and their roles. (3 marks)

Outline the procedure you followed in order to


investigate three factors that affect the generation of
an electrical current. (3 marks)

Discuss the how competition between Edison and


Westinghouse led to the efficient supply of electricity
to cities. (2 marks)

Explain how eddy currents are used in an induction


cooktop to cook a steak. (2 marks)

10

A wire is placed in an external magnetic field as


shown in Figure 7.5.3.
a Determine the direction of the force on the wire.
b Calculate the size of the force experience by
the wire. (3 marks)

12

Compare and contrast how the motor effect is used


in a galvanometer and a loudspeaker. (3 marks)

13

From the four chapters in this module, identify some


of the energy transfers and transformations involving
the conversion of electrical energy into more useful
forms in the home and industry. (4 marks)
Gather, process and analyse information to
identify some of the energy transfers and
transformations involving the conversion of
electrical energy into more useful forms in
the home and industry.

0.1 T magnetic field


within the shaded
area

1 cm

0.5 A

Figure 7.5.3 A wire in a magnetic field

1 cm

11

A simple electric motor has dimensions as shown in


Figure 7.5.4. The single coil carries a 0.1 A current
and sits within a 0.05 T magnetic field at 40 to
the vertical.
a Explain why the magnets in a real motor would
be curved.
b Determine the force generated by the motor
effect on each side of the motor.
c Calculate the torque in the motor. (5 marks)
axis of rotation
m

4c

4 cm
40

I
N

Figure 7.5.4 A simple electric motor

153

3
Context
Figure 8.0.1 Semiconductor integrated
circuits have produced laptop
computers that have much
more computing power than
even the most powerful early
military computers that used
thermionic devices.

154

from ideas to
Implementation
In just over 100 years we have gone from not knowing what constitutes matter to
understanding its subatomic structure. From the discovery of the mysterious
cathode rays came an understanding that these are fundamental particles that hold
solid matter together. Technology, such as X-rays, sprang from this discovery and
showed that their use in medical imaging enabled better patient diagnosis.
An understanding of the way electrons move in space and solids gave rise to the
plethora of electronic devices that have transformed our technology and way of life.
The control of electron motion in a vacuum resulted in early thermionic devices
(valves), such as the triode, to control the flow of current in electronic circuitry.
From these came television, radio, radar and the start of the modern electronic age.
The study of electron motion in solids resulted in miniature semiconductor devices
that replaced thermionic devices and enabled faster, more efficient and much more
sophisticated electronics, such as integrated circuits, to be made. These devices
are the backbone of all modern-day electronics, and include mobile phones and
computers, as well as electronics and instrumentation used in homes, hospitals,
industry, and in space.
The accidental discovery of superconductors (solid material through which
electrons travel unimpeded) may one day transform our technology yet again to
produce much faster computers and more efficient transportation, energy
production and transmission.

Figure 8.0.2 CDs and DVDs can be


used as spectrometers.

Inquiry activity
CDs: windows to viewing photons
Neon signs and most street lights use discharge tubes to create the many
different-coloured lights. In all of these, an electrical current is passed through
a gas at reduced pressure. The light from the glowing gas is characteristic of the
type of gas in the tube. A spectrometer is a device that separates the colours of
the spectrum of the gas by using the same principle of interference as early
research into X-ray diffraction. You can make your own spectrometer of visible
light at home simply by looking at the light reflected from the back of a CD
or DVD.
Ordinary light (from the Sun and an incandescent bulb) will show you the
continuous spectrum of white light that makes up the colours of the visible
spectrum. Wait until its dark and find lighting that does not use incandescent
lamps. Fluorescent tubes or bulbs are commonly available lighting that uses
electrical discharges. These are filled with mercury vapour and an inert gas.
1 Compare the spectrum of colours from a fluorescent lamp with that of an
incandescent lamp. Does it come in discrete colours or is it continuous like
the light from the incandescent lamp?
2 Use your spectrometer under the street lighting. Can you tell if this lighting
has the same gas as your fluorescent lamp or is it different? Or does it use
a mixture of gases?

155

From
cathode rays
to television
Mysterious rays

fluorescence, Geissler tubes,


cathode rays, cathode ray tubes,
discharge tubes, Aston dark space,
cathode glow, Crookes dark space,
negative glow, Faraday dark space,
positive column, anode glow,
cyclotron motion, oscilloscope, CRO,
timebase, sawtooth, blanking,
shadow mask

Try This!
Human-powered lighting
You can make a fluorescent light
tube temporarily give out flashes
of light by rubbing it quickly
with a piece of fur or wool in
a darkened room.

Figure 8.1.1 The high voltage produced


in static electricity will
temporarily light up a
fluorescent tube.
156

In todays terminology, cathode rays are collimated beams of


electrons in an evacuated vessel. They were given this label at
a time when not even the structure of the atom was known.
The discovery of these mysterious rays led to the discovery of the
electron, and paved the way not only for understanding the atom
but has also led to technology such as television, radar,
electronics, the oscilloscope and lighting.

8.1 Cathode ray tubes


Physicists began passing electrical current through air at lower pressure as soon as
the vacuum pump was invented by Otto von Guericke in 1650. Among the most
notable physicists to experiment with this was Michael Faraday, who, in 1838,
reduced the pressure of air in a glass vessel and applied a high voltage to
electrodes imbedded at each end of the tube. He noted that a narrow channel
of light was produced between the negative (cathode) and the positive (anode)
electrodes. Today we use this phenomenon to produce lighting in all shapes and
sizes (such as neon signs) and are generally known as discharge tubes.
In 1855 Heinrich Geissler (18151879), a master glassblower and skilled
Geissler, in collaboration
instrument maker, improved the vacuum pump.
with the physicist Julius Plcker (18011868), reduced the pressure in the tubes
to the point where the colours and patterns of the gaseous discharge disappeared
and were replaced by a green glow (also known as fluorescence) that came from
the glass. Moreover, Geissler was able to seal the tubes and maintain the very low
pressure such that a vacuum pump was no longer needed. Julius Plcker called
these tubes Geissler tubes.

from ideas to
Implementation
near vacuum

We now know that this green glow is caused by electrons leaving the
cathode and striking the glass at high speeds, causing it to glow, but during
Geissler and Plckers time this was a complete mystery. The name cathode rays
was first used by Eugene Goldstein, a German physicist, who coined the name
because it appeared that unknown beams were being emitted by the cathode.
The tubes displaying this property came to be known as cathode ray tubes.
The next great challenge for physicists at the time was to find the composition
of these cathode rays from their behaviour.

cathode

The cathode ray debate: charged particles or


electromagnetic waves?

Figure 8.1.2 A high voltage across

A variety of cathode ray tubes were made by a number of physicists in order


to study the behaviour of cathode rays and hopefully determine their nature.
In about 1875, the most prominent of these physicists Sir William
Crookes placed a piece of metal in the shape of a Maltese cross in the path of
the cathode rays as shown in Figure 8.1.3. This produced a sharp shadow on
the glass behind the cross, indicating that cathode rays travel in a straight line.
A magnetic field at right angles to the direction of the beam caused it
to deflect in the same way that negatively charged particles would deflect (see
Figure 8.1.4). Crookes concluded that cathode rays consisted of negatively
charged particles.
A moveable paddle wheel struck by the cathode rays started to rotate
and roll along the tube (see Figure 8.1.8). This implied that cathode rays carried
energy and momentum. Crookes also went on to show that the properties of
cathode rays did not depend on the type of cathode material.
From all of these experiments, the British physicists (such as Crookes)
concluded that cathode rays were beams of negatively charged sub-microscopic
particles. However, German physicists such as Heinrich Hertz (18571894)
carried out experiments that seemed to contradict the British viewpoint.
Hertz was unable to deflect the cathode rays with an electric field between
two parallel plates placed inside the tube. This suggested that cathode rays did
He also found that cathode rays could pass through very
not have a charge.
thin gold foil without damaging it. Thus Hertz and other German physicists
concluded that cathode rays had no mass or electric charge, and were possibly
a form of electromagnetic radiation.
There were also objections to the paddle wheel experiment because it was
thought that the cathode ray heated one side of a paddle, causing the gas in
contact with it to heat up and expand thus causing the paddle, to move.
The German physicists knew that electrical current can cause a magnetic
field, but they were unable to detect such a field around the cathode rays. These
contradictions strengthened their view of the wave nature of the cathode rays.
From a modern perspective, we can easily resolve the debate between
the British and German physicists. Cathode rays pass through a thin gold foil
because the space between atoms is much larger than the size of an electron, and
so a few atomic layers of gold allow the electrons to pass without much chance
of a collision with a gold atom. However, the atomic structure of matter was
unknown at the time. It was difficult to measure a magnetic field due to the
cathode rays because the current was extremely smalla very sensitive magnetic
field probe is needed to make such a measurement.

anode

+
glass tube

switch

high voltage
(variable)

electrodes at each end of an


evacuated glass tube causes
the glass near the positive
electrode (anode) to glow.
Explain that cathode ray tubes
allowed the manipulation of
a stream of charged particles.
Crookes tube
(cathode ray tube)
cathode

anode

mask holder

Figure 8.1.3 A Maltese cross placed in the


path of cathode rays produces
a sharp shadow on the glass
indicating that cathode rays
travel in a straight line.

cathode

collimator
S

anode

magnet

Figure 8.1.4 Cathode rays are deflected by


a magnetic field in the same
way that negatively charged
particles are deflected.
Explain why the apparent
inconsistent behaviour of
cathode rays caused debate
as to whether they were
charged particles or
electromagnetic waves.

157

From
cathode rays
to television

PHYSICS FEATURE
Anatomy of a discharge tube

anode dark

Aston
Crookes
space
nergy-efficient lighting, such as fluorescent
Faraday
tubes and neon signs, are known as discharge

+
tubes. These are evacuated glass vessels filled with
negative
gas at approximately 1% of atmospheric pressure
cathode
positive
anode
glow
and two electrodes at opposite ends of the tube.
glow
column
glow
A large potential difference between these
electrodes causes an electrical current to flow
Figure 8.1.5 A discharge tube showing the typical bright and
dark spaces
through the gas in the tube, resulting in colours
that depend on the type of gas and its pressure.
Bright regions in the discharge are areas where the
Figure 8.1.5 illustrates an operating discharge
electrons have sufficient energy to ionise the gas.
tube with the characteristic bright and dark spaces.
Some of the ions recombine with electrons, which
The Aston dark space next to the cathode is very thin
results in the emission of light. Not all atoms will be
and may go unnoticed. This is followed by the first
ionised. Some will simply have their electrons gain
luminous region called the cathode glow. Next is the
energy while remaining bound to the atom or
Crookes dark space and the negative glow (a luminous
molecule; this is known as excitation. All excited
region). The Faraday dark space follows this and is the
electrons will fall back to their previous energy state
largest of all the dark spaces. The largest luminous
and, in doing so, give out light. A minimum energy is
region, known as the positive column, follows and is
required to excite or ionise an atom or molecule.
the most prominent feature of the discharge. The
Many discharge characteristics are related to the
positive column may display periodic regions of bright
average
distance that electrons travel between
and dark spaces known as striations. Finally, there
colliding with the gas atoms or molecules. An increase
is the anode dark space and then the anode glow
in gas pressure leads to an increase in the density of
adjacent to the anode.
gas particles, and results in an electron travelling a
An operating discharge tube consists of a mixture
shorter distance between collisions, and vice versa.
of ions, electrons and neutral gas atoms. The
At low pressure, electrons have sufficient distance
discharge starts because energetic particles (such as
to accelerate and reach the required ionisation energy;
electrons and protons) continually stream from outside
however, they may strike the sides of the vessel more
the Earth and strike the surface and atmosphere.
often than they strike the gas. A minimum number of
This so called cosmic radiation can strike gas atoms
ionisations are required to sustain the avalanche of
and produce free ions and electrons in the discharge
electrons that maintain the discharge. Below a
tube. An electric field causes the free electrons to
minimum pressure, it becomes difficult to start an
gain sufficient energy to ionise gas atoms and
electrical discharge along the tube.
produce further free ions and electrons, which in
turn will accelerate to produce more ionisation. This
avalanche of ionisation is the way an electrical
+ Figure 8.1.6 Ions that strike the
discharge is started.
cathode liberate

To maintain the current through the discharge,


secondary electrons,
e
electrons (known as secondary electrons) are
which help sustain

e
the discharge.
continually ejected from the cathode surface by the
+
bombardment of ions that are attracted to it. These
secondary electrons cause further ionisation, which
PRACTICAL
results in further ion bombardment of the cathode
EXPERIEN
CES
surface, and the process repeats.

Activity 8.1

158

Activity Manual, Page


68

from ideas to
Implementation

Anatomy of a discharge tube


(continued)
At higher pressures there is a large density of gas
particles, so there is a high probability that electrons
will strike them. However, too high a pressure results
in very frequent electron collisions such that they do
not reach sufficient energy to ionise gas particles.
Thus, the electric field strength must be increased for
them to reach this energy. There is an optimum
pressure for starting a discharge at a relatively low
electric field.
Electrons in dark regions have not reached
sufficient energy to ionise or excite the gas. Figure
8.1.7 shows the potential difference and the
magnitude of the electric field strength between the
cathode and anode during a discharge. The largest
potential difference occurs between the negative glow
and the cathode. This is due to the ions being
attracted to the cathode and thus crowding around it.
Any electron outside this region will see an almost
equal amount of positive charge (from the ions) and
negative charge (from the cathode), resulting in a
much lower electric field between the negative glow
and the cathode.
Let us now trace the path of electrons from
cathode to anode. Electrons ejected from the cathode
are accelerated by the high electric field until they
have sufficient energy to cause the cathode glow. This
depletes the electron energy gained, so they must
accelerate once more through the cathode dark space
until they have sufficient energy to cause the negative
glow. Once again this process depletes their energy,
and they must accelerate through the Faraday dark
space until they gain sufficient energy to cause the
ionisation and excitation in the positive column.
Striations may result in the positive column for the
same reasons that other bright regions occur. However,
the electric field is more uniform in this region, so the
bright and dark regions (the striations) are uniformly
spaced. Again, electrons gain sufficient energy to
ionise and excite the gas, which causes a luminous
region and depletes the energy of the electrons. The
electrons then accelerate through a region (a dark
space) until they gain sufficient energy to excite and
ionise the gas, and so on.

Distance along the tube

Figure 8.1.7 The potential difference V and the magnitude of


the electric field strength E along the axis of a
discharge tube

Clear luminous and dark regions will be produced


if electrons that leave the cathode remain in step. This
is not possible in practice. Some electrons will strike
gas particles before others and have their energy
depleted sooner. This results in a spread of energies
as electrons move along the tube. Thus, the luminous
regions are diffuse and do not have well-defined
boundaries. If electrons become too out-of-step with
each other, then the striations will start to merge into
each other and will not be distinguishable from the
dark spaces between them. This may happen at too
high a pressure.
As the pressure is lowered, striations become more
widely spaced because there is a lower gas particle
density and the electrons will travel longer, on
average, before striking a gas particle. The converse
happens at higher pressures. Electrons strike gas
particles more frequently, so the striations will be
more closely spaced.
Finally, electrons are collected by the anode, which
also repels any ions, thus forming the anode dark
space. This is a region of high electric field that
causes the electrons to accelerate to the anode and
to excite or ionise the gas around it, causing the
anode glow.
As a rule, the positive column is less bright than
the negative glow and differently coloured. Helium
results in a red cathode glow, a green negative glow,
and a reddish-purple positive column. Neon produces
yellow, orange and red in the respective regions.
Nitrogen emits pink, blue and red. Each gas has a
characteristic set of colours, depending on the degree
of excitation that a bound electron experiences.

159

From
cathode rays
to television

cathode

anode

collimator

paddle wheel

Figure 8.1.8 Cathode rays caused a


paddle wheel to turn,
implying they carried both
momentum and energy.

+ + ++ +

+
cathode rays are not deflected,
and behave in the same way as light

Finally, the apparent lack of deflection of the cathode rays by an


electric field arose because the cathode ray tubes were not in a total vacuum
there was residual air in the tube, albeit at a low pressure. A fraction of the
electrons in the beams (cathode rays) struck air molecules and ionised them. As
shown in Figure 8.1.9, the positively charged ions are attracted to the negatively
charged plate and the electrons are attracted to the positively charged plate. This
essentially neutralises the charge on the plates and results in zero electric field
between them. Therefore, there can be no deflection of the cathode ray.
A resolution to the debate started with the French physicist Jean Perrin
(18701942), who showed that a metal plate acquired a negative charge when it
was struck by cathode rays. British physicist, Joseph John Thomson (18561940)
was able to definitively show that cathode rays can be deflected by an electric
field simply by producing an even lower pressure in the tubethus reducing the
number of ions. Moreover, he measured the charge to mass ratio of electrons.
To quantitatively examine his work, we must first revise the motion of electric
charges in electric and magnetic fields.

Figure 8.1.9 The cathode ray is not deflected by oppositely


charged plates because ions and electrons
neutralise the charge on the plates.

Checkpoint 8.1
1
2
3
4
5
6
7
8

Activity Manual, Page


72

Describe Faradays work with cathode ray tubes.


Outline the contribution of Geissler and Plcker to the development of discharge tubes.
Recall the different types of cathode ray tubes that were developed to understand cathode rays.
Create a table that lists the observations and inferences made from each of the tubes.
List the evidence that led Hertz to believe that cathode rays were not particles.
Give a reason for not being able to detect the magnetic field from cathode rays.
Explain why Hertz could not detect a deflection when he applied an electric field across the cathode rays.
Describe how the debate about whether cathode rays were particles or waves was finally resolved.

Figure 8.2.1 The electric field lines for


(a) a positive point charge
radiate outwards, and
(b) a negative point charge
radiate inwards.

Discuss qualitatively the


electric field strength due to
a point charge, positive and
negative charges and oppositely
charged parallel plates.

160

PRACTICAL
EXPERIENCES
Activity 8.2

8.2 Charges in electric fields


Any region in space in which there is an electric force on a charged object is
said to contain an electric field. The field points in the direction of the force on a
very small positive charge, known as a test charge. The magnitude of the field on
the test chargethe electric field strengthincreases in proportion to the force.
A way of visualising the electric field is to draw lines that indicate the
direction of the force on the test charge. Closely spaced lines mean higher electric
field strength and vice versa. Equally spaced lines indicate it is a uniform field.
Electric field lines are not real lines. They are used to give a qualitative
description of the field. We can only draw a finite number of lines. The field is
actually continuousit exists everywhere in space.
The electric field lines around a point positive or negative charge are shown
in Figure 8.2.1. Although this is a two-dimensional drawing, the lines actually
radiate outwards for a positive charge and inward for a negative charge in three

from ideas to
Implementation
dimensions. The greatest repulsive or attractive force is in the region of the lines
with the closest spacingnear the charge in this case.
There are some guidelines for drawing electric field lines for two or more
charges:
The lines must begin on a positive charge and/or end on a negative charge.
Larger charges have more lines starting or ending on them.
Lines cannot cross.
Lines are always at right angles to a conducting surface.
The electric field lines for a pair of opposite and equal point charges are shown
in Figure 8.2.2. The number of field lines leaving the positive charge is equal to
the number of lines ending on the negative charge. The lines are radial at very
close distances to the charges. These lines are more closely spaced near the charges
to indicate higher electric field strength in that region.
The electric field lines between two equal charges q of the same sign are shown
in Figure 8.2.3. The region at the centre, between the charges, is where the
electric field strength is zero, because the electric fields of the two charges cancel
each other.
The field lines between two charged parallel metal plates of opposite sign but
Equally spaced and parallel
equal magnitude are shown in Figure 8.2.4.
lines indicate that the field is equal in magnitude and direction and is said to be
uniform. The field lines start to curve near the edges and become unevenly
spaced, indicating a non-uniform field (called the edge-effect).
b

Figure 8.2.2 The electric field lines for


two point charges of equal
magnitude but opposite sign

Identify that charged plates


produce an electric field.

negative

positive

Figure 8.2.3 Electric field lines around (a) two equal positive charges,
and (b) two equal negative charges

Electric field strength between parallel plates


A side-view of a uniform field between two plates is shown in Figure 8.2.5
(we have ignored edge effects).
The electric field strength E was defined as the ratio of the force F on a small
positive charge q given by:
F
E =
q
(see in2 Physics @ Preliminary section 10.6).
It has units of force per unit charge, which in SI units is newtons per
coulomb or N/C or NC1. However, in practice neither the force between the
plates nor the charge on them is easily measured. We need a more convenient
expression in terms potential difference V between the plates.
The equal and opposite charges on the plates were produced by applying a
potential difference V between them, using a power supply or a battery. Energy
from the power supply moved the electrons from one plate to the other, resulting
in equal and opposite charges on the plates. Recall that it was shown that the

Figure 8.2.4 The electric field lines between


two oppositely charged parallel
metal plates
negative
+

E
+ + + + + + + + + + + positive

Figure 8.2.5 Parallel plates with a uniform


electric field between them

Describe quantitatively the


electric field due to oppositely
charged parallel plates.

161

From
cathode rays
to television
energy required to move an object in the direction of the force is known as work
and is defined as:
Work = force distance = W = F d
(see in2 Physics @ Preliminary section 4.3).
In this case, the displacement d is the separation of the parallel plates.
Rearranging this formula as follows:
W
F =
d
and substituting this expression for force in the expression for electric field given
previously, we obtain:
W
E=
qd
Recall also that the work done on charges is related to the potential difference
V by the following expression:
W=qV
(see in2 Physics @ Preliminary section 10.8).
Substituting this into the previous expression for electric field strength,
we obtain:
q V
E=
qd
E=

Thus, the electric field strength E is easily calculated from the plate separation
d and the potential difference between them V. From this equation, you can see
that an alternative unit for the electric field strength is volts per metre (V m1).

Solve problem and analyse


information using:
F = qvBsin
F = qE
and
V
E=
d

Worked example
Question

e
E
+ + + + + + + + + + +
L

Figure 8.2.6 Two parallel plates with


a uniform electric field

V
d

Two parallel plates are separated by d = 1.0 cm and have a side length of L = 2.0 cm
(Figure 8.2.6). A potential difference of V=10V is applied between them.
An electron enters halfway between the plates with a velocity of v0 = 2.7 106 ms1
parallel to the plates.
Calculate:
a the electric field strength E between the plates
b the vertical force F experienced by the electron as it travels between the plates
c the acceleration of the electron
d the time it takes for the electron to travel the full length of the plates
e the vertical displacement of the electron just as it exits the plates
f the change in kinetic energy of the electron between its entry to and exit from
the plates.
Note that the electron charge e = 1.6 1019 C, and its mass me = 9.11 1031 kg.

Solution
a The electric field strength E is given by:
V
10
E= =
= 1000 V m1 (or N C1 )
d 0.010
162

from ideas to
Implementation
b The vertical force is given by:
F = e E = 1.6 1019 1000 = 1.6 1016 N upwards
c The acceleration can be obtained from Newtons second law (see section 3.4 of
in2 Physics @ Preliminary):
a=

F
1.6 1016
=
= 1.76 1014 m s2 = 1.8 1014 m s2 downwards
me 9.11 1031

d The motion of the electron is similar to projectile motion (see Module 1 Space).
There is only acceleration in the vertical direction. We are concerned with the
horizontal component of the velocity, which remains constant. Therefore, the time t
to travel the length L of the plates is given by:
t=

L
0.020
=
= 7.4 109 s
v 0 2.7 106

e The electron only accelerates vertically, so we use the acceleration from part c and
one of the SUVAT equations (see in2 Physics @ Preliminary section 1.3) to work out
the vertical displacement s given by:
1
s = ut + at 2
2

where u is the initial vertical velocity = 0 ms1, t is the time to travel the length of
the plates (from part d), and a is the vertical acceleration = 1.8 1014 ms2 (from
part c). Therefore, the displacement is given by:

1
s = 0 + 1.8 1014 (7.4 109 )2 = 0.0049 m = 4.9 mm
2
f The change in kinetic energy is due to the vertical deflection of the electron by the
electric field. This is work done by the field on the electron, and is given by:
W = e V

where V is the change in potential that the electron experiences due to its vertical
deflection s. The relationship between potential difference and the displacement s
is given by:
V = E s

Combining this with the above expression for work gives:


W = e E s = 1.6 1019 1000 0.0049 = 7.8 1019 J

Checkpoint 8.2
1
2
3

Explain the meaning of the arrows and spacing of lines when drawing electric field lines.
Calculate the electric field strength at the location of a charge of 1.28 1018 C that experiences a force of
1.1 1018N.
Sketch the electric field lines around:
a a point positive charge
b two oppositely charged parallel metal plates.

163

From
cathode rays
to television

8.3 Charges moving in


a magnetic field

r
F
+
+q

Figure 8.3.1 A positively charged particle


enters into a magnetic field
directed into the page and
undergoes anticlockwise
circular motion.
Identify that moving charged
particles in a magnetic field
experience a force.
Describe quantitatively the
force acting on a charge
moving through a magnetic
field:
F = qvBsin

A charged particle moving in a magnetic field will experience a force, as discussed


in Module 2 Motors and generators. Consider a magnetic field directed into the
pages (see Figure 8.3.1) and a positively charged particle that enters the field
The particle will experience a force as given by the rightfrom the left.
hand palm rule (refer to Module 2 Motors and generators). The force F on the
particle changes direction every time the particle changes direction, such that the
resulting motion is circular. This circular motion is known as cyclotron motion.
A negatively charged particle will undergo circular motion in the opposite sense.
The magnitude of the force F on a charge q that moves with a speed v
at right angles to a magnetic field B is given by:
F = qvB
The speed of the charged object remains constant even though its
direction changes. This means that the magnetic force simply changes the
direction of the particle without adding or subtracting energy, provided that the
magnetic field is constant. In general, the magnitude of the force on a charged
particle with a velocity at an angle with respect to the magnetic field is given by:
F = qvBsin
Note that a particle that is parallel to the field ( = 0) does not experience
a force. A charged particle that enters a magnetic field at an angle other than
= 0 or 90 travels in a spiral along the magnetic field (also known as helical
motion), as shown in Figure 8.3.2. However, the component of the velocity at
right angles to the field is still circular.
A particle of mass m with charge q enters a uniform magnetic field B at an
angle and a speed v. Recall from the Module 1 Space that the force directed
towards the centre of circular motion is known as the centripetal force. In this
case, the centripetal force is the magnetic force; that is:

centripetal force = magnetic force

Solve problem and analyse


information using:
F = qvBsin
F = qE
and
V
E=
d

mv 2
= qvB sin
r
where r is the radius of curvature of the particle.
Rearranging this expression, we obtain the following expression for the radius:
mv
r=
qB sin
c

b
+

v sin

v
+
B

v
v cos

Figure 8.3.2 (a) A proton enters a magnetic field at an angle . (b) The component of the velocity at right angles to the field undergoes circular motion.
(c) The total motion is helical along the magnetic field.

164

from ideas to
Implementation
Worked example
Question
Protons are occasionally ejected from the Sun at high speeds towards the Earth. These can be
caught in the Earths magnetic field and are trapped along magnetic field lines in a region
known as the Van Allen belt. A proton travelling at a speed of v = 1.0 107 m s1 strikes the
Earths magnetic field at an angle of 30 at a distance of 3000 km above the surface where
the magnetic field B = 3.5 105 T. Assume that the Earths magnetic field at this distance
is uniform. The charge on a proton is 1.61019 C and it has a mass of 1.671027 kg.
a Calculate the magnitude of the force on the proton.
b Calculate the radius of curvature of the proton.
c Determine if the cyclotron motion of the proton will cause it to come in contact with
the Earth.

Solution
a The magnitude of the force is given by:

F = qvB sin = 1.6 1019 1.0 107 3.5 105 sin 30 = 2.8 10 17 N

b The radius of curvature r is given by:


r=

mv
1.67 1027 1.0 107
= 6.0 103 m
=
qB sin 1.6 1019 3.5 105 sin 30

c No, the protons cyclotron motion will not cause it to come in contact with the Earths
surface, since the cyclotron radius is 6 km and the proton is 3000 km above the surface.

Checkpoint 8.3
1
2

Outline what will happen to a negative charge moving to the left at right angles to a magnetic field that is
directed out of the page.
Calculate the magnitude of the force experienced by an electron that travels at right angles to a magnetic field
of 2T at a speed of 3 ms1.

8.4 Thomsons experiment


In 1897 Joseph John (JJ) Thomson not only provided a definitive resolution
to the debate about whether cathode rays were particles or electromagnetic waves,
but he also measured the charge to mass ratio of the main constituent of cathode
raysthe electron.
Thomsons cathode ray tube is shown schematically in Figure 8.4.1. Cathodes
rays (electrons) were accelerated from the cathode to the anode, which consisted
of two anodes aligned along the axis of the tube and separated by a small distance.
Each anode had a horizontal slit cut into it so that cathode rays could pass through.
The separation of the anodes produced a flat beam that passed between two
parallel plates and struck the end of the tube, which had a fluorescent screen
painted on the inside surface. The beam produced a well-defined narrow
horizontal line on the screen.

Outline Thomsons experiment


to measure the charge/mass
ratio of an electron.

165

From
cathode rays
to television
magnetic field
electromagnet
cathode rays

large
voltage

Figure 8.4.1 Thomsons cathode ray tube


apparatus for measuring the
charge to mass ratio of electrons

fluorescent
screen

cathode
anodes

electromagnet

charged plates

The parallel plates deflected the beam vertically upwards by placing a positive
charge on the top plate. A magnetic field (using an electromagnet outside the tube)
was applied at right angles to the electric field and the direction of the beam. The
direction of the magnetic field was such that it deflected the beam downwards.
Adjusting the magnetic and electric forces such that they were equal and
opposite resulted in the beam passing through undeflected.
We now quantitatively describe this experiment and how it leads to the charge
to mass ratio of the electron.
Equating the electric to the magnetic force on a particle with a charge q and
speed v in a magnetic field B and an electric field E, we obtain the following
expression:
Magnetic force = electric force
qvB = qE

From this we obtain an expression for the speed:


E
v=
B
Recall that the relationship between the magnetic force and centripetal force
(see section 8.3) is given by:
mv 2
= qvB
r
This enables an expression for charge to mass ratio to be obtained:
q
v
=
m rB
Substituting the expression for v into this equation:
q
E
= 2
m rB

Thomson was able to calculate the radius of curvature r from the deflection of
the beam on the fluorescent screen when the electric field was switched off and
magnetic field switched on. He could calculate the magnitude of the electric field
E because he knew the spacing d of the plates and the potential difference
V between them, and made use of the relationship E=V/d. Finally, knowing the
number of turns in the wires of the electromagnet and the current flowing through
it, he was able to calculate the magnitude of the magnetic field B.
Thomson found that the charge to mass ratio always came to:
q
= 1.76 1011 C kg 1
m
regardless of the cathode material, indicating that a fundamental particle was
being emitted. This, in essence, marks the discovery of the electron. In 1891,
the Irish physicist George Johnstone Stoney (18261911) suggested that the
fundamental unit of electricity be called an electron6 years before Thomsons
publication of his now famous experiment.
166

from ideas to
Implementation

Checkpoint 8.4
1
2

Explain the purpose of Thomsons experiment.


Describe how Thomsons experiment obtained the charge to mass ratio of the electron.

8.5 Applications of cathode rays


A cathode ray oscilloscope (or simply oscilloscope or CRO)
is a device used to measure the variation of voltage in time
across an electrical component. Both the CRO and TV have
elements in common with Thomsons original cathode ray
tube apparatus. Here we are not referring to the new plasma
or LCD television sets, but the older style scanning electron
beam sets.

Cathode ray oscilloscope (CRO)


The display screen of a cathode ray oscilloscope, as seen by
the user, is shown in Figure 8.5.1. Despite the many knobs
and switches on its front control panel, the CRO simply
displays a real-time graph of voltage (y-axis) versus time (x-axis).
The oscilloscope has had many uses by scientists and
Figure 8.5.1 The display screen and the control
technicians in the design and operation of electronic and
panel of a cathode ray oscilloscope
electrical equipment. The way this real-time graph of voltage
versus time was achieved can be understood by looking at the
path of electrons
horizontally
electron gun
main components inside the CRO. A schematic diagram of a
deflecting plates
bright spot
cathode
anode
CRO (see Figure 8.5.2) consists of an electron gun, horizontal
on screen
and vertical deflection plates and a fluorescent screen.
where
heater
electrons hit
The electron gun (see Figure 8.5.3) produces and
accelerates the electron beam. It consists of a heater, cathode
fluorescent
screen
and anode. The heater is a wire filament with a large enough
current flowing through it so that it reaches a high temperature.
vertically deflecting plates
Electrons in the wire then acquire enough energy to escape the
wire and enter the region between cathode and anode. They accelerate toward Figure 8.5.2 A cathode ray oscilloscope deflects
an electron beam with vertical and
the anode and pass through a hole at its centre. The shape of the anode and its
horizontal electric fields between
separation from the cathode enables a narrow and focused beam of electrons to
parallel plates.
travel to the fluorescent screen. Some electron guns can become more
sophisticated than the description given here, to achieve better focusing.
The horizontal deflection plates cause the beam to sweep horizontally across
the fluorescent screen (from left to right) by periodically changing electric field
Outline the role of:
between the platesthis is the time axis. The speed at which the beam sweeps
electrodes in the electron gun
across the screen is controlled by the timebase dial on the front control panel of
the deflection plates or coils
the CRO. This changes the frequency at which the beam sweeps across the
the fluorescent screen
screen. The waveform of the potential difference across the horizontal plates is

in the cathode ray tube of
shown in Figure 8.5.4 and is commonly known as a sawtooth waveform.
conventional TV displays and
oscilloscopes.
Electronics in the CRO blocks the beam on its way back to the left of the screen
so that it doesnt retrace itself (known as blanking).

167

From
cathode rays
to television
anode (positive)

electrons
'boil' off
the heated
cathode

collimator

electron
beam

heater
cathode (negative)
electrons attracted
to the positive anode

Figure 8.5.3 The components of an electron gun used in both cathode ray
oscilloscopes and CRT televisions

Time

sawtooth voltage for timebase

Time
sinusoidal vertical voltage

Figure 8.5.4 A sawtooth voltage waveform on the horizontal


deflection plates of a CRO sweeps the electron beam
across the screen to display the sinusoidal waveform
on the vertical deflection plates.

The vertical deflection plates cause the beam to move up or down in


synchronisation with an input voltage. For example, a sinusoidal voltage will
display a sinusoidal waveform (known as a trace) on the screen.

Television
electron gun
electron
beam

magnetic
coils

fluorescent screen

Figure 8.5.5 A television picture tube


showing the electron gun,
deflection coils and
fluorescent screen

168

Cathode ray tube (CRT) television sets used the principles of the cathode ray
tube for most of the 20th century. These are now being superseded by plasma
and liquid crystal display television sets, which use different operating principles
and allow a larger display area with a sharper image. However, the CRT
television holds quite a significant historical place in this form of
communication.
A schematic diagram of a colour CRT television set is shown in Figure 8.5.5.
Its basic elements are similar to those of the CRO. The main difference is the
method of deflecting the electrons. Magnetic field coils placed outside the tube
produce horizontal and vertical magnetic fields inside it. The magnitude and
direction of the current determine the degree and direction of electron beam
deflection. Recall your right-hand palm rule for the force on charged particles
in a magnetic field. The vertical magnetic field will deflect the electrons
horizontally; the horizontal field will deflect them vertically.
The picture on the screen is formed by scanning the beam from left to right
and top to bottom. The electronics in the television switches the beam on and
off at the appropriate spots on the screen in order to reproduce the transmitted
picture. However, to reproduce colour images, colour television sets need to
control the intensity of red, blue and green phosphors on the screen. Three
separate electron guns are used, each one aimed at one particular colour. The
coloured dots on the screen are clustered in groups of red, blue and green dots
that are very close to each other and generally cannot be distinguished by eye
without the aid of a magnifying glass. For this reason a method of guiding the
different electron beams to their respective coloured dots was devised. A metal
sheet, known as a shadow mask (Figure 8.5.6) and consisting of an array of
holes, is placed behind the phosphor screen. Each hole guides the three beams to
their respective coloured phosphor as the beams move horizontally and vertically.
Black and white television sets did not need the shadow mask since they had
only one beam.

from ideas to
Implementation
glass

deflecting
coils

mask

electron
guns

blue
beam
red
beam

R
G
B

vacuum

mask

phosphor dots
on screen

fluorescent
screen

green
beam

focusing
coils
fluorescent
screen

Try this!
Do not adjust your
horizontal!
If you have access to an old
black and white TV set or an old
style monochrome computer
monitor, try holding a bar
magnet near the front of the
screen and watch how the
image distorts. This occurs
because the magnetic field
deflects the electrons that strike
the screen. DO NOT do this with
a colour TV set. This can
magnetise the shadow mask and
cause permanent distortion of
the image and its colour. You
can move a bar magnet near the
back of a colour TV set to
deflect the electrons from the
electron gun and therefore
distort or shift the image
without causing permanent
damage to the TV set.

holes in
mask

electron
beams

Figure 8.5.6 A colour CRT television set has three electron guns
that will only strike their respective coloured
phosphor dots with the aid of a shadow mask.

Can an oscilloscope be used


as a television set?

he similarity between the cathode ray oscilloscope (CRO) and CRT


television suggests that a CRO can be used as a television set. In
fact, there have been some devices that have made use of the CRO as
you would a computer monitor. So, in principle, it can be used as a
television. One is then forced to ask why did they need to deflect the
beam in a television set with magnetic fields rather than with electric
fields as in the CRO?
In principle all television sets could be made in the same design as
a CRO; however, it is much easier and cheaper to deflect the beam with
a magnetic field on the outside of the tube rather than embed electrodes
in the glass and inside the vacuumthis is a little trickier. So now
another question arises: why not deflect the beam of the CRO using
magnetic fields, wouldnt it result in cheaper CROs?
Cathode ray oscilloscopes are precision instruments. The horizontal
sweep rate must be able to be increased to very high frequencies in order
to detect signals that change very quickly. Electric fields can be made to
change very quickly without significant extra power requirements.
However, a magnetically deflected system requires higher and higher
voltages with increasing horizontal and vertical deflection frequencies in
order to maintain the same current in the coils, and therefore, the same
angle of beam deflection thus having a significantly greater power
requirement. Cathode ray tube television sets, however, only operate at
fixed and relatively low scanning horizontal and vertical frequencies.
Thus it is simpler and cheaper for the mass market to deflect with a
magnetic field.

Checkpoint 8.5
1
2
3
4

Outline the purpose of a CRO.


List the main parts of a CRO.
Describe the role of each of these parts in the CRO.
State the similarities and differences between the cathode ray tube CRO and CRT TV.
169

From
cathode rays
to television

PRACTICAL EXPERIENCES
CHAPTER 8

This is a starting point to get you thinking about the mandatory practical
experiences outlined in the syllabus. For detailed instructions and advice, use
in2 Physics @ HSC Activity Manual.

Activity 8.1: Changing pressure of discharge tubes


Perform an investigation and
gather first-hand information to
observe the occurrence of
different striation patterns for
different pressures in discharge
tubes.

Connect a set of discharge tubes that are at different pressures to an induction coil
or high voltage power supply and observe the different striation patterns. The
patterns are hard to see unless the room is very dark.
Equipment: induction coil, connecting wires, discharge tubes at different
pressures, DC power supply.
Discussion questions
1 Draw a labelled set of diagrams showing the distinct patterns that occur
during the evacuation of the tube.
2 Describe the change in the striation pattern with changing pressure.

Activity 8.2: Cathode ray tubes evolve


Perform an investigation to
demonstrate and identify
properties of cathode rays
using discharge tubes:
containing a Maltese cross
containing electric plates
with a fluorescent display
screen
containing a glass wheel.
Analyse the information
gathered to determine the sign
of the charge on cathode rays.

170

Connect the induction coil or high voltage power supply to each of the cathode ray
tubes. Observe the behaviour of the electron beam under the influence of a
magnetic or an electric field. Note the result of placing a Maltese cross or a paddle
wheel in the path of the electron beam.
Equipment: induction coil, connecting wires, cathode ray tubes in the
following set-ups: Maltese cross, magnetic field (you can use a permanent
magnet), electric field, paddle wheel, DC power supply.
Discussion questions
1 List which experiment supported the idea that cathode rays were waves and
which supported the particle theory. State the observation that led to the
particle or wave conclusion.
2 Identify the properties of cathode rays that were determined from these
experiments.

Chapter summary


Cathode rays are electron beams in an evacuated vessel.


Cathode ray tubes are the evacuated glass vessels in
which cathode rays travel.
A discharge tube consists of an anode and a cathode at
the ends of a glass vessel filled with a low-pressure gas
that glows when a high voltage is applied between the
anode and the cathode.
Geissler tubes are sealed discharge tubes that do not
require a vacuum pump.
Cathode rays were thought to be negative particles due
to their direction of deflection when subjected to a
magnetic field.
Cathode rays moved a paddle wheel, implying they
carried momentum.
Initial inconclusive experiments led to the debate
between British and German scientists about whether
cathode rays consisted of particles or were a form of
electromagnetic wave.

from ideas to
Implementation

Thompsons experiment provided definitive proof of the


particle nature of cathode rays from their motion in
electric and magnetic fields. Moreover, he measured
the charge to mass ratio of the electron.
Cathode rays (or electron beams) are used in the
cathode ray oscilloscope (CRO) and cathode ray tube
(CRT) television.
The CRO is used to display a changing voltage in time.
The CRO deflects the electron beam with electric fields,
while the CRT television deflects them with magnetic
fields.
Colour CRT TV sets have three electrons beams
one for each of the coloured phosphors (red, green,
blue) on the screen.

Review questions
Physically Speaking

Reviewing

The items in the columns are not in their correct order. Copy
the table and match each of the definitions to the apparatus.

1 State the origin of the name cathode rays.


2 State one inconsistency of the behaviour of cathode

Apparatus

Definition

Oscilloscope

Shows that electrons travel in a straight line by


the shadow cast on the tube

Discharge tube

Uses magnetic fields to deflect electron beams

CRT TV

An instrument used to measure a time-varying


voltage

Maltese cross

A glass vessel containing gas at low pressure


that can be ionised by high voltage

rays and how it was solved.

3 Explain the significance of Jean Perrins experiment.


4 Describe the different components that constitute
a discharge tube.

5 Construct a table that names the features in a


discharge tube, their characteristics and positions.

6 Briefly explain how a discharge is started and


maintained in a discharge tube.

7 Describe how light is produced inside a discharge


tube, in terms of the electrons in an atom.

8 Explain what would happen when a positive charge


is placed in an electric field.

9 Estimate the difference in the force experienced by


a charge placed in an electric field when the charge
is tripled.

10 Describe how the force on a charge in a magnetic


field varies as the angle of entry changes.

11 Compare and contrast the method of producing an


image in a CRO and a CRT TV.

171

From
cathode rays
to television

solving Problems

16 Calculate the acceleration of the electron while in


an electric field strength of 1000V. The mass
of an electron is 9.11 1031 kg and its charge is
1.60 1019 C.

12 Calculate the electric field needed for an electron to

CRT
a force of 3.2 1016N.
phosphor experience SED
electron
phosphor
electron
13 Calculate the work
done in order to move an object
gun
emitter
through 10cm with a force of 3N.

14 Consider Figure 8.6.1.

deflecting
yoke

17 Using the information from Question 16, calculate the


final velocity of an electron initially at rest, when it
had travelled a distance of 1.00cm.

spacer

18 Calculate the force on an electron that enters


a magnetic field of 0.1 T at a speed of
3.2 106ms1.

100 V

10 mm

0V

Figure 8.6.1 A positive charge between two plates


a Calculate the work needed to move a positive
charge from the bottom plate to the top plate.
b Calculate the electric field between these
two plates.
c Determine the direction of the force experienced
by the charge.
d Calculate the magnitude of the force experienced
if the charge is a proton.

15 As a charge moves through an electric field it gains

Re

iew

172

Q uesti o

kinetic energy. Obtain an expression for the kinetic


energy gained in terms of the electric field strength E
and the distance s travelled by the charge q.

19 a Calculate the radius of curvature of a proton that

enters a magnetic field 0.1T at 2.1 106ms1.


b Determine the magnitude and direction of the
force that is experienced by the proton.

20 Using the value that Thomson determined for charge


to mass ratio 1.76 1011 C kg1 and the electron
charge of 1.6 1019 C, determine the mass of
an electron.

from ideas to
Implementation

PHYSICS FOCUS
Where to from here?

cience has come a long way since the first


experiments with discharge tubes. The applications
range from a CRO in scientific fields to TVs for the
general public. But development has continued.
TVs have changed; more and more households now
have either an LCD or plasma screen TV. There are
also surface-conduction electron-emitter display TVs
(SED-TVs), which are still at the planning stage. These
use the traditional cathode ray tube method but
instead of having one electron gun for each colour
(red, blue, green), these sets have a surfaceconducting electron emitter (SCE) for each coloured
phosphor dot on the screenthere may be up to a
million of them!
Each SCE is made of two carbon layers with a gap
between them; one with a negative electrode, the
other with a positive electrode. A voltage of only
10 volts is needed to make an electron appear at one
side of the gap. These cross the vacuum to strike the
different phosphor dots lining the glass, resulting in
a glow.
The big advantage with this system is that the
matrix that controls the SCEs allows each pixel to be
activated simultaneously rather than in the traditional
method of one row at a time.

luminescence
black
matrix

colour filter
glass substrate

electrode
Va

phosphor

metal back film


electron beams electron emitter
glass substrate
Vf

Va

field
emission
nanogap

scattering
several nm

Figure 8.6.2 A diagram of a surface-conduction electron-emitter


display TV

H3. Assesses the impact of particular


advances in physics on the
development of technologies

H4. Assesses the impacts of


applications of physics on society
and the environment

H5. Identifies possible future directions


of physics research

But development is not only on the entertainment


front. Lighting has benefited from discharge tube
technology. Commercial fluorescent lights have been
around since 1938. They consist of a vacuum tube
containing mercury vapour, filled with an inert gas
such as argon and coated with phosphor powder.
Neon lights are similar but are filled with inert
gases such as neon, argon and krypton. These glow
with different gas-dependent colours, when an
electrical current is passed through them.
The most recent and economical is the compact
fluorescent light. Set to eventually replace all
incandescent lights in Australia, these lights are
basically a twisted fluorescent tube.
1 List the parts that are needed to make a discharge
tube.
2 Outline the necessity for having low pressure inside
the tube.
3 Compare and contrast the Geissler tube and the
fluorescent light.
4 Determine why fluorescent lights are more efficient
than incandescent lights.
5 List advantages and disadvantages of SED TVs.
Suggest why a design that seems to encompass all
the features wanted by consumers is not yet being
made commercially.
6 Explain the need for magnetic coils in CRT TV sets.
7 Give reasons why magnetic coils are not needed in
the SED TV.

173

Electromagnetic
radiation: particles
or waves?
The wave nature of light

electromagnetic wave, transmitter,


receiver, standing waves, nodes,
anti-nodes, refraction, polarisation,
hertz, black body, classical theory,
ultraviolet catastrophe, quanta, photons,
spectrometer, Plancks constant,
photoelectric effect, photocathode,
photoelectrons, cut-off frequency, work
function, stopping potential, photocell,
dynodes, photomultiplier tube,
positron emission tomography (PET),
gamma photons

There was considerable debate about whether light was


a wave or a stream of particles, long before there was a
similar debate about cathode rays. However, by the
19th century there was growing experimental evidence
that light had wave properties, but the nature of these
waves was not known.

9.1 Hertzs experiments


on radio waves

PRACTICAL
EXPERIENCES
Activity 9.1

Activity Manual, Page


75

174

A suggestion about the nature of light waves came from the Scottish physicist
James Clerk Maxwell (18311879). From his studies of Michael Faradays
experiments with electricity and magnetism, he derived four fundamental
Remarkably, these
equations that linked electricity and magnetism.
equations predicted that oscillating electric charges should produce a wave that
travels through space at the speed of light. This wave consisted of oscillating
electric and magnetic fields at right angles to each other, and was called an
electromagnetic wave.
Although Maxwells equations did not conclusively prove that light was an
electromagnetic wave, he strongly suspected that it was, because the predicted
speed of these electromagnetic waves was the same as the speed of light.
Moreover, his equations predicted that all electromagnetic waves travelled at the
speed of light, regardless of their frequency. Sadly, he died at age 48 before his
theory could be tested experimentally.
In 1887 Heinrich Hertz (18571894) was the first to experimentally verify
Maxwells electromagnetic wave theory. Hertzs apparatus was essentially a radio
wave transmitter and receiver. Today, almost all of us carry out a more

from ideas to
Implementation
sophisticated version of Hertzs experiment when we tune in to a radio station.
More importantly, we are able to select different radio stations by their
Hertz needed to make an antenna that transmitted
transmitting frequency.
at a specific frequency and a receiver that was tuned to that frequency. This was
not such an easy task when electronics were not available.
Hertz used a number of different transmitters and receivers during the course
of his experiments. Each successive apparatus was refined to give more accurate
results. The basic principle of Hertzs apparatus is shown in Figure 9.1.1.
The transmitter consisted of a pair of metal rods placed end to end with a small
gap between them. He used an induction coil to place charges of opposite signs
on these rods at very large potential difference, causing a spark to jump across the
gap. This caused electrical current to oscillate back and forth across the gap and
along the rods, thus producing an electromagnetic wave. The frequency of the
oscillation was determined by the dimensions of the rods. His first apparatus
produced a frequency of 50 million cycles per second, but he had no way of
The mathematics for calculating the frequency was known
measuring that.
during Hertzs time and so he was able to calculate this oscillation frequency.
The electromagnetic wave emitted by the transmitter was detected by one of
several receivers. The receiver shown in Figure 9.1.1 is a loop of wire with a gap.
The natural oscillating frequency of this loop had to match the frequency
of the transmitter. The dimensions of the loop and gap determined this frequency
and were accurately calculated by Hertz. The receiver showed that a spark jumped
across the gap even if it was placed many metres away from the transmitter, thus
indicating that an electromagnetic wave has been transmitted in air.

Hertz measures the speed of radio waves


Hertz was able to obtain a fairly accurate measurement of the speed
of his radio waves by a number of methods. His most famous paper
for measuring their speed in air relied on the phenomenon of standing waves.
Recall that two waves of the same frequency, wavelength and
amplitude can form standing waves if they overlap while
travelling in opposite directions (see in2 Physics @ Preliminary
section 7.4). Figure 9.1.2 shows a standing wave produced by a
rope tied to a wall. The nodes are points of zero amplitude and
no motion (thus the term standing wave). The points of
maximum amplitude are known as anti-nodes.

plate
rod

spark gap

to induction coil

metal loop

Figure 9.1.1 General schematic representation


of all of Hertzs apparatuses, which
consisted of (a) a transmitting
dipole antenna and (b) a receiver
of various designs
Outline qualitatively Hertzs
experiments in measuring the
speed of radio waves and how
they relate to light waves.

TRY THIS!
Any experiment in a thunderstorm

A
N

A
N

Figure 9.1.2 A standing wave on a rope (A = anti-node, N = node)

Hertz formed a standing wave of electromagnetic


radiation by reflecting the radio waves from a large flat zinc
plate, as shown in Figure 9.1.6, in which only a standing wave
of the electric field is shown. Hertz then moved his receiver
coil along this wave. A spark in the gap was produced at the
anti-nodes but not at the nodes. The distance travelled between
nodes or anti-nodes is half a wavelength. Doubling this

You can reproduce Hertzs experiment at home.


The next time there is a thunderstorm, turn on
your radio to the AM band (FM does not work for
this experiment). Turn the tuning dial to a
frequency at which no radio station is
transmitting and listen. You will hear crackles
and hisses of thunder near and far. Incredibly,
you can hear more crackling than the number of
lightning flashes that you see, because not all
lightning flashes are large enough to be visible.
This is the essence of Hertzs experiment
electromagnetic waves produced by the motion
of charge in lightning travel to your radio
receiver at the speed of light and are detected.
175

Electromagnetic
radiation: particles
or waves?

PHYSICS FEATURE
The evolution of Hertzs
experiment
Hertzs first apparatus
produced an
electromagnetic wave
with a wavelength of
6 m. There are
difficulties with
precision experiments
when using such a long
wavelength. You need to
be far enough away from
the transmitter so that
you are not simply
detecting a spark in your Figure 9.1.3 Heinrich Hertz was a
receiving antenna due to
skilled experimental
physicist.
purely electrostatic effects
as a result of being too
close to the high-voltage induction coil. Moreover, the
spark in the detecting antenna becomes very weak if
the distance between it and the transmitter is too
great. You have to contend with reflections from the
walls of the room and the objects within it. You must
also face the problem of tuning the transmitter and
receiver to the same frequency when there is no
instrument that can help you. Heinrich Hertz
overcame all of these issues.
Figure 9.1.4 shows the evolution of the different
types of transmitters and receivers used by Hertz. The
first transmitter consisted of zinc spheres at the ends
of the two rods (see Figure 9.1.4a). The antenna could
be tuned by sliding the spheres along the rods. The
receiver was a square loop of wire with a gap. The
second type of antenna, was tuned by changing the
area of the flat plates at the end of the rods (see
Figure 9.1.4b). The receiver was a circular loop with a
gap. The first two antennas produced a wavelength of
6 m. The next generation of transmitter and detector
(Figure 9.1.4c) was a smaller antenna and a similar
receiver (no loop) for a wavelength of 0.66m.

176

transmitters
a

receivers

6m

6m

0.66 m

Figure 9.1.4 The different types of antennas and receivers used


by Hertz include (a) a dipole transmitting antenna
with spheres and a square loop detector for 6m
wavelength, (b)a transmitting antenna with square
plates and a circular loop receiver for 6m wavelength
and (c)a small dipole antenna and similar dipole
receiver for 0.66m wavelength.
parabolic
reflector

to induction coil

transmitting
antenna

receiving
antenna

Figure 9.1.5 Hertzs transmitting and receiving antennas used


parabolic reflectors to direct the electromagnetic wave
without much loss of energy with distance.

Because these antennas were smaller, they were


placed in parabolic reflectors (Figure 9.1.5) to direct
and detect the beam without much loss. This
configuration produced the most accurate results.

from ideas to
Implementation
distance gave the wavelength . Hertz was able to calculate
the frequency f of the transmitted radio waves. The speed of
these waves c was then determined using the following
well-known wave speed formula:
c = f
Hertz used a different frequency and found that the
speed of the waves remained the same. Although this didnt
prove that the speed of these radio waves was the speed of
light, it was strong supporting evidence of Maxwells theory
1m
2m
3m
4m
5m
6m
7m
8m
Distance from the wall
that the speed of all electromagnetic radiation was the same.
Moreover, the speed of these waves was exactly the measured
Figure 9.1.6 Standing electromagnetic waves produced by reflection
speed of light.
from a large zinc plate enabled Hertz to measure the
Hertz also showed that the path of these waves
speed of light.
could be bent in the same way as light (refraction) by passing
them through a large asphalt prism. (See in2 Physics @
Preliminary section 8.3.)
Hertz showed that the electric and magnetic fields
Heinrich Hertz: Oops
of the radio waves had a unique direction in space, known as
polarisation. The electric field in Figure 9.1.6 points vertically.
ertz initially published his estimate of
the speed of electromagnetic waves as
When the detecting loop was at an anti-node and the direction
2 108 ms1. This relied on the accuracy of his
of the gap aligned with the electric field (vertically), then a
calculation of the oscillation frequency of the
spark jumped across the gap. When the loop was rotated so
transmitter. It was an embarrassing moment when
that the gap was at right angles to the electric field, there was
Poincar, a great French mathematician, wrote him
no spark.
a letter pointing out that he had not included a
It took great skill for Hertz to show that polarised
factor of the square root of 2 in his frequency
electromagnetic waves exist and have a finite speed equivalent
calculation. As a result, his actual measured speed
to the speed of light. Moreover, these waves shared other
was 2.8 108 ms1, which is only 7% off the
properties with light such as reflection and refraction. This set
actual value of 3108ms1. So dont feel so bad
the scene for the emergence of radio communications and
next time you make a numerical mistakeeven
hence the modern field of telecommunicationsmobile phone
great scientists do it.
technology is a sophisticated version of Hertzs experiment.
Hertz did all of this before his life was cut short at the age of
36. One of the greatest honours that can be bestowed upon a scientist
is to name a unit of measurement after them. The international unit of
frequency is no longer called cycles per secondit is known as the hertz.

Checkpoint 9.1
1
2
3
4
5
6
7
8
9

Describe Maxwells contribution to understanding the connection between electricity and magnetism.
Describe an electromagnetic wave.
Recall the two predictions that Maxwell made about electromagnetic waves.
Describe how Hertz produced and controlled the frequency of electromagnetic waves.
Outline the requirements needed of the receiver in order for it to detect any given frequency.
Recall the definition of a standing wave.
Sketch a diagram of a standing wave in a rope, labelling a node and anti-node.
Outline how Hertz manipulated the equipment in order to determine the wavelength of the electromagnetic wave
produced.
Define polarisation and explain how Hertz showed that electromagnetic waves were polarised.
177

Electromagnetic
radiation: particles
or waves?

9.2 Black body radiation and


Plancks hypothesis

cause these metals to


radiate different colours
from dull red to yellow.

PRACTICAL
EXPERIENCES
Activity 9.2

Activity Manual, Page


78

TRY THIS!
Sun power
Use a magnifying glass to
focus the Suns rays to a point
on a wad of tissue paperit
should catch fire. Now colour
another tissue paper with black
ink, say from a felt-tip pen or
by dipping it in black ink.
Let it dry. Focus the Suns rays
onto this blackened tissue.
You should find that the paper
catches fire much more
quickly. This is because black
absorbs more of the incoming
radiation and therefore heats
up more quickly.

ultraviolet visible

infra-red

10

8
Intensity I (arb.units)

Figure 9.2.1 Increasing temperatures

The success of Maxwells theory of electromagnetic waves led other physicists to


apply it to the long-standing problem of radiation from hot objects. As an object
such as a piece of metal is heated, its temperature rises and it emits colours from
a dull red to orange and then to yellow, eventually becoming a bluish-white
(if the object has not melted). This is shown in Figure 9.2.1 for metal parts at
increasing temperatures.
The brightness or intensity (power radiated per unit area) of radiation
depends on both wavelength and temperature, as shown by the plot in Figure 9.2.2.
The maximum intensity of the curve shifts to smaller wavelengths as the temperature
is increased. This simply means that as an object becomes hotter, its colour changes
The challenge for physicists was to mathematically
from red towards blue.
predict the exact form of this curve using electromagnetic wave theory.
An object that absorbs all the wavelengths of the spectrum is referred to
as a black body absorber. In a strange twist of terminology, an object that emits
all the wavelengths of the spectrum is called a black body emitter. The curves
shown in Figure 9.2.2 are known as black body radiation curves. The Sun is close
to being a black body emitter and so its temperature can be determined from this
plot of intensity versus wavelength.
An object that is painted black is not usually an ideal black body, as some
radiation is always reflected from its surface. An ideal black body is generally
pictured as a cavity with a small hole (see Figure 9.2.3). Any radiation that enters
the hole becomes trapped inside the cavity and undergoes many reflections before
eventually being absorbed. Similarly, a heated cavity with a small hole acts as a black
body emitter. A small fraction of the radiation from within the cavity exits through
the hole, so the intensity of this radiation can be measured (see Figure 9.2.2).
A number of physicists attempted to calculate the mathematical relationship
for the black body radiation curve on the basis of electromagnetic wave theory
and thermodynamics (the physics of the movement of heat). We will call this

6
max

T = 6000 K

4
5000 K

max

4000 K

2
3000 K

1.0

Wavelength (m)

2.0

Figure 9.2.2 The intensity of light from a hot object is dependent on wavelength.
178

3.0

from ideas to
Implementation

These assumptions were radical ones since classical physics said that objects
can have any energy. Max Plancks theory defied this assumption. Incredibly,
Planck himself resisted this theory wholeheartedly for many years and thought
of it as nothing more than a mathematical convenience. However, this idea of the
quantised nature of energy heralded the start of quantum physics, which, together
with relativity, became the foundations of the modern physics we use today.

Einsteins contribution to quantum theory: the photon


Planck originally restricted his concept of energy quantisation to emitters in the
walls of a black body cavity. He still believed that the electromagnetic energy
radiated as a wave. However, in 1905 Einstein called into question this view of
He proposed that
electromagnetic radiation and of light in particular.
radiant energy is made of concentrated bundles of energy that later came to be
called photons. That is, Einstein had proposed that light is made of particles and
is not a wave, as shown in Figure 9.2.5.
Einstein assumed that such an energy bundle is initially localised in a small
volume of space, and that it remains localised as it moves away from the source
with velocity c (the speed of light). Ironically, we still express the speed of photons
in terms of the wave properties of frequency f and wavelength , as follows:

Figure 9.2.3 A spherical cavity with a hole


acts as a black body absorber
of radiation.

Identify Plancks hypothesis


that radiation emitted and
absorbed by the walls of a
black body cavity is quantised.

classical theory
Radiance

the classical theory approach. They assumed that the walls of the cavity were
made from tiny oscillators that emit electromagnetic wavesjust as Hertz had
Although this theory was able to
assumed for his transmitting antenna.
reproduce the shape of the graph for large wavelengths (see Figure 9.2.4), it failed
badly at the shorter wavelengths. Here the calculated intensity increased towards
infinity, which violated the law of conservation of energy. This was called the
ultraviolet catastrophe, because this started to occur at the ultraviolet end of the
radiation spectrum.
Max Planck (18581947) solved this problem in 1900 and was able to
mathematically reproduce the black body radiation curve by making the following
radical assumptions:
The emitters in the walls of the cavity can only have energies E given by
E = nhf
where f is the emitted frequency in hertz, h is a constant (now called the
Planck constant = 6.63 1034Js) and n is an integer. The energy E is
measured in joules.
The emitters can absorb or radiate energy in jumps, or quanta. Two
consecutive energy states of an emitter differ by hf.

experiment and
Planck theory

2
3
Wavelength (m)

Figure 9.2.4 The classical theory of black


body radiation could only
explain the long wavelengths,
Plancks theory could explain
all wavelengths.

c = f
a

y
E

B
z

photon with
energy hf

c
x
z

x
c

Figure 9.2.5 Light can be shown as (a) an electromagnetic wave


and(b) a collection of particles called photons.

Identify Einsteins contribution


to quantum theory and its
relation to black body radiation.
Explain the particle model of
light in terms of photons with
particular energy and frequency.
Identify the relationships
between photon energy,
frequency, speed of light and
wavelength:
E = hf
and
c = f

179

Electromagnetic
radiation: particles
or waves?

PHYSICS FEATURE
Max avoids a catastrophe

etailed understanding of Max Plancks


mathematical manipulations is beyond the scope
of this book. Even Planck, at first, looked on his
solution as nothing more than a mathematical act of
desperation. But we can gain some qualitative insights
into his theory by adopting a view that is slightly
different from that of his original work.
Imagine that the electromagnetic wave emitters in
the walls of the cavity produce standing waves, as
shown in the simplified one-dimensional
representation in Figure 9.2.7 in which the walls on
either side of the waves represent the walls of the
cavity. The intensity at a particular wavelength of the
radiation from the cavity is related to the energy in
the wave at that particular wavelength.
Both the classical theory and Plancks theory
assumed that the energy in the wave is equal to the
energy from the emitter. However, according to
classical theory, the average energy for each emitter
in the walls is kT, where k is a known constant (the
Boltzmann constant) and T is the temperature of the
walls in kelvin. Therefore, the energy per standing
wave is also kT. An understanding of the way intensity
is measured is important to understanding the
difference between the ultraviolet catastrophe and
Plancks radiation law.
An instrument known as a spectrometer is used
to measure the intensity of the black body radiation.
All spectrometers take in a small wavelength range
simultaneouslythey never measure just a single
wavelength. For example, a typically good
spectrometer cannot distinguish between wavelengths
of 6000.0 1010 m and 6000.1 1010 m. Note
from Figure 9.2.7 that the difference in wavelength
between two consecutive standing waves becomes
progressively smaller with increasing frequency. This
implies that at higher frequencies more wavelengths
can enter the spectrometer. Since each wavelength
contributes an equal amount of energy, and more of
them are entering the spectrometer with increasing
frequency, this will register as an increasing intensity
since the total energy entering the spectrometer
increases. As the wavelength becomes shorter, and
180

Figure 9.2.6
Max Plancks radical
hypothesis avoided the
ultraviolet catastrophe
and started a new field
of physics called
quantum physics.

Figure 9.2.7 A simplified representation of the standing waves


inside a metal cavity

therefore the frequency increases, the intensity will


increase towards infinity. This is the origin of the
ultraviolet catastrophe, and which, of course, does
not happen.
Planck restricted the energy of the emitters in the
cavity to integer multiples of hf, which is the same as
only allowing integer multiples of a certain frequency
f. This restricted the number of standing waves that
can occur. He also used a well-established law of
thermal physics that states that a particle is more
likely to have a lower than a higher energy. That means
there is a dramatic decrease in the number of emitters
with higher energies. As the frequency becomes very
high, and therefore the wavelength very short, these
two restrictions ensure that the number of emitters
with these higher frequencies becomes very small and
approaches zero. This registers as a diminishing
intensity in the spectrometer and, thus, reproduces
the actual black body radiation curve.

from ideas to
Implementation
Einstein assumed that the energy E of the photon is related to its frequency
f by the equation:
E = hf
where h is Plancks constant = 6.63 1034Js. In classical theory, the intensity
was determined by the electric field of the electromagnetic wave. In the new
quantum theory, it is the number of photons that determines the intensity.
He then used this idea to explain peculiar properties of metals when they are
irradiated with visible and ultraviolet light, known as the photoelectric effect.

Solve problems and analyse


information using:
E = hf
and
c = f

Worked example
question
A laser pointer emits light with a wavelength of 6.50 107 m. The power of the laser is
1.00 103 W.
Assume that Plancks constant h = 6.63 1034 Js, and the speed of light
c = 3.00 108ms1.
a Calculate the energy in each photon of the laser beam.
b Calculate the number of photons emitted each second.

Solution
a The energy E in a photon is given by
E = hf

where h is Plancks constant and f is the frequency of the light. We can calculate
the frequency from the following:
c = f

Therefore:
E =h

c
3.00 108
= 6.63 1034
= 3.06 10 17 J

6.50 109

The energy per photon is 3.06 1017 J.

b The laser has a power of 1.00 103 W. This means it emits 1.00 103 joules per
second. The number of photons per second is given by:
number of photons per second =

power
1.00 103
=
energy per photon 3.06 1017

= 3.27 1013 photons per second

Checkpoint 9.2
1
2
3
4
5
6

Describe what happens to the wavelength of light emitted as the temperature of an object is increased.
Define a black body.
Outline how classical theory described black body radiation.
Explain the ultraviolet catastrophe.
Outline the assumptions Planck used to solve the problem.
Explain Einsteins revolutionary thoughts on light.

181

Electromagnetic
radiation: particles
or waves?

9.3 The photoelectric effect


Describe Hertzs observation of
the effect of a radio wave on a
receiver and the photoelectric
effect he produced but failed
to investigate.

light
electrons

evacuated quartz tube


V

Figure 9.3.1 Electrons ejected from the


cathode register a current on
the ammeter, when the cathode
is struck by ultraviolet light.

gst
en
pla
tin
um

tun

zin

pot
ass
sod ium
ium

Maximum kinetic energy (eV)

f0
f

W
0.5

1.0

1.5

Frequency ( 1015 Hz)

Figure 9.3.2 The energy of photoelectrons


increases with increasing
frequency. No electrons are
ejected below the cut-off
frequency f0.
182

In his experiments on radio waves, Hertz noticed that the sparks were easier
to produce in the gap of the detector loop whenever it was directly exposed to the
ultraviolet light from the spark of the transmitting antenna. Placing an ordinary
glass plate between the transmitter and the detector reduced the brightness of the
spark, because ordinary glass blocks ultraviolet light. Replacing the glass plate with
a quartz glass plate caused the spark to be more easily produced, because quartz
glass allows ultraviolet light to be transmitted. He concluded that ultraviolet light
has an effect on the receiving loop, but he was not in a position to explain it.
However, he realised that it was an extremely important phenomenon and stopped
his wave research in order to study it. Hertz had discovered the photoelectric effect.
The photoelectric effect is the ejection of electrons from the surface of a
polished metal when light is shone on it. The metal on either side of the gap
of Hertzs detector loop emitted electrons when struck by ultraviolet light, which
assisted in producing the spark. Ironically, the results of this experiment were used
by Einstein to prove the particle nature of light, in contrast to Hertz who wanted
to prove its wave nature.
The photoelectric effect was studied by other physicists who used a cathode ray
tube, as shown in Figure 9.3.1. The tube either had a quartz window or was made
of quartz, which allowed ultraviolet light to pass through and strike the cathode,
known as a photocathode. This caused electrons, known as photoelectrons,
to leave the cathode and be collected by the anode, thus registering a current on
the ammeter.
The photoelectric effect was considered to be strange because:
No electrons are ejected from the metal below a certain frequency no matter
how intense the light. This is called the cut-off frequency f0. This is in
contrast to classical theory, which says that all electromagnetic waves have
energy and, if you wait long enough, all electrons can gain energy and leave
the surface.
The kinetic energy of the electrons increases as the frequency of the incident
light increases (that is, going from red to blue to ultraviolet and beyond) as
shown in Figure 9.3.2. However, there is no change in the electron energy if
the frequency of the incident light is constant but the intensity is increased.
Increasing the intensity simply increases the number of electrons but their
energy remains the same. From classical wave theory, an increase in intensity
should result in an increase in the energy of the electrons.
There is no delay between the time the light is shone on the surface and the
time the electrons are emitted, no matter how dim the light source. From
classical theory, electrons would require a length of time to gain enough energy
from a low-intensity light source in order to escape the surface of the metal.
Einstein explained the photoelectric effect by assuming that an electron is
ejected when it absorbs a photon that has energy E = hf. Although electrons
involved in electrical conduction in a metal are free to move around, they are still
bound to the metal as a whole. Energy is needed to separate them from the metal.
This energy is called the work function W. The value of W depends on the type
of metal. The photoelectric effect is mainly a surface phenomenon, so the surface
must be free of oxide films, grease or other surface contaminants. Einstein then
simply conserved energy by stating that:

from ideas to
Implementation

where Kmax is the maximum kinetic energy of the ejected electrons.


Rearranging this equation, we get:
Kmax = hf W

nte

ractiv

Photon energy = energy of ejected electron + energy needed by the electron to


leave the metal.
This is encapsulated in the equation:
hf = Kmax + W

M o d u le

PRACTICAL
EXPERIENCES
Activity 9.3

Current

Activity Manual, Page


81
From this we can see that the kinetic energy is zero (that is, no ejected
electron) when hf = W. This accounts for the cut-off frequency. In addition, the
kinetic energy increases as the frequency increases. Increasing the light intensity
PRACTICAL
merely increases the number of photoelectrons and therefore increases the current
EXPERIENCES
emerging from the surface, but does not change the kinetic energy of the electrons.
Activity 9.4
Activity Manual, Page
88
Einsteins theory also accounts for the absence of a time delay, because all of the
energy is supplied in a concentrated bundle; it is not spread uniformly over a large
area, as in wave theory. In 1921 Einstein won the Nobel Prize in Physics for his
explanation of the photoelectric effect.
The graph in Figure 9.3.3 shows the dependence of the photoelectron current
on the applied potential difference between photocathode and anode. There are
two interesting features of this graph. The first is that there is a current even when
there is no potential difference. At this point, all the energy to produce the current
comes from the incoming photons. The second, and more interesting, feature is
Vstop
0
Potential difference
that the current is stopped by a certain negative potential difference placed on the
photocathode. Not surprisingly, this potential is called the stopping potential
Figure 9.3.3 Current of photoelectrons versus
Vstop and is used to determine the maximum kinetic energy of the electrons. The
applied potential difference
ammeter will read zero current when eVstop equals maximum electron kinetic energy.
between cathode and anode

Worked example
question
A zinc photocathode is irradiated with ultraviolet light. A voltage of 2.0 V is required to stop
the photoelectrons.
a Calculate the maximum kinetic energy of the photoelectrons. Use the electron charge
e = 1.6 1019 C.
b Calculate the wavelength of the ultraviolet light striking the photocathode, given the
work function of zinc W = 6.88 1019 J and Plancks constant h = 6.63 1034 J s.

Solution
a The maximum photocathode kinetic energy Kmax is given by:
Kmax = eVstop = 1.6 1019 2.0 = 3.2 1019 J
b Use Einsteins photoelectric effect formula: Kmax = hf W

where Kmax is obtained from part a, h is Plancks constant and the zinc work function
W = 6.88 1019 J. In order to calculate the wavelength we first require the frequency,
which is obtained by manipulating Einsteins formula:
f=

K max +W 3.2 1019 + 6.88 1019


=
= 1.52 1015 Hz
34
h
6.63 10

We can now use the relationship between frequency and wavelength:


c 3.00 108
= =
= 1.97 107 m
f 1.52 1015
183

Electromagnetic
radiation: particles
or waves?

Checkpoint 9.3
1
2
3
4

Define the photoelectric effect.


Outline Hertzs observations regarding the photoelectric effect.
Describe the significance of the cut-off frequency.
Complete the table below by listing the observation of the photoelectric effect that classical theory predicted
incorrectly.
Observation

Classical prediction

Electrons are not emitted from the surface of a


metal no matter how intense the light if the
frequency of the light is below the cut-off frequency.

5
6

Explain how the work function helps explain the measured energy of photoelectron.
Outline the significance of the stopping voltage.

photomultiplier outputs an electrical


impulse to electronic circuits

anode

dynode

incoming
light photon

photo cathode

electron

Figure 9.4.1 The photomultiplier tube uses


the photoelectric effect and can
multiply the number of electrons
produced by millions of times.
Identify data sources, gather,
process and present information
to summarise the use of the
photoelectric effect in photocells.

184

9.4 Applications of
the photoelectric effect
The photoelectric effect presents a convenient way of producing an electrical
signal from light. The apparatus used to produce the traditional photoelectric
effect (a vacuum tube with a photocathode and anode), called a photocell
(photoelectric cell), was not of practical use outside the classroom for
demonstrating the photoelectric effect. The current that was produced was far
too small for most applications.
A modification to this tube was made by placing a series of specially coated
surfaces, called dynodes, between the photocathode and the anode (see Figure
9.4.1). This is known as a photomultiplier tube. The dynodes are at successively
increasing positive voltage with respect to the cathode, so they attract the
electrons emitted by the photocathode. Each incident electron causes many other
electrons to be emitted from the dynode surface. This effect is multiplied at the
next dynode, and so on until a large number of electrons reach the anode and
register a relatively large current. This makes the photomultiplier tube extremely
sensitive to low levels of light, such that it is possible to detect individual
photons. The material from which the photocathode is made has been improved
and it can eject electrons from incident light ranging from the ultraviolet to the
infra-red range of the electromagnetic spectrum.
This high sensitivity of the photomultiplier tube makes it suitable for the
detection of small changes in light intensity. This has opened up many
applications in astronomy, nuclear physics, blood testing and medical imaging.
One method of medical imaging is called positron emission tomography
(PET). One use of this method is the imaging of tumour cells within the body.
The patient is injected with a short-lived radioactive chemical that contains
molecules that attach themselves to particular tissue types such as tumour cells.
This chemical emits two very high energy photons (known as gamma photons)
that exit the body in opposite directions and strike a ring of photomultipliers
around the patient (see Figure 9.4.2). Each photomultiplier has a material in

from ideas to
Implementation
front of it, known as a scintillator, that gives out flashes of light when the gamma
photons strike it. The flashes of light are converted to electrical signals by the
photomultiplier tubes. Computers calculate the time of arrival of the individual
gamma photons and are able to locate their point of origin in the body. In this
way, an image of the tumour cells can be generated from the different pairs of
gamma photons arriving from different locations.
Another use of the photoelectric effect is in night-vision devices used by
the military to see in almost total darkness by using the few photons coming
from stars.
If we broaden our definition of the photoelectric effect to mean any process
by which light is converted to electricity, without the need to eject an electron
from a surface, then we can include many devices made from semiconductor
materials. Although the electron stays within the solid, it becomes free from the
atom to which it was bound, when struck by a photon. However, one important
use of this type of photoelectric effect is the conversion of sunlight into electrical
energy in solar cells. More detail will be given on this topic in Chapter 10
Semiconductors and the electronic revolution.

detectors

positron-electron
collision

gamma rays
created

Checkpoint 9.4
1
2
3

Explain why the classic photocell is not used in commercial


applications.
Outline how a photomultiplier tube works.
Give examples of applications in which the photoelectric effect
is used.

Figure 9.4.2 (a) A positron emission


tomography scanner uses
photomultiplier tubes to obtain
(b) an image of tumour cells.

PRACTICAL EXPERIENCES

from ideas to
Implementation

CHAPTER 9

This is a starting point to get you thinking about the mandatory practical
experiences outlined in the syllabus. For detailed instructions and advice, use
in2 Physics @ HSC Activity Manual.

Activity 9.1: Receiving radio waves


Produce sparks with the induction coil. Tune the portable radio to a position on the
AM band at which there is no radio station. Move around the classroom and listen
to the clicking sounds on the radio that are in step with the sparking from the
induction coil.
Equipment: induction coil, power supply, radio.

Perform an investigation to
demonstrate the production
and reception of radio waves.

Discussion questions
1 Explain how your experiment differs from the one carried out by Hertz.
2 Describe what you notice as you move further away from the coil.
185

Electromagnetic
radiation: particles
or waves?

PRACTICAL EXPERIENCES
Activity 9.2: Black body radiation

Identify data sources, gather,


process and analyse information
and use available evidence to
assess Einsteins contribution to
quantum theory and its relation
to black body radiation.

Gather information to assess Einsteins contribution to quantum theory and black


body radiation. Present your information in the form of a newspaper article relating
what you have found to people without scientific backgrounds.
Discussion questions
1 Define the UV catastrophe.
2 Outline how Planck explained the difference between observations and theory.
3 State Einsteins contribution to black body radiation.

Activity 9.3: Photocells


Identify data sources, gather,
process and present
information to summarise the
use of the photoelectric effect
in photocells.
Solve problems and analyse
information using:
E = hf
and
c = f

Part A: Gather information about how the photoelectric effect is used in photocells.
Part B: Set up an electrical circuit to investigate the energy that a photon of light
gives to an electron. You can carry out the experiment first hand or use a computer
simulation.
Discussion questions
1 List devices that can be classified as photocells and explain how their
working principle is related to the photoelectric effect.
2 Use the graph of electron energy (y-axis) and photon frequency (x-axis)
to calculate the Planck constant.

Activity 9.4: Einstein versus Planck


Process information to discuss
Einstein and Plancks differing
views about whether science
research is removed from social
and political forces.

Gather information about the differing views held by Planck and Einstein and use
it to write an informed paragraph on your views.
Discussion questions
1 Outline Einsteins and Plancks view of scientific research.
2 List political pressures on Planck and Einstein during the times of their
main discoveries, and assess the influence of these pressures on Plancks
and Einsteins research.

Chapter summary



186

Maxwells theory predicted that all electromagnetic


radiation travels at the same speed.
Maxwell strongly suggested that light is an
electromagnetic wave.
Hertz performed a number of experiments to
demonstrate the propagation of electromagnetic waves.
Hertz also measured the speed of electromagnetic waves
by causing them to become standing waves, then
measuring the wavelength, calculating the frequency
and using c = f.
A black body is an object that is capable of absorbing or
emitting all wavelengths of the electromagnetic spectrum.

Classical physics could not reproduce the measured


curve of intensity versus wavelength of radiation emitted
from a black body.
The ultraviolet catastrophe is a prediction by the
classical electromagnetic wave theory that the radiation
intensity from a black body becomes infinite at very
short wavelengths, which is false.
Max Planck was able to predict the black body radiation
curve by assuming that the energy of the oscillators in
the black body walls is quantised in integer steps of hf,
where h is Plancks constant and f is the frequency.
Planck thought that the quantisation of energy was only
a mathematical trick and not related to reality.

from ideas to
Implementation

Einstein realised that this quantisation was real and that


light is made of particles, each with a bundle of energy
given by hf.
The particle of light is called a photon.
The photoelectric effect is the emission of electrons
from the surface of a metal that has been struck by light.

Einstein explained the photoelectric effect in terms of


the photon model of light.
The photoelectric effect has applications in all forms of
light detection, medical applications and the conversion
of light into electrical energy.

Review questions
Physically speaking

15 Discuss the significance of the work function in

Create a visual summary of the concepts in this chapter by


constructing a mind map incorporating the following terms
and equation:

16 What determines the maximum kinetic energy of

hertz, radio waves, quanta, photon, Maxwell, photoelectric


effect, black body, UV catastrophe, photomultiplier,
work function, Einstein, kinetic energy, threshold,
frequency, polarisation, E = hf

Reviewing
1 State the significance of Maxwells equations.
2 Outline the purpose of Hertzs experiments.
3 Outline the procedure followed by Hertz to determine
the speed of radio waves.

4 Describe how a standing wave is set up.


5 Explain how Hertz used standing waves to determine
the wavelengths of radio waves.

6 Outline how Hertzs results support Maxwells


predictions.

7 Describe the results that Hertz obtained to support


the idea of polarisation.

8 Recall the relationship between wavelength of


radiation and its intensity.

9 Outline how classical theory predicted the ultraviolet


catastrophe.

10 Explain the idea of quanta.


11 Explain how Einstein expanded on Plancks idea of

understanding the behaviour of photoelectrons.


a photoelectron?

17 What is a photomultiplier tube and how does it work?


18 Describe how the photoelectric effect can be used in
PET scans to detect tumours.

Solving problems
19 Calculate the energy of a photon of red light with
a wavelength of 656 nm.

20 Figure 9.5.1 is a graph of voltage versus frequency


of light.
a Convert the voltage scale to energy of
photoelectrons.
b Determine the cut-off frequency.
c Use information from the graph to determine the
work function of the material.
d Explain what will happen to the graph if the
intensity of light is increased.
e Explain changes in the graph if the cathode was
made of a different material.
f Calculate the Planck constant from the graph.
V0 (V)
3
2
1

quantisation of oscillators in a black body cavity.

13 Justify the use of quartz glass for the photoelectric


effect experiment.
the kinetic energy of the emerging photoelectrons.

0.75

1.0

f (1015 Hz)

Figure 9.5.1 Frequency versus voltage of light

iew

Q uesti o

14 Relate the frequency of light on a photocathode to

0.25 0.50

of the photoelectric effect.

Re

12 Discuss Einsteins contribution to the understanding

187

10

Semiconductors
and the electronic
revolution
Electronics transforms the world

valence electron, valence level,


conduction level, valence, conduction
bands, energy gap, band gap, forbidden
energy gap, solid state physics,
electron volt (eV), hole, doping, n-type
semiconductor, donor, p-type
semiconductor, acceptor, donor energy
level, acceptor energy level, extrinsic,
intrinsic, microprocessor, diode,
diffusion, depletion region, forward
bias, reverse bias, photovoltaic cells,
thermionic devices, transistors, plate,
triode, electronics, bipolar transistor,
emitter, collector, base, field-effect
transistors (FET), source, drain, gate,
MOSFET, integrated circuits

Figure 10.0.1 An old valve computer


weighed several tonnes
but had much less
computing power than
your mobile phone.
188

Electronics has transformed almost every aspects of modern life,


from television, computers and mobile phones to medical
diagnostics and treatment. The discovery of cathode rays led to
devices such as radios, televisions and computers. The earliest
computers, which used cathode ray tube devices, weighed
several tonnes and filled very large rooms. Today, a basic mobile
phone has much more computing power than the most powerful
early computer. This reduction in size (and power consumption)
came with the discovery of semiconductors. The working parts of
semiconductor devices are so small that a microscope is needed
to see their structure. An example of semiconductor circuitry is
the microprocessor, the brains of your computer. It is no larger
than a coin but contains millions of circuits.

from ideas to
Implementation

Figure 10.1.1 An abstract energy level


diagram helps visualise the
ionisation energy of an atom,
which would produce free
charges for an electrical current.

Energy

Energy

Electrical conduction occurs as a result of the flow of charge. Gases, which


are normally electrical insulators, are made to conduct large currents during
lightning strikes, due to the movement of ions and electrons created by the
ionisation of air atoms. Ionisation is the removal of the outer electron of an
atom, which is known as a valence electron. The energy required to remove this
electron is usually supplied by the electric field during lightning. In general, we
are not interested in the exact motion of the charges in a current, we just want to
know whether there will be electrical conduction. A simple and abstract way of
visualising whether there will be conduction in gas atoms is shown in Figure
10.1.1. The vertical axis is energy and the horizontal lines indicate the position
an electron can have. The lower line (or level), known as the valence level, is the
energy it has while being bound to the atom. The upper level, known as the
conduction level, is the energy it must acquire to become free and contribute to
electrical conduction.
Placing two individual atoms together will result in two pairs of these levels
Note that the levels are not at exactly the same energy
(see Figure 10.1.2a).
but become shifted slightly vertically with respect to each other. This odd
behaviour can be explained by the laws of quantum physics, which are beyond
the scope of this section. Basically, energy levels of electrons in close proximity to
each other are forbidden from being the same, so placing five atoms close
together results in five pairs of these lines, as shown in Figure 10.1.2b.
To form a solid we must place many atoms, say 1023, close together, and so
It is impossible to
we have 1023 pairs of energy levels (see Figure 10.1.3).
draw these individual levels because they are very close to each other and too
numerous. We simply represent them by two shaded areas called energy bands.
The lower and upper bands are labelled as valence and conduction bands
respectively. The energy gap between them is referred to as the band gap or
forbidden energy gap.
Electrical conduction in a solid occurs because some electrons have gained
enough energy to be in the conduction band and therefore become free
They are no longer localised to a particular ion but are shared by
electrons.
all ions in the lattice, since they are free to move throughout the solid. Unlike
conduction in a gas, the ions in solids are not free to move, so the only charge
carriers are the electrons.
Quantum physics determines that no electron can have an energy in the
energy gap. Moreover, only a fixed number of electrons are allowed per energy
level in the energy bands. When an energy level is occupied by the maximum
number of electrons allowed by quantum physics, we say that the level is filled.
Any further electrons will start to occupy the level above it until this new level is
filled, and so on until a whole energy band is filled.
Ordinary experience tells you that not all solids conduct electricity. All metals
are good conductors; however, materials such as glass and plastic are insulators.
There is also a third type of solid known as a semiconductor, such as silicon and
germanium, which conduct electricity but much less than metals. The energy
band diagram, shown in Figure 10.1.4, can be used to illustrate the difference in
the conductivities of these three types of materials.

Energy

10.1 Conduction and energy bands

Figure 10.1.2 Energy level diagrams for (a)


two atoms and (b) five atoms
in close proximity

conduction band
energy gap

valence band

Figure 10.1.3 The energy band structure


of solids

Identify that some electrons in


solids are shared between
atoms and move freely.
Describe the difference
between conductors, insulators
and semiconductors in terms of
band structures and relative
electrical resistance.

189

10

Semiconductors
and the electronic
revolution
a

c
conduction band

overlap

energy gap

valence band

Figure 10.1.4 Energy band diagrams for (a) a conductor, (b) a semiconductor and (c) an insulator

Compare qualitatively the


relative number of free
electrons that can drift from
atom to atom in conductors,
semiconductors and insulators.

The type of atom, the bonding between atoms and the number of valence
and inner electrons of an atom are some factors that determine the type of
energy band diagram for the resulting solid. This is the area of physics known as
solid state physics, which relies heavily on quantum physics. Figure 10.1.4a
shows an energy band diagram for conductors. There is no clear energy gap and
therefore no distinct conduction or valence band. We can say that the
conduction band is partially filled and therefore the solid contains free electrons.
The energy band diagrams for a semiconductor and an insulator are similar
The much smaller energy gap of the semiconductor
(Figure 10.1.4b, c).
distinguishes it from an insulator. The valence band of an insulator is full and,
due to the relatively large size of its energy gap, none of the electrons in the
valence band have enough energy to reach the conduction band. However, some
electrons in the valence band of the semiconductor can acquire enough thermal
The
energy to jump across the small gap into the conduction band.
number of conduction electrons that result is much less than for a conductor, so
a semiconductor is a much poorer conductor than a metal but a better conductor
than an insulator such as plastic or glass.

Checkpoint 10.1
1
2
3
4
5

Describe how gases, which usually have insulating properties, can be made to conduct electricity.
Describe why the energy levels of two neighbouring atoms are not exactly the same.
State why only electrons are charge carriers in solids.
Define the terms electrical conductor and insulator.
Describe how it is possible for electrons in the valence band of semiconductors to reach the conduction band.

10.2 Semiconductors
The size of the energy gap determines whether a solid is a semiconductor
or an insulator. A convenient unit for measuring the energy gap is the electron
volt (eV). This is a unit of energy and is related to the joule:
1 eV = 1.6 1019 J
Most semiconductors have an energy gap less than 5 eV. Insulators have
greater energy gaps. Examples of energy gaps for semiconductors and insulators
are given in Table 10.2.1. The most common semiconductors are silicon (Si) and
germanium (Ge).
190

from ideas to
Implementation
Some electrons in the valence band of a semiconductor will gain
enough energy from the ambient thermal (heat) energy in the solid to
jump up to the conduction band. This occurs readily at normal room
temperature. Heating a semiconductor further increases the number of
electrons jumping from the valence band to the conduction band, thereby
making the solid more conducting. For example, heating a piece of silicon
will lower its resistance. At absolute zero (T = 0K), there is no thermal
energy available for the electrons, all of which remain in the valence band;
therefore, at absolute zero the semiconductor behaves as an insulator.

Table 10.2.1 Energy gaps in electron volts of common


semiconductors at room temperature

Semiconductor crystal

TRY THIS!

Energy gap (eV)

Si
Ge
InP
GaP
GaAs
CdS
CdTe
ZnO
ZnS
Diamond

1.14
0.67
1.35
2.26
1.43
2.42
1.45
3.2
3.6
5.4

Semiconducting pencil
Graphite is a form of carbon that is used in lead pencils (there is
no lead in pencils, just graphite and clay, but we will refer to it as
the lead of the pencil). Graphite also behaves as a semiconductor.
Obtain a length of pencil lead (about 10 cm) and attach each end
to the probes of a digital ohm meter. It should measure units of
ohms. Now heat it (with, say, a match) and observe that the
resistance decreases. The resistance may decrease by up to 0.5
and it will be more for thinner leads.
This decrease in resistance is due to an increase in the number
of electrons jumping from the valence band to the conduction band.
The resistance will increase once more when the lead cools.

Electrons and holes


When an electron is excited into the conduction band, it leaves a vacancy in the
The hole
valence band. This vacancy is known as a hole (see Figure 10.2.1).
behaves like a positively charged particle and moves in the opposite direction to
the electron. In reality, the other electrons in the valence band move to fill the
vacancy, but in doing so they leave behind another vacancy, resulting in the
apparent motion of the hole. It is much simpler to treat the hole as a positively
charged particle than to track the motion of all the electrons that move in the
opposite direction. The magnitude of the charge on an electron and a hole are
Therefore, the
the same at 1.6 1019 C, but they are opposite in sign.
motion of both electrons and holes contribute to the motion of electrical
current.

Identify absences of electrons


in a nearly full band as holes,
and recognise that both
electrons and holes help to
carry current.

conduction
band

narrow
energy gap
v

Doping
At normal room temperature (300 K) the average kinetic energy of an electron is
about 0.026eV. However, the energy gap of silicon is 1.14eV. It would appear
that the electrons dont have enough energy to jump to the conduction band, so
why is silicon, or any of the other semiconductors listed in Table 10.2.1, slightly
conducting at room temperature? The answer is that 0.026 eV is an average
energy. This means there are some electrons that have a much lower energy and

valence
band

applied electric field


electrons

holes

Figure 10.2.1 Holes are created in the valence


band of a semiconductor when
electrons are excited up to the
conduction band.
191

10

Semiconductors
and the electronic
revolution

A hole in
a bottle

way of picturing the concept


of a hole in a semiconductor
is to consider a bottle that is
almost full of water with a small
air bubble underneath its closed
cap. Turning the bottle upside
down will cause the bubble to
move upwards. The question is:
did the bubble move up or did
the water move down? The
answer is that both are true, but
it is much simpler to follow the
motion of the bubble than
the more complex motion of the
water. In the same way, we
describe the motion of the hole
in a semiconductor rather than
the more complex motion of all
the valence electrons.

a
+4

+4

+4

+4

+4

+4

+4

+4

+4

+4

+4

+4

+4

+5

+4

+4

+4

+4

+4

+4

+4

+4

+3

+4

+4

+4

+4

electron

192

hole

some that have a much higher energy. It is these higher energy electrons that
make it across the gap, but there are not many of them.
Physicists have found that the conductivity of semiconductors can be
improved by introducing impurities into the crystal lattice, a process that is
called doping. In order to understand how an impurity atom can improve the
conductivity, we must examine what happens at the crystal lattice level.
We will use silicon as an example, but similar explanations can be applied to
all other semiconductors. Silicon belongs to group 4 of the periodic table, which
means that the silicon atom has 4 electrons available for bonding with other
silicon atoms (see Figure 10.2.2a). Silicon can be doped with phosphorus P by
replacing a silicon atom with a phosphorus atom. Phosphorus is in group 5 of the
periodic table and has 5 electrons available for bonding, but it can only bond
with 4 atoms in the silicon crystal lattice. This leaves one electron unbonded (see
This semiconductor is called an n-type semiconductor.
Figure 10.2.2b).
(n represents the negative charge of the unbonded electron.) Impurities that
produce unbonded electrons are called donor impurities. These unbonded
electrons are weakly attached to their atoms and can easily be moved into
the conduction band by ambient heat, and thus can contribute to an electrical
current.
Similarly, silicon can be doped with boron, which belongs to group 3 and
therefore has 3 electrons available for bonding with 3 atoms of silicon. There is
no electron available for the fourth silicon atom to bond with and, as a result,
a vacancy in charge has been created, which we call a hole. An electron from a
neighbouring silicon atom can now jump across to fill this vacancy (see Figure
10.2.2c). This, in turn, creates a vacancy for the atom it left behind, and so can
be viewed as the movement of the hole. As already mentioned, a hole behaves
like a positive charge since it moves in the opposite direction to the electron. The
semiconductor behaves as though it has positive charge carriers and is therefore
called a p-type semiconductor. Impurities that produce a charge vacancy are
called acceptor impurities.
Note that both n- and p-type semiconductors remain electrically
neutral even though there is an unbonded electron or a vacancy is created. The
neutral impurity atoms were added to an already neutral solid, so the net charge
remains zero.
Both n- and p-type doping improve the conductivity of a semiconductor by
creating an excess of negative and positive charge carriers respectively. The
unbonded electron in n-type semiconductors needs a small amount of energy to
move to a different location. Similarly, very little energy is required to cause a
neighbouring electron to occupy the vacancy of a p-type semiconductor.
The energy-band diagram of an n-type semiconductor is shown in Figure
10.2.3. A new energy level has been created in the energy gap just below the
conduction band. This level is occupied by the unbonded electrons of the
impurities and is called the donor energy level. This level is very close to the
conduction band. The difference in energy between the two levels is usually
0.026 eV or less, and so most of the electrons in the donor level are able to jump
up to the conduction band at room temperature, making the semiconductor
much more conducting than the undoped crystal.
Figure 10.2.2 The silicon lattice (a) has no doping, (b) has been doped with phosphorus
and (c) has been doped with boron.

from ideas to
Implementation
The energy-band diagram for a p-type semiconductor in Figure 10.2.4 shows
that there is an extra energy level, called an acceptor energy level, near the
valence band. This level is full of vacancies (holes), allowing the valence electrons
to jump up and occupy these vacancies and therefore contribute to an electrical
current. The gap between the acceptor level and the donor (valence) level must
be equal to or less than 0.026 eV for conduction to take place at room
temperature.
A semiconductor is labelled as extrinsic if conduction is dominated by donor
or acceptor impurities. Otherwise, it is known as an intrinsic semiconductor.

conduction band

Identify differences in p- and


n-type semiconductors in terms
of the relative number of
negative charge carriers and
positive holes.
Describe how doping a
semiconductor can change its
electrical properties.

conduction band

donor impurity energy level

holes
acceptor impurity
energy level

valence band

valence band

Figure 10.2.3 Energy-band diagram of an

Figure 10.2.4 Energy-band diagram of

n-type semiconductor

a p-type semiconductor

PRACTICAL
EXPERIENCES
Activity 10.1

Activity Manual, Page


91

Checkpoint 10.2
1
2
3
4
5
6

Distinguish between a semiconductor and an insulator in terms of the size of the energy gap.
Describe what is meant by a hole.
Describe doping in semiconductors.
Explain the difference between n-type and p-type semiconductors.
Identify elements that can be used to make n-type and p-type semiconductors.
Distinguish between acceptor and donor energy levels.

10.3 Semiconductor devices


Combinations of p- and n-type semiconductors can be used to make all modern
electronic devices, ranging from the simple remote control to the sophisticated
computer microprocessor. The basis of this technology is a result of the
properties of the junction between p- and n-type materials.

The pn junction
A pn junction is a p-type material joined onto an n-type material. In reality,
the pn junction is made from a continuous single crystal in which the
concentration of impurities has been made to change abruptly from p-type to
n-type as we cross the junction. However, to clarify the physics of the junction
region, we will assume that the p-type and n-type crystals are initially separated
and then brought together.
The pn junction is used as the most basic electronic device, the diode.
It allows current to move in only one direction, as shown in Figure 10.3.1.

193

10

Semiconductors
and the electronic
revolution
V

forward
bias

A
+

b
p

forward
bias
O

reverse bias

Figure 10.3.1 (a) A forward-biased diode and


(b) its symbol. (c) The graph
shows the current versus voltage
characteristics of a diode.

electric
field

depletion
layer

p-type

n-type
E
E
free
electrons

free
holes
fixed
negative
ions

fixed
positive
ions

Figure 10.3.2 The electric field in the


depletion region of the pn
region stops further diffusion
of charge.

194

The conventional current easily flows from p- to the n-type material when the
positive terminal of a power supply is connected to the p-type end of the device.
Reversing the connections of the power supply (positive connected to n-type and
negative to p-type) results in a very tiny reverse current, the current is essentially
stopped. The symbol for the diode is an arrow that indicates the direction of the
conventional current when the positive and negative terminals of the power supply
are connected to the p- and n-type materials respectively (see Figure 10.3.1b).
A key to understanding the physics of a pn junction is to first examine
the concept of diffusion. Free particles are in continuous random motion and
move from areas of high to areas of low concentration. For example, a drop of ink
placed in a beaker of water results in the ink particles slowly spreading outwards so
that the water becomes uniform in colour. This occurs because the point of origin
of the free particles is very small (drop of ink) compared with the rest of the
medium (beaker of water). So it is more likely that the particles will move to the
rest of the medium. This is diffusion and it is a property of all randomly moving
particles. The energy for the movement of these particles (kinetic energy) comes
from the ambient thermal energy. At a temperature of absolute zero there will not
be any diffusion.
Similarly, the holes in the p-type material and the electrons in the n-type
material are in constant random motion due to their thermal energies, resulting in
a uniform density of the charge carriers across their respective materials. When the
two crystals are brought into contact, electrons from the n-type crystal will
naturally want to diffuse into the p-type crystal, because the electron density of its
conduction band is lower. Conversely, the holes in the p-type region want to
This results in an
diffuse into the n-type crystal, as shown in Figure 10.3.2.
electrical current, which is quickly stopped because an electric field builds up from
the n to the p region, due to this charge separation, and stops the further flow of
charge. A large part of this diffusion region results in the combination of electrons
with holes, leaving it depleted of charge. This is known as the depletion region.
However, there continues to be an excess of electrons and holes on the p and n
sides respectively.
A depletion region is like two parallel plates with equal and opposite
charges on them, resulting in an electric field in the space between them, which
contains no charge.
The energy that created the electric field essentially came from the
thermal energy of the charge carriers. At a temperature of absolute zero (0 K),
there will not be enough energy for diffusion to occur and therefore no electric
field and no depletion region will be created.
Electrons that diffused across and combined with holes cannot easily
drift back since they lost energy as they fell into the holes. Similarly, holes that
diffused combine with electrons from the n-type material. The trapping of the
diffused holes and electrons has resulted in the conversion of thermal energy into
the electrostatic energy stored in the electric field region.
As shown in Figure 10.3.3a, connecting the positive and negative
terminals of a battery to the p and n sides of the junction respectively creates an
electric field opposing that in the depletion region. This lowers the junction
electric field strength and therefore allows further charge to diffuse across the
junction. A thin pn junction will enable the excess diffused charge to reach the

from ideas to
Implementation
metal terminals on either side of the junction, thus resulting in the flow of
current in the outside circuit. This type of connection to the pn junction is
known as forward bias. This also has the effect of reducing the width of the
depletion region.
Conversely, connecting the negative and positive terminals of a battery
to the p and n sides respectively, as shown in Figure 10.3.3b, increases the
electric field strength in the junction region. This will cause some of the diffused
electrons to travel back to the n region, setting up a small reverse current. This
type of connection is known as reverse bias. It also has the effect of increasing
the width of the depletion region.

Rectifiers

The pn junction or diode is used to convert AC to DC electricity in chargers or


power supplies for electronic devices such as mobile phones and computers. A
simple circuit, but not one that is used often, is shown in Figure 10.3.4. A
sinusoidal alternating voltage, say from the AC mains supply, is applied to the p
end of the diode. The output from the n end is the positive part of this signal.
The negative part of the AC voltage is blocked. A component consisting of
parallel plates (called a capacitor) at the output stores the positive charge and
helps smooth the output signal so that a DC voltage will result.
V

input

output
C

hole
flow
p-type

electron
flow

n-type

E
electron flow +

temporary
hole flow

electron flow

temporary
wide
depletion layer electron flow

p-type

n-type

E
+

Figure 10.3.3 (a) Forward-biased and


(b) reverse biased pn junctions

voltage smoothed

narrow
depletion layer

Figure 10.3.4 A rectifier circuit can be used as a power supply.

Light-emitting diodes (LED) and laser diodes


A current that passes through a forward-biased diode will lead to some
recombination of electron and hole pairs. This recombination occurs when
a conduction electron drops from the conduction to the valence band to occupy
a vacancy (the hole), as shown in Figure 10.3.5. This change in energy can
appear as heat given to the solid, or as the emission of light. The energy of the
photon hf is equal to the energy lost by the electron, which is the energy gap Eg
such that:
hc
E g = hf =

where Plancks constant h = 6.63 1034 Js, f is the frequency, c is the speed
of light and is the wavelength. Manufacturers of LEDs can control the
wavelength of the emitted light by controlling the size of the energy gap. Most
remote-control units around the home have an LED on the end that you point
towards a device. The LED looks like a very tiny plastic light blub, but works
very differently. The wavelength of these LEDs is usually in the infra-red range
of the spectrum and so is not visible to the eye. However, it is visible to most
types of digital cameras such as those in mobile phones and video cameras.

PRACTICAL
EXPERIENCES
Activity 10.2

Activity Manual, Page


94

light emission
conduction
band
E = h
valence
band

hole flow

hole and
electron
recombine

p-type

electron flow
n-type

electron
flow

E = h
electron
flow

Figure 10.3.5 A forward-biased light-emitting


diode (LED)

195

10

Semiconductors
and the electronic
revolution

PRACTICAL
EXPERIENCES
Activity 10.3

PHYSICS FEATURE

Activity Manual, Page


96

Photovoltaic cellssolar cells

region is made thin enough so that most of the


olar cells use the pn junction to convert light
incoming light can be transmitted to the p-type region,
directly into electricity. They have the more formal
name of photovoltaic cells (or PVs). Very little greenhouse where photons are absorbed to create the conduction
electrons. These are swept up into the nregion by the
gas emissions are associated with PVs, which makes
electric field at the pn junction and are collected by
them a strong candidate to replace coal-fired power
the front contact metal grid, which can be connected to
stations for future power generation. Currently they have
an external circuit. The size of the voltage created by
many applications that include the supply of electricity
solar cells depends on the potential difference across
to remote locations or homes, and satellites.
the depletion region.
In a PV, a valence band electron absorbs a photon
The engineering of solar cells has advanced greatly
and is excited up to the conduction band, thus
in recent years so that greater efficiency can be
contributing to an electrical current. In practice, PVs
obtained at lower cost. For example, an antireflective
are simply pn junctions. The photons absorbed in the
coating is placed on top so that very few photons are
p region produce conduction electrons near the
lost due to simple reflection. The size of the energy gap
depletion region. These electrons are swept to the n
determines the wavelength range that will be absorbed
side of the junction by the electric field (see Figure
to produce electronssome parts of the sunlight
10.3.6). If the depletion layer is thin enough, then
spectrum deliver more energy than others. The
most of these electrons can reach the n region without
maximum efficiency of 23% has been achieved in
recombining with a hole. The electrons in the n region
recent years but not on a commercial scale.
have a much higher lifetime before recombining with
a hole. Electrodes placed on the ends of
energy
the semiconductor collect this newly
electrical
from Sun
transmission
created current and deliver it to an
system
external circuit. Electrons will travel
transparent
around a circuit and back to the p region,
adhesive
where they will recombine with holes.
antireflection
The actual structure of a solar cell is
coating
solar
cover glass
shown in Figure 10.3.7. The n-type
front
arrays
contact

n-type layer
(semiconductor)
light absorption

conduction
band

absorbed
photon

substrate

junction
p-type layer
(semiconductor)

valence
band
new hole and
electron created
p-type

hole flow

electron
flow

current

n-type

electron flow

load
resistor

electron
flow
(current)

electron
flow

Figure 10.3.6 A solar cell is a pn junction that


absorb photons to create conduction
electrons.
196

back
contact

freed electrons

holes filled by freed electrons

Figure 10.3.7 How a solar cell operates

from ideas to
Implementation

Try this!
seeing infra-red
Grab a remote control that you use for the TV set or DVD player.
Look at the LED at the end of the remote control while pressing
any button (for example, the volume or channel button). You should
not see anything happening. Now point the remote control at your
mobile phone camera or any other digital camera. While looking at
the screen of the camera, press any button on the remote control.
You should now see the LED flashing, because the camera is
sensitive to the infra-red light from the LED but your eye is not.

Figure 10.3.8 The infra-red light from the LED of a remote control has been
made visible by the digital camera that took this photograph.

Checkpoint 10.3
1
2
3
4
5
6
7

Describe what is used to make a diode.


Outline how a pn junction is connected to a power supply in order for electricity to flow.
Describe what happens to the distribution of electrons and holes when p- and n-type semiconductors are
brought into contact.
Outline how a rectifier works.
Explain why light is emitted in a LED.
Describe the different parts of a solar cell.
Explain why the n-type layer in a solar cell needs to be thin.

10.4 The control of electrical current


We have seen that the diode (or rectifier) enables current to be conducted in one
direction, for example, to convert AC to DC. Diodes are one of the most basic
electronic components, but they have limited control over current. Remarkably,
the first type of radio receiver used only a diode as a means of detecting a radio
signal and obtaining the audible sound without the need for batteries or any
other kind of power supply. This was called a crystal radio set and is still made
today by enthusiasts to detect AM radio stations.
A great deal more control over current is required for the wide variety of
electronic devices we use every day. For example, mobile phones have to transmit
an electromagnetic wave at a precise frequency by making electrical current
oscillate through an antenna. The phone also needs to detect a very weak
electromagnetic wave, then amplify it (make it larger) and decipher the
information contained within it such as voice, SMS, etc.

Figure 10.4.1 Thermionic devices (valves)


were replaced by the much
smaller transistors in modernday electronics.
197

10

Semiconductors
and the electronic
revolution

electrons

A number of discoveries and inventions led to components that were used to


accurately control the direction and magnitude of electrical current. The first of
these were vacuum tubes known as thermionic devices or valves, which were
later replaced by semiconductor technology such as diodes, transistors and
integrated circuits.

anode
electrons

cathode
heater

Thermionic devices

electrons
A

Figure 10.4.2 A thermionic diode enables


current to flow in one direction.

anode
electrons
grid
cathode
heater
heater
(filament)

cathode
grid

anode

Figure 10.4.3 A triode allows the electron


current to be controlled
electrically.

+150 V

The transistor

output

0V

Figure 10.4.4 A small AC signal is amplified


by a triode circuit.

198

During his research on incandescent lamps, Thomas Edison (18471931)


discovered that a current can be made to flow between a heated filament and
a metal electrode in a vacuum by applying a potential difference between them.
The filament and electrode were the cathode and anode respectively. Heating
the filament gives some electrons enough energy to escape the surface. These
electrons are then accelerated to the anode.
John Ambrose Fleming (18491945) applied this effect to the construction
of the first thermionic rectifier (diode) or valve, which allowed current to flow in
only one direction. The name valve comes from an analogous mechanical device
that allows fluids or gases to flow in one direction, such as the valve (or nozzle)
on a bicycle or car tyre that enables air to be pumped in but does not let it out.
A thermionic diode consists of three components: the filament (or heater),
the cathode and the anode (see Figure 10.4.2). The heater is used to heat the
cathode, which releases electrons that are attracted to the anode (the plate). Lee
De Forest (18731961) inserted a metal grid between the cathode and anode of
a thermionic diode. The grid allows electrons to pass through (see Figure 10.4.3).
This device is known as the triode. De Forest found that the electron current
could be stopped by placing a negative voltage on the grid, or it could be allowed
through by using a positive voltage. A voltage between these two extremes could
be used to control the amount of current flowing through.
One application of the triode is the amplifier (see Figure 10.4.4). If the
voltage on the grid is negative to the point that it only allows a very tiny current
through, then a small AC signal applied to the grid will cause current flowing to
the anode to vary in the same way. However, because of the large potential
difference between anode and cathode, this current is larger than that in the
original signal; that is, the triode has acted as a current amplifier. Moreover, this
current could be passed through a resistor, resulting in a varying voltage across it.
In this way, small voltage signals could be amplified. The invention of the triode
was in part responsible for the revolution in electronics, such as in radio and
television, in the early 20th century.
Transistors are semiconductor devices that control and amplify electrical current;
they have largely replaced thermionic devices. Just as the pn junction has
replaced the thermionic diode, the transistor has replaced the thermionic triode.
The transistor was invented in 1947 by John Bardeen and Walter Brattain, who
with their boss William Shockley received the Nobel Prize for it in 1956.
In the early days of the transistor, germanium was used as the semiconductor
because the methods existed for growing germanium crystals of high purity.
Although silicon is more abundant and retains its semiconducting properties at
higher temperatures, it needed a crystal of higher purity than could be achieved
at the time. Germanium diodes and transistors were replaced with those made
from silicon as soon as techniques for growing crystals improved and resulted in
higher purity silicon.

from ideas to
Implementation
The most common transistor is the bipolar (or junction) transistor. A schematic
diagram of a bipolar transistor is shown in Figure 10.4.5. This transistor works in
a similar way to the original transistor of Bardeen and Brattain, but its structure is
very different.
The bipolar transistor (Figure 10.4.5) consists of a p-type semiconductor
sandwiched between two n-type semiconductors. A lead (or wire) is connected
to each layer of the material. The two outer layers are called the emitter and the
collector; the central layer is called the base. This is often called an npn
transistor. The width of the base has been exaggerated for clarity, but is usually
very thin.
An electron current will only flow from the emitter to the collector if there is
a potential on the base with respect to the emitter (see Figure 10.4.6). Recall that
in a pn junction there is an electric field that points from n to p. Applying a
positive voltage on the base of an npn transistor with respect to the emitter
reduces the size of this field to almost zero. This then allows electrons from the
emitter to move into the base region. These electrons are then accelerated into the
collector region by the electric field at the junction between base and collector.
The electrons that moved into the collector can then flow into an external circuit.
Note that no current will flow without applying the potential to the
base. The current to the base is much smaller than the current that it allows to
flow from emitter to collector. Thus the transistor, like the triode, is a current
amplifier. The circuit given in Figure 10.4.7 has a small AC voltage applied to the
base of an npn transistor, which causes a much larger current to flow into an
external resistor between the emitter and the collector. The voltage across the
resistor is larger than the input signal but varies in the same way. This circuit is a
basic amplifier. Bipolar transistors also can be made in the pnp configuration.

Identify that the use of


germanium in early transistors
is related to lack of ability to
produce other materials of
suitable purity.

electrons
electrons
n
collector

p
base

collector

electrons
emitter

small
current

emitter

base

collector

emitter
base

Figure 10.4.5 A schematic diagram


of a bipolar transistor

+
Vbase

large
current

+
Vcollector

Figure 10.4.6 An electron current flows


through an npn transistor.

load
resistor

amplified
output

input
+

+
Vbase

Vcollector

Figure 10.4.7 This small signal amplifier


uses an npn transistor.

A comparison of solid state and thermionic devices


Semiconductor devices such as diodes and transistors do the same job as their
equivalent thermionic devices (the valve and the triode), but they have many
The semiconductor devices are much smaller than their
advantages.
thermionic counterparts and use much less power. They are mechanically
robust, whereas thermionic devices are made of glass and so are fragile, and
semiconductors have longer operating lifetimes due to their lower temperature
of operation.

Describe differences between


solid state and thermionic
devices and discuss why solid
state devices replaced
thermionic devices.

199

10

Semiconductors
and the electronic
revolution

PHYSICS FEATURE
Integrated circuits

nother type of transistor, called the field-effect


transistors (FET), shown in Figure 10.4.8, consists
of two n regions embedded in a larger and lightly doped
p substrate. The two n regions are called the source and
the drain. An insulating layer made from an oxide of the
semiconductor (such as silicon oxide) covers these, and
a metal electrode (called the gate) is placed on top of
this layer. Because the substrate is lightly doped, it is
not very conducting, so that no current will flow
naturally between the source and the drain. When a
positive potential is applied to the gate, it causes
electrons in the substrate to be attracted to the region
between the two n regions and forms a conducting
channel. In this way, current can be controlled by the
voltage on the gate.
This metaloxidesemiconductor field-effect
transistor (MOSFET) is not as common as the bipolar
transistor, but its construction makes it easy to place
many MOSFETs on a single wafer of silicon.
Combinations of MOSFETs, resistors and capacitors
form very complicated circuits known as integrated
circuits (IC). Integrated circuits containing hundreds
of thousands of transistors may be as small as a few
millimetres square. The IC is commonly known as the
silicon chip, and is used in all modern-day electronics
such as computers, televisions, mobile phones, etc.
There are many processes that go into producing an
integrated circuit and they vary between different
manufacturing plants and depend on the specifications
of the required IC. The following description simply
gives an indication of the many-step processes that go
into making an IC.

An n-type layer is sandwiched between a lightly


doped p-type silicon layer (the substrate) and a layer
of silicon dioxide (SiO2) deposited on it. Photographic,
chemical and ion-beam bombardment techniques
are used to dissolve (etch) unwanted areas and
deposit or dope other areas for the required patterns
of transistors, resistors and capacitors.

gate (small voltage)


source ()

drain (+)
insulating
layer

n
few electrons
can pass
substrate (p)

Figure 10.4.8 A schematic diagram of a


metaloxidesemiconductor
field-effect transistor (MOSFET)

silicon
dioxide
film
n-type

p-type
substrate

Figure 10.4.9 A wafer of p-type silicon with n-type


and insulating layers is the starting
point in the manufacture of an
integrated circuit.

Checkpoint 10.4
1
2
3
4
5
6
200

Describe the purposes of a diode and a triode.


Referring to Figure 10.4.2, explain the need for the heater in a thermionic diode.
Explain how a triode works.
Outline how an amplifier works.
Recall the reasons why germanium was originally used in semiconductor devices.
Identify problems associated with the use of germanium in these devices.

PRACTICAL EXPERIENCES

from ideas to
Implementation

CHAPTER 10

This is a starting point to get you thinking about the mandatory practical
experiences outlined in the syllabus. For detailed instructions and advice, use
in2 Physics @ HSC Activity Manual.

Activity 10.1: Electrons and holes


Using the computer simulation, investigate the behaviour of p-type and n-type
semiconductors, and a pn junction.
Discussion questions
1 Describe the changes in the energy diagram when the semiconductors are
doped with p-type or n-type dopants.
2 Use your observations of the simulator diode to explain what happens
inside a diode.

Activity 10.2: History of communication


Gather information about the history of the transistor and the development of
telecommunications, and consider how the materials and technology available at
the time influenced what could be developed.
Discussion questions
1 Explain how the use of semiconductors changed telecommunications.
2 Identify areas in which the introduction of the transistor has influenced
social behaviour.

Activity 10.3: Solar cells


Solar cells are devices that can convert sunlight directly to electrical energy.
Carry out an experiment to determine the effect of sunlight on a solar cell.
Equipment: solar cell, fan or small motor, digital multimeter.

Perform an investigation to
model the behaviour of
semiconductors, including the
creation of a hole or positive
charge on the atom that has lost
the electron and the movement
of electrons and holes in
opposite directions when an
electric field is applied across
the semiconductor.

Gather, process and present


secondary information to discuss
how shortcomings in available
communication technology lead
to an increased knowledge of the
properties of materials with
particular reference to the
invention of the transistor.
Identify data sources, gather,
process, analyse information and
use available evidence to assess
the impact of the invention of
transistors on society with
particular reference to their use
in microchips and
microprocessors.

Identify data sources, gather,


process and present information
to summarise the effect of light
on semiconductors in solar cells.

Discussion questions
1 Determine the angle (with respect to the direction of the Sun) of the solar
cell that delivers the greatest amount of power to the motor, or the greatest
voltage.
2 Determine the effect on power output of different wavelengths of light by
placing, for example, coloured cellophane in front of the solar cell.

201

10



Chapter summary

Semiconductors
and the electronic
revolution

The conduction of electrons can be described with


energy band diagrams.
Electrons in the valence band are not free to move and
do not contribute to conduction.
Electrons in the conduction band are free to move and
enable conduction to take place.
Insulators have a large energy gap between the
conduction and valence bands and not many valence
electrons have enough energy to jump up to the
conduction band.
Metals have no band gap and, therefore, there are always
electrons in the conduction band.
The energy-band diagram of semiconductors is similar
to that of insulators but with a much smaller band gap.
Semiconductors can be made more conducting by
exciting electrons from the valence band to the
conduction band by heat energy or a photon.
The conductivity of semiconductors can be improved
by introducing impurity atoms into the crystal lattice.

A semiconductor with impurities that introduce extra


electrons into the lattice is known as an n-type
semiconductor.
A semiconductor with impurities that introduce a vacancy
in the bonding is known as a p-type semiconductor.
The introduction of impurities into the semiconductor
crystal lattice is known as doping.
The impurities in semiconductors introduce extra
energy levels in the energy gap region and so improve
the conductivity of semiconductors.
Semiconductor devices can be made by using the
properties of the junction between p- and n-type
semiconductors, and include the diode, the solar cell
and the transistor.
Transistors replaced thermionic valve triodes used in the
control of electrical current.
Transistors have the advantage of being faster, smaller
and more energy efficient than thermionic valve triodes,
and thousands can be produced on a small silicon wafer
to make complicated integrated circuits.

Review questions
Physically speaking

Reviewing

Create a visual summary of the concepts in this chapter


by constructing a mind map incorporating the terms and
equation in the following table.

Outline the difference between energy levels and


energy bands.

Describe the difference in terms of energy gaps


between insulators, conductors and semiconductors.

Explain why it is wrong to say n-type semiconductors


are negatively charged or that p-type semiconductors
are positively charge.

Explain why the depletion zone occurs when p- and


n-type semiconductors are placed in contact with
each other.

p-type

n-type

Semiconductor

Energy bands

Depletion zone

Conductor

Valence band

Forbidden gap

Insulator

Thermionic devices

Solid state

Triode

Explain why the depletion zone will not form at


a temperature of absolute zero.

Diode

Transistor

Solar cell

Describe in terms of electric field strength why forward


and reverse bias occurs.

Holes

Electrons

Doping

7
8

Describe the path of an electron through a solar cell.

Explain in terms of the electric field at a pn junction


how current can flow in only one direction.

202

Give reasons why thermionic devices were not energy


efficient.

from ideas to
Implementation
10 Explain how a triode can be used to control the
amount of current flowing in a circuit.

11 Compare and contrast a triode with a transistor.


12 Explain why silicon was not originally used in making
semiconducting devices.

13 Explain the significance of integrated circuits


on society.

Solving problems
14 A 1 milliwatt (103 W) red laser pointer outputs

a wavelength of 650 nm (nm=1109m).


a Calculate the energy of a photon emitted from
the laser.
b Calculate the number of photons being emitted
every second. (Hint: Plancks constant
h = 6.63 1034Js.)

16 A photon of wavelength 3.35 m (1 m = 1 106m)


has just enough energy to raise an electron from
the valence band to the conduction band in a lead
sulfide crystal. Calculate the energy gap between
these bands in lead sulfide. (Hint: Plancks constant
h = 6.63 1034 Js.)

17 Figure 10.5.1 shows a sinusoidal input voltage Vi as a


function of time t for the given circuit. Draw a similar
graph of the output voltage Vo as a function of time.
Vi
+v
Vi
0

Vo

15 The electrical conductivity of undoped silicon can

iew

Q uesti o

Figure 10.5.1

Re

be increased by irradiating it with photons. This has


the effect of exciting valence electrons into the
conduction band. Given that the energy band gap of
silicon is 1.14eV, calculate the longest wavelength
of a photon that can excite a valence electron to
the conduction band. (Hint: Plancks constant
h = 6.63 1034 Js.)

203

11

Superconductivity
Surprising discovery

crystal, constructive interference,


destructive interference, path length,
diffraction grating, Bragg law,
phonons, critical temperature,
type-I superconductors,
type-II superconductors,
critical field strength, vortices,
flux pinning, BCS theory, Cooper pair,
coherence length, energy gap, spin

Just as an improved understanding of the conducting properties of


semiconductors led to the wide variety of electronic devices, research
into the conductivity of metals produced quite a surprising discovery
called superconductivity. This is the total disappearance of electrical
resistance below a certain temperature, which has great potential
applications ranging from energy transmission and storage to public
transport. An understanding of this phenomenon required a detailed
understanding of the crystal structure of conductors and the motion
of electrons through them.

11.1 The crystal structure of matter


A crystal is a three-dimensional regular arrangement of atoms. Figure 11.1.1
shows a sodium chloride crystal (ordinary salt also called rock salt when it comes
as a large crystal). The crystal is made from simple cubes repeated many times,
with sodium and chlorine atoms at the corners of the cubes. Crystals of other
materials may have different regular arrangements of their atoms. There are
14 types of crystal arrangements that solids can have.
The regular arrangement of atoms in crystals was a hypothesis before
Max Von Laue and his colleagues confirmed it by X-ray diffraction experiments.
William and Lawrence Bragg took this method one step further by measuring
the spacing between the atoms in the crystal. Let us first look at the phenomenon

Figure 11.1.1 Crystal structure of sodium chloride. The red spheres represent positive
sodium ions, and the green spheres represent negative chlorine ions.

204

from ideas to
Implementation
of interference of electromagnetic radiation, and examine how this was applied to
crystals using X-rays. Then we will see how the BCS theory of superconductivity
made use of the crystal structure of matter.

Try This!

Checkpoint 11.1

Crystals in the kitchen

Explain what is meant by the crystal structure of matter.

Look at salt grains through a


magnifying lens. Each grain is
a single crystal that is made from
the basic arrangement of sodium
and chlorine atoms shown in
Figure 11.1.1. Although the
grains mostly look irregular due
to breaking and chipping during
the manufacturing process,
occasionally you will see an
untouched cubic or rectangular
prism that reflects the underlying
crystal lattice structure.

11.2 Wave interference


The wave nature of light can be used to measure the size of very small spaces.
Recall that two identical waves combine to produce a wave of greater amplitude
when their crests overlap, as shown in Figure 11.2.1a (see in2 Physics @
Preliminary sections 6.4 and 7.4). The overlapping waves will cancel to produce
t=0s
a resulting wave of zero amplitude when the crest of one wave coincides with the
trough of the other (Figure 11.2.1b). This addition and subtraction is called
constructive and destructive interference respectively and is a property of all
wave phenomena.
t=1s
As an example, two identical circular water waves in a ripple tank overlap (see
Figure 11.2.2). The regions of constructive and destructive interference radiate
outwards along the lines as shown. Increasing the spacing between the sources
t = 3 s (Figure 11.2.2b).
causes the radiating lines to come closer together
a

t=0s

t=4s

Figure 11.2.1 Two identical waves (red, green) travelling in opposite directions can add (blue)
t=1s

(a) constructively or (b) destructively.t = 5 s

The interference of identical waves from two sources can also be represented
by outwardly radiating transverse waves (see Figure 11.2.3). The distance that a
twave
= 3 s travels is known as its path length. t = 6 s Constructive interference occurs
when the difference in the path length of the two waves is equal to 0, , 2, 3,
4 or any other integer multiple of the wavelength . Destructive interference
occurs when the two waves are half a wavelength out of step. This corresponds to
t=4s
t=7s
a path length difference of /2, 3/2, 5/2 etc.

t=5s

waves
in phase

lines of destructive
interference

lines of constructive
interference

Figure 11.2.2 Interference of water waves for


two sources that are (a) close
together and (b) further apart

constructive
interference

t=6s

destructive
interference
constructive
interference

t=7s

Figure 11.2.3 Constructive and destructive interference between


identical transverse waves from two sources
205

11

Superconductivity

Light also has wave properties and produces interference effects when it is
passed through two narrow and closely spaced slits (see Figure 11.2.4). Each slit
acts as a source of light waves that are in step with each other. The resulting
constructive and destructive interference pattern appears as bright and dark
bands on a screen.
The position of the bright bands can be determined by applying the
condition that the path length difference between the two waves must be an
integer multiple of wavelengths (see Figure 11.2.5). This can be expressed by
the following equation:

screen
double
slit

m = dsin
where m = 1, 2, 3 , d is the spacing between the slits and is the angular
position of the bright bands. The central bright band corresponds to m = 0.
The first band on either side of it corresponds to m = 1, and so on.
Interference also occurs when many slits are
constructive
used (see Figure 11.2.6). The bright bands
interference
become narrower and sharper on the edges, with
y
increasing number of slits. In practice, the slits

are parallel straight lines scratched, etched or


moulded onto a glass plate. This arrangement of
multiple slits is known as a diffraction grating.
A very good grating for visible light can
screen
have 2400 lines per millimetre. Such a grating
provides very sharply defined bands whose

Figure 11.2.4 An interference pattern is


formed by light passing
through two narrow slits.

S1
L
S2

slit 1
d

slit 2
d sin

waves
in phase

Figure 11.2.5 The path difference between waves


that produce constructive interference
constructive
interference

waves
in phase

TRY THIS!
Diffraction grating in my stereo
A commonly available diffraction grating is the humble CD
or DVD. It has thousands of tiny and closely spaced pits
that cause ordinary white light to break up into the colours
of the rainbow because there is an angle for constructive
interference for each wavelength contained in white light.
A more clearly defined diffraction pattern can be made by
shining a low power (less than 1 mW) laser pointer on the
CD or DVD so that the beam is reflected to a nearby wall.
You should notice that there are fainter dots near the
reflected dot. These fainter dots arise from constructive
interference of the laser light.

waves
in phase

destructive
interference

constructive
interference

Figure 11.2.6 Interference from multiple slits in a diffraction grating


206

from ideas to
Implementation
position can be accurately determined. The angular positions of the bright bands
Knowledge
are determined by the formula given above for the double slit.
of the wavelength can be used to determine the slit separation and vice versa.
To obtain an interference pattern, the distance between slits must be
close to the wavelength of the light falling on them. The interference pattern
from a diffraction grating is also called a diffraction pattern.

Checkpoint 11.2
1
2
3

Recall the conditions for constructive interference.


Describe how these differ from those for destructive interference.
Compare the interference pattern produced by light and that produced by two interfering water waves.

11.3 X-ray diffraction


X-rays are high-frequency electromagnetic waves first discovered by Wilhelm
Conrad Rntgen (18451923). He was experimenting with cathode ray tubes
and noticed that a fluorescent screen, some distance away from the tube, began
to glow each time the tube was operated. Rntgen had discovered a new type
of radiation, which he called X-rays to indicate that their nature was unknown.
Rntgen also found that X-rays had great penetrating power and could be used
for medical applications. Figure 11.3.1 shows one of the first X-ray pictures of
a hand taken by Rntgen.
Several years of experimentation by scientists after Rntgens discovery
showed that X-rays were electromagnetic waves like light but with an
extremely short wavelength. A schematic diagram of an X-ray tube is shown
in Figure 11.3.2. A beam of electrons strikes a metal target. The rapid
deceleration of electrons on striking the target causes X-rays to be emitted.
The wavelength of the most intense X-rays depends on the material from
which the target is made.
A single narrow slit can also be used to obtain a blurry diffraction pattern.
Using this crude method, rough estimates of 1010 m were obtained for the
wavelength of X-rays. It wasnt physically possible for a proper diffraction
grating with a well-controlled slit spacing of 1010 m to be made, so accurate
estimates of X-ray wavelength were not possible. Max von Laue (18791960)
learned of the hypothesis that crystals were regular arrangements of atoms
with a spacing between atoms of about 1010 m. He realised that crystals were
the gratings he needed for X-ray diffraction.
In 1912 W. Friedrich and P. Knipping followed von Laues suggestions and
passed a narrow beam of X-rays through a zinc sulfide crystal. A photographic
plate placed behind the crystal produced a regular pattern of spots (Figure
11.3.3), which was the interference pattern of the crystal which acted as a
diffraction grating.

Figure 11.3.1 X-ray picture of a hand taken


by Rntgen
cathode
(electron
source)
electron
beam

anode

vacuum
X-rays

heavy
metal
target
metal rod
(removes
heat and
electrons)

cooling fins

Figure 11.3.2 An X-ray tube


207

11

Superconductivity

Checkpoint 11.3

X-rays

1
2

Recall Rntgens observations that led to the discovery of X-rays.


Identify why X-rays are emitted when electrons strike a metal
surface.
Explain the significance of crystals in determining the
wavelength of X-rays.

crystal

3
lead
collimator

photographic
film

Figure 11.3.3 Laues suggestion for


X-ray diffraction

Outline the methods used by


the Braggs to determine crystal
structure.

11.4 Crystal structure


The Laue diffraction patterns qualitatively showed that a crystal indeed consisted
of a regular array of atoms. The idea of using the diffraction pattern to measure
the atomic spacing came from the Australian-born physicist Sir William
Lawrence Bragg (18901971), who, working with his father William Henry
Bragg (18621942), realised that crystals can be considered as consisting of
many planes oriented along different directions (see Figure 11.4.1).
Bragg realised that X-rays could penetrate the crystal structure and be
reflected from a set of parallel planes (see Figure 11.4.2). Constructive
In 1912, Bragg
interference is produced for certain reflection angles.
showed that the wavelength of X-rays was related to the spacing d between
planes by modifying the double slit constructive interference equation as follows:
n = 2dsin
where n is an integer and is the angle of incidence measured between the X-ray
beam and the crystal plane (see Figure 11.4.3). This is the Bragg law that
governs all modern X-ray diffraction.
This pioneering work has now become a standard method of determining
the crystal structure of materials.

Figure 11.4.1 The different planes of a cubic crystal

Figure 11.4.2 Crystal lattices consist of many


parallel planes in many directions
that can reflect X-rays.

incident
angle

reflection
angle

d sin

208

Figure 11.4.3 The parallel planes reflect X-rays to


produce constructive interference
according to the Bragg law.

from ideas to
Implementation

Which came first: or d?

From measurements of the diffraction angle , the Braggs were able to determine
the wavelength of X-rays. This needed an accurate knowledge of the spacing d
between planes in the crystal. However, this spacing was not known accurately,
since there was no method available to measure it. How can you measure
without knowing d and vice versa?
The Braggs solved this dilemma by using the diffraction pattern to determine
the arrangement of the atoms, without the need to know the space between them.
For example, it is possible to know that a crystal has a cubic arrangement from a
Laue diffraction pattern, without the need to know the spacing of atomic planes
Knowledge of the lattice geometry, the density
or the wavelength of X-rays.
of the crystal (mass/volume) and the mass (in grams) of the individual atoms
enables the spacing between atoms to be calculated accurately. This calculated
spacing d can now be used with the X-ray diffraction angle , to determine the
wavelength from the Bragg diffraction equation. Today, we simply use the
database of known crystal lattice spacings to determine X-ray wavelengths.

Checkpoint 11.4
1
2
3

Outline the Braggs contribution to the understanding of X-rays.


State the Bragg law.
Outline how the Braggs determined the spacing of the atoms on crystals.

11.5 Electrical conductivity and the


crystal structure of metals
X-ray diffraction has shown that the atoms of most metals exist in one of three
types of crystal lattices (see Figure 11.5.1). The crystal structure of metals can be
viewed as a lattice of positive ions surrounded by a sea of nearly free electrons,
which makes metals such good electrical conductors. The binding mechanism in
metals is the attractive force between the positive ions and the electron gas.
Remarkably, quantum physics predicts that there should be little or no
resistance to the motion of electrons in a perfect crystal lattice since electrons
behave like waves propagating through it. That is, the perfect regularity of the
crystal enables the electrons to travel unimpeded, as a wave through the crystal.

Identify that metals possess a


crystal lattice structure.
Describe conduction in metals as
a free movement of electrons
unimpeded by the lattice.
Identify that resistance in metals
is increased by the presence of
impurities and scattering of
electrons by lattice vibrations.

Figure 11.5.1 The structure of most metallic


crystals can be (a) body-centred
cubic, (b) face-centred cubic, or
(c) hexagonal close-packed.
209

11

Superconductivity

Anything that disturbs the regularity of the lattice results in electron collisions
and contributes to the electrical resistance of the crystal.
In practice, metals do show electrical resistance, as evidenced by the increase
in temperature of the metal wire in an electric heater or the heating element
on a stove. Resistance in metals originates from collisions of electrons with
irregularities in the crystal lattice. These can be caused by lattice vibrations, or
impurities (a foreign atom substituted for one in the crystal lattice) and defects
of the lattice (such as a missing atom) (see Figure 11.5.2). The crystal lattice of
all metals above a temperature of 0K consists of waves of lattice vibrations
known as phonons. A phonon colliding with an electron causes it to lose energy
and thus contributes to electrical resistance.
Real metal wires consist of many small crystals joined together and separated
by irregular boundaries. The boundaries also serve as places where electron
collisions take place and thus contribute to electrical resistance.
a

Figure 11.5.2 Electrical resistance is caused by electron collisions due to crystal lattice
(a) impurities, (b) defects and (c) vibrations.

The Kelvin temperature scale

he kelvin temperature scale differs from the Celsius scale by the following
expression:
kelvin = Celsius + 273.15

The lowest possible temperature is 0 kelvin (or simply 0K), which is


273.15C. The kelvin is named after the British physicist William Thomson
(18241907), who later was given the title Lord Kelvin. In 1900 he is
reported to have said, there is nothing new to be discovered in physics now.
All that remains is more and more precise measurement. This was before the
discovery of superconductivity, relativity, quantum physics, and all of the
modern physics that has led to a radical transformation of our society through
technology and our understanding of nature and the universe. Even great
scientists can be short sighted.

Figure 11.5.3 William Thomson


(Lord Kelvin)

Checkpoint 11.5
1
2
3
4

210

Outline the significance of X-ray diffraction to the structure of metals.


Explain the effect on electrical resistance of irregularities that are introduced into a crystal.
Give reasons why resistance in metals does not match the near-zero resistance predicted for crystal structures.
Outline the role of a phonon in electrical resistance.

from ideas to
Implementation

11.6 The discovery of


superconductors
0.15
Resistance ()

The phenomenon of superconductivity, in which the electrical resistance of


certain materials completely vanishes at low temperatures, is one of the most
interesting and sophisticated in condensed matter physics. It was first discovered
by the Dutch physicist Heike Kamerlingh Onnes (18531926), who was the
first to liquefy helium (which boils at 4.2 K at standard pressure). In 1911
Kamerlingh Onnes discovered the phenomenon of superconductivity while
studying the resistance of metals at low temperatures. He studied mercury
because very pure samples could easily be prepared by distillation.
The historic measurement of superconductivity in mercury is shown in
Figure 11.6.1. The electrical resistance of mercury decreased steadily when it
was cooled, but dropped suddenly to zero at 4.2K. Soon after this discovery,
many other elemental metals were found to have zero resistance when their
temperatures were lowered below a certain temperature that is characteristic of
the material. This is called the critical temperature Tc, some of which are given
in Table 11.6.1.

Hg
0.10

0.05

0.00
4.1

4.2
4.3
Temperature (K)

4.4

Figure 11.6.1 The resistance of mercury


as measured by
Kamerlingh Onnes

Table 11.6.1 Some superconductors, their critical temperatures and critical magnetic fields
Material

Critical temperature
(kelvin)

Critical magnetic field


strength (tesla)

Type I

Tungsten

0.02

0.0001

(elements)

Titanium

0.4

0.0056

Aluminium

1.18

0.0105

Tin

3.72

0.0305

Mercury ()

4.15

0.0411

Lead

7.19

0.0803

Type II

NbTi alloy

(compounds
and alloys)

NbZr alloy

10.8

11

PbMo6S8

14.0

45

V3Ga

16.5

22

10.2

Process information to identify


some of the metals, metal
alloys and compounds that
have been identified as
exhibiting the property of
superconductivity and their
critical temperatures.

12

Nb2Sn

18.3

22

Nb3Al

18.9

32

Nb3Ge

23.0

30

Type II

YBa2Cu3O7

92

(high-temperature
ceramic
compounds)

Bi2Sr2Ca2Cu3O10

110

Tl2Ba2Ca2Cu3O10

125

HgBa2Ca2Cu3O8

135

Too high to measure,


typically ~200
(estimated)

Superconductivity was an unexpected phenomenon. As shown in Figure


11.6.2, it was expected that by cooling a conductor the lattice vibrations
(phonons) will be gradually reduced in amplitude. The reduction in lattice
vibrations also reduces the number of collisions of electron with the crystal
lattice and therefore reduces the electrical resistance. One would expect the
resistance to gradually decrease to very low values at a temperature of 0 K. This
is why it was surprising to find that the resistance dropped to zero at a relatively
high temperature.

Electrical resistance

Class

superconductor

normal
metal

TC

Temperature (K)

Figure 11.6.2 The resistance of a normal


conductor and a superconductor
211

11

Superconductivity

Checkpoint 11.6
1
2
3

Recall how superconductivity was discovered.


Define critical temperature.
Compare the expected measurements of resistance as temperature is reduced with the experimental results.

11.7 The Meissner effect


T
Tc
Ba
0

Ideal
conductor

Superconductor

Figure 11.7.1 A thought experiment that illustrates


superconductors are not the same
as perfect conductors

In 1933, Walter Meissner and Robert Oschenfeld discovered that


superconductors expel magnetic fields from their interiors in a way that is
different from the behaviour expected of hypothetical perfect conductors.
Figure 11.7.1 illustrates a thought experiment that highlights this difference.
Imagine that both the ideal conductor and superconductor are above their
critical temperature Tc; that is, they both are in a normal conducting state
and have electrical resistance. A magnetic field Ba, is then applied, which
penetrates both materials. Each sample is then cooled to below its critical
It is found that
temperature, so that they both have zero resistance.
the superconductor expels the magnetic field from inside it, while the ideal
conductor maintains its interior field. Note that the energy needed by the
superconductor to expel the magnetic field comes from the superconducting
transition, which is exothermic. Switching off the magnetic field induces
currents in the ideal conductor that prevent changes in the magnetic field
inside it, as stated by Lenzs law (Module 2 Motors and generators).
However, the superconductor returns to its initial state; that is, it has no
magnetic field inside or outside it.

Checkpoint 11.7
Explain what happens to a magnetic field passing through an ideal conductor and a superconductor when the conductors
are cooled to below their critical temperatures.

11.8 Type-I and type-II superconductors


High magnetic fields destroy superconductivity and restore the normal
conducting state. Depending on the character of this transition, we may
distinguish between type-I and type-II superconductors. The graph in Figure
11.8.1 illustrates changes in the internal magnetic field strength Bi (the field
inside the superconductor) with increasing applied magnetic field. It is found
that the internal field is zero (as expected from the Meissner effect) until a
critical magnetic field Bc is reached, at which a sudden transition to the normal
state occurs. This results in the penetration of the applied field into the interior.
Superconductors that undergo this abrupt transition to the normal state
212

Checkpoint 11.8
1
2

Describe the significance of internal magnetic fields and critical


magnetic fields.
Distinguish between type-I and type-II superconductors.

Bc
External field Ba

Figure 11.8.1 Type-I superconductors


abruptly become normal
conductors at field strengths
above a critical magnetic field.

Internal field Bi

above a critical magnetic field strength are known as type-I superconductors.


Most of the pure elements listed in Table 11.6.1 tend to be type-I superconductors.
Type-II superconductors, on the other hand, respond differently to an
applied magnetic field (see Figure 11.8.2). These superconductors have two
critical field strengths, Bc1 and Bc2. As field strength is increased from zero, there
is no change in the internal magnetic field of the superconductor until Bc1 is
reached. At this field strength, the applied field begins to partially penetrate the
interior of the superconductor. However, the superconductivity is maintained at
this point. Superconductivity vanishes above the second, much higher, critical
field Bc2. For applied fields between Bc1 and Bc2, the applied field is able to
partially penetrate the superconductor, so the Meissner effect is incomplete and
the superconductor is able to tolerate very high magnetic fields.
Type-II superconductors are the most technologically useful because the
second critical field can be quite high, enabling high field strength electromagnets
Most compounds listed in Table
to be made out of superconducting wire.
11.6.1 are type-II superconductors. Wires made from, for example, niobiumtin
(Nb3Sn) have a Bc2 as high as 24.5 tesla, though in practice it is lower.
There is a misconception among some non-specialists that the term type
II refers to the copper oxide based high-temperature superconductors discovered
in the late 1980s. Although these are type-II superconductors, so are many
superconductors discovered before that time.

Internal field Bi

from ideas to
Implementation

Bc1

Bc2

External field Ba

Figure 11.8.2 Type-II superconductors have

11.9 Why is a levitated magnet stable?


Figure 11.9.1 shows a spectacular demonstration of the Meissner effect in
which a small permanent magnet floats on top of a high critical temperature
superconductor (YBa2Cu3O7) cooled with liquid nitrogen (at 77 K). It
demonstrates the repulsion of the magnetic field by the superconductor and
thus the levitation of the magnet.
Eddy currents are created on the surface of the superconductor, and,
consistent with Lenzs law, these essentially produce a magnetic field that mirrors
the field of the magnet, resulting in the repulsion and subsequent levitation of
the magnet (see Figure 11.9.2). In reality, when the magnet is first placed over
a small piece of superconductor it is unstable and falls off to the side. This is
because the magnet will float over its mirror image provided that image can keep
moving with it. The superconductor is small and cannot produce a satisfactory
magnetic field image near its edge, which results in ineffective repulsion.
So why does a levitating permanent magnet remain stable on top of a small
superconductor? Even a little nudge causes the magnet to spring back to its
original position as if somehow tied by invisible springs to that point. To explain

a partial penetration of the


magnetic field between two
critical fields.

Figure 11.9.1 A permanent magnet levitates


above a superconductor due to
the Meissner effect.
213

11

Superconductivity

induced
shielding
current

permanent
magnet
high-temperature
superconductor
image of
permanent
magnet

Figure 11.9.2 Eddy currents on the surface of


the superconductor essentially
create a mirror image of the
magnet resulting in repulsion
and levitation.

this, we need to expose a little secret used when demonstrating this levitation
experiment. If the magnet is lightly placed over a newly cooled high-temperature
superconductor, you should find that the magnet does not stay levitated for long
and falls off very quickly. To get the magnet to stay, hold the magnet over the
superconductor and, rather than letting it go, thrust it slightly towards the
superconductor. This is a subtle movement and usually goes unnoticed by the
audience. Now release the magnet and it will remain there stably. Incredibly, if the
magnet is then removed and dropped back over the superconductor, it levitates
stably without the need to thrust the magnet towards the superconductors. It is as
if the superconductor has remembered that the magnet was there. Moving the
magnet back and forth parallel to the surface of the superconductor or allowing
the superconductor to warm up above Tc and then cooling it down again will
make the levitation of the magnet unstable once more. The magnet must again be
thrust towards the superconductor to achieve stability. This is explained in the
following section.

Vortex states and flux pinning

Ba

600

Figure 11.9.3 (a) Illustration and (b)


photograph of a regular array
of normal conducting regions
(dark areas) in a type-II
superconductor where the field
penetrates the material.
214

Stable levitation of a permanent magnet above a small flat superconductor only


occurs with type-II superconductors. Certainly levitation occurs when using type-I
superconductors but with a type-II superconductor the levitation is particularly
stable and robust. The answer lies in the properties of type-II superconductors for
an applied magnetic field between the two critical fields Bc1 and Bc2. Recall that
for type-II superconductors, there is partial penetration of the magnetic field at
field strengths between Bc1 and Bc2. This partial penetration is in the form of a
regular array of normal conducting regions, as illustrated in Figure 11.9.3a.
Techniques have been developed to photograph these regions, which are shown as
a regular array of dark areas in Figure 11.9.3b.
These normal regions allow the penetration of the magnetic field in the form
of thin filaments, usually called vortices. The vortices are aptly named because
each is a vortex or swirl of electrical current that is associated with this state (see
Figure 11.9.3a). While in the vortex state, the material surrounding these normal
regions can have zero resistance and partial flux penetration. Vortex regions are
essentially filaments of normal conductor (non-superconducting) that run through
the sample when an external applied magnetic field exceeds the lower critical field
Bc1. As the strength of the external field increases, the number of filaments
increases until the field reaches the upper critical value Bc2; the filaments then
crowd together and join up so the entire sample becomes a normal conductor.
One can view a vortex as a cylindrical swirl of current surrounding a
cylindrical normal-conducting core that allows some flux to penetrate the interior
of type-II superconductors. Thrusting a permanent magnet towards a type-II
superconductor will cause the applied magnetic field at the superconductor to be
within the region of the two critical fields, which creates the vortex state. In
principle, the motion of a levitating permanent magnet will cause these vortices
In practice, real materials (such as high critical temperature
to move.
superconductors) have defects (missing or misplaced atoms, impurity atoms) in
their crystal lattices. They are also composed of many crystals, all bound together,
resulting in many crystal boundaries. The crystal defects and boundaries stop the
motion of the vortices, which is known as flux pinning. This provides the
stability of a levitating magnet. Pinning the motion of its magnetic field lines
also means stopping the motion of the magnet. Flux pinning can only occur in
type-II superconductors.

from ideas to
Implementation

Checkpoint 11.9
1
2
3
4

Explain what effect is being demonstrated by Figure 11.9.1.


Explain the contribution of eddy currents to the levitation of a magnet over a superconductor.
What are vortices?
Explain how thrusting the magnet towards the superconductor increases the stability of the levitation.

11.10 BCS theory and Cooper pairs


According to classical physics, part of the resistance of a metal is due to collisions
between free electrons and the crystal lattices vibrations, known as phonons.
In addition, part of the resistance is due to scattering of electrons by impurities
or defects in the conductor. As a result, the question arose as to why this doesnt
happen in superconductors.
A microscopic theory of superconductivity was developed in 1957 by
John Bardeen, Leon Cooper and J. Robert Schrieffer, and is known as the
BCS theory (after their initials). The central feature of this theory is that two
electrons in the superconductor are able to form a bound pair called a
Cooper pair if they somehow experience an attractive interaction between them.
At first this notion seems counterintuitive since electrons normally repel one
another because of their like charges.
An explanation of the formation of Cooper pairs relies heavily on quantum
physics; but here we present a classical picture of their formation (shown in
An electron passes through the lattice
Figure 11.10.1) and an explanation.
and at some point the positive ions are attracted to it, causing a distortion in
their nominal positions. The second electron (the Cooper pair partner) is
attracted by the positive charge of the displaced ions. This second electron can
only be attracted to the lattice distortion if it comes close enough before the ions
have had a chance to return to their equilibrium positions. The net effect is a
weak delayed attractive force between the two electrons.
This short-lived distortion of the lattice is sometimes called a virtual phonon
because its lifetime is too short to propagate through the lattice like a wave,
as a normal phonon would.
From the BCS theory, the total linear momentum of a Cooper pair
must be zero. This means that the electrons travel in opposite directions, as
shown in Figure 11.10.1. In addition, the nominal separation between the
Cooper pair (called the coherence length) ranges from hundreds to thousands of
ions! If electrons in a Cooper pair were too close, such as only a couple of atomic
spacings apart, then the electrostatic (coulomb) repulsion would be much larger
than the attraction from the lattice deformation and they would repel each other,
A current flowing in a
and there would be no superconductivity.
superconductor just shifts the total moment slightly from zero so that, on
average, one electron in a Cooper pair has a slightly larger momentum
magnitude than its partner. They do, however, still travel in opposite directions.
The interaction between electrons in a Cooper pair is transient. Each
electron in the pair goes on to form a Cooper pair with another electron, and
this process continues with the newly formed Cooper pairs so that each electron

Describe the occurrence in


superconductors below their
critical temperature of a
population of electron pairs
unaffected by electrical
resistance.
Discuss the BCS theory.

a
e

b
+ + e
+ +

c
+ +
+ +

Figure 11.10.1 Classical description of the


coupling of a Cooper pair.
(a) The first electron
approaches a section of the
lattice and (b) deforms part
of the lattice electrostatically.
(c) A second electron is
attracted to the net positive
charge of this deformation.

215

11

Superconductivity

PHYSICS FEATURE
High-temperature superconductors: The exceptions to the rule

n 1986 a class of materials was


discovered by Bednorz and Mller
that led to the superconductors we use
today on a bench top with liquid
nitrogen to cool them. Bednorz and
Mller received the Nobel Prize in
1987 for this work (the fastest ever
recognition by the Nobel committee).
The material we use mostly in school
science labs is the yttriumbarium
copper oxide compound YBa2Cu3O7,
otherwise known as the 1-2-3
superconductor. It is classified as a
high-temperature (Tc) superconductor.
The critical temperatures of some
high-temperature superconductors
Figure 11.10.2 Bednorz and Mller discovered high-temperature superconductors.
are given in Table 11.6.1. Critical
temperatures as high as 135K have
been achieved. This has made experiments on superconductivity more
accessible, since these need only to be cooled by liquid nitrogen (with
Cu-O chains
a boiling point of 77 K), which is cheap and readily available. This is in
contrast to the expensive and bulky equipment that uses liquid helium
for cooling the traditional types of superconductors.
The crystal lattice structure of YBa2Cu3O7 is shown in Figure 11.10.3.
Cu(2)
Unlike traditional superconductors, conduction mostly occurs in the planes
Y
containing the copper oxide. It has been found that the critical temperature
Ba
CuO2
is very sensitive to the average number of oxygen atoms present, which can
Cu
layer
O
vary. For this reason the formula for 1-2-3 superconductor is sometimes
given as YBa2Cu3O7 where is a number between 0 and 1.
The nominal distance between Cooper pair electrons (coherence length)
in these superconductors can be as short as one or two atomic spacings.
Cu(1)
As a result, the electrostatic repulsion force will generally dominate at
these distances, causing electrons to be repelled rather than coupled.
For this reason, in these materials it is widely accepted that Cooper
Figure 11.10.3 The crystal structure
of YBa2Cu3O7,
pairs are not caused by a lattice deformation, but may be associated with
a high-temperature
the type of magnetism present (known as antiferromagnetism) in the copper
superconductor
oxide layers. This means that high-temperature superconductors cannot
be explained by the BCS theory, since that mainly deals with lattice
deformations mediating the coupling of electron pairs. The research
continues into the actual mechanism responsible for superconductivity
in these materials.

216

from ideas to
Implementation
goes on to form Cooper pairs with other electrons. The end result is that each
electron in the solid is attracted to every other electron, forming a large network
of interactions. Causing just one of these electrons to collide and scatter from
atoms in the lattice means the whole network of electrons must be made to
collide into the lattice, which is energetically too costly. The collective behaviour
of all the electrons in the solid prevents any further collisions with the lattice.
In this case, the
Nature prefers situations that spend a minimum of energy.
minimum energy situation is to have no collisions with the lattice. A small
amount of energy is needed to destroy the superconducting state and make it
normal. This energy is called the energy gap.
In addition to having a linear momentum, each electron behaves as if it is
spinning. This property, not surprisingly, is called spin. (The electron is not
The BCS theory requires
actually spinning, but behaves as though it does.)
that the spins of Cooper pair electrons be in opposite directions.

Checkpoint 11.10
1
2
3

Describe how classical physics explains resistance in metals.


Outline how Cooper pair electrons form.
Describe what is meant by the total linear momentum of a Cooper pair must be zero.

11.11 Applications of superconductors


The main advantage of superconductors is that there is no heat lost when
passing current through them, which means that currents can be made to persist
indefinitely. For example, a superconducting electromagnet is made such that
external power is applied for only a very short time. The electromagnet is then
formed into a closed loop that enables the current (and field) to persist as long as
the superconductor stays below its critical temperature; that is, the external
power supply can be switched off!
The main disadvantage of superconductors is that they must be
maintained at very low temperatures. This requires specialised vessels known as
cryostats, which contain the cooling fluids such as liquid nitrogen and liquid
helium. The brittle nature of high-temperature superconductors has limited their
applicability. As a result, most superconducting applications today still use the
more traditional superconductors such as niobium-tin (Nb3Sn), which can be
made into flexible wires. The disadvantage is that they require cooling with
liquid helium, which is much more expensive than liquid nitrogen and must be
contained in more sophisticated cryostats.

Discuss the advantages of


using superconductors and
identify limitations to their use.

Medical applications
The first large-scale commercial application of superconductivity was in
magnetic resonance imaging (MRI). This is a non-intrusive medical imaging
technique that creates a two-dimensional picture of, for example, tumours and
other abnormalities within the body or brain. This requires a person to be placed
inside a large and uniform electromagnet with a high magnetic field. Although
normal electromagnets can be used for this purpose, their resistance would

Figure 11.11.1 Patients are located inside the


bore of the superconducting
magnet of an MRI machine.
217

11

Superconductivity

dissipate a great deal of heat and have large power requirements. Superconducting
magnets, on the other hand, have almost no power requirements apart from that
required for cooling. Once electrical current flows in the superconducting wire,
the power supply can be switched off because the wires can be formed into a loop
and the current will persist indefinitely, as long as the temperature is kept below
the transition temperature of the superconductor.
Superconductors can also be used to make a device known as a superconducting
quantum interference device (SQUID). This device is extremely sensitive to small
magnetic fields and can detect magnetic fields from the heart (1010 tesla) and
even the brain (1013 tesla). For comparison, the Earths magnetic field is about
104 tesla. As a result, SQUIDs are used in non-intrusive medical diagnostics of
the brain.
Process information to discuss
possible applications of
superconductivity and the
effects of those applications on
computers, generators and
motors and transmission of
electricity through power grids.

Figure 11.11.2 ITER, a proposed test reactor


for future clean energy
production using nuclear
fusion

218

Scientific research
The traditional use of superconductors has been in scientific research requiring
high magnetic field electromagnets. One application of powerful superconducting
electromagnets is in high-energy particle accelerators, such as the Large Hadron
Collider at CERN (see section 15.4), in which beams of protons and other
particles are accelerated to almost the speed of light and made to collide with each
other to create more elementary particles. It is expected that this research will
answer fundamental questions such as those about the origin of the mass of matter
that makes up the universe.
A future use of superconducting electromagnets is in nuclear
fusion energy generation using plasmas. A plasma is a fully ionised gas
that is obtained by heating it to millions of degrees and trapping it
inside a toroidal structure known as a tokamak by large
electromagnets. The nuclei of the ions fuse together, producing
energy. The operating gases of such reactors are deuterium and
tritium, the isotopes of hydrogen. Deuterium is abundant in water,
but tritium will be made inside the tokamak as a by-product of fusion
reactions. There is no long-term radioactive waste with this process,
which is why it is known as clean nuclear energy. Currently an
international research plasma reactorthe International
Thermonuclear Experimental Reactor (ITER)is being built. The
aim of the project is to demonstrate that energy production is possible
with this method. A diagram of the projected reactor is shown in
Figure 11.11.2. The magnetic field coils on the ITER will be made
from superconductors and will need to only be powered once; the
current through them will be sustained indefinitely, as long as the coils
are kept cool.

Levitating trains
Magnetic levitation (maglev) trains have been built that use powerful electromagnets made from superconductors. The superconducting electromagnets are
mounted on the train and kept cool with liquid helium. As shown in Figure
11.11.3, normal electromagnets on a guideway beneath the train repel (or attract)
the superconducting electromagnets to levitate the train while pulling
it forwards. The superconducting electromagnets rely on the conventional
like-pole repulsionnot the Meissner effectto achieve levitation.

from ideas to
Implementation
Although such a superconducting maglev train has been built and has
demonstrated a top speed of 581kmh1, there are several issues that limit its
widespread commercial use. A strong magnetic field inside the train will exclude
passengers with pacemakers or devices that have magnetic data storage including
computers and credit cards. The powered conventional electromagnets on the
guideway that levitate and propel the train are expensive to run over long
distances, so alternative propulsion schemes may have to be used.

Power generation, transmission and storage


Superconductors have the potential to make electricity generation and
transmission more efficient, which will reduce energy costs and the emission
of greenhouse gases. Electricity generators used in coal-fired, nuclear and
hydroelectric power plants use electromagnets, which heat up due to the
resistance of their wires. Replacing these with superconducting wires will at least
halve the amount of power lost in the electromagnets, even when the energy
cost of making the liquid helium or nitrogen to keep the superconducting
electromagnets in the superconducting state is taken into account.
The transmission of power to homes and businesses is carried out by
high-voltage transmission lines. A high voltage enables a small current to be
passed through the transmission lines, to minimise the amount of resistive
heating in the wires. Nevertheless, there is some heating of the transmission
lines and a substantial energy loss associated with it. Superconducting
transmission lines will not suffer from such heating losses. Moreover, problems
associated with high-voltage leakage of power by ionisation of the air can be
overcome by reducing the voltage on the transmission lines and increasing the
current through them.
Energy production at, say, coal-fired power plants varies, depending on the
anticipated demand for electricity. At the moment, any overproduction of
energy is stored by pumping water to higher levels in a large reservoir. Releasing
this water into hydroelectric generators enables this stored energy to be retrieved
as electricity. The problem with this method is that it is highly inefficient.
A third of the excess energy is needed to operate the pumps to the reservoir.
Superconductors offer the possibility of storing an electrical current
indefinitely in superconducting rings. This current could be retrieved at any
time, provided the rings remain in the superconducting state. This technology
may also solve problems such as the variability of supply by solar energy
generation. Night-time power could be obtained from daytime storage of excess
solar power.

superconducting
levitation magnet

superconducting
propulsion
magnet

guideway
propulsion
magnet
vehicle

gliding
skid
guidance
and
braking
levitation
and
propulsion
magnet

armature
windings
(iron core)
guideway

Figure 11.11.3 Levitation of a train using onboard


superconducting magnets on a
guideway that propels it with
conventional electromagnets

PRACTICAL
EXPERIENCES
Activity 11.1

Activity Manual, Page


100

Checkpoint 11.11
1
2

List the advantages and disadvantages of superconductors.


State an application of superconductors and explain why it is an improvement on existing technology.

219

11

Superconductivity

PRACTICAL EXPERIENCES
CHAPTER 11

This is a starting point to get you thinking about the mandatory practical
experiences outlined in the syllabus. For detailed instructions and advice, use
in2 Physics @ HSC Activity Manual.
Perform an investigation to
demonstrate magnetic
levitation.
Analyse information to explain
why a magnet is able to hover
above a superconducting
material that has reached the
temperature at which it is
superconducting.
Gather and process information
to describe how superconductors
and the effects of magnetic
fields have been applied to
develop a maglev train.
Process information to discuss
possible applications of
superconductivity and the
effects of those applications on
computers, generators and
motors and transmission of
electricity through power grids.

Activity 11.1: Applications of superconductivity


Magnetic levitation is the ability to control magnetic repelling forces in order to
balance objects above each other. You will demonstrate that stable levitation of
a permanent magnet can be achieved.
Equipment: 2 circular magnets with holes in the centre, superconductor kit
(optional), liquid nitrogen.
Discussion questions
1 Describe the orientation of the poles of the magnets in order for levitation
to occur.
2 Describe your attempts at achieving levitating magnets without the
supporting rod through the centre of the magnets. Discuss how this might
differ for a levitating magnet over a superconductor.

Figure 11.12.1 Set-up to have magnets levitate

220

Chapter summary

An X-ray diffraction pattern results from the


interference of X-rays to form a pattern of dots on
photographic film that can then be used to deduce the
crystal structure of a solid.
The Bragg law relates the X-ray wavelength to the
spacing d between crystal planes and the diffraction
angle by the equation:
n = 2dsin
where n is an integer.
Resistance in metals originates from collisions of
electrons with irregularities in the crystal lattice.
Crystal lattice vibrations are known as phonons.
Superconductivity is the total disappearance of electrical
resistance below a critical temperature Tc that depends
on the material of the superconductor.
The Meissner effect is the expulsion of the magnetic
field from the interior of a superconductor.
Magnetic fields can destroy superconductivity and
restore the normal conducting state.
Type-I superconductors undergo an abrupt transition to
the normal state above a single critical magnetic field.

from ideas to
Implementation

Type-II superconductors have two critical magnetic


fields. The transition to the normal state occurs above
the higher critical field.
Type-II superconductors form regions of normal
conductivity around which there are circulating
electrical currents, known as vortices.
Magnetic field lines penetrate the interior of a type-II
superconductor through the normal regions of the
vortices.
The BSC theory explains superconductivity by the
coupling of electron pairs, known as Cooper pairs,
through an interaction with a lattice phonon.
The energy gap is the minimum energy required to
destroy the superconducting state.
Applications of superconductors include electromagnets
for MRI machines, and superconducting quantum
interference devices (SQUIDs) for the detection of very
small magnetic fields.
Superconductors have potential applications in fusion
energy research, levitating trains and power generation
and transmission.

Review questions
Physically Speaking

Reviewing

The items in the columns are not in their correct order.


Match each of the key concepts with its closest definition.

1 Outline the work carried out by the Braggs in

Concept

Definition

Superconductivity

The point below which there is zero


resistance
The array of dots on a photographic
film created by X-ray diffraction
A burst of lattice vibrations
The state of matter in which electrical
resistance is zero
Allows a magnetic field to penetrate
while maintaining the superconducting
state
The mathematical relationship between
X-ray wavelength and crystal lattice
spacing
The temporary attraction between
electrons mediated by a lattice
deformation
Expels magnetic fields from the interior
The regular arrangement of atoms in
a solid

Crystal lattice
Laue pattern
Phonon
Bragg law

Tc

Meissner effect

Type-II superconductor
Cooper pairs

understanding the wavelength of X-rays.

2 Describe the property of crystals that make them


useful in understanding X-rays.

3 Explain the difference between constructive and


destructive interference.

4 Outline how the approximate wavelength of X-rays was


found. State the value achieved.

5 State the prediction by quantum physics of


resistance in crystal structures and explain why these
are not always seen.

6 Describe the role of phonons in superconductivity.


7 Outline the contributions of Kamerlingh Onnes to the
understanding of superconductivity.

8 Explain how the drop in temperature allows for zero


resistance.

9 Describe the Meissner effect.


221

11

Superconductivity

10 State the difference between a conductor and a


superconductor when a magnetic field is applied and
the temperature is reduced beyond the critical
temperature.

Solving Problems
19 Calculate the separation of diffraction grating slits
when a heliumneon laser (=633nm) is shone on
it. The angular position of the second-order bright
band is 30.

11 Explain why the superconducting transition is


exothermic.

12 Describe why a magnet levitates above a


superconductor that is below its critical temperature.

13 Explain why there is the formation of eddy currents on


the surface of superconductors.

14 Explain what is meant by a Cooper pair and describe

20

a Identify the critical temperature of each metal in


Figure 11.12.2.
b Identify which of the two plots (diamonds or
squares) shows a material with easily obtainable
superconducting properties. State your reasons.

15 Outline an energy argument as to why electrons travel


through a superconductor unimpeded when it is
below its critical temperature.

16 Outline the BCS theory.


17 Create a table to compare the advantages and

Resistance ()

how they are formed.

1000
800
600
400
200
0

disadvantages of superconductors.

Re

222

iew

Q uesti o

18 List applications of superconductors in everyday life.

50

100

150

Temperature (K)

Figure 11.12.2

200

250

from ideas to
Implementation

PHYSICS FOCUS

H3. Assesses the impact of particular


advances in physics on the
development of technologies

Semiconductors to
Superconductors

H5. Identifies possible future directions


of physics research

1950s

1960s

1970s

1980s

Silicon transistor

TTL Quad gate

8-bit Microprocessor

32-bit Microprocessor

1 transistor

16 transistors

4500 transistors

275000
transistors

1990s

2000s

32-bit Microprocessor 64-bit Microprocessor

3100000
transistors

592000000
transistors

Figure 11.12.3 Moores law states that the number of transistors on a chip doubles every 2 years.
Since the introduction of the commercial transistor in
the late 1940s, the size and processing ability of
computer chips has changed dramatically. According
to Moores law, the number of transistors on a chip
doubles every 2 years. But there is a limitthe
physical size of atoms.
In the distant future, a new generation of
computers that make use of quantum physics may
overcome the size limitations of current (classical)
computer technology. Quantum computers allow for
processes to happen simultaneously rather than
sequentially as in classic computers. For example, to
factorise a 400 digit number would take a quantum
computer a few minutes, but it would take current
computers billions of years to do the same calculation.
Another distant technology that may speed up our
current limitations on computation speed are the
superconducting switching devices known as
Josephson junctions. Although not the only technology
being researched for future computers, this is one
to watch.

1 Define what a semiconductor is.


2 Explain the significance of doping semiconductors.
3 State the significance of semiconductors in the
computing industry.
4 Outline the advantages of silicon over the
previously used germanium.
5 Explain why the pn junction is important to
modern electronics.
6 Discuss the impacts the pn junction has had
on society.
7 Analyse the implication of a limit to the size of
silicon chips.
8 Discuss the need to involve quantum mechanics
in the next generation computers.
9 Describe why quantum computers are so much
faster than standard computers.

Research
10 Research and outline how the Josephson junction
works.
11 Research and describe the possible technologies
that can take over from semiconductors.
12 Outline the role of nano-carbon tubes in speeding
up processing ability.

223

The review contains questions in a similar style and proportion


to the HSC Physics examination. Marks are allocated to
each question up to a total of 25marks. It should take you
approximately 45 minutes to complete this review.

Multiple choice
(1 mark each)
1 Predict the direction of the electron in Figure 11.13.1
as
A
B
C
D

II

III

Conductor
Insulator
Insulator
Semiconductor

Insulator
Conductor
Semiconductor
Conductor

Semiconductor
Semiconductor
Conductor
Insulator

The graph in Figure 11.13.3 shows how the


resistance of a material varies with temperature.
Identify each of the parts labelled on the graph.

A
B
C
D

224

Figure 11.13.1

An electron in a magnetic field

The diagrams in Figure 11.13.2 represent


semiconductors, conductors and insulators. The
diagrams show the conduction and valence bands,
and the energy gaps. Which answer correctly labels
each of the diagrams?

A
B
C
D

II

III

Critical
temperature
Superconductor
material
Critical
temperature
Normal material

Superconductor
material
Critical
temperature
Normal material

Normal material

Superconductor
material

Figure 11.13.2

Normal material
Superconductor
material
Critical
temperature

II

II

Figure 11.13.3

III

Energy bands

Resistance ()

it enters the magnetic field.


Straight up
Left
Right
Down

III

Temperature (K)

Resistance varies with temperature

from ideas to
Implementation
4

Experimental data from black body radiation during


Plancks time showed that predicted radiation levels
were not achieved in reality. Planck best described
this anomaly by saying that:
A classical physics was wrong.
B radiation that is emitted and absorbed is
quantised.
C he had no explanation for it.
D quantum mechanics needed to be developed.
Figure 11.13.4 shows a cathode ray tube that has
been evacuated. Which answer correctly names each
of the labelled features?

III

Figure 11.13.4 An evacuated cathode ray tube

C
D

Explain, with reference to atomic models, why


cathode rays can travel through metals. (2marks)

Outline how the cathode ray tube in a TV works


in order to produce the viewing picture. (2 marks)

Give reasons why CRT TVs use magnetic coils and


CROs use electric plates in order to deflect the
beams, given that both methods work. (2 marks).

In your studies you were required to gather


information to describe how the photoelectric effect
is used in photocells.
a Explain how you determined which material was
relevant and reliable.
b Outline how the photoelectric effect is used in
photocells. (3 marks)

10

Justify the introduction of semiconductors to replace


thermionic devices. (4 marks)

11

Magnetic levitation trains are used in Germany and


Japan. The trains in Germany use conventional
electromagnets, whereas the one in Japan uses
superconductors. Compare and contrast the two
systems. (3 marks)

12

a


b

II

A
B

Extended response

II

III

Striations
Faradays
dark space
Crookes
dark space
Cathode

Cathode
Striations

Anode
Cathode

Anode

Faradays
dark space
Striations

Faradays
dark space

Determine the frequency of red light, which has


a wavelength = 660 nm. (Speed of light
c = 3.00 108 m s1)
Calculate the energy of a photon that is emitted
with this wavelength. (Plancks constant
h = 6.63 1034 J s) (4 marks)

225

4
Context

Figure 12.0.1

226

Bubble chamber tracks


formed by the passage of
ionising particles through
a liquid that is kept at a
temperature above its boiling
point. The curved trajectories
are the result of charged
particles interacting with
a magnetic field, and allow
information concerning the
charge and mass of the
particles to be determined.

Quanta to
Quarks
We all talk about quantum leaps, but did you know that Max Planck made the first
quantum leap in 1901 when he introduced the idea of the quantisation of energy?
At the beginning of the 20th century, scientists were confronted by an accumulation
of experimental observations and explanations that lacked unification. Black body
radiation, the photoelectric effect, radioactivity and the emission of sharp spectral
lines by atoms in a gas discharge tube could not be adequately explained within the
framework of Newtonian classical physics.
A new physics, quantum physics, was born. The story has some inspiring
characters and storylines: how Niels Bohr synthesised the works of Planck, Einstein
and Rutherford and proposed the now commonly recognised RutherfordBohr
atomic model, and so provided an explanation for spectral lines; and how Louis de
Broglie in 1924 took Plancks idea, reversed it and proposed a totally crazy idea that
all matter has wave properties, which, in turn, gave birth to quantum mechanics.
The investigations into radioactivity led to the artificial manufacture of elements
and the dawn of the atomic age. The development of particle accelerators often
referred to as atom smashers in the media, led to the discovery of a particle zoo,
as a plethora of new subatomic particles were identified. Today, the quest to
understand the building blocks and forces of nature continues, with the building of
the Large Hadron Collider (referred to as the LHC), which is designed to produce
conditions that mimic the environment present just after the birth of the universe.

Figure 12.0.2 The blue glow in the core of

Inquiry activity
A chain reaction

a water-cooler nuclear results


from the radiation emitted
when energetic charged
particles travel faster than
light through water.

Obtain some scrap paper! Gather together a crowd of peopleyour class will do
the more the better! And make sure you have safety glasses for all! Screw up the
paper into hundreds of balls, each about the size of a ping-pong ball. Make sure
everyone has at least six balls of paper and then gather everyone in close together.
These are the rules of the chain reaction game.
1 Safety glasses on at all times!
2 If you are hit by a paper ball you throw two balls of paper high up into the air.
Your teacher can lob in the first paper ball. It makes a great video clip!
You can also vary the rules. For example, try throwing one paper ball when you
are hit rather than two. Have fun!

227

12

From Rutherford
to Bohr
Pieces of a jigsaw!

Rutherford, Bohr, spectra, orbit,


spectrum, quantum number, Plancks
constant, quanta, photon, absorption
spectra, emission spectra, Balmer
series, Rydbergs constant, transition,
stationary state, Zeeman effect

By the beginning of the 20th century a large number of experimental


facts had accumulated that could not be explained by existing
theories:
the discovery of ordered series in atomic spectra
the photoelectric effect
radioactivity
evidence that the atom had internal structure.

12.1 Atomic timeline

Ernest
Rutherford

nce, a distinguished
stranger, amazed by
his unscholarly accent and
appearance, mistook him
for an Australian farmer.
At the University of
Manchester, Rutherford
would proclaim to his
recruits that: all science
is either physics
or stamp collecting.

Figure 12.1.1

228

Ernest Rutherford is most well known for his


alpha-particle scattering experiments. He
was awarded the Nobel Prize for Chemistry
in 1908 for his work in investigations into
the disintegration of the elements, and
the chemistry of radioactive substances.

In the early 20th century many prominent scientists


doubted the existence of atoms. Today atomic theory
is widely accepted and taught throughout science
curricula, forming the foundation upon which
scientists, technologists and engineers understand the
properties of matter. The ideas underpinning atomic
models have changed over time, driven on by the
interplay between available technology, theoreticians
and experimentalists. The history of the atom
originates in Greece more 2000 years ago and the
quest to reveal its inner structure continues today.
The atomic age was born, ushered in by the
development and construction of atomic weapons,
nuclear reactors and particle accelerators.

quanta to
quarks
Table 12.1.1

Atomic timeline

Democritus
(c460 bcec.370 bce)

He was a Greek philosopher who proposed that there was a limit to how
small one could divide matterthe smallest indivisible particle was
called an atom (atomos, Greek meaning without slices or indivisible).

Aristotle
(384 bce322 bce)
John Dalton
(17661844)

He criticised Democritus, and proposed a model based upon four


elementsearth, air, fire and water. His view held for some 2000 years.

Henri Becquerel
(18521908)
JJ Thomson
(18561940)

In 1896 he discovered that certain elements emitted radiation and


decayed, suggesting that the atom was divisible.

Ernest Rutherford
(18711937)

In 1911 he proposed the Rutherford planetary model of the atom,


based upon the results of Geiger and Marsdens scattering experiments
at the Cavendish Laboratories.

Niels Bohr
(18851962)

In 1913 Bohr proposed the RutherfordBohr model also commonly


called Bohrs model. Bohr provided a set of three postulates to address
the issues raised by Rutherfords earlier model, and this led to the
development of a mathematical model to account for the spectra of the
hydrogen atom.

Louis de Broglie
(18921987)

In 1924 de Broglie introduced the concept of matter waves. This


concept provided a mechanism for electrons to inhabit a stable orbit by
having an integral number of wavelengths fitting around the
circumference of the orbit, forming a standing wave.

James Chadwick
(18911974)

In 1932 he reported the discovery of the neutron. Rutherford some 12


years earlier had proposed its existence, and this discovery completed
the constituents of the basic atomic model with which most people are
familiar today.

Experimental
surprise

A Scottish teacher, Dalton in 1801 proposed his atomic model, based


upon his studies in chemistry that:
Matter is composed of small indivisible atoms.
Elements contain only one type of atom.
Different elements contain different atoms.
Compounds contain more than one type of atom.

utherfords radium produced


alpha particles, and these
massive particles travelled at
approximately 1.610+7ms1.
Rutherford in a later lecture
described the extraordinary
backscattering of alpha particles
as the most incredible event
that has ever happened to me in
my life. It was almost as
incredible as if you had fired
a 15-inch shell at a piece of
tissue paper and it came back
and hit you.

In 1904 he proposed the plum pudding model of the atom in which


electrons were embedded in a positive sphere like plums in a pudding.
This model was based upon Thomsons experimental work and his
discovery of electrons in 1897.

Checkpoint 12.1
Outline the main atomic models proposed between ancient times and 1913.

~1010 m

12.2 Rutherfords model of the atom


In 1907, New Zealander Ernest Rutherford (18711937) moved to the
University of Manchester in England where, with Johannes Geiger (18821945)
and Ernest Marsden (18891970), he continued his earlier work on firing alpha
particles at metal foils.
They were shocked to find that approximately one alpha particle in
every 8000 was deflected by the platinum and gold foils through angles greater
than 90. The Thomson (plum pudding) model of the atom (Figure12.2.1)
predicted only small scattering because the atom had no large concentrations of
charge or mass to deflect the massive and fast-moving alpha particles. A new
atomic model was needed.

electron

Figure 12.2.1

positively
charged
material

Thomsons plum pudding


atomic model

229

12

From Rutherford
to Bohr

Structure

Discuss the structure of the


Rutherford model of the atom,
the existence of the nucleus
and electron orbits.

The alpha particle experiments posed many questions. Rutherford hypothesised


that for alpha particles to be deflected as observed, it would require a massive,
but tiny, positively charged charge centre (nucleus), approximately 1015 m in
diameter with a set of orbiting electrons (like planets orbiting the Sun). From
Einsteins analysis of Brownian motion, the radius of an atom was approximately
1010m, meaning that the tiny nucleus contains 99.9% of the mass and the
atom is mostly empty space.
By 1910 Rutherford had formalised his atomic model with mathematical
equations and directed Geiger and Marsden to thoroughly test his model. The
series of experiments between 1908 and 1911 provided clear evidence that
Thomsons model was not correct. Interestingly, the scientific community was
not persuaded or even interested. The two people who brought Rutherfords
model into mainstream science were Niels Bohr (18851962) and a young
member of Rutherfords team Henry Moseley (18871915).

Limitations
Despite the success of Rutherfords planetary atomic model in explaining the
scattering of alpha particles, the model failed to explain other important
questions:
What is the nucleus made of?
How are the orbits of the electrons arranged around the nucleus?
What keeps the negatively charged electrons from losing energy and spiralling
into the positive nucleus?

a
++

+ +
+

b
nucleus

Figure 12.2.2

Scattering deflections
predicted by (a) Thomsons
and (b) Rutherfords atomic
models

Rutherford proposed that electrical attraction provided the centripetal force


to keep electrons in orbit. However, orbiting electrons are accelerating, and
Maxwells classical electromagnetic theory predicted that accelerating electrons
would radiate away their energy in a fraction of a second and spiral into the
nucleus, therefore Rutherfords atomic model was not stable. Additionally,
Rutherfords planetary model could not explain the observed line spectra of
excited gases.
viewing screen

pa

rticle

source containing radon

b
metal
foil

particle

+
nucleus

Figure 12.2.3

230

The components used by Geiger and Marsden to measure the deflection


of alpha particles fired at thin metal-foil targets

quanta to
quarks

Checkpoint 12.2
1
2

Explain why Thomsons plum pudding model only predicted small deflections in alpha particles passing through
a thin metal foil.
Outline the significance of Marsden and Geigers scattering experiments to the development of the Rutherford
atomic model.

12.3 Plancks quantised energy


Max Planck (18581947) in 1901 proposed a theory to model the
spectrum of a black body (see section 9.2). This theory dictated that the energy
of oscillations of atoms or molecules cannot have just any value; they can only
possess a discrete amount of energy that is a multiple of the minimum value
related to the frequency of oscillation by the equation:

Discuss Plancks contribution


to the concept of quantised
energy.

E = hf
Plancks assumption suggests that the energy of any vibration could only be
a whole number multiple of hf:
E = nhf
where n = 1, 2, 3, E is energy (in joules), n is called a quantum number, h is
Plancks constant (with a value of 6.63 1034 Js) and f is the frequency in
hertz (s1).
This would mean that energy was not a continuous quantity as had been
believed, but rather that it was quantised into discrete packets. Interestingly, at the
time of this work Planck considered this idea more of a mathematical device to
obtain the right answer rather than an actual physical reality of nature.
In 1905, Einstein extended the concept of quantisation and proposed a new
theory of light. Einstein took Plancks suggestion that the vibrational energy of
atoms or molecules in a radiating object was quantised with energy. He argued
that the vibrational energy of the atoms or molecules could only change by
a multiple of hf and therefore proposed that light would be emitted in discrete
packets (quanta) also obeying the equation E = hf.
Gilbert Lewis (18751946) in 1926 named these discrete packets of light
photons (from photos, Greek meaning light).

ramp

stairs

Figure 12.3.1

This simple analogy shows


the difference between
continuous and quantised
energy states. On the ramp
the box can have any
amount of potential energy,
but the box on a staircase
can only have discrete
amounts of potential energy.

Worked example
Question
A photon has an energy of 2.8eV.
a Calculate its energy in joules.

b Calculate the frequency of the photon.

Solution
a You will recall from section 10.2 that

1eV = 1.602 1019joules

So: 2.8 1.602 1019 = 4.5 1019 joules

b Now rearranging E = hf and substituting in the energy and Plancks constant we obtain:
f=

E 4.5 1019
=
= 6.8 1014 Hz
h 6.63 1034

231

12

From Rutherford
to Bohr

Quantised Energy?

hy dont we notice that energy comes in discrete packets or quanta


in our everyday life? The value of Plancks constant is very small;
therefore, for large everyday objects such as a cricket ball or a car, the
energy appears to be a continuous quantity. At the molecular and atomic
level, however, the quantisation of energy becomes very apparent when
energy is absorbed or emitted.

Checkpoint 12.3
1
2

Outline the idea Planck proposed to explain black body radiation.


Explain the difference between the use of E = hf by Planck and Einstein.

12.4 Spectral analysis

spectroscopes

n a spectroscope, light from


a source passes through a slit
and enters the collimator tube.
The lens at the end of the
collimator makes the light parallel
before it illuminates the grating.
The diffraction grating is viewed
through a small telescope mounted
on a rotatable platform so that the
angle of the observed colour or
spectral line can be measured.

When light is passed through a prism or diffraction grating, the constituent colours
present in the light are revealed in the spectrum produced. The prism separates
the light using the property of refraction, whereas the diffraction grating uses the
property of interference. The traditional device used by scientists to examine
spectra is called a spectroscope. Modern instruments are called spectrometers.
There are two types of spectra: absorption spectra and emission spectra.
Absorption spectra can be produced by passing white light (a continuous
spectrum) through a cool gas. The atoms or molecules in the gas will absorb
certain specific wavelengths (colours) of light. The atom that absorbed the
light is now in an excited state and will spontaneously emit a photon of light,
usually in a different direction. Therefore the original beam of light will now
have certain wavelengths depleted, and these will appear as a series of dark
lines when observed through a spectroscope.
Emission spectra can be produced when a gas is excited. This can be
achieved by heating the gas or by passing an electrical current through a low
pressure gas. The light produced when viewed with a spectroscope will often
be made up of a series of bright coloured lines.
Hydrogen absorption spectrum

source

collimator

slit

lens

grating

telescope

eye

Figure 12.4.1

232

A spectroscope showing
the major components

Hydrogen emission spectrum

400 nm

Figure 12.4.2

700 nm

Absorption and emission


spectra of hydrogen

H alpha line
656 nm
Transition n = 3 to n = 2

quanta to
quarks
Ancient peoples have observed the emission colours of a range of materials.
A sprinkling of common salt (sodium chloride) into a flame produces an intense
golden flame, and during the smelting of copper ore the flames in the furnace are
often coloured an intense vivid green.
Spectra are a window into the hidden atomic structure. Each element has its
own unique spectral fingerprint. Chemists and students of chemistry still use
the flame test to identify the constituent elements present in chemical samples.
A continuous emission spectrum can be produced by a hot object, forming
a rainbow or continuum. For example, if you look at the light emitted from a
white-hot piece of metal through a prism or diffraction grating you will see a full
rainbow of colours.
Anders ngstrm (18141874) was the first to make detailed measurements
of the visible spectrum of hydrogen, and in 1885 Johann Balmer (18251898)
commenced a detailed study of the visible emission spectrum for
hydrogen, which is now referred to as the Balmer series.
Table 12.4.1 The Balmer series for hydrogen
Johannes Rydberg (18541919) generalised Balmers
Spectral line
Balmers generalised equation
equation:
1 1
1
=R 2 2
1
1
1
Name Colour
(nm)

n f ni
= R 2 2

nf ni
nf
ni
H

Red

where is the wavelength of the spectral line, R is Rydbergs


Blue-green
H
constant (1.097 107 m1), nf is the final state of the electron
Blue-violet
H
and ni is the initial state of the electron.
Violet
H
The emission spectrum in Figure 12.4.2 is the Balmer series
emission spectrum for hydrogen corresponding to the values of
nf = 2, and ni = 3, 4, 5, 6, 7 in the generalised Balmer equation.
The improvements in experimental techniques and technology for detecting
infra-red and ultraviolet radiation led to the discovery of additional spectral lines
and other spectral series. For example, Theodore Lyman (18741954) discovered
the first spectral line in the Lyman series in 1906 and took another 8 years to
detect the remaining lines. The generalised Balmer equation could be used
to calculate accurately the wavelength of the spectral lines for all the newly
discovered series.

Worked example
Question
Using the generalised form of Balmers equation, calculate the wavelength of the emitted
photon for a transition between n = 5 and n = 3.

Solution
Using the generalised form of Balmers equation to calculate

1
:

1 1
1
1 1
= R 2 2 = 1.097 107 2 2 = 7.80 105 m1
3 5

n f ni

Now take the inverse to determine the wavelength of the emitted photon:
= 1.28 106 m

656.3

486.1

434.0

410.2

Solve problems and analyse


information using:
1
1
1
= R 2 2

n f ni

The test
of time

ydbergs constant R was


initially determined empirically
from spectroscopy, and then
theoretically by Niels Bohr, who
used a mixture of classical and
quantum ideas. Later its value was
again calculated using more
fundamental constants in terms
of quantum mechanics:
R =

mee 4
8o2h 3c

233

12

From Rutherford
to Bohr

E=0
0.85
1.5
3.4

n=4

ionised atom
(continuous energy levels)

n=5

n=3
Paschen
series

n=2

excited
states

Balmer
series

Energy (eV)

10

Figure 12.4.3

Energy level diagram for the hydrogen


atom with the Lyman, Balmer and
Paschen series emission transitions and
associated spectra for the Balmer series

13.6

ground state

n=1

15

Lyman
series

Table 12.4.2 Spectral series associated with the hydrogen atom


Series

Spectral region

nf

ni

Lyman
Balmer
Paschen
Brackett
Pfund

Ultraviolet
Visible and UV
Infra-red
Infra-red
Infra-red

1
2
3
4
5

2,
3,
4,
5,
6,

Discovered
3,
4,
5,
6,
7,

4,
5,
6,
7,
8,

5
6
7
8
9

19061914
1885
1908
1922
1924

Worked example
Question
a Predict the values for nf and ni for the transition of the second longest wavelength
emission in the Lyman series.
b Calculate the wavelength for the second-longest wavelength in the Lyman series.

Solution
a The second-longest wavelength in the Lyman series will correspond to the secondlowest energy transition. In the Lyman series we know that nf is always 1. Thus
ni=2 will correspond to the lowest energy transition and ni = 3 will correspond to
the second-lowest energy transition. Therefore nf =1 and ni = 3.
1
b Using the generalised form of Balmers equation to calculate :

1 1
1
1
7 1
= R 2 2 = 1.097 10 2 2 = 9.75 106 m1
1 3

n n
f

Now take the inverse to determine the wavelength of the emitted photon for the
transition between ni = 3 and nf = 1.
= 1.03 107 m

234

quanta to
quarks
Worked example
Question
Using the information in the table, construct an energy level diagram for n=1 to n=5
for the hydrogen atom.
a On your diagram, draw and label the Lyman, Balmer and Paschen series.
b Identify the values of ni and nf for a transition of an electron between energy
levels that would absorb the highest frequency photon for the Paschen series,
based on the energy values provided.

Principal quantum number (n)

Energy (eV)

1
2
3
4
5

13.6
3.4
1.51
0.85
0.54

c Identify the values of ni and nf for the transition of an electron between energy
levels that would emit the highest energy photon for the Balmer series.

Solution
a Refer to Figure 12.4.3 and construct a similar diagram for the given values n=1
to n=5.
b From your diagram for part a, we see that for the Paschen series ni = 3 to nf = 5 is
the greatest possible jump in energy, therefore this transition would absorb the
highest energy photon.
c From your diagram for part a, we see that for the Balmer series ni = 5 to nf = 2 is
the greatest possible jump in energy, therefore this transition would emit the
highest energy photon.

Checkpoint 12.4
1
2
3
4
5
6

Recall the two types of spectra.


List the names of five spectral series.
Recall the basic process that produces the spectral series.
Explain the relationship between the energy of an emitted photon and quantum number n.
Calculate the wavelength of a photon emitted by an electron jumping from n=3 to n=2.
Outline the spectral series that corresponds to a transition from n=3 to n=2.

12.5 Bohrs model of the atom


Niels Bohr (18851962), a Danish physicist, spent a short time at the Cavendish
Laboratory, Cambridge, working with JJ Thomson before joining Rutherfords
team at the University of Manchester in 1912. Bohr focused his attention on the
planetary model and was equally impressed by both its successes and its obvious
limitations. Bohr set to work and built upon Rutherfords model by synthesising
a mixture of Plancks quantum theory and classical physics.

Bohrs postulates
In 1913 Bohr announced the revised planetary model of a hydrogen atom based
upon the quantisation of energy and angular momentum of the electron. His
new RutherfordBohr model included a set of three postulates to address the
identified limitations of Rutherfords model.

Figure 12.5.1

Niels Bohr

235

12

From Rutherford
to Bohr

Analyse the significance of the


hydrogen spectrum in the
development of Bohrs model
of the atom.
Define Bohrs postulates.

A Square peg
in a Round
Hole?

espite Bohr showing all the


signs of becoming a theorist
in an experimentalist laboratory,
Rutherford responded to
comments by saying, Bohrs
different. Hes a football player!

Bohrs postulates for his atomic model


1 Electrons exist in stable orbits. An electron can exist in any of several special
circular orbits with no emission of radiation. These orbits are called
stationary states.
2 Electrons absorb or emit specific quanta of energy when they transition
between stationary states (orbits). In contradiction to classical
electromagnetic theory, a sudden transition of an electron between two
stationary states will produce an emission or absorption of quantised
radiation (a photon), described by the PlanckEinstein relation.
3 Angular momentum of electrons is quantised. An electron in a stationary
state (orbit) has a quantised angular momentum that can take only values of
nh where n is the principal quantum number.
2

Structure of the RutherfordBohr model


The RutherfordBohr model has a small, positively charged nucleus that
contains most of the atoms mass. The electrons orbit the nucleus in classical
circular paths. The electrons do not radiate energy continuously as predicted by
Maxwells classical electromagnetic theory (i.e. accelerating charges will radiate
electromagnetic waves, which would result in the electron spiralling into the
nucleus), due to the quantisation conditions of energy associated with each
electron orbit. When an electron jumps to a higher or lower orbit it will absorb
or emit a quantum of energy in the form of a photon.
Despite being a hybrid theory that spanned both classical and the
quantum physics, the RutherfordBohr model proved to be extremely successful
in explaining many experimental observations.

Checkpoint 12.5
1
2

Explain the importance of stationary states to Bohrs model.


Summarise the role of Planck and Einsteins concept of quantisation to the development of Bohrs atomic model.

12.6 Bohrs explanation


of the Balmer series
Diagrammatic illustration
Lets consider the Balmer series emission spectra for a hydrogen atom. When
an excited electron in a stationary state (orbit) of ni>3 jumps down to the
stationary state (orbit) nf=2, a photon is emitted. The energy of this photon
will be equal to the energy difference E between these two stationary states:
Energy of emitted photon = E = Ei Ef
Process and present
diagrammatic information to
illustrate Bohrs explanation
of the Balmer series.

236

This photon will have a characteristic frequency and wavelength, which can
be calculated using the relationships E = hf and v = f (where v is c the speed of
light) rearranged in the forms:
E
c
f =
and =
h
f

quanta to
quarks
Balmer series

Figure 12.6.1 illustrates the RutherfordBohr atomic model


electron transitions for the Balmer spectral series.

Bohrs mathematical model


Bohrs model was not simply a diagrammatic representation of
an atom. He backed up his model with a mathematical framework
that was based upon both classical and quantum ideas. Most
importantly, the model was supported by experimental observations.

n=1

nucleus

n=2
n=3

Quantised energy of the stationary states


of the Bohr atom

n=4
n=5
n=6

The expressions for the quantisation of the angular momentum and


radii can be used to derive a quantised expression for energy:
1
E n = 2 E1
n
Figure 12.6.1 Diagrammatic representation of Bohrs explanation

Theoretically determining spectral lines

of the Balmer line series for the hydrogen spectrum

Now recall Bohrs second postulate concerning the emission or absorption of


a specific quantum of energy when an electron transitions between stationary
states (orbits). This obeys the PlanckEinstein relation hf = Ei Ef.
1
If we now substitute in E n = 2 E1 we obtain:
n
1
1
E between stationary states = hf = 2 E 1 2 E 1
ni
nf
If we take out the common factor of E1 and make f the subject, we obtain
the expression:
E1 1
1
f =
2 2
h nf ni
c
Using the relationship v = f where v is c the speed of light, we have f = .

Substituting this we obtain the expression:


c E1 1
1
=
2 2

h nf ni
Dividing through by c we obtain a theoretically derived expression in a form
that is equivalent to the generalised Balmer equation:
1 E1 1
1
=
2 2

hc nf ni

Describe how Bohrs postulates


led to the development of
a mathematical model to
account for the existence
of the hydrogen spectrum:
1
1
1
= R 2 2

n f ni

PRACTICAL
EXPERIENCES
Activity 12.1

Activity Manual, Page


105

The expression E1|hc corresponds to Rydbergs constant and, when evaluated,


agrees with the experimentally derived value. Bohrs theory had successfully
provided an explanation for some spectral phenomena and permitted the
calculation of Rydbergs constant. The theoretical derivation of Balmers equation
was a major accomplishment and provided strong support for the Rutherford
Bohr atomic model of the hydrogen atom.

237

12

From Rutherford
to Bohr

Worked example
Question
An electron makes a transition between the energy levels 0.85eV and 3.40eV in
a hydrogen atom.
a Determine to which spectral series this photon would belong.
b Calculate the wavelength of the emitted photon without using Balmers equation.
c Predict if the spectral line associated with this transition is visible to the
human eye.
d Evaluate the principal quantum numbers (n) to which each energy level
corresponds.
e Substitute your values of ni and nf into the generalised Balmer equation and
calculate the wavelength of the emitted photon. (This should be the same value
as in part b.)

Solution
a From Figure 12.4.3, the energy level 0.85 corresponds to n = 4 and energy level
and 3.40eV corresponds to n =2. The n = 2 tell us that this transition belongs to
the Balmer series.
b The difference between these two energies corresponds to the energy of the
emitted photon, E = 2.55eV. Convert this to the SI unit joules (remember
1eV = 1.6021019J).
E = 2.55 1.602 1019 = 4.091019 J

Now, using E = hf, we can calculate the frequency of the emitted photon with this
energy:
E 4.09 1019
f= =
= 6.17 1014 Hz
h 6.626 1034
c
Using the relationship v = f where v is c the speed of light, we have = .
f
c 3.0 108
7
= =
= 4.86 10 m
f 6.17 1014

c Light of this wavelength is in the visible range.


d From Figure 12.4.3, the energy level 0.85 corresponds to n=4 and the energy level
3.40eV corresponds to n=2.
1
e Using the generalised form of Balmers equation to calculate :

1 1
1
1 1
= R 2 2 = 1.097 107 2 2 = 2 05 106 m1
2 4

n n
f

Now take the inverse to determine the wavelength of the emitted photon for the
transition between ni = 4 and nf = 2.
= 4.86 107 m

Checkpoint 12.6
Explain the relationships between Bohrs atomic model and observed line spectra.

238

quanta to
quarks

12.7 Limitations of the


RutherfordBohr model
The RutherfordBohr model provided a simple visual model and accurately
predicted line spectra for the hydrogen atom. It synthesised classical mechanics
with the concept of quantisation, and addressed a number of experimental
observations. Despite its successes, it was nevertheless a hybrid model and many
experimental observations remained unresolved.

Discuss the limitations


of the Bohr model of the
hydrogen atom.

The spectra of larger atoms


The observed spectral series of larger atoms appeared to present patterns that
could be explained in a similar manner to the hydrogen atom. Despite the efforts
of Bohr and his colleagues, they were unable to develop an arrangement of
stationary states (orbits) that matched the experimental observations. Atoms
larger than hydrogen have more than one electron, and these electrons were
obviously interacting with each other in a complex manner. Bohrs simple
quantised planetary model explained only the hydrogen atom, the helium ion
(He+) and the lithium ion (Li2+), which all have single electrons orbiting the
nucleus.
a

PRACTICAL
EXPERIENCES
Activity 12.2

Activity Manual, Page


110

Sr

Ba

Ca
400

Figure 12.7.1

450

500

550
600
Wavelength (nm)

650

700

Emission spectra for (a) stronium, (b) barium and (c) calcium

The relative intensity of spectral lines


The spectra of the hydrogen atom and larger atoms all displayed three identifiable
types of spectral lines based upon their width. These were categorised as sharp (s)
lines, primary (p) lines and diffuse (d) lines. Also within these categories the
Bohrs postulates provided no
intensity of the individual lines varied.
explanation for these observations.

Figure 12.7.2

750

Emission spectrum for hydrogen showing the differences in relative widths

Measuring the
Magnetic field
of our Sun

stronomers use a spectroscope


and the Zeeman effect to
measure the strength of the
magnetic field associated with the
surface structures of the Sun.
The splitting of the lines in the
hydrogen spectrum is proportional
to the magnetic field strength.

239

12

From Rutherford
to Bohr

The existence of fine and hyperfine spectral lines

Universal units
for Length
and Time

he plaque on the Pioneer space


probe used the hyperfine
transition of hydrogen, which is the
most abundant element in the
universe, to define a base unit for
length and time.

The fine structure of closely spaced spectral lines that are 0.10.5 nm apart
is a result of an additional property of the electron called spin, which
was proposed in 1925 by Ralph Kronig (19041995). In 1881, using
interferometry, Albert Michelson (18521931) observed that some even
finer spectral lines (called hyperfine) existed; these were about 0.001nm
apart. The observation was not addressed until 1924, when Wolfgang Pauli
(19001958) proposed the existence of a small nuclear magnetic moment
Bohrs model did not
caused by a non-spherical atomic nucleus.
provide any explanation for these observed phenomena.
a

0.1 nm
0.1 nm

0.001 nm

Figure
12.7.3
0.001
nm
Normal spectral line:

With no magnetic field


appliedsingle spectral
line observed.

Normal Zeeman effect:

Magnetic field applied


triple spectral line observed.
Central line polarised parallel
to applied magnetic field.
Other two lines polarised
perpendicular to applied field.

Anomalous Zeeman effect:


When magnetic field applied
more spectral lines observed.

Figure 12.7.4 A single spectral line splits into more


lines when the source is subjected
to a strong magnetic field.

A comparison of (a) fine structure and (b) hyperfine structure

The Zeeman effect


In 1862, Michael Faraday placed a sodium flame between the poles of
a magnet to see if the bright sharp spectral lines were influenced by the
magnetic field, but he observed no change. Some 30 years later (in the
1890s) Pieter Zeeman (18651943) repeated the experiment using both
a more advanced spectroscope and a stronger magnetic field. Zeeman found
that the spectral lines were indeed influenced and he observed that the
previously single lines had each split into three. Classical physics and Bohrs
model could provide an explanation for these triplets. This was called the
Zeeman effect. In 1897 Zeeman used an even stronger magnetic field and
found that the triplet lines were also split. The additional splitting was also
associated with the yet to be discovered spin property of electrons.
Bohrs atomic model provided some explanation for the normal
Zeeman effect, but none for the anomalous Zeeman effect.

Checkpoint 12.7
Compare the origins and observed separations of the splitting in the normal Zeeman effect and the hyperfine spectral lines.

240

PRACTICAL EXPERIENCES

quanta to
quarks

CHAPTER 12

This is a starting point to get you thinking about the mandatory practical
experiences outlined in the syllabus. For detailed instructions and advice, use
in2 Physics @ HSC Activity Manual.

Activity 12.1: Hydrogen and the atom


Connect a high-voltage source to a low-pressure hydrogen discharge tube, and
observe the emitted light through a spectroscope. Identify the visible part of the
hydrogen spectrum.
Equipment: low-pressure hydrogen
gas discharge tube, gas discharge tube
high-voltage power supply,
spectroscope.
Discussion questions
1 List the wavelengths of the
emission lines in the visible part
of the hydrogen spectrum.
2 Identify the energy levels of the
electron transition that creates
these wavelengths.
3 Outline Bohrs explanation of the
Balmer series of emission lines.

Figure 12.8.1

Perform a first-hand
investigation to observe the
visible components of the
hydrogen spectrum.
Process and present
diagrammatic information to
illustrate Bohrs explanation of
the Balmer series.
Solve problems and analyse
information using:
1
1
1
= R 2 2

n f ni

Experiment set-up and the


visible emission spectral
lines of hydrogen

Activity 12.2: Problems with the RutherfordBohr


model
In this activity you will look at the difficulties with the RutherfordBohr model
in explaining certain observations.
Discussion questions
1 Identify the ways in which this model is an improvement over the
previously accepted model of the atom.
2 Explain the limitations of this model.

Analyse secondary information


to identify the difficulties with
the RutherfordBohr model,
including its inability to
completely explain:
the spectra of larger atoms
the relative intensity of
spectral lines
the existence of hyperfine
spectral lines
the Zeeman effect.

241

12

242

From Rutherford
to Bohr

Chapter summary

Atomic theory had its origins in Greece more than


2000 years ago with ideas put forward by Democritus
and Aristotle.
JJ Thomson in 1904 proposed the plum pudding
model comprising a positive sphere in which the
electrons were distributed like plums in a pudding.
Ernest Rutherford in 1911 used the results of Geiger
and Marsdens scattering experiments to propose a
Rutherfords planetary model. Rutherfords model
comprised a massive, positively charged nucleus with
a set of orbiting electrons (like planets orbiting the Sun),
inferring that the atom is mostly empty space.
Despite the success of Rutherfords planetary atomic
model in explaining the scattering of alpha particles,
the model failed to explain:
what the nucleus is made of
how the orbits of the electrons are arranged around
the nucleus
what keeps the negatively charged electrons from
losing energy and spiralling into the positive
nucleus.
Planck in 1901 proposed a theory to model the
spectrum of a black body and introduced the concept
of quantisation E = nhf where n = 1, 2, 3, 4
A spectroscope uses a prism or diffraction grating to
reveal the constituent colours present in the light.
There are two types of spectra:
emission spectra
absorption spectra
Spectra are a window into the hidden atomic structure.
Each element has its own unique spectral fingerprint.

Rydbergs generalised Balmer equation allows you to


calculate the wavelength of the photon emitted or
absorbed for any transition of an electron between
stationary states:
1
1
1
= R
n 2 n 2

f
i
where is the wavelength of the spectral line
R is Rydbergs constant 1.097 107 m1
nf is the final state
ni is the initial state.
Hydrogen has several spectral line series including the
Lyman, Balmer, Paschen, Brackett and Pfund series.
Bohr in 1913 described the revised planetary model of
a hydrogen atom based upon the quantisation of energy
and angular momentum of the electron.
Bohrs three postulates for his atomic model:
1 Electrons exist in stable orbits called stationary
states.
2 Electrons absorb or emit specific quanta of energy
when they transition between stationary states
(orbits), described by the PlanckEinstein relation
hf = Ei Ef.
3 Angular momentum of electrons is quantised.
The RutherfordBohr model, despite its successes, was
nevertheless a hybrid model and many experimental
observations still remained unresolved including:
the spectra of larger atoms
the relative intensity of spectral lines
the existence of hyperfine spectral lines
aspects of the Zeeman effect.

Review questions

quanta to
quarks

Physically speaking
The following table is all jumbled up. Copy the table into your workbook and correctly arrange the information. You may also
add additional features associated with each atomic model. For each atomic model, draw and label a pictorial representation.

Scientist and date

Features of the atomic model

Democritus

Planetary orbits of the


electrons
Three postulates

c 400

b ce

Aristotle
c 300

b ce

Dalton
1801

Thomson

Draw a pictorial representation of the model

A positive sphere with


electrons embedded in it

Planetary orbits of the


electrons
Small positively charged
nucleus

1904

Limit to how small you could


divide matter; the smallest is
called an atom

Rutherford

Electrons act like waves

1911

Bohr
1913

de Broglie
1924

There are four elements:


earth, air, fire and water

Elements only contain one


type of atom
Different elements
contain different atoms
Compounds contain more
than one type of atom

243

12

From Rutherford
to Bohr

Reviewing

Solving Problems

1 Explain why flame tests of unknown samples are so

13 A photon has an energy of 6eV. Calculate:


a its energy in joules
b the frequency of the photon
c the wavelength of the photon.

useful to scientists.

2 Compare Max Plancks and Albert Einsteins ideas


relating to quanta.

3 Outline Rutherfords model of the atom.


4 Explain how emission line spectra are formed.
5 Identify the major events that led Niels Bohr to
propose his model of the atom.

6 Define Bohrs postulates associated with his 1913


model of the atom. Describe how these postulates led
to the development of a mathematical model to
account for the existence of the hydrogen spectrum.
Describe how Bohrs postulates led to the
development of a mathematical model to
account for the existence of the hydrogen
spectrum:
1
1
1
= R 2 2

n f ni

Recall the relationship between energy levels in the


Bohr model of an atom and the observed Balmer
series spectra.

Discuss the limitations of the Bohr model of the


hydrogen atom.

When excited atoms were placed in a strong magnetic


field some previously observed single spectral lines
split. Explain why the RutherfordBohr model could
not completely explain the observed phenomena.

10 Marsden and Geiger in 1909 fired alpha particles at


thin platinum and gold foils.
a Recall what the then current model of the atom
proposed by JJ Thomson theoretically predicted.
b Recall the name of the scientist they were working
with during these experiments.
c Outline their experimental findings.
d Discuss the implications these experiments had
for the development of a new model of the atom.
Discuss the structure of the Rutherford model
of the atom, the existence of the nucleus and
electron orbits.

11 Recall why the RutherfordBohr model could not


explain spectra of atoms larger than hydrogen.

12 Compare the Zeeman effect and hyperfine spectral


lines.

244

14 Using the formula En =


E1 and given that the


n2
energy of the first stationary state (orbit) in the
hydrogen atom is 13.6eV, calculate:
a the energy of the fourth stationary state in the
hydrogen atom
b the energy difference between the fourth and first
stationary states
c the frequency of a photon emitted by an electron
transitioning from stationary state n = 4 to n = 1
d the wavelength of a photon for this transition
e the spectral series to which this photon belongs.

15 An electron makes a transition between the 3.40eV


and 13.6eV energy levels of a hydrogen atom.
a Calculate the wavelength of the emitted photon
without using Balmers equation.
b Predict if the spectral line associated with this
transition is visible to the human eye.
c Evaluate the principal quantum numbers (n) of
each energy level.
d Substitute your values of ni and nf into the
generalised Balmer equation and calculate the
wavelength of the emitted photon. (This should
be the same value as part a.)
e Identify the spectral series to which this photon
would belong.

16 Using the Bohr model, construct diagrams of the


following electron transitions and describe the role of
the photon in these transitions:
a an electron being excited from the ground state
(n=1) to the fourth excited state (n=5)
b an excited electron dropping from the n=6
energy level to the n=2 energy level
c an excited electron dropping from the n=4 to
n=3 and then back to the ground state (n=1)

17 Using the information in the following table, construct


an energy level diagram for n=1 to n=5 for the
hydrogen atom.

Principal quantum number (n)

Energy (eV)

1
2
3
4
5

13.6
3.4
1.51
0.85
0.54

quanta to
quarks
a Identify the electron transition between the energy
levels in your diagram that would absorb the
highest frequency photon.
b Identify which transition of an electron between
energy levels would emit the highest energy
photon.
c Demonstrate on your energy level diagram how
the Balmer series of spectral emission lines is
produced.
d Identify the transition between energy levels in
the Balmer series that would produce the longest
wavelength photon.
Solve problems and analyse information using:
1
1
1
= R 2 2

n f ni

18 Rydberg generalised Balmers equation for the


hydrogen atom:

1
1
1
= R 2 + 2

nf ni

19 A photon is emitted from a hydrogen atom with a


wavelength of 410.12nm. The electron associated
with the transition that produced the photon is now in
energy level nf=2.
a Using Balmers generalised equation, calculate
the value of ni.
b Construct a diagram showing the transition and
identify all relevant features.
c Identify the series associated with this transition.

20 The Lyman series for a hydrogen atom has the


following energy levels: E1=13.6eV, E2=3.40eV,
E3=1.51eV, E4=0.85eV, E5=0.54eV,
E6=0.38eV.
a Construct a diagram using the Bohr model to
represent the series.
b Construct an energy level diagram for the Lyman
series.
c Calculate the energy of the photon emitted in the
transition from n=5 in the Lyman series.
d Calculate the wavelength for this emission.

Re

iew

Q uesti o

a Recall what each term in the equation represents.


b Calculate the wavelength of the emitted photon for
a transition between n=6 and n=1.
c Predict the values for nf and ni for the transition
of the second-longest wavelength in the Balmer
series.
d Calculate the wavelength for the second-longest
wavelength in the Balmer series.

245

12

From Rutherford
to Bohr

PHYSICS FOCUS
Quanta to Quarks Timeline
H1. Evaluates how major advances in
scientific understanding and technology
have changed the direction or nature of
scientific thinking

H3. Assesses the impact of particular


advances in physics on the development
of technologies

Physics is not just about learning facts. Physics is a


human journey of exploration seeking to explain the
universe in which we live. Understanding the history
of events and people will allow you to stand upon the
shoulders of giants and continue this quest. Often the
stories told in modern textbooks skip over the years
of frustration and confusion that scientists faced and
present only the milestones and important discoveries.
When you study the history of scientific
advancement, it can become quite confusing if you
do not see the situation in the context of the period
of history.
For example, many of the experiments carried out
from the late 1800s to the modern particle
accelerators relied on vacuum pump technology. How
did this technology frustrate the pioneers who worked
with low-pressure gas experiments? Consider how the
ability for the scientist to produce a strong magnetic
field may have influenced the outcome of an
experiment. For example in 1862, Michael Faraday
placed a sodium flame between the poles of a magnet
to see if the bright sharp spectral lines were
influenced by the magnetic field; Faraday observed
no change. Some 30years later in 1896, Pieter
Zeeman repeated the experiment using both a more
advanced spectroscope and a stronger magnetic field,
and found that the spectral lines were indeed
influenced. Zeeman also observed that some
previously single lines had each split into three.

246

Figure 12.8.2

JJ Thomson in his Cavendish laboratory

Maybe you are now thinking that the interplay


between technology, people, events and ideas might
be quite interesting.
Construct a timeline that includes people,
experiments, advances in technology, models and
theories for the period from 1885 and Balmers
observations of the hydrogen spectrum through to
1913 when Niels Bohr proposed his atomic model.
You can include pictures of people, models and
experiments. Investigate and explore the relationships
between events. You might construct your own or, as
a class, build a huge timeline and display it in
your laboratory.
Some technologies you might research:
Advances in vacuum pump technology
Advances in spectroscope technology
Sources of electricity

Extension
Some additional technologies you might research:
Methods of detecting the non-visible
electromagnetic spectrum, including infra-red,
ultraviolet light, X-rays and gamma rays
Types of magnets available
Electric and electronic circuitry
As you progress further through this module
extend and include more details on your timeline.

13

Birth of quantum
mechanics
A new paradigm
Niels Bohrs hybrid atomic model, which combined classical and
quantum concepts, held centre stage from 1913 through to the early
1920s. From a historical perspective we must remember that World
War I, 19141918, disrupted research laboratories throughout
Europe. The 1920s saw the move away from classical ideas and
culminated at the 1927 Solvay Conference in Copenhagen where
modern day quantum mechanics was born. In the years leading up
to this conference Louis de Broglie proposed that all matter has wave
properties, Erwin Schrodinger developed a mathematical model for
describing wave mechanics, Heisenberg revealed the statistical
nature of quantum theory and formulated his most famous
discoverythe Heisenberg uncertainty principle
and Pauli developed his exclusion principle.

13.1 The birth


Newtonian classical physics was under attack throughout the 1920s and the
outcome, which was based upon theoretical and experimental evidence, gave
birth to quantum mechanics. The major debates came to a head at the 1927
Solvay Conference in Copenhagen where the old deterministic view of matter
was cast aside in favour of a world that was ruled by probabilities. This outcome,
however, was certainly not unanimously accepted and most notably Bohr and
Einstein continued to debate the interpretations for more than 20 years.

Checkpoint 13.1
Outline how quantum mechanics was started.

quantum mechanics, de Broglie,


Schrodinger, wave mechanics,
diffraction, quantum numbers,
wave function, Heisenberg, uncertainty
principle, Pauli, exclusion principle

A Great Loss

arry (Henry) Moseley was one of


the brilliant young scientists who
worked with Rutherford. In about
1910, using X-ray diffraction of
crystals, he discovered a systematic
relationship between wavelength of
characteristic X-rays and atomic
number, which is now known as
Moseleys law. When World War I
broke out, he enlisted and joined the
British Royal Engineers. He was killed
in action by a sniper shot in 1915 at
Gallipoli. The loss of Moseley and
other scientists during this war
prompted British and other world
governments to implement policy to
not allow scientists to enlist for
combat postings.

247

13

Birth of quantum
mechanics

Figure 13.1.1

Participants at the 1927 Solvay conference


Back row: A Piccard, E Henriot, P Ehrenfest, E Herzen, T De Donder, ESchrdinger, JE Verschaffelt,
W Pauli, W Heisenberg, RH Fowler, LBrillouin
Middle row: P Debye, M Knudsen, WL Bragg, HA Kramers, P Dirac, ACompton, L de Broglie, M Born, N Bohr
Front row: I Langmuir, M Planck, M Curie, HA Lorentz, A Einstein, P Langevin, CE Guye, CTR Wilson,
OW Richardson

13.2 Louis de Broglies proposal


By 1915, William Henry Bragg (18621942) had provided convincing evidence
that X-rays have particle properties; his son William Lawrence Bragg (1890
1971), an Australian, had developed an equation based upon the wave nature
of X-rays that enabled a detailed analysis of X-ray diffraction patterns.
In the early 1920s, Prince Louis de Broglie (pronounced de broy), with the
knowledge that X-rays and light possessed both wave and particle properties,
began to consider the waveparticle duality as a natural symmetry.

Matter waves
In 1923, Louis de Broglie began with a supposition that was based upon the
PlanckEinstein equation linking energy quanta to frequency. He equated
Einsteins special relativity energymass relationship E=mc2 with the Planck
Einstein equation E=hf:
mc2 = hf
He rearranged it to derive an expression for the momentum of a photon:
mc =

hf
c

Now mc is mass times velocity, hence the photons momentum is:


p =

hf
c

c
c
Using the relationship =
in the form f = and substituting,
f

he obtained:
h
p=

248

quanta to
quarks
He had derived an expression for the momentum of a photon in terms of its
wavelength.
In 1924, de Broglie proposed the concept of matter waves by applying
his belief in symmetry to propose that all particles of energy should also possess
an associated wavelength. At this time, particles of matter such as electrons,
atoms and alpha particles were known to possess the properties of mass and
charge, but there was no experimental evidence to indicate that these particles
exhibited wave phenomena.
De Broglies proposed matter waves could be calculated mathematically by
using what is now known as the de Broglie equation:
=

h
h
or =
p
mv

where is the wavelength, h is Plancks constant, m is the mass of the particle,


v is the velocity of the particle and p is the momentum of the particle.
(Remember p = mv.)
Louis de Broglie presented his hypothesis in his doctorial dissertation.
Unfortunately the combination of the involvement of de Broglie and other
French scientists in unfriendly debates with Niels Bohr and his supporters and
the experimentally unsupported proposition that particles such as electrons
possessed wave properties meant that the majority of scientific community did
not take de Broglies ideas seriously.

Figure 13.2.1

Louis de Broglie

Solve problems and analyse


information using:
h
=
mv

Worked example
Question
a Calculate the wavelength of an electron moving with a velocity of 6.0 105 ms1.
b Compare this calculated wavelength with the wavelength of visible light
(400650nm).

Solution

h
and substituting in the following values:
mv
h= 6.63 1034 Js

a Using de Broglies equation =



me = 9.11 1031 kg

v = 6.0 105 ms1

we obtain

6.63 1034 J s
9.11 10

31

kg 6.0 10 m s

= 1.21 109 m

Therefore the wavelength of the de Broglie matter wave is 1.21109 m.

b The visible spectrum ranges from violet light, with the shortest wavelength
(~400nm), to red with a wavelength of 650nm. For the purpose of this comparison
we will use the average wavelength of these extremes 525nm, which corresponds to
a yellow/green colour.

First we need to convert 525nm into metres (5.25 107 m). Now, comparing the
ratio of the wavelength of the electron (1.21 109 m) with the average wavelength
of visible light (5.25 107 m), we discover that the wavelength for an electron
travelling at 6.0 105 ms1 is approximately 430 times smaller than the
wavelength of visible light.
249

13

Birth of quantum
mechanics

Always keep your eye on the physics

n important point to consider when you are calculating the de Broglie


wavelength for macroscopic objects such as cars and balls is that these are
not discrete fundamental particles. Rather they are a bulk collection of such.
Therefore, if you calculate the de Broglie wavelength for an object at near rest
(very close to zero velocity) you would obtain a large wavelength, which of course
is not what we observe. This is because you have not considered that the atoms
and molecules making up the object are actually vibrating at high velocities due
to their thermal energy. Also it is important to remember that, even at absolute
zero (273C), all atomic particles including atoms and electrons will still have
some motion and therefore cannot have zero velocity.

Describe the impact of de


Broglies proposal that any kind
of particle has both wave and
particle properties.

Initially Einstein was the only high-profile physicist to support de Broglies


hypothesis. Erwin Schrodinger, a loner, who was then based at the University of
Zurich, was intrigued and took up the idea proposed by de Broglie. He became the
architect of wave mechanics, a fundamental component of quantum mechanics.

Checkpoint 13.2
1
2
3

Outline the significance of the work of the Braggs in Louis de Broglie developing his theorical research.
Determine the wavelength of an electron moving at 3.0 105ms1.
Calculate the momentum of a proton that has a wavelength of 1.01109m.

13.3 Diffraction
Define diffraction and identify
that interference occurs
between waves that have been
diffracted.

250

A defining property of waves is their ability to flare or seemingly bend


around corners. Waves in water, sound waves and light can all exhibit this
property called diffraction. Lets first look at water waves. If you generate a set
of plane waves in a shallow trough of water using a ruler, you will see the waves
propagate down the length of the trough. Now, if you place an object such as an
edge, a block, a gap or a narrow opening in the path of the waves, you will
observe a strange behaviour. Instead of casting a shadow, each wavefront bends
around the object and enters the shadow region.
Lets now observe the behaviour of light as it passes an edge, through a
narrow slit and a small aperture (Figure 13.3.1). Again you can see that the light
does not cast a sharp shadow, instead it forms a series of bright and dark lines.
These patterns are known as diffraction patterns.

quanta to
quarks
a

Figure 13.3.1

Light passing (a) around a razor blade, (b) through a slit and (c) through a small aperture

You cant
escape them

ometimes when you are sitting back relaxing


or looking upwards into a clear blue sky, you
might occasionally see tiny fuzzy, out-of-focus
hairs or blobs. These are called floaters and
they are, literally, that. They are little bits of jelly
broken off the vitreous humour (the clear gel that
fills the space between the lens and the retina
of the eye) floating just in front of your retina.
The fuzziness is caused by the diffraction
of light around the edges of these floaters.

Try This!
Observing a diffraction pattern
Simply take two pens or pencils (you can even use
your fingers). Place them very close together, almost
touching, to form a narrow slit. Now hold them up in
front of one eye (about 34cm in front works well) now
close the other eye. Look through the slit at a bright light
source such a fluorescent light. Vary the width of the slit
and observe carefully! What do you see?

Checkpoint 13.3
1
2

Define diffraction.
Sketch a diffraction pattern that would result as light moves around a hair.

13.4 Confirming de Broglies


hypothesis
Clinton Davisson (18811958) and Lester Germer (18961971), using
electron scattering, reported the experimental discovery of electron waves in 1927.
Davisson and Germers discovery was preceded by an accident in the laboratory in
which an explosion shattered the evacuated glass vessel holding the nickel target.
The introduction of air oxidised the surface of the nickel target, which then had
to be degassed by heating it to a high temperature. This process allowed a number
of large nickel crystals to form. Experiments with this target produced a new set
of data in which sharply defined currents of electrons were present. Davisson and
Germer quickly realised that these new results were a result of the recrystallisation,
and they constructed new nickel targets of single crystals.

Describe the confirmation


of de Broglies proposal by
Davisson and Germer.

251

13

Birth of quantum
mechanics

In the family

eorge P Thomson, the


co-discoverer of the wave
properties of electrons, was the
son of JJ Thomson the discoverer
of the electron.

Up until this time Davisson and Germer had been unaware of de Broglies
hypothesis, but during a meeting at Oxford in 1926 Davisson heard of
de Broglies proposed matter waves. He immediately realised that the new results
he and Germer had observed were similar to those of X-ray diffraction patterns.
With this insight, Davisson and Germer quickly identified that the scattered
electrons were indeed producing diffraction patterns.
At the same time, George Thomson (18921975) conducted a different
experiment in which he fired a beam of electrons through a thin gold foil and
observed diffraction rings. Therefore, in 1927, two different research teams using
different experimental techniques had verified Louis de Broglies hypothesis
concerning the wave nature of electrons.
a

b I

V = 54 V

electron
gun
50
0

power supply

15 30 45 60 75 90

electron
detector

electron beam (in vacuum)

incident waves
in phase

50

nickel crystal

Figure 13.4.1

scattered waves
in phase

GP Thomson

crystal
surface

Figure 13.4.2

Davisson and Germers apparatus and results. With the single crystal nickel
target, Davisson and Germer detected several peaks in scattered electrons.

Checkpoint 13.4
1
2

Outline how matter waves were first observed.


Discuss what other evidence has been observed to support Louis de Broglies hypothesis.

13.5 Electron orbits revisited


PRACTICAL
EXPERIENCES
Activity 13.1

Activity Manual, Page


112

252

Bohrs first postulate stated that an electron can exist in any of several circular
orbits with no emission of radiation. However, he provided no supporting
reasoning to explain why the orbiting electron would not radiate away its energy,
as predicted by Maxwells classical electromagnetic theory, and simply spiral into
the nucleus.
When de Broglie formulated his concept of matter waves, he envisioned that
electron orbits were standing waves. Each orbit (stationary state) was occupied
by an integral number of wavelengths that fitted around the circumference of
the orbit.
The circumference of a circular orbit is 2R and hence an integral number
of wavelengths:
n = 2R

quanta to
quarks
where n is 1, 2, 3, 4, 5 , is the wavelength and R is the radius of the orbit.
To visualise these standing waves we first imagine a standing wave on a
slinky spring or rope, obeying the condition n where n can have the values n =
1, 2, 3 etc. When you shake a rope or a slinky spring you can produce standing
wave patterns that contain whole or integral numbers of wavelengths.
Now we can take these standing waves and wrap them end-to-end to form a
circular standing wave that corresponds to the circular Bohr orbit. These values
of n are called the principal quantum numbers and they tell you the number of
wavelengths fitting into the Bohr orbit.
Remember that the standing waves in Figure 13.5.1 represent the matter
wave nature of the electron. Often students imagine the electrons travelling in
wavy orbital paths around the nucleusthis is not the correct interpretation.

Explain the stability of the


electron orbits in the Bohr atom
using de Broglies hypothesis.

n=1

Try This!
Standing waves
Using a slinky spring or a rope, generate standing waves by having one
person hold one end firmly while the other person shakes their end.
Once you get a stationary wave you will notice that if you stop shaking,
the standing wave is quite stable and continues to vibrate. On a rope or
slinky you can generate standing waves other than those shown in
Figure 13.5.1. Why are these other standing wave patterns not suitable
for forming circular standing waves? (Hint: Consider interference.)

n=2

Figure 13.5.1

n=3

n=4

Comparison of standing waves


wrapped in a circle and on a rope. For
a standing wave to be produced in a
circle, a whole number of matter
waves must fit into the circumference.

Checkpoint 13.5
1
2

Outline how de Broglies matter waves help to explain the stable orbits of electrons.
Calculate the radius of the smallest orbit of an electron with wavelength 1.21 109m.

13.6 Further developments of atomic


theory 19241930
Quantum mechanics, the modern version of quantum theory, was developed
between 1925 and 1930. During this period, physicists moved from the hybrid
mixture of classical and quantum concepts to a fully mathematical model, to
describe the behaviour of matter. The strangeness of waveparticle duality and
the many associated paradoxes were sidelined as a new physics was born.

Erwin Schrodinger (18871961)


Schrodingers interests were wide and spanned most of modern physics including
statistical mechanics, X-ray diffraction, relativity and field theory. After World
War I, Schrodinger moved between several academic posts and in 1921 he settled
for 6 years at the University of Zurich. Here Schrodinger, encouraged byEinstein,
253

13

Birth of quantum
mechanics

developed wave mechanics by synthesising the works of de Broglie, Planck,


Einstein, ideas from Hamilton and the fundamental differential equations from
optics. In 1926 Schrodinger published the equation known by
all physicists as Schrodingers equation:

h2 d2
+ V ( x ) = E
2m dx 2

When solved, the equation provides a wave function. The wave function
contains all the measurable information about a particle. If you square this
function you obtain a probability density, which allows you to predict the
likelihood of finding a particle. This very quickly successfully addressed a large
variety of atomic and molecular problems. The price paid for this new way of
looking at matter was that the classical and deterministic view of the atomic
world was totally replaced by a system in which you could only calculate the
probability of finding a particle in a certain place and time.

Werner Heisenberg (19011976)


Figure 13.6.1

Erwin Schrodinger

A famous cat
and inspiring a
new career path
to DNA

veryone has heard of


Schrodingers catthe one that
is both alive and deadwell that is
until you open the box and see. But
a little less known fact is that in
1944 Schrodinger published a little
book entitled What is Life?, which
looked at the new field of molecular
biology. Francis Crick the codiscoverer of the DNA double helix
said that this book influenced him
to change his career from physics to
molecular biology.

254

Heisenberg met Bohr in 1922, he was just 20 years old and working toward his
PhD. At the end of a lecture Heisenberg raised an objection to what Bohr had
been discussing. Bohr was so impressed that he invited Heisenberg on a walk and
they discussed the difficulties of quantum theory for more than 3 hours. Bohr
liked physical models of atoms, but Heisenberg thought it was nonsense to talk
about electron orbits when it was obvious that you could not observe the
electrons or their orbital paths.
Heisenberg went on to discover the first complete version of quantum
mechanics. The new theory that was developed in parallel with Schrodinger had
taken a completely different path. Heisenbergs theory had abandoned any
attempt to create a visualisation of the atom, ignored the ideas of waves and
particles, and in itself was purely mathematical.
Heisenberg took de Broglies proposal and Schrodingers wave equation and
was able to ascertain that there was a limit to how precisely you would measure
pairs of quantities. The pair he identified was that of position and momentum.
In 1927 Heisenberg announced his uncertainty principle, which
is expressed mathematically as:
h
h
x p
and E t
2
2
where x is the uncertainty in the position of a particle, p is the uncertainty in
the momentum of a particle and h is Plancks constant.
You can gain some insight into understanding the uncertainty principle by
considering the following example. If you want to know the position of a
particle, you will need to use a high-energy photon that has a small wavelength
as a high-resolution probe to locate your particle. Now consider that the photon
bounces nicely off the particle and you detect the photon and calculate the
position of your particle. You have located the particle very precisely, but the
particle is now recoiling and you will have very little information about its
momentum.
An even more bizarre implication of Heisenbergs uncertainty principle
(pointed out by Einstein) is that particles (i.e. energy) can appear out of the
nothingness of space for a very brief period of time, and then disappear.

quanta to
quarks
These particles are called virtual particles and we will discuss some aspects of them
in Chapter 15 The particle zoo.
In summary, Heisenbergs contributions to atomic theory include:
1 the development of his matrix mechanics, which, like Schrodingers wave
equation, allowed the nature of the atom and especially spectra to be explained
2 the uncertainty principle, which set limits on the precision of measurements.

Wolfgang Pauli (19001958)


Pauli and Heisenberg were great friends and studied as research students under
Arnold Sommerfield at the University of Munich. Paulis introduction to
quantum theory was as a student listening to Sommerfields lectures. Sommerfield
had extended the RutherfordBohr model to include elliptical orbitals, and the
BohrSommerfield theory attempted to describe the hydrogen molecule.
During the decade after 1913, physicists focused on the role of quantum
numbers. A major outstanding question was How many quantum numbers were
required to account for the observed chemical and physical behaviour of atoms?
Pauli responded by presenting an ad hoc solution to address this puzzle.
He found that by assigning four quantum numbers to each electron in an atom,
combined with a set of rules, he was able to produce a system that explained a
number of outstanding issues including the structure of the periodic table.
Several prominent scientists including Heisenberg, Bohr, Compton and Pauli
had considered that a fourth quantum number related to electron spin might be
required to describe the behaviour of electrons in an atom. The more experienced
physicists hesitated, but two Dutch graduate students George Uhlenbeck and
Samuel Gouldsmit published and described the fourth quantum number as spin
on the basis of the ideas presented in a paper Pauli had published on his
exclusion principle.
In summary, Pauli made several important contributions to atomic
theory:
1 He proposed that each electron in an atom would be described by four
quantum numbers.
2 He proposed, through his exclusion principle, that no two electrons in an
atom could have a set of four identical quantum numbers.
3 His exclusion principle and rules provided a system to explain the
arrangement and number of electrons in each atomic orbital, thus providing
an explanation for the structure of the period table.
4 He proposed that a neutrino was also emitted in beta decay, which provided
an explanation for the spectrum of beta-particle energies observed.

Checkpoint 13.6
1
2
3

State what Schrodingers equation allows you to find.


Outline Heisenbergs contributions to quantum mechanics.
Recall how Paulis work explained outstanding problems in the theories.

Figure 13.6.2

Werner Heisenberg

Figure 13.6.3

Wolfgang Pauli

Gather, process, analyse and


present information and use
available evidence to assess
the contributions made by
Heisenberg and Pauli to the
development of atomic theory.

255

13

Birth of quantum
mechanics

PRACTICAL EXPERIENCES
CHAPTER 13
This is a starting point to get you thinking about the mandatory practical
experiences outlined in the syllabus. For detailed instructions and advice, use
in2 Physics @ HSC Activity Manual.

Activity 13.1: Atomic theory


Gather, process, analyse and
present information and use
available evidence to assess
the contributions made by
Heisenberg and Pauli to the
development of atomic theory.

Research and present a 5 minute speech assessing the contributions of Heisenberg


and Pauli to the development of atomic theory.
Discussion questions
1 Outline the contributions of Heisenberg and Pauli to atomic theory.
2 Assess how these contributions have affected our understanding of
the atom.

Chapter summary

256

Louis de Broglie proposed that all matter has wave


properties. These are called matter waves.
The wavelength of a de Broglie matter wave can be
determined by using de Broglies equation:
h
h
=
or =
p
mv
where is the wavelength
h is Plancks constant
m is the mass of the particle
v is the velocity of the particle
p is the momentum of the particle.
Diffraction is a property of waves and it is characterised
by the waves bending around corners or spreading out
after passing through a narrow slit.
Diffraction patterns can only be produced by waves.
In 1927 two different research teams using different
experimental techniques had verified Louis de Broglies
hypothesis concerning the wave nature of electrons:
Clinton Davisson and Lester Germer, using electron
scattering
George Thomson firing a beam of electrons through
a thin gold foil and observing diffraction rings.

Louis de Broglie explained the stability of the electron


orbits in the Bohr atom by stating that each orbit
(stationary state) was occupied by an integral number
of wavelengths that fitted around the circumference of
the orbit.
Heisenbergs contributions to atomic theory include:
1 the development of his matrix mechanics
2 the uncertainty principle, which set limits on the
precision of measurements.
Paulis contributions to atomic theory include:
1 proposing that each electron in an atom would be
described by four quantum numbers
2 proposing through his exclusion principle that no
two electrons in an atom could have a set of four
identical quantum numbers
3 his exclusion principle and rules provided an
explanation for the structure of the periodic table
4 proposing that a neutrino was also emitted in beta
decay, which provided an explanation for the
spectrum of beta-particle energies observed.

Review questions

quanta to
quarks

Physically speaking
Copy and complete the following text.
______________ atomic model published in ______________ was a mix of
______________ and ______________ concepts. In ______________ Louis de Broglie
proposed that particles had both ______________ and ______________ properties.
______________ developed the idea of ______________ waves, which, when applied
to the situation of an electron ______________ in Bohrs atomic model, provided

a mechanism to explain the stability of ______________. The concept of a


______________ wave in which the ______________ of the orbit was equal to the
h
de Broglie ______________, calculated using =
.

mv

1
E1 where E1=13.6eV
n2
to calculate the ______________ of the electron in a given orbit (n). You can then
rearrange the relation E=hf to calculated the frequency. Substituting the
frequency into = c/f you can calculate the ______________ associated with the
electron for the orbit.

For the hydrogen atom you can use the equation E n =

You calculate the wavelength for n=1 and the first orbit (n=1) has a
circumference equal to one ______________. You recalculate the wavelength for
n=2 and the second orbit (n=2) has a ______________ equal to ______________
wavelengths.
In 1927 ______________ and ______________ fired electrons at a ______________
target and observed ______________ phenomena that verified the ______________
______________ duality of electrons.

Reviewing




1
2
3
4
5

Define diffraction.
Describe how interference can be achieved.
Recall instances in which diffraction occurs.
Outline an experiment you could conduct to observe interference.
In terms of the Bohr atomic model, explain how matter waves were used to
account for the stability of electron stationary states.

6 Describe the experiment performed by Davisson and Germer in 1927 and


outline its significance.

7 Louis de Broglie proposed that any kind of particle has both wave and
particle properties. Recall the response given by the scientific community.

8 Recall and list the contributions made by Heisenberg to the development

Define diffraction and identify


that interference occurs
between waves that have been
diffracted.
Explain the stability of the
electron orbits in the Bohr
atom using de Broglies
hypothesis.
Describe the confirmation of
de Broglies proposal by
Davisson and Germer.

of atomic theory.

9 Recall and list the contributions made by Pauli to the development of


atomic theory.

10 If the speed of an electron is increased, predict what would happen to its


de Broglie wavelength.

11 In terms of de Broglies hypothesis, discuss the implications for locating an


electron in the ground state.

257

13

Birth of quantum
mechanics

Solving Problems
Solve problems and analyse
information using:
h
=
mv

12 Louis de Broglie equation is = h .

mv
a Recall what each term represents.
b Calculate the wavelength of an electron in a television tube with
a velocity of 3.0107 ms1.

13 Consider an electron of wavelength of 1.0 m.


a Calculate its velocity.
b Is this wavelength possible?

14 Using de Broglies equation, calculate the wavelength of:


a an electron travelling at 6.0105 ms1
b a proton travelling at 6.0105 ms1.

15 Calculate the velocity of a proton with a wavelength of 1.38 1012m.


16 a Calculate the wavelength of:

i a person (80kg) walking at 2 ms1


ii a soccer ball (0.43kg) kicked at a speed of 5 ms1
iii a car (1000 kg) travelling at 80kmh1.
b Assess the validity of your calculations against real-world observation.

17 Consider an electron at rest.


a Calculate the wavelength of the electron.
b Is this situation possible?

18 A car of mass 850kg is travelling very slowly at 7.01037 ms1.

Re

258

iew

Q uesti o

a Calculate the wavelength associated with the car.


b Will you see the wave properties of the car?

quanta to
quarks

PHYSICS FOCUS

The Microscope:
How small can we see?
H3. Assesses the impact
of particular advances in
physics on the development
of technologies
Describe the impact of de Broglies
proposal that any kind of particle has
both wave and particle properties.

Figure 13.7.1 (a) Light microscope, (b) electron microscope

The maximum magnification of microscopes in


most schools is usually about 400600 times.
More expensive research microscopes can achieve
magnifications of up 2000 times. You would think
that if you made better lenses and optics you could
increase your magnification to whatever you wanted.
Yet this is not the case and the reason for this limit
is related to the wavelength of the light that is
illuminating the object you are examining.
You can think of the wavelength of the light as
your probe, and any details smaller than your probe
would be difficult to see (or resolve). A useful analogy
is to consider that you are blindfolded and asked to
examine an unknown object using only the palms of
your hands instead of your fingers. Your fingers are
smaller and therefore able to detect finer details.
The same is true for the wavelength you use to look
at an object through a microscope.
Now consider using something that has a shorter
wavelength than light to illuminate the object. You
might consider using X-rays or gamma rays, which
certainly have shorter wavelengths. Unfortunately you
cannot easily build an optical system that focuses
these forms of radiation, and if you could the radiation
tends to have such high energies that it would just
pass straight through the object. Of course various
types of X-ray machines use these attributes to image
bones and airport luggage.
Now consider using a beam of electrons. They have
an associated de Broglie wavelength that can be many
times smaller than the wavelength of visible light.

You can easily focus and manipulate the beam using


electrostatic and electromagnetic lenses. You can also
easily see the electrons using detectors ranging from
a simple fluorescent zinc sulfide screen to specialised
complex semiconductor arrays.
1 Discuss how the wavelength of the radiation that
illuminates the object affects the detail you can
see with a microscope.
2 Determine the size of the smallest feature you
could visualise using a light microscope.
3 Research to find the maximum magnification
possible using an electron microscope.
4 Research and investigate the different types
of electron microscopes, including:
a transmission electron microscope
b scanning electron microscope
c scanning transmission electron microscope
d reflection electron microscope.

Extension
5 Outline the different ways in which specimens are
prepared if they are to be examined with an
electron microscope.
6 Discuss why the way in which most specimens
need to be prepared differs from they way they are
prepared when we use an optical light microscope.
7 Investigate the advantages and disadvantages of
using an electron microscope.

259

14
neutron, beta decay, atomic mass,
nucleons, strong nuclear force,
atomic number, nuclides,
transmutation, alpha decay, decay
series, atomic mass unit, fission,
nuclear reactor, controlled nuclear
reaction, atomic pile, critical mass,
uncontrolled nuclear chain reaction,
neutron scattering

20th century
alchemists
Base metals to gold
The idea of changing base metals into gold has been an age-old
quest of alchemists. Their attempts were not successful because
all the types of reactions the alchemists performed were merely
chemical reactions. To achieve their dream, they needed to
change the identity of the atom by changing the number of
protons in the nucleus. In this chapter we will trace the history
of nuclear physics, the modern alchemy, from the early 1930s
through to the construction of the first man-made
nuclear reactor.

14.1 Discovery of the neutron

Figure 14.1.1

260

James Chadwick

Rutherford proposed the idea of the neutron in 1920. At the time he considered
the neutron to be a system comprising a proton and an electron tightly bound
together. This system provided an explanation for the ejection of electrons from
the nucleus during beta decay. James Chadwick (18911974) set to work to
find the neutron.
Chadwick set up two main experiments in which he fired alpha particles at
a beryllium target. He allowed the unknown radiation generated in the first
experiment to pass through paraffin blocks and in the second experiment to pass
through nitrogen gas.
He then applied the conservation laws of momentum and energy, by setting
up simultaneous equations that involved the known masses and velocity
measurements of the alpha particle and ejected nuclei from both experiments.
Chadwick solved these equations in order to determine the likely mass of the
unknown radiation. A month after commencing these experiments, Chadwick
submitted for publication a paper entitled Possible existence of a neutron in
which he reported the neutron to be just slightly more massive than a proton.

quanta to
quarks

Many late nights

Table 14.1.1 The properties of nucleons

P Snow, a research student at the Cavendish Laboratory,


recalled that Chadwick worked night and day for 3 weeks
in his quest to identify the neutron.

Property

Proton

Neutron

Charge
Mass

+1.6 1019 C
1.673 1027 kg

Neutral
1.675 1027 kg

With the discovery of the neutron, the nucleus of the atom was considered to
consist of protons and neutrons; the number of protons equalling the number
of electrons in the neutral atom. The number of neutrons could vary to account
for the observed differences in the atomic mass of the atom. The proton and the
neutron are referred to as nucleons, when they are in a nucleus.
Be
foil

particles

Figure 14.1.2

Be
foil

paraffin

protons
(hydrogen
nuclei)

N2 gas

Discuss the importance of


conservation laws to Chadwicks
discovery of the neutron.
Define the components of the
nucleus (protons and neutrons)
as nucleons and contrast their
properties.

nitrogen
nuclei

particles

Chadwicks two main experiments

Checkpoint 14.1
1
2
3

Describe Rutherfords vision of the neutron.


Outline how Chadwick determined the mass of the unknown radiation from his experiment.
Describe the structure of the atom prior to and after the discovery of the neutron.

14.2 The need for the strong force


The new model of the nucleus posed a new
problem. What forces held the protons and
neutrons together to form a stable nucleus?
The new model of the nucleus posed a
new problem. What forces hold the protons
and neutrons together to form a stable
nucleus? Is it gravity? Assuming the size of
the nucleus is roughly ~1 1015m, the
gravitational force acting on a proton in the
nucleus of a helium-4 atom due to the other
proton and 2 neutrons is approximately
5.6 1034N (can you calculate it?)
The force of Coulomb (electrostatic)
repulsion between the two protons in the
nucleus is approximately 200N, which
is ~1035 times larger! (See Physics Phile
Repulsive protons.)

Evaluate the relative


contributions of electrostatic
and gravitational forces
between nucleons.

Repulsive Protons

he electrostatic force between charged particles follows Coulombs


law, an almost identical law to Newtons gravity, except that its
much stronger and it can be repulsive. Lets calculate the repulsive
force between two protons in a helium nucleus:
qq
Felectrostatic = k 1 22
d
(1.6 1019 C)(1.6 1019 C)
= 9.0 109
(1.0 1015 )2
230 N

where k is the electrostatic constant, q1 and q2 are the charges of the


protons and d is the distance between them.

261

14

20th century
alchemists

ractiv

Clearly, the force of gravity was too weak to overcome the huge electrostatic
repulsive force produced by the interaction between protonsa third
fundamental force of nature was required to explain the stability of the nucleus.
Experiments over the period from 1930 to 1950 showed that this new strong
nuclear force possessed the following properties.
1 The strong nuclear force does not depend on the charge, therefore all
nucleons (protons and neutrons) bind together with the same force. This is
supported by evidence that protons and neutrons are equally likely to be
ejected from a nucleus in a collision.
2 The strong nuclear force acts over short distances of about 1 1015m, the
diameter of a nucleus, and within this range the force is much stronger than
the electrostatic forces. The evidence supporting this is that otherwise the
nucleus would have a tendency to attract more protons and neutrons.
3 The strong nuclear force between the nucleons acts only between adjacent
nucleons. The evidence supporting this is based upon the observed stability
of the nucleus.

nte

M o d u le

repulsion

repulsion

Figure 14.2.1 shows the forces between two nucleons, and from the graph
you can see that the strong nuclear force is at a maximum at a distance of
approximately 1.3 1015 m. If the nucleons are less than 0.5 1015 m apart,
a repulsive force is present.
If we consider the resultant force acting on a proton or an alpha particle
approaching a nucleus, we see that the Coulomb repulsion increases and then
sharply decreases once the particle is within the range of the strong nuclear force
(see Figure 14.2.2).
Coulomb repulsion

Figure 14.2.1

Separation
of nucleons
1015 m

Nuclear force versus


separation between nucleons

Force

attraction

attraction

Force

range of the
nuclear force

Distance
from nucleus

nuclear attraction

Figure 14.2.2

Force acting on a positive charge


as it approaches the nucleus

Checkpoint 14.2
1
2

What is the difference in the magnitude of the electrostatic and gravitational forces between protons in the nucleus?
List the properties of the strong nuclear force.

Account for the need for the


strong nuclear force and
describe its properties.

262

14.3 Atoms and isotopes


The number of protons in a nucleus is referred to as the atomic number (Z).
The total number of protons and neutrons is called the atomic mass number (A).

quanta to
quarks
The number of neutrons (N) can be calculated by subtracting the atomic
number (Z) from the mass number (A).
N=AZ
The nuclear structure of an element is represented by ZA X where X is the
element symbol.
Atoms of each element have the same number of protons in their nuclei,
but the number of neutrons can vary, and these atoms are called isotopes.
There are 91 naturally occurring elements and we have identified more than
2500 different nuclei (or nuclides) associated with these. Of these isotopes,
90% are unstable and often quickly decay into other nuclei.

Checkpoint 14.3
Copy and complete the following table to identify the number nucleons and electrons in elements.
Element

Mass number

Atomic number

Protons

Neutrons

Electrons

C
He
F
Xe

14.4 Transmutation
Artificial transmutation
The process of changing one element into another is called transmutation.
Transmutation occurs naturally in stars through the process of nuclear fusion and
here on Earth via natural radioactivity by which certain elements decay spontaneously.
In 1919, Rutherford fired alpha particles into nitrogen gas and detected
a highly energetic particle that he identified to be a proton. This was actually the
first artificially induced transmutationthe alpha particle had collided with the
nitrogen nucleus to produce an oxygen nucleus and a highly energetic proton.
Rutherford initially detected the production of very high energy protons and,
after further experiments in which he used a cloud chamber, he identified that this
was not just a simple collision. Rather his calculations indicated that the nitrogen
nucleus had absorbed the alpha particle and then ejected a high-energy proton.
Rutherford then showed that the new nucleus was oxygen. He had changed
nitrogen-14 into oxygen-17, as prescribed by this equation:
4
2

He +

14
7

Define the term transmutation.

N 178 O + 11 H

Natural transmutation
Today we know that nuclei with more than 83 protons (Z > 83) or atomic mass
numbers greater than 209 (A > 209) are unstable and decay. In these atoms, the
repulsive electrical forces between the protons overcome the strong nuclear force.
We also know that instability due to odd number pairing of protons and neutrons
The two most common natural decay processes
can occur in smaller nuclei.
are the emission of alpha and beta particles.

Describe nuclear
transmutations due to
natural radioactivity.

263

14

20th century
alchemists

N
238U

146

4.47 109 y

145

234Th

144

24.10 d

143
142
141
140
138
1600 y

137
136
134

131

214Pb
27 min

129

Tl

19.9 min
214Po

decay
decay

1.637 104 s

1.30 min

127

214Bi

19.9 min

130210
128

218Po

3.10 min

210Pb 22.3 y
210Bi
5.01 d

126
125

210Po

138.38 d

206Pb
124
80 81 82 83 84 85 86 87 88 89 90 91 92 93 Z
Hg Tl Pb Bi Po At Rn Fr Ra Ac Th Pa U Np

Figure 14.4.1

Alpha-particle emission usually occurs with large unstable nuclei. An alpha particle
is identical to a helium nucleus 42 He . After the nucleus has emitted the alpha
particle, the atomic mass A decreases by four, corresponding to the loss of two
protons and two neutrons, the atomic number Z decreases by two, corresponding
to the loss of two protons, and the number of neutrons N decreases by two,
corresponding to the loss of two neutrons.
The general equation for this process is:
A
A4
4
AZ 42Y + or more formally Z X Z 2Y + 2 He
X represents the nuclei of the parent element and Y represents those of the
daughter element.
A common naturally occurring alpha decay is that of uranium-238 decaying
to thorium-234:
A
ZX

226Ra

222Rn

3.82 d

135

132

230Th

7.54 104 y

139

133

234Pa

70 s 234U
5
2.46 10 y

Alpha decay

This Segr chart is a plot of


number of neutrons (N) against
the atomic number (Z) for the
uranium-238 decay series.

238
92 U

234
90Th

238
92 U

+ or more formally

234
90Th

+ 42 He

Beta decay
There are three different types of beta-decay processes: beta-minus (), beta-plus
(+) and electron capture.

Beta (minus) decay


The beta-minus particle is identical to an electron. Beta-minus decay occurs when
the ratio of neutrons to protons is too high, and involves the transformation of a
neutron into a proton, an electron and an antineutrino e .
The transformation process is represented as:
n p + +
e

The generic equation for this process is:


A
ZX

The Unseen

n many parts of the world the bricks,


cement and soil contain significant
quantities of isotopes from the
uranium-238 series. Radium-226
decays to radon-222a colourless,
odourless, inert radioactive gas that is
heavier than air and often settles in
poorly ventilated cellars and
basements. Now consider breathing in
a radioactive gas! It decays by ejecting
a high-energy alpha particle that can
easily damage lung cells. But that is
not the worst of it! A radioactive
nucleus of polonium-218 remains; it is
not a gas and is very likely to lodge
itself in your lung tissue and continue
to decay down the 238U series. So
avoid hiding in the cellar or basement!

264

A
Z +1Y

+ + e or more formally

A
ZX

A
Z +1Y

0
1 e

+ 00 e

The atomic mass Z remains the same, as the total number of nucleons is unchanged.
The atomic number A increases by one and the number of neutrons N decreases by
one, corresponding to the transformation of a neutron into a proton.
A commonly occurring beta-minus decay is thorium-234 to protactinium-234:
234
90Th

234
91 Pa

+ + e or again formally

234
90Th

234
91 Pa

0
1 e

+ 00 e

Beta (plus) decay


The beta-plus particle is identical to an anti-electron commonly called a positron.
The process of beta-plus emission involves the transformation of a proton into a
neutron, positron and a neutrino. The transformation process is represented as
p n + + + e .
The general equation for this process is:
A
ZX

A
Z 1Y

+ + + e

or more formally

A
ZX

A
Z 1Y

0
+1 e

+ e

A commonly occurring beta-plus decay is the transformation of neon-19


to flourine-19:
19
10 Ne

199 F + + + e or again formally

19
10 Ne

199 F +

0
+1 e

+ 00 e

quanta to
quarks
A

Electron capture

238

This is the process in which an electron from an inner shell is


captured by the nucleus, resulting in the conversion of a neutron to
a proton, for example:

234

+ e 73 Li + e

The three types of beta decay are mediated by the fourth force of
nature called the weak nuclear force, which was proposed by Enrico
Fermi in 1934 to explain beta decay.
As Rutherford and Soddys 1903 paper stated, often a series
of decays occurs. For example, in nature the decay series for
urainium-238 is common. Figure 14.4.1, a Segr chart, and Figure
14.4.2, a plot of atomic mass against atomic number, both show the
decay series for uranium-238 ( 238
92 U ), which undergoes eight alpha
decays and six beta decays, terminating at the stable isotope
). The original nucleus for each transition is called
lead-206 ( 206
82 Pb
the parent and the resultant nucleus is called the daughter.

Gamma radiation
After the emission of an alpha or beta particle, the daughter nucleus
is sometimes left in an excited state. The change in energy from this
excited energy state of the nucleus to the ground state of the nucleus
will result in the emission of a very high energy photon called a
gamma ray and is represented by the Greek letter . Gamma
radiation does not change the atomic number (Z) or the atomic
mass (A) and therefore it is not an example of transmutation.
A general equation for gamma emission can be written as:

24 d 6.7 h

2.5 x

decay

105

7.5 x 104 y
226Ra

226

1600 y
222Rn

222

3.8 d

218

3.1 min

214

218At

218Po

214Pb
27 min
20 min

3.1 min 1.6 s

218Rn

1.6 s 0.04 s

214Bi 214
Po
20 min

1.6 x 104 s
210Tl 210
Pb 210Bi 210Po
210 1.3
22 y 5 d
min
5 d 138 d
206Tl
206Pb

206 4.2
81min82
Tl
Pb

83
Bi

84
Po

Figure 14.4.2

85
At

86
Rn

87
Fr

88
Ra

89
Ac

90
Th

91
Pa

92 Z
U

Plot of atomic mass (A) against the atomic


number (Z) for the uranium-238 decay series

PRACTICAL
EXPERIENCES
Activity 14.1

nte

Activity Manual, Page


115

ractiv

M o d u le

Checkpoint 14.4
Define a transmutation and outline how Rutherford made this happen.
Identify how Rutherford showed that the high-energy protons were not the result of a collision.
List the properties of an atom that will naturally transmute.
List the types of decay that can occur to help stabilise a nucleus and the properties each possess.

14.5 The neutrino


In alpha-particle decay, the energies of the ejected alpha particles have
well-defined values of kinetic energy, whereas in beta-particle decay a broad
spectrum of kinetic energies for the beta particles is observed. Many scientists
including Otto Hahn (18791968), Lise Meitner (18781968) and Chadwick
had all carefully studied beta decay. By 1930 experiments clearly showed that
the spectra of the kinetic energies of ejected beta particles when graphed was
a smooth and continuous curve ranging from just above zero to a maximum

Relative number of particles

1
2
3
4

234U

230Th

230

A
ZX

+
The * denotes the nucleus is in an excited state.

234Pa

234Th

decay

A *
ZX

4.5 x 109 y

17
4 Be

238U

9
8
7
6
5
4
3
2
1
0

1 2 3 4 5 6 7 8 9 10 11 12
Kinetic energy of the -particles (MeV)

Figure 14.5.1

Energy distribution for beta particles


265

14

20th century
alchemists

Discuss Paulis suggestion of the


existence of the neutrino and
relate it to the need to account
for the energy distribution of
electrons emitted in -decay.

value that was dictated by the parent nuclei. Refer to Figure 14.5.1 for a typical
energy distribution for beta particles.
In 1930, Wolfgang Pauli (19001958), in a letter addressed to
LiseMeitner and Hans Geiger, proposed that a new particle was also emitted
during the beta-decay process. Pauli proposed that this new particle had no
charge and would only very weakly interact with matter. This newly proposed
particle would allow the distribution of energies to be explained. The energy
released during a decay could be shared between the beta particle and the
neutrino in any ratio; thus, the spectrum of energies observed could be explained.
Enrico Fermi formally proposed a theory for beta decay whereby a neutron
in the nucleus of an atom was transformed into a proton, an electron and an
antineutrino. His theory also proposed a fourth force of nature, the weak nuclear
force (commonly referred to as the weak force). The scientific community
accepted the theory, even though Reines and Cowan did not experimentally
detect the neutrino until 1953.

The elusive neutrino

Checkpoint 14.5

he thermonuclear reactions in the core of the Sun


create an environment that produces streams of
neutrinos. As you stand at midday with the Sun
overhead, approximately 1013 neutrinos per second
pass through your body. At midnight when the Sun is
on the other side of the Earth, amazingly still about
1013 neutrinos per second pass through your body.
They have passed straight through the Earth! From
this you can see how weakly neutrinos interact with
matter, and shows the extent of the task physicists
had to surmount to detect these elusive particles.

Explain what led to the need for the neutrino.

Are You Nuclear Free?

id you know that one very widespread application


of nuclear physics is the smoke detector? The
detector commonly used in houses has a small
amount of americium-241, which emits alpha
particles, which ionise the air between two charged
electrode plates. A small current normally flows
between the plates; but if smoke enters the detector,
it disrupts the normal current and the alarm sounds.

14.6 Was Einstein right?


Using a mass spectrometer the masses of electrons, protons and
nuclei (as ions) can be determined by examining their radius of
curvature as they move through a magnetic field. The common
unit used to measure these extremely small masses on an atomic
scale is the atomic mass unit (amu), which is used rather than
the kilogram (kg), the SI unit for mass. Using this unit the
neutral carbon-12 atom has a mass of exactly 12 amu. It is easy
to convert between atomic mass units (amu) and kilograms (kg).
1 amu = 1.6605 1027 kg

Worked example
Question
a Convert 4.05 amu to kilograms.
b Convert 5.023 1027 kg to amu.

Solution
a 4.05 1.6605 1027 = 6.73 1027 kg
b

5.023 1027
1.6605 1027

= 3.025 amu

Einstein was right! Using his energymass equivalence E = mc2 allows us to


also convert an energy value into an equivalent mass or, vica versa, a mass into
an energy. The common unit of energy in atomic physics is the MeV (megaelectron volts 1106eV). Converting between amu and MeV is also easy:
1amu = 931.5MeV.
266

quanta to
quarks
Worked example
Question
Calculate the equivalent energy contained in 0.0015amu.

Solution
0.0015 931.5 = 1.4 MeV

Mass defect
The total mass of a stable nucleus is less than the total sum of the masses
of the protons and neutrons it contains. How can this be? Einstein linked mass
to energy in his equation E = mc2. In our case, when a nucleus forms, some
energy is radiated away. This loss of energy reduces the mass of the nucleus.
Conversely, when the unstable nucleus of a large atom splits into two or more
fragments, the combined mass of the daughter nuclei (and possible neutrons) is
less than the mass of the parent nucleus. This loss of mass is caused by some of
the original parent mass being transformed into the kinetic energy of the
fragments and also possibly gamma radiation.
The difference in mass between the total original mass and the total final
mass is called the mass defect.

Explain the concept of a mass


defect using Einsteins
equivalence between mass
and energy.

Solve problems and analyse


information to calculate the
mass defect and energy released
in natural transmutation and
fission reactions.

Worked example
Question
Calculate the mass defect for a helium-4 atom ( 42 He), given the rest mass of a helium
atom is 4.002602amu.

Solution
A helium-4 atom consists of a nucleus (2 protons and 2 neutrons) and 2 electrons.
Calculate the total mass of these constituent particles:



Mass of protons = 2 1.007276 amu


Mass of neutrons = 2 1.008665 amu
Mass of electrons = 2 0.000549 amu
The total mass of the constituent particles is 4.032980amu.

Calculate the difference in masses by subtracting the total mass of the constituent
particles from the actual mass of the atom:
4.032980 amu 4.002602 amu = 0.030378 amu
Therefore the mass defect for the helium-4 atom is 0.030378 amu.

Figure 14.6.1

The mass of constituent


parts is greater than the
mass of the whole nucleus.

Checkpoint 14.6
1
2
3
4

State what amu stands for and give reasons for its use.
Convert 3.1amu to kilograms.
Calculate the number of MeV from 0.3amu.
Define mass defect.

267

14

20th century
alchemists

14.7 Binding energy


Another useful way of interpreting this mass defect in atoms and nuclei is to look
We can use Einsteins equivalence of mass and
at it in terms of energy.
energy to convert this mass defect into what is referred to as the binding energy of
the nucleus. The binding energy tells you how much energy you would need to
separate the nucleus of the atom back into separate protons and neutrons.

Worked example
Question
Calculate the binding energy for the helium-4 ( 42 He) nucleus.

Solution
In our previous calculation we determined the mass defect for the helium-4 atom ( 42 He) to
be 0.030378 amu.
Using Einsteins equivalence between mass and energy we simply convert the mass
defect into the units of energy by using the relationship 1amu = 931.5MeV.
The binding energy for helium-4 nucleus ( 42 He) = 0.030378 931.5 = 28.3MeV.

To investigate the stability of various nuclei, the binding energy (Eb) per
nucleon can be calculated by dividing the binding energy by the total number
of nucleons (protons and neutrons) in the nucleus.
E
binding energy
Binding energy per nucleon =
= b
atomic mass number A

Worked example
Question
Calculate the binding energy per nucleon for helium-4 nuclei ( 42 He).

Solution
In our previous calculation we determined the binding energy for the helium-4 atom ( 42 He)
to be 28.3MeV. We know that the atomic mass number for helium-4 is 4.
E
binding energy
Therefore:
Binding energy per nucleon =
= b
atomic mass number A
28.3 MeV
= 7.075 MeV
4
We can see from Figure 14.7.1 that elements with atomic mass numbers
between 40 and 80 have nuclei that are tightly bound. The binding energy of
elements with atomic mass number greater than 80 is slightly less, and the
binding energy of elements with atomic mass number less than 40 decreases
sharply. You will notice that 42 He has quite a high binding energy per nucleon,
which makes it very stable and explains why alpha particles 42 He rather than
single protons are ejected from nuclei in alpha decay.
The graph also shows that if you fuse light nuclei together you would
increase the binding energy per nucleon, thus energy would be released. This
process, called fusion, is the nuclear reaction that provides the energy source for
stars. You can see that if the nucleus of a heavy atoms splits, the binding energy
per nucleon will increase. This process is called fission, and is the energy source
used in nuclear reactors.
=

268

Binding energy per nucleon Eb/A (MeV/nucleon)

quanta to
quarks
9

20
16O Ne

12C

4He

Ca Fe

Kr
Hg

greatest
stability

fission

6
5
4
3
fusion

2
1
0
0

Figure 14.7.1

50

100
150
Mass number (A)

200

Binding energy is greatest for elements with atomic mass


numbers between 40 and 80.

Checkpoint 14.7
1 Define binding energy.
2 Referring to the graph (Figure 14.7.1), explain how binding energy can help you understand when energy is
released or absorbed.

14.8 Nuclear fission


Between 1934 and 1938, Enrico Fermi and his fellow researchers fired
neutrons at a variety of target elements and produced many new unstable
radioactive nuclei. In the majority of cases some nuclei in the target would absorb
a neutron and then emit a beta particle. This beta-decay process transforms the
neutron into a proton, an electron and an antineutrino, thus increasing the atomic
number by one. Fermi predicted that he should be able to produce transuranic
elements (elements beyond uranium) with atomic numbers greater than 92
(Z>92). Fermi bombarded uranium with neutrons and tested the properties of
the new radioactive elements. He was sure he had produced element number 93.
Otto Hahn (18791968), Lise Meitner (18781968), Otto Frisch (1904
1979), Fritz Strassman (19021980), Irene Joliot-Curie and Pavle Savitch (1909
1994) all repeated Fermis experiments in order to identify the new isotopes. With
Hitlers invasion of Austria in 1938, Lise Meitner (an Austrian Jew) emigrated to
the safety of Sweden. Hahn and Strassman continued the work and, a few months
later, Hahn wrote to Meitner outlining the final results of their research, which
indicated that smaller nuclei were present. Further research supported these
results, confirming that the decay products of the bombardment of uranium were
not transuranic elements but rather unstable isotopes that included barium
(Z=56), radium (Z=88) and lanthanum (Z=57), all of which have nuclei with
atomic numbers much less than that of uranium (Z=92).

Describe Fermis initial


experimental observation
of nuclear fission.

269

14

20th century
alchemists

Checkpoint 14.8
1 Identify what a transuranic particle is.
2 Outline the experiment performed by Fermi which resulted in nuclear fission and not the production of
transnuranic elements.

Earths Own
Fission Nuclear
Reactor

ong ago, actually about 1.7


billion years ago in what is now
Gabon, Africa, there was a large
natural uranium deposit. At that
time in geological history there was
more urainium-235 present in ore
bodies; approximately 3%
compared to todays average 0.7%.
The scene was set and nature
provided the final ingredient: a
flowing aquifer provided a water
source that could act as a natural
moderator. In the 1970s this ore
body was mined and the ratio of
235
U to 238U was found to be
significantly lower than normal,
revealing the telltale fingerprint of
this ancient natural nuclear reactor.

Describe Fermis demonstration


of a controlled nuclear chain
reaction in 1942.

270

14.9 Chain reactions


Leo Szilard (18981964) realised that if the fission fragments included two
or more neutrons, these neutrons could cause other uranium nuclei to split,
resulting in a self-sustaining chain reaction. Further experiments by American
and French scientists showed that indeed the fission of a uranium nucleus
releases between two and four energetic neutrons. There are many dozens of
possible decay pathways, for example:
238
92

U+n

239
92

90
36

Kr + 146
Ba + 3n
56

The scientists working on this research realised that this chain reaction if
controlled could be used as a useful power source, but if it was allowed to
proceed uncontrolled, it could produce a huge explosion. In August 1939, war
with Germany seemed imminent and the American-based scientists felt that
America should investigate the possibility of building a bomb. Leo Szilard,
Eugene Wigner and Edward Teller drafted a letter that was signed by Einstein
and sent to the US President Franklin D Roosevelt.
It was known that 238
92 U would not support a chain reaction because the
neutrons released during the fission process did not have the right energy to
cause other 238
92 U nuclei to split. However, the rarer naturally occurring isotope
235
acted differently and was known to fission when struck by a slow neutron.
92 U
In 1935 Enrico Fermi and his team set about designing and carrying out an
experiment to see if a controlled nuclear reaction was possible. In 1942 Fermis
team designed and commenced building a fission reactor in a squash court at
Chicago University. Some 40000 graphite bricks, weighing 350 tonnes, were
used as a moderator to slow down the high-energy neutrons. They also provided
the reactor with a structural component that gave the reactor its name, the pile.
Natural uranium in the form of uranium oxide, which contains about 0.7%
235
, provided the fuel for this reactor, and a set of control rods made from
92 U
sheets of cadmium nailed to sticks of timber acted as neutron sponges. These
could be withdrawn or inserted into the pile to control the number of neutrons
and thus the rate of the fission reactions.
The atomic pile took 6 weeks to construct, and in early December 1942 the
control rods were withdrawn 15cm at a time. With each withdrawal the neutron
counters would climb and level off. At 2:00pm, Fermi announced that, based
upon his calculations, the next withdrawal would result in a self-sustaining
reaction. As predicted, the neutron intensity increased more and more rapidly.
1
After 4 2 minutes Fermi raised his hand and announced: The pile has gone
critical. The control rods where pushed back in and the reactor shut down.

quanta to
quarks

Figure 14.9.1

Fermis atomic pile

Controlled and uncontrolled nuclear chain reactions


A chain reaction requires:
1 fuel that is capable of fission; referred to as fissile material
2 an amount of fuel that produces sufficient neutrons to cause new fission;
this is referred to as the critical mass
3 neutrons with an energy that allows them to be absorbed by other fuel
nuclei. Slow (or thermal) neutrons are often best, as the de Broglie
wavelength is larger at low speeds. This larger wavelength increases the
likelihood of a neutron interacting with nearby nuclei and hence the
likelihood of it being absorbed.

Compare requirements for


controlled and uncontrolled
nuclear chain reactions.

The conditions for a controlled nuclear chain reaction are such that the
available neutrons, which cause the fission, are regulated. The control
mechanisms in a nuclear reactor include the use of neutron-absorbing materials
within the physical structure and the manipulation of control rods.
In an uncontrolled nuclear chain reaction, such as in a nuclear
reactor meltdown or in a fission atomic bomb, the production of neutrons goes
unchecked and the fission reactions increase at an accelerating rate. This process
releases an enormous amount of energy in a very short period of time and
results in a significant explosion.

Checkpoint 14.9
1 Explain how a chain reaction can be sustained in nuclear fission.
2 Outline Fermis experiment to show nuclear fission.
3 Give reasons why artificial nuclear fission is a desired process.
271

14

20th century
alchemists

14.10 Neutron scattering

Describe how neutron


scattering is used as a probe
by referring to the properties
of neutrons.

The discovery of the neutron and the further development of nuclear reactors
provided scientists and industry with a powerful analysis tool. Chadwicks work
had revealed that neutrons were subatomic particles with a mass approximately
the same as that of the proton, and no net electric charge. From de Broglies
work we also know that neutrons will exhibit wave properties and their de Broglie
wavelength will vary, depending on their velocity. Interestingly, although
neutrons have no net charge, it was discovered they do have the property of spin;
they therefore possess a magnetic property and act like little subatomic magnets.
These properties make the neutron an excellent tool with which to probe aspects
of atomic structure.
There are four main applications of neutron scattering:
Spacing: The de Broglie wavelength associated with slow (thermal) neutrons
allows investigations to determine the structure and spacing between atoms
in solids and liquids.
Motion: The energy of slow (thermal) neutrons is similar to the energies
associated with the movements of atoms in solids and liquids, allowing
researchers to investigate atomic vibrations and the forces between atoms.
Magnetic structure: The neutron acts like a subatomic magnet, which allows
researchers to examine at the atomic level the structures of semiconductors
and magnetic materials.
Inner structure: X-rays and electrons are scattered by atomic electrons, but
neutrons are scattered by atomic nuclei and, unlike alpha particles, are not
influenced by electrostatic forces. This allows neutrons to penetrate dense
materials to depths of several centimetres, therefore making it possible for
researchers to study material deep inside large pieces of equipment (such as
aircraft engines), and inside containment vessels that have varying conditions
of pressure, temperature and environment.
The first neutron-scattering experiments were carried out by Ernest Wollan
and Clifford Shull in 1945 using another graphite pile reactor built at Oak Ridge.
Today neutron diffraction is used in combination with X-rays to investigate the
structures and properties of a wide range of materials.
The Bragg Institute at the Australian Nuclear Science and Technology
Organisation (ANSTO) OPAL reactor facility, located just south of Sydney,
currently houses eight neutron-beam instruments. The neutron-scattering and
X-ray techniques are used to solve complex research and industrial problems
across a wide range of fields including engineering, pharmaceuticals, mining,
plastics and biology.

Checkpoint 14.10
1 Outline how neutrons can be used to probe atomic structure.
2 List applications of neutron scattering.

272

PRACTICAL EXPERIENCES

quanta to
quarks

CHAPTER 14

This is a starting point to get you thinking about the mandatory practical
experiences outlined in the syllabus. For detailed instructions and advice, use
in2 Physics @ HSC Activity Manual.

Activity 14.1: Detecting radiation


Using different pieces of detection apparatus, determine the type of radiation and
the penetration ability of the radiation being emitted.
Equipment list: alpha, beta and gamma radiation sources, aluminium foil of
varying thicknesses, paper, 2 mm thick lead sheets, GeigerMller tube, spark
counter, cloud chamber, dry ice, methylated spirits, ruler.
Discussion questions
1 List the apparatus best suited to detect each form of radiation.
2 In order of penetrating ability, list each radiation.
3 Identify the properties that make each radiation identifiable.

Perform a first-hand
investigation or gather
secondary information to
observe radiation emitted
from a nucleus using the
Wilson Cloud Chamber or
similar detection device.

273

14

274

20th century
alchemists

Chapter summary

Rutherford proposed the idea of the neutron in 1920.


In 1934 Chadwick fired alpha particles at a beryllium
target and allowed the unknown radiation generated
to pass through paraffin blocks and nitrogen gas.
Chadwick then applied the conservation laws of
momentum and energy to determine the mass
of the neutron.
The proton and the neutron are referred to as nucleons
when they are in a nucleus.
A third fundamental force called the strong force was
required to explain the stability of the nucleus, as the
force of gravity was too weak to overcome the huge
electrostatic repulsive force produced by the interaction
between protons.
The strong nuclear force has the following properties:
It does not depend on the charge, therefore all
nucleons (protons and neutrons) bind together with
the same force.
It acts over a short distance, in the order of the size
of a nucleus, and the force within this range is much
stronger than the electrostatic forces.
It acts only between adjacent nucleons.
An elements nuclear structure is represented by ZA X
where X is the element symbol.
The number of protons in a nucleus is referred to as
the atomic number (Z).
The total number of protons and neutrons is called
the atomic mass number (A).
The number of neutrons (N) can be calculated by
subtracting the atomic number (Z) from the mass
number (A).
Isotopes are atoms of an element that have the same
number of protons but a different number of neutrons
in their nuclei.
The process of changing one element into another is
called transmutation.
The first verified artificially induced transmutation was
reported in 1919 by Rutherford when he fired alpha
particles into nitrogen gas.

The two most common natural decay transmutation


processes are the emission of alpha and beta particles.
Alpha decay usually occurs with large unstable nuclei
and an alpha particle is identical to a helium nucleus
4
2 He .
There are three different types of beta-decay processes:
beta-minus (), beta-plus (+) and electron capture.
In 1930 Pauli proposed the existence of the neutrino to
account for the energy distribution of electrons emitted
in beta decay.
The total mass of a stable nucleus is less than the total
sum of the masses of the protons and neutrons it
contains.
The difference in mass between the total original mass
and the total final mass is called the mass defect.
The binding energy tells you how much energy you
would need to separate the nucleus of the atom back
into separate protons and neutrons.
Fermis initial nuclear fission experiments involved firing
neutrons at a variety of target elements and producing
many new unstable radioactive nuclei.
In 1942 Fermis team designed and built a fission
reactor (an atomic pile) in a squash court at Chicago
University.
A chain reaction requires:
1 fuel that is capable of fission
2 a critical mass of fuel, which produces sufficient
neutrons to cause new fission
3 neutrons with an energy that allows them to be
absorbed by other fuel nuclei.
A controlled nuclear chain reaction requires the number
of available neutrons that cause the fission to be
regulated.
An uncontrolled nuclear chain reaction occurs when the
production of neutrons goes unchecked and the fission
reactions increase at an accelerating rate.
Neutron-scattering techniques are used to solve complex
research and industrial problems across a wide range of
fields including engineering, pharmaceuticals, mining,
plastics and biology.

Review questions

quanta to
quarks

Physically speaking
Pretend you are Charles Wilson, the inventor of
the cloud chamber, shown in Figure 14.11.1.
Name the components of your new invention.
Prepare a short scientific report on your new invention
for presentation to your class.

a Discuss the features of the apparatus; you

could also include a description of what


happens when you place a magnet on top
of the apparatus.

b Complete your report, outlining the importance


of it to reveal the nature of matter.

F
D

Figure 14.11.1 Wilson cloud chamber

Reviewing

7 Copy and complete the following table.


Process

1 Name the scientist who predicted the existence of

Fission

the neutron.

Fusion

2 Copy and complete the following table for the

Alpha emission

properties of the nucleons of an atom.

PROPERTIES

Neutron

Beta emission

Proton

Symbol

Describe Fermis demonstration of a controlled


nuclear chain reaction in 1942.

Charge
Mass (kg)

Compare requirements for controlled and


uncontrolled nuclear chain reactions.

3 Define the term transmutation.


Discuss the importance of conservation laws
to Chadwicks discovery of the neutron.

4 Recall how the conservation laws used by Chadwick

8 Outline Fermis 1942 demonstration of a controlled


nuclear reaction.

9 Describe the requirements for:


a a controlled chain reaction
b an uncontrolled nuclear reaction.

allowed him to identify the neutron.

5 Assess why the invention and development of the


Wilson cloud chamber was important in identifying
transmutation.

a brief description of each process

6 Describe the properties of the strong force.

275

14

20th century
alchemists

10 Copy and complete the following table which outlines


the events associated with the discovery and
identification of the neutron.

Scientist(s)
1920
1930
1932

Contribution

Firing alpha particles into nitrogen gas

Observed transmutation and also predicted the


existence of the neutron
Identified the production of a weak but
penetrating radiation
Published a paper describing

Bothe and Becker


Joliot Curie and
______________

1934

Experiment

______________

Fired alpha particles into a beryllium target and


allowed the radiation to pass through a
paraffin block
Fired ______________ into a beryllium target and
allowed the radiation to pass through a
______________ and ______________

Used the conservation laws on his measurement


observations and identified that the radiation
was most probably composed of ______________

11 Referring to Figure 14.11.2, describe in words the

repulsion

force acting on a proton or an alpha particle that is


fired directly toward the nucleus.

Force

Coulomb repulsion

range of the
nuclear force

distance
from
nucleus

KE of beta particles

Figure 14.11.3

attraction

13 Describe the following processes associated with


nuclear attraction
repulsion in the nucleus

Figure 14.11.2
Discuss Paulis suggestion of the existence
of the neutrino and relate it to the need to
account for the energy distribution of electrons
emitted in -decay.

12 This question relates to the graph in Figure 14.11.3.


a Explain why the distribution of kinetic energies
for the beta particles was so baffling for the
scientists.
b Name and recall the properties of the new particle
suggested by Pauli.
c Account for the energy distribution of the electrons
emitted in beta decay.

276

nuclear transmutations due to natural radioactivity.


Include any other particles that may be present in the
process and write word equations for each process.
a Alpha emission (-decay)
b Beta emission (-decay)
c Emission of gamma radiation ()
d Another naturally occurring transmutation

Solving problems
14 Consider a helium nucleus where the distance

between the protons is 11015m and given that


qq
mm
Felectrostatic = k 1 2 and Fgravitational = G 1 2
d2
d2
a Calculate the electrostatic force between the
two protons.
b Calculate the gravitational force between the
two protons.
c Account for the need of the strong force.

quanta to
quarks
Solve problems and analyse information to
calculate the mass defect and energy
released in natural transmutation and fission
reactions.

15 Calculate the mass defect for a lithium-6 atom ( 63Li),

given the rest mass of a lithium-6 atom is


6.015122 amu and the following rest masses:
proton = 1.007276 amu
neutron = 1.008665 amu
electron = 0.000549 amu

16 Helium ( 42He) has a binding energy of 7.1MeV

per nucleon. Given that the mass of an electron is


0.000549 amu, a proton is 1.0073 amu and
a neutron is 1.0087amu, calculate:
a the total binding energy in the 42He nucleus
in MeV
b the mass defect of the 42He nucleus in amu
c the mass of a 42He atom in amu.

Binding energy per nucleon (MeV )

9
8
7

16O

4He

56Fe

208Pb

12C

235U

9Be

6Li

5
4

2.9 MeV/nucleon

3 3H
3He
2

1.15 MeV/nucleon

2H
1 1H

50

100

150

200

250

Atomic mass (A)

Figure 14.11.4

18 Consider this fission reaction of the uranium-235


nucleus:
235
1
131
92U + 0 n 53I +

? + 3 01n

a Identify the missing product in the above equation.


b From the information in Figure 14.11.4, estimate
the binding energies of the original uranium-235
and the two daughter nuclei.

19 Identify the missing components in the following


reactions:
a

1
65
66
0n + 29Cu 29?

22
?
11Na 10?

+ +01e +

20 Tritium and deuterium are isotopes of hydrogen and


the symbols T and D are often used when writing
nuclear reactions.

Consider the following fusion reaction:


3 T + 2D 4He + 1n +
1

Given that the mass of deuterium is 2.014amu,


tritium is 3.016amu and helium is 4.003amu,
calculate the energy of the -ray.
iew

Q uesti o

energy versus mass number.


a Define the term binding energy.
b Construct an argument based on the information
in Figure 14.11.4 why alpha particles rather than
just protons are ejected from some radioactive
nuclei.
c Estimate the binding energy of iron.

Re

17 Figure 14.11.4 shows a graph of average binding

277

14

20th century
alchemists

PHYSICS FOCUS
Probing Atomic Structure
In 1934 Ernest Rutherfords prediction of neutrons
was proved by his associate James Chadwick, work
that gained Chadwick the Nobel Prize in Physics
in 1935. More recently, Bertram Brockhouse and
Clifford Shull shared the Nobel Prize for Physics in
1994 for pioneering contributions to the development
of neutron-scattering techniques for studies of
condensed matter. This is the modern application of
neutron scatteringdetermining the arrangement
and motion (structure and dynamics) of atoms and
molecules, both of which can determine the physical
properties of the material they form.
Today, these methods are practised at neutronscattering centres such as the Bragg Institute at the
Australian Nuclear Science and Technology
Organisation (ANSTO) located at Lucas Heights,
south of Sydney. Neutrons are used to study atomic
positions, motions and magnetic properties of
materials. Neutron scattering provides unique
information about a material that cannot be obtained
by other methods.

H3. Assesses the impact of particular


advances in physics on the development
of technologies
Define the components of the nucleus
(protons and neutrons) as nucleons and
contrast their properties.

1 Discuss why neutrons are so useful in probing the


structure of materials.
2 Discuss the role of neutrons in fission reactors
and outline how they could be used as a neutron
source.
3 You are a physicist at ANSTO, describe how you
would construct apparatus to provide your
researchers with a beam of neutrons.

Extension
4 Investigate and report on the ways in which you
could detect scattered neutrons.
5 Neutron scattering is very sensitive to different
isotopes of an element (atoms that have nuclei
with the same number of protons but different
numbers of neutrons). Outline some reasons why
this would be the case.

Figure 14.11.5
Dr Vanessa Peterson is changing
a sample in a high-temperature
furnace near Echidna, the highresolution neutron powder
diffractometer at ANSTO.
In this experiment, the structure
of the material was determined,
as it changed with temperature.

278

The particle zoo


The atomic age
The unleashing of the power stored in the nucleus of an atom
dramatically shaped the world we live in today. The detonation
of two atomic bombs dropped on Japan in 1945, and the
post-World War II threat of atomic weapon proliferation, leading
to the Cold War between the Soviet Union and the West,
changed the lives of millions of people. On the scientific front,
the development of particle accelerators allowed the three
constituents of matter known in the 1930s to grow to several
hundred by the mid-1970s. The secrets of the atom were
being uncovered and the actual fabric of our universe was
being unveiled. The giant accelerators were
recreating the processes that occurred during the
first few fractions of a second during the birth of
time and spacethe Big Bang.

15
Manhattan Project, isotope,
Los Alamos, fusion, fission, fuel rods,
core, moderator, control rods, coolant,
radiation shielding, radioisotopes,
half-life, particle accelerator, cyclotron,
positron, radiopharmaceuticals,
radiotherapy, irradiated, cosmic rays,
muons, synchrotron, linear
accelerators, quarks, hadrons, bosons,
pions, Standard Model, mesons,
baryons, leptons

15.1 The Manhattan Project


The United States of Americas top-secret nuclear bomb research project,
code named the Manhattan Project, ushered in the atomic age. The project
comprised many sub-projects including Fermis atomic pile in Chicago. Another
reactor was built near Richland in Washington to produce plutonium, and a
gaseous diffusion plant in Oak Ridge Tennessee was constructed to extract the
from uranium ore. At the famous Los Alamos facility in
fissile isotope 235
92 U
New Mexico, the theoretical and experimental work for the design, development
and testing of the atomic bomb was carried out.
At Los Alamos, research into the construction of two types of atomic bombs
was carried out. A uranium-235 bomb named Little Boy was never tested prior to
being dropped on Hiroshima on 6 August 1945. A more complex plutonium-239
bomb, which was tested at the Trinity site in the New Mexico desert, resulted in
the construction of the bomb named Fat Man, which was dropped on Nagasaki
on 10 August 1945.
The driving fear that fuelled the Manhattan Project centred on the possibility
that Germany was building its own bomb. After World War II had ended, it was
found that Hitlers military had invested the majority of its scientific research and
development into conventional weapons and rocketry. They had access to a heavy
water plant in Norway, but very little progress had been made toward the
construction of an atomic bomb.

PRACTICAL
EXPERIENCES
Activity 15.1

Activity Manual, Page


119

279

15

The particle zoo

The Manhattan Project: its legacy

Gather, process and analyse


information to assess the
significance of the Manhattan
Project to society.

The Manhattan Project left a legacy that shaped the 20th century in ways
that few could have imagined. The project led to the post-World War II nuclear
arms race between the Soviet Union and the West. After the test detonation of
the Soviet Unions first bomb in 1949, relations between the Soviet Union and
the West deteriorated. This tension became known as the Cold War and
continued until the early 1990s; it ended with the dissolution of the Soviet
Union. The United States of America, Britain and the Soviet Union developed
fusion bomb technology, known as the thermonuclear bomb, the H-bomb or
the hydrogen bomb. By the mid-1960s France and China had tested atomic
weapons, and today Israel, India, Pakistan and North Korea also possess atomic
weapons. Despite the Nuclear Non-proliferation Treaty, which pursued a
reduction in weapon stockpiles, today there is a worldwide arsenal of
approximately 20000 nuclear weapons.
In tandem with the military developments, civil projects using nuclear
energy have also proliferated. Today there are 440 nuclear power plants
worldwide, which supply 15% of the worlds electricity demand, and there are
more than 200 research reactors. The use of reactors for nuclear medicine,
neutron scattering and other industry-based applications highlights the peaceful
endeavours pursued by scientists and engineers today.

Checkpoint 15.1
1
2
3
4

Outline the parts and purposes of the Manhattan project.


Justify the project being fuelled by the race to beat Germany to building the bomb.
What was the Cold War?
List developments that have come about due to the knowledge gathered from this project (other than weapons).

15.2 Nuclear fission reactors


Nuclear reactors have a variety of applications in modern society. They are used
as an energy source to generate electricity for power grids and to power naval
vessels. Research laboratories and industry use reactors as a neutron source to
probe the structure and properties of materials, and irradiate materials to make
a range of industrial, agricultural and pharmaceutical isotopes.

Components of a typical nuclear fission reactor

Explain the basic principles


of a fission reactor.

280

Fission reactors are carefully designed to control the rate of fission


(splitting) of large nuclei to ensure their safe operation. The fission process
releases energy, which manifests itself as heat and high-energy gamma radiation.
For the purpose of this course we will limit our discussion to uranium fuel
nuclear fission reactors, which comprise six key components: fuel rods, core,
moderator, control rods, coolant and radiation shielding.
Fuel rods: These are tubes filled with pellets of uranium oxide. The fuel rods
are located in the centre of the reactor known as the core.
Core: The core of the reactor houses the fuel rods, control rods, a coolant
system and a moderator material (which sometimes also acts as a coolant).

quanta to
quarks
pressuriser (prevents
water boiling)

containment building
primary coolant
circuit (high
pressure
water)

boiler

pump

water

condenser

turbines and
electricity
generator
to grid

control
rod

Figure 15.2.1
sea water to
condense steam

fuel
rod

water acts as coolant


and moderator in reactor core

Moderators: These materials slow down neutrons, improving the chance


of the neutron being captured by a nucleus. Commonly used moderators
include ordinary water, heavy water and graphite. The neutrons are slowed
by colliding inelastically with the moderator nuclei. For water, the neutron
has about 20 collisions, which reduce its kinetic energy from about 2MeV
to 1eV.
Control rods: These are usually made from boron or cadmium and are
located between the fuel rods. The control rods absorb neutrons and are
adjusted so that the chain reaction proceeds at a constant rate. A constant
rate is achieved when, for the average fission, each ejected neutron initiates
one new fission. If the reactor needs to be shut down for service or in an
emergency, the control rods are inserted fully into the core. These capture
the neutrons and the chain reaction ceases.
Coolant: The reactor generates heat and the coolant transfers this heat away
from the core. In some reactors normal or heavy water is used as a coolant
and also serves as a moderator. In nuclear electrical power stations, the heat
is used to make steam, which powers a turbine and generator.
Radiation shielding: Reactor cores emit large quantities of gamma radiation
and neutrons. Lead and graphite are used to absorb and reflect radiation,
which protects the containment walls of the reactor vessel. The walls
themselves are made from high density concrete. These radiation shields are
designed to protect people and the environment and prolong the working life
of the reactor facility.

A schematic diagram of a pressurised


water reactor. A typical 1000 MW power
plant consumes about 6000000 tonnes of
black coal each year, or about 25 tonnes of
enriched uranium that has been obtained
from around 75000 tonnes of ore.

Nuclear energy
of fuel

Heat energy
of coolant

Kinetic energy of
steam and turbine

Electric energy

Light, heat,
sound, etc.

Figure 15.2.2

The energy stored in the


nuclear fuel undergoes a
number of transformations.

The nuclear reaction resulting from the absorption of a slow (or thermal)
neutron by uranium-235 has many possible pathways for the decay process
to proceed.
Following are three examples:
235
92

U+n

235
92
235
92

236
92

U+n

236
92

U+n

236
92

140
55

93
Cs + 37
Rb + 3n

147
57

90
36

La + 87
Br + 2n
35

Kr + 143
Ba + 3n
56
281

15

The particle zoo

Checkpoint 15.2
1
2

Define fission.
Construct a table that lists the six key elements in a nuclear reactor and outline their purpose.

15.3 Radioisotopes
Radioisotopes are increasingly being used in medicine, scientific research and
industry. They provide a unique tool that is often cheaper and more effective
than alternative techniques and processes. Radioisotopes are atoms that have an
unstable ratio of protons to neutrons and will decay via alpha or beta decay to
attain a more stable configuration. Some may also emit gamma radiation. Each
radioisotope has a specific half-lifethe time it takes half of the radioisotope
sample to decay. For example, if a sample contains 12 grams of a radioisotope
with a half-life of 5 hours, in 5 hours time half of the radioisotope will have
decayed and 6 grams will remain. Five hours later there will be 3 grams of the
original radioisotope present in the sample, and so on.
Radioisotopes occur naturally or can be produced by changing the ratio
of nucleons in the nuclei of a target material in a nuclear reactor or a particle
accelerator. A nuclear fission reactor provides a source of neutrons that can be
used to irradiate a target material. A particle accelerator such as a cyclotron can
accelerate protons or ions and fire them into a target material. Both processes are
required to produce the range of radioisotopes used in medical and industrybased applications.
In Australia, the Australian Nuclear Science and Technology Organisation
(ANSTO) facilities in Sydney, produces a range of radioisotopes. ANSTO
operates Australias only nuclear research reactor, OPAL, at Lucas Heights and
the National Medical Cyclotron at The Royal Prince Alfred Hospital. It is
necessary to have both the research reactor and the cyclotron, as each produces
different types of radioisotopes. Pharmaceutical companies, research centres and
hospitals operate other medical and industrial cyclotrons.

Radioactive tracers
Often radioisotopes are used in medical and industrial applications to track the
movement, flow or absorption of materials. The radioisotope can be incorporated
into a molecule or introduced into the system. These isotopes then act as tracers
and allow the position, flow or absorption to be mapped. In complex biological
systems it is possible to map the pathway of specific chemicals being transported
and used by cell tissue. For example, glucose is metabolised by brain tissue and so
the radioisotope carbon-11 can be placed into the glucose molecule to act as a
tracer in medical imaging. The carbon-11 via beta-decay is a positron emitter,
and the radioactive labelling of the glucose molecules can be tracked and imaged
using positron emission tomography (PET). An industrial application is the use
of a radioactive tracer in mapping the dispersion of a sewerage ocean outfall.

282

quanta to
quarks

Medical radioisotopes
Nuclear medicine has been used routinely since the 1970s. Radioisotopes that
decay quickly are said to have short half-lives, and are often used in diagnostic
procedures. Radioisotopes with longer half-lives are often used therapeutically to
target diseased organs and tumours.

Describe some medical and


industrial applications of
radioisotopes.

Radiopharmaceuticals
Radiopharmaceuticals can be classified as diagnostic or therapeutic. The
overarching process on which this pharmacology is based is the specific uptake
and absorption of chemicals by organs and specific body tissues. This provides an
ideal mechanism that enables radioisotopes to be incorporated into or attached
to molecules, which are then tracked or used to target specific tissues.
Diagnostic radiopharmaceuticals are used to assess the functioning of organs
including the lungs, heart, liver and brain; identify bone fractures not visible in
X-rays; and assess the flow of fluids such as blood. The decay of the diagnostic
pharmaceuticals can be monitored by detectors that may comprise a simple
device such as a Geiger counter or a complex array of detectors that convert the
information into an image. Diagnostic radiopharmaceuticals subject the body to
a very low radiation dose, usually comparable to a routine diagnostic X-ray.
Therapeutic radioisotopes generally contain radioisotopes with longer
half-lives and once absorbed by the specific tissue or organ will deliver a target
dose of radiation.

Radiotherapy
In radiotherapy the radiation emitted from a radioactive source is directed
at an area of diseased tissue. This procedure is referred to as teletherapy when the
source is located outside the body. In brachytherapy a radioactive implant is used
to target the specific tissue.

Positron emission tomography (PET)


This imaging technique also relies on the principle that specific molecules are
absorbed by specific organs or tissues. In this imaging procedure the radioisotope
used will decay and produce a positron via beta decay. The positron (an
antimatter electron) will collide with an electron and the two will annihilate,
producing two gamma rays. These are detected and triangulated with other
events by a computer, and the location and activity of the targeted organ or
tissue is imaged. (See Chapter 19 Imaging with gamma rays for more details.)

Industrial radioisotopes
Radioisotopes are used widely in industry across applications as diverse as
checking the structural integrity of bridges, determining wear in engine
components, examining welds in gas lines, imaging internal structures in jet
aircraft engines, and thickness control. Radioisotopes commonly used in
measuring the thickness of materials include iridium-190 and cobalt-60. The
most common (often unrecognised) application of a radioactive isotopes is
smoke detection, and americium-241 is the radioisotope most commonly used
in home-based detector systems.

Figure 15.3.1

Brachytherapy for prostate


cancer can be administered
using seeds, small
radioactive rods implanted
directly into the tumour.

283

15

The particle zoo


Eyes

phosphorus-32

Thyroid

technetium-99m
iodine 123
iodine-131
iodine-132
selenium-75

Brain

iodine-131
mercury-197
technetium-99m
oxygen-15
carbon-11

Blood

Heart

iron-59

rubidium-81m
thallium-201
cesium-137

Lungs

xenon-127
xenon-133
nitrogen-13
oxygen-15
carbon-11
indium-113m
technetium-99m

Spleen

chromium-51
rubidium-81
technetium-99m

Kidney

technetium-99m
mercury-197
iodine-131

Liver

technetium-99m
iodine-131
gold-198

Pancreas

selenium-75

Bladder

gold-198

Placenta

iodine-123
iodine-131
carbon-11

Knee

yttrium-90
rhenium-186

Prostate

gold-198
indium-111

Bone

strontium-85
strontium-87
fluorine-18
iron-52
phosphorus-32
technetium-99m

phosphorus-32
arsenic-74
indium-133m
iron-18

Lymph

gold-198

Figure 15.3.2

Commonly used radioisotopes


and the organs in which they act

Agricultural radioisotopes

PRACTICAL
EXPERIENCES
Activity 15.2

Activity Manual, Page


122

Radioactive isotopes are used in the agricultural industry as tracers in


plants to explore chemical and biological processes. As in medical applications,
specific chemicals are taken up and used by specific parts of plants. Radioisotope
tracers are attached or incorporated into molecules, which are then taken up by
plants. For example, the radioisotopes phosphorous-32 or nitrogen-15 can be
added to fertilisers and the uptake of the fertiliser measured by a sensitive Geiger
counter. Similarly, toxic heavy metal compounds such as those of mercury or
cadmium can be introduced into a test soil. The uptake of these tracer
radioisotopes can be used to investigate potential problems associated with
livestock feed or food for human consumption when planted in areas containing
these toxins.

Food irradiation
In many countries certain foods are irradiated in order to increase the
shelf life and make some foods safer to eat. The exposure of the food to gamma
radiation targets disease-causing bacteria or those that cause spoilage. A
common, widely used radioisotope is cobalt-60. The gamma rays passing
through the food possess enough energy to destroy many bacteria without
Identify data sources, and gather,
changing the texture or flavour of the food. The food never comes into contact
process, and analyse information
with the radioactive source or other forms of radiation and therefore is not at risk
to describe the use of:
of itself becoming radioactive. Examples of irradiated foods found worldwide
a named isotope in medicine
include potatoes and onions in which sprouting is reduced, grains, meats
a named isotope in agriculture
a named isotope in engineering. including poultry and some fish, many spices and dried herbs, and fresh fruits.
284

quanta to
quarks

PHYSICS FEATURE
Hydrogen as an
energy carrier

ecent scientific interest in hydrogen as an


alternative energy carrier to carbon-based fossil
fuels such as petrol has generated questions that can
be answered using neutrons generated by a fission
reactor, and neutron-scattering techniques. One of
the problems with using hydrogen as an energy carrier
is the difficulty in storing such a small molecule.
Before we can study new hydrogen-storage materials,
we need to know where the hydrogen is in the
material and how it is interacting with the storage
material.
At Australias OPAL research reactor, neutron
diffractometers such as ANSTOs high-resolution
neutron powder diffractometer (Echidna) can be used
to see hydrogen. The protium (1H) in the material
is replaced with deuterium (2H), and the neutron
scattering by 2H that is obtained in a neutrondiffraction experiment allows the position of the
deuterium in the material to be established.
For example, this technique was used to
determine nine positions on the inner surface
(referred to as adsorption sites for molecular
deuterium) within the porous material Cu3(1,3,5benzenetricarboxylate)2, which adsorbs hydrogen.
The neutron scattering arising from the deuterium
molecules in this material is shown in yellow in
Figure 15.3.3.

Figure 15.3.3

Neutron scattering was used to identify the


hydrogen sites in the porous material
Cu3(1,3,5-benzenetricarboxylate)2.

Neutron spectrometers give information about the


dynamics and motions of atoms and molecules.
Neutron spectroscopy takes advantage of the
exceptionally strong scattering from hydrogen 1H,
which dominates the spectrum obtained. The
scattering from 1H gives information about the
dynamics (motions) of the hydrogen and can be used
to understand the types of interactions that the
hydrogen has with the host material. In the case of
Cu3(1,3,5-benzenetricarboxylate)2, some of the nine
sites for molecular hydrogen have a stronger
adsorption enthalpy with the material than others.

Checkpoint 15.3
1
2
3
4
5
6
7
8
9

Define the term radioisotope.


Explain what half-life is.
Outline how radioisotopes can be made.
Recall what is a radioactive tracer and what it is used for.
Explain the significance of using different half-lives in medical radioisotopes.
Outline what radiopharmaceuticals are used for.
Compare and contrast radiotherapy and brachytherapy.
Recall one use of radioisotopes in industry and one in agriculture.
Why can radiation be used to sterilise food safely?
285

15

The particle zoo

15.4 Particle accelerators


Identify ways by which
physicists continue to develop
their understanding of matter,
using accelerators as a probe
to investigate the structure
of matter.

Prior to the development of particle accelerators, scientists had observed


showers of particles caused by cosmic rays colliding with the Earths atmosphere.
By the late 1940s, Carl Anderson (19051991) had identified positrons and
muons created by cosmic rays. Cosmic rays bombard the Earth continuously,
and they provided many challenges to scientists, who had to use balloons to lift
equipment high into the atmosphere to carry out many experiments. This was
not very convenient and what was required was the development of a more
controllable ground-based laboratory. The subsequent development of a variety
of particle accelerators has provided scientists with the tools to probe and
investigate the structure of matter.

Basic accelerator design and principles


All particle accelerators have three basic components:
a source of charged particles (ions or elementary particles)
a tube or chamber that is highly evacuated so that the particles can travel
without colliding with air molecules
a mechanism to accelerate and control the trajectory of the particles.
There are a number of common designs for particle accelerators including
electrostatic, linear, cyclotron, betatron, synchrotron and storage-ring colliders
for particle accelerators. Modern accelerator facilities often use a combination of
these; for example, a Van de Graaff generator is often used to initially accelerate
and inject a particle beam into a larger linear or synchrotron accelerator.

Electrostatic accelerators
There are two common electrostatic accelerators, which were developed in the
1930s: the CockroftWatson accelerator and the Van de Graaff accelerator. You
maybe familiar with the school version Van de Graaff generator (200kV). The
early accelerators were scaled-up versions capable of accelerating particles across
potentials of 1.5 MV. The Science Museum in Boston, Massachusetts, now
houses the (physically) largest air-insulated Van de Graaff ever built. At the top it
has two joined domes, one housing the top of the belt mechanism and the other
housing the top of the original accelerator tube. The museum now uses the
generator to produce bolts of artificial lightning. Modern Van de Graaff
accelerators are smaller and use a tandem system that can accelerate ions across
a potential of 20MV.

Linear accelerators
Linear accelerators comprise a very long straight evacuated tube that contains
a set of hollow metal drift cylinders (Figure 15.4.1). Each alternate cylinder is
electrically connected to an alternating power supply, which provides an
alternating potential difference. An ion is injected into the tube and is initially
attracted toward the first cylinder. Once the ion is in the cylinder, the electric
field drops to zero and the potential polarity is reversed, so that when the ion
emerges from the cylinder it is repelled from the first cylinder and attracted to
the second. When the ion enters the second cylinder, it enters a region of zero
electric field and drifts at a constant velocity through the cylinder. The polarity
again reverses and when the ion emerges it is repelled by the second cylinder and
286

quanta to
quarks
attracted to the third. This process is repeated for the entire journey of the ion
down the tube. The alternating potential remains at a constant frequency and,
therefore, as the ions increase in velocity, the lengths of the hollow metal drift
cylinders are increased. The Stanford Linear Accelerator Center (SLAC) houses the
most well-known and longest linear accelerator (Figure 15.4.2). It is 3.2 km long.

Cyclotron
In 1932 Ernest Lawrence (19011958) and Stanley Livingston
(19051986) developed a small, compact accelerator that
electrostatically accelerated ions or elementary particles across
a small gap between two semicircular D-shaped hollow metal
cavities called dees (see Figure 15.4.3). Once accelerated
across this gap, the particle is turned through 180 in a near
semicircular path by a constant uniform strong magnetic field.
The particle is again accelerated across the gap and again
turned through 180. The two dees are connected to a
constant high-frequency alternating power source, which
provides the electric field that accelerates the particle across
the gap. The polarity across the gap is reversed as the particles
are being turned. The electric field inside the semicircular
hollow metal dee is zero, and therefore does not affect the
path of the particle. As the velocity of the particle increases,
the radius of its orbit also increases, resulting in an outward
spiral trajectory. One of the dees has an exit point and a beam
of particles is produced. A design limit is imposed upon the
cyclotron because, if you attempt to accelerate particles
beyond energies of 20MeV, the increase in relativistic mass
causes the particles to become out of phase with the constant
high-frequency source. At the ANSTO-operated National
Medical Cyclotron at The Royal Prince Alfred Hospital,
Sydney, the particle beam is used to produce radioisotopes.

source

Figure 15.4.1

Structure of linear accelerator components

electron injector
damping rings

positron
electron

D1
+

transverse magnetic field

bending magnets

focusing
magnets

collision
detector

E
+

D2

high frequency
alternating
voltage

D1

3 km
positron source

Figure 15.4.2
a

D2

The Stanford linear accelerator


accelerates alternate bunches
of positrons (e+) and electrons
(e) to 50 GeV. Magnets
separate the particles and
bend them in arcs to produce
head-on e+e collisions.
The damping rings and
focusing magnets help bunch
and focus the beam.

ion or electron
source

accelerated
particle beam

Figure 15.4.3

(a) Schematic diagram of a cyclotron. (b) Particles are accelerated across the
gap by an electric field, so that each semicircular spiral has a larger radius.
(c) By the time the particle reaches the gap again, the voltage has reversed and
the particle is accelerated. These diagrams show the acceleration of positive ions.

287

15

The particle zoo

Van de Graaff
accelerator

deflecting
magnets

Synchrotron
target

high frequency
alternating
voltage

injection
point
deflecting
magnets

Figure 15.4.4

A Van de Graaff accelerator injects


particles into the synchrotron at high
speeds. The magnets force the particles
into a curved path. Each time the
particle passes the high-frequency
alternating voltage, it is accelerated.

+ proton source

antiproton
source

Figure 15.4.5

main ring

Using the accelerated particle beams

magnets are
coaxial with
rings

Tevatron ring

collision
detector

The Fermilab accelerator accelerates


protons in one ring and antiprotons in
the other. The beams are made to collide
with a total energy of ~2000 GeV.
Superconducting magnets force the
particles to stay in the rings.

Switzerland

Geneve
LHCB
Point 8

The modern accelerators that strive to produce the highest energy


collisions are based upon the synchrotron design (see Figure 15.4.4). Charged
particles are injected into a circular evacuated tube that is shaped into a ring.
The ring has a series of magnets whose fields can be adjusted to maintain a
circular trajectory for the accelerated particles. The ring also has one or more
locations where an adjustable high-frequency (radio frequency) alternating
power source provides a region in which the particle is accelerated. Both the
alternating power source and the magnets are adjusted to increase the speed
of the particles and maintain a circular orbit inside the evacuated tube.
Modern synchrotrons use superconducting magnets to maintain the
trajectories of the accelerated charged particles. The synchrotron at the
Fermilab Tevatron has a circumference of 6.28km (see Figure 15.4.5)
and the newly commissioned Large Hadron Collider (LHC) at CERN has
a circumference of 27km (see Figure 15.4.6). The LHC has 1232
superconducting magnets and will accelerate protons up to 10TeV and
collide them head-on, travelling in opposite directions.
The high-energy particles can either be fired at a stationery target or collided
with other high-energy particles travelling in the opposite direction. Both
collisions are governed by the laws of conservation of energy and momentum.
Fixed target collisions can involve a wide range of target materials, and they
produce secondary beams of particles that can be used in a variety of
experiments to probe the nature of matter. In the late 1970s scientists at
CERN developed a way to collide protons and antiprotonsa collision that
has nearly zero overall momentum. This allowed the total energy of the
collision to be available to produce the new varieties of particles that these
types of experiments were uncovering.

France

CERN
Point 1ATLAS

ALICE Point 2

Point 5 CMS

TI 8
LHC-B

SPS

TI 2

ATLAS
ALICE

CMS

Figure 15.4.6
288

Overall view of the LHC experiments

/LH

LEP

Figure 15.4.7

An engineer checks one of the


magnets in the tunnel of the LHC.

quanta to
quarks

tracking
device

particle
electromagnetic hadronic identification
detector
calorimeter calorimeter

photons

muon chamber
hadron calorimeter
E-M calorimeter
tracking

electrons or
positrons
muons

e+

pions or
protons
neutrons

Figure 15.4.8

The diagram shows typical interactions with sub-detectors


for some common particles.

Figure 15.4.9

Schematic design
for a typical
particle detector

Particle detectors
Particle detectors are designed to record and provide a way of seeing the
explosion of particles that can be generated as a result of the collision. Modern
particle detectors are made up of a series of layers of sub-detectors, each
specialising in identifying particular types of particles or specific properties (see
Figure 15.4.8). These sub-detectors can be classified into three main types:
Tracking devices detect and reveal the trajectory of a particle. Often a
magnetic field is used to bend the trajectory of charged particles into a curve.
The curvature provides physicists with information about the momentum of
the particle. Early detectors such as the cloud chamber and the bubble
chamber provided a direct visual record of the paths of particles. Modern
tracking devices produce electrical signals that are processed by computers to
reconstruct the path of particles. The outer layer of many detectors contain a
muon chamber that detect these weakly interacting particles, which can travel
through many metres of dense material.
Calorimeters used in detectors are of two types: the electromagnetic
calorimeter, which is positioned close to the collision, and the hadronic
calorimeter. These sub-detectors are designed to stop, absorb and measure the
energy of a particle. The electromagnetic calorimeter measures the energy of
lighter particles such as electrons and photons. Hadronic calorimeters absorb
the energy of heavier particles, which contain quarks (these types of particles
are called hadrons, and include pions, protons and neutrons). The
interactions inside both types of calorimeters often cause successive cascades
or showers of new particles and with each interaction or collision the
momentum and energy is shared between more particles, until eventually all
are slowed and stopped. To accurately measure the total energy requires the
detector to stop nearly all particles, and thus explains the huge overall size of
the detectors used. Calorimeters can stop most known particles except muons
and neutrinos.
Particle identification detectors identify the type of particle by various
techniques. Two commonly used methods to detect the nature of charged
particles are Cherenkov radiation, produced when charged particles travel
faster than the normal speed of light of that medium, and transition
radiation, produced when a charged particle crosses the boundary between
certain materials.
289

15

The particle zoo

PHYSICS FEATURE
The worlds largest
microscope

ow do we study the fundamental building


blocks of matter, and the forces that
determine how they interact with each other,
at the very tiniest of distances? Ironically, we
do it by building the worlds largest ever particle
accelerator, which acts a bit like a very
sophisticated microscope. A 100 years or so ago,
Ernest Rutherford showed us that we could learn
about what was inside an atom by bombarding it
with an energetic probe, in this case an alpha
particle, and observing what happens to it. This
principle has served us well, and by applying it
Figure 15.4.10 Jason Lee at the site of the giant ATLAS detector,
which is in its final stages of construction and testing
physicists have burrowed down into the nuclei of
atoms to find protons and neutrons, and into the
These detectors track the emerging charged particles,
protons and neutrons to find quarks and gluons.
bending them in magnetic fields to work out their charge
Many other particles, some of which can only be
and momentum from their trajectories. Further out from
made in accelerators, have turned up along the
the collision point, calorimetry is employed to absorb
way. The ones which we view, at least today, as
particles such as photons and electrons and measure
truly fundamental have been identified, and two of
their energy.
the forces of nature, the electromagnetic and the
Mounting scientific experiments on this scale takes
weak, have been unified.
the combined efforts of thousands of physicists from all
On the outskirts of Geneva, in a 27km circumference
over the world, including Australia. In Figure 15.5.10,
tunnel 50100m underground, the Large Hadron
postgraduate student Jason Lee from the University of
Collider (LHC) has the task of colliding two beams of
Sydney can be seen standing in front of the giant ATLAS
protons at very high energies, in order to allow us to take
detector in its final stages of construction. ATLAS will
the next step along our road to understanding. Using
scour the products of the proton collisions at its centre
Einsteins famous principle relating energy and mass,
to search for things such as the Higgs field, which is
the two colliding protons are destroyed and some of
believed to be involved in giving mass to the other
the energy they carry is transformed into the mass of
fundamental particles; yet-to-be discovered
new particles. We can understand why the protons
supersymmetric particles that are predicted by some
need to have large energy and momentum by recalling
new theories; and possibly even hints of extra
de Broglies idea that particles can be assigned a
dimensions. Jason completed high school in Sydney and
wavelength that is inversely proportional to their
an undergraduate degree at Oxford University before
momentum. The higher the momentum and the shorter
returning to Sydney to do his postgraduate studies. Like
the wavelength, the shorter the distances we can study.
other Australian students in experimental high-energy
Many of the particles produced in the collisions live for
physics, he spends some of his time at CERN, where he
only the tiniest fraction of a second, and their existence
can interact with physicists from all over the world and
must be inferred from the products of the collision
gain experience of the LHC project first hand. For his
(i.e. their decay). To observe all of the products of the
postgraduate studies he is investigating how well ATLAS
collisions in detail, very complex detectors the size of
can identify electrons and their antimatter partners,
several-storey buildings have been constructed.

290

quanta to
quarks

the positrons, and trying to improve the ability of the


detector to pick them up.
Electrons and positrons turn up in the expected
decay products of many of the new particles being
hunted for. An example of what a Higgs particle decay
might look like in the ATLAS detector can be found in
Figure 15.4.11. In this example, the products of the
decaying Higgs particle travel upwards in the picture and
a spray of other particles, known as a jet, travels
downwards (the yellow cone of particles that is absorbed

in the calorimeters, ending in blue and orange blobs).


The Higgs particle decays to two other short-lived
particles known as Z bosons, with one Z boson decaying
to an electron and a positron (shown in blue travelling
upwards) and the other to a muon and antimuon (shown
in red). The other particles in the picture (in yellow) are
the debris from the two protons that collided. The
human shown in the figure gives an indication of scale.
She would not be standing there if the LHC was
operating!

Figure 15.4.11 ATLAS detector schematic with a simulated detection of a Higgs boson

291

15

The particle zoo

Checkpoint 15.4
1
2
3
4

List the three basic parts of an accelerator.


Identify each of the types of accelerators and their uses.
List the types of collisions that can be used to identify information about particles.
Create a table that identifies the types of detectors that can be used and specify what they will show.

15.5 The Standard Model


Discuss the key features and
components of the Standard
Model of matter, including
quarks and leptons.

292

In the early 1930s the list of identified sub-nuclear particles comprised the
electron, the proton and the neutron; the positron and the neutrino had
been proposed by Pauli. It was not long before the muon and the positron
were identified in cloud chambers while examining cosmic showers, and by
the 1960s a hundred other particles had been identified. The physicists
were confronted with this ever-increasing zoo of particles, as the energy of
the collisions increased. They had no overall unifying theory to explain the
behaviour or adequately classify these particles. The quest to uncover
which of these particles were truly fundamental became a primary area
of research.
A few of the major discoveries that led to the development of the
Standard Model of matter are discussed here. In 1964 Murray Gell-Mann
(1929) and George Zweig (1937) proposed the existence of quarks to
explain the properties of a family of particles called hadrons. The hadrons
are further subdivided into two types: mesons, those containing two
quarks (comprising a quark and antiquark pair), and baryons those
containing three quarks. In 1967 the electroweak theory, unifying the
electromagnetic and weak nuclear force, was proposed by the collaboration
of Steven Weinberg (1933) and Sheldon Glashow (1932), and
independently by Abdus Salam (19261996). In 1969 the first evidence of
quarks was reported by Jerome Friedman (1930), Richard Taylor (1929)
and Henry Kendall (19261999). Then, during the period of 19701973,
the Standard Model was formulated. All the components of the model
except the Higgs boson were verified over the next 25 year period.
Today the Standard Model of Matter provides a mechanism to classify
and explain the nature of matter. It includes three of the four fundamental
The model has three main
forces but excludes gravity (Figure 15.5.1).
components: quarks, leptons and force carriers. The experiments at
Fermilab and the new CERN LHC to search for the Higgs boson will either
confirm the Standard Models validity or send the experimentalists and
theorists back to formulate a new or revised model. Either way, the major
challenge facing particle physicists is the need to develop a grand unified
theory that will unite all four of the fundamental forces.

quanta to
quarks

Quarks
There are six varieties (commonly called flavours) of quarks and in all there
are 12 types of quarks. The first generation of quarks are named up and down,
the second generation are charm and strange, and the third are top and bottom.
Each quark is considered to be a fundamental point-like particle that carries the
properties of mass, charge and colour. The electric charge of a quark is fractional
and is either 1/3e or +2/3e, and they possess a colour charge that can be either
red, blue or green. Quarks also have antimatter counterparts that possess the
same magnitude of electric charge with the opposite sign, and colours that are
antired, antiblue and antigreen. Quarks with different colour charge attract each
other, as do quarkantiquark pairs of the same colouranticolour (e.g. red and
antired). The colour charge is associated with the strong force and is constantly
changing as they interact with gluons.
Quarks do not exist as bare entities, and only form composite particles called
hadrons. Within this classification, quarks can either pair with an antiquark to
form a meson, or three quarks can combine to form a baryon. All hadrons have
a total electric charge of 1e, and the total colour charge will sum to white
(achieved by the quarks in mesons having a colour and an anticolour, or in
baryons by the combination of all the quarks having different colours).
atom

neutron

electron

quarks

matter

proton

nucleus
Matter
particles

These
particles
transmit the
four
fundamental
forces
of nature
although
gravitons
have so far
not been
discovered

1st FAMILY

Force
particles

QUARKS

Electron neutrino
Particle with no electric charge,
and possibly no mass; billions
fly through your body every second

Up
Has an electric charge of
plus two-thirds; protons contain two,
neutrons contain one

Down
Has an electric charge of minus
one-third; protons contain one,
neutrons contain two

2nd FAMILY

These
particles
existed just
after the
Big Bang.
Now they are
found only
in cosmic
rays and
accelerators

LEPTONS
Electron
Responsible for electricity
and chemical reactions:
it has a charge of 1
Muon
A heavier relative of the
electron; it lives for twomillionths of a second

Muon neutrino
Created along with muons
when some particles decay

Charm
A heavier relative of the up;
found in 1974

Strange
A heavier relative of the down;
found in 1964

Tau
Heavier still; it is extremely
unstable. It was discovered
in 1975

Tau neutrino
Recently discovered

Top
Heavier still

Bottom
Heavier still; measuring
bottom quarks is an important
test of electroweak theory

Photons
Particles that
make up light;
they carry the
electromagnetic
force

Intermediate
vector bosons
Carriers of the
weak force

3rd FAMILY

All ordinary
particles
belong to
this group

Gluons
Carriers of the
strong force
between quarks

Felt by:
quarks
The explosive release of nuclear energy
is the result of the strong force

Figure 15.5.1

W
Felt by:
quarks and charged
leptons

Electricity, magnetism and chemistry are


all the results of electromagnetic force

Z0

Gravitons
Carriers of
gravity

W+
Felt by:
quarks and leptons

Some forms of radioactivity are the


result of the weak force

Felt by:
all particles with mass
All the weight we experience is the
result of gravitational force

A tabular presentation of the three main components of the Standard Model and the three generations of particles classified
as quarks, as well as leptons and a description of the gauge bosons that mediate the four forces of nature
293

15

The particle zoo

Table 15.5.1 Properties of quarks


Symbol

Charge

Table 15.5.2 Properties of hadrons


Mass

Colour charge

Up

+2/3e

Red, blue or green

Down

1/3e

Red, blue or green

Charm

+2/3e

Red, blue or green

Strange

1/3e

Red, blue or green

Top

+2/3e

Red, blue or green

Bottom

1/3e

Red, blue or green

Hadron

Example

Meson
(2 quarks)

Positive pion

Neutral kaon
Proton
Neutron
Sigma plus

Baryon
(3 quarks)

Symbol

K
p
n
+

Constituent
quarks

ud
ds

uud
udd
uus

Note: ud is an upantidown pair.

Quark road rules

hen remembering what quarks are in protons and neutrons, think of


overtaking lane signs. The passing lane is a protontwo up quarks
and one down quark. No overtaking is a neutronone up quark and two
down quarks.
Passing

No passing

proton
uud

neutron
udd

Leptons
There are six varieties and 12 types of leptons. The most commonly known is the
electron. All leptons appear to be fundamental point-like particles. The electron,
the heavier muon and the massive tau all have an electric charge of 1e and
their antimatter counterparts have a charge of +1e. Each of these leptons has
an associated neutrino that is named accordingly; for example, the electron (e)
has an electron neutrino (e).

Checkpoint 15.5
1
2

294

Draw a chart that shows the classes and subclasses of particles, starting with hadrons.
List the properties of quarks, hadrons and leptons.

PRACTICAL EXPERIENCES

quanta to
quarks

CHAPTER 15

This is a starting point to get you thinking about the mandatory practical
experiences outlined in the syllabus. For detailed instructions and advice, use
in2 Physics @ HSC Activity Manual.

Activity 15.1: The Manhattan Project


Research the Manhattan Project and the events that surrounded it. Assess the impact
it has had on society then and now.
Discussion questions
1 Outline the discoveries that were made during this project.
2 Describe the effect of the outcome of the project on the war.
3 Assess the significance to society today.

Gather, process and analyse


information to assess the
significance of the Manhattan
Project to society.

Activity 15.2: Isotopes in society


Research a series of isotopes that have been developed for medicine, agriculture and
engineering purposes. List their advantages over prior methods.
Discussion questions
1 Name an isotope that is used in:
a medicine
b agriculture
c engineering.
2 Describe the purpose of one of these isotopes and how it is used.
3 Explain how the use of that isotope has replaced previous technology.

Identify data sources, and


gather, process, and analyse
information to describe the
use of:
a named isotope in medicine
a named isotope in
agriculture
a named isotope in
engineering.

295

15


296

The particle zoo

Chapter summary

The Manhattan Project was the code name for the


USAs top-secret nuclear bomb research project.
Nuclear fission releases energy that manifests itself as
heat and high-energy gamma radiation.
Fission reactors are carefully designed to control the rate
of fission (splitting) of large nuclei to ensure their safe
operation.
Uranium-fuelled nuclear fission reactors, comprise six
key components:
Fuel rods are tubes filled with pellets of uranium
oxide and are located in the centre of the reactor
known as the core.
The core of the reactor houses the fuel rods, the
control rods, a coolant system and a moderator
material (which sometimes also acts as a coolant).
Moderators slow down neutrons and so improve the
chance of the neutrons being captured by a nucleus.
Commonly used moderators include ordinary water,
heavy water and graphite.
Control rods absorb neutrons and are adjusted so
that the chain reaction proceeds at a constant rate.
The rods are usually made from boron or cadmium
and are located between the fuel rods. If the reactor
needs to be shut down for service or in an
emergency, the control rods are fully inserted into
the core.
The coolant transfers the heat generated by the
reactor away from the core. In nuclear power
stations the heat is used to make steam, which
powers a turbine and generator.
Radiation shielding protects people and the
environment and prolongs the working life of the
reactor facility. Lead and graphite are used to absorb
and reflect the large quantities of gamma radiation
and neutrons emitted by the core.
Radioisotopes are atoms that have an unstable ratio of
protons to neutrons. They will decay via alpha or beta
decay to attain a more stable configuration; some may
also emit gamma radiation.
Radioisotopes occur naturally, or they can be produced
by changing the ratio of nucleons in the nuclei of a
target material in a nuclear reactor or a particle
accelerator.

Radioisotopes have a specific half-life, which describes


the time it takes half of the radioisotope sample to
decay.
Radiopharmaceuticals can be classified as diagnostic or
therapeutic.
Diagnostic radiopharmaceuticals are used to assess
the functioning of organs including the lungs, heart,
liver and brain; identify bone fractures not visible in
X-rays; and assess the flow of fluids such as blood.
Therapeutic radioisotopes generally contain
radioisotopes with longer half-lives, and once
absorbed by the specific tissue or organ will deliver
a target dose of radiation.
Radioisotopes have applications as diverse as checking
the structural integrity of bridges, determining wear in
engine components, examining welds, imaging internal
structures in jet aircraft engines, and thickness control.
Particle accelerators provide scientists with the tools to
probe and investigate the structure of matter.
Particle accelerators have three basic components:
a source of charged particles (ions or elementary
particles)
a tube or chamber that is highly evacuated so that
the particles can travel without colliding with air
molecules
a mechanism to accelerate and control the trajectory
of the particles.
Common particle accelerator designs include
electrostatic, linear, cyclotron, betatron, synchrotron
and storage ring colliders.
Particle detectors are designed to record and provide
a visualisation of the explosion of particles that can be
generated as a result of the collision.
The four fundamental forces in nature are gravity, the
electromagnetic force, the strong nuclear force and
the weak nuclear force.
The Standard Model includes three of the four
fundamental forces; it excludes gravity. The model has
three main components: quarks, leptons and bosons
(the force carriers).

Review questions

quanta to
quarks

Physically speaking
Very quarky
1
3

6
7
8
9

10

11

12
13

14

15

16
17

18

19

Across
1 Discovered the neutron

Down
2 Type of accelerator

6 Slows things down

3 Family name for the carrier of all the forces

9 Made up from any three quarks

4 The anti one often accompanies the beta particle

10 Atom smasher

5 A project

12 Planck has one

7 You will find them in a nucleus

15 Same number of protons

8 An electron is one

17 A type of scattering

11 Lived 18711937

18 Keeps things cool

12 The mass you require to start a chain reaction

19 Type of reactor

13 A constant R
14 They are the reason for the strong force
16 Fermis first name

297

15

The particle zoo

Reviewing

10 Discuss the purpose of accelerating particles and


colliding them with a target (or their antiparticle) in
terms of:
a the de Broglie wavelength
b Einsteins equation E = mc2
c probing matter
d producing new types of particles.

1 Recall three of Enrico Fermis contributions to physics.


2 Define the term critical mass.
Explain the basic principles
of a fission reactor.

3 Describe the roles of the following components in


a fission nuclear reactor.
a fuel
b control rods
c moderator
d coolant
e radiation shields

4 Explain using a schematic sketch the basic processes


involved in a nuclear-powered electricity station.

5 Name two materials that could be used in a fission


reactor as:
a a moderator
b control rods
c shielding.

6 Define the term radioisotope.


Describe some medical and industrial
applications of radioisotopes.

7 Name and describe the use of a radioisotope used in:


a medicine
b agriculture
c engineering.
Describe how neutron scattering is used as a
probe by referring to the properties of neutrons.

8 Neutron scattering is used to probe structures of


many materials. Explain how each of the following
properties is useful.
a the wave nature of the neutron
b the magnetic moment of the neutron
c strong interaction of the neutron with nuclei
d the range of energies the neutron can possess

9 Draw, label and describe the main features of the

Re

iew

298

Q uesti o

following particle accelerators.


a Van de Graaff
b linear accelerator
c cyclotron
d synchrotron

11 Identify the ways by which physicists continue to


develop their understanding of matter using
accelerators.
Discuss the key features and components
of the Standard Model of matter, including
quarks and leptons.

12
13
14
15

Name the six quarks.


Name three leptons.
Compare fermions and bosons.
Recall the constituents of these particles.
a meson
b baryon
c proton
d neutron
e pion

16 Complete the following table.


Types of boson

Role in Standard Model

Photon
Strong force
Graviton

17 Describe the purpose of the Manhattan Project.


18 Describe the long-term ramifications of the Manhattan
Project on society.

19 The Standard Model classifies the constituents of


matter into three families.
a Name the three families.
b Describe the properties of each family.

20 Explain why the top quark was the last of the six
quarks to be found.

21 Recall why quarks are never found as individual


free particles.

quanta to
quarks

PHYSICS FOCUS
Linking the very big with
the very small
The very big and the very small are inextricably linked
as cosmology and particle physics enter a new era of
research. Many of the recent developments in
cosmology have depended on advances in high-energy
physics.
1 Explain why it is necessary to accelerate particles
to extremely high velocities.
2 Describe how high-energy accelerators such as the
Large Hadron Collider can test aspects of theories
such as the Big Bang.

H1. Evaluates how major advances


in scientific understanding and
technology have changed the direction
or nature of scientific thinking
Identify ways by which physicists continue to
develop their understanding of matter, using
accelerators as a probe to investigate the structure
of matter

Extension
3 Research how high-energy physics was and is still
carried out using cosmic rays.

299

The review contains questions that address the key concepts


developed in this module and will assist you to prepare for the
HSC Physics examination. Please note that the questions on the
HSC examination that address the option modules are different in
structure and format from those for the core modules. Past exam
papers can be found on the Board of Studies NSW website.

Multiple choice

A nuclear chain reaction requires certain conditions.


Which of the following statements provides the best
description?
A Fuel that is capable of fission
B An amount of fuel that produces sufficient
neutrons to cause new fission
C Neutrons with an energy that allows them to be
absorbed by other fuel nuclei.
D All of the above

Today the Standard Model of Matter provides a


mechanism to classify and explain the nature of
matter. Which of the following statements is false in
relation to the model?
A Quarks are the subatomic particles that make up
all matter.
B The model includes three of the four fundamental
forces but excludes gravity.
C The model has three main components: quarks,
leptons and force carriers.
D Hadrons are subdivided into two types: mesons
and baryons.

(1 mark each)
1 Which of the following lists only contains people who
provided science with atomic models in the period
up until 1913?
A Bohr, Democritus, Heisenberg, Schrodinger
B Aristotle, Bohr, Pauli, Rutherford
C Bohr, Dalton, Rutherford, Thomson
D Bohr, Einstein, Rutherford, Schrodinger

Which of the following scientists provided an


explanation for the spectrum of beta-particle
energies observed, and proposed that each electron
in an atom could be described by four quantum
numbers and a system that provided an explanation
for the structure of the period table?
A Erwin Schrodinger
B Werner Heisenberg
C Wolfgang Pauli
D Louis de Broglie

In 1924, de Broglie proposed the concept of matter


waves. Which of the following statements is false in
relation to de Broglies ideas at this time?
A All particles of matter would have an associated
wavelength.
B An electron in orbit around the nucleus would
follow a wavey path.
C The majority of the scientific community did not
take de Broglies ideas seriously.
D There was no experimental evidence to support
de Broglies ideas.

300

quanta to
quarks

Short response
1

The diagram below shows the kinetic energy


distribution of electrons for the beta decay of
bismuth-214.

In 1913 Niels Bohr published a paper On the


Constitution of Atoms and Molecules, which
described the planetary model of a hydrogen atom.
a In terms of the RutherfordBohr model, present
a diagrammatic representation of how ONE of
the Balmer series spectral lines is produced.
(2 marks)
b Calculate the wavelength of the electromagnetic
radiation emitted from a hydrogen atom when an
electron drops from the third shell to the ground
state. (3 marks)
c Outline the ONE spectral observation that the
RutherfordBohr model could not completely
explain. (1 mark)

Account for the energy distribution of the electrons

State the two laws of physics used by James Chadwick


when he discovered the neutron. (1 mark)

The Standard Model has the components shown


in the table.

Quark

Up

Charge

Mass
GeV/c2

Down

Strange

Charm

Bottom

KE

emitted in beta decay. (3 marks)

Top

2
+ e
3

1
e
3

1
e
3

2
+ e
3

1
e
3

2
+ e
3

0.005

0.01

0.2

1.5

4.7

180

a Identify ONE way in which physicists have


developed their understanding of matter using
accelerators. (2 marks)
b Compare neutrons and protons in terms of their
constituents. (2 marks)

Naturally occurring polonium-218 emits an alpha


particle to produce a new element X. This new
element emits a beta particle to become
bismuth-214.
Po X + +
a Identify the new element X. (1 mark)
b Describe, using an equation, the decay of
element X to produce bismuth-214. (2marks)
c Calculate the number of protons and neutrons in
element X. (1 mark)

Extended response
6

You have gathered, processed and analysed


information to assess the significance of the
Manhattan Project.
a Describe how you ensured that the information
you gathered was reliable. (2marks)
b Assess the impact the work carried out at
Los Alamos has had on society and the
environment. (5 marks)

301

5
Context

Figure 16.0.1

This image shows the


pattern of brain tissue loss
in methamphetamine users,
relative to healthy adults,
mapped using MRI scans.

medical
physics
The human body is a complex machine and many things can go wrong with it.
However, unlike a machine, we cannot do a total shutdown to locate and correct a
problem. Most of the functions of our body need to continue working when we are
looking for a problem and when we are trying to fix it. This is where physics comes
to the aid of modern medicine by providing the tools to investigate and, in some
cases, repair problems in the living body.
We see using light, and optical-fibre technology offers a minimally invasive way
of looking inside the body to actually see what may have gone wrong. Listening is also
an important way for doctors to find evidence of problems but, remarkably, we can
now use sound to look deep within the body without ever breaching the surface.
Although they present some potential damage to human tissues, X-rays can also
give us vital information of structural problems below the surface. For more than a
century, X-rays offered the only non-invasive way to see inside the body. Now, when
combined with enhanced computer technologies, X-rays provide powerful tools for
creating images of the structures within the body.
Modern physics also offers magnetic resonance imaging (MRI) as a powerful
method to investigate both the form and function of individual tissues. Carefully chosen
radioisotopes offer similar possibilities, because they can be used
to identify where atoms are accumulating within our
bodies. Other radioisotopes provide a therapeutic
tool to help combat certain problems.
None of these techniques will reveal
everything, but physics has provided
doctors with a range of diagnostic
and therapeutic tools that earlier
generations could never have
imagined.

Areas of greatest loss


emotion, reward (limbic system)
memory (hippocampus)
0% Loss

302

3% Loss

5% Loss

Figure 16.0.2

A drawing of the human


brain by Vesalius

Inquiry Activity
What makes you tick?
The ancient Egyptians knew that the heart was centre of blood supply in the
human body. Its pretty obvious when you look inside the body. However, just how it
worked remained a mystery, because looking inside dead bodies was not permitted
in many societies. Great advances were made by Andreas Versalius (15141564)
in the 16th century precisely because dissection was permitted in Italy in some
circumstances. In 1543 he published De Humani Corporis Fabrica (On the Fabric
of the Human Body), which marks the foundation of the scientific study of
human anatomy.
The function of many of the organs of the human body has been poorly
understood until recentlynone more so than the brain. However, modern imaging
techniques such as PET and MRI are slowly revealing the operation of the brain
by uncovering what parts are involved in different actions or affected by
different diseases.
In this exercise you should seek out reliable sources of information on the web
to investigate the difference between modern structural and functional imaging
of the brain.
1 Define what is meant by the structural and functional imaging.
2 How would you classify the images in Figures 16.0.1 and 16.0.2are they
structural or functional?
3 What imaging techniques allow structural differences to be seen in the brain?
4 What diseases might be revealed by structural images of the brain?
5 What imaging techniques allow functional differences to be seen in the brain?
6 Outline one study of the brain that might be conducted using functional images.
7 How reliable are your sources? Why do you expect them to be reliable?

303

16

Imaging with
ultrasound
See like a bat

transducer, sonogram, Doppler


ultrasound, piezoelectric transducer,
piezoelectric effect, convex array
transducer, sector scan, linear array
transducers, resolution, acoustic
impedance, impedance matching,
acoustic coupling, A-mode scan,
B-mode scan, M-mode scan, 2Dreal-time
scan, 3D ultrasound, 4D ultrasound,
bone density, dual X-ray absorptiometry,
Doppler ultrasound, colour Doppler
imaging, Doppler effect, echocardiography

Bats, dolphins and some birds have evolved the ability to produce
and sense sound waves that are reflected from objects in their
surroundings, enabling them to hunt prey and to navigate in the dark.
The principle used by these animals has been applied to radar
(using radio waves) and sonar (using sound waves).
Since the 1960s, the principles of sonar have also been applied
for medical purposes. Ultrasound imaging is a medical procedure
used to produce pictures of the inside of the body by using sound
waves with a frequency much higher than that audible to humans.

16.1 What is ultrasound?


low
high
low
high
pressure pressure pressure pressure

compression

Air pressure

high

Distance
low

Figure 16.1.1

304

atmospheric
pressure

rarefaction

A longitudinal wave with compression and


rarefaction pressure variations graphed
versus distance

Sound is a vibration in an elastic, mechanical medium that can be


solid, liquid or gas. A sound wave is made up of periodic pressure
variations in the medium through which it travels. It propagates
as a longitudinal or pressure wave of alternating compressions and
rarefactions (see Figure 16.1.1 and in2 Physics @ Preliminary
section 5.5).
The speed of sound is different in different media, as shown in
Table 16.1.1.
As well as speed, you should recall three other important
properties of sound: frequency (the number of vibrations per
second), wavelength (the distance between adjacent compressions)
and amplitude (the maximum displacement of the particles in the
medium from their equilibrium position). The greater the amplitude
of a sound wave, the more energy the wave carries.
Humans can hear sounds with frequencies between about 20 Hz
and 20 kHz. Ultrasound is any sound that has a frequency greater
than this upper limit. Other mammals such as bats and dolphins use

medical
physics
ultrasound waves with frequencies as high as 125 kHz for navigating and
sonar visualisation.
Ultrasound used for medical purposes has frequencies that range
from 500kHz to 30MHz, but for most imaging applications, the
ultrasound used is in the range from 3.5 MHz to about 10 MHz. The
exception is ultrasound used for imaging blood vessels in the body. Tiny
probes on the end of a catheter (tube) are inserted into the blood vessels and
operate at frequencies up to 30MHz.
The choice of ultrasound frequency that is most suitable for
producing a medical image is a compromise between conflicting criteria.
Higher frequency ultrasound produces images with a better resolution of
detail, but they have poorer penetration through tissues. Imaging depth into
tissue is limited by attenuation (absorption) of the ultrasound waves, and
this increases as the frequency is increased.

Checkpoint 16.1
1
2
3

Table 16.1.1 Velocity of sound in air and


ultrasound in different human tissues

Velocity of sound (m s1)

Material
Air
Fat
Water
Brain
Liver
Kidney
Blood
Muscle
Lens of eye
Skull bone
Average soft tissue

330
1450
1480
1540
1550
1560
1570
1580
1620
4080
1540

Identify the differences


between ultrasound and sound
in normal hearing range.

Explain how sound is made by moving air particles.


Outline the difference between sound and ultrasound.
List the advantages of low frequency and high frequency ultrasound.

16.2 Principles of ultrasound imaging


In ultrasound imaging, ultrasound waves are emitted and detected by
a device called a transducer. A transducer is any device that converts energy
from one form to another. For example, a loudspeaker is a transducer that
converts electrical energy to sound energy; a microphone does the reverse. The
ultrasound transducer combines the functions of a speaker and a microphone.
It can transmit and detect ultrasound.
The ultrasound transducer emits pulses of ultrasound for only ~1% of the
time and detects for ~99% of the time. The transducer detects ultrasound waves
that are reflected from boundaries between different tissues in the body. The
amount of reflection depends on differences in the properties of the two tissues
at the boundary.
The intensity of the reflected wave returned to the transducer is
determined by:
1 the difference in the acoustic properties of the tissues at the boundary
the greater the difference, the more energy is reflected
2 the characteristics of the intervening tissuesome tissues absorb more energy
than others
3 the angle at which the ultrasound is reflected from the boundary between
two tissuesthe production of an image is easiest when the reflection occurs
perpendicular to the tissue boundary.

PRACTICAL
EXPERIENCES
Activity 16.1

Activity Manual, Page


127

A strong reflection from a tissue boundary in the body produces a sharply


defined image. A poor image will be produced if the reflection is weak or if
tissues in the body absorb the energy before it returns to the transducer.
305

16

Seeing with
ultrasound

To produce an ultrasound image or sonogram, a sonographer applies a gel


to the area of the patients skin overlying the region to be imaged and the
transducer is moved across the skin. For example, to obtain images of the heart,
the transducer is placed in contact with the chest.
A computer processes information from the reflected waves received by the
ultrasound transducer to produce real-time images, which are displayed on a
screen (Figures 16.2.1
and 16.2.2).
The patient does not feel any discomfort. Because the images are produced
while the procedure is being done, the sonographer can manipulate the
transducer to obtain the best images for the doctor to examine. A variety of
display
different types of images can be produced, depending on the way the data were
collected and how they were processed (Figures 16.2.3 and 16.2.4).
disk
Ultrasound imaging is one of the safest and least
storage
keyboard/
CPU
expensive medical imaging technologies. It is useful for
cursor
producing images of soft tissues and organs. Two major
printer
advantages of ultrasound imaging are that it is non-invasive
Transducer
and it does not involve use of ionising radiation (as in
pulse controls
transducer
X-rays) and it is therefore very safe to patients. More
frequency
duration
invasive procedures carry the risk of infection, and ionising
scan mode
sound backing
radiation damages DNA.
Ultrasound imaging is commonly used to produce
PZT crystals
images of a developing foetus during pregnancy, where its
use can reveal foetal development and movement.
Ultrasound imaging is also used to produce images of the
kidneys, liver, pancreas and heart. Doppler ultrasound
imaging is a special type of imaging used to measure
blood flow rate to diagnose heart disease and blocked
blood vessels.
Figure 16.2.1 Principles of using ultrasound as a diagnostic tool

Figure 16.2.2

306

An ultrasound examination of a pregnant woman.


Note the transducer in the technicians hand and
in contact with the patients abdomen.

Figure 16.2.3

Colour ultrasound of a kidney

medical
physics

PHYSICS FEATURE

3. Applications and uses of physics

Obstetrics

robably the most well-known use of medical


ultrasound imaging is in obstetrics. It began in
1966, but now most women have ultrasound imaging
at some time during their pregnancy. Among the
things that can be determined using ultrasound are
the number, position and size of babies, if a babys
internal organs are growing normally and how much
fluid is around the baby. Measuring the head size is
used to help determine the age of the foetus. Figure
16.2.4 illustrates ultrasound imaging of a foetus.
Some doctors have expressed possible concerns
about the safety of using ultrasound in making threedimensional images (movies) of foetuses simply so
parents can have images of the baby before it is born.
Although the risk is low, they argue that if the
procedure is not necessary for a medical diagnosis,
it should be avoided.

4. Implications of physics for society


and the environment

Figure 16.2.4

Ultrasound of a foetus

Ultrasound can be classified as low or high intensity. For sound, intensity


corresponds to the loudness of a sound, and it is a measure of the amplitude and
the energy of a wave. Low-intensity ultrasound has a minimal effect on tissues as
it passes through them. High-intensity ultrasound heats the tissues it passes
through.
The intensity of ultrasound used to produce images is kept as low as possible
to minimise the risk to the patient. Low-intensity ultrasound is considered safe
enough to produce images during pregnancy and to produce images of vital
organs such as the heart. The use of ultrasound to produce images is referred to
as a diagnostic application.
High-intensity therapeutic ultrasound, on the other hand, affects the medium
and can be used to heat injured muscles or to disintegrate kidney stones.

Limitations of ultrasound imaging


High-frequency sound waves cannot effectively penetrate bone or air, so
ultrasound cannot be used to produce images of parts of the body if there is bone
or air between the ultrasound transducer and the body part of interest. As a
result, ultrasound cannot image the adult brain and parts of the body obscured
by gas in the intestines, and produces poor quality images of the lungs.
Ultrasound can pass through parts of the skeleton of a foetus before the bone
calcifies and hardens.

307

16

Seeing with
ultrasound

Table 16.2.1 Advantages and disadvantages of ultrasound


Advantages

Disadvantages

It is non-invasive and does not require surgical procedures.

Many reflections occur within the body and therefore good imaging
is operator dependent.
An ultrasound image is harder to interpret than an X-ray image.

Patients can be examined without sedation, and relatively quickly


and conveniently.
Since sound is low energy and doesnt ionise the tissue, it does not
damage DNA, cells and tissues.
It is relatively cheap compared with other scanning technologies.

It is difficult to produce clear images in obese patients because of


attenuation and reflection from fat.
The presence of air and bone pose problems because their acoustic
impedances are so different from those of soft tissue.

Checkpoint 16.2
1
2
3
4

Explain the purpose of the transducer in ultrasound imaging.


Define what is meant by intensity of ultrasound imaging.
Explain how high frequency sound waves can make images of internal organs.
State the difference in the effects of low- and high-intensity ultrasound.

16.3 Piezoelectric transducers


Describe the piezoelectric
effect and the effect of using
an alternating potential
difference with a piezoelectric
crystal.

force
+ voltage

voltage
force

Figure 16.3.1

Apply force to a piezoelectric


crystal and it will produce a
voltage; apply a voltage and
the crystal will expand (or
contract) slightly.

Ultrasound scanners use a piezoelelectric transducer made from a crystal


that responds to an oscillating voltage placed across it by oscillating slightly in
size. The oscillating crystal produces a sound wave. It also works in reverse,
converting the sound energy into electrical energy. The process is called the
piezoelectric effect, and was discovered in 1880 by Pierre Curie (18591906)
and his brother Jacques (18561941).
A ceramic called leadzirconatetitanate (PZT) is commonly used to make
piezoelectric transducers. This material responds to relatively small voltages or
forces; that is, it has high electromechanical efficiency. The properties of PZT
can be altered by modifying the ratio of zirconium to titanium, or by adding
small amounts of other materials such as lanthanum. Figure 16.3.2 shows how
PZT rods are embedded in plastic, which is then used to construct different
types of transducers for ultrasound machines.
In ultrasound imaging, a convex array transducer is used to produce
a sector scan. The scanning surface of these transducers is curved outwards
a

b
PZT rods

epoxy resin
matrix

Figure 16.3.2
308

(a) PZT rods are embedded into plastic to construct transducers of various
shapes, such as (b) a convex array transducer.

medical
physics
(convex), resulting in a divergent beam (Figure 16.3.3). This is different from
earlier linear array transducers, which had parallel beams.
Convex array transducers use as many as 512 piezoelectric elements to display
a wedge or pie-shaped image. The size of the area of skin through which the
ultrasound enters the body (the acoustic window) depends on the curvature of
the transducer surface. There is generally a narrow window at the skin surface,
while still providing a wide angle deeper beneath the skin (Figure 16.3.4). The
narrow field of view close to the transducer makes structures near the transducer
difficult to view.
The smallest size of objects that can be imaged (resolution) using
ultrasound depends on the wavelength (or frequency). All waves have the property
of being diffracted by objects; that is, their paths are bent around the object.
After passing around the object, the paths of the waves recombine and undergo
interference (see in2 Physics @ Preliminary section 7.4), which tends to smudge
the reflection of the object.
If the size of the object is approximately one wavelength or less, the smudging
becomes so significant that the individual object is not visible, limiting the ability
to resolve fine detail. This diffraction limit applies to all waves. In ultrasound,
this limits resolution both in the direction along the beam (axial) and across the
beam (lateral) and is summarised in the Table 16.3.1.
Low frequency waves are more effective in penetrating human tissue. This
results in a dilemma for ultrasound imaging. In order to produce a clear image
of an object deep in the body, low frequency waves should be chosen over high
frequency waves. However, to obtain a clearer image of smaller structures,
a high frequency needs to be used. Clearly the final choice is a compromise.

data processing
computer
convex
array

organs
sector scan

Figure 16.3.3

monitor

A convex array transducer

Table 16.3.1 Frequency and resolution for

Figure 16.3.4

linear array transducers with


parallel beams

MHz
3.0
4.0
5.0
7.5
10.0

Axial
resolution
1.1
0.8
0.6
0.4
0.3

mm
mm
mm
mm
mm

lateral
resolution
2.8
1.5
1.2
1.0
1.0

mm
mm
mm
mm
mm

The head of a foetus in the womb.


Note the small acoustic window
the area through which the ultrasound
enters the body at the top of the image,
which is the abdomen of the mother.

Checkpoint 16.3
1
2
3
4

Define the term transducer.


Outline the energy transformations that are happening in an ultrasound transducer.
Define the piezoelectric effect and describe how it is useful in ultrasound imaging.
Recall the advantages of convex sector systems.
309

16

Seeing with
ultrasound

16.4 Acoustic impedance


Define acoustic impedance:


Z =
and identify that different
materials have different
acoustic impedances.
Solve problems and analyse
information using:
Z =

The ability of a medium to transmit sound varies between materials. Even for a
The acoustic
single material, it also varies with the frequency of the sound.
impedance describes how readily a specific sound frequency will pass through
a material. It is defined as the product of the density (in units of kg m3) of the
material and the acoustic velocity u (in m s1) in the material:
Z therefore has units of kg m

Z =
s , which is called a rayl.

2 1

Worked example
Question
Bone has a density of 2 103 kg m3. The speed of sound in bone is 4080 m s1.
Calculate the acoustic impedance of bone.
= 2 103 kg m3
= 4080 m s1

Solution
Z = = 2 103 kg m3 4080 m s1
Z = 8.16 106 rayl

Acoustic impedance differs for different tissues in the body because of


differences in their densities and the speed of ultrasound in the tissue. Acoustic
impedance can be used to predict the proportion of the energy of an ultrasound
wave that is reflected from a boundary between two tissues.

Reflection of ultrasound at tissue boundaries


Define the ratio of reflected
to initial intensity as:
2

Ir Z 2 Z1
=
Io Z + Z 2
2
1
Identify that the greater the
difference in acoustic
impedance between two
materials the greater the
reflected proportion of the
incident pulse.

When ultrasound waves meet a boundary between tissues with different


acoustic impedances, some of the ultrasound energy (intensity) will be reflected
and some will be transmitted. Conservation of energy requires that the sum of
the reflected and transmitted energies must be equal to the original energy
incident on the boundary. Note that for any reflection to occur, the acoustic
impedances of the two media Z1 and Z2 must be different. A large difference in
impedance results in a large reflection.
The incident ultrasound intensity can be represented by Io, with the reflected
intensity Ir and the transmitted intensity It, as illustrated in Figure 16.4.1. At a
tissue boundary, the ratio of the reflected intensity of ultrasound to the original
intensity of the ultrasound is equal to the ratio of the square of the difference
of the acoustic impedances to the square of the sum of the acoustic impedances,
as represented by the equation:
2
I r Z2 Z1
=
Io Z + Z 2
2
1
IO
Ir
Z1 = 11

310

It = IO Ir

Z2 = 22

Figure 16.4.1

Reflection occurs at a
boundary between two
materials with different
acoustic impedance.

medical
physics
Table 16.4.1 Acoustic impedance of ultrasound in human tissue
Density
(kg m3)

Ultrasound velocity
(m s1)

1000

1540

1.54 106

920

1450

1.33 106

Lens of eye

1100

1620

1.78 106

White matter (brain)

1040

1541

1.60 106

Grey matter (brain)

1040

1541

1.60 106

Muscle

1040

1585

1.65 106

Kidney

1040

1561

1.62 106

Blood

1060

1570

1.66 106

Liver

1065

1549

1.65 106

Bone (other than skull)

1810

4080

7.38 106

Skull bone

1910

4080

7.79 106

Substance
Water
Fat

Impedance
(rayl)

Identify that different materials


have different acoustic
impedances.

Worked examples
Question
Calculate the percentage of ultrasound intensity reflected at a boundary between fat and muscle.
Fat:

Z1 = 1.334 106 rayl

Muscle:

Z2 = 1.648 106 rayl

Solution

I r Z 2 Z 1
[(1.648 106 ) (1.334 106 )]2
=
= 0.011
=
Io Z + Z 2 [(1.648 106 ) + (1.334 106 )]2

2 1
Hence only 0.011 of the incident intensity is reflected. This is 1.1% of the incoming intensity.

Solve problems and analyse


information using:
2

Ir Z 2 Z1
=
Io Z + Z 2
2
1

Question
Calculate the percentage of ultrasound intensity reflected at a boundary between air and muscle.
Air:

Z1 = 0.0004 106 rayl

Muscle:

Z2 = 1.648 106 rayl

Solution

(
(

) (
) (

6
6
2
I r Z 2 Z 1 1.648 10 0.0004 10
=
=
I o Z + Z 2 1.648 10 6 + 0.0004 10 6
2 1

) 2 = 0.999
) 2

Hence 99.9% of the incoming intensity is reflected. Since intensity is proportional to energy,
we can say 99.9% of the energy is reflected.

Bone has a much higher acoustic impedance than other tissues, so most of the
ultrasound energy that encounters bone is reflected. For this reason, ultrasound
cannot image the internal structure of bone or a brain inside an adult skull. Some
ultrasound penetrates the skull of a foetus, because the bone has not yet calcified
it is cartilage, similar to the tissue that gives ears their semi-rigid nature.
If ultrasound travelling from air or other gas strikes a soft-tissue boundary of
any type, then most of the ultrasound energy is reflected. For this reason, when
an ultrasound is being carried out, a gel is placed on the skin and the ultrasound
probe is moved over this gel. The most important function of the gel is to exclude
any air from the region between the transducer and the skin. The gel has an acoustic

Describe how the principles


of acoustic impedance and
reflection and refraction are
applied to ultrasound.

311

16

Seeing with
ultrasound

Solve problems and analyse


information to calculate the
acoustic impedance of a range
of materials, including bone,
muscle, soft tissue, fat, blood
and air and explain the types of
tissues that ultrasound can be
used to examine.

impedance similar to that of human tissue. The term used to describe this
Impedance
process is impedance matching or acoustic coupling.
matching between two media results in most of the energy being transmitted
through the interface, with almost no reflection.
A further complication arises at the boundary between two different tissues,
because ultrasound is also refracted at a boundary (see section 18.1 and in2
Physics @ Preliminary section 8.3). This bending of the sound path can
complicate the analysis of the data.

Checkpoint 16.4
1
2

Describe the conditions necessary for reflection of ultrasound waves to occur at a boundary.
Calculate the percentage of ultrasound energy that reflects from a muscle and bone boundary. (Use values in
Table 16.4.1 to help.)
Explain why it is necessary to use gel on the transducer in order to get clear images.

16.5 Types of scans


To perform an ultrasound scan, a transducer is placed against the patients skin,
directly over the region to be imaged. The transducer sends a very brief pulse of
ultrasound into the tissue. The pulse travels into the body as a beam, similar to
a searchlight. Interfaces along the way reflect some of the ultrasound energy back
to the transducer. The transducer converts the energy of the reflected echo into
electrical signals, which are sent into amplifiers and signal-processing components
of the ultrasound machine. The exact delay between when the transducer first
emits the ultrasound pulse and when it picks up an echo allows the machine to
calculate how far the reflecting interface is from the transducer, given the known
speed of ultrasound in the tissue.
Ultrasound can be used in different ways, called modes, which produce
different data and images.

Describe the situations in


which A scans, B scans and
phase and sector scans would
be used and the reasons for
the use of each.

A-mode scans
1200
high-frequency signal
absolute value of signal
Hibert envelope of signal

Amplitude

800
400
0
400
800
1200

Time of flight (s)

Figure 16.5.1

312

An A-mode scan signal

When a single-element transducer is used and the transducer is


stationary, only a single dimension can be recorded. That dimension
is the distance of the echoing interface from the transducer. A typical
A-mode scan (or A scan) is shown in Figure 16.5.1. Returning echoes
produce a vertical displacement of the signal display. The amount of
displacement is proportional to the amplitude. This was the first use of
ultrasound as it is relatively simple and requires little signal processing to
display the data. New ultrasound devices use computers to analyse the
various time values.
Typically the A scan is used to examine midline structures in the
brain, solid or cystic structures, foreign bodies in the eye, abnormal fluid
accumulation around the heart or lungs, or to guide biopsy and
amniocentesis needles.

medical
physics

The most common type of medical ultrasound is called a B-mode scan


In a B-mode scan the returning
(or brightness-mode scan).
echoesare displayed on the screen as dots, rather than as a signal plotted
versus time, as in A-mode scans. The intensity or brightness of the dot
is proportional to the amplitude of the reflected pulse (Figure 16.5.2).
In the simplest scanning mode, a single-element transducer can be
moved around to provide different lines probing through the body. Like
the beam of a searchlight, it picks out structures along the line of the
beam. An image can be built up from these scans (Figure 16.5.3).
probe placenta
legs

foetal
skull

probe

spine
probe

organ

to scan
display

vertebra

pulse
echo

transducer
A scan
display

Echo strength

B-mode scans

Time

B scan
display

Figure 16.5.2

The timing of a return echo


indicates depth in the body.
The intensity of reflection may
be indicated as a signal versus
time in A-mode scans or as a
bright spot in B-mode scans.

ultrasound image
synthesised
from scans

Figure 16.5.3

B-mode scan with a moving transducer can be used to reconstruct


a two-dimensional ultrasound image of a foetus.

M-mode scans
A single-ultrasound beam can be used to
produce an M-mode image in which movement
of an individual structure such as a heart valve
can be displayed versus time. An M-mode scan
uses a high-sampling frequency (up to 1000
pulses per second) to capture the rapid motion.
This display is essentially a series of B-mode
scans lined up side-by-side in time (Figure
16.5.4).

2D real-time scans
Using a convex-array transducer with many
piezoelectric elements, beams of ultrasound can
be launched in different directions into the
body. Each beam produces a B-mode scan.
Once all the echoes from the first beam have
been picked up, the transducer sends a second
Figure 16.5.4 An M-mode scan through the left ventricle of the heart allows
pulse along a slightly different beam direction
measurement of the ventricle size (green markers) as the heart beats.
into the tissue. Echoes are again picked up and
sent for processing to produce an image, and
another pulse is launched in another direction, and so on. Like the beam of
a searchlight swept across the night sky, the pulsed beam from an ultrasound
transducer is swept through the body, mapping out reflecting surfaces and forming
a 2D real-time scan. The echos image is two-dimensional and shows detail along
313

16

Seeing with
ultrasound

one slice through the body, originating at the transducer. Beams are swept and
ultrasound images are formed very rapidly in real-time images. As the operator
holds the transducer in contact with the skin, the image appears live on a video
monitor. Real-time images are usually made with a sector scanner, so the images
will be pie shaped.
The B-mode scan is continuously updated and displayed in real time, with
the image changing as the tissues move in the field of the ultrasound beam.
The movement of the object being scanned can be monitored on the image.
Interpreting ultrasound images requires considerable skill and knowledge,
as the images in Figures 16.5.5 and 16.5.6 demonstrate. An untrained observer
would have difficulty making sense of the image without explanation.
Figure 16.5.5

This sector scan image of a


gall bladder reveals a strong
(bright) echo from an
abnormally thick wall.

Where to from here?


Complex ultrasound transducers are used for producing real-time 3D
ultrasound images. These simultaneously produce sector scans in many planes
(Figure 16.5.7). Combined with tilting or moving the transducer, this yields
ultrasound data of body structures from many angles. High-speed digital signal
processing permits the large amount of gathered data to be assembled into a 3D
image (Figure 16.5.8).

Figure 16.5.6

This image of a kidney


reveals a large, dark cyst
that shows no internal
echoes because of its
homogeneous structure.

Figure 16.5.7

Examples of scanning patterns produced by modern transducers.


Each small blue triangle indicates a sector scan (a) moved in an angle or
(b) rotated relative to the transducer.

Ultrasound using this 3D imaging technique is sometimes used to produce a


movie. This is usually called 4D ultrasound. There is a section of the ultrasound
industry that promotes 4D ultrasound for non-medical reasonstrying to
persuade parents that it is fun to have such a record of the child before it is born.

Checkpoint 16.5
1
2
Figure 16.5.8

314

Example of a 3D ultrasound
image of a foetus

Summarise the process of (a) A-mode scanning, (b) B-mode


scanning and (c) M-mode scanning.
Outline examples of when each mode of scanning would be used.

medical
physics

16.6 Ultrasound at work


Bone-density measurements
Osteoporosis literally means porous bones. Bones that once were strong typically
become fragile as a person gets older, due to loss of calcium. The problem is
common in postmenopausal women, but a significant number of men also suffer
from a loss of bone density. A person may not realise they have osteoporosis until
they suffer a vertebral fracture when doing ordinary activities such as lifting a bag
of groceries, or break a hip in a fall.
To determine if a person has osteoporosis, it is necessary to measure bone
density (or bone mineral density). Bone-density testing can measure the amount
of bone in different parts of the skeleton, and can predict the risk of future fractures
and monitor changes in bone-mineral density due to medical conditions or therapy.
There are several ways to measure bone density, most of which involve the
use of X-ray radiation. X-ray based methods usually take a bone-density
measurement of the hip, spine, forearm or heel by passing two X-ray beams of
different energy through the bone. The technique is called dual X-ray
absorptiometry (DXA or DEXA).
Alternatively, ultrasound can be used to estimate the bone density of the heel,
as the heel bone contains a high percentage of the kind of bone most affected by
osteoporosis. During an ultrasound examination, two soft rubber pads are placed
in contact with either side of the heel. Transducers within these pads send and
The speed and absorption
receive ultrasound waves through the heel bone.
of ultrasound vary with bone density. The test takes about a minute and is
performed in a seated position (Figure 16.6.1). No injections or invasive
procedures are necessary and the test results are processed immediately.

What are the relative advantages of ultrasound imaging and DXA?


Ultrasound systems are smaller and less expensive than DXA systems.
Ultrasound requires no exposure of the body to ionising radiation (although
X-ray levels are very low with DXAlower than in most other X-ray
procedures).
Studies show that ultrasound mainly measures the bone mass while DXA is
better able to predict bone strength.
Reproducibility of results using ultrasound is not quite as good as with DXA.
Relative risk for hip fracture is predicted as well by ultrasound of the heel as
by DXA of the hip.

Blood flow and the Doppler effect


Doppler ultrasound scanning is used to image any structure or tissue that
pulsates or moves. It is based on the principle that whenever the reflector surface
moves with respect to the transducer, there is a shift in the frequency of the
ultrasound received by the transducer compared with the frequency it emitted
(the Doppler effect; see in2Physics @ Preliminary section 13.8). This concept is
illustrated for blood flow in Figure 16.6.2, while Figure 16.6.3 illustrates a case in
which Doppler imaging reveals abnormal blood flow in a heart. Colour Doppler
imaging is colour-coded, using redder colours to show flow or movement towards
the transducer and bluer colours to show flow away from it. (Interestingly, this is the
opposite of the choice an astronomer would make in presenting Doppler images.)

Identify data sources, gather,


process and analyse
information to describe how
ultrasound is used to measure
bone density.

Figure 16.6.1

A machine used to measure


the bone density of your heel

PRACTICAL
EXPERIENCES
Activity 16.2

Activity Manual, Page


128

Describe the Doppler effect


with respect to sound and how
it is used in ultrasonics to
obtain flow characteristics of
blood flow through the heart.

315

16

Seeing with
ultrasound

Outline some cardiac problems


that can be detected through
the use of the Doppler effect.

The Doppler shift effect has been used for a long time in foetal heart rate
detectors. Further developments in Doppler ultrasound technology in recent
years have resulted in its increased use in obstetrics to assess and monitor the
well-being of a foetus.

PRACTICAL
EXPERIENCES
Activity 16.3

Activity Manual, Page


130

skin
beam direction

Doppler again
and again

he Doppler effect (see in2


Physics @ Preliminary
section 13.8) is one of the most
versatile techniques for
measuring speed. As well as its
use in Doppler ultrasound, it is
used by astronomers to measure
the motion of stars and by police
radars to detect speeding
drivers.

Figure 16.6.3

Colour Doppler ultrasound of


a newborns heart, revealing
blood flow (green) through
a hole in the septum (grey)
between the ventricles

vessel

flow
B

Sonogram

Figure 16.6.2

Different transducers (AD) in this system see blood flowing in different


directionswith a component towards or away from the transducer. This
produces a Doppler effect that can be used to calculate a velocity at that point.

Echocardiography
Echocardiography is the name given to ultrasound imaging of the heart. It is
used to investigate heart-valve function, ventricular function, congenital heart
diseases (including holes in the septum separating the left and right halves of
the heart), cardiac tumours and obstructions in cardiac blood vessels.
Pulsed ultrasound from a transducer is used and a continuous
recording is made of the echoes received from the various parts of the heart,
showing the motion of the various parts. A standard transthoracic
echocardiogram can be conducted through the thorax (chest wall). Higher
resolution imaging can be achieved by a trans-oesophageal echocardiogram in
which a transducer on a specialised probe is passed down the patients
oesophagus, thereby getting much closer to the heart.
Blood-flow characteristics in the foetal blood vessels can also be examined
using Doppler ultrasound. Diminished flow, particularly in the diastolic
phase of a pulse cycle (as the heart muscle relaxes) is associated with problems
with the circulation of blood in the foetus. Colour Doppler can also be used
to assess the success of coronary bypass surgery, and is particularly useful in
the diagnosis and assessment of congenital heart abnormalities such as atrial
or ventricular septal defects (a hole between the atria or the ventricles; see
Figure 16.6.3).

Checkpoint 16.6
1 Discuss why the heel is an ideal place to use ultrasound to determine bone density.
2 Explain how the Doppler effect can help diagnose the well-being of a foetal heart.
3 Outline how echocardiography works.
316

PRACTICAL EXPERIENCES

medical
physics

CHAPTER 16

This is a starting point to get you thinking about the mandatory practical
experiences outlined in the syllabus. For detailed instructions and advice, use
in2 Physics @ HSC Activity Manual.

Activity 16.1: Ultrasound images


Collect some images made using ultrasound and interpret information from
the image.
Discussion questions
1 Interpret the grey scale that accompanies most ultrasound images.
2 Determine what parts of the body are best imaged with ultrasound.
3 List things that can be determined from ultrasound images.

Activity 16.2: Doppler ultrasound


Use several data sources to find and observe a Doppler ultrasound image that shows
the flow of blood through the heart.
Discussion questions
1 Define the Doppler effect.
2 Explain how different directions in the video image are shown.
3 Outline what can be determined from these images.

Activity 16.3: Bone density


Find out how bone density measurements are made using ultrasound and present
the information in a 5 minute talk to the class.
Discussion questions
1 Define bone density measurements and discuss why they are needed.
2 Outline how the measurements are taken.
3 Explain why ultrasound is a good method for obtaining these
measurements.

Gather secondary information


to observe at least two
ultrasound images of body
organs.

Identify data sources and


gather information to observe
the flow of blood through the
heart from a Doppler
ultrasound video image.

Identify data sources, gather,


process and analyse
information to describe how
ultrasound is used to measure
bone density.

317

16


Seeing with
ultrasound

Chapter summary

Ultrasound can travel through the body but is reflected


from interfaces between different tissues.
For most medical imaging applications, ultrasound in
the range from 3.5 MHz to about 10 MHz is used.
The reflected sound can be detected at the skin surface,
allowing imaging of structures deep within our bodies in
a non-invasive way with little stress to the patient.
Higher frequency ultrasound produces images with
a better resolution, but has poorer penetration
through tissues.
Lower frequency ultrasound has better penetration
through tissues, but produces images with a lower
resolution.
The intensity of the reflected wave is determined by the
difference in the acoustic properties of the tissues and
the absorption and angle of the intervening tissue.
Ultrasound waves are emitted and detected by
a transducer.
A piezoelectric transducer uses a crystal that responds to
an oscillating voltage by oscillating slightly in size and
responds to the pressure of a sound wave by producing
an oscillating voltage.
A convex-array transducer uses a diverging beam to
produce a sector scan.
Acoustic impedance describes how readily a specific
sound frequency will pass through a material.
Acoustic impedance: Z =

When ultrasound waves meet a boundary between


tissues with different acoustic impedances, some of the
ultrasound energy will be reflected and some will be
transmitted. The greater the difference in acoustic
impedance, the greater the intensity of the reflected
signal.
2
I r Z2 Z1
=
Io Z + Z 2
2
1

Impedance matching (acoustic coupling) between two


media results in most of the energy being transmitted
through the interface, with almost no reflection.
An A-mode scan shows returning echoes as a vertical
displacement, proportional to the amplitude of the
reflected signal.
A B-mode scan shows the returning echoes as dots with
intensity proportional to the returning echo amplitude.
These dots are used to construct a 2D real-time scan.
A movie made using 3D ultrasound images is called
4D ultrasound.
Bone density scans use the fact that the speed and
absorption of ultrasound vary with bone density.
Doppler ultrasound scanning uses the Doppler effect to
image any structure or tissue that moves.
Colour Doppler imaging is colour-coded to show flow
or movement towards or away from the transducer.
Echocardiography is ultrasound imaging of the heart.

Review questions
Physically Speaking
Below is a list of topics that have been discussed throughout this chapter. Create a visual summary of the
concepts in this chapter by constructing a mind map linking the terms. Add diagrams where useful.

318

Ultrasound

Transducer

Doppler effect

Frequency

Piezoelectric

Acoustic impedance

Reflection

Linear array

Impedance matching

Bone-density measurement

Echocardiography

A-mode scan

B-mode scan

Sector scan

Blood flow

medical
physics

Reviewing

11 Describe the effect of increasing the difference in


acoustic impedance at a tissue boundary on the
proportion of the incident ultrasound energy that is
reflected at the boundary.

1 Compare quantitatively the frequencies of medical


ultrasound with sound in the normal hearing range
of humans.

12 Describe the conditions that must be satisfied for


reflection to take place at a tissue boundary in
the body.

2 Contrast medical ultrasound and the sound that


humans hear in terms of the media in which they
mainly occur.

3 Figure 16.7.1 is a colour ultrasound image. Identify


the organ that is imaged here and outline the main
benefit of this type of image.

13 Explain why ultrasound imaging is not used for


investigating the function and structure of the brain.

14 Outline how refraction complicates the process of


obtaining clear images of organs in the ultrasound
process.

15 Discuss current issues associated with the use of


obstetric ultrasound imaging.

16 Describe the Doppler effect and outline how this


effect may be observed using sound waves.

17 Explain how ultrasound can be used to obtain


information about the flow of blood through the heart.

18 Describe how ultrasound is used to measure


bone density.

19 Compare the nature and use of A-mode and B-mode


ultrasound scans.

20 Explain why ultrasound is not used to scan the lungs.


21 Assess the statement Tissues having the same
Figure 16.7.1

density have the same acoustic impedance.

Example of an ultrasound image

4 Outline the piezoelectric effect and its place in


ultrasound imaging.

5 Assess the advantages of ultrasonic examination of


a pregnant woman against the dangers associated
with an ultrasound examination.

6 Define acoustic impedance in words (without using


equations).

7 Identify the units for the quantities represented in the


equation Z = that defines acoustic impedance.

8 Account for the fact that different materials have


different acoustic impedances.

Solving Problems
22 Bone has a density of 2103 kg m3. The speed of

sound in bone is 4080ms1. Calculate the acoustic


impedance of bone.

23 Calculate the density of kidney tissue, given that the

acoustic impedance is 1.623106rayl and that the


ultrasound velocity in kidney tissue is 1561ms1.

24

Calculate the percentage of the ultrasound energy that


would be reflected from an interface between blood
and brain tissue.

9 The acoustic impedance of two tissues in the body is


the same, and yet one has a greater density than the
other. Explain how this is possible.

10 Identify the quantities represented by each symbol


in the equation

I r Z 2 Z 1
=
Io Z + Z 2
2
1
Re

iew

Q uesti o

State the units for each quantity.

319

17
X-rays, cathode, radiographs,
tomography, computed axial
tomography (CAT), characteristicX-rays,
Bremsstrahlung, anode, radiographer,
filter, collimator, grid, fluorescent,
contrastagents

Imaging
with X-rays
The invisible reveals the hidden
Wilhelm Rntgen (18451923) was studying cathode rays in a Crookes
tube wrapped in cardboard, when he noticed a faint green glow from
the fluorescent screen painted with barium platinocyanidethe rays
emitted from the tube were passing through the cardboard. He found
the rays could also pass through other papers and books. This
phenomenon intrigued Rntgen and he dedicated the next few months
to systematically investigating the properties of these rays. He named
this new type of radiation X-rays.
On 22 December 1895, Rntgen took the first X-ray, an image of the
hand of his wife Anna. After this, many scientists and companies began
investigating X-rays. Thomas Edison began a rigorous investigation into
fluorescent materials. He developed the fluoroscope using calcium
tungstate, which became the standard for medical X-ray examinations.
Today, with the development of semiconductor detectors and imageprocessing software, X-rays are used in computed axial tomography
(CAT scans) to provide medical doctors with high-resolution 3D images
of skeletal, vascular and soft tissue.

17.1 Overview and history:


types of X-ray images
X-rays are a form of electromagnetic radiation that lies between ultraviolet and
gamma radiation on the electromagnetic spectrum. X-rays are able to pass
through our body. In medicine, they are used for diagnosis of medical problems
and injuries, and for therapeutic purposes such as treatment of certain tumours.
From 1896, many applications of X-rays were immediately recognised and
generated a wide range of interest. Companies began researching and developing
a range of specialised versions of Crookes tubes to produce X-rays. However,
Crookes tubes are classified as cold cathode tubes and were often unreliable, as
they needed to contain a small quantity of gas to operate. In 1913 William D
Coolidge (18731975) invented the Coolidge X-ray tube, which permitted a
stable and controllable continuous production of X-rays. Coolidge drew upon
the thermionic diode (vacuum tube), which used a hot cathode (a filament) that
permitted current to flow in a high vacuum. Most modern X-ray tubes are
variations on Coolidges basic design.

320

medical
physics
A range of film-based and fluoroscope devices were developed and refined,
providing the familiar two-dimensional X-ray radiographs. In 1930, Alessandro
Vallebona (18991987) proposed the idea of tomography in which a single
slice of the body could be recorded on radiographic film. In the 1970s the
development of affordable computers capable of processing image data, and
the parallel development of semiconductors and image intensifiers allowed a new
generation of X-ray devices to be developed. The technology was now available
to re-engineer Vallebonas tomography. Allan McLeod Cormack (19241998)
developed the theory and Godfrey Hounsfield (19192004) independently built
a prototype of the new generation of X-ray tomography scanner. This became
known as computed tomography (CT) or computed axial tomography (also
called CAT).
Today, with the development of more sensitive detectors and advances in
image processing software, CAT scans can provide two- and three-dimensional
images with resolutions of 1cm. The combination of traditional radiographs and
CAT scans provides the medical profession with important therapeutic and
imaging diagnostic tools.

Checkpoint 17.1
1
2

Describe the properties of X-rays.


Construct a timeline that shows the development of technology in
X-ray devices.
Figure 17.1.1

17.2 The X-ray tube


X-rays are emitted when high-energy electrons strike a target. There are
two processes by which X-rays can be produced: characteristic X-rays and
Bremsstrahlung. The efficiency of both processes in converting the kinetic
energy of the electrons into X-rays is often less than 2%; the remainder
of the energy is converted into heat.
The Coolidge X-ray tube developed in 1913 has remained the basis for the
majority of todays diagnostic devices. Several improvements have been made to
increase efficiency, control and cooling of the anode target. Figure 17.2.1 shows
a schematic representation of a modern rotating-anode diagnostic X-ray tube.
rotating anode

Wilhelm Rntgens first X-ray


taken on 22 December 1895.
It is the hand of his wife
Anna Berthe Rntgen and you
can see her wedding ring.

Describe how X-rays are


currently produced.

stator of induction motor

glass envelope
bearings
rotor/anode
support

cathode
block
rotor
filament

focusing
cup
electrons

target
exit window

Figure 17.2.1

Schematic representation of
a rotating-anode diagnostic
X-ray tube
321

17

Imaging
with X-rays

The tube is highly evacuated and generally constructed from glass. It contains
an anode and a cathode. A potential difference of between 20000 and 300000volts
is applied between the cathode and the anode. The cathode has a small filament
(similar to an incandescent lamp filament), which is heated to a dull red glow by
a small current. The hot filament readily releases electrons and the high potential
difference then accelerates these electrons from the cathode towards the target
anode. The cup shape of the cathode behind the filament focuses the electron
beam towards a point on the target anode. When the electrons hit the angled
tungsten anode, X-rays are emitted through the exit window at the side of the
X-ray tube. Because of the inefficiency of the process, a huge amount of thermal
energy is produced. To ensure the tungsten anode does not melt, it is often rotated
to more evenly distribute the energy. The anode is often mounted on a copper
heat-sink that may also have an additional cooling system attached. Shielding, to
reduce the emission of X-rays in unwanted directions, surrounds the tube itself.

Checkpoint 17.2
1
2
3

Name the two types of X-rays.


Discuss the energy conversion of electrons as they strike a surface to produce X-rays.
Describe the structure of a Coolidge tube.

17.3 Types of X-rays


Compare the differences
between soft and hard X-rays.

X-rays are often categorised as hard and soft X-rays, descriptive terms
that indicate the relative penetrating ability of the X-ray beam. Soft X-rays do
not penetrate through body tissue and are absorbed easily. These absorbed X-rays
are not useful for producing radiograph images and they pose an increased risk
to patients, of genetic mutations and cancers. X-ray machines have aluminium
filters that attenuate these lower energy soft X-rays from the X-ray beam. Hard
X-rays have a higher energy and greater penetration into and through tissue, thus
producing a sharper image.
Changing the potential difference between the cathode and anode can vary
the distribution of hard and soft X-rays produced by the X-ray tube. Increasing
the voltage to the tube will produce more hard X-rays. The radiographer can
adjust the voltage, to obtain the best quality image of the particular organ, tissue,
vessel or bone under examination.
Table 17.3.1 Properties of X-rays

322

Hard X-rays

Higher potentials are applied to the X-ray tube


Produced by the impact of high-energy electrons onto the anode
Shorter wavelength ~0.01nm
Higher penetration (the X-ray photons have more energy)
Produce higher resolution images

Soft X-rays

Lower potentials are applied to the X-ray tube


Produced by the impact of lower energy electrons onto the anode
Longer wavelength ~1nm
Lower penetration (the X-ray photons have less energy)
Produce lower quality images

medical
physics

Characteristic X-rays

ejected

ei

2p

1s
2s
e

electron
e

2p

1s
2s

characteristic X-ray

tic

is
ter

c
ara
ch
ay
X-r
2 posible paths
Au
to fill the gap
ger
ele
ctr
o

n
e
2p

e
1s
2s

0.02 0.04

0.06 0.08 0.10

0.12

Wavelength (nm)

Figure 17.3.1

X-ray wavelength
10

Characteristic X-rays show up


as distinct sharp peaks within
the X-ray spectrum.

X-ray continuum radiation


(Bremsstrahlung)
50 kV

X-rays can be produced by firing high-speed electrons at a metal target. These


electrons can eject electrons from the inner shells of the atoms of the target.
Vacancies will be quickly filled by electrons dropping down from higher levels,
emitting X-rays with specific defined wavelengths. Occasionally an emitted
characteristic X-rays photon is re-absorbed by another electron, which is then
ejected from the atom (an Auger electron).

Relative intensity

Figure 17.3.2

X-rays from a
molybdenum
target at 35 kV

a gap remains

characteristic
X-rays

Brehmsstrahlung
continuum

X-ray intensity

incident
electron

3
Relative intensity

The accelerated electrons can interact with and knock out an electron from
the inner electron shell of the anode. If an outer shell electron drops down and
fills the vacant position, an X-ray photon of a specific energy and wavelength,
peculiar to the elements of the target will be produced. In the total X-ray
spectrum, the characteristic X-rays appear as sharp well-defined peaks.
Figure 17.3.1 shows the characteristic peaks for X-rays produced by a
molybdenum target anode bombarded by electrons accelerated through
a potential of 35kV. The details of the process are shown in Figure 17.3.2.

Bremsstrahlung
The word Bremsstrahlung means braking radiation in German. It is used
to describe the radiation emitted when electrons are decelerated or braked, when
fired into a metal target. When an electron rapidly changes its velocity, the lost
kinetic energy is converted into photons of electromagnetic radiation. When the
energy of the incident electrons is high enough, the radiation emitted will be
X-rays. Decelerating electrons from the beam interact and produce a continuous
distribution of radiation.
Figure 17.3.3 shows a set of curves for a tungsten target with electron beams
of four different energies. The details of the Bremsstrahlung process are shown
in Figure 17.3.4.

40 kV
6

30 kV

2
20 kV
0.02

0.04

0.06

0.08

0.10

Wavelength (nm)

Figure 17.3.3

ei incident electron

Bremsstrahlung (braking radiation)


is characterised by a continuous
distribution of wavelengths. The
curves in this graph are based on
the 1918 paper of Clayton Ulrey
(18841963).

incident electron
ei

hard X-ray

ei

Figure 17.3.4

X-ray intensity

soft X-ray

Wavelength (nm)

ei

The processes associated with high-energy and low-energy Bremsstrahlung


323

17

Imaging
with X-rays

Checkpoint 17.3
1
2
3
4

Distinguish between hard and soft X-rays.


Create a table to list the properties of hard and soft X-rays.
Explain how characteristic X-rays are produced.
Explain how Bremsstrahlung X-rays are produced.

17.4 Production of X-ray images


The key components of a medical X-ray machine are an X-ray tube, a filter, a
collimator, a platform to situate the patient, a grid and a detector. Figure 17.4.1
shows a typical X-ray machine.
The filter is designed to remove soft X-rays, which pose a risk to the patient
and are not useful for imaging purposes. The collimator provides a mask to shape
the output X-ray beam to minimise unwanted X-ray exposure to the patient.
The grid situated just in front of the detector absorbs scattered secondary X-rays
that would make the image fuzzy.

Conventional radiographs
In traditional radiographs, fluorescent screens absorb the energy in the X-ray
beam that has penetrated the patient. This energy is converted into visible light
that has same information as the original X-ray beam. This light is then used to
expose the X-ray film. The more efficient this light conversion, the less the patient
needs to be exposed to X-ray radiation. In most systems, the film is sandwiched
between two fluorescent screens in a cassette so that the film emulsion is exposed
from both sides. In modern X-ray machines, film is replaced by arrays of
electronic detectors.
Conventional X-ray systems are used to image a variety of patient conditions
including suspected skeletal bone fractures (Figure 17.4.2), cancer in the breast
(Figure 17.4.3) and lung conditions.

Figure 17.4.1
324

A basic diagnostic X-ray machine

Figure 17.4.2

(a) An incomplete (greenstick) fracture of the radius and ulna


of the lower arm and (b) a simple fracture of the ulna

medical
physics

Imaging vascular and hollow structures

X-ray imaging does not show vascular (veins and arteries)


or hollow soft tissue structures such as the digestive tract
To provide radiologists with a
in great detail.
mechanism to reveal these structures, non-toxic contrast
agents containing barium or iodine, which absorb X-rays,
are used. These contrast compounds can be ingested, or
injected into an artery or vein.
These procedures were first carried out between 1906
and 1912 and allowed blood vessels, the stomach, digestive
tract, the gall bladder and bile ducts to be seen in situ for
the first time. Today the process is very similar. An initial
X-ray is taken, then contrast material containing iodine is
injected into the bloodstream and a second X-ray taken.
These two images are then digitally subtracted, leaving only Figure 17.4.3 This breast X-ray, called a mammogram, shows
the image of the blood vessels (Figure 17.4.4).
(a) a healthy breast and (b) a breast with
a tumour (arrowed).
Another once common procedure is called a barium
meal X-ray (Figure 17.4.5). This procedure involves the
patient drinking a suspension of barium sulfate and then a series of
radiographs are taken of the oesophagus, stomach and duodenum. Barium
meal tests are declining with the increasing use of endoscopy (section 18.2),
which allows the doctor to directly visually inspect the oesophagus,
stomach and duodenum.

Figure 17.4.4

Renal artery angiogram showing blood vessels,


a kidney and the spine

Figure 17.4.5

Barium swallow X-ray of the large intestine of a patient


with cancer of the sigmoid colon

Checkpoint 17.4
1
2
3

List the components of an X-ray machine and their functions.


List the possible uses of conventional X-rays.
Outline how it is possible to image hollow structures within the body.
325

17

Imaging
with X-rays

17.5 X-ray detector technology


In the early 1900s a head X-ray would require the patient to be exposed to
10 minutes of radiation. Today a similar higher resolution image would expose
the patient to one-fiftieth of the radiation used 100 years ago. Technological
advances over the last 100 years have improved the quality of X-ray images and
dramatically reduced the X-ray exposure to patients.
Originally radiographic images were produced on glass photographic plates.
These were replaced by film cassettes with scintillating screens which intensified
the images. In 1955, the X-ray image intensifier was developed and X-ray images
were displayed on a television monitor. The image intensifier allowed doctors to
image blood vessels and the heart in real time.
New digital technology and semiconductor detectors (silicon or germanium
doped with lithium) began to be developed in the 1970s. Today, with the advent
of large semiconductor array detectors, it has become possible to collect digital
data that can be processed into high resolution images. These images can be
enhanced and processed, transmitted for remote diagnosis and easily stored
and retrieved. Over the next 15 years most conventional X-ray systems will be
upgraded to all digital technology.

Checkpoint 17.5
1
2

Discuss how image intensifiers have changed what doctors can do in diagnosis.
List the benefits of digital technology.

17.6 Production of CAT X-ray images


A computed axial tomography (CAT) or computed tomography (CT)
scanner is a diagnostic device that uses an X-ray tube that is rotated around the
patient. A detector collects the X-rays and digitises the information. A series of
linear scans is processed by a computer and a series of two-dimensional slices or
a three-dimensional rotatable image can be constructed. The development of
CAT scan technology was enabled by the development of microcomputers and
detector technology that could digitise data. In medical circles, the term CT is
more commonly used than CAT, but they are interchangeable.
Figure 17.6.1

A 64-slice CAT scanner

Explain how a computed axial


tomography (CAT) scan is
produced.

326

Basic design
There are several designs of CAT scanners in use. One commonly used scanner
has the patient placed in a fixed position on a bed. The X-ray tube and detector
array are attached to a movable C-shaped gantry that is rotated 180 around the
patient and then progresses forward and records the next set of scans. Another
more modern system has a moveable bed is that moved slowly through a fixed
toroidal structure (a donut) that houses the X-ray tube and detector. As the bed
progresses, the X-ray tube and detectors continually rotate around the toroidal
structure, scanning the patient in a spiral motion.

medical
physics

The scanning process


In older style devices (Figure 17.6.2a), a linear scan of data is taken at a large
number of positions, producing the digitised data for that slice through the
patients body. The source and detector are then rotated to produce a new scan.
Modern devices speed up this process and direct a narrow fan-beam through the
patient. The intensity of the emerging transmitted radiation is recorded by a line
of adjacent detectors (Figure 17.6.2b and c). Fast CAT scans can now also be
performed with a cone-shaped X-ray beam and an area-array detector panel.
a

X-ray
tube

45
n

sc
a

13
detectors

detector
array

c
fan
beam

direction of
X-ray tube
motion

patient

Figure 17.6.2

n
180

X-ray source

collimator

X-ray
source

odfrey Hounsfields original


CT scan in 1972 took
several hours to acquire a single
slice of image data and then
more than a day to reconstruct
the data into a single image.
Today a CAT scan can construct
an image in less than 1 second.

scan 90

a
sc

detector

A Long Wait

X-ray
source

ring of
stationary
detectors

The history of CAT: (a) A pencil-beam X-ray source and a detector scan linearly
across the patient from a series of angles. (b) A fan-beam X-ray source and an arc
of detectors rotate around the patient. (c) Only the fan-beam X-ray source rotates;
the detector ring is stationary.

Constructing the two-dimensional slice image


To form an image, we need to calculate the radiation that is absorbed at each
small volume element (voxel) pixel within the scanned structure. These scans are
three-dimensional versions of two-dimensional picture elements (pixels) that
make up a picture from your digital camera. With this information we can
construct a digital image. To assist in the explanation of the process used in CAT
scans, we will use the simplified example shown in Figure 17.6.3.
In this example, two linear scans of a hollow cross have been made (Figure
17.6.3a). One linear scan has been made from the left and the other from the
top. The intensities for each beam have been measured by the detectors opposite
the X-ray sources. The computer then translates the information from the
detectors into the total amount of the incident X-ray radiation that was absorbed
as it passed through the structure. These values have been recorded down the
right-hand side and along the bottom edge of the diagram (Figure 17.6.3a). This
is the same information as obtained in a normal radiograph.
Now to obtain specific information about each voxel, we can sum the vertical
and horizontal absorption values and record the value in each corresponding
voxel (Figure 17.6.3b). A grey scale can then be overlaid, with the value 10
being white, 0 being black and the other values appropriate shades of grey.
Figure 17.6.3c shows this grey-scale overlay.

X-rays

X-rays

5 4 2 4 5

10
9
7
9
10

9
8
6
8
9

7
6
4
6
7

9
8
6
8
9

5
4
2
4
5

10
9
7
9
10

Figure 17.6.3

Mechanism for constructing


the 2D image slice

327

17

Imaging
with X-rays

The image generated shows some sort of cross structure but it is fuzzy. Now
if we reassign the grey scale and assign light grey to all voxel values greater than 6
and a dark grey to all voxels with values of 6 or less, we obtain a clearer image of
the original structure (Figure 17.6.3d).
This is the basic process of a CAT scan. For a real CAT scan in which there
are complex sets of absorption calculations, a lot of data processing is required to
recreate a high-resolution image from the data within a second. Figure 17.6.4
shows a single two-dimensional slice from a scan.
Because the data contains three-dimensional information, the imaging
software can combine a series of slices to produce a three-dimensional image.
Computer imaging software can also remove distracting tissues from the scan
images and generate an image that shows the specific organ, tissue or structure
under examination. Figure 17.6.5 shows a CAT angiogram of the blood vessels
in the brain, with the surrounding tissue removed.
Common applications for CAT scans include:
identifying trauma injuries to the lungs, heart, spleen, kidneys and liver
planning for and assessing the results of surgery
planning radiation treatments for tumours
detecting osteoporosis by measuring bone density.

Figure 17.6.4

A CAT scan of the lungs with the window level set to


demonstrate the vessels and airways. This is used to look for
diseases such as pneumonia or lung cancer.

Checkpoint 17.6
1
2
3
4

328

Outline how CAT images are produced.


Describe how a two-dimensional scan is produced.
Outline the benefits of a CAT scan.
Outline the uses of a CAT scan.

Figure 17.6.5

An intracranial angiogram showing the blood


vessels of the brain

medical
physics

17.7 Benefits of CAT scans over


conventional radiographs
and ultrasound
Conventional radiographs provide an image of all the structures present,
superimposed on each other. For example a chest X-ray of the lungs will show all
the ribs. The ribs therefore obscure details associated with the lungs. A CAT scan
allows the ribs to be removed from the image and provides the doctor with a full
image of the lungs. When the head is imaged, CAT scans provide a detailed
image that is not as affected by the skull bone as a radiographic image. A CAT
scan of the head, for example, can distinguish between the white matter, grey
matter and spinal fluid. This enhanced discrimination allows the doctor to more
easily identify vascular problems, tumours and subtle abnormalities. CAT scans
contain the information to produce a three-dimensional image, allowing many
angles of a particular area of interest to be viewed by the doctor. Software can be
used to remove distracting tissues or structures.
Modern ultrasound equipment can also produce three-dimensional images
(see section 16.5). Ultrasound can provide an initial diagnosis but cannot clearly
identify the level of tissue damage or sites of internal bleeding. It is unable to
penetrate bone or gas, and therefore tissues beyond these regions cannot be
imaged. For this reason, ultrasound cannot be used to examine the brain, which
is surrounded by the skull, or tissue masked by gas pockets in the abdominal
region. A CAT scan can image all these regions, with resolution that is superior
to that of ultrasound imaging.

Describe circumstances where


a CAT scan would be a superior
diagnostic tool compared to
either X-rays or ultrasound.

PRACTICAL
EXPERIENCES
Activity 17.1

Activity Manual, Page


133

Checkpoint 17.7
1
2

Identify the uses of CAT for which a radiograph can not be used.
List problems that CAT scans can overcome that ultrasound imaging can not.

329

17

Imaging
with X-rays

PRACTICAL EXPERIENCES
CHAPTER 17
This is a starting point to get you thinking about the mandatory practical
experiences outlined in the syllabus. For detailed instructions and advice, use
in2 Physics @ HSC Activity Manual.

Gather information to observe


at least one image of a fracture
on an X-ray film and X-ray
images of other body parts.
Gather secondary information
to observe a CAT scan image
and compare the information
provided by CAT scans to that
provided by an X-ray image for
the same body part.

330

Activity 17.1: CAT versus X-rayS


Find a CAT scan and an X-ray radiograph of the same part of the body and
compare the information that can be determined from each.
Discussion questions
1 Outline characteristics that allow you to determine that an image is
a CAT scan.
2 Outline characteristics that allow you to determine that an image is an
X-ray radiograph.
3 Identify applications that would make a CAT scan more appropriate than
an X-ray radiograph.
4 Identify applications that would make an X-ray radiograph more
appropriate than a CAT scan.

Chapter summary

X-rays are a form of electromagnetic radiation that lies


between ultraviolet and gamma radiation on the
electromagnetic spectrum.
X-rays are used for the diagnosis of medical problems
and injuries, and for therapeutic purposes such as the
treatment of certain tumours.
The modern X-ray tube is based upon the 1913 design
by William Coolidge, which used a hot cathode and
permitted a stable and controllable continuous
production of X-rays.
The CAT scan was independently conceived by
Allan McLeod Cormack and Godfrey Hounsfield.
X-rays are emitted when high-energy electrons strike
a target.
There are two processes by which X-rays can be produced:
characteristic X-rays and Bremsstrahlung.
X-rays are often categorised by the terms hard X-rays
and soft X-rays, which are descriptive terms indicating
the relative penetrating ability of the X-ray beam.
The basic components of an X-ray machine are an X-ray
tube, a filter, a collimator, a platform to situate the
patient, a grid and a detector.
Contrast compounds can be ingested or injected to
assist X-ray imaging of vascular (veins and arteries) or
hollow soft tissue structures.

medical
physics

Benefits of digital technology to all X-ray systems


include:
lower X-ray doses to patients
images that can be enhanced and processed,
transmitted for remote diagnosis and easily stored
and retrieved.
A computed axial tomography (CAT) or computed
tomography (CT) scanner is a diagnostic device that
uses an X-ray tube that is rotated around the patient.
A detector collects the X-rays and digitises the
information.
The development of CAT scan technology was enabled
by the development of microcomputers and detector
technology that could digitise data.
CAT scanners use powerful image-processing software
to recreate images from the scan data collected.
Common applications for CAT scans include:
identifying trauma injuries to the lungs, heart,
spleen, kidneys and liver
planning for and assessing the results of surgery
planning radiation treatments for tumours
detection of osteoporosis by measuring bone density.
CAT scans generally provide higher resolution images
across a greater range of body tissues than either
radiographs or ultrasound.

Review questions
Physically speaking
This is a list of topics that have been discussed
throughout this chapter. Create a visual
summary of the concepts in this chapter by
constructing a mind map linking the terms.
Add diagrams where useful.

X-ray

Hard

Soft

Characteristic

Bremsstrahlung

Penetration
ability

Resolution

Diagnostic

Benefits

2D scan

Image
intensifier

331

17

Imaging
with X-rays

Reviewing
1 Outline two ways in which X-rays are produced in an
X-ray tube.

2 Describe the differences between characteristic X-rays


and Bremsstrahlung.

3 Using a chest X-ray as an example, recount how an


X-ray image of the body is produced.

4 Compare the wavelengths of hard and soft X-rays.


5 A student wrote the statement: Soft X-rays could also
be called ultraviolet rays. Assess this statement.

6 Assess the impact on society of doctors being able to


examine images such as Figure 17.4.2, showing part
of a patients arm.

7 For more than 100 years, X-ray images have been


used to examine one type of tissue in the body more
than any other. Identify this tissue and outline why
X-rays are so successful in producing images of this
type of tissue.

8 Figure 17.8.1 shows the chest area of two patients.


Identify the types of X-ray images shown and compare
the information provided by each image.

Re

iew

332

Q uesti o

Figure 17.8.1

9 Explain why it is sometimes necessary for


radiographers to take two radiographs of the chest
area, one from the front and one from the side to
provide the same information that can be seen in
a single CAT scan of the chest.

10 Compare the effectiveness of CAT scans and


radiographs to resolve soft tissue in the body of
a patient.

11 Compare the advantages of the 2D and 3D images.


12 Compare the passage of the X-rays through a patient
given a conventional chest X-ray (radiograph) and
a CAT scan of the chest region.

13 Explain how a CAT scan is produced.


14 Outline the main technological development that took
place in the 1970s that made CAT scans possible.

15 Assess the impact of particular advances in physics


on the development of CAT scans.

Imaging
with light
Look and see
Optical fibres have had a great impact on society, although the fibres
themselves are usually hidden from view. One of those impacts is in a
doctors endoscope, a device to peer inside the human body without
major surgery. Optical fibres allow light to be directed onto an area of
interest and then lenses relay an image back to the doctor. Combined
with tiny surgical instruments, these devices allow minimally invasive
surgery to reduce the trauma of surgery, having a radical effect on
the treatment and recovery of patients with problems ranging from
a torn knee cartilage to leaky heart valves.

18
endoscope, endoscopy, total internal
reflection, critical angle, optical fibres,
core, cladding, coherent fibre bundle,
non-coherent fibre bundle, biopsy

18.1 Endoscopy
An endoscope is a medical device used to shine visible light into a patients
body and relay an image out, to allow organs, tissues and cavities to be seen.
Light is directed to the area of interest through a flexible bundle of optical fibres.
Lenses are used to focus an image of the area of interest onto a separate bundle
of optical fibres that transfer the image out of the body. The image is usually
processed electronically to display it on a screen in real time or to record still or
moving images for later use.
The process of using an endoscope to examine a patient is called endoscopy.
The tube-like part of the endoscope, which enters the body, is typically less than a
centimetre in diameter. Depending on the purpose of the endoscopy, the tube is
inserted via a natural opening of the body, or through a small cut made in the skin.
Miniature surgical instruments can be attached to the part that enters the body and
these are controlled from outside the body by a doctor using control wires that pass
through another narrow tube that is also a part of the endoscope. Tissue samples
can be removed from the patient and surgical procedures can be carried out.
The main advantages of endoscopy are that it is minimally invasive
and provides real-time images in true colour. The operator can manipulate the
endoscope to obtain the best view of the area of interest inside the patient.

Figure 18.1.1

A gastric ulcer seen through


an endoscope

333

18

Imaging
with light

PHYSICS FEATURE

3. Applications and uses of physics

Optical fibres

Explain how an endoscope works in relation to


total internal reflection.

ou will recall learning about the properties and


behaviour of light (see in2 Physics @ Preliminary
Chapter 8). The path of a light ray meeting a boundary
between two transparent materials changes if there is
a difference in the refractive indices of the two media.
This process is called refraction.
Total internal reflection may occur at a boundary
if the light is travelling from a medium with a higher
refractive index ni to one having a lower refractive
index nr and the angle of incidence exceeds the
critical angle (c) given by:
sinc =

optical fibre is transmitted along the optical fibre by


total internal reflection, as illustrated in Figure 18.1.4.
Most optical fibres used for communications
purposes transmit light in the near infra-red region
of the electromagnetic spectrum at wavelengths of
8001600nm. The glass used in optical fibres has
maximum transparency at these wavelengths.
(Remember that the wavelength of visible light is
between about 400 and 700nm.)
Fibres used in medical endoscopy must be able to
transmit light in the visible region of the spectrum.
Because of the short distances involved (usually less
than about 2 m), the fibres do not need to have to
have the same transparency as fibres used for
communication technology, nevertheless endoscopic
fibres are made from high quality, homogeneous, high
transparency glass. Homogeneity is an important
property of the fibre because any irregularities would
distort and degrade the image.

ni
nr

Total internal reflection (Figure 18.1.2) is the


basic principle underlying the operation of relatively
large diameter optical fibres.
Optical fibres are usually made of glass with
two layersthe light-carrying, higher index inner core
and the surrounding lower index cladding (Figure
18.1.3). Light entering the core at one end of the
normal

normal

normal

normal

air

nr

air

water

ni

water

air

ni

water

Figure 18.1.2

nr

90

nr

air

nr

ni

water

ni

Light moving from high to low refractive index materials may be totally internally reflected.

jacket

core

core
cladding
cladding

Figure 18.1.3
334

Figure 18.1.4
A cross-section of an optical fibre

Light is transmitted along an optical fibre by


total internal reflection.

medical
physics
The main parts of an endoscope are shown in Figure
18.1.5. The endoscopic tube that is inserted into the patient
typically contains the following parts:
a bundle of optical fibres to transmit light to the point
of observation
a coherent bundle of optical fibres to carry the image of
the tissue to the observer
a system of lenses to focus an image of the tissues under
examination onto the optical fibre bundle. At the viewing
end of the optical fibre bundle, more lenses allow direct
viewing of the image by eye, or connect to a camera that
feeds a video screen or computer
suction tube to remove blood and other loose, obscuring
tissue material from the area under inspection
an inlet and an outlet to permit the area under observation
to be flushed with clear saline (salt water) solution to
increase visibility
control lines to manipulate the tube inside the patient.
These vary in complexity, depending on the task undertaken.
A simple gastroscopy may not require complex controls.
Endoscopes used for surgical procedures may require fine
control over the position of the lens at the tissue end
miniature remote-controlled surgical instruments may be
present, ranging from simple suction tools to more
elaborate surgical tools used in operations.

power source
video out etc.

The endoscope
is a thin,
flexible fibreoptic telescope.

light to
see inside
Various devices
can be passed
down side channels.
These can be
manipulated by
the doctor to take
specimens etc.

doctor looks
down endoscpe

oesophagus

stomach

duodenum

Figure 18.1.5

Worked example
Question
Calculate the critical angle for a glass optical fibre with a core refractive index of 1.48
and a cladding refractive index of 1.46.

endoscope passed down


oesophagus into stomach

Gastroscopy is the examination of the upper gut


(oesophagus, stomach and duodenum) using a
flexible fibre optic endoscope.

PRACTICAL
EXPERIENCES
Activity 18.1

Activity Manual, Page


136

Solution
The critical angle will be a value of i such that the angle of refraction is 90.
ni = 1.48, nr = 1.46, r = 90
Snells law states:

ni sini = nr sinr

Rearrange this to make i the subject:



Substitute values:

n sin r
i = sin1 r

ni
1.46 sin 90
= 80.5
i = sin1
1.48

Coherent and non-coherent bundles of fibres


If all of the fibres in an optical fibre bundle are parallel along the full
length of the bundle, so that there is a uniform one-to-one correspondence
between the positions of the fibres at one end of the bundle and the positions
of the opposite ends of each fibre at the other end of the bundle, then the bundle
335

18

Imaging
with light

is said to be a coherent fibre bundle. The light from an object projected onto
the ends of the fibre bundle by a simple lens will travel along the fibres and
emerge as a corresponding image of the object at the other end of the bundle
(Figure 18.1.6). This is the principle of the medical endoscope.
In a non-coherent fibre bundle, one or more of the fibres swap positions
relative to each other at opposite ends of the fibre. Typically, the arrangement of
fibres along the bundle is random, so that although the light travels along each
fibre, no clear image is produced at the exit end (Figure 18.1.7). A non-coherent
fibre bundle is adequate to simply transmit light to the point where observations
are being made with the endoscope. White light is used so that the doctor can
observe the tissues in true colour.

Discuss differences between


the role of coherent and noncoherent bundles of fibres in
an endoscope.

object

object

no image

image
1
2
3
4
5

1
2
3
4
5

Figure 18.1.6

1
2
3
4
5

The arrangement of fibres in a coherent bundle allows


an image to be transmitted by the bundle.

Figure 18.1.7

2
4
5
1
3

A non-coherent bundle does not transmit


a sensible image.

Checkpoint 18.1
1
2
3
4

Outline how an optical fibre allows you to see something inside the body.
Define total internal refraction.
Explain why infra-red light is used in communication applications but visible light is used in medical applications.
Identify the difference between coherent and non-coherent bundles of fibres.

18.2 Medical uses of endoscopes


Endoscopes are used to visually examine the inside of a patients body. Being able
to see tissues allows doctors to diagnose diseases such as ulcers and tumours, and
also to determine the nature and extent of injuries such as damaged cartilage and
ligaments in joints.
Endoscopes used for specialised purposes have different names. An arthroscope
is used to examine joints, a bronchoscope to view the lungs, a laparoscope to
view female reproductive organs and an otoscope is used to view the ear.
A disadvantage of endoscopic examination is that it only allows the surface
of tissues to be viewed, and so its use is limited to problems that cause visible
changes to the surfaces of tissues. It can, however, be linked with ultrasound
imaging to produce better results than ultrasound scans from outside the body.
336

medical
physics
A risk in using an endoscope is that the part of the endoscope inside the body
can tear tissues while it is being moved about. Endoscopic examination of the
bowel presents a particular risk, because the bowel contains bacteria, which, if they
enter the bloodstream, can produce a fatal infection. However, the risk in using
an endoscope is much less than the risk encountered if the abdomen had to
be opened up in conventional surgery. The fact that a patient usually has to be
sedated or anaesthetised presents another minor risk to the patient, although this
type of risk is the lesser problem when weighed against the alternative of an
undiagnosed or untreated problem.
Endoscopes that have been modified with surgical instruments can be used
to remove tissue samples for testing. This process is called a biopsy, and this is
one of the most common endoscopic procedures.
A common example of a biopsy is the removal of polyps or other growths from
the intestine for further examination and testing. Endoscopy reduces this risk because
the incisions and amount of cutting is minimised. Figure 18.2.1 shows a biopsy
being performed within the abdomen. The sample of tissue cut off can then be placed
or sucked into a tube attached to the endoscope and withdrawn from the body.
Minimally invasive surgery is conducted using optical fibre instruments that
are often an integral part of the endoscope. Surgery that is commonly carried out
with the aid of an endoscope includes removal of the gall bladder and the prostate,
and repairs to damaged tissues in joints. A common joint operation is the repair
of the anterior cruciate ligament in the knee (Figure 18.2.2). This part of the
anatomy is frequently torn in sports such as netball and football, which involve
vigorous twisting forces on the knees. People once condemned to months off the
sporting field by knee injuries are now returning to their sport within weeks,
because of endoscope-aided surgery.

Explain how an endoscope is


used in:
observing internal organs
obtaining tissue samples of
internal organs for further
testing.

Figure 18.2.1

A biopsy from within


the abdomen

Figure 18.2.2

External view of surgery to


repair an anterior cruciate
ligament

Developments in endoscopy

n endoscopic capsule (Figure


18.2.3) is an endoscope with
no optical fibres! This small
capsule can be swallowed by the
patient and contains a wireless
camera that can pass through the
intestinal system and report via
video link what is observed.
This will improve endoscopic
observation of the digestive tract.

optical dome
lens holder
illuminating LEDs
lens
battery

antenna

Figure 18.2.3

An endoscopic capsule

Checkpoint 18.2
1
2

List the advantages and disadvantages of endoscopy.


Outline how a biopsy is done.
337

18

Imaging
with light

PRACTICAL EXPERIENCES
CHAPTER 18

This is a starting point to get you thinking about the mandatory practical
experiences outlined in the syllabus. For detailed instructions and advice, use
in2 Physics @ HSC Activity Manual.

Perform a first-hand
investigation to demonstrate
the transfer of light by optical
fibres.
Gather secondary information
to observe internal organs
from images produced
by an endoscope.

Activity 18.1: Optical fibres


Many light shops sell products known generally as optical fibre lights, which
consists of numerous fibre optic tubes through which coloured light is passed via
some colour-changing mechanism. Obtain one of these tubes and use a LED or
laser light to shine light through the tube. Change the direction of the illumination
by moving the end of the tube in different directions.
Equipment: optical fibre (e.g. from an optical fibre lamp), light source (LED),
power supply.
Discussion questions
1 Identify what optical fibres are made of and explain how something
so brittle can be made so flexible.
2 Explain how light is transferred down an optical fibre.
3 Explain how optical fibres are used in an endoscope to transfer images
from inside the body.

Figure 18.3.1

338

An optical fibre lamp

Chapter summary




An endoscope is an optical instrument that allows


real-time observation of internal organs.
Light is transmitted through optical fibres by total
internal reflection.
Cheaper non-coherent bundles of fibres carry the light
into the body.
More expensive coherent bundles carry the image out
of the body.
Advantages of endoscopy
The tissues and organs are seen in real colour.
Imaging is in real time, enabling the doctor to
respond to what is seen.

medical
physics

It allows minimally invasive tissue sampling and


minor surgery that is safer and cheaper, with quicker
recovery than open surgery.
The process uses non-ionising radiation, namely
light, an advantage over X-rays.
Disadvantages of endoscopy
It is more time consuming than ultrasound and
X-rays.
It presents minor risks to the patient, especially if
an anaesthetic is required. Operations on the bowel
involve a risk of infection.
Only the surface of tissues is visible.

Review questions
Physically Speaking

5 Explain how an endoscope is used to obtain tissue

Use some of the chapter key words to complete


the following paragraph.

6 Recall an investigation that you carried out to

The medical technique known as ________________ allows


minimally invasive procedures such as ________________
to be performed via a body opening or a small incision.

samples from the stomach of a patient.


demonstrate the transfer of light by optical fibres.

7 Assess the advances in medical techniques as the


result of the use of endoscopes.

8 Explain why endoscopic surgery is often referred to


as keyhole surgery.

The ________________ allows a surgeon to view internal tissues


through the ________________ while it is illuminated via the
________________. An ________________ works because light is

Solving problems

confined to the ________________ enclosed by the

9 Determine the angle of refraction of light that passes

________________ of a fibre by ________________.

Reviewing
1 Explain the importance of total internal reflection to
the operation of an endoscope.

2 Compare the structure of coherent and non-coherent


fibre bundles.

3 Compare the function of coherent and non-coherent

from water (n = 1.33) to glass (n = 1.48) at an


incident angle of 30.

10 Determine the critical angle for a material with


refractive index of 1.4 that is immersed in:
a glass (n = 1.48)
b water (n = 1.33)

11 A critical angle of 48.75 is measured at the


boundary between air (n = 1) and another medium.
Calculate the refractive index of the medium.
Can you identify the probable medium?

fibre bundles in an endoscope.

4 Explain how an endoscope is used to observe

Re

iew

Q uesti o

internal organs.

339

19
radioactive decay, radiation,
radioactive, radioisotopes, nucleons,
atomic number, mass number, isotopes,
alpha decay, alpha particle, beta decay,
beta particle, positron decay, antiparticle,
positron emission tomography (PET),
gamma decay, half-life,
radiopharmaceuticals, nuclear reactor,
cyclotron, gamma camera, bone scan,
collimator, scintillator, single-photon
emission computed tomography (SPECT)

Imaging with
gamma rays
Radioactivity can be good!
Images made using ultrasound, X-rays and visible light can show
anatomical structures rather well. They all involve sending various
forms of energy into the body. A rather different approach is to
introduce radioactive elements into a persons body and study the
radiation that emerges. Images can be made of the bones as well
as soft tissues including the brain, heart, liver and thyroid. Rather
remarkably, this approach allows the production of images from
outside the body that show how a persons organs are functioning.
Here we consider two of these minimally invasive but powerful
diagnostic tools: bone scans using radioactive tracers
and positron emission tomography.

19.1 Isotopes and radioactive decay


Outline properties of
radioactive isotopes and their
half-lives that are used to
obtain scans of organs.

340

For the 30 or so lightest elements, the number of protons is roughly the same
as the number of neutrons in the nucleus in most of their naturally occurring
isotopes. These isotopes are stable. However, many elements have isotopes whose
nuclei have too few or too many neutrons. These isotopes are unstable and
undergo radioactive decay in which they change and emit radiation. The type
of radiation that is emitted depends on the nature of the decay (see in2 Physics
@ Preliminary section 15.5).
There are 82 elements that have at least one stable isotope. The stability
depends on the ratio of protons to neutrons. As the atomic number increases,
the ratio of neutrons to protons needed for stability also increases.
Many elements have naturally occurring unstable isotopes. These are
called radioactive isotopes or radioisotopes. The nucleus of a radioisotope
(the parent nucleus) usually transforms itself into another nucleus (the daughter
nucleus) by emitting particles and energy. It will decay repeatedly until it forms
a daughter nucleus that is stable.

medical
physics

increasing
distance

Isotopes

maximum
shielding

reducing time of exposure

Figure 19.1.1

Reducing the danger from radiation involves increasing distance,


maximising shielding and reducing the time of exposure.

Alpha decay
Some unstable nuclei decay by emitting a particle that
contains two protons and two neutrons in a process known as
alpha decay. The remaining nucleus has a mass number that is
reduced by 4 and an atomic number that is reduced by 2. This
particle emitted from the nucleus is called an alpha particle
(-particle). Alpha particles are helium nuclei ( 42 He ) and they
rapidly become helium atoms, as they gain electrons from the
surroundings. Such reactions are the source of most of the
helium on Earth.
For example, radioactive uranium-238 undergoes alpha decay
to produce thorium-234. The daughter nucleus has 2 protons
less than the parent nucleus and so it is a different element. In
a nuclear reaction, both mass number and charge are conserved,
and the decay process can be described by an equation:
238
92 U

234
90Th

n atomic nucleus consists of nucleons

protons and neutrons. The number of


protons in the nucleus is called the atomic number,
while the total number of nucleons is called the
mass number. Atoms of the same element with
different numbers of neutrons are called isotopes of
that element. Many isotopes occur naturally, but
some are made artificially.
In section 15.4 of in2 Physics @ Preliminary
we represented this information in a compact form.
For example, an important isotope of fluorine is:
Mass number
18
Atomic number
9

It is called fluorine-18, with 18 being the mass


number. Other isotopes that are important in
medicine include carbon-14, iodine-131,
phosphorus-31 and technetium-99. Hydrogen is the
only element that has special names for its three
isotopes: hydrogen, deuterium and tritium.

electron
proton

Beta decay
When a radioactive nucleus undergoes beta decay, a
neutron changes into a proton, releasing a high-energy electron
in the process. The electron is ejected from the nucleus with such
a high velocity that it totally escapes the atom. An electron
(represented as e or 10e) emitted from the nucleus in this way is
called a beta particle (-particle). An electron has only 1/1836

1
1H hydrogen

neutron
2 neutrons

+ 42 He

or illustrated by a diagram (see Figure 19.1.3).


An alpha particle can only travel a few centimetres in air
before it loses its kinetic energy and gains electrons to become a
helium atom. In living tissue the range is about 50 m, about
half the width of a human hair. Due to their relatively large mass,
alpha particles carry a lot of energy and have a high ability to
ionise the surrounding medium, making them very dangerous
to living cells. They are not used very much in medicine.

proton
electron

Figure 19.1.2

proton
3
1H tritium

electron
2
1H deuterium

Isotopes of hydrogeneach with one proton


and one accompanying electron

daughter nucleus
Th-234
parent nucleus
U-238

Figure 19.1.3

4
2He

alpha particle
(helium nucleus)

Uranium-238 undergoes alpha decay

341

19

Imaging with
gamma rays

14
7N
14
6C

daughter
nucleus

parent
nucleus

Figure 19.1.4

beta
particle

of the mass of the proton, and its mass number is 0. Its negative charge gives it
an atomic number of 1.
Beta decay increases the atomic number of the nucleus by one but the mass
number does not change. There is one less neutron but one more proton and
hence the total number of nucleons does not change.
Carbon-14 decays by beta decay to nitrogen-14. The process can be described
using an equation or a diagram (Figure 19.1.4).

Carbon-14 decays to
nitrogen-14 and an electron.

14
6C

147 N +

0
1 e

The two products are a stable nitrogen nucleus and a beta particle (electron).
Beta particles have a range that depends on their energy. In air they travel several
metres, but they travel only a few millimetres in human tissues.

Beta decay: positron production

Identify that during decay of


specific radioactive nuclei
positrons are given off.

Some nuclei of radioactive elements are unstable because they have too many
These become
protons in the nucleus, relative to the number of neutrons.
more stable by a form of beta decay called positron decay. A positron
0
(represented as e+ or + or sometimes +1 e ) is the antiparticle of the electron.
It has the same mass as an electron and the same magnitude charge as the
electron, but it is positively charged. In positron decay, a proton in the nucleus
decays to a neutron and a positron. The atomic number of the decaying nucleus
decreases by one but the mass number remains the same. The positron is ejected
with high kinetic energy from the nucleus.
The most important positron-emitting isotope is fluorine-18, an artificially
produced isotope. Its decay equation is:
18
18
0
9 F 8 O + +1 e

Discuss the interaction of


electrons and positrons
resulting in the production
of gamma rays.

Carbon-11 and oxygen-15 are also artificially produced positron emitters.


When a positron and an electron collide, they mutually annihilate
(see Physics Phile Evil twins p 73). Their total mass is converted into energy,
producing two identical gamma (g) ray photons with a total energy consistent
with Einsteins famous equation E = mc2 where m is the total mass of the two
particles (the mass of each electron is 9.1 1031 kg) (see section 3.4). The
energy of each gamma ray is
E = 9.1 1031 (3 108)2 = 8.2 1014 J = 511 keV
The two gamma rays emerge in opposite directions to conserve momentum
(Figure 19.1.5). In positron emission tomography (PET) (section 19.5) it is
these gamma rays, not the positrons, that are detected and used to produce
medical images.
Both kinds of beta decay are always accompanied by the creation of neutrinos,
which are almost undetectable and not relevant to medical applications.

positron
emitter
e+

Gamma ray emitters


gamma ray
photon

Figure 19.1.5

342

e +
e

gamma ray
photon

A positron and an electron


annihilate to produce two
gamma rays.

The most widely used medical isotope is technetium-99. It is a beta emitter that
decays to ruthenium-99. The higher energy form of technetium-99, called
metastable technetium-99 or technetium-99m, is also unstable and becomes
more stable by emitting electromagnetic radiation. At the energy involved, this
The gamma ray carries away no charge or mass
radiation is a gamma ray.
and so the nucleus remains technetium-99. It is the gamma decay of
technetium-99m that is the medically important decay.

medical
physics
Gamma decay also frequently accompanies alpha or beta decay. Cobalt-60 is
a well-known beta and gamma emitter.
Gamma rays have no charge and so can pass easily through matter. To achieve
the 95% absorption of the gamma energy from a cobalt-60 source, the rays must
pass through 60 mm of lead, 100 mm of iron, or 330 mm of concrete.

alpha
beta
X-ray (medical)
gamma ray
neutrons

concrete
human hand

Figure 19.1.6

aluminium

thin
lead

thicker
lead

The penetrating ability of different forms of radiation

Checkpoint 19.1
1
2
3

Describe what happens to an unstable nucleus.


Identify two naturally occurring isotopes of the same element.
Compare an electron with a positron.

Atom-sized
energy units

articles or gamma rays


produced in radioactive decay
carry away energy. The SI unit for
energy is the joule, but thats a lot
of energy for a tiny particle or even
a single gamma ray. A more atomsized unit of energy is the electron
volt (eV)the energy acquired by
an electron in accelerating through
a potential difference (voltage) of
one volt. More generally, the energy
acquired by a particle of charge q
is given by:
E = qV
So the energy acquired by the
electron is:
E = (1.6 1019 C)(1 V)
= 1.6 1019 J
So 1.6 1019 J = 1 eV
As with other units, a large
amount of energy can be expressed
as keV (kilo-electron volts) and
MeV (mega-electron volts) and so on.

19.2 Half-life

Amount (grams)

The time it takes for half the mass of the parent isotope to decay into
Outline properties of
daughter nuclei is defined as the half-life of the isotope. After one half-life, only
radioactive isotopes and their
50% of the original parent isotope remains; 50% of that remaining amount
half-lives that are used to
obtain scans of organs.
decays after another half-life, leaving just 25% of the original parent isotope
and so on.
The half-life of a radioactive isotope can be deduced
from a graph showing the mass of the remaining
2000
Strontium-90 decay
radioactive atoms of the element plotted against time. In
1800
Figure 19.2.1, the time taken for 2000 g of strontium-90
1600
to be reduced to 1000 g is 28.1 years, the half-life of
1400
1200
strontium-90. The daughter isotope is yttrium-90, which
1000
rapidly decays to zirconium-90, which is stable.
800
This mathematical model, called exponential decay, is
600
applicable to all forms of radioactive decay. The rate of
400
200
radioactive decay of an isotope is not affected by changes
0
in physical conditions such as temperature or pressure.
0.0 28.1 56.2 84.3 112.4 140.5 168.6 196.7 224.8 252.9 281.0 309.1
Time (years)
The decay rate is unchanged by any chemical reactions
(or compounds in which the radioactive isotope may be
Figure 19.2.1 The mass of strontium-90 remaining versus time,
incorporated). Every radioisotope has its own half-life.
from an original sample of 2000 g
343

19

Imaging with
gamma rays

Checkpoint 19.2
1
2

Define half-life.
Outline how you would find the half-life of an isotope from a graph showing its mass in a sample versus time.

19.3 Radiopharmaceuticals:
targeting tissues and organs

Outline properties of
radioactive isotopes and their
half-lives that are used to
obtain scans of organs.

Radioisotopes are used to produce functional images of the body. They are used
to examine blood flow to the brain, to assess functioning of the liver, lungs, heart
or kidneys, to assess bone damage, and to confirm other diagnostic procedures.
In contrast to the imaging techniques already discussedultrasound images,
X-ray images and CAT scansthe use of radioisotopes can show how the body is
functioning, rather than simply showing detailed images of tissues and organs in
the body. Diseases such as cancer alter chemical processes in the body and images
produced using radioisotopes can reveal these changes. Radioactive chemicals
used in medicine are called radiopharmaceuticals.

Choose your element

Describe how radioactive


isotopes may be metabolised
by the body to bind or
accumulate in the target organ.

344

Different isotopes of the same element have identical chemical properties.


This is important in medical applications because radioactive isotopes can
be substituted for non-radioactive atoms normally used by the body.
Radiopharmacologists are able to attach various radioisotopes to biologically
active substances and introduce them into the body. The radioisotope is then
incorporated into the normal biological processes.
Radiopharmaceuticals are chosen so that when they enter the body they will
circulate around the body and be absorbed by the organ of interest. The
radiopharmaceutical may be a substance that is specifically used by a particular
organ or it may be part of a molecule that is used by the organ to be imaged.
Radiopharmaceuticals are prepared by replacing one of the atoms in the
molecules of that substance with a radioactive atom. This process is sometimes
referred to as tagging or labelling. When the radiopharmaceutical is placed in
the body, it accumulates in the target organ and so the radiation that is emitted
from that organ during the imaging process will be greater than the amount
emitted from other organs and tissues.
The choice of a radioisotope used for medical imaging is based on the
following criteria. It must:
produce gamma rays (directly or indirectly) since only gamma radiation is
likely to leave the body
have a half-life that is long enough for the molecule to enter the metabolic
processes, yet short enough to minimise the radiation dose to the patient
be taken up rapidly into the desired tissuethis is achieved by incorporating
the radioisotope into an appropriate compound that is metabolised by the
target tissue in the body
be rapidly excreted from the body. Appropriately chosen compounds are
broken down and excreted as part of the normal body chemistry.

medical
physics
Radiopharmaceuticals are used in very small quantities for diagnostic work.
Just enough is administered to obtain the required information before the
radiopharmaceutical decays, therefore minimising cell damage from the radiation.
The radiation dose received is similar to that from diagnostic X-rays.

Where to get your radioisotopes


Radioisotopes are produced in two main ways: in a nuclear reactor or in a
cyclotron particle accelerator.
In a nuclear reactor, fission (splitting) of heavy nuclei produces large numbers
of neutrons. The target element to be converted into a radioisotope is placed in
the path of these neutrons and the nuclei absorb one or more neutrons,
producing an unstable isotope with an excess of neutrons. These typically decay
via beta decay.
Different isotopes can be produced in a cyclotron. In a cyclotron, protons
are accelerated in a vacuum and fired into the nucleus of a target atom, to
create isotopes that have an excess of protons. These isotopes typically decay
via positron emission.
In Australia, most radioisotopes for medical purposes are prepared at the
Australian Nuclear Science and Technology Organisation (ANSTO) OPAL
nuclear reactor at Lucas Heights in Sydney and at the National Medical Cyclotron
at the Royal Prince Alfred Hospital in Sydney (see Physics Focus p353). The
ANSTO reactor and the cyclotron make various radiopharmaceuticals, including
some of those listed in Tables 19.3.1 and 19.3.2.
Nuclear reactors and cyclotrons are expensive to build and operate. The cost
of these machines and the increasing demand for radioisotopes has added
significantly to health costs in Australia and this has an impact on society through
increased medical insurance costs. However, it is not practical to import many
medical radioisotopes from overseas because of the short half-lives and the
difficulty of transporting them safely.

Figure 19.3.1

The National Medical


Cyclotron Facility at Royal
Prince Alfred Hospital

Table 19.3.1 Examples of radioisotopes produced in a nuclear reactor


Isotope

half-life

Cobalt-60

5.3 years

Iodine-131

8 days

Phosphorus-32

14.3 days

Molybdenum-99

2.75 days

Technetium-99m

6 hours

Emission
,
,

Uses
External beam radiotherapy
Cancer diagnosis and imaging of the thyroid gland
Treatment of excess red blood cells
Parent isotope used in a generator to produce technetium-99m, the most
widely used isotope in nuclear medicine
To image the skeleton and heart muscle in particular, but also for brain,
thyroid, lungs, liver, spleen, kidney, gall bladder, bone marrow, salivary and
lacrimal glands, heart blood pool, infection and numerous specialised
medical studies

Table 19.3.2 Examples of radioisotopes produced in a cyclotron


Isotope

Half-life

Carbon-11

20.3 minutes

Emission Uses
+

Nitrogen-13

10 minutes

Oxygen-15

2.03 minutes

Fluorine-18

109.8 minutes

Iodine-124

4.5 days

+
+
+
+

In PET for studying brain physiology and pathology, for investigating


epilepsy; have a useful role in cardiology
In PET scans to tag ammonia and trace protein metabolism
In water for PET scan on blood flow in muscle tissue
For investigating tumours in the breast, prostate, liver and brain
To investigate cancer, notably in the thyroid gland

345

19

Imaging with
gamma rays

PHYSICS FEATURE

3. Applications and uses of physics

Technetium-99m

he radioisotope technetium-99m (Tc-99m) is


used in more than half of all nuclear medicine
procedures. It is an isotope of the artificially produced
element technetium and is almost ideal for use in
nuclear medicine scanning.
Technetium-99m has a half-life of 6 hours and
when it decays it emits a single gamma ray with
energy of 140keV. It does not emit any alpha or beta
particlesan advantage since these cause biological
damage. A further advantage of Tc-99m is that it has
several electron valence states, so it has versatile
chemical properties and can be used to produce a
wide variety of pharmaceuticals.
A technetium generator, consisting of a lead vessel
enclosing a glass tube containing the radioisotope
molybdenum-99, is supplied to hospitals by ANSTO.
Molybdenum-99 (Mo-99) has a half-life of 66 hours
and it decays to Tc-99m by normal beta decay. The

Tc-99m is washed out of the lead or glass pot as


required by passing a saline solution through the glass
tube, a process called elution. After about 2 weeks the
generator is returned for recharging.
The two nuclear reactions involved in Tc-99m
production can be represented as:

99 m
99
42Mo 43 Tc

99 m
43 Tc

+ 01e half-life 2.75 days

99
43Tc +

half-life 6.01 hours

Technetium-99m has a half-life of 6 hours, which


is long enough to examine metabolic processes, yet
short enough to minimise the radiation dose to
the patient. It is usually injected into the patients
body and eliminated from the body via the kidneys.
Technetium-99m allows the diagnosis of many
conditions in the brain, lungs, heart, bones and
other organs, avoiding the use of surgery.

Checkpoint 19.3
1
2
3
4

Explain the process of tagging.


Discuss why only small amounts of radioisotopes are used in patients.
Outline how radioisotopes are made.
Explain why we do not just import all radioisotopes from overseas.

digital signal processing unit


photomultipliers
NaI (Tl) detector
lead collimator

gamma photons

radioactive tracer in body

Figure 19.4.1

346

Principle of a
gamma camera

19.4 The gamma camera


A gamma camera (Figure 19.4.1) detects gamma rays emitted by
a radiopharmaceutical in the patients body. The camera reveals the
distribution of radioactive material in a patient and this distribution is
determined by the uptake of the radiopharmaceutical, which is dependent
on the functionsnormal or otherwiseof the body.
In a bone scan, for example, technetium-99m methylene diphosphonate
(Tc99m-MDP) is usually injected into the patient. The radioactive
phosphate travels through the blood and is metabolised and accumulates
in the bones.
The patient lies under a gamma ray camera. The gamma rays
coming from the radioactive tracer in the body travel out of the body in all

medical
physics
directions. Unlike visible light, gamma rays cannot be focused to form an image.
Instead a lead collimator with parallel holes passing from the bottom to the top is
used to allow only those gamma rays directed through the holes to
reach the detector itself.
The detector is a scintillator, made of a crystal of sodium iodide containing
traces of thallium. The thallium impurity causes the crystal to scintillate
(i.e. to emit a small flash of light) when a gamma ray enters the crystal. The flash
is very faint, so sensitive devices called photomultipliers are used to detect the
flash and amplify its effects, converting the result into an electrical signal.
A computer processes the signals to produce an image that may be viewed
directly on a screen, printed or stored electronically, since the information is
all digitally encoded.
Regions of bone having a high metabolic activity appear darker on the bone
scan because more of the radiopharmaceutical is absorbed (Figure 19.4.2).
Conditions that may cause increased metabolic activity include cancer cells
multiplying in the bone, healing of a fracture or damage from arthritis.
A gamma camera is also used in single-emission computed tomography
(SPECT). The camera acquires two-dimensional images from multiple directions.
These are then assembled into a three-dimensional image in a computer, in a
similar way to other three-dimensional tomographic imaging techniques.

PRACTICAL
EXPERIENCES
Activity 19.1

Activity Manual, Page


140

Checkpoint 19.4
1
2

Outline how a gamma camera creates an image of the body using


gamma radiation.
Explain how the grey scale in the bone scan (Figure 19.4.2) can
be interpreted.
Figure 19.4.2

19.5 Positron emission tomography


Positron emission tomography (PET) is a functional imaging technology that
allows physicians to assess chemical changes related to how the body is
functioning. In this way, PET is like a bone scan. PET imaging can analyse sugar
metabolism, blood flow, oxygen use and a long list of other vital physiological
processes. It is often used to investigate brain function and diseases affecting the
brain, such as epilepsy, schizophrenia and Parkinsons disease, as well as
investigating which parts of the brain are active during specific activities.
PET imaging uses positron-emitting radiopharmaceuticals to obtain
images. Positron-emitting isotopes exist for carbon, oxygen, nitrogen, fluorine
and others, which allows these radioisotopes to be substituted into many
naturally occurring substances used by the body (e.g. proteins, water and
sugarsespecially glucose). An increased level of metabolic activity in a part of
the body associated with the chosen radiopharmaceutical causes a higher
radiopharmaceutical concentration in that part and hence the gamma rays from
those parts are more intense than those from parts in which there is less
metabolic activity.

This bone scan shows numerous


hotspots along the spine, pelvis,
shoulders and ribs that indicate
that in this patient the breast
cancer has spread to the bone.

347

19

Imaging with
gamma rays

seeing words

hearing words

Figure 19.5.1

This PET image contrasts


the visual and auditory
stimulation of the brain.

For example, if glucose labelled with fluorine-18, a positron emitter, is given


to a patient who is then asked to perform a task, the regions in the brain in which
the isotope concentrates most are the regions that are actively involved in that
behaviour. Figure 19.5.1 shows PET images that reveal the different active parts
of the brain of a person reading (seeing) written words or hearing the same words
spoken by another person.
The radiopharmaceuticals are usually injected into a patient, although oxygen-15
can be inhaled and absorbed into the body through the lungs. The radioactive atoms
decay, emitting positrons that travel about 2mm before encountering an electron.
Mutual annihilation occurs and two gamma rays are emitted in opposite directions.
In a PET scanner these gamma rays trigger pairs of gamma-ray detectors that are
arranged in a ring around the patient (Figure 19.5.2).
When two detectors directly opposite each other record photons within a few
nanoseconds of each other (called a coincidence pair), the data are stored for
processing to produce the image. Non-coincident hits on a single detector are
ignored because they do not provide useful information about the location of the
annihilation that produced them.
Signals reaching opposite detectors are analysed by a computer, which
determines the position of the source of the annihilation event using the
intersections of the gamma ray trajectories. Regions that emit the most gamma-ray
photons produce a stronger signal than regions that emit fewer photons. A large
number of measurements from the ring of detectors surrounding the patient is
required to compute the distribution of the radioisotope in the body.
PET images dont show anatomical structures well, so they are usually shown
superimposed onto other medical images such as CAT scans, to clarify the location
of the PET signal (Figure 19.5.3).

b
detectors

coincidence
unit

Figure 19.5.2

348

valid
event

(a) A PET scanner showing coincidence detection of gamma rays by opposite detectors and (b) an example of a PET scanner

medical
physics
PET is a valuable diagnostic tool because the images that it produces show
differences in chemical activity in different parts of the body. These differences
may result from normal processes or they may be the result of diseases such as
cancer or other functional abnormalities in the body. The great diagnostic
benefit of PET results from the fact that it is possible to incorporate positron
emitters into a wide variety of radiopharmaceuticals which can be produced to
target specific organs or chemical processes in the body.
PET has provided new knowledge, in particular about the functions of the
normal brain, as well as of diseases that affect the brain. This knowledge has also
greatly modified brain surgery.

Figure 19.5.3

PRACTICAL
EXPERIENCES
Activity 19.2

Activity Manual, Page


143

Describe how positron emission


tomography (PET) technique is
used for diagnosis.

In this false colour PET scan (centre) two regions of high metabolic activity typical
of cancerous tumours are indicated by the bright white hot-spots (in the spine and
the prostate). This image is combined with a CAT image (left) to produce the fused
image (right) that accurately indicates their location.

Checkpoint 19.5
1
2
3

Explain what features in the body PET scans can be used to identify that X-rays and ultrasound can not.
Outline how the radioisotopes for a PET scan can be introduced into the body.
Explain how coincidence pairs in a PET scanner allow an image to be constructed.

349

19

Imaging with
gamma rays

PRACTICAL EXPERIENCES
CHAPTER 19
This is a starting point to get you thinking about the mandatory practical
experiences outlined in the syllabus. For detailed instructions and advice, use
in2 Physics @ HSC Activity Manual.

Activity 19.1: Bone scans


Perform an investigation to
compare a bone scan with
an X-ray image.

A bone scan is performed to obtain a functional image of the bones and so can be
used to detect abnormal metabolism in the bones, which may be an indication of
cancer or other abnormality. Because cancer mostly involves a higher than normal
rate of cell division (thus producing a tumour), chemicals
involved in metabolic processes in bone tend to accumulate in
higher concentrations in cancerous tissue. This produces areas
of concentration of gamma emission, indicating a tumour.
Compare the data obtained from the image of a bone scan
with that provided by an X-ray image.
Discussion questions
1 Identify the best part of the body for each of these
diagnostic tools to image.
2 Compare and contrast the two images in terms of
the information they provide.

Figure 19.6.1

Comparison of an X-ray and bone scan of a hand

Gather and process secondary


information to compare a
scanned image of at least one
healthy body part or organ with
a scanned image of its
diseased counterpart.
a

Figure 19.6.2

350

Activity 19.2: Healthy or diseased?


Typical images of healthy bone and cancerous bone are shown. The tumours show
up as hot-spots. Use the template in the activity manual to research and compare
images of healthy and diseased parts of the body.
Discussion questions
1 Examine Figure 19.4.2. There is a hot-spot that is not cancerous near the
left elbow. Explain.
2 In the normal scan (Figure 19.6.2a), the lower pelvis has a region of high
intensity. Why is this? (Hint: It may be soft tissue, not bone. Looking at
Figure 19.6.2b might help you with this question.)
3 State the differences that can be observed by comparing an image of
a healthy part of the body with that of a diseased part of the body.

Bones scans of (a) a healthy person and


(b) a patient with a tumour in the skeleton

Chapter summary

The number of protons in a nucleus is given by the


atomic number, while the total number of nucleons is
given by the mass number.
Atoms of the same element with different numbers of
neutrons are called isotopes of that element.
Many elements have naturally occurring unstable
radioisotopes.
In alpha decay an unstable nucleus decays by emitting
an alpha particle (-particle).
In beta decay, a neutron changes into a proton and
a high-energy electron that is emitted as a beta particle
(-particle).
In positron decay, a positronthe antiparticle of the
electronis emitted.

medical
physics

When a positron and an electron collide, their total


mass is converted into energy in the form of two
gamma-ray photons.
In gamma decay a gamma ray (g) is emitted from a
radioactive isotope.
The time it takes for half the mass of a radioactive
parent isotope to decay into its daughter nuclei is the
half-life of the isotope.
Artificial radioisotopes are produced in two main ways:
in a nuclear reactor or in a cyclotron.
A gamma camera detects gamma rays emitted by
a radiopharmaceutical in the patients body.
PET imaging uses positron-emitting
radiopharmaceuticals to obtain images using gamma
rays emitted from electronpositron annihilation.

Review questions
Physically Speaking

Reviewing

Below is a list of topics that have been discussed throughout


this chapter. Create a visual summary of the concepts in
this chapter by constructing a mind map linking the terms.
Add diagrams where useful.

1 Recall how the bone scan produced by a radioisotope

Radioactive
decay

compares with that from a conventional X-ray.

2 Analyse the relationship between the half-life of


a radiopharmaceutical and its potential use in the
human body.

3 Explain how it is possible to emit an electron from the


Radiation

Radioisotope

Nucleon

nucleus when the electron is not a nucleon.

4 Assess the statement that Positrons are radioactive


particles produced when a proton decays.

5 Discuss the impact that the production and use of


Neutron

Proton

Isotope

Alpha decay

radioisotopes has on society.

6 Describe how isotopes such as Tc-99m and F-18 can


be used to target specific organs to be imaged.

7 Use the data in Table 19.6.1 to answer the questions:


Beta decay

Gamma
decay

Antimatter

PET

Half-life

Bone scan

Positron
decay

Scintillator

a Which radioactive isotope would most likely be


used in a bone scan? Justify your choice.
b Propose two reasons why cesium-137 would not
be a suitable isotope to use in medical imaging.

Table 19.6.1 Properties of some radioisotopes


Radioactive source

Radiation emitted

Half-life

C-11
Tc-99m
TI-201
I-131
Cs-137
U-238

b+, g
g
g
b, g
a
a

20.30 minutes
6.02 hours
3.05 days
8.04 days
30.17 years
4.47 109 years

351

19

Imaging with
gamma rays

8 Figure 19.6.3 shows a bone scan and an X-ray image.

14 Describe the interaction between electrons and

Identify which is which and justify your choice.


a

positrons.

15 Explain how a PET scan is produced and compare


the image with that obtained from a CAT scan.

16 Identify the key characteristic of a PET image that


gives it a diagnostic advantage over a CAT image.

Solving problems
17


Figure 19.6.3

Two different images showing the chest region


of the human body

9 Compare the processes used to produce a bone scan


image and an X-ray image.

10 Compare the radiation used to produce a bone scan


image and an X-ray image.

11 Compare the resolution of a bone scan with that of


an X-ray.

146
234Th

144

dependent on the design of the lead collimator used


in the gamma camera.

13 Explain why tumours are visible in the patient in


Figure 19.6.4b.

234U

230Th

140

226Ra

138
222Rn

136

218Po

134

214Pb

130

210TI

128

218At
214Bi
214Po

210Pb

126

210Bi

210Po

206Pb

124
122
78

238U
234Pa

142

132

12 Explain why the resolution achieved in a bone scan is

Complete the decay series of U-238 shown in Figure


19.6.5 by including the decay productan , or
particlein each step, as shown for the -emission
seen in the first step.

80

82

Figure 19.6.5

84

86

88

90

94 Z

92

The decay series of U-238

18 Figure 19.6.6 shows the percentage of a radioactive


isotope that remains as a function of time.
a Estimate the half-life of this isotope.
b Would the isotope represented by this decay
curve be suitable for medical imaging?
Justify your answer.

50.0%

25.0%
12.5%

Re

iew

352

Q uesti o

A bone scan of (a) a healthy patient and


(b) a patient with tumours

6.3%
0

Figure 19.6.4

Figure 19.6.6

8
Years

10

Decay curve of a radioisotope

12

3.1%
14

16

medical
physics

PHYSICS FOCUS

4. Implications of physics for society


and the environment

ANSTO Radiopharmaceutical
production
Lucas Heights production facility
The Lucas Heights Facility in Sydney, operated by
ANSTO, manufactures radiopharmaceuticals such as
molybdenum-99 generators for technetium-99m and
iodine-131, together with other associated products
including chromium-51, thallium and gallium. The
facilities are ideal for simultaneous production of large
quantities of different isotopes used for diagnosis and
therapy. An average of 3500 medical isotopes and
2500 packages are dispatched per month. The
packaging, transport and delivery of all products
comply with strict national and international
regulations.

Molybdenumplant
ANSTOHealth is in the process of commissioning
a new plant that will be used to manufacture
molybdenum-99. ANSTO is currently importing this
product from overseas to meet domestic needs;
however, this is costly (as a heavy airfreight product)
and shipment is not always reliable, which emphasises
the importance of local Australian production.
Molybdenum-99 is used as a raw material for 80%
of nuclear medicine procedures performed around
the world.
Molybdenum-99 is formed by the fission of
uranium-235 that is itself formed by irradiation of low
enriched uranium (LEU) in ANSTOs nuclear reactor.
The molybdenum-99 is then used as a generator for
technetium-99m, which can then be formulated into
a plethora of radiopharaceuticals.
The new plant is located behind shielding because
of radiation, and has been carefully designed to
protect workers and the external environment during
and after processing.

5. Current issues, research and


developments in physics

National Medical Cyclotron Facility


(NMC)
The NMC was established in 1991 and produces
specialised isotopes for SPECT (single photon
emission computed tomography)imaging. The facility
houses a 30 MeV cyclotron, the largest in Australia.
Located at Sydneys Royal Prince Alfred Hospital,
Camperdown, the cyclotron accelerates protons to the
required energy and then fires them atdifferent
targets to generate the radioisotope required. The
radioactive isotope is then recovered and purified for
dispensing into the finished dosage at Lucas Heights.
The products produced at the cyclotron facility are
gallium-67, thallium-201, iodine-123 and iodine-123.
1 Discuss why the correct packaging and transport
of radiopharmaceuticals is important.
2 Discuss why it is important to have facilities to
produce radiopharmaceuticals in Australia.
3 Explain why molybdenum-99 is an especially
important radioisotope.

Extension
4 The OPAL nuclear reactor opened in 2007,
replacing the earlier HIFAR reactor. Some critics
of the nuclear reactor program claim a new reactor
was unnecessary.

Investigate this issue and discuss your conclusions


in terms of:
a the need for radiopharmaceuticals
b the safety of a nuclear reactor.

353

20
spin, magnetic moment, parallel,
antiparallel, precession, Larmor
frequency, radio frequency (RF),
magnetic resonance imaging (MRI),
resonance, relaxation, longitudinal
relaxation time constant, transverse
relaxation time constant,
RF transceiver coils, gradient coils,
functional MRI,

Imaging with
radio waves
Hydrogen calling
Magnetic resonance imaging (MRI) uses radio waves emitted by
hydrogen atoms to produce high-resolution images of the body.
Magnetic resonance has been used as a tool for studying atomic
structure since 1946, but it was not until 1973 that medical imaging
was suggested, and another 15 years before useful images were
obtained. Like CAT scans, MRI can produce tomographic images
(slices) of the body, enabling detailed two- and three-dimensional
images to be constructed from the data. Magnetic resonance imaging
was initially used to produce images showing structural detail;
however, recent developments have led to functional MRI,
which has greatly advanced knowledge of how the
brain and other organs work.

20.1 Spin and magnetism


Identify that protons and
neutrons in the nucleus have
properties of spin and describe
how net spin is obtained.

spin vector

protons

spin vector

354

opposite spins resulting


in zero net spin

Particles such as protons, neutrons and electrons have a property called


spin, which can be visualised as a rotation (spinning) of the particles. However,
spin is a quantum-mechanical property of the particles and the classical view of
a spinning ball is a useful model, but it is not reality. Protons are not simply tiny
spinning charged balls, but some of their properties are similar.
For protons, neutrons and electrons the spin property can only have
one of two possible directions: up or down (sometimes called spin up and spin
down). If the total number of protons and neutrons in a nucleus is even, then
their spins align in pairs in opposite directions so that the net spin of each pair
and the whole nucleus is zero (Figure 20.1.1). If there is an odd number of
nucleons, then the nucleus has a net spin, since there must be one unpaired
nucleon. Nuclei having a net spin include hydrogen, phosphorus-31, fluorine-19,
nitrogen-15 and carbon-13.

Figure 20.1.1

Using our model of spin, we can visualise two particles


of opposite spin resulting in zero net spin.

medical
physics
Spin produces a magnetic effect called a magnetic moment or
Identify that the nuclei of
magnetic dipole. You can think of this as being similar to the magnetic field
certain atoms and molecules
produced by moving charge (a current) around the wire of a solenoid (see Figure
behave as small magnets.
4.1.10). Therefore, charged particles such as electrons and protons behave like
tiny magnets. Even uncharged neutrons have a magnetic moment, because they
contain fundamental charged particles called quarks (see section 15.5). Nuclei
with an odd number of nucleons will therefore have a net spin and always have
an unpaired magnetic moment and so behave like tiny magnets. When placed in
a strong magnetic field these nuclei behave a little like a compass needle in the
A compass needle always points north, but the
Earths magnetic field.
Explain that the behaviour of
nuclei magnetic moments may assume one of two possible alignmentsthey can
nuclei with a net spin, particularly
align parallel or antiparallel to the magnetic field. External magnetic fields or
hydrogen, is related to the
magnetic field they produce.
the magnetic field of electromagnetic radiation may affect the nuclei because
of this magnetic moment.
north
Hydrogen is the most important element in the
process of magnetic resonance imaging because of
magnetic
the magnetic properties of the nucleus and the fact
moment
that it is present in water molecules, proteins, fats
and carbohydratesin fact, in most of the
molecules in the body. Since it normally has just
a single proton forming the nucleus, a hydrogen
nucleus will clearly have a magnetic moment. The
magnetic properties of hydrogen are not normally
evident, because the magnetic moments of the
south
nuclei are randomly aligned, resulting in a zero
net magnetic effect (Figure 20.1.2).
Figure 20.1.2 The net spin of hydrogen results in a magnetic moment that is often
not apparent in large groups of hydrogen nuclei.

Checkpoint 20.1
1
2
3
4

Outline what is meant by the spin of a particle.


Describe the situations in which an atomic nucleus has zero net spin.
Define the term magnetic moment.
Describe the two possible alignments of the magnetic moment of a proton.

20.2 Hydrogen in a magnetic field


When placed in a very strong magnetic field, the proton will tend to align with
the applied field, because of the interaction between the external field and the
magnetic properties of the proton. In a medical context, the strength of the
magnetic field used is typically between 0.5 and 5 T, although fields used in
research applications may be more than 10 T. The parallel and antiparallel
alignments to the field are not strictly parallel to the field. They are actually
The parallel configuration has a
angled as illustrated in Figure 20.2.2.
slightly lower energy than the antiparallel configuration; just how much lower
depends on the strength of the magnetic field. In a 1 T field, the difference is
just 0.18 eV (0.18 106 eV or 2.8 1026 J). As a lower energy state, it is

Describe the changes that


occur in the orientation of the
magnetic axis of nuclei before
and after the application of a
strong magnetic field.

355

20

Imaging with
radio waves

no external field

therefore slightly preferredbut only about 0.0003% more protons favour the
parallel configuration. Injecting the energy difference that corresponds to a photon
with a radio frequency of 42.6 MHz can cause the proton to flip from the parallel
to the antiparallel configuration. This same energy will be released if it flips back.
When the applied external field is removed, the protons become randomly
orientated again because of collisions occurring between the randomly moving
atoms.

Precession
external magnetic field

parallel

antiparallel

Figure 20.2.1

Randomly orientated
hydrogen nuclei align to
an externally applied
magnetic field.

Define precessing and relate


the frequency of the precessing,
(i.e. Larmor frequency), to the
composition of the nuclei and
the strength of the applied
external magnetic field.

When you have a magnetic moment angled to the direction of a magnetic


field, the field will exert a force and hence a twist (a torque) on the magnetic
moment. This will cause the direction of the magnetic moment to rotate about the
magnetic field direction. This is called precession. It is similar to the precession
of a toy spinning top caused by the force of gravity. In our model of a spinning
proton, the direction of the magnetic moment corresponds to the imagined spin
axis of the proton, which is analogous to the spin axis of the top (Figure 20.2.2).
The frequency of precession of
external magnetic field
a magnetic moment such as a proton is
called the Larmor frequency. It depends
on the magnetic properties moment of
the proton (p) and is proportional to
precession
the magnitude of the external field (B). It
is given by:
2 p B
f Larmor =

h
where h is Plancks constant.
For a proton in a 1 T magnetic field,
the Larmor frequency is 42.6 MHz,
corresponding to radio frequency (RF)
electromagnetic waves.

Try this!

Figure 20.2.2

Precession of a hydrogen
nuclei around the applied
magnetic field

precession

A spinning top
When you try to push over a childs spinning top, it doesnt fall
over. Try it! Instead, the tops axis of rotation itself starts to
rotate around the vertical axisit precesses (at least until
frictional forces cause it to tip too far and it hits the ground).
The force of gravity produces a twisting effect (a torque) on
the top. If the top werent spinning, this would cause it to
simply fall over. Precession is the motion that results when the
top is spinning. The axis of rotation of the top sweeps out
a conical motion.

356

rotation

force of gravity

Figure 20.2.3

Precession of a spinning top

medical
physics

Checkpoint 20.2
1
2
3

Explain what happens to a proton in an external magnetic field.


Using the photon energy given by E = hf, demonstrate that the frequency f required to cause a proton to flip its
alignment in a 1 T magnetic field is 42.6 MHz.
Explain how the frequency required to flip a proton will change if the applied magnetic field is doubled.

20.3 Tuning in to hydrogen


Excitation

PRACTICAL
EXPERIENCES
Activity 20.1

Activity Manual, Page


146

In magnetic resonance imaging (MRI) a person is placed in a strong


Discuss the effect of subjecting
magnetic field and a pulse of RF electromagnetic radiation is sent into their
precessing nuclei to pulses of
body. The frequency is tuned to match the Larmor frequency of protons within
radio waves.
the field.
The RF pulses have two effects on the protons in the persons body. First,
they cause a spin flip to occur. A few of the protons aligned parallel to the
external field absorb the RF energy because it matches the energy necessary to
flip from the parallel to the slightly higher energy antiparallel configuration.
Before the RF pulse there was a small imbalance in numbers, with slightly more
protons in the parallel configuration. The net magnetic
a
b
effect of the alignment of protons along the magnetic
field direction (usually called the longitudinal
S
S
magnetisation) was not zero. The RF pulse results in the
number of parallel and antiparallel protons becoming
more nearly equal, causing the net magnetic effect in this
Z
direction to become zero (Figure 20.3.1). This direction
X
is along the head-to-toe axis of the person in the MRI
Y
machine.
N
N
The second effect of the RF pulses is to cause the
protons to precess in step (in phase) with each other.
Figure 20.3.1 (a) Parallel (blue) and antiparallel (red) protons are aligned
This can be compared to soldiers all marching in step.
within a strong magnetic field. (b) RF radiation causes some
The protons precessing in phase with each other create
protons to jump to the antiparallel configuration.
a net magnetic effect, Bxy, in the plane perpendicular
a
b
to the field (Figure 20.3.2). This net magnetic effect
S
S
rotates at the Larmor frequency.
The RF wave is used to transmit energy into
the tissues to be imaged. The energy of the RF
electromagnetic waves is absorbed effectively by protons
Bxy
in hydrogen, because the frequency of the radiation and
the precession frequency of the protons are the same.
Said another way, the energy of the RF photons matches
N
N
the energy difference between the parallel and antiparallel
Figure 20.3.2 (a) The protons precess in phase in response to the RF pulse.
configurations. This energy exchange in which properties
(b) The net magnetic field is in the xy plane.
are matched is called resonance.

357

20

Imaging with
radio waves

Resonance

1st and 2nd strings

esonance occurs between two oscillating objects if they


have the same natural frequency and if there is some
means by which energy can be transferred from one object to
the other. The everyday experience of pushing a child on a
swing demonstrates the principle. To make the child swing
higher, you must push at the same frequency as the child is
swingingpush every time they get to the top of the back
swing. If you push at the wrong time, you will just interfere
with the swing (and hurt yourself!).
Resonance can also be observed in a guitar when pressing
down on the fifth fret of the first string and plucking the
second string. If the guitar is tuned correctly, the first string
will also vibrate. This occurs because under these conditions
the two strings have the same natural frequency and the energy
can be transferred from one string to the other via the body of
the guitar.

nut
1st fret

5th fret

Figure 20.3.3

Resonance can be observed in a guitar.

Longitudinal magnetisation

Relaxation
100%

Time

T1

Figure 20.3.4

The relationship between the longitudinal


relaxation time constant T1 and the
increasing longitudinal magnetisation

Table 20.3.1 Relaxation times for various tissues in a 1 T field


Tissue
Brain grey matter
Brain white matter
Cerebrospinal fluid
Fat
Muscle
Blood
Water

358

T1 (ms)
520
390
2000
180
600
800
2500

T2 (ms)
100
90
300
90
40
180
2500

After the RF pulse, the protons return to their original state


and, as they do so, they re-emit the energy they absorbed from the
radio wave. They do this over a period varying from about 0.01 to
0.1 s. The emitted energy is again an RF wave that is detected
using an antenna. The signals emitted by the protons are used to
create the MR image. The return of the proton to the less excited
state after absorbing the RF energy is called relaxation.
The emission of the energy from the hydrogen nuclei returning
to the lower energy state is described by two time constants, T1
and T2, associated with two different electromagnetic processes
(Table 20.3.1).
The longitudinal relaxation time constant T1 is a
measure of the time taken for protons to return to their normal
ratio of parallel and antiparallel configurations relative to the field.
In this process, the longitudinal magnetisation returns to its
non-zero strength. The signal that is detected as a result of this
process increases in strength from zero as the number of protons
aligned parallel to the external field approaches its equilibrium state
(Figure 20.3.4).
The time constant T1 is defined as the time taken for the signal
from the protons aligned in this state to reach a particular value
(about 63% of the final equilibrium value).
T1 is dependent on the interaction between the precessing
protons and the other atoms in the material. It is sometimes called
the spinlattice interaction, because the process was first observed
in crystals, which have a lattice structure.

medical
physics

100%

Transverse magnetisation

The transverse relaxation time constant T2 is


a measure of the time taken for the magnetic component
perpendicular (transverse) to the external field to return to zero.
This happens as the precessing protons become out of phase
with each other (in the analogy, this is like the soldiers getting
out of step). As a result of the loss of phase between the
precessing protons, the transverse magnetisation, Bxy, decreases
exponentially to zero (see Figure 20.3.5).
The time constant T2 is dependent on the interaction
between the precessing protons and is sometimes referred to
as the spinspin relaxation time.

Figure 20.3.5

Checkpoint 20.3
1
2
3
4

Outline the sequence of events during an MRI scan.


Explain the two consequences of applying correctly tuned RF
pulses to the hydrogen in the body.
Outline the general concept of resonance.
Define the relaxation times T1 and T2.

Bxy

T2

Bxy

Time

The relationship between the transverse


relaxation time constant T2 and the decreasing
transverse magnetisation. The arrows in the
circles represent the components of the
magnetic properties of the protons becoming
progressively more random, causing the
transverse magnetisation, and the RF signal
associated with it, to decrease to zero.

Explain that the amplitude of


the signal given out when
precessing nuclei relax is
related to the number of
nuclei present.

20.4 It depends on how


and where you look
Each type of tissue in the body contains different
amounts water and therefore different amounts of
hydrogen. Hydrogen is also present in carbohydrates,
proteins and fats, and the amounts of these also vary
The amplitude of the radio
in different tissues.
signal emitted as the relaxation processes occur
increases as the number of protons in the tissue
increases. An MRI can be created which shows these
differences in proton density and thereby highlights
the difference between different tissues.
Each tissue has unique values for T1, and T2.
Magnetic resonance images can also be obtained to
show specific tissues more clearly using the
differences in the values of T1 and T2 that are
characteristic of each tissue type. If the contrast in an
image results mainly from differences in the T1
values, the images are called T1 weighted. Images
dominated by T2 differences are called T2
weighted. Figure 20.4.1 illustrates the difference in
contrast between T1 and T2 weighting.

Bxy

Figure 20.4.1

Contrast differences in (a) T1 weighted and (b) T2 weighted


images centred on the knee of a 9-year-old boy. The arrows
indicate the lesion responsible for his knee pain.

359

20

Imaging with
radio waves

Explain that large differences


would occur in the relaxation
time between tissue containing
hydrogen-bound water
molecules and tissues
containing other molecules.

Table 20.4.1 Water content of various tissues


Tissue

Water content (%)

Brain grey matter


Brain white matter
Heart
Blood
Bone

70.6
84.3
80
93
12.2

The difference in relaxation times T1 and T2 is significantly greater for tissues


As a result, MRI is very
containing relatively large amounts of water.
sensitive to variations in the water content of tissues (see Table 20.4.1) and this is
a significant factor in MRIs ability to produce high-resolution, high-contrast
images (see Figure 20.4.2).
Because tumours are characterised by rapid cell division and high growth
rates, they typically have a higher percentage of water than similar non-cancerous
tissue, and can thus be clearly imaged using MRI. Haemoglobin molecules in red
blood cells also provide a clearly identifiable resonance signal and so MRI can be
used to compare the blood content of different tissues. This is often greater in
cancerous tissue, again because of the high growth rates.
MRI contrast agents are unique in radiology because it is not the
chemical of the contrast agent that is detected, but rather the effect that the
chemical has on the protons in nearby hydrogen atoms.
Gadolinium (bonded to a carrier molecule) is a contrast agent that, when
injected into the bloodstream, shortens the relaxation time T1 of protons near it,
making the tissues appear brighter in a T1-weighted image. Because gadolinium
normally stays in blood vessels, it has the effect of making these blood vessels and
and areas in the body where blood leakage is occurring appear brighter.
Another powerful tool used to enhance image contrast is to vary the time
intervals between successive repetitions of a sequence of applied RF signals. This
changes the weighting of the relaxation times and can improve the contrast in
the image between different tissues.

Checkpoint 20.4
1
2
3
4

Figure 20.4.2

MRI scans of the brain show


more contrast and detail than
conventional X-ray or CAT
scans because MRI is more
sensitive to the differences in
water content of grey matter
and white matter.

PRACTICAL
EXPERIENCES
Activity 20.2

Activity Manual, Page


152

360

Account for the different MRI signal amplitudes observed from


different tissues in the body.
Explain how different parts of the body can be highlighted in an
MRI image.
Outline why tumours are so clear on scans.
Explain why gadolinium is injected during an MRI scan.

20.5 The MRI scanner


There are approximately 200 MRI scanners in Australia. All major hospitals and
many private imaging facilities in Australia have one, although they cost more
than $2 million to purchase and about $1 million per year to operate and staff.
Part of this cost is for liquid helium to cool the superconducting electromagnets.
These produce the strong magnetic fieldsup to 5 Trequired for MRI.
Compare this with the field of about 50 106 T at the Earths surface.
The patient is placed in the MRI scanner tunnel and the current in the
superconducting coils is turned on to produce the strong magnetic field that will
align the proton spins. These coils are shown in Figure 20.5.1, but not in Figure
20.5.2. When the patient lies in the MRI scanner, the strong magnetic field is

medical
physics
RF
transceiver

Y coil

Z coil
patient
patient
table

gradient
coils

X coil

superconducting
magnet coils
scanner

RF transceiver
X

Figure 20.5.1

Cutaway view of an MRI scanner

Figure 20.5.2

parallel to a line running in the head-to-toe direction, which is normally referred


to as the longitudinal axis or the z direction.
The pulse of radio waves from the RF transceiver coils is transmitted
through the patients body. This flips the magnetic axes of some of the protons,
resulting in a zero magnetic field from the protons in the z direction. It also
causes the protons to precess in phase with each other. As some of the protons
flip back and return to precession that is out of phase, the protons in each
volume element (voxel) emit radio waves with an intensity proportional to the
number of hydrogen atoms in the voxel.
Gradient coils produce a small systematic variation in the net
magnetic field throughout the patients body. The Z coils (Figure 20.5.2)
produce variation in the z direction, which defines a slice through the body,
represented by the blue region in Figure 20.5.3. The X and Y coils change the
net magnetic field in their respective directions so that its strength is different in
each voxel. As a result of these variations in the imposed field, the protons
precess at slightly different frequencies. These frequencies allow the positions of
the protons emitting RF signals to be determined.
Hundreds of pulse cycles are used to obtain proton density data from each
voxel in the plane of interest.
The RF signals emitted by the protons provide data that is processed by a
computer to produce an image for each slice through the body, and hence to
create a 3D image that can be manipulated and viewed from any angle.

Checkpoint 20.5
Explain why the cost of running MRI scanners is so high.

patient

Gradient in an MRI scanner

Figure 20.5.3

Image slices and voxels


in an MRI scan

PRACTICAL
EXPERIENCES
Activity 20.3

Activity Manual, Page


155

361

20

Imaging with
radio waves

20.6 Applications of MRI

Figure 20.6.1

One of the first highly successful applications of MRI was its use in diagnosing
multiple sclerosis, an incurable disease that affects the nervous system. MRI clearly
shows the scar-like plaque resulting from this disease, marking the destruction of
the insulating myelin sheaths around the axons of nerve cells (Figure 20.6.1),
something other imaging technologies are unable to do. Other examples of MRI
imaging are shown in Figures 20.6.2 and 20.6.3.
MRI has changed scientific thinking about the progress of diseases such as
cancer and multiple sclerosis, because these diseases can now be detected earlier
and different treatments can then be used. MRI also allows the progress of a disease
and its treatment to be monitored more safely, increasing the chances of survival.
False-colour MRI of the brain
of a person with multiple
sclerosis shows regions
of myelin destruction
(blackorange).

Advantages of MRI
MRI is non-invasive and has no known side effects, although a few patients
experience anxiety in the confined, noisy space of the MRI scanner.
MRI provides excellent soft-tissue imaging, providing better contrast than CAT
or conventional X-rays and much better resolution than ultrasound.
Image contrast can be altered by changing the T1 and T2 weightings.
There are few cases of adverse reaction to MRI contrast agents.

Disadvantages of MRI

Figure 20.6.2

MRI shows blood flow in the


arteries of the brain as
white, relative to stationary
brain tissue.

Patients with heart pacemakers or metal parts in their bodies cannot be imaged
because of the strong magnetic fields.
Scans may take up to an hour to complete.
MRI scanners are very expensive and the running costs are high.
MRI scanners are very sensitive and must be screened from outside radio and
magnetic interference.
The strong magnetic field produced can interfere with nearby electronic
equipment and is dangerous in the presence of iron tools or furniture.
Most current MRI images show structure within the body. An important
area of technical development is the production of functional MRI images that
reveal differences in chemical processes happening in the body. Other
developments include the use of even stronger magnetic fields, allowing better
resolution and contrast, and ultimately imaging using elements other
than hydrogen.
The possibility of future high temperature superconductors, requiring only
liquid nitrogen cooling, could impact significantly on MRI technology by
removing the need for expensive, non-renewable liquid helium. Research and
development has already produced portable MRI scanners for specialised
applications. These machines do not require the use of superconducting
magnets, greatly reducing the size and cost of the machine.

Checkpoint 20.6
1
Figure 20.6.3

362

MRI shows cancer


(colour coded orange)
that has spread from the
lungs to the brain.

Describe what is seen in an MRI scan in a patient that has


multiple sclerosis.
Create a table to contrast the advantages and disadvantages of
MRI and CT scans.

PRACTICAL EXPERIENCES

medical
physics

CHAPTER 20

This is a starting point to get you thinking about the mandatory practical
experiences outlined in the syllabus. For detailed instructions and advice, use
in2 Physics @ HSC Activity Manual.

Activity 20.1: magnetic resonance images


Look at the MRI scans and list their characteristics. Use images of healthy and
damaged tissue and identify what characteristics allow you to distinguish between
the two.
Discussion questions
1 List characteristics of an MRI scan.
2 Outline how damaged tissue can be identified on an MRI scan.
3 Give reasons why MRIs are used to identify areas of high blood flow.
4 Construct a table that lists the parts of an MRI, identifying their
functions.

Activity 20.2: A comparison of scanning techniques


Research each of the diagnostic methods and compare the advantages and
disadvantages of each technique. Prepare a summary of your findings.
Discussion questions
1 Identify the most appropriate application for each of the methods listed.
2 List advantages for each of the diagnostic methods listed.
3 Identify limiting factors for each method.

Activity 20.3: Medicine and physics


Research how medical applications of physics have affected society and write
a report to assess these impacts.
Discussion questions
1 Recall some medical applications of physics.
2 Identify the areas of society these applications have affected.

Perform an investigation to
observe images from magnetic
resonance image (MRI) scans,
including a comparison of
healthy and damaged tissue.
Identify data sources, gather,
process and present information
using available evidence to
explain why MRI scans can be
used to:
detect cancerous tissues
identify areas of high blood
flow
distinguish between grey and
white matter in the brain.
Gather and process secondary
information to identify the
function of the electromagnet,
radio frequency oscillator, radio
receiver and computer in the
MRI equipment.

Identify data sources, gather and


process information to compare
the advantages and
disadvantages of X-rays, CAT
scans, PET scans and MRI scans.

Gather, analyse information


and use available evidence to
assess the impact of medical
applications of physics on
society.

363

20

Imaging with
radio waves

Chapter summary

Particles have a property called spin, which can be


visualised as a rotation (spinning) of the particles,
although this is not reality.
Protons, neutrons and electrons can only have two spin
states: spin up and spin down.
If there is an odd number of nucleons in a nucleus, then
the nucleus has a net spin.
Spin produces a magnetic effect called a magnetic
moment or magnetic dipole.
Nuclear magnetic moments mayalign parallel or
antiparallel to a magnetic field. The parallel
configuration has a slightly lower energy and is therefore
slightly preferred.
Injecting energy equal to the energy difference between
the spin states can cause the proton to flip from the
parallel to the antiparallel configuration. This same
energy will be released if the proton flips back.
A magnetic moment angled to a magnetic field direction
experiences a force and hence a twist (a torque) that causes
precession of the direction of the magnetic moment.
The frequency of precession of a magnetic moment is
called the Larmor frequency given by:
2 p B

f Larmor =
h

Magnetic resonance imaging (MRI) involves sending a


pulse of radio frequency (RF) radiation into a persons
body while located in a strong magnetic field. The
frequency is tuned to match the Larmor frequency of
protons within the field.
The return of the proton to the less excited state after
absorbing the RF energy is called relaxation.
The longitudinal relaxation time constant (T1) is a
measure of the time taken for protons to return to their
normal ratio of parallel to antiparallel configurations
relative to the field.
The transverse relaxation time constant (T2) is a
measure of the time taken for the magnetic component
perpendicular to the external field to return to zero.
MRI is very sensitive to variations in the water content
of tissues.
MRI contrast agents affect the protons in nearby
hydrogen atoms.
Gradient coils produce small variations in the net
magnetic field throughout the patients body.
Functional MRI images reveal differences in chemical
processes happening in the body.

where B = magnetic field and h = Plancks constant.

Review questions
Physically Speaking
Complete the passage below by filling in the missing words
from the list provided.
antiparallel, field direction, energy, flip, higher energy,
intensity, relaxation time constants, magnetic resonance
imaging, quantum mechanical, parallel, protons, model,
radio, randomly, reality, rotation, magnetic field, spin

MRI stands for _____________. This diagnostic tool uses a


property of atoms called _____________ which can be
visualised as a _____________ of the particles. However, spin is
a _____________ property of the particles and the classical
view of a spinning ball is a useful _____________, but is not
_____________. Single _____________ (hydrogen nuclei)

generally all have their spins oriented _____________. When a


_____________ is applied, they all line up _____________ or
_____________ to the _____________. _____________ waves

transmitted through the patients body can cause protons to


momentarily _____________ to the _____________ antiparallel
state. As they return to their original orientation, they re-emit
the _____________. The _____________ of the emission tells us
how many protons there are in that area of the body. The
_____________ tell us about the environment of the protons in

that area of the body.

364

medical
physics

Reviewing
1 Explain why fluorine-19 has a nuclear magnetic
moment, but nitrogen14 does not.

2 Identify the property of tissues that permits images


like those in Figure 20.7.1 to be produced using an
MRI machine.

3 Describe the effect on the energy of protons in an MRI


scanner that allows an image to be produced.

4 Outline why cancerous tissues usually have a greater


amount of blood flowing to them than normal tissue
of the same type.

5 Describe the behaviour of a proton in a strong


magnetic field.

6 Describe the process of precession in a mechanical


system such as a childs toy top.

7 Describe the process of precession involving protons


in a strong magnetic field.

8 Summarise the conditions required for radio waves to


cause protons precessing in a strong magnetic field to
undergo a spin flip.

9 Justify the need for superconducting electromagnets


in an MRI scanner.

10 Describe the function of the radio frequency


transceiver coils in an MRI scanner.

MRI machine.

13 Describe what is meant by the term relaxation time.


14 Compare the advantages and disadvantages of MRI
and CAT scans.

15 Briefly assess the impact of MRI scanners on


medicine.

16 Assess the impact of superconductivity on the


development of MRI.

17 Identify possible future directions of physics research


that would contribute to improvements in MRI.

Solving problems
18 When a 1 T field is applied to a portion of the body
containing 2 million protons, estimate how many
extra protons favour the parallel configuration over
the antiparallel configuration.

19 Calculate the magnetic moment of a proton, given


that the Larmor frequency for a proton in a 1T
magnetic field is 42.6MHz.

20 Calculate the Larmor frequency of an electron, given


that the magnetic moment for an electron is
9284.764 1027 JT1.

iew

Q uesti o

MRI images show a normal brain (left) and the brain of a patient with a tumor.

Re

Figure 20.7.1

11 Explain the role of the computer in an MRI machine.


12 Describe the purpose of the gradient coils in an

365

The review contains questions that address the key concepts


developed in this module and will assist you to prepare for the
HSC Physics examination. Please note that the questions on the
HSC examination that address the option modules are different in
structure and format from those for the core modules. Past exam
papers can be found on the Board of Studies NSW website.

Multiple choice

(1 mark each)
1 Two differences between the normal hearing range
of sound and ultrasound are:
A
B
C
D

Using the data in the table below, determine which


answer is closest to the percentage of ultrasound
intensity that is reflected from a boundary between
kidney and fat.

Substance

Normal sound

Ultrasound

Not easily scattered by


human tissue, 20 Hz
to 20kHz
Shorter wavelength,
audible
20 Hz to 20kHz,
longer wavelength
Above 20kHz, audible

Easily scattered by
human tissue, above
20kHz
Longer wavelength,
inaudible
Above 20kHz, shorter
wavelength
20 Hz to 20kHz,
inaudible

An advantage of a CAT scan over an X-ray as


a diagnostic tool would be:
A seeing broken ribs more clearly.
B the images contain three-dimensional information
that allows for greater discrimination of structure.
C the ability to carry out a biopsy.
D lower cost.

Figure 20.8.1 shows two images of the same


patients brain, using two different medical imaging
techniques. The two techniques were:
A Left: Ultrasound; Right: MRI
B Left: Ultrasound; Right: CAT
C Left: CAT; Right: MRI
D Left: MRI; Right: CAT

Fat
Kidney

A 0.29%
B 1%

Ultrasound
velocity (m s1)

Impedance
(rayl)

920

1450

1.33 106

1040

1561

1.62 106

C 12%
D 18%

In medical imaging, the term relaxation time


refers to:
A how long it takes for an MRI machine to
recharge.
B the time for proton spin to return to equilibrium.
C the minimum time between scans.
D the maximum time before a radioisotope decays
to untraceable amounts.

Figure 20.8.1

366

Density
(kg m3)

medical
physics

Short response

Extended response

The ultrasound image in Figure 20.8.2 shows an


organ in a child that is not usually imaged in adults.
Identify the organ and describe the features of the
image that make it recognisable as an ultrasound
image. (4 marks)

10

Explain how an endoscope can be used to obtain


tissue samples from the bladder of a patient.
(2 marks)

For each of the radioactive isotopes in the table


below explain why it would or would not be a good
choice for use in a bone scan. (5 marks)

Radioactive
source

Radiation
emitted

Half-life

C-11

b+, g

20.30 minutes

Tc-99m

6.02 hours

I-131

b, g

8.04 days

Cs-137

30.17 years

U-238

4.47 109 years

Assess the impact of MRI on diagnosis of disease,


in comparison to other medical imaging techniques.
(5 marks)

The nucleus of 1H has magnetic properties that


enable it to be used for MRI.
4
a Explain why 2He is not used. (2 marks)
b Identify the primary factor that determines the
1
amplitude of the signal produced by 1H nuclei
after an RF pulse at the resonant frequency has
been applied to a part of a persons body.
(2 marks )

Figure 20.8.2

367

6
CONTEXT

Astrophysics

I render infinite thanks to God, for being so kind as to make me alone the first
observer of marvels kept hidden in obscurity for all previous centuries.
Galileo Galilei in Siderius Nuncius (Sidereal Messenger)

Figure 21.0.1 The 2-degree field top end


of the Anglo-Australian
telescope (AAT) uses optical
fibres to direct light from up
to 400 astronomical targets
to spectrographs.

368

The Italian scientist Galileo Galilei (15641642) was not quite right in his claim to
be the first to see the universe revealed by the newly invented telescope. That
accolade may belong to Englishman Thomas Harriot (15601621). However, it was
certainly Galileos short record of his observations in Siderius Nuncius (Sidereal
Messenger), published in March 1610, that announced a revolution in astronomy to
the educated world in Europe. Observations with telescopes revealed previously
unknown objects, unimagined by thousands of years of naked-eye observers.
Nonetheless, astronomy remained largely a collectors science for another two
centuries. Observers used telescopes of increasing size to discover objects and note
their positions and details, but knew little of their true nature. It was not until the
early years of the 19th century that physicists discovered lines in the spectrum of
sunlight, and later realised that they were the fingerprints of the elements making
up the Sun and the stars. Although Newtons law of universal gravitation explained
motion in the sky, it was the understanding of spectral lines that really put physics
into astronomy. Spectroscopy remains at the heart of modern astrophysics.
In this module we will explore some aspects of both gravity and spectroscopy in
astrophysics, leading to our current understanding of the life history of stars.

Figure 21.0.2 The stars of the Southern Cross

(right) and the pointers (alpha)


and (beta) Centauri (left)

INQUIRY ACTIVITY
Learn about a star
A lot of astronomical data gathered by astronomers is now available on-line in
various catalogues. One important catalogue of stars was produced by the
Hipparcos satellite in the early 1990s. It observed the positions and basic
observable properties of more than 100000 stars with high precision and more
than 2.5 million stars to lower precision.
In this exercise, you will use the Hipparcos catalogue to find out about one
of the bright stars of the Southern Cross (the constellation Crux) or a neighbour
in the constellation of Centaurus.
1 Find a map of Crux and Centaurus,
perhaps in a star atlas or a computer
program that shows the stars (e.g. Google
Sky or WikiSky). Youll need to orient the
map to match Figure 21.0.2 above.
2 Identify the brightest stars of Crux and
Centaurus seen in Figure 21.0.2.
3 Select one of these stars and look it up
in the Hipparcos online catalogue, which
can be accessed on the companion
website at Pearson Places
<www.pearsonplaces.com.au>.
To do this, enter the name
(e.g. Alpha Crucis) into the Target Name
field and then click on Submit Query.
The database should respond with a single
line of data about your star.

4 Can you interpret the data? Use


the cursor to select each column
heading to understand what each
value means.
5 Record the following important
values (and the units) from the
catalogue for each star:
Trigonometric parallax and the
standard error in this value
V magnitude
Colour index B V
At the end of this Module, you should
have a better understanding of many
of these data values.

369

21

Eyes on
the sky
Who needs a telescope?

telescope, magnification, refracting


telescope, field of view, focal length,
reflecting telescope, radio telescope,
sensitivity, angular resolution,
diffraction, Airy disc, active optics,
seeing, scintillation, adaptive optics,
interferometer, interferometry

Looking at the sky with the naked eye involves an optical system with
a maximum diameter of about 7mmthe size of the pupil of your
dark-adapted eye. The brightness of the image and the ability to
resolve fine detail are both set by that size. Using a telescope
increases the diameter of the light-collecting aperture to perhaps
60mm in a small backyard telescope, or 10m in the largest optical
telescopes currently in use. The aperture is the most important
property of any telescope operating at any wavelength because it
governs the ability to see faint objects and resolve fine detail.

21.1 The first telescopes


The magnifying ability of lenses has been known since ancient times, but it
seems that the combination of two lenses to form a practical optical telescope
was first achieved by spectacle makers in the Netherlands in 1608. By mid-1609,
Galileo Galilei (15641642), at the University of Padua, had built his first
telescopes with magnification of 3. He quickly progressed to instruments with
increasing magnification and, for just a few months, his superior instruments
gave him an unchallenged ability to observe the sky (Figure 21.1.1). This turning
point in the history of science prompted the United Nations to declare 2009,
the 400th anniversary of Galileos initial observations, as the International Year
of Astronomy.

Figure 21.1.1 Two of Galileos original telescopes,


mounted for display. The longer telescope
had an aperture of 26mm, a magnification
of 14 and was about 1m long.
370

astrophysics

Galileos influence was immense because, in March 1610, he quickly


published a short record of his initial observations in Sidereus Nuncius (Sidereal
Messenger). Among his major discoveries was the presence of mountains and
valleys creating shadows on the Moons surface, which changed as the lighting of
the surface changed. This rugged nature of the surface of the Moon contradicted
Aristotles concept of heavenly perfection (Figure 21.1.2).

What is
magnification?

he magnification of an optical
telescope describes how much
bigger an object appears in the
telescope compared to the nakedeye view. Technically, it is the
ratio of the angular size of the
object with and without the
telescope. A pair of 7 50
binoculars, for example, has an
angular magnification of 7 times
(7) and 50mm diameter main
lenses. The view through the
binoculars is as if the object was
only 1/7th as far away.

Figure 21.1.2 One of Galileos drawings of the Moon compared with a photograph

He also saw four stars moving back and forth relative to Jupiter in a pattern
he recognised as orbital motion around Jupiter. The observation of these
Galilean satellites almost led to the discovery of the planet Neptune, more than
two centuries before its actually discovery.
Galileos instruments were refracting telescopes that used a planoconvex lens (Figure 21.1.3) as the objective lens. The properties of the glass
and the shape of this lens bend the light and bring it to a focus (Figure 21.1.4).
The smaller eyepiece lens then relays the light to your eye, which creates a new
image on the retina at the back of the eyeand you see the object!
Galileos telescopes used a plano-concave lens (Figure 21.1.3) as the eyepiece.
This results in an image that is upright; but the telescope then has a very narrow
field of viewthe area of sky you can see at any momenta little like looking
down a narrow pipe at the object.
In 1611, Johannes Kepler (15711630) pointed out that a plano-convex
eyepiece lens would also work (Figure 21.1.4), although the image was inverted.
Christoph Scheiner (15731650) popularised this form of Keplerian refractor
by pointing out that the field of view was larger than in the Galilean telescope
(as illustrated in Figure 21.1.5).
With the field-of-view problem reduced in the Keplerian (or astronomical)
refractor, a race began for higher magnification. This was most easily achieved by
increasing the focal length of the objective lensthe distance between the lens
and the image it forms (see section 21.3 for more on calculating magnification).
By the 1670s, long telescopes achieved magnifications of more than 100, but at
the cost of enormous size and resulting difficulty in mounting and operation.

Discuss Galileos use of the


telescope to identify features
of the Moon.

plano-convex

plano-concave

Figure 21.1.3 Early telescopes used simple lenses


that were flat on one side and
either concave or convex on the
other. Modern single lenses are
often curved on both sides.
371

21

Eyes on the sky

Keplerian telescope
objective lens

focus
to eye

starlight
eyepiece

focal length
b

Galilean telescope
objective lens
to eye

starlight

eyepiece

focal length

Figure 21.1.4 (a) Keplerian and (b) Galilean telescopes using an objective lens

of the same focal length for easy comparison


b

Figure 21.1.5 These images, shown through


(a) Keplerian and (b) Galilean
telescopes at the same
magnification, have very
different fields-of-view.

Figure 21.1.6 A replica of Newtons original


reflecting telescope

Another advantage of a long focal length objective lens was to minimise


distortions inherent in the lens design. One of these was chromatic (colour)
aberration caused by the fact that different colours of light come to a focus at
slightly different distances from the lens. This causes a coloured halo around a
white star image. It was Isaac Newton (16421727) who realised that this was
caused by the properties of the glass itself. Combining different types of glass
offered the possibility of partially cancelling the colour effect of one glass with
the effect of the other. It was not until 1733 that such an achromatic objective
lens was first made. Today, any good-quality small telescope or set of binoculars
has an achromatic objective lens and an eyepiece also composed of several lenses.
Despite these developments, and the construction in the late 19th century of
some large refractors with apertures up to 1m, the dominance of the refracting
telescope in astronomy is long past. Again it was Newton who made the
breakthrough by constructing a reflecting telescope in 1668, using a mirror
rather than an objective lens to collect and focus the light (Figure 21.1.6). It
offered better images, without a coloured halo, in a telescope that was much
smaller than any comparable refractor.
Newtons telescope used a polished metal primary mirror to focus the
light, and a smaller, flat secondary mirror to divert the light out the side of the
telescope tube to the eyepiece. In 1672, a variation of the Newtonian telescope
was developed by Laurent Cassegrain (~16291693), who used a small
hyperbolic secondary mirror to reflect light back through a hole in the primary
mirror (Figure 21.1.7). By the mid-1700s reflecting telescopes with up to 15cm

Newtonian telescope
to eye
flat secondary
mirror

Cassegrain telescope
hyperbolic
secondary mirror

eyepiece

prime
focus

parabolic
primary
mirror

starlight

eyepiece

prime
focus

to
eye

starlight

focal length

focal length

Figure 21.1.7 Newtonian and Cassegrain telescopes using a primary mirror of the same focal length for easy comparison.
In large telescopes, if no secondary mirror is in place, the prime focus position can be used.
372

parabolic
primary
mirror

astrophysics
apertures were available, and in 1789 William Herschel (17381822) built a
reflector with a 124cm aperture! This was the first giant reflectorthe
forerunner of the 8 and 10m aperture reflecting telescopes of today.

Checkpoint 21.1
1
2
3

Identify what sets the light-gathering ability and resolving power of an optical system.
Outline key astronomical observations made by Galileo.
Describe chromatic aberration and its cause.

21.2 Looking up
For more than 200 years from the time of Galileo, all telescopes were optical
telescopes designed to focus visible light onto the retina of the human eye.
From about 1840, the use of photography revolutionised astronomy by allowing
images of the sky to be permanently recorded. For the next century telescopes
grew in size but remained fundamentally unchanged.
Discuss why some wavebands
In 1933, Karl Jansky (19051950), a physicist working for Bell Telephone
can be more easily detected
Laboratories in New Jersey, realised that part of the background hiss heard in
from space.
Discuss the problems associated
radio communication was coming from the sky. Janksys discovery was followed
with ground-based astronomy
up in 1937 by Grote Reber (19112002), who built a reflecting telescope in his
in terms of resolution and
backyard in Chicago (Figure 21.2.1). This was a radio telescope, with a mirror
absorption of radiation and
made from sheet metal and a diameter of 9.5malmost four times the size of
atmospheric distortion.
the largest optical reflecting telescope of the era. The basic
optical principles of Rebers radio telescope were exactly the
same as those of an optical reflecting telescope, with the
exception that the radio light was detected by a radio receiver
rather than by eye or photography.
Radio astronomy really began to develop after World
WarII, spurred by technical developments during the war
This was possible because a wide range of radio
years.
wavelengths, as well as visible light, penetrate the Earths
atmosphere without significant absorption and can therefore
be observed by telescopes on the ground. Electromagnetic
(EM) radiation of most other wavelengths is absorbed by the
atmosphere before it reaches the ground (Figure 21.2.2).
At wavelengths longer than about 10m, the Earths
ionosphere (see in2 Physics @ Preliminary section 8.4) blocks
radio energy from space. The radio window opens at metre
wavelengths and remains open down to millimetre wavelengths
for telescopes on the ground. However, observing at these
shortest radio wavelengths requires a high, dry location such as
the site of the ALMA mmsub-mm radio telescope, under
construction in the Atacama Desert in Chile.
Figure 21.2.1 Grote Rebers radio telescope

373

Antarctic
astronomy

t may surprise you to learn that


the highest and driest sites
on Earth are in Antarctica. The
altitude in the centre of the
Antarctic continent and the
extreme cold of the atmosphere
make it a tempting site for an
infra-red observatory. Australian
astronomers have studied the
conditions for several years and a
design study for a 2.4m aperture
optical/IR telescope called PILOT
has been conducted. However, a
large Antarctic telescope is still
in the future.

Far infra-red (FIR) light at wavelengths of about 100m (0.1mm)


is absorbed by water vapour in the atmosphere. However, this blocking becomes
patchy as wavelength decreases, with windows of less absorption at certain infra-red
(IR) wavelengths between 1 and 10 m. As a result, some infra-red astronomy can
be done from mountaintop observatories or even aircraft. However, observing
across the full IR range requires a telescope above the atmospherea role most
recently and spectacularly filled by the 0.85m aperture Spitzer Space Telescope.
At wavelengths shorter than IR (i.e. below 1 micron), the atmospheric
window opens again for the narrow optical band between about 350 and 750nm.
It then quickly closes in the ultraviolet (UV) and the shorter wavelength, higher
energy X-ray and -ray wavebands. All of these are now observed from space.

Absorption by atmosphere

21

Eyes on the sky

Galex

Fermi

Spitzer

Hubble

100%

50%

0%

Parkes
Gemini
0.1
nm

1
nm

10
nm

100
nm

1
m

10

100
m

1
mm

1
cm

10
cm

1m

10 m 100 m 1 km

Wavelength

Figure 21.2.2 The Earths atmosphere stops light from many parts of the electromagnetic spectrum
reaching the ground.

Checkpoint 21.2
1
2

Discuss how and why radio astronomy started.


Construct a table listing the various wavebands of the electromagnetic spectrum and describe their ability
to penetrate the Earths atmosphere.

21.3 The telescopic view


Most telescopes operate on the same basic principles, no matter what
their operating wavelength. They use some type of mirror to collect the light
(EM radiation) and focus it onto an appropriate detectoroften using the
same prime focus or Cassegrain configurations illustrated in Figure 21.1.7.
The combination of telescope and detector will set the field of view.

Magnification
Perhaps surprisingly, the concept of magnification (sometimes called
power) is not too important in most astronomical telescopes. It is easily
calculated if the focal length of the objective lens or mirror, fo, and the focal
length of the eyepiece, fe, are known. The magnificationm is then given by:
m=
374

fo
fe

astrophysics
Worked example
Question
A small refracting telescope has an objective lens diameter D of 70mm and focal length fo
of 700mm.
If this telescope has a typical low power eyepiece focal length fe of 25mm, what would
be its magnification?

Solution
m =

fo
fe

700
= 28 (i.e. 28)
25

Sensitivity
The sensitivity of a telescope system describes its ability to see faint objects.
Sensitivity is sometimes called light-gathering power. It depends on how much
light the telescope collects and how much of that light is delivered to the detector.
The primary factor controlling light-gathering power is the diameter D of the
telescopes objective lens or mirror. Most telescopes have circular mirrors and so
the collecting area is proportional to D2. A 10 m diameter telescope therefore
collects 100 times the light of a 1m diameter telescope. This is the primary
reason for building bigger telescopes, although it is often not quite that simple,
as illustrated by the example of the Anglo-Australian Telescope (AAT).

Define the terms resolution


and sensitivity of telescopes.

Worked example
Question
What is the collecting area of the 3.9m AAT, and how does it compare with that of a
70mm diameter refracting telescope?

Solution
The refracting telescope has D = 70mm, so its collecting area can be calculated using:
Collecting area = p

2
D

The AAT has D = 3.9m; however, it also has an obstruction caused by the secondary
mirror and its housing, as seen in Figure 21.3.1. In the f/8 Cassegrain configuration, this
obstruction is about 20% of the collecting area. So the ratio of the collecting areas of the
two telescopes is:
3900 2
p
0.8
2
Collecting area AAT
=
2
Collecting area 70mm
70
p
2

2500
The AAT has 2500 times the light-gathering power of the 70mm telescope.

Once the light has entered the telescope, it must be delivered to the detector
and this may involve a series of lenses and mirrors. Each of these may reflect,
absorb or scatter a little of the light and reduce the amount that reaches the
detector. In the AAT there are just two mirrors between the sky and a detector
placed at the Cassegrain focus, yet 10% of the light is lost.

Figure 21.3.1 The 3.9m Anglo-Australian


Telescope, showing the black
2-degree field (2dF) secondary
mirror structure at the top of
the telescope
375

21

Eyes on the sky

Apart from the telescope itself, the overall sensitivity also depends on several
other factors:
Atmospheric transmissionFor telescopes on the Earths surface, the
proportion of the light that penetrates the atmosphere depends on
wavelength, as illustrated in Figure 21.2.2.
Detector efficiencyOften much less than 100% of the light striking the
detector is actually recorded.
Observing timeUnlike the human eye, most detectors record more light
if they are left exposed for longer times.
BackgroundThe sky is not truly dark because of natural sources and
man-made pollution at optical and radio wavelengths.

Angular resolution

Angular resolution is often simply called resolution. Adding angular reinforces


the idea that we are concerned with an ability to discern (resolve) detail separated
In a practical sense, angular resolution
by very small angles in the sky.
describes whether we can see two closely spaced objectsor do they look like one?
The limit to resolution is set by the wave-like properties of light. Whenever
light encounters an obstacle, such as the aperture of a telescope, it doesnt cast a
sharp shadow. Instead diffraction caused by the edges of the obstacle alters the
For a telescope with a circular objective
path of the light near the edge.
lens or mirror this creates a pattern of light known as an Airy disc (Figure 21.3.2)
at the focusnot an image of a star. For a telescope of diameter D using light of
wavelength , the angular size R of this pattern is given by the equation:

R (in arc seconds) = 1.22


RD

R2D

Figure 21.3.2 (a) A view of an Airy disc,


and (b) the same view
through an aperture with
twice the diameter

D 206265

(Recall the arc second as a unit of angle from in2 Physics @ Preliminary section
13.4.)
The angular size of this pattern depends only on the wavelength of the light
and the diameter of the telescope. For most telescopes, R is much bigger than the
apparent size of even the largest star as seen from Earth.
If two stars are close together, their Airy discs may overlap so that it is
impossible to see that there are two stars. They are unresolved. The stars can
theoretically be resolved and seen as two stars (Figure 21.3.3) if they are
separated by at least
R (in arc seconds) = 1.22

D 206265

Resolution also affects images of larger objects such as the Moon or planets,
since each point of the object acts like a distant star and is blurred into an Airy
disc (Figure 21.3.3).
Figure 21.3.3 A magnified view of an Airy

pattern from an unresolved


pair of stars (left), and a
resolved pair (right)

Figure 21.3.4 Simulated views of Saturn, showing the effect of resolution using (a) a 10cm diameter
telescope and (b) a 30cm diameter telescope
376

astrophysics
Notice that the angular resolution of a telescope is expected to be better
(R is smaller and the image is sharper) for a big telescope. It is also better if the
wavelength is small (e.g. much better for visible light than for radio waves). This
is illustrated by the theoretical resolution values for 10m diameter telescopes
listed in Table 21.3.1. For comparison, the size of the Sun or Moon in the sky is
around 1800arc seconds, while a $1 Australian coin seen from a distance of
100km would appear just 0.05arc seconds across.
Table 21.3.1 Theoretical resolution of 10m diameter telescopes at different wavelengths
Resolution R for D = 10m

Band

Typical wavelength

Ultraviolet (UV)

100nm

0.0025

Optical

500nm

0.013

Near infra-red (IR)

2000nm (2 m)

0.050

Radio (millimetre)

1mm

25

Radio (centimetre)

21cm

5300

(arc seconds)

PRACTICAL
EXPERIENCES
Activity 21.1

Activity Manual, Page


158

Identify data sources, plan,


choose equipment or resources
for, and perform an investigation
to demonstrate why it is
desirable for telescopes to have
a large diameter objective lens
or mirror in terms of both
sensitivity and resolution.

TRY THIS!

Checkpoint 21.3

The man in the Moon

When you look at the Moon, do


you see the face of the man in
the Moon?
Were all limited by the angular
resolution of our eyes. If the pupil
of your eye is open to, say, 5mm,
the theoretical resolution is about
1/100 the size of the Moon in the
sky, and not even quite that in
practice. The result is that you
can only see the broad details of
the dark mare and bright lunar
highlands. Your imagination does
the restand often in not the
same way as someone else!

2
3
4

Calculate the magnification of a telescope that has an objective


lens with a focal length of 500mm and an eyepiece with a focal
length of 12.5mm.
Define sensitivity of a telescope system.
Define angular resolution.
Describe an Airy disc.

21.4 Sharpening the image


In practice, the theoretical resolution of a telescope may not actually be achieved
if the imaging performance of the telescope is not good enough. How good is
good enough?
As a rule of thumb, the surface of a lens or mirror must be within
one-eighth of a wavelength of its correct shape if it is going to produce sharp
images. At optical wavelengths of about 500nm, this means the surface must be
smooth and the correct shape to within about 60nmabout one-thousandth
the thickness of human hair! This extraordinary precision is routinely achieved
with optical polishing techniques. At radio wavelengths of 21cm, one-eighth of
a wavelength is 2.6cm and this is easily achieved with metal plates and even wire
mesh attached to a metal frame.

Active optics
These tight tolerances may be impossible to achieve if the mirrors are so large
that they bend under their own weight. Alternatively, the telescope itself may
bend as it points in different directions. In the past, both effects could largely be
overcome by building massive telescopes. The dish of the Parkes radio telescope
(Figure 21.4.1) is 64m across and weighs 300tonnes, but also has a clever design
to allow for flexure of the structure.

Figure 21.3.5 The face of the man in the


Moonin the southern
hemisphere
377

21

Eyes on the sky

Outline methods by which the


resolution and/or sensitivity of
ground-based systems can be
improved, including:
adaptive optics
active optics.

This approach has been abandoned in recent optical telescopes in


favour of much lighter mirrors and structures that are controlled by an active
optics system that keeps the telescope at optimum performance. For example,
the mirrors of the 8.1m diameter Gemini telescopes (Figure 21.4.2) are only
20cm thickmuch too thin to hold their shape. Instead, a system of 180
computer-controlled actuators push and pull on the back of the mirror to
keep it in shape. The distortions are measured and corrections made about
once a minute.

Figure 21.4.1 The 64m Parkes radio telescope

Figure 21.4.2 One of the 8m Gemini telescopes, in


which Australia owns a small share

Seeing
Discuss the problems associated
with ground-based astronomy
in terms of resolution and
absorption of radiation and
atmospheric distortion.

PRACTICAL
EXPERIENCES
Activity 21.2

Activity Manual, Page


161

378

For all wavelength ranges, coping with the slowly changing distortions in the
telescope itself is an issue. In contrast, the much faster effects of seeing are
problems restricted to ground-based optical and infra-red telescopes.
Seeing refers to the effects on images of tiny, rapidly changing
temperature variations in the Earths atmosphere. These effects distort the path
of the visible and, to a smaller extent, the infra-red light passing through the
atmosphere so that even a perfect optical/IR telescope will not produce a sharp
image. Images in a large telescope are distorted into a constantly moving and
changing speckle pattern that averages over time into a blurred seeing disc
(Figure 21.4.3). The same effects produce the familiar scintillation, or twinkling,
of stars. Scintillation describes rapid changes in the brightness of a star, while
seeing refers to blurring of a star image.
The turbulence that produces seeing is usually concentrated near the ground
and in discrete layers in the atmosphere. It depends on local weather conditions
and geography. The best locations for good seeing are typically tall mountains
that project above the relatively smooth stream of air flowing across open oceans.
Most of the worlds largest optical or IR telescopes are therefore sited on islands,

astrophysics

Figure 21.4.3 Simulated images of a binary star seen through a large telescope in good seeing(a) without seeing effects, (b) the speckle pattern
seen in a 10ms exposure time and (c) the seeing disc seen in longer exposure times. The arrow indicates 1arc second.

or mountains on the western edges of continents. Siding Spring Observatory in


New South Wales, site of the Anglo-Australian Telescope, is relatively poorly sited
in comparison.
The long-wavelength radio waves are unaffected by atmospheric turbulence
and so a mountaintop site is unnecessary for most radio telescopes. Protection
from artificial sources of radio waves is essential, and so
many radio telescopes are in very remote sites, away from
natural guide star
population centres.

science target
laser guide star created
in sodium layer at 90 km

Adaptive optics
Obviously, the ultimate way to beat the seeing effects is to
place a telescope in space, above the atmospheric
turbulence. The 2.4m diameter Hubble Space Telescope
(HST) has spectacularly demonstrated the power of
observing from space since its launch in 1990. However, it
is incredibly expensive and the largest optical/IR telescopes
are on mountain tops, not in space.
For at least 40 years, astronomers and engineers have
been working towards beating the seeing effects for
ground-based telescopes by using adaptive optics.
Adaptive optics is essentially active optics, working much
fastercorrecting hundreds of times a second to overcome
the rapidly changing seeing effects.
An adaptive optics system (Figure 21.4.4) works
by using a wavefront sensor to measure the image
distortion of a target star. A tilting mirror is used to stop
the image moving and a deformable mirror is used to
sharpen the image. The result is a much sharper image,
although still not perfect (Figure 21.4.6).
The target star may not be the real target of scientific
interest, but it must be very close and bright enough for
the adaptive optics system to work. A single system can
measure and correct only a tiny patch of sky, since the
blurring is different just a few arc seconds away. If there is
not a bright enough adaptive optics target star close to the
scientific target, then astronomers can provide one! A laser
guide star is created by firing a high-power laser up from
the telescope, causing the atmosphere to glow at about
90km altitude. To the adaptive optics system it looks like
a star that can be moved to where it is needed.

layers of turbulence
in the atmosphere
laser projection system
telescope

sodium laser

distorted wavefronts
producing a moving
blurred image

control signals for


deformable mirror
deformable
mirror

tilting
mirror

control signals for


tilting mirror

smoothed wavefronts
producing a sharp image

real-time
control computer

wavefront
sensor

moving, blurred
image of
guide star

observing
instrument

sharp image of science target

Figure 21.4.4 Layout of an adaptive optics system, including the science


target, a natural guide star and a laser guide star
379

21

Eyes on the sky

Outline methods by which the


resolution and/or sensitivity of
ground-based systems can be
improved, including:
adaptive optics
interferometry
active optics.

All large optical/IR telescopes now have some sort of adaptive optics system.
The next generation of even larger telescopes will rely on multiple adaptive
optics systems and laser guide stars.

Seeing is not
believing!

he water in a swimming pool


provides a very clear illustration
of how light is affected by the
medium through which it passes.
In Figure 21.4.5 the surface of the
water bends the light paths,
concentrating the light in places
and generally distorting the view of
the bottom of the pool. A similar
effect makes stars twinkle.
Another example is the mirage
seen above water or a road on a hot
day, due to the effects of
temperature variations in the air.

Figure 21.4.6 The blurred image (left) of the star IW Tauri reveals a pair of stars separated by
0.3arc second (right) when imaged using adaptive optics on the 5m Hale Telescope.

Checkpoint 21.4
1
2
3
4
5

Identify the factors that limit the resolution of an optical telescope.


Explain the physical problems that make active correction necessary.
Describe what is meant by the term seeing.
Outline the need for adaptive optics.
Distinguish the differences between active and adaptive optics.

21.5 Interferometry

Figure 21.4.5 A distorted image of the


bottom of a swimming pool,
caused by ripples on the
surface of the water

The goal of adaptive optics is to achieve the resolution limit set by the diameter
of the telescope and the wavelength of the light. Given the long wavelengths of
radio waves, it is impractical to build a single large radio telescope to achieve
The
the resolution possible with even a very small optical telescope.
solution is to use two or more radio telescopes and link them together to form an
interferometer. Provided the effective path to the detector via each telescope is
the same to within a few wavelengths, interference will be seen. The interference
pattern contains information about the image of the astronomical object.
The ability of an interferometer to resolve fine details is governed by the
equation:
R (in arc seconds) = 1.22

D 206265

where D is now the distance between any two telescopes, not their individual
diameters. The resolution of the interferometer is about the same as having a
large telescope of diameter Dbut only in the direction along the line between
the telescopes. At right angles to that direction the resolution is the same as using
only one telescope.
If the interferometer is formed from an array of several telescopes, then each
pair provides good resolution in one direction. With enough telescopes, the array
380

astrophysics
can produce good resolution in many different directionsenough to build up
an image of the target. Also, as the Earth rotates, these directions move relative
to the target and the array can synthesise an even better image.
An interferometer produces the resolution of a single large telescope, but
with much less sensitivity, because only a small proportion of the light is
collected by the array of telescopes (Figure 21.5.1).
Interferometry is fundamental to the operation of almost all large radio
telescopes. The main interferometric array of radio telescopes in Australia is the
Australia Telescope Compact Array (ATCA), located near Narrabri, NSW
(Figure 21.5.2). It has six 22m dishes spread over 6km in a pattern that can be
varied. However, it is sometimes used with other dishes spread around Australia
or the world to form a much larger array.
Interferometry is also now being
applied in optical/IR astronomy. The
Sydney University Stellar
single telescope
mirror of the
Interferometer (Figure 21.5.3), also
same resolution
located at Narrabri, is one of the
pioneering instruments in this field.
The much shorter optical
wavelengths make the tolerances
much tighter on the optical
components and their positions, and
seeing effects are once again a major
problem.
individual telescopes

Figure 21.5.2 Some of the dishes of the Australia Telescope in a compact configuration

2
3

enerally when you combine two


beams of light from a source you
just get more light. If you split the
light and then recombine those two
beams you may get something
different. Provided the two beams
travel the same distance, they
interfere with each other in a way
that depends on the path of the
beams and the characteristics of the
light source. An optical device used
to do this is called an interferometer
(see section 3.2).

Figure 21.5.1 The resolution of a large


telescope can be achieved
using an array of small
telescopes. Each white line
joins an interferometric pair
of telescopes.

Figure 21.5.3 The Sydney University Stellar Interferometer

Checkpoint 21.5
1

What is
interferometry?

has an array of fixed stations along a 640m


baseline. Interference of the light occurs in the
central building (upper centre of the image).

Outline how greater resolution is achieved in interferometry than


with single telescopes.
Calculate the maximum theoretical resolution of the Australia
Telescope Compact Array that has six 22m dishes spread over 6km.
Explain why the resolution of an optical interferometer differs from
that of a radio interferometer.
381

21

Eyes on the sky

21.6 Future telescopes


The forefront ground-based optical/IR telescopes in the world today are 8m
and 10m in diameter. Planning is now underway for the next generation of
Extremely Large Telescopes (ELTs) with apertures up to 42m! Two or three of
these may be operational after 2016. Smaller telescopes still have a role of course,
but even 4m telescopes are only truly competitive if, like the AAT, they have
excellent instrumentation.
Also by about 2016, radio astronomers hope to be commissioning their own
new instrumentthe Square Kilometre Array (SKA). This will actually be an
interferometric array of radio telescopes spaced over 3000km, with a collecting
area of one square kilometre.
By then, the replacement for the 2.4m diameter Hubble Space Telescope will
be operational. The James Webb Space Telescope (JWST) will be a 6.5m diameter
IR telescope. Other smaller space telescopes will continue to observe the sky at
other wavelengths inaccessible from the ground. Observations at many wavelengths
are essential to understanding most astronomical objects (Figure 21.6.1).
a

Figure 21.6.1 (a) A composite image of the nearby active galaxy Centaurus-A, with (b) an X-ray image and
(c) a radio image revealing much more than is seen in (d) the optical image they overlay.

Checkpoint 21.6
Contrast the maximum theoretical resolution of a 42m diameter ELT, the JWST and the SKA. (You will need to assume
some appropriate wavelength values.)
382

PRACTICAL EXPERIENCES

astrophysics

Chapter 21

This is a starting point to get you thinking about the mandatory practical
experiences outlined in the syllabus. For more detailed instructions and advice,
use the in2 Physics @ HSC Activity Manual.

Activity 21.1: Mountains on the moon


Use an image of the Moon to make the same type of measurements that Galileo
used to estimate the size of the mountains on the Moon.
Discussion questions
1 State what assumptions are made in estimating the size of the mountains.
2 Explain how advances in technology in astrophysics have allowed for more
accurate methods to determine the height of lunar mountains.

Figure 21.7.1 A portion of the Moons surface


around the crater Plato

Activity 21.2: A better telescope


Measure the amount of light collected by lenses of differing diameter. Also
determine the size of the lens that can best separate (resolve) two close objects.
Discussion questions
1 Determine the relationship between diameter of a lens and its lightgathering ability.
2 Explain how the diameter of the lens is related to resolution and explain
how this relationship was determined.

Identify data sources, plan,


choose equipment or resources
for, and perform an investigation
to demonstrate why it is
desirable for telescopes to have
a large diameter objective lens
or mirror in terms of both
sensitivity and resolution.

383

21

Eyes on the sky

Chapter summary

The magnification of an optical telescope describes


how much bigger an object appears in the telescope
compared with the naked-eye view.
Refracting telescopes use an objective lens to collect
and focus light.
Reflecting telescopes use a mirror to collect and
focus light.
Field of view describes the area of sky an optical
instrument can see at any moment.
The focal length of a lens or mirror is the distance
between the lens or mirror and the image it forms of
a distant object.
The basic optical principles of most telescopes at all
wavelengths are the same as for an optical reflecting
telescope.
Visible light and a wide range of radio wavelengths
penetrate the atmosphere without significant
absorption and can therefore be observed by telescopes
on the ground. Most other wavelengths are absorbed
by the atmosphere.
Sensitivity (or light-gathering power) of a telescope
system describes its ability to see faint objects.
Diffraction in a telescope with a circular objective lens
or mirror creates a pattern of light known as an Airy
disc at the focusnot an image of a star.

Angular resolution describes whether we can discern


two closely spaced objects as separate (resolved) or not
(unresolved).
The surface of a lens or mirror must be within about
one-eighth of a wavelength of its correct shape if it is
going to produce sharp images.
Active optics systems control telescope mirrors and
structures to keep the telescope at optimum
performance.
Seeing describes the blurring of images caused by tiny,
rapidly changing temperature variations in the Earths
atmosphere.
Scintillation, or twinkling, describes rapid changes in
the brightness of a star caused by tiny, rapidly changing
temperature variations in the Earths atmosphere.
Adaptive optics measures the image distortion of a
target star by using a wavefront sensor, and then
corrects it hundreds of times a second, to overcome the
rapidly changing seeing effects.
Two or more telescopes linked appropriately form an
interferometer, provided the effective path to the
detector via each telescope is the same to within
a few wavelengths. The interference pattern contains
information about the image of the astronomical
object.

Review questions

Physically speaking

Copy and complete the following table with the knowledge that you have gained after studying this chapter.

Concept
Telescope

Angular resolution

Sensitivity

Active optics

Adaptive optics

Interferometry

384

Definition of concept

Diagram to illustrate the concept

astrophysics

Reviewing
1 Recall how the introduction of technology has influenced the discoveries
made in astronomy, with particular reference to Galileo and the telescope.

2 Describe some of the observations Galileo made with his telescope.


3 Name some telescopes that are used to view the universe using wavebands
other than visible light, identifying the wavebands they use as well as
where they are located.

4 Define the following terms and describe how they relate to a telescope:
collecting area
sensitivity
magnification

5 A small value for angular resolution is considered good. Explain what it


means and how it can be achieved in a telescope.

6 Discuss the implications of seeing in optical astronomy.


7 Twinkle twinkle little star is a line from a popular nursery rhyme.
Explain what physical phenomenon is being described in this phrase.

8 Outline how active optics works.


9 Compare and contrast active and adaptive optics.
10 Adaptive optics systems are complex because what they are attempting
is difficult to achieve. Outline some of the factors that make adaptive
optics difficult.

11 Justify the use of interferometry to increase resolution.


12 Assess the importance of telescopes observing at different wavelengths
in gathering information about the universe.

Solving problems
13 Compare the sensitivity of a telescope with a diameter of 10cm with one
of 20cm.

14 The nearby spiral galaxy M31 in the constellation of Andromeda is


approximately 2.5million light-years from the Sun. What is the size
(in light-years) of the smallest features in M31 that can theoretically be
resolved by a 10m telescope at the visible light wavelength of 550nm?

15 What is the size (in light-years) of the smallest features in M31 that can

Re

iew

Q uesti o

theoretically be resolved by the 6km long baseline of the Australia


Telescope Compact Array operating at the radio wavelength of 21cm?

385

21

Eyes on the sky

PHYSICS FOCUS
Beautiful death-star could
threaten Earth
The Earth may be in the firing line when one of the
skys most beautiful objects explodes, according to
University of Sydney astronomer Peter Tuthill.
Dr Tuthill discovered the elegant rotating pinwheel
system, named WR104, in the constellation
Sagittarius. It includes a highly unstable star known
as a Wolf-Rayet, widely regarded by astronomers as a
ticking bombthe last stop in a stars life before
a cataclysmic supernova explosion.

5. Current issues, research and


developments in physics

When it finally explodes as a supernova, it could


emit an intense beam of gamma rays coming our way,
says Dr Tuthill.
New images of WR104 taken with the Keck
Telescope in Hawaii by Dr Tuthill in 2008 show a
glowing plume of hot dust and gas flung out into
a whirling spiral as the two stars at the centre of the
system orbit one another every 8 months.

Figure 21.7.2 Spiral pattern around the massive star WR104, as seen using masked aperture interferometry

386

astrophysics

But something odd about the images caught the


attention of Dr Tuthill and his team: Viewed from
Earth, the rotating tail appears to be laid out on the
sky in an almost perfect spiral. It could only appear
like that if we are looking nearly exactly down on the
axis of the binary system.
Dr Tuthill and his team worry this box-seat view
might put us in the firing line when the system finally
explodes. Sometimes, supernovae like the one that
will one day destroy WR104, focus their energy into a
narrow beam of very destructive gamma-ray radiation
along the axis of the system. If such a gamma-ray
burst happens, we really do not want Earth to be in
the way, warns Dr Tuthill.
At only 8000light-years distance, WR104 is just
down the road in galactic terms, only one-third of the
way to the centre of our Milky Way Galaxy. Earlier
research has suggested that a gamma-ray burst
if we are unfortunate enough to be caught in the
beamcould be harmful to life on Earth out to these
distances. Scientists have speculated that, eons ago,
a gamma-ray burst from a distant star could explain
mass extinctions seen in the fossil record, he said.
I used to appreciate this spiral just for its
beautiful form, but now I cant help a twinge of
feeling that it is uncannily like looking down a rifle
barrel.
But Dr Tuthill is not panicking just yet. There are
still plenty of uncertainties: the beam could pass
harmlessly to the side if we are not exactly on the axis,
and nobody is even sure if stars like WR104 are
capable of producing a fully fledged gamma-ray burst
in the first place.
We probably have hundreds of thousands of years
before it blows, so we have plenty of time to come up
with some answers.

1 What do the colours in the image (Figure 21.7.2)


represent?
2 Do you believe that every detail you see in this
image is real? Why or why not?
3 This image is ~0.5 arc seconds across. How does
the resolution in this image compare with one you
might expect from one of the Keck telescopes?

Extension
4 Peter Tuthill used a technique called masked
aperture interferometry to obtain this image. It
uses a single telescope instead of two or more.
How does interferometry work on a single
telescope?
5 How does the resolution of this technique compare
with that possible using an interferometer such as
the Sydney University Stellar Interferometer or the
Australia Telescope Compact Array?
6 How much of a risk to Earth do you think is posed
by the star WR104?

387

22
astrometry, parallax, astronomical
unit, light-year, parsec, spectroscopy,
spectrum, Fraunhofer lines, continuous
spectrum, emission line spectrum,
absorption line spectrum, black body,
black body curve, Planck curve,
spectroscope, prism, spectrograph,
diffraction grating, spectral classes,
brightness, luminosity, Doppler effect,
photometry, magnitudes,
apparentmagnitude, absolute
magnitude, distance modulus,
colourindex, spectroscopic parallax

Define the terms parallax,


parsec, light-year.
Explain how trigonometric
parallax can be used to
determine the distance to stars.
more distant stars
photo
taken
now
Earth now
r

Sun

Earth
6 months
from now

Figure 22.1.1

388

p
d

relatively
nearby star

photo taken
6 months
from now

The parallax motion of a


nearby star, showing the
parallax angle p

Measuring
the stars
Fingerprinting the stars
The invention of the telescope led to a revolution in astronomy.
The Moon and planets were found to be unique worlds, faint
stars were revealed and new mysteries such as spiral nebulae
were discovered. However, astronomers still couldnt answer the
most basic questions: How far away are the stars? What are they?
The answer to the first question came in 1838 with the first
successful measurement of the parallax of a star. The clue to the
second question was already in place by then with Fraunhofers
observation of spectral lines in light from the Sun. The
understanding of these lines as fingerprints of the stars
is the basis of modern astrophysics.

22.1 How far?


Prior to the invention of the telescope, astronomy was the science of the positions
and motions of the stars and planets. We now call this astrometry. The telescope
made these measurements much more precise, but one of the prizes of astrometry
remained elusivea parallax measurement of the distance of a star.
Stellar parallax (see in2Physics @Preliminary Figure 13.3.2) is the
apparent change in position of a nearby star relative to more distant stars. Over
the period of a year, any nearby star will trace out a tiny ellipse in the sky,
mimicking the Earths orbital motion (Figure 22.1.1). The closer the star, the
larger the ellipse will be. The parallax angle p measures the angular length of half
the major axis of the ellipse.
Parallax, as a surveying concept, was well known to ancient astronomers.
It offers an unambiguous distance measurement based on simple geometry.
Unfortunately, even the largest stellar parallax motion is less than 1 arc second
about the size a star appears because of seeing effects (see section 21.4). The first
successful measurement was made by Friedrich Bessel (17841846) in 1838,
using visual observation of the star 61 Cygni. Later in the 19th century
photography replaced visual observations.

astrophysics
The distance d of the star and the parallax angle p are simply related by:
r
tan p =
d
Using the small angle approximation (sinp tanp p) since p is so small, and
measuring p in arc seconds, r in astronomical units (AU) and d in parsecs (pc)
results in an even simpler form for this equation:
p=

1
d

Worked example
Question
Proxima Centauri is the nearest star to Earth. It has a parallax p = 0.76887 arc seconds.
What is its distance in parsecs, light-years and astronomical units?

Solution
1
1
d= =
= 1.3006 pc = 4.2421 ly = 26 827 AU = 4.0133 1016 m
p 0.76887

Ground-based measurements of parallax are hindered by seeing effects, yet the


uncertainty in the measured angles is, remarkably, only about 0.01 arc seconds.
Realistically, the limit of ground-based parallax measurements with an acceptable
uncertainty (of, say, 30%) is about 0.03 arc seconds, corresponding to a distance
of 30 pc.
Between 1989 and 1993 Hipparcos (high precision parallax collecting satellite)
measured the parallax of 118000 stars to a precision of 0.001 arc seconds and
more than 1 million stars to lower precision. This represents a factor of 10
increase in the distance range of parallax measurements to, say, 300 pc. However,
compared with the distance of about 8000 pc to the centre of the Milky Way
galaxy, the stars with accurate Hipparcos distances are still very local!
In about 2012 the astrometric boundaries should begin to expand even
further, with the launch of the Gaia spacecraft. It will survey 1 billion stars with a
typical precision of 0.00002 arc seconds, reaching out to the centre of the galaxy.

Checkpoint 22.1
1
2

Explain exactly what the parallax angle p measures.


Outline the reasons for using satellites to measure parallax angles.

Units of
distance

arallax measurement is based


on the size of the Earths orbit,
so a useful unit of distance is the
astronomical unit (AU). This is
approximately the average distance
between the Earth and the Sun.
1 AU 1.4960 1011 m
The most familiar distance unit
in astronomy is the light-year (ly).
This is the distance that light
covers in 1 year, travelling at
2.998 108 m s1.
1 ly 63241 AU

9.4605 1015 m.
The parsec (pc) is an alternative
unit that represents the distance of
a hypothetical star that has a
parallax angle of 1 arc second.
1 pc 206265 AU 3.2616 ly

3.0857 1016 m

Discuss the limitations of


trigonometric parallax
measurements.

PRACTICAL
EXPERIENCES
Activity 22.1

Activity Manual, Page


167

PRACTICAL
EXPERIENCES
Activity 22.2

Activity Manual, Page


171

22.2 Light is the key


In his book The Positive Philosophy, the 19th century French philosopher August
Compte (17981857) wrote about our knowledge of the stars and planets.
... We see how we may determine their forms, their distances, their bulk, and
their motions, but we can never know anything of their chemical or mineralogical
structure ...
389

22

Measuring
the stars

The distances in astronomy are an enormous barrier to learning about the stars and
planets, but Compte was wrong in thinking we would never know about their
composition. Astronomical spectroscopy is the study of the light from objects in
the universe to reveal their composition and physical characteristics.
The first step had been made long before when Isaac Newton
(16431727) realised that white light is composed of component
colours, revealed when the light is passed through a glass prism. In
1802, William Wollaston (17661828) looked more closely and
noticed dark lines in the rainbow of colours that form the spectrum
of the Sun. In 1814 Joseph von Fraunhofer (17871826) used a
spectroscope to rediscover the lines and eventually catalogued over 570
of these Fraunhofer lines (Figure 22.2.1). By 1860, Gustav Kirchhoff
(18241887) and Robert Bunsen (18111899) and others discovered
that chemical elements produced bright spectral lines when heated and
They
some of these lines matched the dark Fraunhofer lines.
deduced that the Fraunhofer lines were produced by absorption of light
Figure 22.2.1 Fraunhofers spectral lines superimposed
by these elements in the Sun, providing the key to understanding the
on the rainbow spectrum of the Sun
composition of the stars.
Kirchhoff s empirical laws of spectrum analysis describe how to produce
three of the key types of spectra we observe in astronomy:
Account for the production of
A hot, dense gas produces a continuous spectrum.
emission and absorption
A hot, low-density gas produces an emission line spectrum.
spectra and compare these
A continuous spectrum source viewed through a cool, low-density gas produces
with a continuous black body
spectrum.
an absorption line spectrum.
Describe the technology
needed to measure
astronomical spectra.

PRACTICAL
EXPERIENCES
Activity 22.3

Activity Manual, Page


174

400

500

600

Wavelength (nm)

Figure 22.2.2

A continuous spectrum

Continuous spectra
When you heat an object such as a steel bar it begins to glow red and orange and
then perhaps white before melting. A very low resolution spectrograph reveals that
at any of these temperatures the bar is emitting a continuous range of
wavelengthsa continuous spectrum (Figure 22.2.2).
Any relatively dense materiala solid, liquid or high density
gaswill behave in the same way. They approximate a black body (see
section 9.2), which is the idealised example of a hot object. A black
body emits light at all wavelengths, with a distribution described by a
700
black body curve (or a Planck curve) that depends only on
temperature (Figure 22.2.6), not composition. In astronomy, the
interior of a star is very close to a black body source.
The wavelength of peak emission of a black body curve max is given by
Wiens law (see in2 Physics @ Preliminary section 14.2):
2.9 103
max =
T
where T is the temperature on the kelvin scale. The change in max with
temperature alters the balance of the visible colours and leads to a change
in the overall colour of the object. Cool objects (T 4000 K) look red but emit
most of their light in the infra-red. Peak emission from a hot object (T 10000 K)
is in the ultraviolet (UV) and the object looks blue-white to the eye.
The total energy emitted per second per unit area of the surface is
represented by the area under the black body curve. This power increases very
rapidly with temperature according to the StefanBoltzmann law:

P = 5.67 108 T 4
390

astrophysics

PHYSICS FEATURE

light from
telescope

Spectroscopes
and spectrographs

slit

grating
focusing
mirror

camera

Figure 22.2.3

A basic spectrograph

b
Intensity

spectroscope is a device that disperses light into


its component colours to be viewed by eye. Early
spectroscopes used a prism to refract the various
colours to different angles, according to their
wavelengths. Replacing the eye with a camera creates
a spectrograph, the key instrument in astronomical
spectroscopy.
The spectral resolution of a spectrograph defines
how much detail can be seen in the spectrum, in the
same way that angular resolution is used for images.
Optical spectral resolution ranges from low (~0.5 nm)
to ultra-high (~0.001 nm).
Anything beyond the lowest resolutions requires
greater dispersion of the light than is possible with a
prism. This can be achieved using a diffraction grating,
often made by ruling very fine lines onto an optical
surface. Interference between light beams diffracted
by the lines results in a spectrum of the light striking
the surface.
Most spectrographs use a slit to restrict the
light beam. The light is dispersed in the direction
perpendicular to the slit (Figure 22.2.3). This
direction measures wavelength. Depending on the
optical set-up, distance along the slit may measure
distance across the target object.

collimating
mirror

measure intensity
along a slice
through the spectrum
H

H
H
500

600

700

Wavelength (nm)

Figure 22.2.4

A spectrum as (a) coloured light and


(b) an intensity profile

The resulting spectrum of a single point such as


a star can be shown as a coloured band (Figure
22.2.4a), but it is usually presented as an intensity
profile versus wavelength (Figure 22.2.4b).
Spectrographs are the key instruments on all
optical telescopes. At the Anglo-Australian Telescope,
AAOmega (Figure 22.2.5) is a modern spectrograph,
fed by optical fibres bringing light from multiple
targets in the image.

5. Current issues, research and


developments in physics

Figure 22.2.1 Fraunhofers spectral lines superimposed


rainbow Rob
spectrum
the Sun the grating feeding
Figure 22.2.5 on the
Astronomer
Sharpofexamines
a camera (the assembly in the right half of the
image) in the AAOmega spectrograph.

391

22

Measuring
the stars

Worked examples
PRACTICAL
EXPERIENCES
Activity 22.4

Activity Manual, Page


178

Question
In Figure 21.0.2, the two bright stars on the left are and Centaurithe pointers to the
Southern Cross. a Centauri is actually a double star, and the brighter star has a surface
temperature of ~5800 K. b Centauri is a triple system, with the brightest star having a
surface temperature of ~23000 K. Which star is which in the image?

Solution

Intensity

Using Wiens law for the two stars:


Centauri
2.9 10 3
max =
5800
500 nm
This is in the middle of the visible band, so we expect a yellow overall colour.

12 000 K
7500 K
6000 K
4500 K
3000 K

500

1000

1500

2000

Wavelength (nm)

Figure 22.2.6

Black body curves for different


temperatures show that the peak
moves to shorter wavelengths with
increasing temperature and the
curves move up, indicating greater
energy output at all wavelengths.

Centauri
2.9 10 3
max =
23 000
130 nm
This is in the ultraviolet, so we expect a blue-white overall colour.
Clearly the left star in the image is Centauri and the right star is Centauri.

Question
What is the relative power output of the two stars?

Solution
Using the StefanBoltzmann law for the two stars:
Centauri
P = (5.67 108 Wm2 K 4 )(5800)4
= 6.4 107 Wm2
Centauri
P = (5.67 108 Wm2 K 4 )(23 000)4
= 1.6 1010 Wm2
So, each square metre of the surface of the hotter star Centauri emits nearly 250 times
more energy per second than the surface of Centauri. The hotter star is also about 8 times
larger and so has 64 times the surface area of the cooler star. Thus the luminosity of the
hotter star is about 16000 times the intensity of the cooler one.

Emission line spectra


447.1

471.3 492.1 501.5

587.5

667.8

Wavelength (nm)

Figure 22.2.7

392

An emission spectrum

When you inject energy into a low-density gas, the spectrum is very
different. It shows bright emission lines at wavelengths characteristic of the
elements present in the gas (Figure 22.2.7). Familiar examples include the spectra
from gas discharge lamps and neon signs. Low-density gas clouds in interstellar
space also glow with an emission line spectrum.

astrophysics

The origin of emission lines lies in the structure of the atom (see Chapter
12 From Rutherford to Bohr). When an atom absorbs energy, an electron will
make a transition to a higher energy state, provided the absorbed energy exactly
matches the energy of the transition. It will often then quickly make one or more
downward transitions back to lower energy states, emitting energy.
This process is described as the absorption or emission of photons. In making
a downward transition from energy E2 to energy E1, an atom emits a photon with
energy E related to its wavelength (or frequency f) by the relation:
E = E 2 E1 =

h
= hf
c

where Plancks constant h = 6.626 1034 J s and c = 3.00 108 m s1 is the


speed of light. These photon energies (and hence wavelengths or frequencies)
correspond to the observed emission lines. The pattern of emission lines is a
fingerprint for each element because it is matched to the energy levels within each
atom of that element.

Absorption line spectra


The spectrum of the Sun and stars is a continuous spectrum, but crossed by dark
The origin of these absorption lines
lines, not bright ones (Figure 22.2.8).
lies in the relatively cool gas overlying the hotter, denser gas deep in the star. The
denser gas produces a continuous spectrum. Atoms in the overlying gas absorb
light from the continuous spectrum, but only at the wavelengths matching
differences in their energy levels. They then re-emit the light, but in all directions,
not just outwards. The net effect is less light in the outward direction at those
wavelengths, creating dark lines in the spectrum.
The spectral fingerprint of a gas is the same, whether seen in absorption
or emission. In the Sun and stars, it is the outer layer of the star whose composition
is imprinted on the spectrum as absorption lines (Figure 22.2.9).
The spectra of other astronomical objects depends on their physical conditions
of temperature, density and composition. Table 22.2.1 gives some examples.

black body source


(e.g. deeper layers
of star)

continuous spectrum

Where does
black body light
come from?

f light is emitted by atoms at very


precise wavelengths, how is a
continuous spectrum produced?
The answer lies in the density of
the material emitting the light.
If the density is low, the atoms are
relatively far apart and they emit
light at the discrete wavelengths
expected. However, if the density is
high, the atoms are closer together,
and they influence one another and
alter the energy levels of the atoms.
The effect is to blur the lines into
a continuous distribution.

Wavelength (nm)

Figure 22.2.8

An absorption spectrum

Describe how spectra can


provide information on surface
temperature, rotational and
translational velocity, density
and chemical composition of
stars.
Identify the general types of
spectra produced by stars,
emission nebulae, galaxies
and quasars.

absorption spectrum

diffuse gas
(e.g. outer layers of
a star or a nebula)
emission spectrum

Figure 22.2.9

All three types of spectra are produced by the Sun: continuous spectrum from
deeper layers, absorption lines from the photosphere and emission lines from the
chromosphere and corona.
393

22

Measuring
the stars

Table 22.2.1 Spectral characteristics of some astronomical objects


Description

Spectrum

Emission
nebulae

Regions of gas (mostly


hydrogen and helium) that
glow due to intense UV
light from embedded young
hot stars

Dominated by strong
emission lines
characteristic of the gas
composition

Example
1
0.8
Relative flux

Object

0.6
0.4
0.2

450

500

550

600

650

(nm)

Collections of billions of
stars, gas and dust; light
output is generally
dominated by the mix of
stars of various types

Absorption spectra, often a


mix of bright blue-white
types with many more
numerous but fainter yellow
stars

0.3

Relative flux

Normal
galaxies

0.2

0.1

400

500

600

700

800

(nm)

A type of galaxy with an


active nucleus dominating
the total energy output

A continuous spectrum
with emission lines (that
may be variable) suggesting
fast-moving gas clouds

0.8

0.6
Relative flux

Quasars

H + [OIII]

0.4

0.2
400

500

600

700

800

(nm)

Checkpoint 22.2
1
2
3

Describe the Fraunhofer lines.


Describe how emission lines are produced.
Describe the relationship between the absorption lines and the emission lines of an element.

22.3 The stellar alphabet


In 1885 astronomers at Harvard College Observatory began to compile a
photographic catalogue of spectra that became the Henry Draper Catalogue of
Stellar Spectra. When the final extension was published in 1949, almost 360000
stars had been classified.
394

astrophysics

Spectral classes
The Harvard classification scheme developed during the project remains in use
today. It started as groups of spectra with similar spectral lines, given letters
from A to N, with O, P and Q added for some unusual stars. Re-ordering and
simplification of the sequence led to the current, seemingly random sequence
The major spectral classes are:
of letters.

Brightness
and Luminosity

ecall the definitions of


brightness and luminosity
from in2 Physics @ Preliminary
section 15.1. The brightness of a
star is a measure of the energy
received in a certain time per unit
of collecting area (or power per
unit area) W m2. Its luminosity is
the total power output of the star
in watts. A star will look brighter
when seen through a telescope.
It obviously isnt really more
luminous; the telescope has a
larger collecting area than your
eye alone, making the star
appear brighter.

OBAFGKM
(We can remember this odd sequence using various mnemonics, the best
known being Oh Be A Fine Girl [Guy] Kiss Me). With additions of other classes,
the sequence can be represented as:
WOBAFGKMLT
The black body curves underlying the shape of the spectra clearly imply
a sequence in surface temperature from hot O stars to cool M stars. It was not
until the 1920s that the understanding of electron energy levels in atoms
progressed enough to understand how surface temperature also controlled the
appearance of the spectral lines. Composition differences between stars are
relatively small and are not the reason why spectra vary so much.
A sample of spectra from various spectral classes is shown in Figure 22.3.1.
Some of the key features of each spectral class are described in Table 22.3.1,
emphasising that the colour of a star, its spectral class and surface temperature are
all closely related.
Each spectral class has been divided into 10 sub-classes. The Sun, for example,
is a yellow G2 star with a surface temperature of ~5770 K, placing it at the hot end
of spectral class G.

Luminosity classes
When plotting a HertzsprungRussell (HR) diagram (see in2 Physics @ Preliminary
section 15.3), we can use the spectral class on the horizontal axis, since it is closely
related to the surface temperature (Figure 22.3.2).

Describe the key features of


stellar spectra and describe how
these are used to classify stars.

Table 22.3.1 Characteristics of spectral classes of stars


Spectral class
W

Effective temperature
(K)
>50 000

Colour
Blue

Strength of
hydrogen lines
Weak

Other spectral
features
He, C, N emission
lines
+

% of main sequence
stars
Extremely rare

3100050000

Blue

Weak

Ionised He lines,
strong uv continuum

0.00003

1000031000

Blue-white

Medium

Neutral He lines

0.1

750010000

White

Strong

Ionised metal lines

0.6

60007500

White-yellow

Medium

Weak ionised Ca+

53006000

Yellow

Weak

Ionised Ca+, metal


lines

38005300

Orange

Very weak

Ca , Fe, strong
molecules, CH, CN

12

21003800

Red

Very weak

Molecular bands, e.g.


TiO, neutral metals

76

12002100

Red

Negligible

Neutral metals,
metal hydrides

Brown dwarf
numbers uncertain

<1200

Red

Negligible

Methane bands

Brown dwarf
numbers uncertain

395

22

Measuring
the stars

1200

Table 22.3.2 Luminosity classes


Luminosity class

1000

Ia
Ib
II
III
IV
V
VI
VII

H
H

800

05
09
B6
A2
A6
F0
F7
G2

Na

400

G7
F0
K5

200

M2

Ca II

M4
TiO

0
350

400

450

Figure 22.3.1

TiO

TiO

500
550
Wavelength (nm)

TiO

600

Effective temperature (K)


7 000

6 000

4 000

10

10

Supergiants (I)

10

0
Ma

10

Giants (II, III)


in s

4
6

Subgiants (IV)
equ
enc
Sub e (
dw V)
arf
(VI
)

10
1
1

10

8
10

10

White
dwarfs
(VII)

12

10

14

10
O5 BO

0.5

396

AO

FO
GO
Spectral class

KO

0.0

+0.3
+0.6
Colour index

+0.8

MO
+1.4 +2.0

Luminosity compared to Sun

10

The HR diagram clearly illustrates that stars


with the same surface temperature may have very
different luminosities. This can only imply that one star
has more surface area than the otherone is a giant in
comparison to the other (look again at the second
worked example in section 22.2). These differences in
luminosity, and hence size, are described by the
luminosity classes listed in Table 22.3.2 and illustrated
in Figure 22.3.2.
The Sun is a G2 V star on the main sequence of the
HR diagram. A G-type Ib supergiant like Cephei
(see section 23.4) has a similar spectrum but is about
2000 times more luminous than the Sun.

More from spectral lines

650

Examples of spectra representing stars of


different spectral classes, with hot stars at
the top and cool stars at the bottom.
Spectral lines due to hydrogen and calcium
and some molecular bands are identified.

30 000 10 000

Bright supergiant
Supergiant
Bright giant
Giant
Subgiant
Main sequence dwarf
Subdwarf
White dwarf

600

Absolute magnitude (Mv)

Relative flux

He

Description

Another pointer to the size of stars can be found in their


spectral lines. Supergiant stars have very sharp spectral
lines because the atoms in the low-density outer layers
are widely spaced. Dwarf stars like the Sun have higher
densities in their outer layers, causing the atoms to
interact more, slightly changing their energy levels and
hence slightly broadening their spectral lines.
This is just one of many extra clues about stars
hidden in the spectral lines. Many others are related to
the Doppler effect (see in2 Physics @ Preliminary
section 13.8) caused by motion of the gas emitting or
absorbing the light. At velocities much less than the
speed of light c, the Doppler shift of a spectral line
of wavelength is related to the velocity v along the line
of sight (Figure 22.3.3) by:
v
=

c
The wavelength shifts lead to broadening of spectral
lines that encode information about the temperature
and motion of the gas.

Figure 22.3.2

An illustration of the distribution of stars in the


HR diagram, emphasising that stars of different
luminosity classes may have the same spectral type

astrophysics
Worked example
city
velo
t)
rse f sigh
sve
o
tran s line
ros
(ac

veloc
ity th
ro
space ugh

Since the first confirmed detection in 1992, more than 300 planets have been detected
around other stars. Most have been found by looking for a shift in the stars spectral lines
due to the motion of the star caused by the orbiting planets.

Question
1

Jupiters motion causes the Sun to wobble around a small circle at 12 m s . What shift in
wavelength does this represent at an observing wavelength of 500 nm?

Solution

12 ms 1
=
500 109 m 3.00 108 ms 1

line

ght

i
of s

star

city t)
elo
al v f sigh
i
d
o
ra
line
ng
(alo

Earth

The periodic shift in wavelength = 2 1014 m = 2 105 nm.


This periodic back and forth shift of all the spectral lines is very small and comparable
to the broadening of spectral lines due to the density of gas in the Suns photosphere, but
1000 times less than the thermal broadening due to the motion of the hot gas atoms.

Figure 22.3.3

The Doppler effect measures


the radial velocity along the
line of sight. This is generally
only a component of the true
velocity through space.

Checkpoint 22.3
1
2
3
4

Describe how the spectral classes came about.


Describe what is meant by a luminosity class.
Explain why dwarf stars have broader spectral lines than larger stars.
Explain the origin of blue and red shifting of spectral lines in stars.

22.4 Measuring magnitudes


Astronomical photometry is the measurement of the brightness of an
astronomical source. In optical astronomy, this used to require a photoelectric
photometer. This usually employed a photomultiplier tube that produced a pulse
of electrons when struck by a photon from a target star. Photographic film could
also be used, but is inefficient. Both have now been replaced in most situations
by charge-coupled device (CCD) cameras, the same devices used for modern
astronomical imaging and similar to the detectors in a modern digital camera.
CCDs offer very high detection efficiency and can record many objects at once.
Sensible SI units for brightness are W m2. However optical astronomy has
inherited the traditional system of magnitudes (see in2 Physics @ Preliminary
section 15.1) that is much harder to use!
The magnitude system is usually said to have originated with the Greek
astronomer Hipparchus (~190120 bc e ). He called the brightest stars
In 1856
magnitude 1 and the faintest stars seen by eye magnitude 6.
Norman Pogson (18291891) suggested formalising the system and making
the magnitude scale a multiplying scale, so each step up in magnitude represents
a factor of about 2.5 in brightness. More precisely, he defined a first magnitude
star to be exactly 100 times the brightness of a sixth magnitude star (a difference
of 5 magnitudes). Each magnitude is then a factor of 1001/5 2.512. This
behaviour is logarithmic and roughly mimics the behaviour of the eye.

Describe the advantages of


photoelectric technologies
over photographic methods
for photometry.

397

22

Measuring
the stars

Mathematically, this can be expressed as:

PRACTICAL
EXPERIENCES
Activity 22.6

or equivalently:

Activity Manual, Page


183

Define absolute and


apparent magnitude.

I
mB mA = 2.50 log10 A
IB

IA
m m
= 100( B A )
IB

These equations show how a difference in magnitudes (mB mA) is related to


a ratio of brightnesses (or intensities) IA/IB.
The system is difficult because it is logarithmic, not linear, and larger
magnitudes represent fainter stars.

Apparent magnitude
The apparent magnitude is a measure of how bright an object is in our
sky. The exact zero-point of the system must be defined carefully, but the Sun,
Moon, some planets and even some stars are clearly brighter than zero
magnitude. Their values are therefore negative. Many stars are invisible to the
unaided eye and therefore have magnitudes much greater than 6 (Figure 22.4.1).

Worked examples
Question
Show that a brightness ratio of 100 corresponds to a difference of five magnitudes.

Solution

30
25

Sun

20

Apparent magnitude (mv)

15

full moon
10
5
0
5

Venus at
brightest
Sirius
Polaris
naked eye limit

10

Figure 22.4.1

398

Question
The two stars of the Centauri binary system have apparent magnitudes of 0.01 and 1.34.
a Centauri is believed to be a triple system, with the star Proxima Centauri much fainter
at apparent magnitude 11.09 and some distance away on the sky. What is the relative
brightness of these three stars?

Solution
Choose mA = 0.01, mB = 1.34 and mC = 11.09
IA
= 1001.33 5 3.4
IB

(mC mA) = 11.09 0.01 = 11.08, then

20

30

mB m A = 2.50 log10 (100) = 2.5 2 = 5

Then: (mB mA) = 1.34 0.01 = 1.33, then

15

25

IA
=100
IB

faintest
objects seen
by HST

The brightness of some


astronomical objects on
the magnitude scale

So the brightness ratios A : B : C are 1 :

IA
= 10011.08 5 27 000
IC

1
1
:
.
3.4 27 000

astrophysics

Absolute magnitude
Apparent magnitude doesnt tell the whole story, since it does not indicate the
The magnitude system handles this by calculating
true luminosity of a star.
how bright the stars would appear to be in our sky if all were moved to a standard
distance of 10 pc (32.6 ly). To calculate this absolute magnitude we use the
inverse square law (see in2 Physics @ Preliminary section 15.1), which can be
2
expressed as:
I A 10
=
I 10 d A
I
where A is the ratio of the brightness (intensity) at its actual distance dA
I 10
(in parsecs) compared to its brightness at 10 parsecs. Using this expression in the
equation defining magnitudes leads to:
I
m10 mA = 2.50 log10 A
I 10
10
= 5 log10
dA

The absolute magnitude is usually represented as M. The relationship


between apparent magnitude m and absolute magnitude M for a star at distance
d can then be written as:
d
m M = 5 log10
10
or

d
M = m 5 log10
10

The quantity m M is often called the distance modulus.


Absolute magnitude is a measure of the luminosity of a star. It is often used
as the vertical axis in an HR diagram.

Worked examples

Explain how the concept


of magnitude can be used
to determine the distance
to a celestial object.

Question
Assuming the apparent magnitude of the Sun is 26.72, what is its absolute magnitude?

Solution

1
The distance of the Sun is 1 AU = 206 265 pc
d
M = m 5 log10
10

1
= 31.57
26.72 M = 5 log10
2 062 650
Therefore M = +4.85.
If the Sun were placed at a distance of 10 pc, it would appear as one of many relatively
faint stars visible with the unaided eye.

399

22

Measuring
the stars

Solve problems and analyse


information using:
d
M = m 5 log
10
and

Question
The brightest star in the sky is Sirius ( Canis Majoris) with m = 1.43 and M = +1.47.
How far away is Sirius?

Solution

IA
m m 5
= 100( B A )
IB

d
1.43 1.47 = 5 log10
10
2.90
d
= log10
10
5

to calculate the absolute or


apparent magnitude of stars
using data and a reference star.

d
102.90 5 =
10
Thus d = 10 0.263 = 2.63 pc.
This makes Sirius one of the closest stars to Earth. Although it is more luminous than
the Sun, it is the brightest star in our sky mainly because it is so close.

Checkpoint 22.4
1
2
3

Briefly outline the history of the magnitude system.


Deduce (do not calculate) the intensity ratio between star A with m = +3 and star B with m=2.
Deduce (do not calculate) the absolute magnitude of a star that is d = 100 pc away and has a magnitude of m = +5.

PRACTICAL
EXPERIENCES
Activity 22.5

Activity Manual, Page


180

Figure 22.5.1

400

A star field imaged through


(top) red and (bottom) blue
filters. Note the change
in relative brightness of
many stars.

22.5 Colour matters


As we saw earlier, the colour of stars arises from the distribution of the wavelengths
they emit. The black body curve (Figure 22.2.6) shows that hot stars emit most
of their light at blue and UV wavelengths. Why is such a star usually described
as blue-white and not simply violet?
The answer lies in the response of the human eye to light. The spectral
sensitivity of the eye peaks in the yellow-green part of the visual band and falls
The overall colour impression is therefore a
quickly at blue wavelengths.
result of both the spectrum of the star and the spectral sensitivity of the eye.
Coloured filters are often used to define the spectral sensitivity of a detector.
The importance of this is illustrated in Figure 22.5.1 in which stars of different
colours have different relative brightness when observed through different
coloured filters. Different observers using different filters may then measure
different brightness for a single star. Who is right?
The solution to this problem is to agree on standard filters to use when
measuring brightness. The first standard was the Johnson UBV (ultraviolet, blue
and visual) set. Magnitude values quoted previously were all visual magnitudes
measured through a V filter or converted to that standard. An apparent visual
magnitude is often represented as simply V (although mV is better). An absolute
visual magnitude would then be represented as MV.

Colour index
The difference in brightness seen through different filters is a measure of the
colour of a star. A blue-white star will appear brighter (with a lower magnitude
value) through a filter that passes blue light than through a filter that passes only

astrophysics
red light. Using these magnitude measurements, the concept of colour can be
A colour index is the difference in the brightness of a
assigned a value.
star, in magnitudes, when measured through two different filters.
Many colour indices are possible, but the best known is the (B V) colour
index (or mB mV), the difference between the blue and visual apparent
magnitudes. The system is set up so that white A0 stars have (B V) = 0.
The colour index of the Sun is +0.66. Blue-white stars have small negative
colour index values, while orange-red stars have larger positive colour indices
(Figure 22.5.2).

Explain how two-colour values


(i.e. colour index, BV) are
obtained and why they are useful.

Worked example
Question
Assuming the colour index of the Sun is +0.66, what is
the apparent B magnitude (or mB) of the Sun? What is
the absolute B magnitude (MB) of the Sun?

Crucis
V = +1.59
B V = +1.60

a Centauri A
V = 0.01
B V = +0.71

d Crucis
V = +2.79
B V = 0.19

Solution
From an earlier worked example, V = mV = 26.76 and
MV = +4.85.
Colour index = B V

0.66 = B (26.76)
Therefore B = 26.10

b Crucis
V = +1.25
B V = 0.24
b Centauri
V = +0.61
B V = 0.23

Similarly:

a Crucis
V = +0.77
B V = 0.24

e Crucis
V = +3.59
B V = +1.39

Colour index = MB MV

0.66 = MB 4.85

Therefore MB = 5.51
The fact that the blue magnitudes are greater than the visual magnitudes
implies the Sun is fainter in the blue, as expected for a yellow star.

Figure 22.5.2

Magnitudes and colour


indices for some stars in and
around the Southern Cross

Because colour index quantifies the concept of colour, it is closely related to


both surface temperature and spectral class. Any of these three may be used to
plot an HR diagram (Figure 22.5.3), but colour index has the advantage of being
a measured value.

Spectroscopic parallax
Measuring the colour index of a star places it within a narrow strip on
the HR diagram. Studying details of the spectral lines identifies the stars
luminosity class within that strip. With these two measurements we have
approximately located it on the diagram, without knowing its absolute
magnitude (see Figure 22.5.3). However, reading the vertical scale now reveals
absolute magnitude. Comparison of the absolute magnitude and the apparent
magnitude will then yield a distance, using the equation given earlier.
This method of determining distance is called spectroscopic parallax.
This is a poor name since parallax is not involved. This procedure is essentially
identifying the type of star from the characteristics of its light. Then, knowing
how bright such stars are, a distance can be calculated using the inverse
square law.

Outline spectroscopic parallax.

401

22

Measuring
the stars

Effective temperature (K)


30 000 10 000

7 000

6 000

4 000

10

10

Crucis

Absolute magnitude (Mv)

Crucis
4
Crucis
2 Centauri
Crucis
0
Giants (II, III) Crucis
Ma
in
seq
2
u
Subgiants (IV)
Sub ence
(V)
dw
Centauri A
arf
4
(VI
)
Sun
6
Centauri B
8
10

10
10
1

10

10

White
dwarfs
(VII)

12

10

Luminosity compared to Sun

10

Supergiants (I)

This is a powerful technique since the colour index


can be measured and the luminosity class can be estimated,
provided there is enough light to obtain a good spectrum.
Often there is no other way to know the distance to a star.
However, the uncertainties are often large and the
distances derived are only approximate.

10

14

10
O5 BO

0.5

AO

FO
GO
Spectral class

KO

0.0

+0.3
+0.6
Colour index

+0.8

MO

Figure 22.5.3

Using spectroscopic parallax to estimate


the distance of Crucis

+1.4 +2.0

Worked example
Question
One of the stars identified in Figure 22.5.2 is Crucis, the brighter reddish star in the
Southern Cross. Estimate its distance using spectroscopic parallax, given that it is a
M3.5 III star with V = 1.59 and colour index +1.60.

Solution
From the HR diagram in Figure 22.5.3, the absolute magnitude of Crucis is just above zero,
say 0.5.
Then:

d
m M = 5 log10
10
d
1.59 ( 0.5) = 5 log10
10
2.09
d
= log10
10
5
d
102.09 5 =
10

Thus d = 10 2.62 26 pc.


The distance measured by the Hipparcos satellite is 27.0 0.5 pc, which shows remarkable
agreement, considering the estimates involved in the spectroscopic parallax value.

Checkpoint 22.5
1
2
3

402

Outline the need for standard filters in measuring brightness.


Interpret what a colour index of zero tells you about a star.
Explain how knowing the colour index and luminosity class of a star can lead to finding its absolute magnitude
using the HR diagram.

PRACTICAL EXPERIENCES

astrophysics

CHAPTER 22
This is a starting point to get you thinking about the mandatory practical
experiences outlined in the syllabus. For detailed instructions and advice, use
in2 Physics @ HSC Activity Manual.

Activity 22.1: Parallax


Determine the angle of parallax of an object at the back of the classroom in order to
determine how far away it is. Use this method to understand the use of parallax to
determine the distance to nearby stars.
Equipment: protractor, metre ruler.
Discussion questions
1 Compare the method used in the classroom with the method used to
measure the distance to the stars. What differences are there between
the methods?
2 Explain how the accuracy in results changes when the angle is increased.

Solve problems and analyse


information to calculate the
distance to a star given its
trigonometric parallax using:
1
d=
p

Activity 22.2: Limits of parallax


Research the limits of parallax measurement for ground-based and space-based
telescopes. Use the template to help focus your search. Summarise your findings in
a table.
Discussion questions
1 What minimum angle of parallax can be measured using ground-based
telescopes before the uncertainty of the measurement is too great?
2 Explain what other factors contribute to the minimum useful parallax
measured by ground-based telescopes.
3 Determine the difference between this ground-based limit and the limit
from space-based telescopes.

Gather and process information


to determine the relative limits
to trigonometric parallax
distance determinations using
recent ground-based and
space-based telescopes.

Activity 22.3: Spectra


Observe spectra of different objects with a spectroscope and determine if they are
emission, absorption or continuous spectra.
Equipment: spectroscope, fluorescent light, incandescent light.
Discussion questions
1 Compare and contrast the different types of spectra.
2 Identify an example of each type of spectra.

Perform a first-hand
investigation to examine a
variety of spectra produced by
discharge tubes, reflected
sunlight, or incandescent
filaments.

403

22

Measuring
the stars

PRACTICAL EXPERIENCES
Activity 22.4: Temperature prediction

Analyse information to predict


the surface temperature of
a star from its intensity/
wavelength graph.

Examine each of the given black body radiation curves of stars and determine their
surface temperature.
Discussion questions
1 State the relationship between peak wavelength and surface temperature
of a star.
2 Outline the method that you followed in order to determine the
temperature of the star.

Activity 22.5: Magnitude and colour index


Perform an investigation to
demonstrate the use of filters
for photometric measurements.

Observe a light through different filters to determine how brightness changes when
only selected colours are viewed.
Equipment: light meter, 2 red filters, 2 blue filters, light source.
Discussion questions
1 Compare the brightness of a red star when viewed through a red filter with
that of the star when viewed through a blue filter.
2 Compare this with viewing a blue star through the filters.
3 Explain how the change in brightness when viewed through the filters
would affect the measured magnitude of each star.

Activity 22.6: Technological advances


Identify data sources, gather,
process and present
information to assess the
impact of improvements in
measurement technologies on
our understanding of celestial
objects.

Research some of the instruments and techniques used in astronomy that have
aided in developing our understanding of celestial objects.
Telescopes
Photomultipliers
CCDs
Optical fibres
Thin telescope mirrors and active optics
Adaptive optics
Optical interferometry
IR array detectors
Radio telescopes
Radio interferometry
X-ray mirrors
Scintillators
Balloon-borne telescopes
Space telescopes
Discussion questions
1 Outline what has been discovered or developed because of the use of the
item you have researched.
2 Discuss what technological developments had to occur in order for this
piece of equipment to be used.

404

Chapter summary

Stellar parallax angle p is the apparent change in


position of a nearby star at distance d, relative to more
distant stars. It is caused by the orbital motion of the
Earth. p (in arc seconds) and d (in parsecs) are related by:
1
p=
d
Fraunhofer lines are produced by the absorption of light
by atoms in the outer layers of the Sun.
A spectroscope or spectrograph disperses light by
wavelength, and enables the composition and physical
characteristics of the source to be studied.
Three types of spectra are observed from hot sources:
A hot, dense gas produces a continuous spectrum.
A hot, low-density gas produces an emission line
spectrum.
A continuous spectrum source viewed through a
cool, low-density gas produces an absorption line
spectrum.
A black body is an idealised hot object that emits light
at all wavelengths with a distribution described by a
black body curve.
The wavelength of peak emission of a black body curve
max is given by Wiens law:
2.9 103
max =
T
The total energy emitted per second per unit area of
the surface of a black body is given by the Stefan
Boltzmann law: P = 5.67 108 T 4
The spectral fingerprint of an element is the same,
whether seen in absorption or emission.
The major spectral classes are O B A F G K M.
The surface temperature of a star controls the
appearance of the spectral lines. Composition
differences between stars are not the main reason why
spectra vary so much.

astrophysics

Differences in luminosity (and hence size) between


stars of the same spectral class are described by the
luminosity classes.
The Doppler effect is caused by motion of the gas
emitting or absorbing the light. At velocities much less
than the speed of light c, the Doppler shift of a
spectral line of wavelength is related to the velocity v
along the line of sight by:
v
=

c
Photometry is the measurement of the brightness of an
astronomical source.
Each stellar magnitude represents a factor of about 2.5
in brightness. A difference of five magnitudes is
equivalent to exactly 100 times in brightness.
Apparent magnitude is a measure of how bright an
object appears in our sky.
Absolute magnitude describes how bright a star would
appear in our sky if moved to a standard distance of
10 pc (32.6 ly).
The overall colour impression from a star is a result of
both the spectrum from the star and the spectral
sensitivity of the detector.
A colour index is the difference in the brightness of
a star in magnitudes, when measured through two
different filters. The best known is the B V colour
index.
Spectroscopic parallax is a distance measurement made
by identifying the type of star from the characteristics of
its light and then comparing its apparent and absolute
magnitudes to derive a distance.

Review questions
Physically Speaking
On the right is a list of topics that have been
discussed throughout this chapter. Create a
visual summary of the concepts in this chapter
by constructing a mind map linking the terms.
Add diagrams where useful.

Parallax

Colour index

Distance

Colour

Spectroscopy

Temperature

Photometry

Absorption
spectra

Emission
spectra

Magnitude

Luminosity

Continuous
spectra

405

22

Measuring
the stars

Reviewing

Solving problems

1 Using a diagram, explain how parallax can be used to

13 Calculate the distance in metres of one kiloparsec

determine the distance to a nearby star.

2 Explain why parallax measurements are so important


in astronomy.

3 Outline the factors that set limits on the angle of


parallax that can be measured from the ground.

(1000 pc).

14 A star has an annual parallax of 0.017 arc seconds.


Calculate the distance to the star in parsecs and
light-years.

15 A star has an annual parallax of 0.029 0.005 arc


seconds. Calculate the range of possible distances
(in parsecs) to this star suggested by this value.

4 Describe the significance of the Hipparcos and Gaia


astrometry missions.

5 List four pieces of information about a star that can

16 The hydrogen lines in the visible part of the spectrum

be obtained using spectroscopy.

6 Compare and contrast the three types of spectra


described in this chapter.

of energy levels of electrons in an atom.

8 Identify the spectral class of stars with:

used to classify stars.

10 Relate temperature to the colour of stars.


11 Recall how the magnitude scale was developed and
hence explain why the scale has negative values.

12 Justify the need for a colour index in describing a star.

440
420
400

Flux

9 Describe the major observational properties that are

the following equation:


1 1
2 2
2 n

a Calculate these wavelengths for n = 3, 4, 5, 6, 7.


b Identify the hydrogen lines in the spectrum of the
F-type star shown in Figure 22.6.1.

7 Explain how emission spectra are created, in terms

a strong hydrogen lines but weak calcium emission


b relatively weak hydrogen lines and few other
strong absorption lines
c relatively weak hydrogen lines and broad
absorption bands.

have wavelengths given by


1
= 1.097 107

380
360
340
320
300
350

400

450

500

550

600

650

Wavelength (nm)

Figure 22.6.1

Spectrum of an F-type star

17 The Sun and the brightest star of the Centauri


system are very similar. Assuming the apparent
magnitude of the Sun in 26.72 and that of
Centauri A is 0.01, calculate how much brighter
the Sun appears in our sky than Centauri A.

18 Star A has an apparent magnitude of +0.3 and a


distance of 5 pc. Star B has an apparent magnitude
of +4 and a distance of 100 pc. Assess which star
has the greater luminosity.

19 A star has a B magnitude of +9.6 and a V magnitude


of +8.0. Deduce the approximate spectral type of
this star.

20 Betelgeuse is a spectral class M Ib supergiant star. It

Re

iew

406

Q uesti o

has an apparent visual magnitude of about 0.45 (but


varies). Using the spectroscopic parallax technique,
calculate its approximate distance from the Earth.

Stellar
companions
and variables

23

Not quite typical


The Sun is often said to be a typical star. It is a not-quite middle-aged
G-type dwarf among billions of similar stars in the disc of the Milky
Way galaxy. It leads a relatively quiet life, with little visible change
apart from a cycle of small sunspots and occasional flares at the
surface. In this it is unlike much larger and brighter G-type stars that
fluctuate significantly in brightness over periods of days or weeks.
The Sun itself will also become variable in its old age. Even today,
the Sun is a little unusual in living alone, without
a close stellar companion. This is not true for the
majority of stars in the galaxy.

binary star, centre of mass,


visual binary, proper motion,
astrometric binary, spectroscope binary,
eclipsing binary, light curve,
cataclysmic variable, X-ray binary,
massluminosity relationship,
variable star, extrinsic variables,
intrinsic variables, non-periodic
variables, periodic variables,
Cepheid variables, RR Lyrae variables,
periodluminosity relationship

23.1 Binary stars


Unlike the Sun, the majority of stars do not live alone in space. Most stars have
one or more stellar companions, or are members of larger clusters of hundreds
or thousands of stars.
The simplest possibility is a binary star system composed of two stars
orbiting their common centre of mass. Each of the orbits is generally elliptical
and this centre of mass point is the balance point that always lies between the
two stars (Figure 23.1.1). The balance is given by the equation:
m1r 1 = m 2r 2
where m1 and m2 are the two masses and r1 and r2 are the distance of each from
the centre of mass. If we use r = r1 + r2 as the distance between the stars, then:

m1r1 = m2(r r1)

= m2r m 2r 1
Rearranging, we get:

r1 =

m2
r
1 + m2

If we make the two stars of equal mass, then m1 = m2 and therefore r1


is half of rthe balance point is half way between the two stars. If star 1 is
more massive than star 2 (m1 > m2), then the balance point is closer to star 1.
407

23

Stellar
companions
and variables
orbit of
star 2

star1
m1

r1

focus 2

focus 2

centre
of mass

r2

orbit of
star 1

star2
m2

Figure 23.1.1

Try this!

Worked example

Make your own binary system

Question

Some of the principles of a binary star system are easily


demonstrated with some balls and a rod.
Cut a hole through each ball so that it can be pushed onto
the thin wooden or metal rod but not slide off easily. Rest the
rod on your finger and slide it along until it is balanced
horizontally. Tie a string to the balance point (the centre of
mass) and suspend your binary system by the string. Gently
rotate the binary by giving one of the balls a push. Try a
different ball and see how the centre of mass changes.

m1

m2

r1

Figure 23.1.2

r2

The centre of mass of the binary system is the balance point.

Explain the importance of


binary stars in determining
stellar masses.

A planet orbiting the Sun is a similar situation to a binary


star, but with a larger ratio between the two masses.
Where is the centre of mass between the Sun and Jupiter?

Solution
mSun = 1.99 1030 kg
mJupiter = 1.90 1027 kg 0.001 mSun
Use the equation:
rSun =

mJupiter
m Sun + mJupiter

Even without calculating exactly, we can see that,


since Jupiter has only ~one-thousandth the mass of the
Sun, rSun is only about one-thousandth of r. Knowing the
size of the Sun (see in2 Physics @ Preliminary section
16.1) leads us to the conclusion that the balance point
between the two is very close to the surface of the Sun.
For smaller planets such as the Earth, the balance point
is within the Sun. So, although we usually say that the
planets orbit the Sun, this isnt quite true.

The apparent size and shape of the orbit is interesting, but the real
importance of binary stars is the possibility of determining their masses. Mass
cannot be determined from spectroscopic information alone. We need to study
the gravitational effect on another object, and a binary star system provides a
laboratory in which to do this.
We can see how this works if we assume the orbit is circular and use equations
from section 2.2. The centripetal force needed to maintain circular motion of
star 1 around the centre of mass is provided by gravitational force between the
two stars:
Fgravity = Fcentripetal
Gm1m2

408

The centre of mass of a binary system is


the balance point between the two stars.

r2

m v2
= 11
r1

astrophysics
2 r1
We can include the period T of the orbit of star 1 by using v1 =
.
T
This gives:
2
Gm2 4 r1
=
r2
T2

Now we can insert the result for r1 derived earlier:


Gm2 4 2
= 2
r2
T

Rearranging this produces:

mm22
m + m rr
(m1 1+ m22)

m1 + m2 =

or

4 2r 3
GT 2

r 3 G (m1 + m2 )
=
T2
4 2

This is a form of Keplers third law (section 2.2; see also in2 Physics @
Preliminary section 13.4) but involves the combined mass of both stars. It also
requires r, the true distance between the stars, with allowance made for any tilt
of the orbit relative to the plane of the sky. This means that the distance to the
system must be known.
If r1, the distance from star 1 to the centre of mass can also be determined,
then the masses m1 and m2 can be calculated individually by using a slight
rearrangement of the earlier result for r1. The problem is that only rarely do we
have enough information to do this.

Worked examples
Question
The Centauri system is 1.338 pc away. The A and B components orbit each other with
a period of 79.92 years and an average distance of 23.7 AU. What is the total mass of
the system?

Solution
Use

m1 + m2 =
m1 + m2 =

4 2r 3
GT 2

but ensure the correct SI units are used.

4 2 3.55 1012 m

binary status
In binary or multiple star
systems we generally call the
brightest star the primary and
assign it the letter A. The
secondary star is B and any
other stars in the system are C,
D etc. Unlike the system in
Figure 23.1.1, a binary system
is often drawn centred on the
primary, which is considered
stationary. For example, in
Figure 23.1.3 the system is
centred on Centauri A, with
the very close companion
Centauri B in orbit around it.
Notice that the orbit is
elliptical, although not the
same ellipse you would draw if
the motions were represented
relative to the centre of mass
(as in Figure 23.1.1). However,
even this elliptical orbit is only
the apparent orbit on the
plane of the sky. The real
elliptical orbit is tilted relative
to the plane of the sky.

(6.672 10 )(2.522 10 s)
11

Try this!

= 4.16 1030 kg
2.1 solar masses

This is usually written with a special symbol for solar mass as 2.1M.
In fact the orbit is highly elliptical, but the result is still valid.

Question
Measurements indicate that Centauri A has an average distance of 11.2 AU from the
systems centre of mass. What is the mass of each star?

409

23

Stellar
companions
and variables

Solution
Use

m2
1.68 1012 m
=
3.55 1012 m 4.16 1030 kg

r1 m2
=

r (m1 + m2 )

Therefore m2 = 1.97 1030 kg 0.99M, leaving m1 = 2.19 1030 kg 1.1M.


See the examples in Chapter 22 for further information on these two stars.

PHYSICS FEATURE

5. Current issues, research and


developments in physics

Centauri

Orbit
Period: 357.00 0.07 days
Semi-major axis: 25.30 0.19 milli-arc seconds
Eccentricity: 0.821 0.003
Inclination to the plane of the sky: 67.4 0.3 degrees

Primary

Mass: m1 = 9.1 0.3M

Absolute V magnitude:

Secondary

M1 = 3.85 0.05

Mass: m2 = 9.1 0.3M

Absolute V magnitude:

M2 = 3.70 0:05

The high accuracy of these results offers an


important starting point for understanding the
structure of the stars themselves.
25

Position (milli-arc seconds)

Centauri has long been known as a visual binary


with the two stars separated by more than 1arc
second. Early observations with an interferometer
suggested that the brighter star was itself a very close
binarymuch too close to be seen as separate
through a conventional telescope.
Figure 23.1.3 shows the orbit of Centauri B.
Its orbit has been determined from high resolution
observations using the Sydney University Stellar
Interferometer (see section 21.5). The stars are very
close, so angles on the sky are measured in milli-arc
seconds (one-thousandth of an arc second).
Combining this with spectroscopic data yields all
the parameters of the system.

20
15

orbit of Centauri B

10
5
0
5
10

Centauri A

10

15

20

25

Position (milli-arc seconds)

Distance 102.3 1.7 pc


Figure 23.1.3

Observations and orbit of Centauri B relative to


Centauri A

Checkpoint 23.1
1
2

410

Draw a diagram of a binary system that shows the two stars, the centre of mass of the system and their elliptical
orbits around the centre of mass.
Determine where the centre of mass is located between Jupiter and its moon Callisto.
MJupiter = 1.901027kg
MCallisto = 1.08 1023kg
Average distance of Callisto from Jupiter = 1883000km

astrophysics

23.2 Doubly different


Although most stars are in binary or multiple systems, they dont all reveal
themselves in the same way. We can classify the types of binaries according to
how they are observed.

Describe binary stars in terms


of the means of their detection:
visual, eclipsing, spectroscopic
and astrometric.

Visual binaries
A visual binary (or multiple system) can be resolved into two (or more)
stars by a telescope under sufficiently good seeing conditions. Like Centauri
(Figure 23.2.1), many of the brighter stars are doubles well known to amateur
astronomers. A few are merely chance alignments of unrelated stars, but most are
genuine binaries.
A simple calculation of angles will reveal that if a star is to be seen as a double
in a telescope with angular resolution about 1 arc second, the stars must be at
least tens of AU apart and the system must be relatively close. Such stars orbit
each other slowly; but over many years the orbital motion may become apparent.
More than 100000 visual double stars are catalogued, but the orbits of only a
few thousand are known.

B
a

Figure 23.2.1 Centauri A and B


1990

1980

Astrometric binaries
Some stars are sufficiently close that their motion through space is apparent as
A few of these stars reveal
motion across the sky, that is, as proper motion.
their binary nature by the wobbling of their paths across the sky (Figure 23.2.2).
The centre of mass of such an astrometric binary follows a straight path, but
the individual stars appear to wobble as they orbit.
Few binaries are discovered in this way, since it usually requires long-term
observations of nearby stars. The Hipparcos astrometry mission (section 22.1)
revealed many new examples, because its very high precision position
measurements revealed the motions of stars much more quickly.

Spectroscope binaries

1970
1960

Sirius B

1950
1940

Sirius A

1930
1920
1910

Most binary systems are too distant to appear as either visual or astrometric
1900
centre of mass
However, the spectrum of an unresolved binary will have light
binaries.
contributed by both stars. As the stars orbit, one will typically have a component
Figure 23.2.2 Sirius is a visual and an
of its motion towards us, while the other is moving away. The light from the two
astrometric binary.
stars will show small blue and red Doppler
A is moving towards the
shifts that change as the stars orbit. Provided
observer: lines blue-shifted
A
B
the stars are close in brightness, the result is
B is moving away from the
a doubled-lined spectroscope binary in
A BA B
A B
observer: lines red-shifted
which each spectral line will appear doubled
by these small shifts in wavelength (Figure
B
A and B are moving across
23.2.3). At other times, the motions of the
the observers line of sight:
stars may be entirely across the line of sight
no Doppler shift
A
A&B
A&B
A&B
and show no Doppler shift and therefore
display only a single set of lines.
A is moving away from the
If the stars are significantly different in
observer: lines red-shifted
B
A
brightness, the spectrum of the brighter star
B is moving towards the
will flood the fainter one and only one set
observer: lines blue-shifted
B AB A
B A
of moving spectral lines may be visible,
Figure 23.2.3 The changing pattern of spectral lines in a spectroscopic binary
forming a single-lined spectroscopic binary.
(very exaggerated)

411

23

Stellar
companions
and variables
To be visible as a spectroscopic binary, the component of the orbital velocity
measured by the Doppler shift must be relatively large. This means that the stars
must be closemuch closer than most visual binaries, so they orbit quickly. Also,
the orbit must be orientated so that the orbital motion has a component in the
line of sight. This is not enough to really understand the system since, generally,
the tilt of the orbit to the line of sight is unknown.
The brighter star of Centauri is a singled-lined spectroscopic binary that has
now been resolved by the SUSI interferometer (see Physics Feature p 410). This
combination of spectroscopic and astrometric data is rare and valuable, since it
allows all the parameters of the system to be derived, in particular the mass.

Eclipsing binaries

PRACTICAL
EXPERIENCES
Activity 23.1

Activity Manual, Page


185

Visual magnitude

orbit of
secondary
1.6
1.8
2.0
2.2
2.4
2.6
2.8
3.0
3.2
3.4
3.6

primary
secondary
primary
eclipse

Figure 23.2.4

Figure 23.2.5

412

secondary
eclipse

2.867321 days

If the stars of a binary system are close together and orientated so that the
orbital plane is close to edge-on, then it may be seen as an eclipsing binary.
In these systems the stars regularly eclipse one another, periodically blocking out
some of the light from the system. To be well enough aligned with our line of
sight means that the stars are usually very close together, with orbital periods of
just a few hours or days. They are therefore usually also spectroscopic binaries.
They may be so close together that the stars are distorted by their mutual
gravitational forces.
The change in brightness is apparent in the light
curve of the systemsa plot of the apparent magnitude
of the system versus time (Figure 23.2.4) that repeats
with every orbit. If the stars are different, the two
eclipses will not be identical. A primary eclipse will
primary
result in a greater loss of light than the secondary
eclipse
eclipse, with the details determined by the tilt of the
orbit, the relative size of the stars, their surface
temperatures and even the structure of their
atmospheres.
The importance of eclipsing binaries lies in the
wealth of information we can glean from observations,
including masses and distances.

Algol ( Persei), a famous example of an eclipsing


binary system, and its corresponding light curve

This artists impression of Circinus X-1 shows a binary


system composed of an ordinary star losing material to
the accretion disc around a neutron star.

Cataclysmic variables and X-ray binaries


Some binaries are so close that they exchange
mass between the stars. When one of the stars is a
compact stellar remnant such as a white dwarf, a
neutron star or a black hole (see section 24.5), it may
accrete gas from its companion. This gas releases
gravitational potential energy, becomes very hot and
emits high-energy radiation.
If the compact remnant is a white dwarf, its light
output may vary dramatically as the system suffers one
or more outbursts as a cataclysmic variable. If the
remnant is a neutron star or black hole, it may be
apparent as a source of X-rays from the infalling gas.
It is then known as an X-ray binary (Figure 23.2.5).

astrophysics

The massluminosity relationship


1 000 000
100 000

Luminosity (solar luminosities)

ne important product of our ability to


determine the masses of some stars in
binary systems is the massluminosity relationship
(Figure 23.2.6). This is a plot of the mass M
of a main sequence star versus its luminosity L,
which can be approximately fitted by the
relationship L M3.5.
It indicates that the luminosity of a star
increases very rapidly with its mass. As
luminosity is based on the consumption of
hydrogen fuel in the core of the star (see in2
Physics @ Preliminary section 15.4), this
indicates that fuel consumption increases much
more rapidly than fuel availability. The result is
that high-mass stars on the main sequence have
shorter lifetimes than low-mass stars.

10 000
1000
100
10
1
0.1

L M 3.5

0.01
0.001
0.1

10

100

Mass (M/M )

Figure 23.2.6

The massluminosity relation for main sequence stars

Checkpoint 23.2
1
2

Recall the different types of binary systems and outline the characteristics of each type.
Explain the significance of the massluminosity relationship.

Classify variable stars as either


intrinsic or extrinsic and
periodic or non-periodic.

23.3 Variable stars


Eclipsing binaries and cataclysmic variables can also classified as
variable stars. They vary in brightness because they are part of a
close binary system. Other stars vary by themselves, whether part
of a binary system or not, as a normal stage in their lives.
More than 30000 variable stars have been catalogued, and
many thousands more are suspected to be variable. In fact, all
stars vary in brightness to some degree. The Sun varies by about
0.1% within an 11-year cycle because of the solar activity cycle
(see in2 Physics @ Preliminary section 16.3). Also, rapid variations
are associated with tiny oscillations in the Sun measured by
helioseismology (see in2 Physics @ Preliminary section 16.2).
These are both just part of the normal behaviour of a Sun-like
star, and the Sun is not regarded as a variable star.
Other stars vary more in brightness and we track these
changes with a light curve. There are many types of variable stars
and the main groupings are shown in Figure 23.3.1.

GROUP

TYPE

CLASS
Cepheids
PULSATING
STARS

INTRINSIC
VARIABLE

Type II
W Virginis

RV Tauri
Long-period
variables

ERUPTIVE
(cataclysmic)
STARS

VARIABLE
STARS

RR Lyrae

Type I
Classical

Mira type

Semiregular
Supernovae
Novae
Dwarf novae
Symbiotic stars
Flare stars
R Coronae Borealis
T Tauri stars

ECLIPSING
BINARIES
EXTRINSIC
VARIABLE
ROTATING
VARIABLES

Figure 23.3.1

Different types of variable star systems


413

23

Stellar
companions
and variables
Effective temperature (K)
30 000 10 000

7 000

6 000

4 000

10

10

Type I

4
Absolute magnitude (Mv)

RV Tauri
Semiregular
stars

Classical Cepheids

Long-period
variables

Type II

RR Lyrae

Ma

W Virginis

10
10

equ

enc

T Tauri stars

Instability

strip

10

Miras

in s

10

10

Luminosity compared to Sun

10

10

12

Flare stars

14

10

10
O5 B0

0.5

Figure 23.3.2

A0

F0
G0
Spectral class

K0

0.0

+0.3
+0.6
Colour index

+0.8

M0
+1.4 +2.0

The location of some types of variable stars


on the HR diagram

It divides variables stars into extrinsic variables and


intrinsic variables. Extrinsic variables are perhaps less
interesting, at least as variable stars, because their variation
is due to a process external to the body of the star itself.
Eclipsing binaries are the best example, although the group
also includes stars that vary because of their rotation
(e.g. the effect of spots on their surfaces).
Intrinsic variables vary because of physical
changes in the star or the stellar system. The HR diagram
(see in2 Physics @ Preliminary section 15.3) summarises the
properties of stars, and therefore different types of intrinsic
variables occupy specific regions of the HR diagram
(Figure 23.3.2).
Intrinsic variables can be split into non-periodic
variables and periodic variables. As the names suggests,
the difference is whether their variation repeats at
reasonably regular intervals.

Non-periodic variables
The non-periodic variables cover a wide range of
different types of stars that are physically very different,
as illustrated by the systems described in Table 23.3.1.

Table 23.3.1 Major classes of non-periodic variables


Variable type

Brightness change

Physical description

Stars in binary systems


Type I Supernovae
Novae

Dwarf novae

Symbiotic stars

~20 magnitudes within hours, then


gradually fades over weeks
Between 7 and 16 magnitudes,
usually within a few days, then fades
overs years to its initial brightness;
some have been seen to brighten again
Between 2 and 5 magnitudes,
repeating semi-regularly; different
types behave differently
Vary semi-regularly over a range of
about 3 magnitudes

Accretion of gas onto a white dwarf from its companion,


leading to a runaway nuclear explosion
A close binary composed of a Sun-like star leaking gas
onto a white dwarf that eventually will accumulate
enough material to generate a surface nuclear explosion
A close binary composed of a Sun-like star leaking gas
onto a white dwarf; instability in the accretion of gas
appears to produce the outbursts
A close binary composed of a red giant and a white
dwarf; outbursts from the red giant fall onto the white
dwarf

Single stars (may be in a binary system)


Type II supernovae
Flare stars

R Coronae Borealis

T Tauri

414

~20 magnitudes within hours, then


gradually fade over weeks
Typically 1 or 2 magnitudes and fade
within hours
Usually at maximum brightness but
fade at irregular intervals by up to 9
magnitudes, returning to normal over
months
Vary irregularly in brightness

Core collapse of a massive star blows off its outer layers


Solar-like flares on the surface of faint red dwarfs
contribute dramatically to the overall visible light
output of the star
Carbon-rich dust clouds obscure the surface of the
yellow supergiant star

Very young stars with an accretion disc, still


approaching the main sequence

astrophysics

Periodic variables
Periodic variables show regular or semi-regular changes in brightness,
with periods ranging from hours to hundreds of days. The major types are
shown in Table 23.3.2. Their brightness changes as the stars pulsate in size,
surface temperature and colour.
The pulsation arises from a slight instability in the balance between the
inward pull of gravity and the outward pressure of the gas and radiation
(see in2 Physics @ Preliminary section 15.4). This instability only occurs in the
outer layers of the stars and does not affect the energy production in the core.
Conditions in pulsating stars are just right to allow the pulsations to continue
instead of dying away, as they would in most stars. These conditions are found
in stars in the instability strip on the HR diagram, with another similar zone
of long-period variables among the red giants (Figure 23.3.2).
Table 23.3.2 Major classes of pulsating variables
Variable type

Brightness change

Physical description

Cepheid

Between 0.5 and 2 magnitudes with


periods from 1 to 70 days
Up to 2 magnitudes with periods of
less than 1day
Alternating deep and shallow minima
with periods from 20 to 100 days
Between 2 and 10 magnitudes with
periods from 80 to 1000 days
Up to 2 magnitudes with periods from
80 to 1000 days, but with
irregularities

Luminous yellow
supergiants
Old giants stars with
MV +0.6
Yellow supergiants

RR Lyrae
RV Tauri
Mira
Semi-regular

Red giants and supergiants


Red giants and supergiants

Naming of stars

tar names and designations


follow many different systems for
different types of stars. There are
also many historical names and
catalogues with designations still
commonly in use. For example, the
brightest star in the sky has many
designations such as Sirius, Canis
Majoris, 9 Canis Majoris, HD 48915
and Hipparcos 32349 to name a few.
Variable stars have one of the oddest
systems. The first variable discovered
in a constellation is called R, e.g.
R Canis Majoris. Next comes, S, T, U
Z. Then what to do? We use RR,
RS, RT all the way to ZZ, and then
AA, AB to QZ. Finally, if we have
more than 334 variables in a
constellation, we start a sensible
system with V335, V336 etc!
Different classes of variable stars
often take their name from the first
one identified, for example
RCoronae Borealis stars.

Checkpoint 23.3
1
2

Distinguish between extrinsic and intrinsic variable stars.


Explain how the pulsation of periodic variables can tell us about
the structure of those stars.

23.4 Cepheid variables


Of all the variable stars, the Cepheid variables are the most important, followed
by their cousins, the RR Lyrae variables. Both serve as standard candles in
distance measurements, because their absolute magnitudes can be estimated from
their pulsation properties.
The Cepheids are named for the prototype of the class, the northern nakedeye star (delta) Cephei. They vary regularly and have a characteristic light
curve. As they oscillate in size they change in surface temperature and therefore
spectral class (Figure 23.4.1).

415

23

Stellar
companions
and variables

Brightness

PERIOD

Time

Figure 23.4.1

Explain the importance of the


periodluminosity relationship
for determining the distance
of cepheids

In the early 20th century, Henrietta Leavitt (18681921) was studying


Cepheids in the Magellanic Clouds, two satellite galaxies to the Milky Way. The
Magellanic Clouds are relatively small compared with their distance, so all Cepheids
in each Cloud are at essentially the same distance from us. The different apparent
magnitudes of these stars therefore reflect true differences in luminosity (or absolute
With this simplification, Leavitt realised that Cepheids with
magnitude).
longer periods were more luminous than Cepheids with shorter periods. This is the
periodluminosity relationship for Cepheids (Figure 23.4.2).
It was discovered later that there are actually two classes of Cepheids. Type I
(or classical) Cepheids are massive young stars crossing the instability strip as they
evolve. Type II Cepheids (or W Virginis stars) are much older stars also crossing
the instability strip.
The power of the periodluminosity relationship lies in the fact that period can
be determined by following the brightness variation of any Cepheid we can see.
Then its absolute magnitude can be estimated from the periodluminosity curve
(Figure 23.4.2) and the distance determined by comparing the apparent and absolute
magnitudes (see section 22.4). However, reality is always a bit more difficult; for
example, interstellar dust can make the stars appear dimmer than they should.
Because they are supergiant stars, Cepheids can be identified in some nearby
galaxies. As a result, Cepheid distances are a fundamental stepping stone in
measuring much larger distances in the universe.
RR Lyrae variables are not as bright as Cepheids, but they are simpler since
they all appear to have about the same luminosity (Figure 23.4.2). Once a star is
recognised as an RR Lyrae variable, its distance can be determined by comparing
its apparent and absolute magnitudes.

Type I (Classical)
Cepheids

104

Luminosity (L )

Variations in the brightness, size and colour


of a Cepheid variable during its pulsation

Type II (W Virginis)
Cepheids

103

102

RR Lyrae
1
0.5

Figure 23.4.2
1

10

Period (days)

416

30 50

100

The periodluminosity relations for Cepheid and RR Lyrae


variable stars. The green arrows indicate the luminosity relative
to the Sun of a type I Cepheid that has a period of 10 days.

astrophysics
Worked examples
Question
(zeta) Geminorum is a Type I Cepheid variable star in the constellation of Gemini. It varies
in brightness between 3.7 and 4.2 magnitudes every 10.2 days. Using the periodluminosity
relation, estimate its absolute magnitude.

Solution
Using the arrows on Figure 23.4.2, a period of 10.2 days indicates
a luminosity of approximately 3500L.
This can be converted to a magnitude difference using:
I
mB m A = 2.50 log10 A
IB
but working with absolute magnitudes. MV for the Sun is +4.85, so:
4.85 MA = 2.50log10 (3500)
and thus MA is 4.0.

Question
Using Geminorums average apparent magnitude, estimate its distance.

Solution
If we assume the average apparent magnitude is +3.95 then, using the relation between
absolute and apparent magnitude:
d
3.95 ( 4.0) = 5 log10
10
d
7.95 5
Then = 10
and d 390 pc
10
This is close to the accepted value.

Checkpoint 23.4
1
2
3

Recall why Cepheids and RR Lyrae variables are so important in astrophysics.


Outline how Leavitt discovered the periodluminosity relationship.
Describe the characteristics of the two types of Cepheids.

417

23

Stellar
companions
and variables

PRACTICAL EXPERIENCES

CHAPTER 23

This is a starting point to get you thinking about the mandatory practical
experiences outlined in the syllabus. For detailed instructions and advice, use
in2 Physics @ HSC Activity Manual.

Activity 23.1: Eclipsing binaries


Go to the website given to observe the light curves of eclipsing binary stars.
Record your observations and use them to interpret any light curves.
Discussion questions
1 Identify the basic features of the light curve of an eclipsing binary.
2 Sketch a light curve and explain the positions of the stars at specific points
of the curve.

Perform an investigation to
model the light curves of
eclipsing binaries using
computer simulation.

Chapter summary




418

A binary star system is composed of two stars orbiting


their common centre of mass.
The major importance of binary stars is their use in
determining the masses of the stars.
4 2r 3
Keplers third law in the form m1 + m2 =
GT 2
involves the combined mass of both stars. With more
information, the individual masses can be determined.
A visual binary (or multiple system) can be resolved into
two (or more) stars by a telescope under sufficiently
good seeing conditions.
An astrometric binary reveals its binary nature by the
wobbling of its path across the sky.
In a doubled-lined spectroscope binary each spectral line
will periodically appear doubled by the Doppler shift of
the individual stars.
In a single-lined spectroscope binary the spectrum of
the brighter star will flood the spectrum of the fainter
one and only one set of moving spectral lines will be
visible.
If the stars of a binary system are close together and
orientated so that the orbital plane is close to edge-on, it
may be seen as an eclipsing binary in which the stars
regularly eclipse one another.

The light curve of an eclipsing binary or other variable


star is a plot of the apparent magnitude of the system
versus time.
Some binaries are close enough to exchange mass
between the stars, and become apparent as cataclysmic
variables or X-ray binaries.
The massluminosity relationship is a plot of the mass
M of a main sequence star versus its luminosity L,
which can be approximately fitted by the relationship
L M3.5.
Variable star systems vary in brightness.
Variation in extrinsic variables is due to a process
external to the body of the star itself.
Variation in intrinsic variables is due to physical changes
in the star or the stellar system.
Non-periodic variables cover a wide range of different
types of stars that are physically very different.
Periodic variables show regular or semi-regular changes
in brightness due to pulsation.
Cepheid variables and RR Lyrae variables serve as
standard candles in distance measurements.
Cepheid variables obey a periodluminosity relationship
that can be used to determine their distance.
RR Lyrae variables all appear to be about the same
luminosity, which can be used to determine their
distance.

Review questions

astrophysics

Physically Speaking
Complete the passage below by filling in the missing words from this list:
brightness, companion, eclipse,
eclipsing binaries, eruption,
extrinsic, intrinsic, magnitudes,
periods, pulsating variables,
pulsation, rotation, variability

Variable stars are stars that change ______________. This change can range from
0.001 to as much as 20 ______________ over ______________ of a fraction of a
second to years. There are a number of reasons why the brightness of variable
stars will change. ______________ swell and shrink due to internal forces, while a
star in an ______________ will dim when it is eclipsed by a faint ______________ and
then brighten when the occulting star moves out of the way.
Variable stars are classified as either ______________, when variability is caused
by physical changes such as ______________ or ______________ in the star or stellar
system, or ______________ when ______________ is caused by the ______________ of
one star by another or by the effects of stellar ______________.

Reviewing
1 Define what is meant by the term binary star.
2 Discuss why binary star systems are important in astronomy.
3 Draw a typical light curve that would result from an eclipsing binary
system that consists of two identical stars.

4 Outline the characteristics that can be determined if two stars form a


spectroscopic binary.

5 Explain why relatively few binary systems are discovered as astrometric


binaries.

6 On the basis of the other types of binary systems described, propose an


idea of why you might describe a system as an interferometric binary
system.

7 Define what is meant by the term variable star.


8 List examples of variable star types that would be called extrinsic or
intrinsic.

9 List the properties of a star that vary in a pulsating variable star.


10 Construct a block diagram indicating the relationship between all the types
of binary stars and variable stars described in this chapter.

11 Explain the properties that distinguish a Cepheid variable star.


12 Outline the importance of the periodic change in luminosity of Cepheid
variables.

419

23

Stellar
companions
and variables

Solving problems
13 If a binary system consists of a 0.5M red dwarf star and a 3M red giant
star separated by 20 AU, calculate the location of the centre of mass.

Solve problems and analyse


information by applying:

4 r
GT 2

m1 + m2 =

2 3

14 Sketch the orbits around the centre of mass of the 0.5M and 3M stars
in the previous question.

15

a Sirius, the brightest star in the sky is a binary composed of a main


sequence star and a white dwarf. It is 2.631 pc away. The A and B
components orbit each other with a period of 50.1 years and an
average distance of 19.8 AU. Calculate the total mass of the system.
b Measurements indicate that Sirius A has an average distance of 6.5AU
from the systems centre of mass. Calculate the mass of each star.

16 V Puppis is a very closely spaced eclipsing binary in the southern


constellation of Puppis. Its light curve is shown in Figure 23.5.1. Estimate
the period of the orbit and the amplitude of the eclipses, in magnitudes.

Apparent magnitude

4.2
4.4
4.6
4.8
5.0
0.3

0.0

0.3

0.6

0.9

1.2

1.5

1.8

2.1

2.4

Time (days)

Figure 23.5.1

Light curve of V Puppis

17 Doradus is a Type I Cepheid variable in the southern constellation of


Dorado. It varies in magnitude from 3.46 to 4.08 in 9.94 days. Calculate
its distance.

18 The star RR Lyrae is spectral class F5 with an average apparent magnitude

Re

iew

420

Q uesti o

of 7.1. Calculate its distance.

astrophysics

PHYSICS FOCUS
Variables and supergiants
by Jonathon Nally
For most of the 20th century, professionals and
amateurs were separated by a wide gap in technology.
Only the professionals could afford the large
telescopes and complex support gear required for
modern astronomy and astrophysics. That gap has now
closed to some degree. Amateurs now have affordable
access to sophisticated telescopes, electronic
detectors, software, and all the other paraphernalia
needed to contribute to many areas of astronomy.
But its great to see that theres still a place for the
good old human eye. And this is nowhere more evident
than in the field of variable star research.
Some variable stars change brightness over a period
of days, weeks, months or even years. Many change
their brightness with perfect reliability, while others are
totally unpredictable. This means that constant, longterm monitoring is needed, which is the sort of thing
that professional astronomers really are unable to do
but its a job perfectly suited to amateurs.
Every night, all over the world, scores of dedicated
variable star observers are outside, making
observations and brightness readings of hundreds of
variable stars. They tabulate their data and send them
in to one of several umbrella organisations scattered
around the world. The data is then made available to
the professionals who, it cannot be stressed enough,
rely so heavily on this work by the amateurs.
Usually, it takes many years of patient monitoring
for enough data to be built up to make a difference in
research programs. But sometimes things can happen
much, much faster.
On the night of 6 January 2002, Aussie amateur
astronomer Nick Brown spotted something unusual.
A certain star was much brighter than it should have
beenV838 Monocerotis had gone into outburst.
It was a nova, and Nick was the first person to spot it.
He quickly reported it, and it came to the attention
of a bunch of professional astronomers who were
interested in just this particular kind of star. For the
past year, this team has been studying V838, and

20 May 2002

2 September 2002

28 October 2002

17 December 2002

Figure 23.5.2

Hubble images of the light echo surrounding


V838 Monocerotis

have found it to be very unusual indeed. It had


expanded into a supergiant star over a period of just
a few months, brightening three times before fading
dramatically. At first they thought the fade was caused
by obscuring dust, but now it turns out that the star
has actually cooled down. In fact, it has become the
coolest supergiant star ever found and all because of
a single observation made by a dedicated amateur
astronomer in Australia.
Source: Transcript from ABC Science Show, Radio National, 10 May 2003

1 Outline the advantages of an electronic detector


over the human eye for variable star observation.
2 Describe why it takes years of observations of a
variable star to build a useful data set. Is this true
of all types of variables?

Extension
3 V838 Monocerotis is now well known because of
images taken by the Hubble Space Telescope
(Figure 23.5.2) showing a light echo from the
outburst. Define the term light echo.
4 Recount what has happened to V838 Monocerotis
since the time of the radio broadcast in 2003.
5 Propose some other fields of astronomy in which
you think amateur astronomers can make valuable
contributions.

421

24

Birth, life
and death
Stellar evolution: How do we know?

interstellar medium, nebula,


emission nebulae, dark nebulae,
reflection nebulae, giant molecular
clouds, protostar, zero-age main
sequence, accretion disc, jets, proton
proton chain, carbonnitrogenoxygen
cycle, red giant, triple alpha process,
helium flash, horizontal branch (HB),
asymptotic giant branch, planetary
nebula, supernova, supernova remnant,
neutron star, black hole, pulsar

Astronomers talk about the evolution of a star, but not quite in the
sense a biologist uses the word. Astronomers mean that an individual
star transforms itself during its lifetime as it consumes its nuclear fuel.
However, stars have also evolved over generations of birth and death.
Stars born today are somewhat different from stars born closer to the
time of the Big Bang.
How do we know about the evolution of the stars? Its a little like
biology in that we almost never see evolution happening in our
lifetimes, but the evidence is there to be found. Some
of the ancient stars are still around to see today.

24.1 The ISM


Within the Milky Way galaxy, the space between the stars is not quite
empty. It is filled by a patchy medium of gas and dust called the interstellar
medium (ISM). The medium is visible as a nebula (meaning cloud), when it
interacts with starlight. In visible light images such as Figure 24.1.1, we see three
types of nebulae:
bright emission nebulae in which the gas is energised by hot young stars to
emit an emission spectrum (see section 22.2)
dark nebulae in which the dust scatters starlight, reddening or completely
blocking our view of background stars
reflection nebulae in which we see the light scattered by the dust, especially
at blue wavelengths.
The importance of gas and dust in our galaxy is apparent by looking at its
distribution in other similar galaxies (Figure 24.1.2), although it makes up only
about 1% of the mass of all the stars.
Figure 24.1.1
422

The Trifid nebula, composed of an emission nebula (pink) and


a reflection nebula (blue), crossed by dark dust lanes

astrophysics

The gas is about 70% hydrogen and 28% helium (if measuring
mass), with traces of other elements and molecules. This is spread
through several different components of the ISM with very different
properties (Table 24.1). Of particular interest are the cold giant
molecular clouds in which the gas is most dense, and simple molecules
can form. Individual molecular clouds have masses up to millions of
solar masses. They make up only about 1% of the volume of the ISM
but contain 90% of the mass and are the sites of star formation.
Where massive stars have formed in a giant molecular cloud, their
intense ultraviolet (UV) radiation eats into the cloud and creates an
emission nebula. Elsewhere the embedded dust may be revealed by dark
nebulae and reflection nebulae.
The dust is just a fraction of a millimetre in size and represents
about 1% of the mass of the gas. Its characteristics vary with location,
but it seems to consist of silicate or carbon grains, sometimes with a
coating of various ices. At least some of it is formed in the cool outer
atmospheres of red supergiant stars and blown outwards by the stars
stellar wind.
The ISM is part of a giant recycling system that includes the
stars (Figure 24.1.3). Gas from the ISM forms stars, is processed by
nuclear reactions inside the stars, and some is returned to the ISM when
the largest stars age and die.
Table 24.1.1 Major components of the ISM, with the air around you included for comparison
Component

Temperature (K)

Atmosphere at Earths surface


Molecular clouds
Neutral atomic clouds
Partly ionised intercloud medium
Highly ionised coronal gas

300
2050
50150
103104
105106

Figure 24.1.2

neutral atom
clouds
ionised clouds

molecular clouds

ISM

Density (atoms per m3)


25

~10
1091011
106109
~104
2
10 103

stellar outflows
and explosions

Checkpoint 24.1
1
2
3

IR image of the galaxy M81, with the spiral


arms traced by emission from dust

Recall the three types of nebulae and their characteristics.


State the main components of ISM.
Outline the significance of giant molecular clouds.

star formation

nuclear fusion
in stars

Figure 24.1.3

The galactic recycling system

24.2 Star birth


The first step in the recycling scheme is star formation. If a part of a giant
molecular cloud is sufficiently cool and dense, it can be pushed into gravitational
collapse. Likely triggers include the shockwave from the explosion of a nearby
star, or the density waves that sweep through the disc of the galaxy.
Once the trigger has boosted the density sufficiently, gravity takes over and
that piece of the cloud starts to collapse because of the gravitational forces
between its own particles. Initially the gas falls freely inward and the density

Describe the processes


involved in stellar formation.

423

24

Birth, life
and death

increases quickly at one or more centres to form cores


with more slowly collapsing envelopes.
10
As each core collapses, the infalling gas releases
5
10
8
gravitational potential energy (see in2 Physics @
4
6
10
15M .
Preliminary section 4.1). Some of this energy is

4
3
converted to kinetic energy and heats the gas while the
9M
104 years 10

2
rest is radiated as infra-red light. As each core gets
5M
105 years
2

10
0
hotter, the gas pressure increases, slowing the collapse
3M

Ma
2
in s
106 years 10
of the core while the surrounding gas continues to fall
equ
enc
inwards. The collapsing fragment of the cloud is now a
e
4
1
protostar, probably one of many, perhaps destined to
1M
6

1
10
form part of a binary system or even an entire cluster
8
ZAMS
7
2
of stars. It is a luminous source of infra-red light
10
10
10
0.5M
years

tracing a path on the HR diagram (Figure 24.2.1), but


3
12
10
buried deep in the larger cloud.
14
4
10
As the collapse continues, the core
O5 B0
A0
F0
G0
K0
M0
temperature
rises, eventually passing 10 million K and
Spectral class
initiating
the
nuclear fusion of hydrogen into helium
0.5
0.0
+0.3
+0.6
+0.8
+1.4 +2.0
Colour index
(see in2Physics@Preliminary section 15.4)the star
turns on! The increased energy production slows and
Figure 24.2.1 Evolutionary tracks for protostars of various masses
eventually stops the collapse of the core. Lower mass
stars like the Sun go through a highly active phase as a
T Tauri variable star, but eventually settle into a stable
equilibrium between the inward pull of gravity and the outward force of gas and radiation
pressure. The star is born on the zero-age main sequence (ZAMS) of the HR diagram, a
line along the lower edge of the broader main sequence band on Figure 24.2.1.
The time taken to reach this stage is a function of the mass of the collapsing fragment
of cloud. More massive protostars heat up more quickly and begin nuclear reactions
sooner. Very low mass stars will reach the ZAMS as red dwarfs long after their higher
mass counterparts have lived their entire lives and died! Objects with mass below about
0.08M form brown dwarfs and will never fuse hydrogen into helium.
Just how many stars form in each mass range is uncertain and undoubtedly varies.
However, for every one 10M star, there are typically about 300 stars like the Sun formed
and many more M-type red dwarfs.
Effective temperature (K)
6 000

4 000

Luminosity compared to Sun

7 000

Absolute magnitude (Mv)

30 000 10 000

Discs and jets

urrounding the protostar is an accretion disc that


forms naturally as the protostar collapses and
spins faster. Infalling material accumulates in the
disc, but it is accompanied by jets of outflowing
materiala situation often seen in astronomy. When
the stars turns on, the system is swept clean by
radiation from the star and may reveal a planetary
system that formed within the disc.

jet
protostar
outflow

inflow

accretion disc

jet

Figure 24.2.2
424

The accretion disc and jet surrounding a protostar

astrophysics

Checkpoint 24.2
1
2
3
4

Outline some of the events that trigger the birth of stars.


Describe a protostar.
Describe the ZAMS.
Recall the forces that are in balance when a star is in equilibrium.

24.3 Stars in the prime of life


A place on the ZAMS is a starting point for the main part of any stars life.
That place, how long the star took to get there and everything that happens
afterwards are determined almost entirely by the mass of the star. This arises
because mass determines the gravitational forces within the star and therefore the
pressure that must resist those forces if the star is to be stable. The pressure is
generated by the high temperatures inside a star as a result of the energy produced
by the nuclear reactions in its core. So the stability of the star depends on it having
enough nuclear fuel available and processing it fast enough.
Stars change very slowly as they age, so they spend most of their lives on the
main sequence, not far from the ZAMS, and that is where most stars are found in
the HR diagram. How long they can maintain this peaceful existence depends on
how much hydrogen fuel they start with and how quickly they consume it.
The massluminosity relationship for main sequence stars (section 23.2) tells
Despite their larger
us that high-mass stars spend their fuel reserves recklessly.
supply, more massive stars have shorter lives on the main sequence before their fuel
runs low and they need to evolve (Table 24.3.1). In comparison, the Sun is a G2
star with an expected main sequence lifetime of about 11 billion years.
Table 24.3.1 Properties of main sequence stars relative to the Sun
Spectral
class

Mass
(m)

O5
B0
A0
F0
G0
K0
M0

40
15
3.5
1.7
1.1
0.8
0.5

Luminosity
(l)
400000
13000
80
6.4
1.4
0.46
0.08

Surface temperature
(K)

Radius
(r)

Time on main sequence


(million years)

40000
28000
10000
7500
6000
5000
3500

13
4.9
3.0
1.5
1.1
0.9
0.8

1.0
11
440
3000
8000
17000
56000

To understand why the high-mass stars are so rash with their fuel, compared
to their more frugal lower mass cousins, we need to consider the two processes at
work in main sequence stars converting hydrogen into helium: the proton
proton chain and the carbonnitrogenoxygen cycle (see in2Physics
In both cases the net reaction is a combination
@Preliminary section 15.4).
of four hydrogen nuclei (protons) into one helium nucleus with the release of
energy and some other light particles. Energy is available because the mass of the
four hydrogen nuclei is more than that of one helium nucleus. The lost mass is
converted to energy according to Einsteins famous equation E = mc2.

Describe the types of nuclear


reactions involved in Main
Sequence and post-Main
Sequence stars.
Discuss the synthesis of
elements in stars by fusion.

425

24

Birth, life
and death

1H
1
2H
1

1
1H

1
1H

3
2He

1H
1
1
1H

4He
2

3
2He

1H
1

1
1H

2H
1

proton
neutron
positron
neutrino
gamma ray

1
1H

Figure 24.3.1

In cooler stars like the Sun, the dominant


reaction is the protonproton chain (Figure
24.3.1). Several pathways are possible but, for
core temperatures from 10 to 14 million K,
the main reaction sequence is:

A series of reactions must occur to form


helium in the protonproton chain.

12
6 C

1H
1

4
2 He

12

C re

cyc

led

12
6 C

13
7 N

1H
1

15
7 N

13
6 C

positron
neutrino
gamma ray

15
8O
1H
1

Figure 24.3.2

14
7 N
1H
1

3
2

H + 11 H 21 H + e + +

2
1

H + 11 H 23 He +

He + 23 He 42 He + 2 11H

The reaction emits e+ (positrons, i.e.


positive electrons), (neutrinoselusive
particles with very small mass) and -rays.
At temperatures below 10 million K, this
reaction barely works. At higher temperatures
the energy production rises quickly
(proportional to ~T4).
In more massive main sequence stars, the
core temperature is even higher and the
carbonnitrogenoxygen (CNO) cycle takes
over as the dominant reaction. This process
also produces helium from hydrogen but uses
carbon nuclei as a catalyst, as illustrated in
Figure 24.3.2.
12
6

C + 11 H 137 N +
13
7

13
6
14
7

A series of reactions starting with carbon make up


the carbonnitrogenoxygen cycle.

1
1

C + 11 H 147 N +

N + 11 He 158 O +
15
8

15
7

N 136 C + e + +

O 157 N + e + +

N + 11 He 126 C + 42 He

CNO energy production is negligible below about 13 million K, but


dominates protonproton energy production by 20 million K. The energy
production rises dramatically with temperature (proportional to ~T20!), with
some other reaction sequences also possible.
The enormous increase in reaction rate produced by small changes in
temperature, especially in the CNO cycle, explains why high-mass stars expend
the reserves of hydrogen in their cores so quickly. It also explains the differences in
later evolution of low- and high-mass stars. These different evolutionary paths are
illustrated in Figure 24.3.3 and described in the following sections.
As it ages, a main sequence star builds up helium in its core. This dampens
the reaction rate, leading to a loss of pressure in the core, which responds by
contracting a little. This releases gravitational potential energy that raises the
temperature and therefore boosts the reaction rate. As the reaction is so sensitive
to temperature, the overall effect is to increase the energy output and inflate the
whole star. This moves the star on the HR diagram and gives the main sequence
some width (Figure 24.3.4).
426

astrophysics

ionised clouds

neutral atom clouds


molecular
clouds

ISM

STAR FORMATION
High mass 8M M

Main sequence star


H fusion in core via
CNO cycle

Major mass loss


via stellar wind

Red supergiant
He fusion in core via triple- process,
H fusion in shell

Major mass loss


via stellar wind

AGB red supergiant


C fusion in core, He, H fusion
in shells, finally multiple shells
of heavy element fusion

Dramatic mass loss,


including heavy elements
via supernova remnant

Type II supernova
produces heaviest elements

Black hole M 3M
No fusion reactions

Small mass loss


via stellar wind

Larger mass loss


via stellar wind

Red giant
He fusion in core via triple- process,
H fusion in shell

Significant mass loss


via planetary nebula shell

Planetary nebula
White dwarf
M 1.4M
No fusion reactions, slowly cooling

Figure 24.3.3

A summary of the evolution of single stars of various masses.


Gas is returned to the ISM as raw material for future stars.

Effective temperature (K)


30 000 10 000

7 000

6 000

4 000

10
2 million years
initial 30M .

5
10

Supergiants (I)

4
10

30 million years
initial 10M .

4
Absolute magnitude (Mv)

he Sun is 4.6 billion years into its main


sequence lifetime. It is now about 5% bigger
and brighter and 200K warmer at the surface
than it was when it landed on the ZAMS. Over the
remaining 6 billion years of its main sequence life,
the Sun will gradually double in luminosity and
get 25% larger. This is a small change on the HR
diagram (Figure 24.3.4), but one that will
dramatically alter the Earth, eventually boiling
away its oceans.

Brown dwarf
No ongoing fusion

AGB red giant


Possible C fusion in core,
He fusion in shell,
H fusion in shell

Increasing mass loss


via stellar wind

Neutron star
1.4M M 3M
reactions

No fusion

Climate prediction:
getting hotter!

Main sequence star


H fusion in core via
pp chain and CNO cycle

3
10

180 million years

2 initial 5M

initial 3M

2
10

640 million years

Giants (II, III)

Ma

in s

Subgiants (IV)
enc
e (V 11 billion years
)
present sun

initial 1M

equ

1
4.6 billion years
56 billion
1
10
years
ZAMS

8
10

White
dwarfs

12

initial 0.5M

(VII)

14

Figure 24.3.4

The motion of a star


on the HR diagram
during its life on the
main sequence

10

Luminosity compared to Sun

Significant mass loss


via stellar wind

Very low mass


M 0.08M

Low to medium mass


0.08M M 8M

2
10
3
10
4
10

O5 B0
0.5

A0

F0
G0
Spectral class

K0

0.0

+0.3
+0.6
Colour index

+0.8

M0
+1.4 +2.0

Checkpoint 24.3
1
2
3

Explain why the lifetime on the main sequence is different for stars of different mass.
Write the net nuclear reaction equations that summarise the protonproton chain and CNO cycle.
If the CNO cycle produces negligible energy at 13 million K, explain why it can dominate energy production at
20 million K.
427

24

Birth, life
and death

24.4 Where to for the Sun?


Outline the key stages in a
stars life in terms of the
physical processes involved.

Eventually a main sequence star runs very low on hydrogen fuel in its core. For
a star like the Sun, there is no mixing of new gas into the core and no increase in
the reaction rate can counteract the lack of fuel. The core collapse accelerates,
raising the temperature as gravitational potential energy is released, and igniting
hydrogen fusion in a shell around the helium core.
Computer modelling shows that the outer layers of the star respond by
expanding outwards and cooling. The plotted position on the HR diagram rapidly
moves to the right across the sub-giant range (Figure 24.4.1). The luminosity of the
star then begins to grow and the outer layers inflate further as the star climbs the
red giant branch. About 1.3billion years after leaving the main sequence, the G
class V main sequence Sun becomes a K or M class III giant, about 200 times its
current size.
The star now has a dense core that is perhaps one-third its original size, but the
grossly extended outer layers reach out to nearly envelope the orbit of the Earth.
These outer layers are of very low density and only weakly held. As a main sequence
star, the Sun looses mass at a rate of ~1014M per year as solar wind (see in2 Physics
@ Preliminary section 16.4). As a red giant, the mass loss will be perhaps 107 times
larger, and it will simply blow away a significant fraction of its mass.
In the core of the red giant the temperature eventually rises to about
100 million K and the fusion of helium into carbon begins via the triple alpha
process:
4
He + 42 He 48 Be +
2
8
4

Be + 42 He 126 C +

Adding another 42 He can produce 168 O.


For stars of less than about 2.6M, the onset of helium fusion occurs as the
helium flash, since conditions in the cores of these stars cause the entire core to
begin helium fusion almost at once. Slightly more massive stars begin helium
fusion a little more sedately, because the physical state of the core is a little
different.
We believe that less massive stars that begin life as red dwarfs probably never
go through a red giant phase. They never attain the temperatures needed to fuse
helium. Its hard to be sure, as their time on the main sequence is much greater
than the current age of the universe and so there are no evolved red dwarfs to
look at.
In stars that become red giants, the new energy supply from helium fusion
causes a reduction in the size of the star and the surface temperature begins to rise
again. Stars fusing helium in their cores, with hydrogen fusion in a surrounding
shell, appear as smaller yellow G- and K-type giants.
The Sun may spend 100 million years in this horizontal branch (HB) phase,
but its evolution is accelerating. Helium fusion occurs much faster than the
hydrogen reactions and, all too quickly, the helium in the core is consumed and
The outer layers expand
the core must again begin to collapse and heat.
again and the star moves up the asymptotic giant branch (AGB) in the HR
diagram. The star now has a core of carbon and oxygen surrounded by shells of
helium and hydrogen fusion. In higher mass stars, other elements are formed by

428

astrophysics

Effective temperature (K)


7 000

6 000

4 000

10

5
10

Supergiants (I)
planetary nebula

AGB

6
4

H He

2 initial 5M
0

He C
H He

helium
ignition
RGB

HB

helium
flash

AGB
Giants (II, III)
He C HB
H He
Ma
in s
equ Subgiants (IV)
enc
e (V
)
H He

2
4

RGB

H He

8
White
dwarfs
(VII)

12

no nuclear
reactions

2
10

10

3
10

10

initial 1M

4
10

initial 0.5M

14

1
10

Luminosity compared to Sun

30 000 10 000

Absolute magnitude (Mv)

nuclei capturing neutrons. The Mira-type long-period variable


stars are AGB giants.
After a further 20 million years these shells become
unstable and begin to rapidly switch on and off, because the
helium reaction is spectacularly dependent on the temperature
(proportional to ~T40!). The star has been losing mass
throughout the giant phases, but now the pulsations gently
puff off the outer layers, exposing deeper layers of the star.
The intense UV light from the remains of the star lights
up the ejected gas as a planetary nebula (Figure24.4.2). The
nebula returns gas to the ISM that has been enriched in
carbon, nitrogen and oxygen by its passage through the
nuclear fires of the star.
At the centre of the planetary nebula, the remains of the
star contract and heat up to about 100000 K on the surface,
as the final fusion of hydrogen occurs in a shell around the
The fusion quickly ends and the
carbonoxygen core.
star, now a white dwarf, begins a long, slow cooling that will
last for tens of billions of years. With surface temperatures of
about 10000 K, but a size comparable to that of the Earth,
the white dwarfs are small and faint and appear near the
bottom of the HR diagram (Figure 24.4.1).
The star has reached its stellar graveyard with only about half
the mass it had on the main sequence and having only consumed
about 12% of its original supply of hydrogen fuel. Half its mass
has been returned to the ISM, somewhat enriched in heavier
elements, as raw material for the next generation of stars.

2
10
3
10
4
10

O5 B0
0.5

A0

F0
G0
Spectral class

K0

0.0

+0.3
+0.6
Colour index

+0.8

Figure 24.4.1

M0
+1.4 +2.0

Evolution of stars of low to medium mass


after they leave the main sequence

Explain the concept of star


death in relation to:
planetary nebula
white dwarfs.

PHYSICS FEATURE
When is a star not a star?

hat is a star? A sensible definition would say that a star produces


energy from nuclear reactions in its core and is stable because of
the balance between the inward pull of gravity and the outward forces of
gas and radiation pressure. By that standard, a white dwarf is not a star!
A white dwarf is a glowing ember. There are no nuclear reactions to
generate energy, despite a core temperature as high as 10 million K.
The energy trapped within the white dwarf from its energetic past slowly
leaks out through its relatively small surface area.
With as much as 1.4M of material in an object the size of the
Earth, the density of a white dwarf is enormousabout 109 kg m3
a million times the density of water. Gas pressure cannot support the
white dwarf against the correspondingly enormous gravitational force.
The balancing force comes from a new physical effect: degenerate
electron pressure. Essentially, the electrons reach a state in which they
refused to be packed more tightly.

Figure 24.4.2

An example of planetary nebulae


429

24

Birth, life
and death

Checkpoint 24.4
1
2
3

Outline the stages in the evolution of a solar mass star from main sequence star to white dwarf.
Describe the processes by which a star loses sufficient mass to end its life as a white dwarf.
Recall the characteristics of a white dwarf.

24.5 The fate of massive stars

430

Luminosity compared to Sun

Absolute magnitude (Mv)

For any star the basic problem is the same throughout its life. To be stable,
it must generate enough energy to ensure that internal pressure balances the
gravitational force. In a massive star, this means higher core temperatures to
generate the energy required and a correspondingly shorter lifetime.
For stars that start life with more mass than the Sun, but still less than about
8M (the middle of spectral class B), evolution is faster than for lower mass stars
(Figure 24.4.1), but they have the same fate in store. They lose more mass via
stellar winds and all end up below the 1.4M limit for the mass of a stable
white dwarf.
Stars starting life more than about 8M evolve much more quickly
Describe the types of nuclear
and have a different ending (Figure 24.3.3). Their main sequence lifetimes are
reactions involved in
measured only in millions of years (Table 24.3.1). Even at this stage, their stellar
Main Sequence and post-Main
winds lead to significant mass loss (perhaps 106M per year for a 60M star).
Sequence stars.
Their evolutionary path is flatter on the HR diagram (Figure 24.5.1), indicating
Discuss the synthesis of
elements in stars by fusion.
that their size changes dramatically but their luminosity does not. These stars
become supergiants, perhaps 1000 times the size of the Sun. They cross back
and forth across the instability strip, becoming Cepheid variables for part
of their lives.
In their cores, the onset of helium burning is gradual,
Effective temperature (K)
but as the temperature increases to more than 300 million K
30 000 10 000 7 000 6 000
4 000
10
supernova
the fusion of carbon with helium to produce oxygen
AGB
5
helium
10
He C
8
ignition
becomes the dominant reaction. When the temperature
H He
HB
RGB
4
6
climbs above 500 million K, carbon nuclei can fuse together
10
Supergiants (I)
H He
4 initial 10M
to produce sodium, neon and magnesium. All the carbon is
3

10
2
consumed quickly and the core collapses further, pushing
2
10
the temperature ever higher. Ultimately, fusion of silicon
0
Giants (II, III)
Ma
in s
produces iron at core temperatures of 7 billion K.
2
10
equ Subgiants (IV)
enc
The core of the supergiant resembles the layers of
e
4
(V)
1
an
onion
in which an iron core is surrounded by shells in
6
1
which
silicon
and sulfur, oxygen and carbon, helium and
10
8
hydrogen are all undergoing fusion reactions (Figure
2
10
10
White
24.5.2). However this is a fleeting phase. A 25M star will
dwarfs
3
12
10
(VII)
fuse hydrogen on the main sequence for about 7 million
14
4
10
years. It then spends 500000 years also consuming helium,
O5 B0
A0
F0
G0
K0
M0
600 years consuming carbon, half a year consuming oxygen
Spectral class
and just a day consuming silicon.
0.5
0.0
+0.3
+0.6
+0.8
+1.4 +2.0
Colour index
Synthesis of iron is the limit of normal fusion processes,
since
any further reaction costs the star energy rather than
Figure 24.5.1 Evolution of high-mass stars after they leave
generating it, accelerating its collapse. Within seconds a
the main sequence

astrophysics
portion of the core the size of the Earth collapses to just a few
kilometres across, achieving densities of 1017 kg m3. The speed
Fe core
core
of collapse causes the core to bypass the electron degeneracy
region
At 1012 K, degenerate
pressure that supports a white dwarf.
neutron pressure finally halts the collapse at the centre; however,
the surrounding layers hit the core and bounce back, blowing the
star apart in a supernova explosion. In a few seconds 1046 J of
energy is released, 99% of it as elusive neutrinos. Just 1% of the
H shell
supergiant star
Si, S shell
He shell
energy appears as visible light, but this is sufficient for it to
C, O shell
outshine all other stars in the galaxy for a few days!
Figure 24.5.2 The shell structure in the core of a high-mass star
During the explosion, iron nuclei are ripped apart and
just before it explodes
a large number of neutrons are released. These neutrons can be
captured by existing heavy nuclei to produce heavier nuclei than
Explain the concept of star
could be produced by any process earlier in the stars history.
death in relation to:
The blast wave and several solar masses of gas ejected by the supernova
supernovae
explosion speed outward at several thousand km s1, sweeping up gas blown away
neutron stars/pulsars
earlier by the stellar wind (Figure 24.5.3). The result is a supernova remnant
black holes.
(Figure 24.5.4), glowing across the full range of wavelengths and carrying gas
enriched in a full range of heavy elements back into the ISM. After perhaps
100000 years, the expanding remnant fades and merges into the ISM, but it may
have already triggered a new burst of star formation nearby.
Back at the site of the explosion, some of material very quickly falls back onto
Computer simulations remain very
the core of degenerate neutrons.
uncertain, but they indicate that stars with initial masses of between 8M and
roughly 25M will produce a core of less than 3M. The core will then be stable
as a neutron star. Much rarer stars with initial masses of between roughly 25M
and 40M will have more material falling back. The core mass will exceed the
3M limit of stability for a neutron star. No known force will stop the collapse
and the result is the formation of a black hole. Computer simulations suggest
that even more massive stars may form a black hole before an explosion can occur!

Figure 24.5.3

1994.4

2000.9

2005.2

2008.3

The expanding remnant from supernova 1987A


brightens in its radio emission as the outrushing gas
crashes into the surrounding medium.

Figure 24.5.4

The 14 light-year diameter remnant from Keplers 1609


supernova is shown in this image assembled from images
taken by the Hubble, Spitzer and Chandra space telescopes.
431

24

Birth, life
and death

PHYSICS FEATURE

rotation axis

magnetic axis
beamed
radiation

Neutron stars: Pulsars

neutron star packs a mass about


that of the Sun into an object
magnetic field
typically only 10km across. As the core
shrinks to this size, the rotation of the
neutron star increases to a rate that is
neutron star
typically dozens of times a second! The
acceleration due to gravity at the surface is
possible
about 1012ms2 (compared with 9.8ms2
line of
at the surface of the Earth) and its magnetic
sight to
Earth
field near the surface is about 108T!
beamed
Neutron stars were first proposed in the
radiation
1930s, but no-one expected to be able to
Figure 24.5.5 The lighthouse model sweeps beams of radiation
see such tiny objects. In 1967 radio emission
around the sky as the pulsar rotates.
was detected from a rapidly pulsing source that
was quickly recognised to be a rotating neutron
very precise rate that reduces only very slowly as the
stara pulsar.
pulsar loses energy.
Pulsars produce two beams of radiation from
A few pulsars have been observed at visible, X-ray
near their magnetic poles. As these poles are usually
and -ray wavelengths, but most of the 1700 known
not aligned with the rotation axis, these beams sweep
pulsars have been discovered by radio telescopes.
around the sky as the pulsar rotates rapidly (Figure
Many of these, including the first binary pulsar
24.5.5). As it spins, the beams may sweep across the
system, have been found by the 64 m Parkes radio
Earth, to be seen as a pulse. The pulses arrive at a
telescope (Figure 21.4.1).

Black holes

lack holes are even more bizarre than neutron


stars. With no known force able to resist the
collapse, all the mass is concentrated at a point
and surrounded by an intense gravitational field.
They are black because not even light can
escape once it has crossed a theoretical boundary
called the event horizon. At the event horizon, the
escape velocity (see section 1.3) is the speed of light.
Black holes have many intriguing physical
properties, both within the event horizon and outside
it. Many of these do not directly affect their behaviour
as astronomical objects, as their results cannot be
observed.
Others enable us to detect the hole,
because of the effect on surrounding matter, despite
the tiny size and blackness of the hole.
A black hole in a close binary system may draw gas
off its companion. The gas will be heated as it falls
onto an accretion disc (something like that around a
protostar) around the black hole. The gas emits highenergy radiation at UV, X-ray and -ray wavelengths.
432

Figure 24.5.6 An artists impression of an accretion disc


around a black hole with jets of emission

Stellar mass black holes are thought to range in


mass from 3M to about 15M. A supermassive
black hole of 3 to 4 million M is be believed to mark
the centre of the Milky Way galaxy, and even larger
black holes are thought to be present in some
distant galaxies.

astrophysics

Checkpoint 24.5
1
2
3

Outline what happens in a supernova explosion.


If iron is the heaviest element made through normal fusion reactions, explain how heavier elements are made.
Outline the properties of a neutron star.

24.6 How do we know?


How do we know that the picture of stellar evolution presented in this chapter is
correct, when almost every step takes far longer than a human lifetime?
Our understanding is based on computer modelling of the structure of stars.
This takes the physical laws we know and carefully applies them to the situation
inside stars. Comparison of our models with reality gives us confidence that our
understanding of stellar structure and evolution is largely correct.
A key set of evidence lies in the HR diagrams of star clusters. Open
(or galactic) star clusters typically contain a few hundred relatively young stars
in a loose grouping about 10 light-years across (Figure 24.6.1). In contrast,
a globular star cluster usually contains hundreds of thousands of old stars in a
sphere about 100 light-years across (Figure 24.6.2).
Stars in an open cluster are expected to have formed from a single cloud with
the same initial composition. We therefore expect a well-defined ZAMS line to
mark the birth of the cluster stars on an HR diagram. Plotting that diagram is
made easier since all the stars are at essentially the same distance from us, so the
apparent magnitudes of the stars reflect their true relative luminosity.

Figure 24.6.1

The open star cluster M44

Figure 24.6.2

Explain how the age of a globular


cluster can be determined from
its zero-age main sequence plot
for a HR diagram.

The globular star cluster M92

433

24

Birth, life
and death

Figure 24.6.3a is an HR diagram for the open cluster M44. The line on the
diagram estimates the ZAMS. Most of the stars are clearly still on the main
sequence. However, stars at the high-mass end are starting to evolve away from
the main sequence, and a few more-evolved supergiants are apparently also
present. This is consistent with our expectations that stars move off the main
sequence to the right on the HR diagram and massive stars evolve most quickly.
Figure 24.6.3b is a rather different HR diagram for the globular cluster M92.
The turn-off point is much lower down the main sequence, and a well-developed
The turn-off point is an indicator of the age of the
giant branch is apparent.
cluster; the further down the main sequence the turn-off point is, the older the
cluster. The M44 cluster is estimated to be about 600 million years old, while the
M92 cluster was formed early in the universeabout 13 billion years ago.
a

4.0

12

6.0

14

8.0
V

V 10.0

Activity Manual, Page


188

16

18

12.0

PRACTICAL
EXPERIENCES
Activity 24.1

ZAMS

ZAMS
20

14.0
22
0.0 0.4 0.8 1.2 1.6 2.0
BV

0.4 0.8 1.2


BV

1.6

2.0

Figure 24.6.3 Colour index versus apparent magnitude for stars in the (a) M44 cluster and
(b) M92 cluster

PHYSICS FEATURE
A final thought

imon Newcomb (18351909) was a CanadianAmerican astronomer and mathematician who,


in the late 19th century, said: We are probably
nearing the limit of all we can know about astronomy.
One hundred and twenty years later his comments
seem a bit premature, since much of our modern

understanding of the stars was unknown to Newcomb.


We should take this as a warning not to think that our
impressive knowledge today is complete. There is
much we dont understand and probably even more we
dont know anything about. Thats what makes
astrophysics so exciting!

Checkpoint 24.6
1
2

434

Explain how the HR diagrams of star clusters support our understanding of stellar evolution.
If black holes emit no light, explain how we can observe them.

PRACTICAL EXPERIENCES

astrophysics

CHAPTER 24
This is a starting point to get you thinking about the mandatory practical
experiences outlined in the syllabus. For detailed instructions and advice, use
in2 Physics @ HSC Activity Manual.

Activity 24.1: Star clusters on the HR diagram


Revise what you learnt in the Module 4 The Cosmic Engine of in2 Physics @
Preliminary by briefly reviewing the properties of different types of stars.
Discussion questions
1 Explain the differences in properties of open clusters and globular clusters.
2 Describe how these differences are revealed on an HR diagram.
106
105

10R

100R
Blue giants

Luminosity (L )

1000R

Supergiants (I)

104
103 1R

Red giants

Dwarfs (V)

Giants (III)

102
101

Present information by plotting


HertzsprungRussell diagrams
for:
nearby or brightest stars
stars in a young open cluster
stars in a globular cluster.

0.1R

100
101

0.01R

102
103

Red dwarfs

White dwarfs
0.001R

104
40 000

20 000

10 000

5000

2300

Temperature (K)

Figure 24.7.1

Simulation of HR diagram

435

24




Chapter summary

Birth, life
and death

The space between the stars is filled by the gas and dust
of the interstellar medium (ISM).
The gas of emission nebulae is energised by hot young
stars to emit an emission spectrum.
The dust of dark nebulae scatters starlight, reddening or
completely blocking our view.
The light of reflection nebulae is scattered by dust,
especially at blue wavelengths.
Giant molecular clouds are regions where the gas is most
dense and simple molecules can form. They are the sites
of star formation.
Gas from the ISM forms stars, is processed inside the
stars, and some is then returned to the ISM.
Giant molecular clouds can be pushed into gravitational
collapse by triggers such as supernovae.
Protostars warm up as infalling gas releases gravitational
potential energy and raises the central temperature until
nuclear fusion of hydrogen into helium begins.
Surrounding the protostar is an accretion disc and jets
of outflowing material.
Stars are born onto the zero-age main sequence (ZAMS).
The location of a star on the ZAMS, the time to get
there and its subsequent evolution are almost entirely
determined by the mass of the star.
The protonproton chain and the carbonnitrogen
oxygen cycle both combine four hydrogen nuclei into
one helium nucleus, with the release of energy and some
other light particles.
When a main sequence star runs very low on hydrogen
fuel in its core, its core shrinks and is surrounded by a
shell of hydrogen fusion, while the outer layers inflate
to create a red giant.
When the temperature of the core of a red giant is
greater than 100 million K, the fusion of helium into

carbon begins via the triple alpha process, explosively in


some stars as the helium flash.
A star moves up the asymptotic giant branch (AGB)
when it has a core of carbon and oxygen surrounded by
shells of helium and hydrogen fusion.
A planetary nebula is puffed off by a low- to mediummass star and returns gas to the ISM.
A white dwarf cools slowly for tens of billions of years,
supported by electron degeneracy pressure.
Stars starting life with more than about 8M evolve
much more quickly and have strong stellar winds,
leading to significant mass loss.
The core of a red supergiant briefly resembles the layers
of an onion in which an iron core is surrounded by
shells of silicon and sulfur, oxygen and carbon, and
helium and hydrogen fusion.
Core collapse and bounce back by the surrounding
layers blows a massive star apart in a supernova
explosion.
The blast wave and ejected gas from a supernova sweep
up other gas to form a supernova remnant.
Stars with initial masses of between 8M and roughly
25M will produce a neutron star that packs a mass of
up to 3M into a rapidly spinning, highly magnetised
object typically about 10 km across.
Pulsars are neutron stars that produce beams of radiation
from near their magnetic poles that sweep across the
Earth to be seen as very precisely timed pulses.
Stars with initial masses greater than roughly 25M
will produce a black hole with mass greater than 3M.
Black holes are black because not even light can escape.
They are detected because of their effect on the
surrounding matter.
The turn-off point on the main sequence is an
indicator of the age of a star cluster.

Review questions
Physically
Speaking

Match each term to the best definition.

436

Term

Definition

Planetary nebula

Pre-main sequence stage of a star

Supernova

Final state of a low mass star

Giant molecular cloud

Relatively gentle ejection of the outer layers of a star

Protostar

Location in space of the birth of a star

ZAMS

Possible final state of a high mass star

White dwarf

Location on the HR diagram of the birth of a star

Neutron star

Explosive ejection of the outer layers of a star

astrophysics
14 Compare the HR diagrams of an open cluster and
a globular cluster.

1 Stars A and B form in the same giant molecular

Analyse information from an HR diagram


and use available evidence to determine
the characteristics of a star and its
evolutionary stage.

15 For each of the stars marked on the HR diagram in


Figure 24.7.2, identify the spectral class and
luminosity class and describe its future.

2 Why do stars need core temperatures in excess of


10 million K before fusion commences?
sits on the main sequence.

Effective temperature (K)


30 000 10 000

on the main sequence.

6 Construct a table of the major nuclear reactions in


the life of the Sun. In the table, list:
a the temperature needed for the process to occur
b the stage in the Suns life at which it occurs.

7 Recall the nuclear reactions that occur during


different stages of the life of a 5M star.

8 Explain how a white dwarf can produce light with no


nuclear fusion in the core.

9 Outline the properties of a neutron star.


10 Outline the properties of a black hole.
Present information by plotting on an HR
diagram the pathways of stars of 1, 5 and 10
solar masses during their life cycle.

11 Construct a flow chart that maps the evolution of stars


of masses 1, 5 and 10M. Add as much detail as
possible.

12 On HR diagrams, construct the complete evolutionary


path from birth to death of stars of 1, 5 and 10M.

13 Estimate the time associated with the different stages


in the lifetime of stars of 1, 5 and 10M. Construct
a timeline to scale of the life of each star.

Absolute magnitude (Mv)

branch, giant molecular cloud, red giant branch,


protostar, helium flash, main sequence, T Tauri star

4 000

5
10

Supergiants (I)

4
10

5 A 1M star goes through several stages in its life,

planetary nebula, white dwarf, asymptotic giant

6 000

4 Explain how the mass of a star determines its lifetime

many of which are listed here but are out of order.


Construct a list of these terms in the correct time
sequence:

7 000

10

3
10

2
0

Ma

2
4

2
10

Giants (II, III)

in s

equ

10

enc Subgiants (IV)


e (V
)

1
10

2
10

White
dwarfs
(VII)

10
12

3
10

14
O5 B0
0.5

Luminosity compared to Sun

3 Explain how the mass of a star determines where it

4
10

A0

F0
G0
Spectral class

K0

0.0

+0.3
+0.6
Colour index

+0.8

M0
+1.4 +2.0

Figure 24.7.2

Solving problems
16 The Sun is mainly powered by nuclear reactions of
the protonproton cycle. Each reaction contributes
4.2 1012 J to the solar luminosity of
3.84 1026 W.
a Calculate how many hydrogen atoms fuse to form
helium every second to produce this luminosity.
b If the mass of the proton is 1.67 1027 kg,
calculate how much mass this represents.
c Assuming 70% of the Suns initial mass of
1.99 1030 kg was hydrogen, calculate how
many years it will take to convert all the hydrogen
to helium.
d Contrast this result to the expected lifetime of
the Sun. Explain any discrepancy.
iew

Q uesti o

cloud. Star A is 1M, star B is 5M.


a Predict which star will reach the main sequence
first.
b Predict which wavelength bands you would best
use to observe the early stages of the formation
of the stars.

Re

Reviewing

437

The review contains questions that address the key concepts


developed in this module and will assist you to prepare for the
HSC Physics examination. Please note that the questions on the
HSC examination that address the option modules are different in
structure and format from those for the core modules. Past exam
papers can be found on the Board of Studies NSW website.

Multiple choice
(1 mark each)
1 Which of the following telescopes must be used
above Earths atmosphere to observe astronomical
objects?
A Optical
B Radio
C X-ray
D Infra-red

438

The star Canopus ( Carinae) has an apparent visual


magnitude of 0.7 and the star Deneb ( Cygni)
has an apparent visual magnitude of +1.3. Which of
the following statements about Canopus is true?
A 2.0 times fainter than Deneb
B 2.0 times brighter than Deneb
C 6.3 times fainter than Deneb
D 6.3 times brighter than Deneb
The spectra of two stars indicate that they are of the
same spectral type, but star A has relatively broad
absorption lines while star B has very narrow lines.
What does this tell us about the two stars?
A Star A is more massive than star B.
B Star A has a more dense atmosphere than star B.
C Star A is older than star B.
D Star A is moving away from us faster than star B.
Consider two single stars with different masses, both
on the zero-age main sequence. What properties
would you expect for the more massive star, relative
to the less massive star?
A Lower luminosity and shorter lifetime
B Lower luminosity and longer lifetime
C Higher luminosity and longer lifetime
D Higher luminosity and shorter lifetime

What properties would you expect to be approximately


common between stars within an open star cluster?
A Age
B Luminosity
C Age and initial chemical composition
D Age, initial chemical composition and luminosity

Short response
6 Describe the problems associated with viewing
astronomical objects through ground-based optical
telescopes. (2 marks)

7 Explain why the limit of accurate parallax


measurement from Earth is different to that
from space. (2 marks)

8 The apparent magnitude of star A is 2.4 while that


of star B is 0.3.
a Calculate the brightness ratio of star A to star B.
b Explain which star is brighter. (2 marks)

9 Outline how to calculate the distance to a Cepheid


variable star. (2 marks)

10 Describe the evolution of a 1M star. Make sure to


include the various stages reached and nuclear
reactions taking place. (2 marks)

11 Explain why the HR diagram of a globular cluster is


different from the HR diagram of an open cluster.
(2 marks)

astrophysics

Extended response
12 Outline an experiment you undertook in class to
observe emission, absorption and continuous spectra.
Describe the differences you observed between the
spectra, and identify an object that produces each
type of spectra. (3 marks)

13 For each of the stars marked on the HR diagram in


Figure 24.8.1, list the properties and probable stage
of evolution. (3 marks)
Effective temperature (K)
30 000 10 000

7 000

6 000

4 000

10

10

8
A

10

Absolute magnitude (Mv)

10

Ma

in s

equ

10

10

enc

1
1

10

Luminosity compared to Sun

10

10

12

14

10

10
O5 B0

0.5

A0

F0
G0
Spectral class

K0

M0

0.0

+0.3
+0.6
Colour index

+0.8

+1.4

+2.0

Figure 24.8.1

439

7
Context

Figure 25.0.1

440

It is important to fine tune


your skills in order to get the
most out of your experiment.

skills

In the preliminary text you were introduced to many skills that are needed to get
through Physics. In the HSC year it is important to further develop these skills, and
to make sure you know how to use the skills without thinking twice. You will now be
able to fine tune and add to what you have learnt.
One of the most important skills for a physicist is being unambiguous, because
the mathematical nature of physics demands that everything be defined strictly and
clearly. Its also important as a scientist to have the skill of communicating clearly
the results of research to other scientists.
The HSC year is about clearly getting across what you have learnt. It is important
not to be ambiguous, so you must learn how to interpret questions and construct
your answers. Throughout this module we will focus on the skills you need to make
sure what you have learnt is conveyed clearly.

Inquiry Activity
The Bungee jumper
Lets say you are responsible for getting the bungee cord length (lo)
correct for the next jumper.
The elasticity (k) of the cord is always known. For simplicity,
assume that the energy conversion is close to 100% efficient.
You are given the chance to take any measurements of the jumper.
How could you calculate the length of the cord you need to attach
to the jumper if the bridge stands 100 m above the river below and
the jumper is to stop between 10 and 100cm from the water for
maximum thrill. Not too close for safety, but not too far for thrill.

Figure 25.0.2

A bungee jumper

Hints
A sketch of the situation (Figure 25.0.3) has been given to you to help you
along the way.
1 List the types of energy that is available at the top of the jump
1 22
EPE== kx
kx
(this will include gravitational U = mgh, elastic potential energy EPE
2
1 22
=
kx
and kinetic energy EEPE
=
mv
)
and
the
values
of
things
that
are
known.
K
2
2 Using the law of conservation of energy, write an equation that includes all
the types of energies you have listed.
3 hi is the maximum height that the cord can stretch to. It is made of the
original length of the cord plus the stretched length, i.e. hi = lo + x.
Figure 25.0.3 A sketch of the situation
4 Rearrange the equation to solve for the bungee cord length (lo).
Think about how to do this and then have a go at doing it in a scaled-down
experiment.
What will you need to know, or take measurements of? How accurate will you
need to be?
Set up your experiment to test it.

Questions
1 List any assumptions that you have made in your calculations.
2 Determine how it is possible for you to test your prediction accurately.
3 List problems involved with your experiment and how it is possible to
reduce them.
441

25

Skills
stage 2
Skills introduction

prefix, systematic error,


random error, accuracy, precision

Skills are a very important component of the Year 12 syllabus.


In Year 11 you started learning them and in Year 12 you will
develop them further. The skills that are important to develop
include planning and carrying out investigations, researching
and communicating what you have found, and problem
solving. Most of these skills are developed while you learn the
material in the modules, but there are a few specific concepts
that will be discussed separately in this chapter, to aid with
your development.

25.1 Metric prefixes


In the preliminary course, you were taught to express numbers using scientific
notation. This is a way of representing a large or small number in an
unambiguous, compact way. To add to this, we will introduce the idea of
prefixes. Although you may not know the term prefix, you are probably very
familiar with some of these in everyday life, for example kilogram and millimetre
and centimetre. They are usually printed on the back of exercise books.
Prefixes help reduce the written size of the unit of measurement. A list of
common prefixes (and some less common ones) is given in Table 25.1.1. For
example, the basic SI unit of distance is the metre (m). A centimetre (cm) is then
1/100th of a metre. The prefixes that are commonly used in your course have an
asterisk placed next to them. You are expected to know the values of these, as
they will be used in questions and their value will not be given in any exam you
undertake, so it is in your best interests to become familiar with them.

442

SKILLS
Sometimes you will need to convert from one prefix to another. The easiest way to
carry out the conversions is to follow the steps set out in the worked example below.

Worked example
Question
Convert 5.49mm to m.
Step 1: Express the number in scientific notation.
5.49 103m

Step 2: Look at the prefix that you need to convert the number to. Express the prefix in
scientific notation.

micro = 106

Step 3: Look at the difference between the two.


3

Number

Prefix

101

deci-

10
103
106
109

centimillimicronano-

c
m

1012

pico-

1015

femto-

10

18

atto-

10

21

zepto-

1024

yocto-

Solution

Table 25.1.1 Common metric prefixes

Symbol

10 m to 10 m

10

deka-

da

To change the exponent to 106, the decimal place must be moved 3 places to
the right.

102

hecto-

10
106

kilomega-

k
M

109

5.490 10

5490 106 = 5490 m

Checkpoint 25.1
1

2
3

Express the following masses using prefixes in terms of the


specified unit.
a 0.45 g in mg
b 345 g in kg
c 5600000 g in Mg
Convert 4.5 km to Mm. (Note the difference between Mm and mm.)
Convert 0.54 nm to km.

giga-

12

10

tera-

1015

peta-

10

exa-

1021

zeta-

yotta-

18

24

10

*
*
*
*

*
*

25.2 Numerical calculations


In terms of exam performance, exam technique can be just as important as
understanding the physics that you have been studying. If you cannot decipher
the question and communicate your answer clearly, you cannot show that you
understand physics. Chapter 26 Revisiting the BOS key terms will go through
the technique of answering questions that require a written response. Here we
will look at numerical answers.
When attempting any question involving numerical calculations, follow this
series of steps:
1 Highlight numerical values within the question.
2 Identify the unknown that must be determined.
3 Write the relevant equation.
In each of steps 13, write the item down at the top of your answer, then:
4 Show all working.
5 Write your answer with units.
443

25

Skills
stage 2

Worked example
Question
A driver of a car travelling at 16.0 m s1 slams on the brakes when he sees a ball on the
road in front of him. The ball is 25 m away at the time the brakes are applied. Will the car
stop in time, given that the deceleration is 2.5 m s2?

SOlution
Step 1: The data have been highlighted in the question.
Step 2: List them below.

Assume the direction of initial velocity is positive.

u = +16.0 ms1

v=0

a = 2.5 ms2

s = +25 m

The unknown is actual displacement s taken to stop.

Even though our list shows that there is a displacement, this is not the actual
displacement of the car, but the initial displacement of the ball from the car.

s=?

Step 3: v2= u2 + 2as


Step 4:

0 = (16.0)2 + 2(2.5)s

s = 256/5

s = 51.2 m

Step 5: This is more than 25 m, therefore the car will not stop in time.

Recall from the preliminary course that the answer to a calculation needs to
be expressed correctly in order to reflect the accuracy of the measurements.
First you must look at the calculation itself. The calculation should be
completed with all the available digits in each value. When multiplication or
division is used, the answer should be expressed to the least number of
significant figures used in the values in the question. With addition and
subtraction, the answer is expressed to the lowest number of decimal places
within the question.
Remember that a calculator can invent accuracy. Just because the calculator
gives you an answer with 10 digits doesnt mean that the answer has become that
accurate. Refer back to the question and reduce your answer accordingly.

Worked example
Question
Correctly express the answer from the worked example above.

Solution
The equation has multiplication and division in it, therefore we will need to look at the
answer in terms of significant figures.
The least number of significant figures in the question is 2.
Therefore the answer to the calculation is 51m.
444

SKILLS

Checkpoint 25.2
1
2

An aeroplane has touched down on the runway at 290 kmh1. The braking ability allows it to decelerate at a rate of
11 m s2. The runway is 3.9km long. Will the aeroplane stop in time?
A satellite needs to be launched into a circular near-Earth orbit. At what height above the Earths surface would the
satellite need to be placed in order to maintain a period of 90 minutes?

25.3 Sourcing experimental errors


There are errors involved in all measurements. The magnitude of the error and
how many errors are present can be defined by the equipment you choose to use
in an experiment as well as the method you use. So how can you minimise the
effect of errors and make sure your experiment is both valid and reliable?
A valid experiment is one that actually tests what the aim has stated. This
sounds obvious but it can be easy to stray from the task at hand.
A reliable experiment is one that can be repeated each time with the same
results, no matter who is carrying it out. How do you achieve this?
Make clear statements in your method.
Express clearly the need for repetition and averages.
Use the most accurate equipment available.
It is also important to understand the difference between systematic and
random uncertainities (often simply called errors).
Systematic error is something within the experiment that causes the readings
to be consistently high or low; they are always the same magnitude and sign no
matter how often the experiment is repeated. This includes things such as errors
in calibration, or a repetitive mistake in experimental technique. Systematic error
can be minimised by careful experimental techniques and proper understanding
of the experimental apparatus.
Random error is the unpredictable fluctuation of results, due to rapidly
changing external influences, limitations of the readability or sloppiness of an
instrument or method. If a measurement is taken repeatedly, it would be noted
that the values would be readings scattered around a number. Although random
error cannot be eliminated, it can be reduced by good experimental practice and
by repeating the measurements many times and taking an average.
Two words closely related to these ideas are accuracy and precision. An
accurate measurement is one with small systematic error, while a precise
measurement is one with small random error (see Figure 25.3.1).
It is not always clear if an uncertainty (error) is systematic or random. Some
causes could be considered partly systematic and random. Consider the effect on
the result and how best it fits in the definitions before applying the rule blindly.
Remember to record your values to the accuracy of your instruments. If you
have equipment that is accurate to 1/1000th of a unit, record it in its entirety.
Remember to place any measured zeros at the end if appropriate (e.g. 7.600 m).
If you dont, you have lost that accuracy and use of the equipment is pointless.
Then perform any calculations to the full number of figures available in the data
and only reduce the number of figures expressed in the answer at the end.

precise but inaccurate

accurate but imprecise

inaccurate and imprecise

accurate and precise

Figure 25.3.1

Accuracy and precision

445

25

Skills
stage 2

You may be asked in an exam to express how you made sure that your
experiment was valid and reliable, so make sure you do this when you are doing
the experiment (refer to Chapter 17 Physics skills in in2 Physics @ Preliminary).

Checkpoint 25.3
1

You are required to carry out an experiment that determines the reaction time of your lab partner. Hold a ruler just
above your partners fingers (see Figure 25.3.2). Your partner must catch the ruler after you drop it without warning.
Record how far the ruler has dropped by the time it is caught. Use your knowledge of physics to determine the
reaction time of your lab partner.

Repeat the process five times at an interval of 30s. Does your partner become better?

After a day or so, do a similar experiment, this time using a data-logger sensor. Does your partner become better?
2
3
4

Determine if the experiment is reliable.


Is the experiment valid?
How can the experiment be made more reliable and valid?

Figure 25.3.2

Your partner must catch the ruler after you drop it without warning.

25.4 Presenting research for an exam


During the course of your study you were required to gather a lot of data from
primary and secondary sources. It is important to know how much of this you
need in order to accurately answer questions based on this material.
In the activity manual you have found templates to help guide your search so
that you had information that covered the content to the right depth. The
discussion questions in the student book also help guide you in reaching the
depth that is required. If you can answer these from your research you are on the
right track.
Once you have extracted the information, practise questions on the topic from
past exam papers so you can get a feel for what can be asked.
The biggest mistake you can make is storytelling. Remember to answer the
question, dont just regurgitate everything that you have researched. How to
answer questions as they are asked will be looked at in more detail in Chapter 26
Revisiting the BOS key terms.

Checkpoint 25.4
During your study of Module 1 Space, you were required to research a scientist in relation to their contribution to space
exploration. Use this information to answer the following question: How has your scientist influenced those who have
come after him/her?

446

SKILLS

25.5 Australian scientist


One skill that you need to undertake in Physics is the study of an Australian
scientist. In this student book there is information about Australian scientists.
Pick one that interests you to study in detail. Things that you are looking to
know about will include their name, the area of physics in which their current
research is being carried out, what the research is, where they are currently
working and what area of your study relates to this scientists work.

Checkpoint 25.5
For your chosen scientist, state the research carried out and explain how the research relates to the Physics course.

25.6 Linearising a formula


When looking for a relationship between two variables it is usually simpler to
deal with a linear relationship. What happens if the relationship between two
variables in a formula is not a straight line? It is not necessarily a problem, as
sometimes we can rearrange the formula in order to graph the two variables so
that they relate to each other in a linear way. Confused? Lets look at the steps
to follow with an example.

Worked example
Question
A student is looking at how the velocity v of a ball relates to its displacement s as it falls
through the air when dropped. To do this, the student drops a ball and records the start velocity,
the displacement it goes through and the end velocity.
What does the student need to graph in order to get a linear relationship?

Solution
The formula that we can use to relate these two variables is:
v2 = u2 + 2as
From this equation, we can see that there is not a linear relationship between v and s, but there
is a linear relationship between v2 and s. Graphing v2 against s would produce a straight line.

Although it is a good skill to know, linearising is less important today when


Excel (and similar applications) can fit other functions such as power laws directly
(a power law with power 2 in this example). The fit may not be identical because
the least squares fitting will be minimising slightly different values.

Checkpoint 25.6
1
2

L
.
g
What variables would need to be placed on the axes of a graph if a straight line was to result?

Determine the linear relationship between length and period from the formula T = 2

447

26

Revisiting the
BOS key terms
Verbs in action
In the Preliminary course, you were introduced to the idea of
grouping the BOS key terms in order to help you answer questions.
Below is a table with the BOS key terms in the allocated groups.
From working through the examples in the Preliminary course,
you should be very familiar with how to formalise your answers.
In the HSC year you will not have as much of an opportunity to fill
in a table and analyse your answer as completely as you have up to
now. Because you are now familiar with the structure of the answers,
we will introduce the short examination technique to help you. If you
are still not confident with structuring your answers, continue to work
as outlined in in2Physics @ Preliminary.

Table 26.1.1 BOS key terms in their allocated groups


Group 1: Knowledge

Group 2: Comprehension

Group 3: Application

Define

Account

Apply

Extract

Clarify

Calculate

Identify

Compare

Classify

Outline

Contrast

Demonstrate

Recall

Describe

Examine

Recount

Discuss
Distinguish
Extrapolate
Interpret
Predict

Group 4: Analysis

Group 5: Synthesis

Group 6: Evaluation

Analyse

Construct

Appreciate

Explain

Propose

Assess

Investigate

Summarise

Deduce

Synthesise

Evaluate
Justify
Recommend

448

SKILLS

26.1 Steps to answering questions


1 Highlight the verb.
2 Highlight the main topics that make the question.
3 Recall the group the verb falls into and determine the depth of information
needed.
Group 1: List information.
Group 2: Interpret details of topic.
Group 3: Apply knowledge.
Group 4: Look at relationships and organisation.
Group 5: Put together information to decipher.
Group 6: Make judgements and draw conclusions based on fact.
4 Record in point form in the margin what needs to be statedinclude
diagrams. This should be a brief outline to organise your ideas so the order is
correct and you dont go off topic as you progress in your writing.
5 Elaborate on each point in the space providedrefer to your diagrams.
The best way to make sure you can write a good answer is to practise writing
good answers.
What follows is a set of worked examples for each of the verb groups.

Group 1: Knowledge

Group 2: Comprehension

Worked example 1

Worked example 2

Question

Question

Define orbital velocity with reference to satellites.

During an experiment carried out in class, students had two currentcarrying wires, one placed near an electronic balance, the other on
it. They noticed that the readings fluctuated as they changed the
current moving through one of the wires. Discuss how these
readings changed in relation to the current.

Notes
Define falls into group 1, therefore this requires you to list
information.
Orbital velocity: tangential velocity of satellite when moving in
an orbit around the Earth
The speed of the satellite is given by the following formula:
Gm
r
This shows that the velocity is affected by the mass of the central
objectthe Earthand the radius from the centre of the object.
v=

Notes
Discuss falls into group 2, therefore this requires you to list
information and interpret details of topic.
A current-carrying wire has a magnetic field around it.
Two current-carrying wires near each other will attract or
repel, depending on the direction of the current.

Solution

One wire is on the balance; balance will register the weight


of it.

Orbital velocity of an object is the tangential velocity of an object


that is in orbital motion around another object.

If the wires are repelling, the reading also registers the


extra force.

If the orbit is circular, the satellite must have a velocity that is


determined by the formula:
Gm
v=
r
The speed of the satellite will be determined by the radius from
the centre of the Earth as well as the mass of the Earth. The velocity
will vary if the orbit is eliptical.

If the size of the current or the direction of the current is


changed, the reading will change by the amount the force
is changing.
Force is determined by:
II
F
=k 12
l
d
449

26

Revisiting
the BOS
key terms

Solution
Two current-carrying wires will either attract or repel. If the current
is travelling in the same direction in both wires, the wires will
attract. If the current is in different directions, then they will repel.
The size of the force is determined by:
II
F
=k 12
l
d
As one wire is resting on the balance, the balance is registering
both the weight of the wire and the force created by the interaction
of the wires.

r3
2

86 400

r=

6.67 1011 6.0 1024


4 2
3

6.67 1011 6.0 1024 86 4002

= 4.2 107 m

42000 km

4 2

Group 4: Analysis
Worked example 4

Force is proportional to the amount of current. If there is a


repulsive force originally, then the registered reading will decrease
as the current is decreased and increase as the current is increased.
If there is an attractive force originally, then the registered reading
will decrease as the current is increased and increase as the current
is decreased.

Notes

If the direction of the current is reversed, then the force will


change from attractive to repulsive or vice versa, with a change in
the readings as mentioned above.

Explain falls into group 4, therefore this requires you to list


information and interpret details of topic and apply knowledge and
consider relationships and organisation.

Worked example 3

English versus German research.

Question
Calculate the radius of orbit for a geostationary satellite.

Notes
Calculate falls into group 3, therefore this requires you to list
information and interpret details of topic and apply knowledge.
Geostationary = 24 h, orbit = T
It is in orbit around the Earth, therefore M = 6.0 1024kg.
Equation that applies is
T

Solution
r3
T2

GM
4 2

r=?
M = 6.0 1024 kg
T = 24 60 60 = 86400s
G = 6.67 1011

450

Explain how the debate over the apparent inconsistency in behaviour


of cathode rays was finalised.

Inconsistent behaviour refers to cathode rays displaying


both wave and particle properties.

Group 3: Application

r3

Question

GM
4 2

German findings supported wave theory as electric field did


not cause deflection.
Hertz showed that cathode rays could pass through thin
layers of metal such as gold and silver.
Resolved when JJ Thomson could deflect the ray with an
electric field.

Solution
Both Hertz and Crookes observed properties of the cathode rays that
showed properties of both waves and particles. The main reason for
Hertz believing that the ray was a wave was from two experiments:
one showed that when an electric field was applied there was no
evidence of the ray moving, and the second showed that the rays
could pass through thin metal. This resembled the behaviour of
light, which was known to be a wave.
It was not until JJ Thomson performed the same experiment
again that the debate was put to rest. Thomson produced a nearperfect vacuum, eliminating any trace atoms within the tube, and
coated the end of the tube with a fluorescent screen. It was noted
that when the electric field was applied the ray was bent in the
direction in which a negative particle would move.

SKILLS

Group 5: Synthesis

Group 6: Evaluation

Worked example 5

Worked example 6

Question

Question

Propose an experimental method to be able to determine Plancks


constant.

Justify the extensive safety precautions that are evident on the


Space Shuttle in order to protect the astronauts.

Notes:

Notes

Propose falls into group 5, therefore this requires you to list


information and interpret details of topic and apply knowledge and
consider relationships and organisation and put together
information to decipher.

Justify falls into group 6, therefore this requires you to list


information and interpret details of topic and apply knowledge
and consider relationships and organisation and put together
information to decipher and make judgement and draw conclusion
based on fact.

E = hf
W = qV so measuring V will allow you to get a measure of E.
f can be varied by using coloured filters.
Set up apparatus as shown.
Measure the stopping voltage for each frequency.
Produce a graph of V against f.
Convert V to E by multiplying by 1.6 1019.
Determine the gradient of the graph. This is equal to h.

Solution
Set up the equipment as shown. (Draw a diagram like
Figure 9.3.1.)
Measure the stopping voltage for each frequency of light.
(Stopping voltage is the voltage to stop current flowing in
the circuit.)
Produce a graph of E against f. To determine E multiply V
by 1.6 1019. (Draw a graph like that shown below.)
The gradient of the graph is h.
EK

EK = hf
slope = h

threshold
frequency

Frequency

Heat shields: heat energy not allowed to travel into


the Shuttle
Lying down on take off: improves tolerance of g-forces
Parachutes: remaining kinetic energy is removed on landing
Entry angle
Nose cone: rounded to change kinetic to heat energy
efficiently

Solution
The Space Shuttle was the first reusable space transport. It was
designed to fly many missions and be reused quickly while
protecting those inside.
On take off, the g-forces that are applied to the astronauts are
massive. As they need to be able to withstand the g-force, the
astronauts lie down during take off, so the blood does not rush
away from their vital organs and they remain conscious throughout
the launch.
Most of the precautions are taken for the re-entry. On the
approach the astronauts must aim the shuttle to come in at an angle
of between 1 and 2, in order to avoid burning up in the atmosphere
or skipping off the atmosphere. The burning is due to friction. A small
amount of this has to occur, and so heat shields are used to minimise
the effects on the astronauts. The underside of the surface of the
Shuttle is covered with tiles that can be superheated and dissipate
the energy quickly. This avoids extensive heat inside and converts the
massive amounts of KE to heat quickly, slowing the Shuttle.
The nose cone of the Shuttle is fairly blunt. This also aids in the
conversion of KE to heat and therefore helps to slow the shuttle.
A pointed nose would be more likely to melt.
There is a limit to how much energy can be converted to heat
safely, so the final means of slowing down is the use of parachutes.
These parachutes are deployed on landing and allow the Shuttle to
slow and stop safely.
The main purpose of the Shuttle is to transport humans safely.
These precautions must be taken in order for the occupants
to survive.

451

Numerical answers

Numerical answers
These are selected numerical answers only. A complete set of
answers can be found in the Teacher Resource.

Module 1 Space
Checkpoints: 1.1 1 ah = 0, av = 9.8ms2 down 2 =45,
=90 3 drag 4 both at the same time. 6 see Table 1.1.1
1.2 1 F = Gm1m2)/d2 3 1/d2
1.3 2 EP = (Gm1m2)/r 3 0
Review questions: 3 2.5s 5 a 2Fi b 4Fi c Fi/4 8 11.2kms1
10 a 1.38s b 4.37m above ground c 18.0m (right)
d 14.3ms1, 25 below horizon 11 a 25.3ms1, 0.883s
b 17.9ms1, 1.25s 13 5.35ms1 14 260.5N, 89.99
from x-axis 15 7.0 m
17 5.30108J 18 10300ms1 19 a 42100ms1

Chapter 2
Checkpoints: 2.1 1 Sergey Korolyov, Wernher von Braun
6 a g-force = 1 b g-force = 0 c g-force = 0
2.2 3 ac = v2/R 8 True 9 a = r 10 Mercury
2.3 1 circle, ellipse, hyperbola and parabola 2 hyperbola
2.4 2 hyperbolic 3 they are equal 4 vmax = 2VP
2.5 2 false
Review questions: 15 6.25107 J, 2.121012m
16 a 10690kgs1 b 5.2ms2, 12.1ms2 17 a w = mg
b N = 0 c ac = g d v > 7.00ms1 e 0 19 32.7 h1
20 16.8 days 22 a 4.22107 m b 2.66107 m
23

KE

Geostationary 9.47109 J

Review questions: 16 1380 Nm 17 5.4 104 Nm


18 b 2.7 c nBAcos (= nBA when 0 = 0) d 30T

Chapter 7

Chapter 1

Type of
orbit

Chapter 6

GmM
2a

Two-body PE

ME =
KE + PE

1.8871010 J

9.40109 J 9.44109 J

Checkpoints: 7.2 4 1200V


Review questions: 22 3600V 23 8%
24

Coils in
primary

Primary
voltage (V)

Coils in
secondary

Secondary
voltage (V)

100
320
50000
25

6
240
393

200
66.7
30500

12
50
240

Current in
primary
coil (A)

Voltage in
primary
coil (V)

Current in
secondary
coil (A)

Voltage in
secondary
coil (V)

5
0.1
0.5

6
240
200

0.125
2
0.1

240
12
1000

Extended questions: 10 a up b F = BIlsin = 5 104 N


11 b 2 104 N c 2.5 103 Nm

Module 3 From Ideas to Implementation


Chapter 8
Checkpoints: 8.2 2 0.86NC1 8.3

2 9.6 1019N

24 hyperbolic, no 26 a 2.221011J b 2.451011 J

Chapter 3

Chapter 9

Review questions: 1 D 13 1.73ms2 16 a 80kg b 121 kg


17 0.781c, 5.60 years 18 1.641013J 19 6.91 MeV
20 6.31013 J 21 v = 0.994c

Module 1 review
Multiple choice: 1 B 2 D 3 A 4 A 5 C

Review questions: 19 3.0 1019 J 20 b 0.25 1015 Hz


c 1.66 1019 J f 6.4 1034 J s

Chapter 10
Review questions: 14 a 3.06 1019 J b 3.27 1015
15 1.09 106 m 16 5.94 1020 J = 0.37 eV

Short response: 7 2.361010 J 8 0.9679c


10 greater by 1.08108 kg 11 g at equator

Chapter 11

Module 2 Motors and Generators

Module 3 review

Chapter 4
Checkpoints: 4.3 1 at 90 to each other
2 F B, F I, F l, F sin
Review questions: 13 4.3 17 3N 18 b gradient = B/F = 2.47
c 1/gradient = 0.4, Il = 0.4, 100%

Chapter 5
There are no numerical answers for this chapter.

452

Turns
ratio

Multiple choice: 1 A 2 D 3 A 4 C 5 B

Review questions: 12 2 10 NC 13 0.3J 14 a 100q J


b 1.0 103N c q 103N d 1.6 1016N
15 EK = q E s 16 1.76 1014 m s2
17 1.88 106ms1 18 5.1 1014 N
19 a 0.22 m b 3.4 1012 N 20 9.09 1031 kg.

1.491010 J 1.501010 J

Step-up
Step-down
Step-down

Module 2 review

Circular semi1.5041010 J 2.991010 J


synchronous

Step-up or
step-down

Review questions: 19 2.53 106 m 20 a 75 K, 90 K


Multiple choice: 1 C 2 C 3 A 4 B 5 B
Extended response: 12 a 4.5 1014 Hz b 3.0 1019 J

0.025
20
0.2

Numerical answers

Module 4 Quanta to Quarks


Chapter 12
Checkpoints: 12.4 5 6.6 107 m 6 Balmer series
Review questions: 13 a 9.61 1019 J b 1.45 1015 Hz
c 2.07 107 m 14 a 0.85 eV b 12.75 eV c 3.08 1015 Hz
d 9.73 108 m e Lyman series 15 a 1.22 107 m
c n=2 and n=1 d 1.22 107 m or 122 nm e Lyman series
17 a n=1 to n=5 b n=5 to n=1 c n=3, n=4, n=5 to
n=2 d n=5 to n=4 18 b 9.38 108 m or 94 nm c ni=4
and nf=2 d 4.86 107 m or 486 nm
19 a ni=6 c Balmer series 20 c 2.09 1018 J
d 9.50 108 m

Chapter 13
Checkpoints: 13.2 2 2.4 109 m 3 6.56 1025 kg m s1
13.5 2 1.93 1010 m
Review questions: 12 b 2.4 1011 m 13 a 7.3 104 m s1
14 a i 1.2 109 m ii 6.6 1013 m 15 2.87 105 m s1
16 a i 4 1036 m ii 3 1034 m iii 3 1038 m
17 a + infinity b no 18 a 1.1 m

Chapter 14
Checkpoints: 14.2 1 a factor of ~ 1035 greater
14.6 2 5.2 1027 kg 3 280 MeV
Review questions: 14 a Fe = 231 N b Fg = 5.6 1034 N
15 0.034348 amu 16 a 28.4MeV b 0.03049amu
c 4.0026 amu 17 c 8.9 MeV per nucleon
18 a 102
(yttrium) b Uranium 7.6 MeV/nucleon,
39 Y
iodine 8.5 MeV/nucleon, yttrium 8.6 MeV/nucleon
1
0n

65
66
23
+ 29
19Cu
a a=29
b
Na +
11Cu

22
10 Ne

0
20 +1
17.406

e + v MeV

Chapter 15

Chapter 19
Review questions:
17 a U-238 Th-234:

Th-234 Pa-234:

Pa-234 U-234:

U-234 Th-230:

Th-230 Ra-226:

Ra-226 Rn-222:

Rn-222 Po-218:

Po-218 Pb-214: , Po-218 At-218:

At-218 Bi-214:

Pb-214 Bi-214:

Bi-214 Tl-210: , Bi-214 Po-214:

TI-210 Pb-210:

Po-214 Pb-210:

Pb-210 Bi-210:

Bi-210 Po-210:

Po-210 Pb-206:
18 a 3 years

Chapter 20
Review questions: 18 6 19 1.41 1026 J 20 2.8 1010 Hz

Module 5 review
Multiple choice: 1 C 2 B 3 B 4 B 5 B

Module 6 Astrophysics
Chapter 21
Checkpoints: 21.3 2 40 21.5 2 0.1 arc second
Review questions: 13 14 ~0.2 ly 17 ~90 ly

Chapter 22

Multiple choice: 1 C 2 C 3 B 4 D 5 A

Review questions: 13 3.0857 1019 m 14 59 pc, 190 ly


+ 6.6
pc
15 59 pc, 34.5
5.1

16 a 656.3 nm, 486.2 nm, 434.1 nm, 410.2 nm,
397.0 nm 17 ~5 1010 20 d 200 pc

Short response: 1 b 1.03107 m or 103 nm

Chapter 23

There are no numerical answers for this chapter.

Module 4 review

Module 5 Medical Physics


Chapter 16
Checkpoints: 16.4 3 40%
Review questions: 22 8.1610 6 rayl 23 1039.7kgm3
24 0.034%

Chapter 17
There are no numerical answers for this chapter.

Chapter 18
Review questions: 9 26.7 10 a 43.98 b 67.5
11 n2 = 1.33, water

Review questions: 13 3 AU from the centre of the red giant


15 a 6.16 1030 kg (~3MSun)
b m1 = 4.14 1030 kg 2.1 MSun, m2 = 2.02 1030 kg 1.0 MSun
16 T = 1.5 days; amplitude (main eclipse) = 0.6 magnitudes,
(secondary eclipse) = 0.5 magnitudes 17 360 pc or ~1170 ly
18 d 220 pc or ~720 ly

Chapter 24
Review questions: 16 a 3.66 1038 b 6.11 1011 kg
c 72 billion years d ~6 times the expected lifetime of the Sun

Module 6 review
Multiple choice: 1 C 2 D 3 B 4 D 5 C
Short response: 8 a 1.4 b star B

Module 7 Skills
Chapter 25
Checkpoints: 25.1 1 a 450 mg (milligrams) b 0.345 kg
(kilograms) c 5.6 Mg (megagrams) 2 0.0045 Mm
3 0.00000000000054 km
25.2 2 300km above surface

T 2
25.6 1
g = L 2 s and t2
2

Chapter 26

There are no numerical answers for this chapter.

453

Numerical answers

Glossary

2D real-time scan a scan in which a convex array transducer


launches a rapid series of beams along different directions (sector
scan) to build up a fan-like two-dimensional image of B-mode
scans in real time
3D ultrasound a rapid series of sector scans are performed
(oriented over a range of angles) to yield a three-dimensional
image

alternating current (AC) electric current that changes direction


periodically
A-mode scan an ultrasound scan in which the wavefront is
launched in a single direction, yielding a one-dimensional
oscilloscope plot of echo intensity versus depth data
angular resolution resolution that describes the ability to resolve
details separated by very small angles in the sky

4D ultrasound 3D ultrasound images are acquired so rapidly that a


series of three-dimensional images can be displayed as a movie

anode in physics, the positive electrode. (Caution: The definition in


chemistry is more complicated.)

ablation excess heat is carried away from a heat shield by material in


the heat shield melting or vaporising

anode glow small luminous region in a discharge tube adjacent to


the anode

absolute magnitude the apparent brightness of an astronomical


object (usually a star) as it would appear in our sky if it were
moved to a standard distance of 10pc (32.6ly), measured in
magnitudes and often represented as M

anti-nodes the points on a standing wave where the amplitude is


maximum

absorption line spectrum a spectrum with dark emission lines at


wavelengths characteristic of the elements present in the light
source, usually produced in astrophysical situations by a source of
a continuous spectrum source viewed through a cool, low-density
gas; most stars produce an absorption line spectrum
acceptor energy level an extra energy level is created in the energy
gap near the valence band of a semiconductor as a result of the
introduction of acceptor impurities
acceptor impurities produce a deficiency of electrons in the crystal
lattice of a semiconductor

antiparallel when one vector points in the opposite direction to


another
antiparticle a particle with opposite properties to another particle;
for example, a positron is the antiparticle of an electron
aphelion in a satellites orbit around the Sun, the farthest position
from the Sun
apoapsis in a two-body system, the farthest position of a satellite
from the central mass
apogee in a satellites orbit around the Sun, the farthest position
from the Earth

accretion disc a disc of material around a compact object


(e.g. a protostar or a black hole) formed from infalling gas

apparent magnitude the apparent brightness of an astronomical


object as it appears in our sky, measured in magnitudes and often
represented as m

accuracy an accurate measurement is one with small systematic


error

armature the part of a motor or generator that contains the main


current-carrying coils or windings

acoustic impedance is a measure of how easily a medium oscillates


in response to a sound. It is given by the formula Z=v where
is density of the medium and v is the velocity of sound in the
medium

Aston dark space the non-luminous region nearest the cathode in a


discharge tube

active optics a system to detect and overcome the slowly changing


effects of gravity and temperature that distort the mirror or
structure of a telescope and degrade the image quality

astrometry the measurement of the positions and motions of


astronomical objects

adaptive optics a system to detect and overcome the rapidly


changing effects of turbulence in the Earths atmosphere (seeing)
that are apparent as motion and blurring of the image produced by
a telescope
aether drag the hypothesised tendency for aether to be trapped and
dragged along by mountains and valleys on the surface of the
rotating, orbiting Earth
aether wind the movement of the hypothetical aether relative to the
surface of the rotating, orbiting Earth; also called aether drift.
Airy disc the diffraction pattern caused by light passing through a
circular aperture such as the objective lens or mirror of a telescope
alpha decay a form of radioactive decay in which an atomic nucleus
emits an alpha particle
alpha particle a highly energetic helium nucleus (two protons plus
two neutrons) produced by radioactive decay
454

astrometric binary stellar system that reveals its binary nature by


the wobbling of its path across the sky

astronomical unit approximately the average distance between the


Earth and the Sun: 1 AU 1.49601011m
asymptotic giant branch the portion of the evolutionary path of a
red giant in which the star has a core of carbon and oxygen
surrounded by shells of helium and hydrogen fusion
atomic mass is the mass of an atom made up from the total mass of
the protons, neutrons and electrons
atomic mass unit (amu) it is equal to the mass of one twelfth of an
atom of Carbon-12, which is approximately the mass of a proton
or neutron
atomic number the number of protons in a nucleus
atomic pile the name given to a type of nuclear fission reactor that
was constructed using a pile of graphite bricks
back emf an emf that is produced in accordance with Faradays law
within the coils of a motor and opposes supply emf

Glossary

ballistic trajectory the trajectory of a projectile subject only to


gravity and air resistance
ballistics the science of projectile motion

Bragg law mathematical relationship between wavelength, slit


spacing and wavelength for constructive interference of
electromagnetic radiation from a diffraction grating

Balmer series a set of hydrogen spectral lines containing all the


visible spectral lines that are produced when an electron transitions
to the 2nd energy level or orbital (n=2)

Bremsstrahlung braking radiation; X-rays with a broad


wavelength range that result from the conversion of kinetic energy
of rapidly braking high-speed electrons directly into X-ray photons

band gap same as energy gap

brightness is a measure of the energy received in a certain time per


unit of collecting area of a source (e.g. a star)or power per unit
areaW m2
carbonnitrogenoxygen cycle the chain of reactions that
dominates the conversion of hydrogen into helium in the hot cores
of stars of higher mass than the Sun

baryon a family of particles that are made up of three quarks


(protons and neutrons are both baryons)
base part of a bipolar transistor
BCS theory microscopic theory of superconductivity developed
Bardeen, Cooper and Schrieffer
beam splitter a partially silvered mirror that allows a fraction of
a light beam to pass through and another fraction to be reflected
beta decay a form of radioactive decay in which an atomic nucleus
emits a beta particle
beta particle an electron ejected from the nucleus of a radioactive
nuclide
binary star a system of two stars orbiting their common centre
of mass
biopsy a procedure in which a tissue sample is obtained for
medical tests
bipolar transistor a semiconductor device used to finely control
the flow of electric current
black body an idealised example of a hot object that produces a
continuous spectrum accurately described by a black body curve;
an object that absorbs or transmits all the wavelengths of the
electromagnetic spectrum
black body curve (or Planck curve) the distribution of light versus
wavelength produced by a black body, an idealised example of
a hot object; the shape, peak wavelength and intensity depend
simply on the temperature of the object
black hole a remnant of a supernova explosion with a mass greater
than about three solar masses, and a gravitational field so strong
that even light cannot escape
blanking the blocking of the electron beam, in a cathode ray
oscilloscope, on its way back to the left of the screen so that it does
not retrace over itself
B-mode scan or brightness mode scan; similar method to an
A-mode scan except that the echo intensity versus depth data are
represented as a one-dimensional line of pixels, with pixel intensity
proportional to echo intensity, yielding a one-dimensional slice of
an image
bone density or bone mineral density, is related to the mass per
unit volume of bone and depends on its mineral (calcium)
content. The lower the calcium content, the lower the bone
density and so the weaker the bone
bone scan a diagnostic imaging technique in which a patient is
given technetium-99m methylene diphosphonate, which
accumulates preferentially in excessively active bone and can
indicate the presence of cancer or other diseases
boson a family of particles within the Standard Model and includes
photons, gluons and the proposed Higgs particle

CAT (computed axial tomography) a form of imaging in which


multiple X-ray radiographs (projections) are collected at different
angles through a patient, from which a computer reconstructs a
3D stack of slices of the patient or tomogram; also known as CT
cataclysmic variable a binary system in which one star is so close to
its white dwarf companion that it pours mass onto the white
dwarf, causing the systems light output to vary dramatically as the
system suffers one or more outbursts
cathode in physics, the negative electrode (Caution: The definition
in chemistry is more complicated.)
cathode glow first luminous region near the cathode in a discharge
tube
cathode ray tube glass tube that contains two electrodes (an anode
and cathode) that produce electron beams
cathode rays collimated beams of electrons in an evacuated vessel
central body a large mass around which other much smaller masses
orbit, via gravitational attraction
centre of mass the balance point that always lies between the two
stars about which each of them can be considered to orbit
Cepheid variables periodic variables that vary between 0.5 and 2
magnitudes with periods from 1 to 70 days. They are yellow
supergiants that are important distance indicators to nearby
galaxies
characteristic X-rays X-rays of sharply defined wavelengths that are
characteristic of each individual chemical element and are given off
when a target is bombarded by accelerated electrons, resulting
from rearrangements of electrons in the inner shells of target atoms
cladding in a clad optical fibre, the lower index coating that
protects the core from damage and ensures total internal reflection
classical theory electromagnetic wave theory used to calculate the
mathematical relationship for the black body radiation curve
closed or stable orbit an orbit that repeats indefinitely. Its twobody mechanical energy is negative
coherence length the nominal separation between Cooper pairs
coherent fibre bundle a bundle of optical fibres in which the order
of fibres is the same at each end, ensuring that an image projected
on one end is transmitted accurately to the other
coils loops of current-carrying wire in motors, generators and
transformers
collector part of a bipolar transistor
collimate to make a beam of radiation more focused or parallel by
using opaque barriers to remove components of the radiation
travelling at the wrong angle
455

Glossary

collimator any kind of cylindrical, conical or planar barrier used to


collimate radiation that cant be focused using mirrors or lenses

critical field magnetic field that can destroy the superconducting


state

colour Doppler imaging a sonogram employing the Doppler effect


in which the colour of the pixel represents the velocity of the tissue
being observed

critical mass the minimum mass of nuclear fuel which is required to


produce sufficient neutrons to cause a sustained fission reaction

colour index the difference in the brightness of a star in


magnitudes when measured through two different filters,
indicating the colour of the star. The B V or mB mV colour
index is most commonly used
commutator brushes conducting contacts that connect the
commutator to the external circuit in an electric motor
commutator slip-ring a cylindrical metal contact in a simple AC
motor or generator that provides a continuous connection between
the coils and the external circuit
commutator split-ring a segmented cylindrical metal contact in a
simple DC motor or generator that reverses the connection
between the coils and the external circuit
conduction band range of energy levels of free electrons in a solid
conduction level energy a valence electron must acquire to become
free and contribute to electrical conduction
constructive interference two identical waves combine to produce
a wave of greater amplitude when their crests overlap
continuous spectrum a spectrum with light of all wavelengths,
usually produced in astrophysical situations by a hot, dense gas
contrast agent substance that when injected into a patient increases
the contrast for target organs or tissues in medical imaging
techniques such as X-ray, CAT or MRI
control rods in a nuclear fission reactor the control rods absorb
neutrons and are adjusted so that the chain reaction proceeds at a
constant rate
controlled nuclear reaction the conditions for a controlled nuclear
chain reaction are that the available neutrons which cause the
fission are regulated, resulting in a constant rate of nuclear
reactions
conventional current the conventional direction assigned to a
current is the direction of motion of positive particles, opposite to
the actual flow of electrons
convex array transducer a piezoelectric transducer with a convex
front face; the most commonly used shape of transducer for
medical ultrasound imaging
coolant a substance, commonly water, used to transfer heat energy
away from the core of a nuclear reactor
Cooper pair the temporary binding (or coupling) of two electrons
in a superconductor mediated by a lattice deformation (phonon)

critical temperature characteristic temperature below which


superconductivity occurs
CRO cathode ray oscilloscope
Crookes dark space the second non-luminous region near the
cathode of a discharge tube separated from the Aston dark space
by the cathode glow
crystal a three-dimensional regular arrangement of atoms in a solid
current (I) the net flow of charges through a region per unit time
in amperes
current-carrying conductor a conducting material (typically
copper wire) through which an electric current is flowing.
current-carrying loop a circular coil of wire through which an
electric current is flowing
cut-off frequency the frequency of electromagnetic radiation below
which no electrons are emitted from a photocathode
cyclotron is a type of particle accelerator that accelerates particles in
a spiral trajectory. For medical purposes, the cyclotron produces
accelerated protons that can convert non-radioactive nuclei into
short-lived radioisotopes, usually emitters
cyclotron motion circular motion of a charged particle at right
angles to the direction of the magnetic field
dark nebulae a portion of the interstellar medium where the dust
scatters starlight, reddening or completely blocking our view of
background stars
decay series a map or sequence of nuclear transformations from one
element or isotope to another
density mass of a material per unit volume
depletion region the region in the centre of a pn junction that
is depleted of charge due to the recombination of electrons
with holes
destructive interference two identical waves will cancel to produce
a resulting wave of zero amplitude when the crest of one wave
coincides with the trough of the other
diffraction the tendency of wavefronts to bend around the edges of
small obstacles, instead of casting a sharp shadow
diffraction grating an optical surface, often reflective, that uses fine
lines ruled onto its surface to disperse visible light into its
component colours using diffraction and interference
diffraction pattern interference pattern from a diffraction grating

core in a clad optical fibre, the higher index central fibre that carries
the light

diffusion movement of free particles from places of high to low


concentration

core in a nuclear fission reactor the core houses the fuel rods,
control rods, a coolant system and a moderator material; it is
where fission occurs

diode a device that allows current to travel in only one direction

cosmic rays extremely energetic particles that originate from outer


space

discharge tubes glass tubes in which there is conduction as a result


of ionisation of the contained gas initiated by electron motion
between imbedded electrodes

critical angle the smallest angle of incidence (approaching a


boundary with a lower refractive index material) that will allow
some of the incident light to pass through the boundary instead of
being reflected by it
456

direct current (DC) an electric current that travels in only one


direction along a conductor

distance modulus the difference (m M) between the apparent


magnitude m of an object and its absolute magnitude M; related to
its distance

Glossary

donor energy level an extra energy level created in the energy gap
just below the conduction band of a semiconductor as a result of
the introduction of donor impurities

electronics method of finely controlling the flow of electrical


current

donor impurities produce unbonded electrons in a semiconductor

emf a measure of the strength of a source of electrical potential


energy that produces a separation of charge; typically used to
describe the energy per unit charge that produces a potential
difference in an open circuit

doping the introduction of impurities into the crystal lattice of


a solid
Doppler effect in which the frequency (perceived by an observer)
of any kind of wave (including sound and light) is changed by the
motion of the wave source (or a reflecting surface). Increased
frequency means the observer and source are approaching;
decreased frequency means they are moving apart

ellipse a compressed or stretched circle

emission line spectrum a spectrum that shows bright emission lines


at wavelengths characteristic of the elements present in the light
source; usually produced in astrophysical situations by a hot, lowdensity gas

Doppler ultrasound a form of sonogram in which detection of


a change in reflected ultrasound frequency (due to the Doppler
effect) is used to deduce the velocity of a tissue, usually blood

emission nebulae a portion of the interstellar medium where the


gas is energised by hot young stars to emit an emission spectrum

drag the resistive force exerted on an object moving through any


fluid (for example air or water). In air, it is also known as air
resistance

endoscope a device employing optical fibres and/or small cameras


to collect images of internal organs with minimal surgical invasion

drain one of the three components of a field effect transistor


dual X-ray absorptiometry imaging technique that compares the
X-ray absorption by a specimen using X-rays of two different
energies, making it more sensitive to differences in bone density
than a single energy X-ray
dwarf novae a small outburst, repeating semi-regularly, produced
by instabilities in the flow of gas from a Sun-like star onto a close
white dwarf companion; classified as a non-periodic intrinsic
variable star
dynodes electrodes in a photomultiplier tube that are used to
increase the number of electrons emitted from a photocathode
eccentricity a measure of how far the position of the centre of
a satellites orbit lies from the central mass. A measure of how
elliptical an orbit is; a circle has zero eccentricity
echocardiography a form of colour Doppler ultrasound used to
diagnose conditions of the heart
eclipsing binary binary systems in which the stars are close
together and orientated so that the orbital plane is close to edgeon, causing the stars to regularly eclipse one another, periodically
blocking of some of the light from the system
eddy current circulating current in a conducting material caused by
a changing magnetic field
eddy current braking a braking system that utilises eddy currents
to slow a moving object
effectively weightless the effect of being in free-fall or orbit. The
only significant force acting on a body is gravity. Because there are
no other opposing forces acting, the body experiences no
compression or tension and so behaves like a truly weightless
object in an inertial reference frame
electromagnet a magnet in which the magnetic field is produced
by the flow of electric current
electromagnetic induction the generation of an electric current
by a changing magnetic field
electromagnetic wave consists of oscillating electric and magnetic
fields at right angles to each other and travelling at the speed
of light
electron volt an atomic-sized energy unit; 1 eV is the energy gained
by an electron when accelerated through a potential of one volt
(1 eV = 1.6 1019 J)

emitter part of a bipolar transistor

endoscopy procedures involving an endoscope


energy gap range of forbidden energy levels between the valence
and conduction bands; energy is needed to destroy the
superconducting state
escape velocity the minimum velocity required for a projectile to
permanently escape the gravitational field of an astronomical body
exclusion principle the Pauli exclusion principle was formulated by
Pauli in 1925 and states that no two fermions can possess the same
quantum numbers in the same system; explains the energy states
and orbitals in the atom
exhaust velocity the velocity of the exhaust gas as it leaves the
rocket nozzle
extrinsic semiconductor one in which conduction is dominated by
donor or acceptor impurities
extrinsic variables variable stars whose variation in brightness is due
to a process external to the body of the star itself, for example
eclipsing binaries or stars that vary because of their rotation
Faraday dark space a non-luminous region in a discharge tube
between the negative glow and the positive column
Faradays law the induced emf in a coil is proportional to the
product of the number of turns and the rate at which the magnetic
field changes within the turns
fictitious forces apparent forces used for convenience to explain
apparent changes in velocity when a system is observed from
within a non-inertial (accelerating) frame of reference
field of view the area of object (e.g. the sky) you can see at any
moment with, for example, a telescope
field-effect transistor uses an electric field to control the flow
of current through a conducting channel in a semiconductor
filter a slab or sheet of material designed to selectively remove
certain wavelengths, frequencies or energies from a beam of
radiation
fission the splitting of an atomic nucleus into two or more
fragments
flare star a faint red dwarf that flares in visible brightness because of
solar-like flares on the surface; classified as a non-periodic intrinsic
variable star

457

Glossary

fluorescence the phenomenon by which a material absorbs photons


of one energy (frequency) and immediately re-emits photons of
lower energy (frequency)

gravity and acceleration. It is the ratio of the effective weight to


true weight on Earth. g-force is measured in units of Earth
gravity g

fluoroscope a traditional apparatus that produces real-time moving


X-ray images by exposing a fluorescent screen to a broad beam of
X-rays passing through a patient or specimen. Modern digital
versions use electronic detectors in place of the screen

giant molecular clouds a portion of the interstellar medium where


the gas is most dense; simple molecules can form and star
formation occurs

flux leakage loss of magnetic flux as it passes from the primary coil
through the secondary coil, resulting in loss of induction in the
secondary coil and, hence, loss of energy

gimbal a pivoted frame that allows rotation around more than one
axis; common example is the mount that supports a gyroscope

flux pinning the stopping of the motion of vortices through a type


II superconductor by crystal defects and boundaries

gradient coils coils in an MRI machine that apply gradients in the


magnetic field across a patient so that variations in the detected
Larmor frequency of relaxing nuclei can be used to obtain the
position information needed to create a tomogram

focal length the distance between a lens or mirror and the image it
forms of a distant object

gravimeter a device for measuring the local value of the


gravitational field (or equivalently, the gravitational acceleration

forbidden energy gap see energy gap

gravitational field g a vector representing the direction and


strength of gravity at a point in space. The magnitude g of the
vector equals the acceleration due to gravity at that point

force (F) a push, pull or twist measured in newtons (N)


forward bias connecting the positive and negative terminals of a
power supply to the p and n sides respectively of a pn junction
fuel rod tubes filled with nuclear fuel and located in the centre of
the reactor known as the core
functional MRI a form of MRI in which the contrast agent is
designed to concentrate in parts of the brain that are active during
certain activities, making it possible to identify the functions of
different regions of the brain
fusion the joining of two nuclei to form a new nucleus
Galilean transformation the formula for transforming velocities
relative to a different frame of reference; vB(rel.toA)=vBvA
galvanometer a device used to measure the relative strength and
direction of electric current
gamma camera an array of collimated gamma detectors for imaging
patients who have been given diagnostic gamma-emitting
radiopharmaceutical
gamma decay decay involving emission of a gamma ray
gamma photons high-energy photons mostly emitted in nuclear
reactions and positron emission tomography
gamma rays a form of electromagnetic radiation (photons) with
wavelength <~10pm. This definition overlaps with X-rays, so this
term is usually reserved for photons produced by atomic nuclei
gasoline US term for petroleum or petrol
gate one of the three components of a field effect transistor
Geissler tubes sealed tubes that contain gas at low pressure and used
to produce an electric discharge
general relativity Einsteins relativity theory (see special relativity)
generalised to include non-inertial reference frames; it was the
replacement for Newtons laws of gravity
generator a device that transforms kinetic energy into electrical
potential energy
geostationary a circular, geosynchronous orbit directly over the
equator. A satellite in such an orbit will appear stationary to an
observer on Earths surface; also called a Clarke orbit
geosynchronous an orbit with the semimajor axis chosen so the
orbital period equals the rotational period of Earth
g-force an overall measure of the magnitude of internal forces and
reactions within bodies resulting from the combined effects of
458

gravitational potential energy potential energy associated with a


gravitational field
gravitationally bound a body is locked in an indefinite orbit
around a central body it is said to be gravitationally bound. Its
two-body mechanical energy is negative and its speed is never
greater than the escape velocity
gravity assist using the gravitational influence of a planet to transfer
some of the planets momentum (and kinetic energy) to a satellite
that has been deliberately steered to pass close to it
hadron a particle comprising of quarks. There are two types:
mesons, which contain two quarks (comprising a quark and antiquark pair), and baryons, which contain three quarks
half-life the time taken for half of the nuclei of a radioisotope
sample to decay
heat shield a layer of material that protects the vulnerable surfaces
of a re-entering spacecraft from air resistance-induced heating
using the principles of thermal insulation, radiative cooling and/or
ablation
helium flash the explosive ignition of helium burning in red giant
stars with masses comparable to that of the Sun
hertz the SI unit of frequency (Hz)
hole a vacancy left behind by the removal of an electron from the
valence band in a solid
horizontal branch (HB) stage of stellar evolution that immediately
follows the red giant branch in stars whose masses are similar to
the Suns. Horizontal branch stars are powered by helium fusion in
the core and by hydrogen fusion in a shell surrounding the core.
hyperbola the shape of a graph of y=k/x
hypersonic usually defined as speeds five times the speed of sound
in air
impedance matching any material or device of intermediate
impedance, connecting a material of higher impedance and one of
lower impedance in order to reduce reflections, for example gel
smeared between an ultrasound transducer and human skin; also
known as acoustic coupling
impulse change in momentum
induction cooktop an appliance for cooking which uses rapidly
changing magnetic fields to cause resistive heating in metallic
cookware

Glossary

induction motor type of AC motor in which the power is induced


in the rotating device by means of electromagnetic induction
inertial frame of reference a non-accelerating frame of reference

lift upward force on an object (moving through air) due to resulting


pressure differences above and below the object. This keeps an
aeroplane in the air

insulator a material that resists the flow of electric current

lift-off the launch of a rocket, also known as take-off

integrated circuit a thin wafer of semiconductor on which many


elements (transistors, resistors, capacitors) are combined to form
a useful circuit

light curve a plot of the changing apparent magnitude of a variable


star system versus time

interferometer an instrument that performs highly sensitive


measurements (usually of distance or time) by exploiting light
interference
interferometry splitting the light from a source (e.g. by observing
a star with two telescopes) and then combining the beams may
produce interference between the beams, provided they have
travelled the same distance. The interference effect depends on the
path of the beams and the characteristics of the light source
interstellar medium (ISM) a patchy medium of gas and dust in
the space between the stars
intrinsic semiconductor one in which conduction is dominated
by the creation of electrons and holes as a result of ambient
thermal energy
intrinsic variables variable stars whose variation in brightness is due
to physical changes in the star or the stellar system, for example
pulsating stars
invariant any quantity or relationship that remains unchanged
in moving from one inertial frame of reference (or one position)
to another
inverse square law a mathematical formula describing any physical
quantity (e.g. light intensity or strength of gravity) that decreases
in magnitude proportionally to the distance squared

lightning protector wires placed above transmission lines to


significantly reduce the chance of lightning striking the
transmission lines
light-year the distance that light covers in one year, travelling at the
speed of light: 1ly63241AU 9.4605 1015m
linear accelerator a type of particle accelerator that accelerates
particles in a straight line
linear array transducer a piezoelectric transducer with a flat front
face; it has been largely replaced by the convex array transducer for
medical ultrasound imaging
longitudinal relaxation time constant T1 a measure of the time
taken for protons to return to their normal ratio of parallel and
antiparallel configurations relative to the field, after the initial
RF pulse
Lorentz factor the factor by which the time between ticks on a
clock moving relative to the observer is observed to increase, or the
factor by which the length of a ruler moving relative to the
observer is observed to decrease
Los Alamos a US national research laboratory set up during World
War II to develop the atomic bomb
low Earth orbit orbits around Earth at an altitude lying between
(roughly) 160 and 2000km above the Earths surface

ionising radiation radiation energetic enough to generate ions

luminiferous aether a hypothetical (non-existent) medium once


supposed to be the medium required to carry light waves

irradiation a process in which a substance is exposed to a form of


radiation

luminosity is the total power output of a source (e.g. a star) in


watts

isotope atoms of an element which have nuclei with the same


number of protons but different numbers of neutrons

luminosity classes describe differences in luminosity and hence size


between stars of the same spectral class

jet a stream of material emitted from the polar regions of an accretion


disc around a compact object (e.g. a protostar or a black hole)

magnetic field a region of space that can exert a force on a compass


needle

Keplers laws the three laws that govern the motion of planets
around the Sun

magnetic field strength (B) is a measure of the magnetic field


strength per square metre measured in tesla (T) or equivalently in
webers per square metre (Wbm2)

Larmor frequency the frequency of precession of the axes of


protons aligned by a magnetic field
launch angle the angle between the horizontal and the initial
velocity vector of a projectile
law of conservation of energy states that energy cannot be created
or destroyed, only converted from one form to another
law of universal gravitation Newtons theory of gravitation
which was assumed to apply to all massive objects in the universe
length contraction the phenomenon in which the length of an
object (e.g. a ruler) moving relative to an observer is seen to
decrease
Lenzs law an induced current in a closed conducting loop will
appear in such a direction that it opposes the change that
produced it
lepton a family of elementary particles (e.g. electrons, muons and
neutrinos) in the Standard Model; do not participate in the strong
interaction

magnetic flux (B) is a measure of the amount of magnetic field


passing through a given area in webers
magnetic flux density see magnetic field strength
magnetic hysteresis method of heat production in induction
cookware in which a magnetic field is applied to and then removed
from a magnetic material and a second field in the opposite
direction is applied
magnetic moment a measure of the strength of the magnetic field
around a magnetic object
magnetic resonance imaging (MRI) a tomographic imaging
technique that detects RF radiation given off by precessing nuclei
(usually protons) aligned in a strong magnetic field
magnification a concept most commonly used with optical devices
such as telescopes to describe how much bigger an object appears
compared to the view with the unaided eye. Technically, its the
ratio of the angular size of the object with and without the
telescope
459

Glossary

magnitudes a measure of the brightness of a star in which 5


magnitudes represents a factor of 100 in brightness; brighter
objects have smaller (or negative) magnitudes

neutron star a remnant of a supernova explosion with a mass about


that of the Sun in an object typically only 10 km across

Manhattan Project the code name for the US secret atomic bomb
project during World War II

non-coherent fibre bundle a bundle of optical fibres for which the


order of fibres is not same at both ends, making it unsuitable for
imaging but suitable for remote illumination

mass number the total number of nucleons in a nucleus


mass-energy many physicists dislike the definition of relativistic
mass and prefer not to distinguish mass and energy, but rather
lump them together as a single quantity called mass-energy
massive possessing mass
massluminosity relationship a relationship between the mass M
of a main sequence star and its luminosity L that can be
approximately fitted by the relationship LM3.5
Maxwells equations four equations (in their modern versions) that
summarise all possible phenomena of electromagnetism
mechanical medium any material (e.g. air or water) through which
a mechanical wave can travel
medium Earth orbit orbit higher than a low Earth orbit but lower
than a geostationary orbit
meson a particle belonging to the hadron family; it is comprised of
one quark and one antiquark

nodes the null points on a standing wave

non-periodic variables intrinsic variable stars whose variation in


brightness repeats at irregular intervals
novae a smaller outburst than a supernova, produced when a Sunlike star leaks enough gas onto a close white dwarf companion to
generate a surface nuclear explosion; classified as a non-periodic
intrinsic variable star
n-type semiconductor a doped semiconductor that results in extra
unbonded electrons in the crystal lattice
nuclear reactor a device in which a controlled nuclear chain
reactions occurs at a sustained and steady rate. The energy can be
used to produce electricity and the neutrons emitted can be used
for industrial and medical purposes
nucleon the name given to protons and neutrons when they are
present in a nucleus
nuclides the name given to a specific isotope of an element

MichelsonMorley experiment the historical experiment designed


to measure the effect of aether wind on the speed of light

null result when a well-designed, carefully performed experiment


fails to observe an expected effect

microprocessor the main component of a computer that is at the


centre of all computer operations

optical fibre a thin, highly transparent (usually silica) fibre that


conducts light with little loss because of total internal reflection at
the outer boundary

Mira variable star a pulsating variable red giant or supergiant star


with a period of between 80 and 1000 days
M-mode scan a rapid series of B-mode scans displayed side by side
to represent echo intensity versus depth as a function of time; used
to detect organ motion

optical telescope a telescope designed to image optical (visible)


light
orbit the path an electron takes around the nucleus in classical
atomic theory

moderator a material used in a nuclear fission reactor to slow down


neutrons and so improve their chance of being captured by a
nucleus

orbital decay decrease in the orbital radius as orbital kinetic energy


is converted into thermal energy, e.g. by drag

MOSFET metaloxidesemiconductor field-effect transistor

oscilloscope a device used to measure the variation of voltage in


time across an electrical component

motor a device that converts electrical potential energy into kinetic


energy
motor effect the force experienced by a current-carrying conductor
in a magnetic field
muon a lepton; it is the heavier cousin of the electron in the
Standard Model
mutual annihilation the process in which two antiparticles meet
(e.g. electrons and positrons), their opposite properties cancelling,
leaving only their rest mass-energy, which is released in the form of
two gamma rays
nebula the portion of the interstellar medium where interaction
with starlight reveals the gas and dust
negative glow the luminous region in a discharge tube next to the
Crookes dark space and before the Faraday dark space
neutron uncharged subatomic particle
neutron scattering the neutron can be fired into sample materials
and analysis of the resulting interactions can determine the
motion, spacing, magnetic structure and inner structure of many
materials

460

orbital velocity the tangential velocity of a satellite in orbit

osteoporosis a condition of weakened bone due to low bone


density
parabola the shape of the graph of a quadratic equation
paraffin a common name for a hydrocarbon wax
parallax the apparent change in position of a nearby object relative
to a more distant background caused by a change in viewing
position, such as the apparent change in position of nearby stars
caused by the Earths orbital motion
parallel when one vector points in the same direction as another
parsec the distance of a hypothetical star that has a parallax angle of
1 arc second: 1pc206265AU 3.2616 ly 3.0857 1016 m
particle accelerator a device for accelerating charged particles
path length distance travelled by a wave
payload the cargo, instruments or passengers delivered into space
by a vehicle
periapsis in a two-body system, the position of closest approach of
a satellite to the central mass
perigee in a satellites orbit around the Earth, the position
of closest approach

Glossary

perihelion in a satellites orbit around the Sun, the position of


closest approach
periodic variables intrinsic variable stars whose variation in
brightness repeats with a regular period
periodluminosity relationship a relationship between the period
of variation of a Cepheid variable star and its luminosity L that
allows the luminosity and hence the distance of the star to be
estimated
perpendicular area the area of a wire loop perpendicular to the
magnetic field lines passing through the loop
PET positron emission tomography
phonons lattice vibrations with discrete energy; analogous to
photons
photocathode an electrode that emits electrons in an evacuated
vessel when struck by electromagnetic radiation
photocell a device that converts light to an electrical signal
photoelectric effect electron emission from the surface of metals
when irradiated with electromagnetic radiation, mostly visible or
ultraviolet

emitting isotope is used to identify the position of diseased tissue.


A tomogram is built up by deducing the original positions of the
positrons by detecting pairs of gamma rays that result from mutual
annihilation with surrounding electrons
postulate a fundamental principle assumed to be true but for which
there is no direct proof, which forms the basis upon which a
theory can be built
potential difference a measure of the difference in electrical
potential energy between two points in a circuit. This quantity is
measured in volts and is often replaced by the term voltage
power station an industrial facility for the generation of electric
power
precession the conical motion of the axis of rotation that results
when an external torque is applied to a spinning object
precision a precise measurement is one with a small random error
prefix mathematical word that can be added in place of scientific
notation
principle of relativity physical laws remain invariant in all inertial
frames of reference

photoelectrons the electrons emitted from an electrode in an


evacuated vessel when struck by electromagnetic radiation

prism a wedge-shaped glass block that disperses visible light into its
component colours using refraction within the glass

photometry the measurement of the brightness of a light source

projectile an object (e.g. a cannonball) projected through open


space

photomultiplier tube an evacuated tube with a photocathode,


anode and dynodes used to convert light to an electrical signal
photons radiant electromagnetic energy consisting of concentrated
bundles of energy; a particle of light
photovoltaic cells semiconductor devices used to convert light into
electricity
piezoelectric effect the phenomenon in which some materials
produce a voltage when squeezed and conversely become slightly
compressed (or expanded) when a suitable voltage is applied
piezoelectric transducer a transducer (often used to generate
ultrasound) composed of a material (such as PZT) that exhibits
the piezoelectric effect
pion the lightest meson
pixel abbreviation of picture element; the smallest dot represented
in a digital image
Planck curve or black body curve is the distribution of light versus
wavelength produced by a black body, an idealised example of a
hot object. The shape, peak wavelength and intensity depend
simply on the temperature of the object
Plancks constant a fundamental constant h = 6.63 1034 J s
planetary nebula a short-lived, small cloud of gas expelled by a low
mass star to reveal its core as a white dwarf star
plate part of a thermionic device used to collect the electron current
polarisation the direction of the electric field in an electromagnetic
wave

propellant material (usually combustible) used to generate exhaust


through a rocket nozzle, hence producing thrust
proper length the length of an object as measured by an observer
stationary relative to that object
proper mass see rest mass
proper motion the apparent motion of a star across the sky due to
its the transverse motion through space
proper time the time interval between two events that take place in
the same position as measured by an observer stationary relative to
those events
protonproton chain the chain of reactions that dominates the
conversion of hydrogen into helium in the relatively cool cores of
lower mass stars like the Sun
protostar the hot core of a collapsing fragment of a gas cloud,
perhaps destined to form a star
p-type semiconductor a doped semiconductor that results in a
deficiency of electrons in the crystal lattice
pulsar a neutron star that is visible because beams of radiation
produced near its magnetic poles sweep across the Earth to be seen
as a pulse as the pulsar rotates
PZT lead-zirconate-titanate, a material commonly used in
piezoelectric transducers
quanta the emission or absorption of energy in discrete units

positron an antimatter electron

quantum mechanics a set of principles that describe physical


reality at the atomic level of matter (molecules and atoms) and the
subatomic (electrons, protons and even smaller particles),
including the simultaneous wave-like and particle-like behaviour
of matter and radiation (waveparticle duality)

positron decay subset of beta decay involving emission specifically


of a positron

quantum number a set of numbers used to describe quantities


in a quantum system

positron emission tomography a form of imaging in which a


biologically accumulating substance tagged with a positron

quark a set of particles developed to explain the properties of a


family of particles called hadrons

positive column largest luminous region in a discharge tube and the


most prominent feature of the discharge; situated near the anode

461

Glossary

R Coronae Borealis star a yellow supergiant star that fades


significantly at irregular intervals as carbon-rich dust clouds
obscure the surface; classified as a non-periodic intrinsic
variable star

relativistic mass the phenomenon in which the mass of an object


moving relative to the observer is appears to increase

radiation particles or waves that propagate outwards from a source

resistive heating heating that occurs in a conductor when energy is


transferred from the moving charges of an electric current to the
atoms of the conductor

radiation shielding protects people and the environment against


excessive radiation and prolongs the working life of a nuclear
reactor facility

relaxation the process of a previously aligned system (e.g. nuclear


spin) to gradually become misaligned

radio frequency (RF) electromagnetic radiation frequencies in the


range (roughly) 3kHz to 300GHz

resonance the tendency for one object possessing a natural


frequency of oscillation to oscillate strongly in the presence of an
external source at that same frequency

radio telescope a telescope designed to collect radio wavelengths


of electromagnetic radiation

rest mass the mass of an object as measured by an observer who is


stationary relative to that object

radioactive any nucleus that undergoes radioactive decay

reverse bias connecting the negative and positive terminals of a


power supply to the p and n sides of a pn junction respectively

radioactive decay the process whereby certain unstable atomic


nuclei approach a more stable state by releasing alpha, beta or
gamma rays

RF transceiver coils the coils that both transmit the RF pulse and
receive the RF echo of relaxing nuclei in MRI

radiograph the oldest form of X-ray image; produced by exposing


film using a broad beam of X-rays passing through a patient or
specimen

rotor coils the central rotating component of a motor or generator


consisting of wire coils wound around a laminated iron frame
attached to an axle or shaft

radiographer a technician who takes medical images of patients

RR Lyrae variables pulsating variable stars, commonly found in


globular clusters, that vary up to 2 magnitudes with periods less
than 1day

radioisotopes atoms that have an unstable ratio of protons to


neutrons and will decay via alpha or beta decay to attain a more
stable configuration; may also emit gamma radiation
radiopharmaceuticals radioisotopes incorporated into compounds
that are used in medicine and can be classified into diagnostic and
therapeutic

RV Tauri star a pulsating variable yellow supergiant star with


a period of between 20 and 100 days
Rydbergs constant constant with a value of 1.097107m1 used
in the calculation of hydrogen spectral line wavelengths

radiotherapy a medical procedure in which radiation emitted


from a radioactive source is directed at an area of diseased tissue

satellite any object in orbit under the gravitational influence


of a much larger body

random error an uncertainty in a measurement governed by random


statistical fluctuations

sawtooth the waveform of the potential difference across the


horizontal plates inside the tube of a cathode ray oscilloscope

range the maximum horizontal displacement of a projectile

scintillation the rapidly changing effects of tiny, rapidly changing


temperature variations in the Earths atmosphere that are apparent
as rapid changes in the brightness of a star; commonly known as
twinkling

reaction device any device (such as a rocket) that is driven along by


the reaction force from material being expelled
real-time image when image data are collected and processed so
rapidly that the delay between an event and its displayed image is
negligible
receiver apparatus used to detect an electromagnetic wave
red giant a star that has evolved off the main sequence and grown in
size and luminosity, having largely exhausted the hydrogen fuel in
its core
re-entry the process of bringing a spacecraft back through the
atmosphere
reflecting telescope a telescope that uses a mirror as the objective
element to gather the light (not just visible light). The mirror
reflects the light and brings it to a focus
reflection nebulae a portion of the interstellar medium in which
we see the light scattered by the dust, especially at blue
wavelengths
refracting telescope a telescope that uses a lens as the objective
element to gather the light (almost always visible light). The lens
refracts the light and brings it to a focus
refraction bending of the path of a wave as it passes from one
medium to the next at an angle with respect to the surface
(interface) of these media
462

scintillator any substances that produces light flashes when struck


by ionising radiation
sector scan fan-shaped ultrasound beam that emanates from a
convex array transducer. The wavefronts of ultrasound are launched
at different angles at separate times, to avoid simultaneously
detecting confusing echoes from different directions
seeing rapidly changing effects of tiny, rapidly changing temperature
variations in the Earths atmosphere that are apparent as motion
and blurring of the image produced by a telescope
semiconductors materials used for the manufacture of modern day
electronic components
semimajor axis the distance between the apoapsis of an orbit and
the centre of the orbit
semi-regular variable star a pulsating variable red giant or
supergiant star with an irregular period of between 80 and 1000 or
more days
semi-synchronous an orbit with the semimajor axis chosen so the
orbital period equals half the rotational period of Earth
sensitivity (light-gathering power) describes the ability of a
telescope system to see faint objects; depends on how much light
the telescope collects and how much of that light is delivered to
the detector

Glossary

shaded pole induction motor an induction motor in which four


small copper shading rings are inserted into the stator on each side
of the rotor on opposite poles. The currents induced in these
shading rings act to delay the magnetic flux passing through the
rotor, producing an asymmetric magnetic field
shadow mask a metal sheet with an array of holes that is placed
behind the phosphor screen of a CRT television screen. Each hole
guides the three beams to their respective coloured phosphor as the
beams move horizontally and vertically
shock wave the wavefront of sharp pressure increase that builds up
in front of an object moving through a gas at speeds faster than the
speed of sound in the gas
simultaneity the state of being simultaneoustwo or more
phenomena taking place at the same time as seen by an observer
single-phase AC refers to the distribution of AC electric power
using a system in which only a single voltage oscillation is supplied
to the user; used for household power supply
slingshot effect see gravity assist
solid state physics branch of physics that includes the study of
properties of solid materials
sonogram an image created using ultrasound
source one of the three components of a field-effect transistor
spacetime a 4D set of coordinate axes consisting of the usual x, y
and z axes of space plus an extra axis representing time. The
equations of special relativity and Maxwells equations are
expressed in these four dimensions
special relativity Einsteins theory applied to inertial reference
frames in which the speed of light is assumed to be invariant and
the principle of relativity is assumed to apply to all physical laws.
It expands on Galileo and Newtons laws of mechanics
spectral classes groups of stars with similar spectral lines, indicating
similar surface temperatures
spectrograph an instrument that disperses light into its component
wavelengths to be recorded by a detector
spectrometer an instrument that measures the intensity of
electromagnetic radiation for a range of wavelengths
spectroscope an instrument that disperses visible light into its
component colours to be viewed by eye
spectroscope binary stellar systems that reveal their binary nature
by the motion of spectral lines in the spectrum
spectroscopic parallax a poor name for a stellar distance estimated
by identifying the type of star from the characteristics of its light
spectroscopy the study of the light from objects to reveal their
composition and physical characteristics
spectrum the pattern that results when light from a source is spread
into its component wavelengths (colours if visible light)
spin the property of subatomic particles that gives them a magnetic
moment. It is (very roughly) the microscopic equivalent of the
spin of a spinning top
squirrel cage rotor the rotor of an AC induction motor containing
parallel conducting bars around its circumference
Standard Model the current, scientifically accepted model to
describe the nature of matter
standing wave a wave that remains in a constant position. It
consists of two equivalent waves overlapping and travelling in

opposite directions so that the maximum amplitude and null


points on the wave do not move in space
stationary state a definite energy; in atomic theory relates to a
stable orbital
stator the stationary part of a motor or generator surrounding the
circumference of the rotor
step-down transformer a transformer that produces an output
voltage that is less than the input voltage
step-up transformer a transformer that produces an output voltage
that is greater than the input voltage
stopping potential the potential difference required to stop
electrons from leaving the surface of a photocathode
strong force the nuclear force holding the nucleus together
substation a subsidiary station of an electrical distribution network
where voltage is either stepped up or down using transformers
superconductivity total disappearance of electrical resistance in a
material when it is cooled to below a certain temperature
supernova a cataclysmic explosion of a star, producing a massive
outburst of light that fades over many weeks. They are sometimes
classified as non-periodic intrinsic variable stars. There are two
major types: type Iaccretion of gas onto a white dwarf from its
companion leads to a runaway nuclear explosion; type IIcollapse
of the core of a massive star
supernova remnant the cloud of gas that was expelled by a star and
made to glow by a supernova explosion
supersonic speeds greater than the speed of sound in a gas
supply emf the emf applied to a circuit
symbiotic star a close binary composed of red giant and white
dwarf stars, with irregular outbursts from the red giant falling onto
the white dwarf; classified as a non-periodic intrinsic variable star
synchrotron a type of particle accelerator
systematic error consistent error that is in every measurement
T Tauri star a very young star with an accretion disc that varies
irregularly in brightness as it approaches the main sequence;
classified as a non-periodic intrinsic variable star
telescope a device that collects electromagnetic radiation (light)
and focuses it to create an image on a detector that is brighter and
with greater spatial resolution than could be achieved using the
detector alone
test mass a small mass used (in practice or in theory) to measure the
gravitational field. Its mass should be so small as to make a
negligible contribution to the gravitational field being measured
thermionic device a cathode ray tube device used to control the
flow of electrons
three-phase AC three circuit conductors carry three alternating
currents (of the same frequency) which reach their instantaneous
peak values at different times, resulting in constant power transfer
over each cycle of the current; used by high power or industrial
machines
thrust the reaction force exerted back onto a rocket by the exhaust
gas it expels; this reaction is responsible for driving the rocket
time dilation the phenomenon in which the time between ticks on
a clock moving relative to the observer is observed to increase
time of flight the time elapsed between the launch of a projectile
and when it hits a barrier
463

Glossary

timebase the time (x) axis on an oscilloscope controlled by the


timebase dial
tomogram a 3D image in the form of a stack of 2D slices produced
by any form of tomography
tomography any imaging technique that produces tomograms
torque the turning effect (or turning moment) of a force
total internal reflection the complete reflection that occurs
when light in a higher index material meets a boundary with
a lower index material at an angle of incidence larger than the
critical angle
trace the plot of voltage against time displayed on the screen of an
oscilloscope

uncontrolled nuclear reaction chain reaction occurs when the


production of neutrons goes unchecked and the fission reactions
increase at an accelerating rate
universal gravitational constant G the constant
G=6.671011Nm2kg2 that appears in Newtons equation
for gravitational force; it is assumed to be constant throughout
the universe
valence band range of energy levels of electrons bound to an atom
valence electron bound outer electron of an atom
valence level the energy of a valence electron while being bound to
its atom
variable star a star or star system that appears to vary in brightness

transducer any device that converts energy from one form


to another

vertical and horizontal components two vectors parallel to the


vertical and horizontal directions respectively that add up to the
vector being analysed

transformer a device that alters the voltage and current of


AC electricity

visual binary a binary star system that can be seen as two stars
by a telescope under sufficiently good seeing conditions

transistor a semiconductor device used to finely control the flow of


electrical current in a circuit

voltage the SI unit for potential difference; voltage is also


commonly used to replace the term potential difference

transition the movement between two orbitals or energy states

vortices swirls of electrical current surrounding a normally


conducting region that are embedded in a type II superconductor

trajectory the path of a projectile in flight

transmission tower typically a steel tower used to support wires for


the long distance transmission of electricity
transmitter apparatus used to generate an electromagnetic wave
transmutation the process of changing one element into another
transverse relaxation time constant T2 a measure of the time taken
for the magnetic component perpendicular (transverse) to the
external field to return to zero, after the initial RF pulse
triode a thermionic device used to finely control the flow of electric
current
triple alpha process the chain of reactions that describes the
conversion of helium into carbon in the hot core of a red giant star
twin paradox a thought experiment that explores the symmetry of
time dilation by supposing one identical twin takes a journey in a
spacecraft travelling at high speed while the other stays on Earth
type I superconductor one in which the internal magnetic field
remains zero until a critical applied magnetic field strength is
reached, at which a sudden transition to the normal state occurs
type II superconductor one that has two critical magnetic field
strengths and the superconductivity can be maintained up to the
upper critical field, but there is partial penetration of the field into
the superconductor at between these fields
ultraviolet catastrophe the erroneous prediction by classical theory
that the intensity of radiation from a black body will increase
towards infinity at the ultraviolet end of the electromagnetic
spectrum
uncertainty principle the Heisenberg uncertainty principle states
that there is a limit to how precisely you can measure pairs of
quantities such as positionmomentum, and energytime and that
both cannot be known to arbitrary precision

464

voxel volume element; the smallest part of a three-dimensional


image (or tomogram)
wave function a solution to Schrodingers wave equation; the square
of this function provides you with a probability density that allows
you to predict the likelihood of finding a particle
wave mechanics commonly used interpretation of quantum
mechanics
wavefronts a line or surface joining all points of equal phase in
a wave, e.g. the circular crest of a single ripple in a pond
white dwarf the core of a low mass star revealed when the outer
layers are gently blown away, and visible as a faint star slowly
cooling off over tens of billions of years
work function energy required to just remove the electrons from
the surface of a metal
X-ray binary a binary system in which one star is so close to its
neutron star or black hole companion that it pours mass onto the
companion and emits X-rays from the infalling gas
X-rays a form of electromagnetic radiation (photons) with
wavelength 10nm
Zeeman effect the splitting of spectral lines in the presence of
a magnetic field
zero-age main sequence (ZAMS) the line on the Hertzsprung
Russell diagram where collapse stops and a newborn star is
powered entirely by nuclear reactions in its core

Index

Index
A-mode ultrasound scans 312
ablating materials 49
absolute magnitude, of stars 399
absorption spectra, lines in 232, 393
AC electric motors 1214
synchronous 1301
AC generators
compared with DC 135
simple 1313
AC generators and transformers, affect on
society 1467
AC induction motors
activity 126
single-phase 124
three-phase 1223
AC power
generation and delivery 1424
losses during transmission and
distribution 1446
accelerated particle beams, uses of 288
acceleration due to gravity, activities 201
acceptor energy level 193
accuracy in experiments 445
acoustic coupling 312
acoustic impedance (Z) 31012
active optics in telescope mirrors 3778
adaptive optics in telescopes 37980
aether drag 64
aether model for light transmission 612
agricultural radioisotopes 284
air resistance (drag) 7
Airy disc 376
Algol ( Persei) eclipsing binary star system
412
alpha decay 264, 341
alpha () particles 341
alternating current (AC) 85
electric motors 1214
amplification of currents using triodes 198
Anderson, Carl, and cosmic rays 286
angle (), in motor effect 93
ngstrm, Anders, and hydrogen spectrum
233
angular resolution of telescopes 3767
anode glow 158
ANSTO (Australian Nuclear Science and
Technology Organisation)
Bragg Institute at 272
Echidna neutron diffractometer 285
National Medical Cyclotron at Royal
Prince Alfred Hospital 282
OPAL reactor facility at Lucas Heights
272, 282, 345
radiopharmaceutical production at 353
antennae, radio 1756
apparent magnitude of stars 398
apparent weight 312
artefact standards of mass and length 79
Aston dark space in discharge tubes 158
astrometric binary stars 411

astrometry 3889
astronauts, forces on during take-off 315
astronomical unit (AU) of distance 389
asymptotic giant branch 428
ATLAS particle detector in Sydney 2901
atom
Bohrs postulates for model of 2356
Rutherfords model 22830
atomic bombs 279
atomic mass, and the neutron 261
atomic mass number (A) 262
atomic mass unit (amu) 266
atomic number (Z) 262
atomic piles 270
atomic spectra
for hydrogen 2325
for larger atoms 239
atoms, historical understanding 229
Australia Telescope Compact Array (ATCA),
for interferometry 381
Avogadro project (CSIRO) 79
B-mode ultrasound scans 313
back emf in DC electric motors 120
ballistic trajectories 3
Balmer, Johann
Bohrs explanation of series 2368
emission spectrum series for hydrogen
233
band gap (forbidden energy gap) 189
barium emission spectrum 239
barium meal X-ray procedure 325
baryons 292
BCS (Bardeen, Cooper and Schrieffer) theory
215, 217
Bessel, Friedrich, and stellar parallax motion
388
beta () particles 341
beta decay 260, 3412
positron production 342
beta (minus) and (plus) decay 264
Centauri A and B orbits 410
bias, forward and reverse 1945
binary stars 40710
Centauri 410
types of 41112
binding energy of nucleus 2689
bipolar transistors 1989
black body absorbers and emitters 178
black body radiation 1789
black holes 431, 432
blood flow and Doppler effect 31516
Bohr, Nils
explanation of Balmer series for hydrogen
2368
postulates for atomic model 2356
and Rutherfords atomic model 230
bone-density measurement using ultrasound
315
bone scans with gamma camera 3467

BOS key terms 44851


Bragg, William Henry, particle properties of
X-rays 208, 248
Bragg, William Lawrence (Sir), law of 208,
248
Bragg Institute at ANSTO OPAL reactor
facility 272
brain, MRI scans of 360, 362
brain function 303
braking using eddy currents 11213
Bremsstrahlung 323
brightness of stars see magnitudes
brushless DC motors 115
bungee jumping 441
Bunsen, Robert, and spectral lines 390
calcium emission spectrum 239
carbonnitrogenoxygen cycle 4256
Cassegrain, Laurent, telescope 372
CAT (computed axial tomography) 321
CAT scans
benefits over other methods 329
process and image construction 3278
CAT X-ray images 3269
cataclysmic variables 412
cathode glow 158
cathode ray oscilloscope (CRO) 1678
cathode ray tubes, history of 1567
cathode rays
nature of 157, 160
see also electrons
Cavendish, Henry, measuring density of
Earth 25
CDs and DVDs
as diffraction gratings 206
as spectrometers 155
Centaurus-A galaxy images 382
centripetal force 36
Cepheid variable stars 41517
Chadwick, James, discovers neutron 2601
chain reaction game 227
chain reactions 2701
characteristic X-rays 323
charge to mass ratio of electron 166
charged particles
in electric fields 1603
forces on in magnetic fields 89, 1645
circular orbits 412, 57
classical theory approach to black body
radiation 1789
closed orbits 41
coherence length between Cooper pairs 215,
217
coherent bundles of optical fibres 3356
coil, torque on 11617
Colour Doppler imaging 315
colour index of stars 4001
compact fluorescent lights 173
Compte, August, knowledge of stellar objects
38990

465

Index

computed axial tomography see CAT


conduction bands 18990
conduction level 189
conic sections as orbits 42
conservation of energy, law of 105
continuous spectra 390, 392
contrast agents in X-ray imaging 325
control rods in nuclear reactors 281
conventional current 85
convex array transducers 3089
coolant in nuclear reactors 281
Coolidge, William, X-ray tube 320
Coolidge X-ray tubes 3212
Cooper pairs 215, 217
core losses, in transformers 140
core of nuclear reactor 280
Cormack, Allan McLeod, and tomography
321
cosmic rays 286
critical angle in optical fibres 334
critical field strengths (Bc1 and Bc2) 213
critical mass 271
critical temperature (Tc) for superconductors
211
Crookes, William (Sir), and cathode rays 157
Crookes dark space, in discharge tubes 158
Crookes magnetic deflection tube 90
crystal structure, and X-ray diffraction
gratings 2089
crystal structure of metals, and electrical
conductivity 20910
crystals, plane spacing (d) in 209
Curie, Jacques, and piezoelectric effect 308
Curie, Pierre, and piezoelectric effect 308
current amplification by triode 198
current-carrying loop (coil), torque on
11617
current (I) in motor effect 93
currents, electric 848
cut-off frequency (f0) in photoelectricity 182
cyclotron motion 164
cyclotrons 287
producing radioisotopes in 282, 345
dark nebulae 422
Davisson, Clinton, and electron waves 251
Davy, Humphrey (Sir), mentor of Faraday
100
DC electric motors 11420
back emf in 120
DC generators
compared with AC 135
simple 1334
de Broglie, Louis (Prince)
hypothesis confirmed 2512
matter wave equation 24850
De Forest, Lee, invents triode 198
decay series for uranium-238 265
deceleration during space shuttle re-entry 48
degenerate electron pressure 429
depletion regions 194
diffraction
of waves 2501
of X-rays 207

466

diffraction angle () 209


diffraction gratings 2067, 391
diffusion of charges, in semiconductors 194
diodes (pn junctions) 1935
direct current (DC) 85
electric motors 11420
transmission at high voltage 146
discharge tubes, structure of 1589
Discovery space shuttle, re-entry problems 49
distance modulus for stars 399, 400
donor energy level 192
doping of semiconductors 1913
Doppler shift () of spectral lines 396
Doppler ultrasound imaging 306
for blood flow 31516
drag (air resistance) 7
dual X-ray absorptiometry (DXA or DEXA)
315
dynodes 184
Earth, weighing of 25
Echidna (high-resolution neutron powder
diffractometer) at OPAL 285
echocardiography 316
eclipsing binary stars 412
eddy currents 1068
braking in trains and roller-coasters
11213
in induction cooking 108
losses in transmission lines due to 145
resistive heating due to 140
Edison, Thomas
and DC electricity 141
and incandescent lamps 198
Einstein, Alfred
and photoelectric effect 1823
principle of relativity 589
proposal of photons 179, 181
special theory of relativity 648
electric current 848
control of 197200
electric field lines 161
electric field strength (E) 1601
between parallel plates 1613
electric motors
activity 126
alternating current 1214
characteristics of different types 125
compared with generators 135
direct current 11420
home-made 83
universal 122
electric power (P)
generation and delivery 1424
losses during transmission and
distribution 1446
and resistance 86
in transformers 138
electrical conduction and energy bands
18990
electrical conductivity of metals, and crystal
structure 20910
electrical resistance see resistance, electrical
electricity supply network 143

electromagnetic induction 1003


without relative motion 103
see also induction cooking; induction
motors
electromagnetic wave emitter energy 180
electromagnetic wave theory, Maxwells
1745
electromagnetism, exploring 136
electron capture 265
electron orbits, as standing waves 2523
electron spin 217
electron volt (eV) 190, 343
electrons
charge to mass ratio 166
de Broglies hypothesis of wave nature
24852
electrons (cathode rays) 1656
electrostatic particle accelerators 286
elliptical orbits
deductions from perturbations of 40
of Halleys Comet 39
Keplers laws for 379
properties 37
emf () 85, 102
in DC electric motors 120
emission nebulae 422
emission spectra
for emission nebulae, normal galaxies and
quasars 394
lines in 232, 3923
endoscopy 333, 335
medical uses 3367
energy, equivalence with mass 724
energy band diagrams 18990
energy gap 18990, 217
environment, effect of widespread electricity
generation 147
equal areas, Keplers second law of 389
escape velocity (ve) 1819
Esnault-Pelterie, Robert (REP), space trip
calculations 27
evolution of stars 4257
excitation of hydrogen proton by RF pulses
357
exclusion principle, Paulis 255
experimental errors 4456
extremely large telescopes (ELTs) 382
extrinsic and intrinsic semiconductors 193
extrinsic variable stars 414
Faraday, Michael
dark space in discharge tubes 158
law of 1003
Fermi, Enrico
and controlled nuclear reactions 270
and transuranic elements 269
Fermilab Tevatron 288
fictitious forces 59, 60
field-effect transistors (FETs) 200
fission, nuclear 269
fission reactors 2801
Fitzgerald, George, and aether 63
Fleming, John Ambrose, builds first diode
198

Index

flight time of projectile 6


fluorescence, in discharge tubes 1567
fluorescent lights 173
flux leakage, in transformers 139
flux pinning, in magnetic levitation 214
focal length of lens 3712
food irradiation 284
forbidden energy gap (band gap) 189
force carriers 292
formulae, linearising 447
forward bias in pn junctions 1945
forward-biased LEDs 195
4D ultrasound 314
frames of reference, inertial and non-inertial
589
Fraunhofer, Joseph von, lines in spectrum of
Sun 390
Friedman, Jerome, and quarks 292
Fritsch, Otto, and radioactive isotopes 269
fuel rods in nuclear reactors 280
functional MRI images 362
fundamental physical property standards of
mass and length 79
fusion bombs 280
g-force 314
galactic recycling system 423
galaxies (normal), emission spectra for 394
Galilean transformation 5
Galileo Galilei 45
and telescope 368, 3702
Galileo space probe, use of gravity assist 45
galvanometers 119
gamma camera 3467
gamma photons 184
gamma radiation 265
gamma ray emitters 3423
Crucis, distance of 402
gastroscopy see endoscopy
Geiger, Johannes, and model of atom 22930
Geissler, Heinrich, and vacuum pump for
discharge tubes 1567
Geissler tubes 156
Gell-Mann, Murray, and quarks 292
generators, electricity 1305
compared with motors 135
comparing AC and DC 135
geographic poles 88
geostationary satellites 43
geosynchronous satellites 43
Germer, Lester, and electron waves 251
giant molecular clouds 423
Glashow, Sheldon, and electro-weak theory
292
global positioning system (GPS)
receivers 67
satellites 434
globular star clusters 4334
Goddard, Robert, US rocket physicist 27
gradient coils in MRI scanners 361
gravitational fields
variations in 1415
weight in 1314
gravitational potential energy (GPE) 1619

gravity 1015
activity 20
effect on orbits 3540
gravity assist (slingshot effect) 445
hadrons, properties 294
Hahn, Otto
aand radioactive isotopes 269
and beta decay 265
half-life of radioisotopes 282, 343
Halley, Edmund, comet of 39
Harriot, Thomas, and telescope 368
heart imaging by echocardiography 316
heat shields 49
heating during space shuttle re-entry 4850
Heaviside, Oliver, and Maxwells equations
61
Heisenberg, Werner, uncertainty principle
of 2545
helium flash 428
Herschel, William, and reflector telescope
373
Hertz, Heinrich
and cathode rays 157
discovers photoelectric effect 182
measures speed of radio waves 61, 1757
verifies electromagnetic wave theory
1745
HertzsprungRussell (HR) diagram 3956
activity on star clusters 435
and distance of Crucis 402
life of star on main sequence 427
showing evolutionary tracks for protostars
424
showing variable stars 414
for star clusters 434
Higgs particle 291
high-mass stars, evolution of 4301
high-temperature superconductors 216
high-voltage DC (HVDC) transmission 146
Hipparchus, and star magnitudes 397
Hohmann transfer orbits 57
holes in valence bands 191
hollow structures, X-ray images of 325
homopolar electric motor 83
Hooke, Robert, law of 15
Hounsfield, Godfrey, and tomography 321,
327
hydrogen atom
Balmer series for 233
emission spectrum 239
spectra and energy levels of 2325
hydrogen bomb 280
hydrogen proton
as energy carrier 285
precession of 356
spin in magnetic field 3556
subjected to RF pulses 3579
hyperbolic orbits 412
hypersonic flight 489
impedance matching 312
induction cooking 108
see also electromagnetic induction

induction motors
single-phase AC 124
see also electromagnetic induction
industrial radioisotopes 283
inertial frames of reference 589
inertial and non-inertial frames of reference
activity 75
infra-red light 197
insulators
energy band diagrams for 18990
in high-voltage AC power transmission
1456
integrated circuits (ICs) 200
interference, constructive and destructive 205
interferometry
in MichelsonMorley experiment 624
in radio telescopes 3801
International Thermonuclear Experimental
Reactor (ITER), superconductors in 218
interstellar medium (ISM) 4223
intrinsic variable stars 414
inverse square law 11
isotopes
of atoms 2623, 341
radioactive decay of 3403
James Webb space telescope (JWST) 382
Jansky, Carl, and background radiation 373
Joliot-Curie, Irene, and radioactive isotopes
269
junction transistors 1989
Kamerlingh Onnes, Heike, liquefies helium
211
Kelvin, Lord (William Thomson), and
temperature scale 210
Kendall, Henry, and quarks 292
Kepler, Johannes
laws of planetary motion 359
telescope 3712
third law 409
Kirchhoff, Gustav, and spectral lines 390
Korolyov, Sergey (aka Sergei Korolev), USSR
space program leader 28
Kronig, Ralph, and electron spin 240
Lagrange points 29
landing of spacecraft 501
Large Hadron Collider (LHC) at CERN
288, 2901
Larmor frequency 356
laser diodes 195
Laue, Max von, X-ray diffraction patterns
2079
launch angle of projectile 6
law of universal gravitation 11
in finding new planets 40
in predicting small deviations in planetary
orbits 39
Lawrence, Ernest, and linear accelerator 287
length contraction 6970
length (l ) in motor effect 93
Lenz, Heinrich
application to DC motors 121
law of 1045

467

Index

leptons 2924
levitated magnets 21314
Lewis, Gilbert, names photons 231
light
aether theory of transmission 612
infra-red 197
intensity and wavelength 178
speed of 65
as wave and particles 179
wave properties of 2067
light-emitting diodes (LEDs) 195
light-year (ly) distance unit 389
lighting, fluorescent and neon 173
lightning protection 147
linear array transducers 309
linear electric motors 129
linear generator principle 133
linear particle accelerators 2867
linearising formulae 447
Livingston, Stanley, and linear accelerator
287
longitudinal relaxation time constant (T1)
358
Lorentz, Hendrik
and aether 63
factor () 67
loudspeaker-making activity 126
loudspeakers 91, 93
low Earth orbits (LEOs) 43
luminiferous aether 61
luminosity classes 3956
Lyman, Theodore, spectral series for
hydrogen atom 2334
M-mode ultrasound scans 313
Mach, Ernst (18381916), and ratio of speed
of sound 48
magnetic field strength (B)
as magnetic flux density 101
in motor effect 92
magnetic fields
and electric currents 87
forces on charged particles in 89
magnetic flux (B) 1013
magnetic hysteresis losses
in induction cookware 108
in transformers 140
magnetic levitation (maglev) 21314
trains 129, 21819
magnetic moment 355
magnetic poles 88
magnetic resonance imaging (MRI) 21718
applications 362
behaviour of hydrogen proton in 357
see also MRI scanners
magnetism, and spin of atomic particles
3545
magnification of telescopes 371, 3745
magnitudes of stars 397400
main sequence stars, properties relative to
Sun 425
mammograms 325
Manhattan Project 27980
Mariner 10, use of gravity assist by 45
Marsden, Ernest, and model of atom 22930

468

mass
equivalence to energy 724
relativistic 71
mass defect 267
massenergy 73
massluminosity relationship 413
massive stars, fate of 4301
matter, crystal structure of 2045
matter waves, de Broglies equation 24850
Maxwell, James Clerk
electromagnetic wave theory 1745
equations of 612
medical radioisotopes 283
medium Earth orbits (MEOs) 43
Meissner effect 212
Meitner, Lise
and beta decay 265
and radioactive isotopes 269
mesons 292
metal-oxide-semiconductor field-effect
transistor (MOSFET) 200
metals, crystal structure and electrical
conductivity 20910
methamphetamines, brain tissue loss in users
302
metric prefixes 4423
Michell, John, weighing the Earth 25
Michelson, Albert, and speed of light 62
MichelsonMorley experiment 624
results interpretation activity 756
microscopes, optical to electron 259
Minkowski, Hermann, and spacetime 72
moderators in nuclear reactor 281
molybdenum-99 production by ANSTO 353
momentum conservation, in slingshot effect
44
Moores law 223
Morley, Edward, and speed of light 62
Moseley, Henry 247
and model of atom 230
motion, components of 5
motor effect (F) 903
activity 97
MRI scanners
operation of 3601
see also magnetic resonance imaging
(MRI)
muons 286
n-type semiconductors 192
National Medical Cyclotron at Royal Prince
Alfred Hospital, Sydney, radioisotope
production at 282, 345, 353
nebulae, types 422
negative glow 158
neon lights 138, 173
neutrinos, Paulis proposal 2656
neutron, discovery of 2601
neutron diffractometers, ANSTOs Echidna
285
neutron scattering, applications of 272, 278
neutron stars 431, 432
neutrons, number (N) in atomic nucleus
2623

Newcomb, Simon, on future of astronomy


434
Newton, Isaac
composition of light 390
law of universal gravitation 1011
telescope of 372
non-coherent bundles of optical fibres 3356
non-elliptical orbits 412
non-inertial frames of reference 59
non-periodic variable stars 414
nuclear chain reactions, controlled and
uncontrolled 2701
nuclear fission 269
nuclear fission reactors, components 2801
nuclear fusion energy generation 218
nuclear reactors, producing radioisotopes in
345
nucleons, gravitational attraction of 2612
numerical calculation skills 4434
Oberth, Herman, space flight and travel 27
Ohms law, and electrical resistance 86
ONeill, Gerard, US physicist 289
OPAL nuclear research reactor
Bragg Institute at 272
Echidna at 285
radioisotope production at 282, 345
open orbits 412
open star clusters 4334
optical fibres 334
in endoscopy 333, 3357
orbital decay 467
orbital velocity 36
of Earth 35
orbits
elliptical 379
manoeuvres between 57
of planets 3540
of satellites 434
types 412
osteoporosis detection using ultrasound 315
pn junctions (diodes) 1935
as solar cells 196
p-type semiconductors 192
parabolic orbits 412
parabolic trajectories 37
parallax angle (p) 3889
parallax measurement of star distance 3889
parallel wires, forces between 936
parsec (pc) distance unit 389
particle accelerators
and radioisotopes 282
types 2869
particle detectors 289
path length of waves 205
Pauli, Wolfgang
exclusion principle 255
and neutrino 266
pendulum, oscillation period T 13
periodluminosity relationship, for Cepheid
stars 416
periodic variable stars 415
periods, Keplers third law of 38

Index

Perrin, Jean, and cathode rays 160


PET see positron emission tomography
phonons 210
photocathode and photoelectrons 182
photocell 184
photoelectric effect
applications 1845
discovery 1823
photometry, astronomical 397
photomultiplier tube 184
photon energy (E) 393
photons, Einsteins proposal 179, 181
photovoltaic cells (PVs) 196
piezoelectric effect and transducers 3089
Planck, Max
and black body radiation curve 179
constant (h) 181, 2312
and electromagnetic wave emitter energy
180
planetary nebula 429
planets, finding new 40
Plcker, Julius, and vacuum discharge tubes
1567
plum pudding model of atom 229
plutonium-239 atomic bomb 279
Pogson, Norman, and star magnitude scale
397
Poincar, Jules Henri, and speed of light 65
polarised electromagnetic waves 177
poles, geographic and magnetic 88
positive column 158
positron decay 342
positron emission tomography (PET) 1845,
283, 3479
potential difference 86
power, electric see electric power (P)
power generation, transmission and storage
using superconductors 219
power-line transmission structures 1456
precession in rotation of magnetic moment
356
precision in experiments 445
prefixes, metric 4423
presenting research 4467
pressurised water reactor 281
principle of relativity, Einsteins 589
projectile motion 410
activity 20
proper length 69
proper mass (rest mass) 71
proper time 67
protonproton chain 4256
protons, electrostatic repulsion of 2612
protostars, evolution of 424
pulsars 432
quanta of energy 179
quantum computers 223
quantum mechanics, Solvay Conference
(1927) 2478
quantum number (n) 231
of wavelengths in Bohr orbit 253
quantum physics 226

quarks 2924
quasars, emission spectra for 394
racing magnets 107
radiation shielding, in nuclear reactors 281
radio antennae 1757
radio frequency (RF) waves 356
radio telescopes 3723
radio transmitters and receivers 1745
radio waves, speed of 1757
radioactive decay of isotopes 3403
radioactive tracers 282
radiographs, conventional 324
radioisotopes
decay of 3403
half-life of 343
production of 345
range of uses 2824
radiopharmaceuticals 283
choice of 3445
radiotherapy 283
random error 445
Reber, Grote, and radio telescope 373
rectifiers 195
re-entry into Earths atmosphere 4650
safe corridors 478
reflecting telescopes 3723
reflection nebulae 422
refracting telescopes 3712
regenerative braking 134
relativistic mass and speed 71
relativity, Einsteins principle of 589
relaxation of hydrogen protons after cessation
of RF pulse 3589
relaxation time constants (T1 and T2) of
body tissues 35960
research, presenting 4467
resistance, electrical
from crystal structure 210
losses in transmission lines due to 1445
and Ohms law 86
and power 86
resistive heating (Q)
in induction cooktops 108
in transformers 140
resonance 358
rest mass (proper mass) 71
reverse bias in pn junctions 195
RF transceiver coils in MRI scanners 361
right-hand grip rule 878
right-hand palm (or push) rule 89, 901,
123
rocket engines and stages 301
rocketry
history of 267, 29
researchers in 20th century 279
rockets
forces during take-off 334
thrust on 30
roller-coasters, eddy current braking in
11213
Rntgen, Wilhelm Conrad, discovers X-rays
207
RR Lyrae variable stars 41516
on HR diagram 414

Rutherford, Ernest, atomic model of 22830


RutherfordBohr model of atom 236
limitations of 23940
Rydberg, Johannes, constant (R) of 233
Salam, Abdus, and electro-weak theory 292
satellites, orbits of 434
Savitch, Pavle, and radioactive isotopes 269
sawtooth waveform on CRO 1678
scans
MRI 21718, 3601, 362
ultrasound 31214
Schrodinger, Erwin
and de Broglies hypothesis 250
wave function of 2534
seeing in telescopes 3789
Segr chart for uranium-238 decay series 264
semi-synchronous satellites 43
semiconductor devices 1936
semiconductors, explanation 1903
sensitivity of telescopes 3756
shaded-pole AC induction motors 124
shock waves from projectiles 49
silicon doping 192
simple DC motors, operation of 11719
simultaneity, relativity of 656
single-emission computed tomography
(SPECT) 347
single-phase alternating current 121
induction motors 124
sinusoidal waveform (trace) on CRO screen
168
slingshot effect (gravity assist) 445
solar cells 196
solid-state devices, compared with
thermionic devices 199
solid state physics 190
Solvay conference (1927) participants 248
sonograms 306
Space Shuttle launch, g-force during 34
Space Shuttle re-entry and landing 4851
spacecraft, launching 2635
spacetime interval 72
special relativity, Einsteins theory 648
spectra, continuous 390, 392
spectral classes 395
spectral lines
fine and hyperfine 240
relative intensity of 239
and size of stars 3967
spectrographs 391
spectrometers and black body radiation 180
spectroscope binary stars 41112
spectroscopes, CDs and DVDs as 155
spectroscopes (spectrometers) 232
and spectrographs 391
spectroscopic parallax, for star distance
4012
spectroscopy, astronomical 390
spin of atomic particles, and magnetism
3545
spin of electrons 217
springs, behaviour of 15
Square Kilometre Array (SKA) telescope 382

469

Index

squirrel-cage rotors 123


stable orbits 41
Standard Model of matter 2924
standards for mass and length 79
standing waves 1756
as electron orbits 2523
Stanford linear accelerator 287
star catalogues 369
star clusters
activity 435
age of 4334
stars
ageing of 4257
binary 40710
birth of 4234
colour index 4001
composition of 38994
distance modulus 399400
evolution 4257
evolution of low to medium mass 429
evolution of massive 4301
magnitude measurement 3979
naming 415
size from spectral lines 396
spectroscopic parallax 4012
variable 41317
step-down and step-up transformers 137
Stoney, George Johnston, names electron 166
Strassman, Fritz, and radioactive isotopes 269
strong nuclear force 262
strontium emission spectrum 239
substations 1423
Sun
future of 4289
life on main sequence of HR diagram
427
superconducting quantum interference
device (SQUID) 218
superconductors
applications 21719
critical temperature (Tc) 211
discovery of 211
high-temperature 216
Meissner effect in 212
type-I and type-II 21213
supergiant stars 421
supernova 431
supersonic flight 489
supply emf in DC electric motors 120
surface-conduction electron-emitter (SCE)
display TV sets (SED-TVs) 173
synchronous AC motors 1301
synchrotrons 288
systematic error 445
Szilard, Leo, and chain reactions 270
tangential velocity at rocket launch 345
Taylor, Richard, and quarks 292
technetium-99m, generation and use 346
telescopes
angular resolution 3767
early optical 3703
of the future 382
interferometry in 3801

470

magnification 374
methods of sharpening images 37780
radio 3723
sensitivity 3756
television
CRT tubes used in 1689
LCD, plasma and SED-TVs 173
Tesla, Nicola, and AC motors and generators
141
thermionic devices, compared with solidstate devices 199
thermionic diodes and triodes (valves) 198
thermonuclear bombs 280
Thomson, George P, and electron beam
diffraction 252
Thomson, Joseph John 160
discovery of electron 1656
Thomson, William (Lord Kelvin), and
temperature scale 210
three-phase AC induction motors 1223
three-phase alternating current 1212
3D real-time ultrasound images 314
thrust on rocket (FT) 30
time dilation 667
time of flight of projectiles 6
torch without batteries 133
torque () on a rotating coil 11517
total internal reflection, in optical fibres 334
trains, eddy current braking in 11213
trajectories 3
components 57
transducers
piezoelectric 3089
for ultrasound 305
transformers
efficiency and design 13740
in electricity distribution 1445
in the home 144
principles 1367
transistors 1989
transmission towers for high-voltage AC
power 1456
transmutation, artificial and natural 2635
transverse magnetisation (Bxy) 359
transverse relaxation time constant (T2) 359
triodes as current amplifiers 198
triple alpha process 428
Tsiolovsky, Konstantin, rocket equation 27
twin paradox 68
2D real-time ultrasound scans 31314
2D slice image construction from CAT
scanning 3278
type-I and type-II superconductors 21213
ultrasound 3045
reflection at tissue boundaries 31012
ultrasound imaging
and acoustic impedance 31011
limitations 3078
in obstetrics 307
principles 3057
types of scans 31216
using piezoelectric transducers 3089
ultrasound scans, types 31214

ultraviolet catastrophe 179, 180


uncertainty principle, of Heisenberg 2545
uniform circular motion, activity 52
universal electric motors 122
universal gravitation
Newtons law of 1011
see also law of universal gravitation
universal gravitational constant G 11
uranium-235 atomic bomb 279
V838 Monocerotis, supergiant star 421
valence bands 18990
holes in 191
valence electrons 189
valence level 189
Vallebona, Alessandro, and tomography 321
valves (thermionic devices) 198
Van de Graaff accelerator 288
variable stars 41315
observed changes in 421
vascular structures, X-ray images of 325
verbs, use of 44851
visual binary stars 411
voltage (V) 86
von Braun, Werner, Moon mission leader 28
vortex states, in magnetic levitation 214
Voyager 1 and 2 space missions 46
wave function, Schrodingers 254
wave interference 2057
wave mechanics of Schrodinger 250
wave nature of electrons, de Broglies
hypothesis 24852
wave properties of light 2067
wavelength of X-rays (e) 209
waves, path length 205
weak nuclear force 266
weight in gravitational fields 1314
weightlessness effect 31
Weinberg, Steven, and electro-weak theory
292
Westinghouse, George, and AC electricity
141
white dwarfs 429
Wollaston, William, and lines in Sun
spectrum 390
work and gravitational potential energy
1617
WR 104 star system 3867
X-ray binary stars 412
X-ray detector technology 326
X-ray diffraction by crystals 207
X-ray images
production 3245
types 3201
X-ray tubes 3212
X-rays, types and properties 3223
Zeeman, Pieter, effect of 240
zero-age main sequence (ZAMS) 424
Zweig, George, and quarks 292

Acknowledgements
We would like to thank the following for permission to reproduce photographs, texts and illustrations. The following
abbreviations are used in this list: t = top, b = bottom, c = centre, l = left, r = right.

Barnaby Norris: pp. 368, 375, 391.

Physics Stage 6 Syllabus Board of Studies NSW for and


on behalf of the Crown in the right of the State of New
South Wales, 2007: pp. xxxiii, Formulae and Data sheet:
p. 473, Periodic Table of the Elements: p. 474. The Board
of Studies does not endorse model answers prepared by or
for the Publisher and accompanying the material. The
Office of the Board of Studies takes no responsibility for
errors in the reproduction of the material supplied by the
Office of the Board of Studies to the Publisher.

BioMed Central: p. 313.

Images.com: p. 2.

Brian James: p. 96.

Institut International de Physique Solvay: p. 248.

CartoonStock: p. 299.

Intel: p. 223.

CERN: pp. 226, 288, 289, 293.

iStockphoto: pp. 138, 139, 142, 178, 217, 307, 326, 338,
348br, 350tl, 380b, 441.

AIP Emilio Segre Visual Archives: p. 216.


Alamy Pty Ltd: p. 259r.
Andrew Dunn: p. 372b.
ANSTO Australian Nuclear Science & Technology
Organisation: p. 345/Vanessa Peterson: pp. 278, 285.
Australia Telescope National Facility: pp. 378l, 431l.

Corbis: pp. 61, 303.


CSIRO Publishing: p. 188.
Damian Peach: p. 376.
Dave McKinnon: p. 411.
David Malin Images: pp. 369, 400, 401.
Dreamstime: pp. 14, 314b, 324l, 352tl, 356, 360.

ITER: pp. 218.


Jason Lee: p. 290.
Jim Mosher: p. 372t.
John Rowe Animation: p. 412.
Lorojon Pty Ltd: p. 197b.

Peter Tuthill and WM Keck Observatory: p. 386.

NASA: pp. 17, 26, 28t, 28b, 33, 36, 49a,b,d, 50, 60, 72,
382, 421, 423, 429, 431r.

D. Smyth: p. 381l.

NRAO/AUI/NSF: p. 373.

Fourmilab: p. 11.
Fundamental Photographs, NYC: p. 251l.

PA/Jeff Stanger: pp. 83, 90, 115, 118, 124, 132, 133, 144r
/John OByrne: p. 381r.

Gemini Observatory: p. 378r.

Palomar Observatory, Caltech: p. 380t.

Greg Konkel: p. 383.

Photolibrary Pty Ltd: pp. 3, 12, 13, 15, 27, 28c, 91, 114,
119, 122, 127, 141, 146, 154b, 155, 167, 176, 180, 185,
197t, 204, 207, 210, 213, 227, 228, 235, 246, 249, 251c,
251r, 252, 254, 255, 260, 271, 306, 308, 309, 314t, 314c,
315, 319, 324r, 325tr, 328, 332r, 333, 337, 348tl, 349,
352tr, 362, 367, 390b, 392, 422, 433, 440.

Harald Hess: p. 214.

471

Acknowledgements

Sebastian Terfloth: p. 112.


The Picture Source: p. 7.
Radiation Oncology Department of the Prince of Wales
Hospital, Randwick NSW: p. 366.
Rod Nave: p. 241.
Reproduced with permission from the Ministero per i Beni
e le Attivit Culturali, Italy/Biblioteca Nazionale Central,
Firenze: p. 4.
Sebastian Egner: p. 379.
Shutterstock: pp. 59, 82, 86, 121, 129, 144l, 145, 154t,
259l, 325tl, 332l, 365l, 377.
Smithsonian National Air and Space Museum: p. 49
(Soyuz).
U.C.L.A./Dr. Paul Thompson: p. 302.
University of Arizona/Tim Hunter: p. 324c.
UBC & Vancouver Coastal Health Research Institute/
Department of Radiology: pp. 347, 352br.
Every effort has been made to trace and acknowledge copyright. However, should any infringement have occurred, the
publishers tender their apologies and invite copyright owners to contact them.

472

Formulae and data sheets

FORMULAE SHEET
v =f

F = qvB sin

m1m2
r

Ep = G
F = mg

FORMULAE SHEET continued


E =

d2

v1
sin i
=
v2
sin r

vx 2 = ux 2

E =

F
q

v = u + at

R=

V
I

vy 2 = uy 2 + 2ay y

T2

aav =

vu
v
therefore aav =
t
t

F =

F = ma
F =
Ek =

mv 2
r
1
mv 2
2

1
a t2
2 y

4 2

Gm1m2
d2

E = mc 2
v2

lv = l0 1

c2

t0

tv =

v2

v2
c2

p = mv
Impulse = Ft
F
l

= k

I1I 2
d

F = BIl sin

1
d =
p
M = m 5log
IA
IB

= Fd
= nBIA cos

Vp
Vs

np
ns

d
10

( mB mA )

= 100

m1 + m2 =

Rf
Ri

[ Z2 Z1] 2
[ Z2 + Z1] 2

Charge on electron, qe

1.602 1019 C

Mass of electron, me

9.109 1031 kg

Mass of neutron, mn

1.675 1027 kg

Mass of proton, mp

1.673 1027 kg

Speed of sound in air

340 m s1

Earths gravitational acceleration, g

9.8 m s2

Speed of light, c

3.00 108 m s1

Magnetic force constant, k =

c2

m0

mv =

DATA SHEET

GM

1
W = Fs

Ir
I0

y = uy t +
=

Vin

Z = v

Energy = VIt

r3

Vout

Vin

c = f

x = ux t

r
t

V
d

Vout

E = hf

P = VI

vav =

A0 =

4 2r 3

0
2

2.0 107 N A2

Universal gravitational constant, G

6.67 1011 N m2 kg2

Mass of Earth

6.0 1024 kg

Planck constant, h

6.626 1034 J s

Rydberg constant, R (hydrogen)

1.097 107 m1

Atomic mass unit, u

1.661 1027 kg
931.5 MeV/c2

1 eV

1.602 1019 J

Density of water,

1.00 103 kg m3

Specific heat capacity of water

4.18 103 J kg1 K1

GT 2

1
1
1
= R 2 2

n f ni

h
mv

473

474

20
Ca
40.08

38
Sr
87.62

56
Ba
137.3

88
Ra
[226]

19
K
39.10

37
Rb
85.47

55
Cs
132.9

87
Fr
[223]

Francium

Caesium

Rubidium

Potassium

Sodium

Radium

Barium

Strontium

Calcium

Magnesium

12
Mg
24.31

Beryllium

11
Na
22.99

Lithium

4
Be
9.012

3
Li
6.941

Hydrogen

1
H
1.008

Thorium

Protactinium
Uranium

92
U
238.0
Neptunium

93
Np
[237]

Promethium

61
Pm
[145]

Plutonium

94
Pu
[244]

Samarium

62
Sm
150.4

Hassium

108
Hs
[277]

107
Bh
[264]

Bohrium

76
Os
190.2
Osmium

Ruthenium

44
Ru
101.1

Iron

26
Fe
55.85

75
Re
186.2

Rhenium

Technetium

43
Tc
[97.91]

Manganese

25
Mn
54.94

Americium

95
Am
[243]

Europium

63
Eu
152.0

Curium

96
Cm
[247]

Gadolinium

64
Gd
157.3

110
Ds
[271]

Platinum

78
Pt
195.1

Palladium

46
Pd
106.4

Nickel

28
Ni
58.69
Copper

Gold

Berkelium

97
Bk
[247]

Terbium

65
Tb
158.9

111
Rg
[272]

79
Au
197.0

Silver

47
Ag
107.9

29
Cu
63.55

Name of element

Symbol of element

Meitnerium Darmstadtium Roentgenium

109
Mt
[268]

Iridium

77
Ir
192.2

Rhodium

45
Rh
102.9

Cobalt

27
Co
58.93

Gold

79
Au
197.0

KEY

Californium

98
Cf
[251]

Dysprosium

66
Dy
162.5

Mercury

80
Hg
200.6

Cadmium

48
Cd
112.4

Zinc

30
Zn
65.41

Einsteinium

99
Es
[252]

Holmium

67
Ho
164.9

Thallium

81
Tl
204.4

Indium

49
In
114.8

Gallium

31
Ga
69.72

Aluminium

13
Al
26.98

Boron

5
B
10.81

Fermium

100
Fm
[257]

Erbium

68
Er
167.3

Lead

82
Pb
207.2

Tin

50
Sn
118.7

Germanium

32
Ge
72.64

Silicon

14
Si
28.09

Carbon

6
C
12.01

Mendelevium

101
Md
[258]

Thulium

69
Tm
168.9

Bismuth

83
Bi
209.0

Antimony

51
Sb
121.8

Arsenic

33
As
74.92

Phosphorus

15
P
30.97

Nitrogen

7
N
14.01

Nobelium

102
No
[259]

Ytterbium

70
Yb
173.0

Polonium

Lawrencium

103
Lr
[262]

Lutetium

71
Lu
175.0

Astatine

Iodine

53
I
126.9

Bromine

35
Br
79.90

Chlorine

17
Cl
35.45

Fluorine

9
F
19.00

Radon

Xenon

54
Xe
131.3

Krypton

36
Kr
83.80

Argon

18
Ar
39.95

Neon

10
Ne
20.18

Helium

2
He
4.003

84
85
86
At
Rn
Po
[209.0] [210.0] [222.0]

Tellurium

52
Te
127.6

Selenium

34
Se
78.96

Sulfur

16
S
32.07

Oxygen

8
O
16.00

For elements that have no stable or long-lived nuclides, the mass number of the nuclide with the longest confirmed half-life is listed between square brackets.
The International Union of Pure and Applied Chemistry Periodic Table of the Elements (October 2005 version) is the principal source of data. Some data may have been modified.

Actinium

91
Pa
231.0

90
Th
232.0

Actinoids
89
Ac
[227]

60
Nd
144.2

Seaborgium

106
Sg
[266]

Tungsten

74
W
183.8

Molybdenum

42
Mo
95.94

Chromium

24
Cr
52.00

Praseodymium Neodymium

Cerium

Lanthanum

Dubnium

59
Pr
140.9

Rutherfordium

Actinoids

105
Db
[262]

Tantalum

73
Ta
180.9

Niobium

41
Nb
92.91

Vanadium

23
V
50.94

Lanthanoids
57
58
La
Ce
138.9
140.1

104
Rf
[261]

Hafnium

72
Hf
178.5

Zirconium

40
Zr
91.22

Titanium

22
Ti
47.87

89103

Lanthanoids

5771

Yttrium

39
Y
88.91

Scandium

21
Sc
44.96

Atomic Weight

Atomic Number

PERIODIC TABLE OF THE ELEMENTS

Вам также может понравиться